Sunteți pe pagina 1din 416

1

11

DAY 23 - SOLUTION .. . .. . . .. ... . . .. . .. ... .. . . . . . .. . .. ... .. . . .. ... ... . . . .. . . .. . .. .. . . .. ... . 23 - 9


DAY 24- EXAM .. ........ ... ............ .. .,, ... .... ... ..... ....................... ... ..... .. 24-1
PART 1 - ENGINEERING MATHEMATICS DAY 24- SOLUTION .................................... .. .. ... ........................... : 24-5
DAY 25- EXAM ........... ........................................ ............. ..... ......... 25- 1
DAY 1- EXAM ........................ ....... ... .......... . .... .. ........ .......... ......... ... 1 -1 DAY 25- SOLUTION ........ .. ... .. .. .... ... ......................................... ...... 25- 10
DAY 1- SOLUTION ~ ....................................... .... ........... .... .............. 1-9 DAY 26- EXAM ........... ....... ...... ... .............................. ....... .... ... .... ... 26-1
DAY 2- EMM ......., ...... ... .... ............................... .. .... :........ ............... 2- 1 DAY 26 - SOLUTION . . . . . . . . . ... . . .. . .. ... .. . .. . . .. . .. . .. .. . . .. ... .. . . . . .. . . .. . .. . .. . .. . .. . 26 - 6
~~~ ~ ~ ~~~:.~.~~-~::·:·~·.;·~.:·.·.·.·.·.·.·:.·.·.·.·:.·:.·.·:.·:.·.·.·.·. ·.·.:.·.·.·. ·.·.·.·.·.·:.·.·.·::.·.·.·:::.·.·:.·.·:.·.·.·:.·:.·:.:.· ~ =~ DAY 27- EXAM ....................................... .. ................. .. ... .... .... ....... 27- 1
DAY 27 - SOLUTION .. .. .. . .. ... . .. . . .. .. .. .. .. . .. .. .. .. .. .. .. .. .. ... ... . .. .. .. .. .. .. .. .. .. 27 - 6
DAY3-SOLUTION ... ......~ ... ~..... ............. ........... ...... ......... ............... 3-6 DAY28-EXAM ........ , ......................... ........ ~ ....................... ...... ...... 28-1
DAY 4 - EXAM .. .... .................... .......................... .. ... .. ... ...... . ..... ....... 4 - 1 DAY 28- SOLUTION ........ .. ............ ~ .............................................. .. 28-6
DAY 4- SOLUTION ............... ... ........................ ........... ..... ..... ....... .... 4 -7
DAY 29 - EXAM ........................ .'..... .. . . . .. .. .. .. .. ... .. . .. .. .. ... .. .. .. . .. .. . .. .. .. 29 - 1
DAY 5 - EXAM .............................................................................. ... 5 - 1 DAY 29- SOLUTION .................................................................... ... 29-6
DAY 5- SOLUTION ......................... .... .... . .. . ..................................... 5-8
DAY 30- EXAM .............. ... ........... . ........................ .......... .. .......... ... 30- 1
DAY 6 - EXAM ... .............. ... ..................................... ..... . ...... ..... ....... 6- 1 DAY 30- SOLUTION ... ... .. ...... . .. ... ... .. ... ......... .. .... ... .. . .. . ...... .... .. .. .. . 30 - 5
DAY 6- SOLUTION ... .. .................... ........... ... ................ .... .. ............. 6-6 DAY 31 -EXAM ... .... ..... .. ...... .. .. .. ... . .... ...... .... .. ............ .. .. ............. 31- 1
DAY 7 - EXAM ........ . ........ ...... .......... .... .. ....................................... ... 7 - 1 DAY 31 -SOLUTION ... ... ... .... ... ... ... ... ... ... .. . ... ... ...... .. . ... .. . ... ... ... .. . ... 31 - 6
DAY 7- SOLUTI ON ... .. ............. ... . ........ ......................... . .. .. .... ....... ... 7-7 DAY 32- EXAM ....................... .. ................................... .. : .. .. ........... 32- 1
DAY 8- EXAM ............. .. ....... .. ... ....... .. ............ ..... ......... ..... .............. 8- 1 DAY 32- SOLUTION .. .... .. .. ... ............... ....... :................ ..... ...... ...... 32 - 6
DAY 8- SOLU TION ................... .... ......... .. .. ............... : ....... ........ ... 8-7 DAY 33- EXAM ...... .. .. ... .......... . .... ................................................... 33- 1
DAY 9 - EXAM .... ....... ... . ...... .. . ............ ... ........ .............. .. ...... ............ 9 - 1 DAY 33 - SOLUTION .. . . .. . .. . .. . . ... ... .. .. .. .. ... .. .. .. .. .. .. ...... .. .. . .. . .. ... . .. .. .. .. 33- 6
DAY 9- SOLUTION .................. . ............. .... . ... ........... ........... ............ 9-6 DAY 34- EXAM .. .. ........................... .. ... ..... .. ............................. ....... 34 -1
DAY 10- EXAM ........................ ....................................................... 10-1 DAY 34- SOLUTION ............................... ........ ............ .. ................ 34-6
DAY 10- SOLUTION........................................ ... ......... .............. .... 10 - 7 DAY 35- EXAM ..................... ........ ..... .... .... ... ................. ......... ... .... 35 - 1
DAY11-EXAM ............................................................................... 11-1 DAY 35- SOLUTION ...... .......... ...... ... .. . ... .. .... ... ... ... .. .. ..... ...... ......... 35-6
DAY 11 - SOLUTION .. . .... .. ...... ..... .. .. .. .. ... ... .. . .. .. .. .. .. .. ... .. .. .. .. .... .. .. .. 11 - 5
DAY 36- EXAM ................................................................ . ............. 36- 1
DAY 12- EXAM ............................................................................... 12 - 1 DAY 36 - SOLUTION . .. . .. . .. .. .. . .. ... .. . .. . . .. . .. .. . .. . . .. . .. ... .. . .. . . .. . .. . .. ... ... ... 36 - 6
DAY 12- SOLUTION .................................................·........... .. ........ 12-6
DAY 37- EXAM .. ........ ..................................................................... 37 -1
DAY13-EXAM ............................................ .'.................................. 13-1
DAY 37- SOLUTION .............................. .... .............. .. .......... ... ....... 37-9
DAY 13- SOLUTION.... .. ... .. ................................ ....... ....... ........ ..... 13-7 DAY 38 - EXAM .. .. .. ..... .... .. ... . : ... .-............. ..... ................................... 38 - 1
DAY 14- EXAM ................................ . ................................. ........ ..... 14-1
DAY 38 - SOLUTION . .. .. . .. . .. .. .. .. .. .. .. .. .. . .. . .. .. .. .. . .. . .. .. .. .. . .. . .. .... ... ... .. . 38- 6
DAY14-SOLUTION ..................................... .... .. ........................ ... 14-6 DAY 39- EXAM ...... .... ..... .... .. ......... ........ .. .. .............. . ...... ... ........ ..... 39-1
DAY 15 - EXAM.. . .. . .. . .. . .. . . .. . .. .. . .. .. .. .. . .. . .. .. .. .. . ... .. . ... . .. .. . .. . ... ... .. . . .. .. .. 15 - 1 DAY 39 - SOLUTION ... ... ... ..... .. ........................ ................... .. ... ... ... · 39 - 7
DAY 15- SOLUTION ............. ... ...... ... .......................................... ... 15-6 DAY 40- EXAM .. . ...... ... .... .. ... ....'. . ........... .. ... .......................... .. . .. ..... 40-1
DAY 16- EXAM .................. ... .......... .............. ......... ................... ... .... 16-1 DAY 40 - SOLUTION .. .... .. .. . .. .... .. ... ... . .. .. .. .. .. .. . .. . .. .. . .. .. .. .. .• . .. . .. .. . .. . 40 - 6
DAY 16- SOLUTION......... .... ......... .. ... ................. .......................... 16-6
,;
DAY 17- EXAM ............ ......................... ....... ...... .. ....... . ...... ... ... ... .... 17 - 1
DAY 17- SOLUTION............... . ......... .......... ........................ ........... 17-7 PART 3- POWER AND INDUSTRIAL PLANT ENGINEERING
DAY 18 - EXAM ........ ...... . ...... .. ................ .. ...... ...... ...................... .... 18-1
DAY 18- SOLUTION............ ..... ... ............. ...... .... ......... ..... ............. 18-7 DAY 41- EXAM ........ . .. ....... ..... . .... ... ........................................... . ... 41- 1
DAY 19- EXAM ........ .......... ................... ........... ...... ........... ........... . 19-1 DAY 41- SOLUTION ........... ....... .. .... ............................................... 41 -5
DAY 19- SOLUTION.. .. ............. ............. . .... .. .... .. ... .. . .... ................. 19-6 DAY 42- EXAM .. ...................... .. .. .. .... ...................................... ...... 42 - 1
DAY 20 - EXAM .. . ...... ... ... .... ..... ... ... ...... .. ...... . ... ................ .. ............. 20 -1 DAY 42- SOLUTION .............................. ..... ... ... . ............................ 42-5
DAY 20 - SOLUTION .. .... .. .. ...... ... .. .. .. .. .. .. ... .. .. .. .. . ... .. . .. .. ... ... .. .. . .. .... 20- 6 DAY 43- EXAM ............................... .... ............ .... ........... ... .... ..... .... 43- 1
DAY 43 - SOLUTION .............................................................. ......... 43- 6
DAY 44- EXAM ............................................................................... 44- 1
PART 2 - POWER AND INDUSTRIAL PLANT ENGINEERING • DAY 44 - SOLUTION ......................, ................................................ 44-9 ·
DAY 45- EXAM ............ ...... .............. ........ ... ................................... 45- 1
DAY 21- EXAM ........... ..... ........... .. ........ ........................................ 21 - 1 DAY 45- SOLUTION ...................... :.. ....... , .............. .. ........ .. ............ 45 - 9
DAY 21- SOLUTION .............. ... ........... .. ..... ...... .. ............................ 21-6 DAY 46- EXAM ............................................................. ................. 46- 1
DAY 22- EXAM............... ...... ... .. .. .. .. .... .................. ......... ........ .. ... .. 22- 1 DAY 46 - SOLUTION ... .. .. .. .. .. .. . .. . .. .. ... .. ... .. .. ... ... .. .. .. .. . .. .. .. .. ... .. .. .. .. .. 46 - 5
DAY 22- SOLUTION .......................................................... ...... ..... ... 22 -5 'DAY 47- EXAM ............... .. ......................... ... .. ~ ................. .. ... ........ 47-1
DAY 23- EXAM ............... .. .... ........................... ........ .... .. ........ .. .... .. 23 - 1 DAY 47- SOLUTION ..... .. ....... ......... .. .. .. ............... ....... ...... ... ..... ... ... 47 - 6
DAY 48- EXAM ..... . .. .. ..·... .. .. ..'... .. . .... .. ... ... ... .. . ... ... ....... .. ............ ... ... 48 - 1
DAY48 -SOLUTION ........ ................ .... .............. ....... . . ... ........... 48-6
iii
DAY 49- EXAM ..... .... .. . ............ ... ................. ...... ..... ............. .... ...... 49- 1
DAY49-SOLUTION ...................... ................................................. 49-5
DAY 50- EXAM ............................................................................... 50-1
DAY 50- SOLUTION ...... ... .... ... ......... ... ... ...... ... ......... ... ... ... ...... ... ... 50 - 5
DAY 51- EXAM ......... ... ......... ................ ................................ .. . ....... 51 - 1
DAY 51 - SOLUTIOIIT .:.. ...... .... ..... .... ... ... ... ... ... ... ......... ... ... ... ... .... .. ... 51 - 6
DAY 52- EXAM .......... '..'":...................................................... .. ........ 52 - 1
PART 1
DAY 52- SOLUTION .... ..... ~..... ............ ..... ....... .................... .. ......... 52-6
DAY53-EXAM ............................................................................... 53-1
DAY 53- SOLUTION .. .... ......................... ..... : ................................. 53 - 6
DAY 54- EXAM ........................... ............ ........ .. .. .... ,.... :.................. 54 - 1
DAY 54 -SOLUTI ON .......... .. .. .. ............... ....... ......... ....................... 54-7
DAY 55- EXAM ........ .......... ................................. ............................ 55- 1
DAY 55-SOLUTION .. ............. .. .... .... ..... ... ... .'................................ 55- 5
DAY 56 - EXAM .. . .............. . ... ... ...... .. . ...... .................................... .. .. 56 - 1
DAY 56- SOLUTI ON ... .... .... ... ..... .. ........... . ........................ ........... 56-5
DAY 57- EXAM .............. .. .. .. ............ ... .... .. ....... ............... ........ ... ..... 57- 1
DAY 57- SOLUTION ......... ........ :.... ... ... ............. .. ........... ... ..... ........ 57 -6
ENGINEERING MATHEMATICS
DAY58-EXAM ............. .... .... ............ ........ ....... ............................... 58-1
DAY 58- SOLUTION .................................................... ........... . ...... 58 - 10 ENGINEERING SCIENCES
DAY.59- EXAM ............................................................................... 59- 1
DAY 9- SOLUTION ....... .. ........................................ .. ......... .'.........
DAY 60- EXAM ............................................................................... 60-1
59 - 9
ENGINEERING ECONOMICS
DAY 60- SOLUTION ...... .......... ...... ... ...... .... .. ......... ............... ... ...... 60 - 7


DAY 1 -EXAM
ENGINEERING MATHEMATHICS
11-1
6. Find the remainder if we divide:
DAY l- EXAM 4y3 + 18y2 + 8y - 4 by (2y + 3)
A. 10
1. At what time between 6 and 7 o'clock B. 15
are the hands of the clock at right C. 11
angles? D. 13
"~....·, A. 6:16.32
B. 6:42.09 7. A man finds the angle of elevation of
C. 6:32.32 the top of a tower to be 30 degrees. He
D. 6:55.09 walks 85 m nearer the tower and finds
its angle of elevation to be 60 degrees.
2. How many pounds of salt must be What is the height of the tower?
added to a solution containing 5 lb. of A. 76.31 m
salt and 35 lb. of water so that the B. 73.16 m
resulting mixture will be 40% salt? C. 73.31 m
A. 12.3 lbs D. 73.61 m
B. 25.31bs
C. 18.3 lbs 8. The sides of a triangle are 8 em, 10
D. 36.3lbs em and 14 em. Determine the radius of
the inscribed and circumscribed circle.
3. How many liters of a 60% solution of A. 3.45, 7.14
nitric acid should be added to 40 liters of B. 2.45, 7.14
a 30% solution to obtain a 50% solution C. 2.45, 7.14
of the acid? D. 3.45, 8.14
A. 50
B. 80 9. If the length of the latus rectum of an
C. 65 ellipse is three-fourth of the length of its
D. 95 minor axis, find its eccentricity.
A. 0.15
4. A tank can be filled by one pipe in 9 B. 0.55
hours and by another pipe in 12 hours. C. 0.33
Starting empty, how long will it take to fill D. 0.66
the tank if water is being taken out by a
third pipe at rate per hour equal to one- 10. If the distance x from the point of
sixth the capacity of the tank? departure at time t is defined by the
A. 18 hrs equation x = -1 6t2 + 5000t + 5000, what
B. 15 hrs is the initial velocity?
C . 21 hrs A. 2000
D. 36 hrs B. 5000
C. 0
5. Mary is 24 years old. Mary is twice D.3000
as old as Ann was when Mary was as 11. How many years will P100,000
old as Ann is now. How old is Ann? earned a compound interest of P50,000
A. 16 yrs old if the interest rate is 9% compounded
B. 18 yrs old quarterly?
C. 24 yrs old A. 4.55
D. 72 yrs old B. 5.68
C. 3.55
D. 2.35

REFRESHER MANUAL 2nd Edition by JAS TORDILLO


1-21 DAY 1 - EXAM DAY 1 - EXAM 11-3
ENGINEERING MATHEMATHICS
12. By the condition of a will, the sum of 17. Cast iron weighs 650 pounds per
P20,000 is left to a girl to be held in trust cubic foot. The weight of a cast iron 28. The measure of the ability to transfer
and oil density is 0.85 gm/ml. Find the
fund by her guardian until it amounts to block 18" x 12" x 20" is: heat to other bodies . based on a
volume flow in liters per sec.
P50,000. When the girl received the A. 730 kg reference temperature where a body
A. 28.27
money, if the fund is to be invested at B. 2000 kg has given up all the thermal energy it
B. 72.22
8% compounded guarterly? C. 1625 kg
A. 7.98 yrs • D. 2570 kg
c. 22.22 possibly can;
D. 82.62 A. zero pressure
B. 11.57 yrs , .. B. gage pressure
C. 10.34 yrs 18. A solid disk flywheel (I= 500 kg-m 2)
23. Air is compressed in a diesel engine C. vacuum pressure
D. 10.45 yrs is rotating with a speed of 1800 rpm.
from an initial pressure of 100 kPa and a D. absolute temperature
What is its rotational kinetic energy?
temperature of 30 deg C to 1/12 of its
13. A P10,000.00 loan is to be paid off A. 888 x 10 to the 3~ power J
initial volume. Assuming the 29. In fluids flow, if the fluid travels
in 10 equal payments. The annual · 8. 668 x 10 to the 3'd power J parallel to the adjacent layers and the
compression to be adiabatic, calculate
interest rate is 15%. How much t interest C. 6.8 x 10 to the 61h power J
paths of individual particles do not
will be paid in the first two (2) years? x
D. 8.8 10 to the 61h power J the final temperature.
A.;318.7°C cross, the flow is said to be:
A. P3,005.1 0 A. uniform
19. What is the name for a vector that
B. 545.7 °C
B. P2,940.20 B. turbulent
C. 218.7 °C
C. P2,970.20 represent the sum of two vectors?
D. 818.7 °C C. laminar
D. P2,926.10 A. moment
D. critical
B. scalar
24. The first derivative of kinetic energy
14. Find the nominal rate which if C. torque
with respect to time is: 30. Deflection of a beam is
converted quarterly could be used D. resultant
A. power A. proportional to the modulus of
instead of 12% compounded semi- elasticity and moment of inertia
B. force
annually. 20. Determine the super elevation of the B. proportional to the load and inversely
C. work
A. 10.58% outer rail of a 3-m wide railroad track on
D. momentum to the length squared
B. 11.28% a 10° curve is one which a chord 30 m C. inversely proportional to the modulus
c. 9.38% long subtends an angle of 10° at the 25. The measurement of fluids of elasticity and moment of inertia
D. 11.82% center). Assume velocity of 45 mph. D. inversely proportional to the weight
resistance when acted upon by external
A. 0.69 m times the length
force is:
15. XYZ has a selling price of P400. If B. 1.5m
A. viscosity
its selling price is expected to decline at C. 1.0 m 31. Varignon's theorem is used to
B. flash point
a rate ·of 10% per annum due to D. 1.69 m
C. density determine.,.-..,..,--.--
obsolescence. what will be its selling· A. moment of inertia
D. tackiness
price after 5 years? 21. An ideal ·gas is contained in a B. mass moment of inertia
A. P213.10 vessel of unknown volume at a pressure C. location of centroid
26. It is an increased pressure
B. P200.00 of 1 atmosphere. The gas is released D. moment of area
developed on impact with a pilot tube as
C. P249.50 and allowed to expand into a previously
a result of localized kinetic energy
D. P236.20 evacuated bulb whose volume is 1-liter. 32. Study of motion with reference to
reduction to zero.
Once equilibrium has been reached, the the force which causes the motion is:
A. critical pressure
16. Assuming a shaft output of 5 MW temperature remains the same while the A. kinematics
B. saturation pressure
and a fuel rate of 40 lbs/min. What is pressure is recorded as 650 millimeters B. kinetics
C. stagnation pressure
the overall thermal efficiency of the of mercury. What is the unknown C. statics
D. vacuum pressure
machine? (HHV of fuel is 23,000 volume, V. of the first bulb? 27. If a process is carried out in· such a D. dynamics
Btu/lb). A. 3.51i manner that at every instant. the system
A. 25.2% B. 5.9 li
departs only infinitesimally from an 33. An impulse causes:
B 30.9% C. 0.91i A. the object's momentum to decrease
equilibrium state.
c. 28.9% D. 4.5 li A. Isentropic process B. the object's momentum to increase
D. 29.9% C. the object's momentum to change•
B. Non-flow process
22. Oil flow through a 30 tubes on a D. the object's momentum to remain
C. Throttling process
single cooler with a velocity of 3 m/s. constant
D. Quasi-static process
The internal diameter of tube is 20 mm
REFRESHER MANUAL 2nd Edition by JAS TORDILLO
REFRESHER MANUAL 2"d Edition by JAS TORDILLO

1-41 DAY 1 - EXAM DAY 1 -EXAM ~1-5
I: 34. It is the ratio of the ultimate stress ENGINEERING MATHEMATHICS
~ .
40. Ratio of the outlet stagnation
to the allowable stress. pressure to the inlet stagnation pressure
A. proportionality constant is referred as: 46. A hoist with a 80 hp engine is C. 383.68 m
B. stratn A. diffuser efficiency capable of lifting a 8,000 pounds load a D. 483.68 m
C. bulk modulus B. nozzle efficiency height of 30 feet in 10 seconds. The
D. factor of safety C. isentropic flow efficiency efficiency of this machine in % is: 52. A shot is fired at an angle of 35° with
D. weir flow efficiency A. 60% the horizontal and a velocity of 150 m/s.
35. An open system. first law should be B. 50% Calculate the range of the projectile in
utilized for all the fbiiOV>(ing, EXCEPT: 41. A device used to keep moisture C. 40% meters.
A. a turbine ' from passing through the system is D. 20% A. 1055.25 m
B. a pump called: · B. 1155.25 m
C. a piston-cylinder device with no inlet A. trap 47. 1 torr is equivalent to a pressure of C. 2155.25 m
and exhaust valves B. aerator D. 3155.25 m
D. a nozzle C. humidifier A. 1 atmosphere
D. dehydrator B. 14.7 psia 53. A highway curve has a super
36. An increase in heat enthalpy of a C. 2 mm Hg elevation of 10 degrees. What is the
substance where it undergoes a change 42. The locus of the elevations to which D. 1/760 atmosphere radius (in feet) of the curve such that
of phase at constant pressure and water will rise in a piezometer tube is there will be no lateral pressure between
temperature. termed: 48. 1 calorie is equivalent to: the tires and the roadway at a speed of
A. heat of fusion A. critical depth A. 4.187 j 60 mph?
B. heat of transformation B. stagnation pressure B. 4.187 kJ A . 1363.92 ft
C. heat of crystallization C. hydraulic gradient C. 252 Btu B. 2363.92 ft
D. heat of vaporization D. energy gradient D. 778 ft-lb C. 3363.92 ft
D. 4363.92 ft
37. A very important consequence of 43. Amount of heat needed to raise the 49. Which of the statements is true?
the ideal gas model is that the internal temperature of one pound of that A. Entropy always decreases. 54. A ball is thrown upward with an
energy of an ideal gas is a function of substance one degree Fahrenheit is B. Entropy increases up to the critical initial velocity of 20 m/s. How high does
_ _ only. A. BTU • temperature, then it decreases. it go?
A. pressure B. Calorie C. Theoretically, entropy may be zero at A. 10.38 m
B. point function C. Se11sible heat a less er'lough temperature. B. 20.38 m
C. temperature D. Centigrade heat unit D. Entropy does not change in a C. 30.38 m
D. volume thr.ottling process. D. 40.38 m
44. An ideal gas is contained in a rigid
38. A thermodynamic system which container. There is no work of a rotating 50. An adiabatic process: 55. A traveler in a truck on a level road
undergoes a cyclic process during a shaft associated with the container. Any A. allows heat transfer into the system sees an airplane traveling in the same
positive amount of work is done by the heat transfer is a function of; but not out of the system direction climbing at an angle of 35°
system. A. pressure B. allows heat transfer out of the system above the horizontal. By driving at 120
A. heat engi ne B. heat transfer equal to work but not into the system kph the traveler is ab le to stay ciirectly
B. heat pump c. volume only C . may be reversible below the airplane. Find the airplane's
C. reversible-irreversible process D. temperature only D. is one in which enthalpy remains velocity.
D. re~ersed rankine cycle unchanged A. 97.7 kph
45. If a particle has a velocity of 10 B. 146.5 kph
39. It is an increased pressure meters per second and a kinetic energy 51. A rubber ball is thrown from a C. 127.5 kph
developed on impact with a pitot tube as of 1000 joules, then the mass, in building at an angle of 60° with the D. 210.5 kph
a result of localized kinetic energy kilograms of this particle must be ; horizontal at an initial velocity of 38
reduction to zero. A. 100 m/sec. After hitting level 'ground at the 56. A motorcyle weighing 15 kN is on a
A. critical pressure B. 20 base of the building, it has covered a hill that makes an angle of 20° with the
B. stagrjation pressure C. 80 total distance of 200 m. How tall is the horizontal. Find the components of the
C. saturation pressure D. 10 building in m? motorcycle's weight parallel and
D. vacuum pressure A. 183.68 m perpendicular to the road.
B. 283.68 m A. 5.1kNand14.1kN

REFRESHER MANUAL 2nd Edition by JAS TORDILLO


REFRESHER MANUAL 2"d Edition by JAS TORDILLO
~· - -- -

DAY 1 - SOLUTION ~1-7


1- 6 I DAY 1 - SOLUTION ENGINEERING MATHEMATHICS
B. 5.1 kN and 13.3 kN
C. 7.1 kN and 15.3 kN DAY 1 -SOLUTION
4. A tank can be filled by one pipe in 9 c. 11 *
D. 9.1 kN and 17.3 kN hours and by another pipe in 12 hours. D. 13
1. At what time between 6 and 7 o'clock
Starting empty, how long will it take to fill
are the hands of the clock at right

r r s(
57. A box suspended by a rope is pulled the tank if water is being taken out by a f(y) = 4l + 18y2 + 8y- 4
to one side by a horizontal force. The angles? ·
third pipe at rate per hour equal to one-
tension in the rope
A. is less than before
A. 6 :16.32 *
B. 6:42.09 of a certain commodity
sixth the capacity of the tank?
A. 18 hrs
t( -23) = 4( -23 + 1{ -23 + -23 )- 4
B . is unchanged ' c. 6:32.32 sale at a certain price at
D. 6:55.09 B. 15 hrs
C. is greater than before time: C. 21 hrs t(-;)=11
D: any of the above ' X D. 36 hrs *
X+ 15 = 30 +-
12
58. A ball is thrown upward from the Let x =the number of hours required for 7. A man finds the angle of elevation of
edge of a cliff with an initial velocity of 7 ~ = 15 pipe A and pipe B to fi ll the tank the top of a tower to be 30 degrees. He
m/s. How fast is it moving 5 sec later? 12
walks 85 m nearer the tower and finds
A. 42 m/s upward direction 1
ao l buyers and sellers 1 1 1 1 its angle of elevation to be 60 degrees.
B. 42 m/s downward direction x = 11 or 16.36
x= 9+ 12-6 What is the height of the tower?
C. 33 m/s upward direction Time: 6:16.32 A. 76.31 m
D. 33 m/s downward direction 1 4+3-6 B. 73.16 m

59. A ball dropped from the top of a


X = 30 + .=_ + 15
12
.1
x-= 36 C. 73.31 m
D. 73.61 m *
building and strikes the ground 5 .sec llx X = 36 hrs
later. How high is the bridge? 12 = 45 , of P50,000 is deposited h = c sin 30°
A. 122.6 m · · much money are you 5. Mary is 24 years old. Mary is twice
B. 140m x = ~ = 49.1 after 8 years at 8%
11 as old as Ann was when Mary was as Solving for c:
C. 135.6 m Time: 6:49.1 Jally? old ·as Ann is now. How old is Ann? c 85
D. 160m A. 16 yrs old = sin300
2. How many po~_salt must be
sin120°
B. 18 yrs old • c = 147.22 m
60. A boat travels 10 km in 30 min. added to a solution containing· 5 lb. of C. 24 yrs old
What is its speed in kilometers per salt and 35 lb. of water so that the D. 72 yrs old
hour? h = (147.22) sin 30°
resulting mixture will be 40% salt? h = 73.61 m
A. 10 kph A. 12.3 lbs -- -· ¥'"' vovc::; mat Let x = Ann's present age
B. 15 kph B. 25.3 lbs human and will be 24 = Mary's present age
c. 20 kph c. 18.3 lbs * ney is available after the 8. The sides of a triangle are 8 em, 10
D. 30 kph Present Ages Past Ages em and 14 em. Determine the radius of
D. 36.3 lbs """"e been obtained.
the inscribed and circumscribed circle.
)~ )
35 + x(O) = (40 + x)(0.60)
Mary
Ann x
24 ; s ( ? )
? = x- 4- x)
A. 3.45, 7.14
B. 2.45, 7.14
E DAY 1- ANSWER KEY 35 = 24 + 0.6x c. 2.45, 7.14 *
( l.A 16. B 31. C 46. A X ·= 18.331bs 24 -
difference 24 - x X
D. 3.45, 8.14
[ 2.C 17.A 32.A 47.0
3. B 18. D 33. C 48. A
3. How many liters of a 60% solution of A. Inscribed circle
4. D 19. D ~.DGC 24 = 2(x- (24 - x)]
s.;,e liC~C
nitric acid should be added to 40 liters of
20.A 24 = 2(x + x- 24) At= r s
l.f ~-C 21. 8 ~ana a 30% solution to obtain a 50% solutio1.1
24 = 4x -48 s= 14 + 10 +8= 16
7. D 22. A 37. C 52.C of the acid? .e same quantity even
8. C 23. B aA~A A. 50 ·ice varies considerably: x = 18 yrs old 2
9. D 24. A -B~B B. 80 * s Hero's formula:
10. B 25. A -A~B c. 65 goods 6. Find the remainder if we divide: At= Jt6{16-14XJ6-IOX16-8)
11. A 26. C no~A
D. 95 4l + 18y2 + 8y- 4 by (2y + 3)
12. B 27. 0 ncnc A. 10 At =3~.19
13. D 28. D UA~B
14. D 29. C 44. D 59. A 0.30(40) + 0.60x = 0.50(40 + x) B. 15
15. D 30. C 45. B 60. C 12 + 0.60x = 20 + 0.50x ,492.13
0: x = 80 liters and the REFRESHER MANUAL 2nd Edition by JAS TORDILLO
REFRESHER MANUAL 2nd· Edition by JAS TORDILLO
1- 8 I DAY 1 - SOLUTION
DAY 1 - SOLUTION 11 ·9
ENGINEERING M ATH EMATHICS
39.19 V= dx
r=--
16 dt C. P2,970.20 obsolescence, what will be its selling
r =2.45cm v = -2(16)t + 5000 D. P2,926.1 0 * price after 5 years?
A. P21 3.10
B. Circumscribing circle B. pzoo,oo
[~]
Initial velocity means the velocity of t he
A = abc body when t 0. = P=R
~l +iY'
C. P249.50
1 D. P236.20 * I
4r '
_ abc _14(to}(8) • v = -32(0) + 5000 = 5000 10,000 = R [ (1 + 0. 15)
10
- I]
I
r- ----,,...__,~
Selling Price= P(1 - i)" )
4A I 4(39.19) 0.15(1 + 0.15)10 = 400(1 - 0.10) 5
11. How many years will P100,000
r = 7.14 em earned a compound interest of P501000 = P236. 20
it the interest rate is 9% compounded R = P1992.52 .
9. If the length of the latus rectum of an quarterly? s 1 = 1o,ooo(1 + 0.15)1 = P11 ,5oo·· 16. A ssuming a shaft output of 5 MW
ellipse is three-fourth of the length of its A. 4 .55 * l1 = 11,500- 10,000 = P1 ,500 · and a fuel rate of 40 lbs/min. What is
minor axis, find its eccentricity. B. 5.68 p1 ':: 11 ,500-1992.52 = 9507.48 the overall thermal efficient:y of the
A. 0.15 C. 3.55 s 2 = (9507.48) (1 +0.15) 1 = P10 1933.6 machine? (HHV of fuel is 23,000
B. 0.55 D. 2.35 l2 = 10,933.6- 9507.48 = P1426.12 Btu/lb).
C. 0.33 Interest Paid in 2 years= l1 + l2 A. 25.2%
D. 0.66 * S = P(1 + i)" = 1500 + 1426.12 = P2926.12 B. 30.9% *
0 09 C. 28.9%
i= · = 0.0225 20,000
D. 29.9%

~~ ~~-
4
+ + -· + . +...J
LR = 2b2
S = P +I I · ..
a 1001000 + 50,000 = 100,000 (1 + Thermal Eff = Output
0 1 .2 3 4 ... .. ......... .. .... 10 Input
~ 0.0225)."

I~! t t! !
LR = (length of the minor axis)
3 4 n log (1.0225) = log 1.5 soooR
n = 4.55 yrs = sec
=~(2b) 40 ~ (23000 Btu~ . min _ x 1.0551.<_,!
2b1
a
3 .
3 12. By the condition of a will, the sum of
P20 1000 is left to a giri to be held in trust
p
10,000
m10
= 30.9%
lb 60 sec Btu

b = -~ fund by her guardian until it amounts to


4 17. Cast iron weighs 650 pounds per
P50 1000. When the girl received the
= a2- b2 14. Find the nominal rate which if

r
c2 cubic foot. The weight of a cast iron
money, if the fund is to be invested at converted quarterly could be used block 18" x 12" x 20" is:
2
c = a2- (~a 8% compounded quarterly?
A. 7.98 yrs
instead of 12% compounded semi-
annually.
A. 730 kg *
B. 2000 kg
B. 11.57 yrs * A. 10.58% C. 1625 kg
7 2 C. 10.34 yrs B. 11 .28% D. ~570 kg
c2 = 16 8 D. 10.45 yrs C. 9.38% I
. . c
Eccentnc1ty, e = - = 0.66
S =P(1 + i)" D. 11.82% *
.
We1ght = - (I8xl2x20)
12 -12 -12(650) = 1625 lbs
a 4 Effective rate Quarterly
50 000 =20 000 (1 + O.OS) n
= Effective rate Semi-annually = 738.6 kg

0.12)2-1 I
I I

10. If the distance x from the point of


4
4 2
departure at time t is defined by the
4
=
(1 .02) " 2.5 in) (
( 1+4 -1 = 1+ 2
18. A solid disk flywheel (I = 500 kg-m )
equation x = -16f + 5000t + 5000, what n = 11.57 yrs is rotating with a speed of 1800 rpm.
is the Initial velocity? in = 11.82% compounded quarterly What is its rotational kinetic energy?
A. 2000 13. A P10,000.00 loan is to be paid off 15. XYZ has a selling price of P400. If A. 888 x 10 to the 3«~ power J
B. 5000 * in 10 equal payments. The annual its selling price is expected to decline at B. 668 x 10 to the 3'd power J
c. 0 interest rate is 15%. How much t interest a rate of 10% per annum due to C . 6.8 x 10 to the 6th power J
D. 3000 will be paid in the first two (2) years? D. 8.8 X 10 to the 61h power J *
A. P3,005.1 0
X = -16t2 + 5000t + 5000 I B. P2,940.20 REFRESHER MANUAL 2nd Edition by JAS TORDILLO
REFRESHER MANUAL 2"d Edition by J~S TORDILLO
1- 1ol DAY 1 - SOLUTION DAY 1 - SOLUTION 11 - 11
ENGINEERING MATHEMATHICS
w = 1800"ev x lmin x 27tfad = _ rad B. 5.9 1i"'
188 5 C. 0.91i
min 60sec rev sec C. work C. laminar*
D. 4.5 li D. momentum D. critical
KE = Y. lro2 = Y. (500 kg-m2 )(188.5)2
KE = 8,883,062.5 N-m ,P1 =
1 atm = 760 mm Hg KE = Y. mv2 30. Deflection of a beam is
KE = 8.8 x 108 .I PN1 = P2V2 A. proportional to the modulus of
dKE dv
760V1 = 650(1 + V1)
I. 19. What is the-.ni(lrTle for a vector that
represent the sum of'two vectors?
v1 = s.91i
--=mv- = mva=Fv -+power
dt dt elasticity and moment of inertia
B. proportional to the load and inversely
to the length squared
22. Oil flow through a 30 tubes on a 25. The measurement of fluids
A. moment C. inversely proportional· to the
B. scalar single cooler with a velocity of 3 m/s. resistance when acted upon by external
modulus of elasticity and moment of
The internal diameter of tube is 20 mm force is:
C. torque inertia*
.
D. resultant * an,d oil density is 0.85 gm/ml. Find the
volume flow in liters per sec.
A . viscosity *
B. flash point
D. inversely proportional to the weight
times the length
A. 28.27 * C. density
20. Determine the super elevation of the
B. 72.22 ~ - tackiness
outer rail of a 3-m wide railroad track on 31. Varignon's theorem is used to
a 10° curve is one which a chord 30 m c. 22.22
determine - - -- - -·
D. 82.62 26. It is an increased pressure
long subtends an angle of 1oo at the developed on impact with a pitot tube as A. moment of inertia
center). Assume velocity of 45 mph. a result of localized kinetic energy
B. niass moment of inertia
Q = AV
A. 0.69 m * reduction to zero.
C. location of centroid *
B. 1.5 m Q = ~(0.02) 2 {3) (30) A . critical pressure
D. moment of area
C. 1.0 m 4
B. saturation pressu re
D. 1.69 m = 0.0282 m 3/sec 32. Study of motion with reference to
C. stagnation pressure "'
Q = 28.27 lilsec the force which causes the motion is:
D. vacuum pressure
V = 45 mph = A. kinematics *
23. Air is compressed in a diesel engine B. kinetics
Sin 5°= ~ from an initial pressure of 100 kPa and a
27. If a process is carried out in such a
C. statics
r manner that at every instant, the system
r = 172.1 m temperature of 30 deg C to 1/12 of its
departs only infinitesimally from an
D. dynamics
y2 initial volume. Assuming the
equilibrium state.
e=
~h
tan compression to be adiabatic, calculate 33. An impulse causes:
gr
A. Isentropic process
the final temperature. A. the object's momentum to decrease
B. Non-flow process
20.12
A . 318.7 °C B. the object's momentum to increase
C. Throttling process
tan e= 9.81(172.1)
B. 545.7 °C *
D. Quasi-static process •
C. the object's momentum to
C. 218.7°C change*
e= 13.4° (banking angle) D. 818.7°C
28. The measure of the ability to transfer
D. the obj ect's momentum to remain
sin 13.4°
3
= ~ T1 = 30 + 273 = 303°K . heat to other bodies based on a
reference temperature where a body
constant

h = 0.69 m (superelevation of railroad V2 = 1112 v 1 34. It is the ratio of the ultimate stress
has given up all the thermal energy it
to the allowable stress.
track) Tz =(vi )k-1 possibly can;
A. proportionality constant
TJ v2 A. zero pressure
21. An ideal gas is contained in a B. strain
B. gage pressure
vessel of unknown volume at a pressure 1.4- 1 C. bulk modulus
C. vacuum pressure
T2 Yt D. factor of safety *
of 1 atmosphere. The gas is released
and allowed to expand into a previously rr
303 = [ Yt )
D. absolute temperature *
35. An open system, first law should be
evacuated bulb whose volume is 1-liter. 29. In fluids flow, if the fluid travels
Once equilibrium has been reached, the
T2 = 818.7 °K = 545.7°C utilized for all the following, EXCEPT:
~aral lel to the adjacent layers and the
temperature remains the same while the A . a turbine
24. The first derivative of kinetic energy paths of individual particles do not
pressure is recorded as 650 millimeters B. a pump
w ith respect to time is: cross, the flow is said to be:
of mercury. What is the unknown A. uniform
A. power*
volume, V, of the first bulb? B. turbulent
B. force
A. 3.51i REFRESHER MANUAL 2nd Edition by JAS TORDILLO
REFRESHER MANUAL2"d Edition by JAS TORDILLO
1-12J DAY 1 - SOLUTION
DAY 1 - SOLUTION 11-13
ENGIN~ERING MATHEMATHICS
C. a piston-cylinder device with no B. aerator
inlet and exhaust valves * C. humidifier
C. 383.68 m
D. a nozzle D. dehydrator* Efficiency = Power Output D. 483.68 m
Power Input 2
36. An increase in heat enthalpy of a 42. The locus of the elevations to which
8,000-lbx30-ft hp y=xtane - ~
substance where.it undergoes a change water will rise in a piezometer tube is - - X .....- . --
2V0 2 cos2e
) 0-s SOOft-1%••
of phase at constant pressure and termed:
80hp
temperature. , A. critical depth -h = 150 tan 60o _ 9.81(200)2
A. heat of fusion B. stagnation pressure Efficiency = 0.60 = 60%
• .- · 2(38)2 cos2 60
B. heat of transformation * C. hydraulic gradient*
47. 1 torr is equivalent to a pressure of h = 283:68 m
C. heat of crystallization D. energy gradient
D. heat of vaporization
A. 1 atmosphere 52. A shot is fired at an angle of 35° with
43. Amount of heat needed to raise the
B. 14.7 psia the horizontal and a velocity of 150 mls.
37. A very important consequence of temperature of one pound of that
C. 2 mm Hg Calculate the range of the projectile in
the ideal gas model is that the internal substance one degree Fahrenheit is
D. 11760 atmosphere* meters.
energy of an ideal gas is a function of A. BTU*
A. 1055.25 m
_ _ only. B. Calorie
48. 1 calorie is equivalent to: B. 1155.25 m
A. pressure C. Sensible heat
A. 4.187 J * C. 2155.25 m *
B. point function D. Centigrade heat unit
B. 4.187 kJ D. 3155.25 m
C. temperature*
D. volume 44. An ideal gas is contained in a rigid C. 252 Btu
container. There is no work of a rotating D. 778 ft-lb v2 . 28
Range = 0 s1n = (150)2 sln2(35)
38. A thermodynamic system which sh.aft associated with the container. Any 9 9.81
undergoes a cyclic process during a heat transfer is a function of; 49. Which of the statements is true?
A. Entropy always decreases. = 2155.25 m
positive amount of work is done by the A. pressure
system. B. heat transfer equal to work B. Entropy increases up to the critical
temperature, then it decreases. 53. A highway curve has a super
A. heat engine * C. volume only
C. Theoretically, entropy may be zero elevation of 10 degrees. What is the
B. heat pump D. temperature only *
radius (in feet) of the curve such that
C. reversible-irreversible process at a less enough temperature. *
D. Entropy does not change in a there will be no lateral pressure between
D. re~ersed rankine cycle 45. If a particle has a velocity of 10
throttling process. the tires and the roadway at a speed of
meters per second and a kinetic energy
60 mph?
39. It is an increased pressure of 1000 joules, then the mass, in
50. An adiabatic process: A. 1363.92 ft *
developed on impact with a pitot tube as kilograms of this particle must be ;
A. allows heat transfer into the system B. 2363.92 ft
a result of localized kinetic energy A. 100
but not out of the system · C. 3363.92 ft
reduction to zero. B. 20*
B. allows heat transfer out of the system D. 4363.92 ft
A . critical pressure C. 80
B. stagnation pressure * D. 10 • but not into the system
C. saturation pressure
2
C. may be reversible * V = 60 mi(5280ft)(~)
D. vacuum pressure KE = Y2 mv D. is one in which enthalpy remains hr ml 3600
1000 = · Y2 m(10)2 unchanged V = 88 ftlsec.
40. Ratio of the outlet stagnation m = 20 kg
pressure to the inlet stagnation pressure 51 . A rubber ball is thrown from a v2
tan e = -
IS referred as: 46. A hoist with a 80 hp engine is building at an angle of 60° with the gr
A . diffuser efficiency * capable of lifting a 8,000 pounds load a horizontal at an initial velocity of 38
B. nozzle efficiency height of 30 feet in 10 seconds. The mlsec. After hitting level ground at the tan10°= (88)2
base of the building, it has covered a (32.2)(r)
C. isentropic flow efficiency efficiency of this machine in % is:
D. weir flow efficiency A. 60% * total distance of 200 m. How tall is the r = 1363.92 ft
B. 50% building in m?
41 . A device used to keep moisture c. 40% A. 183.68 m 54. A ball is thrown upward with an
from passing through the system is D. 20% B. 283.68 m * initial velocity of 20 m/s. How high does
called: it go?
A. trap REFRESHER MANUAL 2nd Edition by JAS TORDILLO
REFRESHER MANUAL 2"d Edition by JAS TORDILLO
1-14j DAY 1 - SOLUTION DAY 2- EXAM ~2-1
A. 10.38 m 58. A ball is thrown upward from the
ENGINEERING MATHEMATHICS
B. 20.38 m * edge of a cliff with an initial velocity of 7
C. 30.38 m 6. The volume of water in a spherical
D. 40.38 m
m/s. How fast is it moving 5 sec later? DAY 2 - EXAM tank having a diameter of 4 m is .5.236
A. 42 m/s upward direction 3
B. 42 m/s downward direction* m . Determine the depth of the water in
1. The sum of two numbers is 35 and
V? = Vo2 - 2gh C. 33 m/s upward direction the tank.
their product is 15. Find the sum of their
o = (2W- 2(9.B1)(h) D. 33 m/s downward direction reciprocal.
A. 1.6
h = 20.38 m B. 1.2
A. 2/7
' ' V = Vo+gt =7+(-9.8)(5) = -42m/s B. 2/3
C. 1.4
D. 1.0
55. A traveler in a truck on a level road C . 7/3
sees an airplane traveling in the same 59. A ball dropped from the top of a D. 5/2
direction climbing at an angle of 35° building and strikes the ground 5 sec 7. Given of diameter x and altitude of h.
above the horizontal. By driving at 120 later. How high is the bridge? What percent is the volume of the largest
2. The seating section in a coliseum has cylinder which can be inscribed in the
kph the traveler is able to stay directly A. 122.6 m* 30 seats in the first row, 32 seats in the
below the airplane. Find the airplane's B. 140m cone to the volume of the cone?
second row, 34 seats in the third row, A. 44%
velocity. C. 135.6 m and so on, until the tenth row is reached , B. 46%
A. 97.7 kph D. 160m after which there are-ten rows each C. 56%
B . 146.5 kph *
contain ing 50 seats. Find the total D. 65%
C. 127.5 kph h = Y:z gf = Y:z (9.81)(5)2 = 122.6 m number of seats in the section.
D. 210.5 kph
A. 900 8. The major axis of the elliptical path in
60. A boat travels 10 km in 30 min. B. 890
Let v. = truck's velocity as the horizontal What is its speed in kilometers per C. 810
which the earth moves around the sun is
component of V hour? approximately 186,000,000 miles and the
D. 390 eccentricity of the ellipse is 1/60.
V = the airplane's velocity A. 10 kph
B. 15 kph Determine the apogee of the earth.
3. A bookstore purchased a bestselling
v. = v cos 9 c. 20 kph* book at P.200 per copy. At what price
A. 93,000,000 miles
B. 91 ,450,000 miles
v = -vx- = 120
-- = 146.5-kph
D. 30 kph should this book be sold so that, giving a C. 94,335,000 miles
cosa cos35o 20% discount, the profit is 30%.
30 min= 0.5 h D. 94,,550,000 miles *
56. A motorcyle weighing 15 kN is on a A. P450
hill that makes an angle of 20° with the v = -s =10- = 20 kph B. P357 9. Determine B such that 3x + 2y- 7 = 0
horizontal. Find the components of the 0.50 C. PSOO is perpendicular to 2x - By + 2 = 0.
motorcycle's weight parallel and D. P400 A.S
perpendicular to the road. ~
B.3
A. 5.1 kN and 14.1 kN * 4. If the sides of a parallelogram and in C. 4
B. 5.1 kN and 13.3 kN included aragle are 6, 10 and 100 degrees D. 2
C. 7.1 kN and 15.3 kN
D. 9.1 kN and 17.3 kN .. respectively, find the length of the shorter
diagonal. 10. What is the x-intercept of the line
A. 10.63 passing through (1, 4) and (4, 1).
F, = w.sin 9 = 15 sin 20° = 5.1 kN B. 10.73 A. 4.5
F2 = w cos 9 = 15 cos 20° = 14.1 kN C. 10.37 B.6
D. 10.23 C. 5
57. A box suspended by a rope is pulled D.4
to one side by a horizontal force. The 5. Determine the area of a regular 6-star
tension in the rope polygon if the inner regular hexagon has 11. A man on a wharf 3.6 m above sea
A. is less than before 10 em sides. level is pulling a rope tied to a raft at 0.60
B. is unchanged A. 441.66 cm 2 m per second. How fast is the raft
C. is greater than before* B. S19.60 cm2 approaching the wharf when there are 6
D. any of the above ' C. 467 .64 cm 2 m of rope out?
D. 493.62 cm 2 A. -0.95 m/s
B. -0.75 m/s
REFRESHER MANUAL 2nd Edition by JAS TORDILLO
REFRESHER MANUAL 2"d Edition by JAS TORDILLQ~
\
DAY 2- EXAM ~ 2-3
2-2 DAY 2 - EXAM
ENGINEERING MATHEMATHICS
C. -0.85 m/s 17. A man borrowed P300,000 from a
D. -0.65 m/s lending institution which will be paid after increase the withdrawal by $1000 at the 27. is the locus of a point
10 years at an interest rate of 12% end of each successive year. How much that moves in a plane of that the
12. A flywheel of radius 14 inches is compounded annually. If money is worth money must the engineer have in his difference of the distances from two fixed
rotating at the rate of 1000 rpm. How fast 8% per annum how much should be savings account at the start of his points of the plane is constant.
does a point on the rim travel in ftlsec? deposit to a bank monthly in order to retirement, if money earns 6% per year, A. hyperbola
A. 122 • discharge his debt 10 yrs hence? compounded annually? B. circle
B. 100 ...; ., A. P2,798.52 A. $206 116.59 C. ellipse
C. 1456 ~ -.: B. P4,672.31
B. $126 116.59 D. parabola
D. 39 C. P3,952.50 C. $146 116.59
D. P5,093.06 D. $106 116.59 · 28. Intermittent and filters are primarily
13. A higt:lway curve has a super used to:
elevation of 7 degrees. What is the radius 18. A young engineer borrowed P10,000 22. What is the annual rate of simple A. trap the substance
of the curve such that there will be no tit 12% interest and paid P2000 per
interest if $265 is earned on four months B. determine the flow
lateral pressure between the tires and the annum for the first 4 years. What does he on an investment of $15 000? C. oxidize putrescible matter
roadway at a speed of 40 mph? have to pay the end of the 51h year in
A, 5.3% D. convert water to gas
A. 265.71 m order to pay off the loan?
B. 4.5%
B. 345.34 m A. P6,922.93 c. 3.5% 29. To divert excessive flow from
C. 438.34 m B. P9,622.93 D. 7.5% combined sewers designers often use:
D. 330.78 m C. P6,292.93 A. a overhead trap
D. P9,262.93 23. The place where buyers and sellers B. an extension pipe
14. A marksman fires a rifle horizontally come together. C. a spillway
at a target. How much does the bullet 19. The corporation purchased a D. a leaping weir
A. market
machine for 1 million. Freight and
drop in flight if the target is 150 m away B. recreation center
and the bullet has a muzzle velocity of installation charges amounted to 3% of 30. The most important factor in
C. business
500 m/sec. the purchase price. If the machine shall determining high-temperature behavior of
D. buy and sell section
A. 0.34 m be depreciated over a period of 8 years an alloy is:
B. 0.64 m with a salvage value of 12%, determine A. pressure
24. The quantity of a certain commodity
C. 0.44 m the depreciation charged during the 5th B. materials
that is offered for sale at a certain price at
D. 0.54 m year using the sum of the years digit
a given place and time. C. parameter
method. D. composition
A. demand
15. Engine oil at 409C (p = 875 kg/m 3 ) A. P101, 107.11
B. stocks
flow into a 2.5 em diameter horizontal B. P107,110.11 31. The new Mechanical Engineering
C. supply
pipe with a velocity of 1 m/s and exits C. P170,110.11 D. goods law which was signed last February 12,
through a nozzle of 0.5 em diameter. If D. P100,711.11 1998, is known as:
2
the pressure drop is 538 kN/m , what 25. A statement the truth of which follows A. RA 9584
must be the exit velocity. 20. A debt of P10,000 with 10% interest
with little or no proof from a theorem . B. RA 8495
A. 35 m/s compounded semi-annually is to be
A. axiom C. RA 2002
B. 30 m/s amortized by semi-annual payments over D. RA 7680
B. conclusion
C. 25 m/s the next 5 years. The first due is 6
C. hypothesis
D. 20 m/s months. Determine the semi-annual
D. corollary 32. Under the provisions of the PSME
payment. Code, what is the color symbol for air
16. What annuity is required over 12 A P1 ,200.00 26. What curve is represented by the piping?
years to equate with a future amount of B. P1, 193.90
equation r = ae. A. red
P20,000? Assume i = 6% annually. C. P1 ,295.05
A. Spiral of Archimedes B. light blue
A. P4, 185.54 D. P1 ,400.40
B. Four-leaves rose C. black
B. P2,185.54 D. green
C. cardioid
C. P3, 185.54 21. An engineer is planning for his 15-
D. Three-leaves Rose
D. P1,185.54 year retirement. In order to supplement
his pension and offset the anticipated
effects of inflation, he intends to withdraw
$5000 at the end of the first year, and to REFRESHER MANUAL 2nd Edition by JAS TORDILLO
REFRESHER MANUAL 2nd Edition by JAS TORDILLO
2-4 DAY 2 - EXAM DAY 2 -EXAM ~ 2-5
33. What is the ong1n of the energy 39. A check drawn from a depositor ENGINEERING MATHEMATHICS
conservation equation used in the flow against his account which the bank has
systems? marked certified. 46. Supplementary angles are angles
A. Daltons law A. manager's check whose sum is:
B. First law of thermodynamics B. personalcheck A. 45 deg G )
C. Pascals law C. certified check B. 180 deg DAY 2 - ANSWER KEY
D. Second law of thermodynamics D. company check C. 90 deg 1. c 16. 0 31. 8 46. 8
. ~
D. 360 deg 2. 8 17. 0
3. 8 18.A
32. 8 47. 0
33. 8 48. B
34. The process of exoh~nge where no 40. A written contract, under seal,
whereby a corporation binds itself to pay 47. The median of a triangle is the lien 4. 8 19. D 34. B 49. B
single medium of exchange Is used.
5. B · 20. C 35.C 50.0
A. exchange a specific sum of money to the owner of connecting a vertex and the midpoint of
6.0 21.0 36. B
B. liquidated damages tl)e bond. the opposite side. For a given triangle, 7.A 22. A 37. B
C. partial A. COrPoration bond these medians intersect at a point which 8.0 23. A 38.A
D. hunter B. coupon bond is called the: 9. B 24. c 39. c
C. collateral bond A, orthocenter 10. c 25.0 40. A
35. Means the sum agreed upon in D. sealed bond B. circumcenter 11. B 26. A 41.0
C. incenter 12. A 27. A 42. B
advance by the parties as compensation
13. A 28. C 43. c
for a breach. 41 . The actual accounts passing into and D. centroid
14. c 29.0 44. A
A. unliquidated damages out of the treasury of financial venture:
B. advance wages A. money market 48. Any number multiplied by
equals unity.
I (?3 I
15. A 30.0 45. A
)
C. liquidated damages B. bond
D. all of these C. check out A. infinity
D. cash flow B. its reciprocal
36. A written acknowledgement by· a C. itself
carrier that he has received the goods for 42. The number of several outcomes D. zero
shipment. divided by the number of possible
A. crate bill outcomes is: · 49. A sequence of decreasing numbers
B. Bill of Lading A. change or when the succeeding term is lesser
C. ticket B. probability than the preceding term is:
D. con.tract to ship C. combination A. convergent series
D. permutation B. divergent series
37. A letter of request whereby one C. polyhedral series
person, usually a merchant or banker, 43. The unit prefix nano is opposite to: D . discrete series
requests some other person to advance A. Mega
money or to give credit up to a certain B. Tera 50. A n angle greater than a straight line
amount. C. Giga and less than two straight angles is
A. check D. Hexa ca lled:
B. letter of credit A. rig ht angle
C. letter of debit 44. The altitudes of the sides of triangles B. acute angle
D. acknowledgement intersect at the point known as: C. obtuse angle
A. orthocenter ' D. reflex angle
38. lf an increase in the capital stock is B. centroid
made when actually no additional value C. circumcenter
has been paid either in cash or in D. incenter
money's worth:
A. stock watering 45. The arc length equal to the radius of
B. inflation the circle is called:
C. deflation A. 1 radian
D. depletion B. pi radian
C. 1 quarter circle
D. 1 grad
REFRESHER MANUAL 2nd Edition by JAS TORDILLO
REFRESHER MANUAL 2nd Edition by JAS TORDILLO
2-6 DAY 2 - SOLUTION ' DAY 2- SOLUTION ~2-7
B. P357 * ENGINEERING MATHEMATHICS
DAY 2 - SOLUTION C. P500
D. P400 6. The volume of water in a spherical approximately 186,000,000 miles and the
1. The sum of two numbers is 35 and tank having a diameter of 4 m is 5.236 eccentricity of the ellipse is 1/60.
their product is 15. Find the sum of t heir Let x = selling price of the book m3 . Determine the depth of the water in Determine the apogee of the earth.
reciprocal. 0.2 x = discount price of the book the tank. A. 93,000,000 miles
A. 1.6.
A.2n
B. 2/3 ..
...... ,tr_i"
0.30(0.8x) = profrt
B. 1.2
B. 91 ,450,000 miles
C. 94,335,000 mi l ~s
c. 7/3 * x- 0.2x = 200 + 0.3(0.8x) C. 1.4 D. 94,550,000 m iles *
D. 5/2 X= P 357.00 D. 1.0 *
1
x = first number 4, If the sides of a parallelogram and in c = ae = 93,000,000 ( )
y = second number V=
7th 2 [3r -h ]
-
60
includeo angle are 6, 10 and 100 degrees
3 c = 1,550,000 miles
respectively, find the length of the shorter
X+ y = 35 Eq. 1 diagonal. 5.236 = 7th 2(3(2) - h] Apogee= 93,000,000 + 1,550,000
xy =·15 Eq. 2 A 10.63 3 Apogee = 94,550,000 miles
B.10.73 ..
Sum of their reciprocal:
1 I x+y
C. 10.37
D. 10.23
3
h - 6h2 + 5 = 0
(h - 5)(h - 1) =0 ,. ... .... - ---' ...
-+-=-- h = 5 m (absurd ; for 5 > 4) Earth / b Sun ' Earth
X y xy
Cosine Law
h =1m ap;ge-;,·Q·····~·-··<>-·· ·c·-~0--.-~
~ / Pengee
1 1
-+-=-or-
x y IS 3
35 7 d2 = (6)2 + (1 0)2 - 2 (6)(10) cos 80°
d=10.73m 7. Given of diameter x and altitude of h. Apogee=a+c
,
"',
b
.... __ _ ., ,;
10 W hat percent is the volume of t he largest

~
cylinder which can be inscribed in the
2. The seating section in a coliseum has cone to the volume of the cone? 9. Determine B such that 3x + 2y- 7 = 0
6 d 6
30 seats in the first row, 32 ·seats in the A. 44% * is perpendicular to 2x - By + 2 = 0.
second row, 34 seats in the third row, 80 ° B. 46% A. S
and so on, until the tenth row is reached, 10 c. 56% B. 3*
after which there are ten rows each D. 65% C.4
contaiAing 50 seats. Find the total D. 2
5. Determine the area of a regular 6-star
number of seats in the section. For maximum cylinder inscribed in a
polygon if the irmer regular hexagon has 1
A. 900
10 em sides. cone: m2= - -
B. 890 * m1
C. 810 A 441.66 cm 2 2
x = - r and y = - h
I
3x + 2y - 7 = 0 eq. of Line 1
D. 390 B. 519.60 cm 2 * 3 3
2

$
C. 467.64 cm
2 .; Vcylinder 1iX 2y 3x 7
a1 = 30 a2 = 32 a3 = 34 D. 493.62 cm = -2- y =- - + -
.
. Vcone 1ir h 2 2
Astar=A1+3A2
The given numbers are in arithmetic . 3 m1 = - ~ slope of line 1
.
- {~ rr(;)
progression, d = 2: A 1= .!_ b 2 sin 9 h 2
2 2x - By + 2 =0 eq. of Line 2
S = .'::[2a1 +(n-l)d] 2 2
2 A 1= .!. (30) 2 sin 60° - 1ir 2 h y= - X+-
. 8 8
2
s = .!.Q[2(30)+9(2)]=390 A 1 = 389.71 cm
2 3
2
2
Total seats= 390 + 10(50) = 890 seats -- .i9 = 44% m2= B slope line 2

3. A bookstore purchased a bestselling


A 2 = .!. (10) 2 sin 60° 2 -1
2
book at P200 per copy. At what price A 2 =43.30 cm 2 8. The major axis of the elliptical path in
w hich the earth moves around the s un is
8 -3/ 2
should this book be sold so that, giving a B =3
= 389.71 + 3 (43.30) =519.60 cm
2
20% discount, the profit is 30%. A star ·
A P450 REFRESHER MANUAL 2nd Edition by JAS TORDILLO
REFRESHER MANUAL 2nd Edition by JAS TORDILLO
2-8 DAY 2 -SOLUTION DAY 2 - SOLUTION 12- 9
y-axis dx _ 6{-0.6, = _ 0.75 m/s
ENGINEERING MATHEMATHICS
• dt- 4_8

12. A flywheel of radius 14 inches is


y=xtane- ~ 20,000=R [(1+0.06)12_1]
2Vo2cos2s 0.06
rotating at the rate of 1000 rpm. How fast
2 R = P1 , 185.54
does a point on the rim travel in ft/sec?
A.122 *
o
-y = (150) tan 0 _ 9.81(150)
2(500)2 cos2 0
B. 100 17. A man borrowed P300,000 from a
c. 1456 y = 0.44 m lending institution which will be paid after
D. 39 10 years at an interest rate of 12%
15. Engine oil at 40°C (p = 875 kg/m 3 ) compounded annually. If money is worth
w = 1000 rpm = 104.72 rad/sec flow into a 2.5 em diameter horizontal 8% per annum how much should be
r = 14 in = 1.1667 ft pipe with a velocity of 1 m/s and exits deposit to a bank monthly in order to
10. What is the x-intercept of the line through a nozzle of 0.5 em diameter. If discharge his debt 10 yrs hence?
passing through (1, 4) and (4, 1). V.= rw , the pressure drop is 538 kN/m 2, what A. P2,798.52
A. 4.5 v = 1.1667 ( 104. 72) must be the exit velocity. B. P4,672.31
B. 6 V = 122.17 ft/s A. 35 m/s * · C. P3,952.50
c. 5 * B.·30 m/s D. P5,093.06 *
D. 4 13. A highway curve has a super C. 25 m/s
elevation of 7 degrees. What is the radius D 20 m/s SA= P(1 + i)"
y - Y1 = m (X- X1) SA= 300,000 (1 + 0.12) 10

r-
of the curve such that there will be no
y-4 = m (x-1) lateral pressure between the tires and the Bernoulli's Equation;
1-4
m= - = -1 roadway at a speed of 40 mph? Sa =· R [ (1 + I]
4-1 A. 265.71 m * P =875 kg/m 3 =8.583 kN/m3

l
y-4=-1(x-1) B. 345.34 m SA =Sa
x+y-5=0 C. 438.34 m P1 - P2 = 538 kN/m2
when: y = 0 (x -intercept} D. 330.78 m 300,000 (1.12) 10
x=5 P1 V1 2
-+-+ZI = P2
-+-+Z2
vi
v = 40 mi (528<ttx~) y 2g y 2g 1+ -0.08)10(12) -1
The x -intercept of the line is 5. hr mi 3600 = R [( 12

11 . A man on a wharf 3.6 m above sea


V = 58.67 ft/sec. V2
2
= (11 -~
y
+ v? +(Z1-Z2l)2g
2g
0.08
12
level is. pulling a rope tied to a raft at 0.60 v2
tan 9 = -
(~+~+0]2(9.81)
m per second. How fast is the raft gr R = P5,093.06
approaching the wharf when there are 6 v 22 = 8.583 2(9.81)
m of rope out? tan 7 o = (58.67 J2 18. A young engineer borrowed P10,000
A . -0.95 m/s (32.2Xr) .• v2 = 35 m/s at 12% interest and paid P2000 per

(~)
B. -0.75 m/s * annum for the first 4 years. What does he
C. -0.85 m/s r = 870.53 ft 16. What annuity is required over 12 have to pay the end of the 51h year in
3.28
D. -0.65 m/s years to equate with a future amount of order to pay off the loan?
r = 265.41
P20,000? Assume i = 6% an nually. A. P6,922.93 *
S 2 = x?- + 3.62 14. A marksman fires a rifle horizontally
A. P4, 185.54 B. P9,622.93
B. P2,185.54 C. P6,292.93
2S dS = 2x dx + 0 at a target. How much does the bullet
dt dt C. P3,185.54 D. P9,262.93
drop in flight if the target is 150 m away
D. P1,185.54 *
dx =~(dS) and the bullet has a muzzle velocity of
P1 + P2 = 10,000
dt

When S
X

.
dt

= 6' x = - -.
500 m/sec.
A. 0.34m
B. 0.64 m
C. 0.44m *
S = R [ (1 + r- 1] P1=2000 [(1 + 0.12)4-1]
0.12(1 + 0.12)4
= P6,074.69
x= J62 -3.62 D. 0.54 m
x = 4.8 m
REFRESHER MANUAL 2nd Edition by JAS TORDILLO
REFRESHER MANUAL 2"d Edition by JAS TORDILLO
2- 10 DAY 2 - SOLUTION DAY 2 - SOLUTION 12 -11
ENGINEERING MATHEMATHICS
p2 = X 21. An engineer is planning for his 15-
(1 + 0.12)5 = 0.567 X year retirement. In order to supplement
24. The quantity of a certain commodity B. materials
his pension and offset the anticipated
that is offered for sale at a certain price at C. parameter
effects of inflation, he intends to withdraw D. composition *
0.567 X+ 6,074.70 = 10,000 a given place and time.
$5000 at the end of the first year, and to
X= P6,922.93 A. demand
increase the withdrawal by $1000 at the
B. stocks 31. The new Mechanical Engineering
end of each successive year. How much law which was signed last February 12,
19. The corporation purchased a C. supply*
money must the engineer have in his
machine for 1 ~-~lion. Freight and D. goods 1998, is known as:
savings account at the start of his
installation charges amounted to 3% of A. RA 9584
retirement, if money earns 6% per year,
the purchase price. If the machine shall 25. A statement the truth of which follows B. RA 8495 *
compounded annually? C. RA2002
be depreciated over a period of 8 years with little or no proof from a theorem.
A. $206 116.59
with a salvage value of 12%, determine A. axiom D. RA 7680
B. $126116.59 B. conclusion
the depreciation charged during the 5th
C. $146116.59 32. Under the provisions of the PSME
year using the sum of the years digit C. hypothesis
D. $106116.59 * Code, what is the color symbol for air
method .. D. corollary *
A. P1 01,107.11 piping?
B. P107,110.11
C. P170,110.11
P= R[(l+i'f-!]+A[(l+i'f-J
i(1 + i'f
_n_J
i2(1 + i'f i(1 + i'f
'26. What curve is represented by' the
equation r = ae.
A. red
B. light blue*
D. P100,711.11 * A. Spiral of Archimedes * C. black
R = 5000, A = 1000, i = 6%, n =15 B. Four-leaves rose D. green

SYD =!2 (1 + 8) =36 P = 500J (1 + o.o6)15 -I J C. cardioid


33. What is the origin of the energy
FC = 1,000,000 (1.03) = P1 ,030,000
10.06(1 + 0.06)15 D. Three-leaves Rose
conservation equation used in the flow
27. is the locus of a point systems?
= =
SV 0.12 (1 ,030,000) P123,600 + 100} (1+0.06)15 -1 15 ]
that moves in a plane of that the A. Daltons law
1
0.062 (1 + 0.06)15 0.06(1 + 0.06)15 difference of the distances from two fixed B. First law of thermodynamics *
Deps = (FC - SV) ( -n-4) = $5000(9.7123) + $1000(57.55) points of the plane is constant C. Pascals law
SYD = $106 116.59 A. hyperbola * D. Second law of thermodynamics
4 B. circle
Deps = (1.oJo.ooo -123.6ooe; ) 22. What is the annual rate of simple C. ellipse 34. The process of exchange where no
6 single medium of exchange is used.
interest if $265 is earned on four months D. parabola
Deps = P100,711 .11 A. exchange
on an investment of $15 000?
A. 5.3% * 28. Intermittent and filters are primarily B. liquidated damages *
20. A debt of P10,000 with 10% interest C. partial
B. 4.5% used to:
compounded semi-annually is to be D. hunter
C. 3.5% A. trap the substance
amortized by semi-annual payments over
D. 7.5% B. determine the flow
the next 5 years. The first due Is 6 • 35. Means the sum agreed upon in
C. oxidize putrescible matter*
months. Determine the semi-annual advance by the parties as compensation
F = P(1 + ni) D. convert water to gas
payment. for a breach.
A. P1 ,200.00
B. P1,193.90
$15 265 = $15 000 (1 +I: 1) 29. To divert excessive flow from A. unliquidated damages
B. advance wages
combined sewers designers often use:
C. P1,295.05 * $15 265 = $15 000 + $5000i A. a overhead trap c. liquidated damages *
D. P1,400.40 B. an extension pipe D. all of these
i = $265 = 5.3%
$5000 C. a spillway
D. a leaping weir * 36. A written acknowledgement by a
P=R [(1+if-1] carrier that he has received the goods for
i(1 + i'f 23. The place where buyers and sellers
come together. 30. The most important factor in shipment.
determining high-temperature behavior of A. crate bill
1ooo = R [ (1 + o.os)IO -I J A. market*
B. recreation center an alloy is: ' B. Bill of Lading *
o.os(1 + o.os)10 A. pressure
C. business
R = P1 ,295.05 D. buy and sell section REFRESHER MANUAL 2nd Edition by JAS TORDILLO
REFRESHER MANUAL 2"d Edition by JAS TORDILLO
2-12 DAY 2 - SOLUTION
DAY 2 - SOLUTION 12- 13
ENGINEERING MATHEMATHICS
C. ticket C. combination
D. contract to ship D. permutation C. polyhedral series
D. discrete series
37. A letter of request whereby one 43. The unit prefix nano is opposite to:
person. usually a merchant or banker, A. Mega 50. An angle greater than a straight line
requests some oth~r person to advance B. Tera and less than two straight angles is
money or to givev credit up to a certain C. Giga" called:
amount.
A. check '" . D. Hexa A. right angle
B. acute angle
B. letter of credit * 44. The altitudes of the sides of triangles
C. obtuse angle
C. letter of debit intersect at the point known as: D. reflex angle *
D. acknowledgement A. orthocenter *
B. centroid
38. If an increase in the capital stock is C. circumcenter
made when actually no additional value IJ. incenter
has been paid either in cash or in
money's worth: 45. The arc length equal to the radius of
A. stock watering * the circle is called:
B. inflation A. 1 radian*
C. deflation B. pi radian
D. depletion C. 1 quarter circle
D. 1 grad
39. A check drawn from a depositor
against his account which the bank has 46. Supplementary angles are angles
marked certified. whose sum is:
A. manager's check A. 45 deg
B. personal check B. 180 deg *
C. certified check* C. 90 deg
D. company check D. 360 deg

40. A written contract, under seal, 47. The median of a triangle is the lien
whereby a corporation binds itself to pay connecting a vertex and the midpoint of
a spec;:ific sum of money to the owner of the opposite side. For a given triangle,
the bond. these medians. intersect at a point which ·
A. corporation bond * is called the:
B. coupon bond A. orthocenter
C. collateral bond B. circumcenter ·•
D. sealed bond C. incenter
D. centroid *
41 . The actual accounts passing into and 48. Any number multiplied by _ __
out of the treasury of financial venture: equals unity.
A. money market A. infinity
B. bond B. its reciprocal *
C. check out C. itself
D. cash flow* D. zero

42. The number of several outcomes 49. A sequence of decreasing numbers


divided by the number of possible or when the succeeding term is lesser
outcomes is: than the preceding term is:
A. change A. convergent series
B. probability * B. divergent series *
REFRESHER MANUAL 2nd Edition by JAS TORDILLO
REFRESHER MANUAL 2"d Edition by JAS TORDILLO
DAY 3 -EXAM ~3-t ·
ENGINEERING MATHEMATHICS

smaller, the quotient is 3 and the


DAY 3- EXAM remainder is 6. What is the smaller part?
A. 15 units
1. One pipe can fill a tank in 8 hours and B. 23 units
another pipe can fill the same tank in 5 C. 32 units
.... hours. A drain pipe can empty the tank in D. 90 units
. ~~. 20 hours. With all three pipes open, how
long will it take to fill in the tank? 7. A company has a certain number of
A. 3.63 hours machines of equal capacity that produced
B. 4.64 hours a total of 180 pieces each working day. If
C. 5.63 hours 2 machines breakdown, the work load of
D. 6.63 hours t~e remaining machines is increased by 3
pieces each per day to maintain
2. How many liters of water must be producfion. Find the number of machines.
added to 40 liters of 92% hydrochloric A. 10
acid solution to reduce its strength to B. 12
80%7 C. 18
A. 2 liters D. 25
B. 61iters
C. 10 liters 8. A man bought 40 chickens for P40.00.
D. 18 liters The cocks cost P3.00 each, the hens
P1.50 each and the chicks at P0.50 each.
3. Find the 501h term of an A.P. How many chicks did he buy?
7,10,13, ........ ... ... . A. 2
A. 102 B.5
B. 154 C. 13
C. 178 D. 25
D.220
9. How many minutes after 6 o'clock at
4. How many liters of a 60% solution of the time between 6 and 7 o'clock are the
nitric acid should be added to 50 liters of hands of the clock at right angles?
a 30% -solution to obtain a 45% solution A. 16.36 min
of the acid? · B. 21.14 min
A. 20 liters C. 32.40 min
B. 30 liters D. 30 min
..... C. 40 liters
D. 50 liters 10. A bookstore purchased a bestselling
book at P350 per copy. At what price
5. James is four times as old as Harold. should this book be sold so that, giving a
ln 'six years James will be twice as old as 25% discount, the profit is 35%.
Harold. What is the age of James now? A. 512
A. 6 years old B.625
B. 12 years old C. 718
C. 20 years old D.892
D. 30 years old
11 . Given a triangle with side a = 140
6. Separate 98 into two parts so that, units, side b = 230 units and angle
when the greater is divided by the C = 32° Determine side c.
A. 133.7 units •
REFRESHER MANUAL 2nd Edition by JAS TORDILLO
DAY 3- EXAM ~3-3
3-2 .DAY 3- EXAM ENGINEERING MATHEMATHICS
B. 177.3 units many times has the front wheel turned if
C. 211.3 units the bicycles has traveled 1.75 km? B. 2141.59 em 29. Find the eccentricity of an ellipse
D. 288.3 units A. 525 rev C. 3141.59 cm
3 when the length of the latus rectum is 2/5
B. 675 rev D. 4141.59 cm
3 of the length of the major axis. ·
12. Evaluate the loge 950 = x. C. 743 rev A. 0.775
A. 1.15 D. 925 rev 23. Compute the surface area of the cone B. 0.995
B. 2.25 having a slant height of 10 em and a C. 0.225
C. 3.29 { -...-:'- . 17. Find the length of the side of a diameter of 10 em. D. 0.325
D. 4.75 ·~ pentagon if the line perpendicular to its A. 457.08 cm 2
side is 20 units from the center. B. 357.08 cm
2 30. The volume of the sphere is
3
13. A tower and a monument stand on a A. 11 units C. 257.08 cm
2 increasing at the rate of 10 cm /hr. At
level plane. The angles of depression of B. 20 units D. 157.08 cm
2 what rate is its radius increasing (in
the top and bottom of the monument C. 29 units cm/hr) when the radius is 30 em?
viewed from the top of the tower are 13° D. 45 units 25. A group of children playing with Ar O.OOQ111
and 35 5 respectively. The height of the marbles placed 70 pieces of the marbles B. 0.000151
tower is 75 m. Find the height of the 18. Find the area the triangle whose inside a cylindrical container with water c. 0.000191
monument. sides are: 30, 40 and 50. filled to a height of 30 em. If the diameter D. 0.001101
A. 20.3 m A.400 of each marble is 2.0 em and that of the
B. 50.3 m B.500 cylindrical container is 10 em. What 31. Water is pouring into a swimming
C.J0.5 m C.600 would be the new height of water inside pool. After t hours, there are t + Jt
D. 95.5 m D. 700 the cylindrical container after the marbles gallons in the pool. At what rate is the
were placed inside? water pouring into the pool when t = 16
14. If the sides of a parallelogram and 19. The area of a circle circumscribing a A. 23.73 em hours?
included angle are 8, 12 and 100 degrees hexagon is 180TT m2 . Find the area of the B. 33.73 em A. 7/6 gph
respectively, find the length of the shorter hexagon. C. 43.73 em B. 6/5 gph
diagonal. A. 467.2 m2 D. 53.73 em C. 5/4 gph
A. 43.2 m B. 567.2 m2 D. 4/3 gph
B. 33.2 m C. 667.2 m2 ' 26. Find the equation of the line passing
2
C. 23.2 m D. 767.2 m through (8, -4) and (-4, -6).
D 13.2 m A. 6x - y - 32 = 0 32. Evaluate the integral of J4t -1 dt.
20. A regular dodecagon is inscribed in a B. X+ 6y + 32 = 0 A. 1/6(4t -1)312 + C
15. A transit set up 120 feet from the circle of radius 20. Find the perimeter of C. X - 6¥- 32 = 0
base of a vertical chimney reads 35.4° the dodecagon. D. 6x- y + 32 = 0 B. 3/2(4t- 1)312 -t C
with the cross hair set on the top of the A. 90.5 units c. 6(4t-1)312 + c
chimney. With thE;! telescope leyel, the B. 112.2 units 27. What is the equation of the normal to
D. 1/4(4t -1)312 + C
vertical rod at the base of the chimney is
5.8 feet. How tall is the chimney?
C. 124.2 units
D. 138.5 units
... the curve y = 3x' - 2x+ 7 at the point
(2, 6)? 33. The stress on a wire that supports a
A. 75.2 ft A. X + 1Oy + 58 = 0 load depends on which of the following?
B. 82.4 ft 21. The circumference of a great circle of B. X+ 10y- 58 = 0
A. All of these
C. 89.3 ft a sphere is. 30TT. Find the volume of the C. 10x + y- 58 = 0
B. the wire's diameter
D. 91 .1 ft sphere. D ..10x- y +58 = 0 C. acceleration of gravity
A. 10,137.16 cu. units
D. the mass of the load
16. What is the inverse natural function of B. 14,137.16 cu. units 28. Find the distance from the line
4x - 3y + 5 = 0 to the point (3, 2). 1
the cosecant? C. 24,137.16 cu. units 34. Integrate - -with respect to x and
A. sine D. 34,137.16 cu. units A. 2.2 units 3X+4
B. cosine B. 3.3 units = =
evaluate the result from x 0 to x 3.
C. tangent 22. When an irregular shaped object is C. 4.4 units A. 0.29288
D. cotangent placed in a cylinder vessel of water D. 5.5 units B. 0.39288
whose radius is 10 em, the water rises 10 c. 0.49288
16. The front wheel of a bicycle has a em. What is the volume of the rock if it is D. 0.59288
diameter of 75 em. Approximately how completely submerged?
REFRESHER MANUAL 2nd Edition by JAS TORDILLO
A. 1141.59 cm3

REFRESHER MANUAL 2nd Edition by JAS TORDILLO


DAY 3- EXAM ~3-5
3-4 IDAY 3 - EXAM ENGINEERING MATHEMATHICS
35. What is the area between y = 0, 42. What is the length of the line with
y = =
3x2 , x 0 and x = 3? slope 4/3 from a point (9, 5) to the y - C. 75.48 em A. 7x + 11y + 79 = 0
A. 12 axis? D. 85.48 em B. 11 X - 7y - 79 = 0
B. 15 A. 12 C. 7x- 11y- 79 = 0
C. 27 B. 15 49. An equilateral triangle has an altitude D. 11x-7y+79=0
D. 35 c. 20 of 10 em long. Find its area~
36. A father is 30 years older than his D. 30 2 55. A hoist with a 120 hp engine is
A. 47.735 cm
son. In 8 years he"wiU be twice as old as capabl~ of lifting a 12,000 pounds load a
B. 57.735 cm2
his son. What is th~ present age of the 43. Find the equation of a line where x-
C. 67.735 cm 2 height of 25 feet in 30 seconds.
father? intercept is 4 and y-intercept is - 3.
D. 77.735 cm2 Determine the·efficiency of this machine.
A. 35 A. 3x + 4y + 12 = 0 A. 16.66%
B. 45 B. 4x -3y- 12 = 0 50. The circumference of a great circle of B. 26.66%
C. 52
D. 60
C. 3x-4y-12=0
D. 4x + 3y -12 = 0
a sphere is 30TT. Find the volume of the c: 33.33%

37. At how many minutes after 3 P.M. will 44. Evaluate the limit: Lim(x
3-27) sphere.
A.10,137. 16cubicunits
D: 44.44%

B. 14,137.16 cubic units 56. An elevator weighing 5000 lb attains


the hands of a clock be at right angles X-4>4 x-3
C. 20,137.16 cubic units an upward velocity of 25 fps in 5 sec w ith
with each other? A. 17 D. 24,137.16 cubic units uniform acceleration. What is the tension
A. 12.72 min B. 27 in the supporting cable in lbs?
B. 22.72 min c. 37 51 . When an irregular shaped object is A. 4776.4 lbs
C. 32.72 min D. 47 placed in a cylinder vessel of water B. 5776.4 lbs
D. 42.72 min whose radius is 10 ern, the water rises 10 C. 6776.4 lbs
45. The distance a body travels is a em. What is the volume of the rock if it is D. 7776.4 lbs
38. What is the product of the complex function of time and is given by x(t) = 20t completely submerged?
numbers 3 + 4i and 7 - 2i? + 1Of. Find its velocity at t = 2. 57. Traffi'c travels at 90 mi/hr around a
A . 10001t cm3
A. 21 + 28i A. 20 banked highway curve with a radius of
B. 15001t cm 3
B. 29- 22i B. 30 2500 ft. What banking angle that is
C. 20001t cm 3
C. 21- 22i c. 60 3 necessary such that friction will not be
D. 29 + 22i D. 25001t cm
D.80 required to resist the centrifugal force?
52. One pipe can fill a tank in 4 hours and A. e = 10.2°
39. Given the triangle ABC in which A = 46. Find the area in the first quadrant B. e = 12.2°
another pipe can fill the same tank in 5
35°, b = 110 em and c = 220 em. Find the bounded by the parabola f =
4x, x 1 = hours. A drain pipe can empty the tank rn c. e = 22.2°
length of the side a. and x = 5.
A. 444.4 em
20 hours. With all three pipes open, how D. e = 32.2° ·
A. 13.57 unif long will it take to fill in the tank?
B. 344.4 em B. 15.57 unif A. 1 hr , 58. A 90 ton rail car moving at 5.5 miles/
C. 244.4 em C. 18.57 unif hr is instantaneously coupled to a
B. 1.5 hr
D. 144.4 em D. 33.57 unif stationary 60 ton rail car. What is the
C. 2 hr
D. 2.5 hr speed of the coupled cars in mi/hr?
40. Find tkle area of a regular hexagon 47. Determine the sum of the odd A. 3.3 mil hr
inscribed in a circle of radius 3 em. numbers of the following integers from1 53. What is the radius of a circle inscribed B. 4.3 mil hr
A. 53.38 cm 2 to 999. C. 5.3 mi/ hr
in a triangle with sides of 7, 9 and 13?
B. 33. 38 cm 2 A . 100,000 A . 1.07 D. 6.3 mi/ hr
C. 23.38 cm 2 B. 150,000
2 B. 1.57
D. 13.38 cm C. 200,000 c. 2.07 59. If a 3 kg gun fires a 45 g bullet at a
D. 250,000 D. 2.57 speed of 600 m/sec, what is the recoil
41 . How many cubic meters is 500
speed of the gun in m/sec?
gallons of water? 48. A central angle of 35 degrees
3 54. Determine the equation of the line A. 5 m/s
A. 1.8927 m subtends an arc of 20 ern. What is the that is perpendicular to the line segment B. 9 rn/s
B. 2.8927 m3 diameter of the circle? (-5, 3) and (2, -8) and passes through the C. 15 m/s
3
C. 3.8927 m A. 55.48 em point x = 5 and y = - 4. D. 25 m/s
3
D. 4.8927 m B. 65.48 em
REFRESHER MANUAL 2nd Edition by JAS TORDillO
REFRESHER MANUAL 2"d Edition by JAS TORDILLO
3-6 !DAY 3 - SOLUTION
DAY 3 - SOLUTION 13- 7
ENGINEERING MATHEMATHICS
60. Determine the super elevation of the
outer rail of a 5-ft wide railroad track on a
DAY 3 - SOLUTION 5. James is four times as old as Harold. pieces each per day to maintain
10° curve. (A 10° curve is one which a In six years James will be twice as old as production. Find the number of machines.
chord 100 ft long subtends an angle of 1. One pipe can fill a tank in 8 hours and Harold. What is the age of James now? A. 10
10° at the center). Assume a velocity of another pipe can fill the same tank in 5 A. 6 years old B.12 *
60 mph. hours. A drain pipe can empty the tank in B. 12 years old * C. 18
A. 1.93 ft .. 20 hours. With all three pipes open, how C. 20 years old D.25
B. 2.25 ft
C. 5 ft
' .
~.-
long will it take to fill in the tank?
A. 3.63 hours *
D. 30 years old
Let X = number of machines
D. 6.25 ft B. 4.64 hours J = present age of James 180 .
C. 5.63 hours H = present age of Harold = work load per machine
D. 6.63 hours X
J + 6 = age of James after six years

ra DAY 3- ANSWER KEY


)
1 1 I
-+---=-
8 5 20 t
1
H + 6 = age of Harold after six years

J = 4H Eq. 1
180
x
+3'=~
x-2
1. A 16. A 31. A 46. A t = 3.63 hours J + 6 = 2(H + 6) 180(x-2) + 3x(x-2) :;: 180x
2. 8 17. c
3. 8 18. c
32. A
33. A
47, D
48. B
4H + 6 = 2H + 12 3,(! - 6x - 360 o =
34. B 49. B 2. How many liters of water must be 2H = 6 X 2 - 2x-120 = 0
4.0 19. A
S.B 20. c 35. C 50. B added to 40 liters of 92% hydrochloric H·= 3 (x-12)(x+10) = 0
6. B 21.8 36. C 51. A ~cid solution to reduce its strength to
7. B 22. c 37.C 52.0 80%? J = 4(3) = 12 age of James X = 12
8. c 23. D 38.0 53. c A. 2 liters x = -10 (absurd)
9.A 24.0 39.0 54. c
B. 611ters * 6. Separate 98 into two parts so that,
10. c 25.8 40. C 55. A 8. A man bought 40 chickens for P40.00.
11. A 26. C 41. A 56.8
C. 10 liters when the greater is divided _ by the
D. 181iters smaller, the quotient Is 3 and the The cocks cost P3.00 each, the hens
12. c 27.8 42.8 57.8
13.8 28. A 43. C 58. A remainder is 6. What is the sm.aller part? P1 .50 each and the chicks at P0. 50 each.
14.0 29.A 44.( 59.8 =
40(0.92) + x(O) (40 +X) (0.80) A. 15 units How many chicks did he buy?
r)jl 15.0 30. c 45. C 60. A =
36.8 32 + 0.80x B. 23 units • A.2
B.5
x = 61iters C. 32 units
D. 90 units C.13 *
3. Find the 50tn term of an A.P. D.25
7,10,13, .............. . Let x ~greater part
A. 102 98 - x = smaller part Let x = no. of cocks
B.154 * y = no. of hens
C. 178 98~x =J(98~x) z = no. of chicks
D. 220
..
, X+ y + Z = 40 (1)
L=a1+(n-1)d Multiply all by 98 - x: 3x + 1.5y +0.50z = 40
=7+(50-1)(3) x = 3(98- x) + 6 30x + 15y + 5z = 400 (2)
L = 154 = 294 -3x + 6
= 75 6x + 3y + z = 80
4. How many liters of a 60% solution of X+ y + z = 40
nitric acid should be added to 50 liters of 98 - X = 23 5x + 2y = 40
a 30% solution to obtain a 45% solution 40-Sx
7. A company has a certain number of y= - -
of the acid? 2
A. 20 liters machines of equal capacity that produced
Try X =4
B. 30 liters a total of 180 pieces each working day. If
2 machines breakdown, the work load of 40-5(4)
C. 40 liters y= - -
the remaining machines is increased by 3 2
D. 50 liters*
y = 10
0.30(50) + 0.60x = 0.45(50 + x)
15 + 0.60x = 22.5 + 0.45x
REFRESHER MANUAL 2nd Edition by JAS TORDILLO
x = 50 liters
REFRESHER MANUAL 2"d Edition by JAS TORDILLO .,
3• s !DAY 3 - SOLUTION DAY 3 - SOLUTION 13- 9
z = 40 - 4 - 10 = 26 12. Evaluate the loge 950 = x.
ENGINEERING MATHEMATHICS
A. 1.15
No. of cocks = 2 B. 2.25 12
s = r6
c. 3.29 * = 0·75 ()
No. of hens = 5

~
No. of chicks = 13 D. 4.75 1750
8 d 8 2
9. How many minutes after 6 o'clock at
6 = 4666.67 rad
x =loge 950 80 °
the time betweert 6 and 7 o'clock are the
x=---
log 950 12 6 = 4666.67 rad x ~
hands of the clock M Toigpt angles? 211" rad
log8
A.16.36 min* 6 = 2333/Tt rev = 742.72 rev
B. 21.14 min X= 3.29 15. A transit set up 120 feet from the
C. 32.40 min base of a vertical chimney reads 35.4°
17. Find the length of the side of a
D. 30 min 13. A tower and a monument stand on a with the cross hair set on the top of the
level plane. The angles of depression of chimney . With the telescope level, the pentagon if the line perpendicular to its
the top and bottom of the monument vertical rod at the base o·f the chimney is Side is 20 units from the center.
X+ 15 = 30 + ~ A. 11 units·
12 viewed from the top of the tower are 13° 5.8 feet. How tall is the chimney?

~
and 355 respectively. The height of the B. 20 units
A . 75.2 ft
~ = 15 tower is 75 m. Find the height of the B. 82.4 ft
C. 29 units*
IZ D. 45 units
monument. c. 89.3 ft
x = ~ or 16.36 min (ans)
11
A. 20.3 m
B. 50.3 m *
C. 70.5 m
D. 91.1 ft *
360
B=~ = 72o
5
'\&) X X
10. A bookstore purchased a bestselling
D. 95.5 m tan cp = Y.. B
book at P350 per copy. At what price X
-=36°
should this book be sold so that, giving a y = (120) tan 35.4° 2
25% discount, the profit is 35%.
h s = 85.3 ft
A. 512 sin 22° sin 103° H= 5.8 + y tan 36o = x
B.625 = 5.8 + 85.3 20
c. 718 * h = s(sin22°) = 91.1 ft x = 14.53 units
D.892 slnl03°
16. What is the inverse natural function of
the cosecant?
I
length of side, 2x = 2(14.53)
= 29.06 units
Let x = selling price of the book
15 A. sine*
0.25 x = discount price of the book S= - -- = 130.76 m
sin 35° B. cosine 18. Find the area the triangle whose
0.35(0.75x) =profit
C. tangent sides are: 30, 40 and 50.
130.76(sin 22°) D. cotangent A.400
x - 0.25x = 350 + 0.35(0.75x)
h= B. 500
0.75x = 350 + 0.2625x sin 103°
X= P 718.00 = 50.27 m •"" I c.soo• ~
h esc e= sine D.700 a =~
11 . Given a triangle with side a = 140 14. If the sides of a parallelogram and
units, side b = 230 units and angle included angle are 8, 12 and 100 degrees 16. The front wheel of a bicycle has a c= 50
C = 32°. Determine side c. respectively, find the length of the shorter diameter of 75 em. Approximately how
A. 133.7 units* diagonal. many times has the front wheel turned if A = Js(s - aXs -bX.s- c)
B. 177.3 units A. 43.2 m the bicycles has traveled 1.75 km?
B. 33.2 m A. 525 rev s=
30+40+50 = 60
C. 211.3 units C. 23.2 m B. 675 rev 2
D. 288.3 units 0.13.2 m * C. 743 rev*
D. 925 rev
A= J6o(6o-3oX6o-4oX6o-5o)
By Cosine Law: Cosine Law: A=600
rf = a2 + b2 2ab cos C

d 2 = (8)2 + (12)2 - 2 (8)(12) cos 80°
2 2 2
c = (140) + (230) • 2(140)(230) cos 32° d= 13.21 m
c = 133.7 units
REFRESHER MANUAL 2nd Edition by JAS TORDILLO
REFRESHER MANUAL 2"d Edition by JAS TORDILLO
3- 10 IDAY 3 - SOLUTION
DAY 3 - SOLUTION 13 -11
ENGINEERING MATHEMATHICS
19. The area of a circle circumscribing a C. 24,137.16 cu. units
hexagon is 180TT m 2 • Find the area of the D. 34,137.16 cu. units 25. A group of children playing with dy
hexagon. marbles placed 70 pieces of the marbles slope - = 6x- 2
A. 467.2 m 2
• C = 2 TTR ' dx
inside a cylindrical container with water m = 6(2)-2 = 10
B. 567 .2 m 2 30 TT = 2 TTR filled to a height of 30 em. If the diameter
C. 667.2 m2 ·R = 15 of each marble is 2.0 em and that of the Slope of normal line, mn = I
D. 767.2 m 2 10
V= ~TTR 3 cylindrical container is 10 em. What
3 would be the new height of water inside Y- Y1 = mn (x- X1)
'

2eJ
the cylindrical container after the marbles l
A= [_!_r2sin6oo}6) v = ~lt(t5)3 were placed inside?
y- 6 = --(X
10 .
- 2)
3
V = 14,137.16 cu. units A 23.73 em 10y- 60 = -x + 2
r=
r¥ 1t 22. When an irregular shaped object is
placed in a cylinder vessel of water
B. 33.73 em*
C. 43.73cm
D. 53.73 em
X+ 10y-58 = 0

28. Find the distance from the line


r= p&~K whose radius is 10 em, the water rises 10
= TT~h
4x- 3y + 5 = 0 to the point (3, 2).
A. 2.2 units *
em. What is the volume of the rock if it is Vtotal

r=13.41m completely submerged? Vtotal = Vwarer + Vmarbtes B. 3.3 units


3 C. 4.4' units
A. 1141.59 cm 4
2
31t 3

[~(13.41)2 sin 60° Jc6)


Vtotal = TT(5) (30) + (1.0) (70)
A= B. 2141.59 cm3 D. 5.5 units
C. 3141.59 cm3 * Vtotat = 2649A cm
3

A= 467.2 m
2 D. 4141 .59 cm 3 d = Ax1 +BY! +C
2
VR = _:: (20)2 (I 0) 2649.4 = TT(5) h ±JA2+82
20. A regular dodecagon is inscribed in a 4 h = 33.73 em
3 3
circle of radius 20. Find the perimeter of = 10007t cm = 3141.59 cm
d = 4 ( 3 )- 3( 2 )+ 5
-J42
the dodecagon. 26. Find the equation of the line passing
A. 90.5 units 23. Compute the surface area of the cone through (8, -4) and (-4, -6). +(- 3'f
B. 112.2 units having a slant height of 10 em and a A. 6x - y - 32 = 0 d = 2.2 units
C. 124.2 units* diameter of 10 em. B. X + 6y + 32 = 0
D. 138.5 units A. 457.08 cm2 C. X - 6y - 32 = 0 * 29. Find the eccentricity of an ellipse
B. 357.08 cm 2 D. 6x- y + 32 = 0

~-10~
when the length of the latus rectum is 2/5
C. 257.08 cm 2 of the length of the major axis.
Dodecagon has 12 s i d
.. e
. .s. .Q
.. .· 0.157.08 cm 2 * Y - Y2 = m(x- x2)
360° . A. 0.775 *
8=- B. 0.995
n A=rrrl
r a :r A= TT (5) (10)
m = ~2- Yi = -4 + 6 = 2_ =..!. C. 0.225
8 = 360 =300 r=5cm X2-XJ 8+4 12 6 D. 0.325
A= 50TT ~
. 12 X I
A= 157.08 cm 2 y + 4 = -(x-8)
6 LR = 2b2
x = length of the side of a dodecagon 6y + 24 =X- 8
24. How many cubic meters is 500 a
gallons of li~uid? x-6y-32 = 0
Using_ cosine law:
A. 4.8927 m
LR = 3.. (length of the major axis)
r
x2 = + ~- 2(r)(r) cos 8
B. 3.8927 m 3 27. What is the equation of the normal to
5
x2 = (20)2 + (20)2 - 2(20)(20) cos 30° 3 the curve y = 3x" - 2x+ 7 at the point 2b2 2
C. 2.8927 m -=-(2a)
x = 10.35 units 3 (2, 6)?
D. 1.8927 m • a 5
Perimeter= 12x A. X + 1Oy + 58 = 0
= 12(10.35) = 124.23 units .
3.785411 ~
3 B. X+ 10y- 58 =0 * b2=3_a2
5
-g;;J" C. 10x + y- 58 = 0
21. The circumference of a great circle of
V = 500 gal X X !OOOii c2 = az- bz
D. 10x- y +58 = 0
a sphere is 30rr. Find the volume of the = 1.8927 m 3 ~ = a2- ~a2
sphere. y = 3x2 - 2x + 7 I 5
A. 10,137.16 cu. units
B. 14,137.16 cu. units* REFRESHER MANUAL 2nd Edition by JAS TORDILLO
REFRESHER MANUAL 2nd Edition by JAS TORDILLO
3- 12 IDAY 3 - SOLUTION DAY 3 - SOLUTION 13 -13
ENGINEERING MATHEMATHICS
c2 = ~a2
5 fJ4t+1 = ±Jdt (4t -t)ll2(4dt)

'~I:
(3 + 41)(7 - 2t) = 21 - 8f + 28i- 6i
~a = vs
fi = o.7746 = 21 + 8 + 28i- 6i

~[,··-r ]·c
eccentricity, e = = 29 + 22i
=
= = (3)3 - 03 :: 27
30. The volume o'f the sphere is 39. Given the triangle ABC in which A =
3 35°, b = 110 em and c = 220 em. Find the
increasing at the l'ate of 10 cm /hr. At 36. A father is 30 years older than his
what rate is its radius , increasing (in = ~(4t-l~/ 2 +C length of the side a.
12 son. In 8 years he will be twice as old as A. 444.4 em
cm/hr) when the radius is 30 em? his son. What is the present age of the B. 344.4 em
A. 0.000111 =.!.(4t-1~ 12 +C father? C. 244.4 em
B. 0.000151 6
A. 35 0.144.4 em*
c. 0.000191 * B.45
33. The stress on a wire that supports a
D. 0.001101
load depends on which of the following?
c. 52*
D. 60
a = (110) +(220)
2 2 2
- 2 (110) (220) cos 35°
A. All of these * a·= 144.4 em
V = ~11:r 3 B. the wire's diameter
3 Let x = son's age at present 40. Find the area of a regular hexagon
C. acceleration of gravity
X + 30 = father's age at present
~~ = ( 411: )(3r2 )(
3
~~) D. the mass of the load
X + 8 = son's age in 8 years
inscribed in a circle of radius 3 em.
A. 53.38 cm 2
X + 30 + 8 = father's age in 8 years B. 33.38 cm 2
10 = 4rr(30)2 ~
1
dt
34. Integrate - -with respect to x and
3X+4
c. 23.38 cm 2 *
X + 38 = 2(x + 8) D. 13.38 cm 2
evaluate the result from x = 0 to x = 3.
~ = 0.000191
A. 0.29288
X+ 38 = 2x + 16
dt X = 38 -16 = 22
si{3~0)]
2
B. 0.39288 *
A= n; [
31. Water is pouring into a swimming c. 0.49288 X + 30 = 22 + 30 = 52 years old
D. 0 .59288
Jt
pool. After t hours, there are t +
gallons in the pool. At what rate is the
37. At how many minutes after 3 P.M. will A= 6(~)2 [ si{3:0)J
3 3 the hands of a clock be at right angles

J(-
water pouring into the pool when t = 16 1 A= 23.38 cm 2
) dx = [.!.ln(3x + 4)1 with E1ach other?
hours?
A. 7/6 gph *
3X+4 3 Jo A. 12.72 min
0 B. 22.72 min 41. How many cubic meters is 500
B. 6/5 gph
C. 32.7~ min* gallons of water?
C. 5/4 gph = .!.[ln(9 + 4)-ln(o + 4)]
3 D. 42.72 min A. 1.8927 m3 *
D. 4/3 gph B. 2.8927 rl) 3
3
C. 3.8927 m 3
v =t+ Ji J(3x ~ 4) dx = 0.39288 ~ 15+~+15=x
12 D. 4.8927 m
3

dV = l+-1-
0 11x = 360 _ 3.785411 m3
V - 500 ga 1 X - - X --
dt 2J! gal 100011
35. What is the area between y = 0, X= 32.!. min = 32.72 min
when t = 16: II = 1.8927m3
y = 3x2, x = 0 and x = 3 ?
dV I 7 A. 12
dt = I + 2M = 6 gph
B. 15
38. What is the product of the complex 42. What is the length of the line with
numbers 3 + 4i and 7 - 2i? slope 4/3 from a point (9, 5) to the y -
c. 27*
A. 21 + 28i axis?
D. 35
32. Evaluate the integral of J4t -I dt. B. 29- 22i A 12
3
C. 21 - 22i 8.15 *
A. 1/6(4t- 1)312 + C * A= Jydx
2
:: J3x dx D. 29 + 22i * C.20
B. 3/2(41- 1)312 + C 0
D. 30
C. 6{4t - .1)312 + C
D. 1/4(41 - 1)312 + C REFRESHER MANUAL 2nd Edition by JAS TORDILLO
REFRESHER MANUAL 2nd Edition by JAS TORDJLLO
• - - - -r-- -
--· ----------------~~

3 - 14 JDAY 3 - SOLUTION DAY 3 - SOLUTION 13 -15


y - y, = m(x- x,) 8 . 30 ENGINEERING MATHEMATHICS
y - 5
4
= - (x- 9)
c. 60* • 11
3 D. 80 Sum = -(at + L) V = irrR 3
2 3
when x = 0:
4
x(t) = s = 20t + 1of = 500 (I + 999)
y - 5 = - (0 - 9) 2
v = i:rr(Is)3
3 dS = 20 + 20t 3
dt = 250,000 V = 14,137.16 cubic units
y = -7
P(O, -7) and P(9, 5)
at t = 2: 48. A central angle of 35 degrees 51. When an irregular shaped object is
D = ~(X2-Xt)2 + (Y2-YI)2 dS = velocity = 20 + 20(2) = 60
subtends an arc of 20 em. W hat is the pll')ced in a. cyli nder vessel of water
dt
diameter of the circle? whose radius is 10 em, the water rises 10
D = J c9 - o>2 + cs + 1>2 A. 55.48 em em. What is the volume of the rock if it is
B. 65.48 em • completely submerged?
D = 15 46. Find the area in the first quadrant
C 75.48 em
bounded by the parabola y2 = 4x, x = 1 A:10001t cm 3 *
D. 85.48 em B. 15007t cm 3
43. Find the equation of a line where x- and x = 5.
intercept is 4 and y-intercept is - 3. A. 13.57 unie * C. 20007t cm 3
A. 3x + 4y + 12 = 0 B. 15.57 unif s =r e D. 25007t cm3
B. 4x -3y - 12 = 0 C . 18.57 unit2
20 = rx
35° (--;)
C. 3x -4y - 12 = 0 * D. 33.57 unif ISO VR = 2:(20)2 (10)
D.'4x + 3y - 12 = 0· r =32.74 em 4
3 d = 2r =65.48 em = 3141.59 = 10007t cm3
Intercept form: A= I ydx
I
~ +r=I 49. An equilateral triangle has an altitude 52. One pipe can fill a tank in 4 hours and
a b 3 of 10 em long. Find its area. another pipe can fill the same tank in 5

~+L=t A= JJ4X dx A . 47.735 cm 2


B. s7.735 cm 2 •
hours. A drain pipe can empty the tank in
20 hours. With all three pipes open, how
4 -3 I C. 67.735 cm 2 long will it take to fill in the tank?
3X -4y =I 5 D. 77.735 cm 2 A. 1 hr
I2 x3/2 ]
A=2 - - B. 1.5 hr
3x - 4y = 12 [ 3/2 C. 2 hr
3x-4y- 12=0 1 Tan 60° =

~
X D. 2.5 hr*
A= iKsf312 _(I)312] x = 5.773 em
Lim(x
3-27) 3
A = 13.57 unie
B = 2x =
2(5.773) =
11 .547 em 1 1 I I
-+---=-
44. Evaluate the limit: H = 10
x-+4l x-3 · 4 5 20 t
Area = Y. B x H t = 2.5 hours
A. 17 47. Determine the sum of the odd
= y, (11.547)(1 0)
numbers of the following integers from1
B. 27 = 57.735 cm 2 53. What is the radius of a circle inscribed
c. 37 * to 999.
A. 100,000 in a triangle with sides of 7, 9 and 13?
D.47 50. The circumference of a great circle of A . 1.07
B. 150,000
a sphere is 30rr. Find the volume of the B. 1.57
. (x3-27) _ (x - 3) (x 2+3X+9)
C. 200,000
sphere. c. 2.07 *
L 1m - -
X-+3 x-3
-
x-3
c..._---.:..~----'-
D. 250,000 *
A. 10,137.16 cubic units D. 2.57
B. 14,137.16 cubic units *
L = a, + (n - 1)d
= (4)2 + 3(4) + 9 C. 20,137.16 cub!c units
s = 7 + 9 + 13 = 14.5
= 37 D. 24,137.16 cubic units
Where: a 1 = 1, L = 999 and d = 2 2
999 = 1 + (n- 1)2 Area = J's('s-
--a"'Xs---=-b-vX~
s --c')
45. The distance i a body travels is a C = 2 rrR
n = 500 30 TT = 2 rrR
function of time and is given by x(t) = 20t
+ 10f. Find its velocity at t = 2. R = 15
A . 20
REFRESHER MANUAL 2nd Edition by JAS TORDILLO
REFRESHER MANUAL2"d Edition by JAS TORDILLO
3 -16 IDAY 3 - SOLUTION
DAY 3 - SOLUTION 13- 17
ENGINEERING MATHEMATHICS
Area= Jt4.5(l4.5-7X14.S-9XJ4.5-i3} A. 4776.4 lbs
B. 5776.4 lbs * Mgun = Mbullet
Area= 29.95 C. 6776.4 lbs (mV)gun = (mV)bullet
D. 7776.4 lbs 3000 Vgun =45 {600)
s = 14.5
Vgun = 9 mlsec
At,
R= - a = V2 - V2 = 25 - o = 5 ft/s2
s v I 5 60. Determine the super elevation of the
29.95 = 2.065 " ··~ ' .... L Fv = 0 outer rail of a 5-ft wide railroad track on a
R = 14.5 10° curve. (A 10° curve is one which a
T = W + -
w a = 5000 + - (5)
5000
chord 100 ft long subtends an angle of
. g D2 ·
10° at the centerj. Assume a velocity of
54. Determine the equation of the line
that is perpendicular to the line segment = 5776.41bs 60 mph. ·
(-5, 3) and (2, -8) and passes through the A. 1.93 ft *
point x = 5 and y =- 4. 57. Traffic travels at 90 mi/hr around a B. 2.25 ft
A. 7x + 11 y + 79 = 0 banked highway curve with a radius of c. 5 ft
B. 11 X - 7y - 79 = 0 2500 ft. What banking angle that is D. 6.25 ft
c. 7x-11y-79=0* necessary such that friction will not be
D. 11x- 7y + 79 = 0 required to resist the centrifugal force? . 50
A. 8 = 10.2° S1n 5°=-
r
P1(-5, 3) and P2(2, -8) B. e = 12.2° *
c. e = 22.2° r = 573.68 ft
_ YJ-Y2 3-(-8) 11
m1- - - = - - = - - D. e = 32.2° y2
X1-x2 -5-2 7 tan 8 = -
-1 -1 7 gr
mz= - = - = - V = 65 mi/hr = 132 ft/s
m1 _ 11 11
v2 = ---=---...:.-
(132) 2 882
7 Tan 9 = - tan e= 32.2(573.68)
Solving for the equation of a line at point gr 32.2(2500)
e = 12.2°
e = 22.74° (banking angle)
(5, -4) and slope, m = 7/11.
Y- Y1 = m(x- x1)
y-(-4}= -(x-5)
7 58. A 90 ton rail car moving at 5.5 miles/ sin 22.74° =~
hr is instantaneously coupled to a 5
II h = 1.93 ft (super-elevation of railroad
stationary 60 ton rail car. What is the
11 y + 44 = 7x- 35 track)
speed of the coupled cars in mi/hr?
7x- 11 y - 79 = o
A. 3.3 mi/ hr *
B. 4.3 mil hr
55. A hoist with a 120 hp engine is
C. 5.3 mil hr
capable of lifting a 12,000 pounds load <.i
D. 6.3 mil hr ..•
height of 25 feet in 30 seconds.
Determine the efficiency of this machine.
Let V = speed of coupled cars
A. 16.66% * By conservation of momentum:
B. 26.66% m1V1 + m2V2 = m1V1' + m2V2'
C. 33.33%
D. 44.44%
V1' =V 2' =
V (coupled cars)
90 (5.5) + 60 (0) = 90V + 60V
12,000-lbx25-ft
. ·X
hp
.. V = 3.3 mil hr
Eff = ~ = 30-s 50Qfl-l~ec
Pin 120hp 59. If a 3 kg gun fires a 45 g bullet at a
Eff= 0.1666 = 16.66% speed of 600 m/sec, what is the recoil
speed of the gun in mlsec?
56. An elevator weighing 5000 lb attains A. 5 mls
an ·upward velocity of 25 fps in 5 sec with B. 9 m/s *
uniform acceleration. What is the tension C. 15 m/s
in the supporting cable in lbs? D. 25 m/s REFRESHER MANUAL 2nd Edition by JAS TORDILLO
REFRESHER MANUAL2"d Edition by JAS TORDILLO
DAY 4 - EXAM 14 -1
ENGINEERING MATHEMATHICS
7. What is the hydraulic radius of a 6-cm
DAY 4- EXAM diameter pipe filled to depth of 2 em?
A. 0.90 em
1. Calculate the weight, in lbf, of a 5-lbm B. 1.12 em
object in a gravitational field of 30 ft/sec2. C. 2.11 em
A. 1.66 1bf D. 3.11 em
{ B. 2.661bf
"·. . ,_
'
C. 3.661bf 8. What is minimum distance between
D. 4.661bf the
y = 3x + 4 and the origin (0,0)?
2. A fighter plane with a mass of 5000 A. 0.95
Ibm travels at 30,000 ft/sec. What is its B. 1.26
kinetic energ~ in ft-lbf? C. 2.25
A. 6.98 X 101 ft-lbf D. 5.25
B .• 7.98 X 1010 ft-lbf
C. 8.98 X 1010 ft-lbf 9. Find the angle between the lines.
10
D. 9.98 X 10 ft-lbf y =- 0.752x + 3
y = +0.752x- 6
3. A 10-kg block hangs from a cable. A. 13.48°
What is the tension in the cable? B. 23.48°
2
(Standard gravity equals 9.81 m/s .) c. 33.48°
A. 9.81 N D. 38.48°
B. 19.8 N
C. 18.9 N 10. What is the slope at x = 4 of the curve
D. 98.1N f (x) = i' - 3x?
A. 25
4. A 50 kg block is raised vertically 5 B.35
meters. What is the change in potential C. 45
energy? D. 55
A. 1452.50 J
B. 1952.50 J 11. A circle is expressed parametrically
c. 2452.50 J by the equations
D. 5452.50 J x = 10 cos 6 and y = 10 sin 6
Express the derivative dy/dx as a function
5. The surv1v1ng fraction, x, of a
.
~ radioactive isotope is given by x = e-ooost.
of x andy.
A. y/x
For what value of t will the surviving B. - x/y
percentage be 9%? C.x/y
A. 267 ' D. x-y
B. 276
c. 357 12. What is the partial derivative az!ax of
D. 897 the following function?
z = 4~ - 7l + xy + 6y - 10
5 A. 4x + y
6. Evaluate the limits lim = Jx-
x~co 4x +S B. 8x + y
A. 1/3 C. 4~ -7
B. Y:z D. 8x -7y2 +X+ 6y
C. o/.
D. 18
REFRESHER MANUAL 2nd Edition by JAS TORDILLO
4-2 DAY 4- EXAM DAY 4- EXAM ~4-3
13. Find the derivative dy/dx of 19. Maintenance costs for a machine are ENGINEERING MATHEM~THICS
f(x,y) = 2x2 + xy + 2y3 = 0 P500 each year. What is the present
A. -(4x + y)/(x + 6{) worth of these maintenance costs over a 24. An asset is purchased for P9000. Its B. P9,181.95
B. (2x + y)/(x + 3y ) 12 year period if the interest rate is 8%7 estimated economic life is ten years, after C. P10,181.95
C. (2x + y)/(x + 3/) A. P1,768 which it will be sold for P400. Find the D. P18,181.95
D. (4x + y)/(x + 3y 2) B. P2,768 depreciation in the first three years using
C. P3,768 sum-of-the-years' digits depreciation 28. A ski resort installs two new ski lifts at
14. Eight people are on a sinking boat. D. P5,768 methods. a cost of P2,000,000. The resort expects
There are five life jl!cke~~- How many A. P4221.81 annual gross revenue to increase
combinations of survivors ate there? 20. Your are currently paying P620 per B. P5221.81 P600,000 while it incurs an annual
A_ 56 month to lease your office phone C. P6221.81 expense of P75;000 for lift operation and
B. 51 equipment. You have three years left on D. P7221.81 maintenance. What is the pay-back
c. 45 the five-year lease. What would have period?
D. 30 been an equivalent purchase price two 25. A company purchases complete and A. 2.5 years
years ago? The effective interest rate per exclusive patent rights to an invention for B. 3.0 years
15. A pianist knows five pieces but will month is 1%. P1.000.000. It is estimated that, once C. 3.8 years
have enough stage time to play only A. P17,872.12 commercially'produced, the invention will D. 4.5 years
three of them. How many different B. P27,872.12 have .a specific but limited market of 800
programs can be arranged? C. P37,872.12 units. For the purpose of allocating the_ 29. What is the uninflated present worth
A. 24 D. P67,872.12 patent right cost to production cost, what of P12,000 in two years if the average
B. 45 is the amortization rate in pesos per unit? inflation rate is 8% and interest is 12%?
C. 60 21. Maintenance on an old machine is A. P1000 A. P5,201.60
D. 90 P1000 this year but is expected to B. P1;250 B. P6,201.60
increase by P250 each year thereafter. C. P1500 C. P7,201.60
16. A box contains six white balls, three What is the present worth of five years of D. P1850 D. P8,201.60
red balls and four green balls. What is the maintenance? Use an interest rate of
probability of getting either a white ball or 10%. 26. A corporation that pays 55% of its 30. A P15,000 simple interest loan is
a red ball in one draw from the .bowl? A. P3506.25 profit in income taxes invests P1 0,000 in taken out at 15% per annum interest rate.
A. 6/13 B. P4506.25 a project that will produce P3000 annual The loan matures in two years with no
B. 3/13 C. P5506.25 revenue for eight years. If the annual intermediate payments. How much will be
C. 9/13 D. P8506.25 expenses are P800, salvage value after due at the end of the second year?
D. 3/5 eight years is P600, and 9% interest is A. P17,250
22. An asset is purchased for P9000. Its used, what is the after-tax present worth? B. P19,500
17. One box contains five white balls, two estimated economic life is ten years, after Disregard depreciation. C. P22,500
red balls, and three green balls. Another which it will be sold for P400. Find the A.- P4219.43 D. P30,000
box contains three yellow and seven total depreciation for the first three years B. - P5219.43
black balls. What is the probability of using straight line. c_- P6219.43 31. P6000 is ·b orrowed for 90 ·days at
getting a red ball from the first box and a A. P2580.00 D.- P7219.43 18% per annum simple interest. How
yellow ball from the second box in one B. P3580.00 much will be due at the end of 90 days?
draw from each bowl? C. P4580.00 27. One year, a company makes a PSOOO A. P6270
A. 3/50 D. P5580.00 investment in a historic building. The B. P7260
B. 1/5 investment is not depreciable, but it does C. P8350
C. 3/10 23. An asset is purchased for P9000. Its qualify for a one-time 20% tax credit. In D. P9950
D. 1/10 estimated economic life is ten years, after that same year, revenue is P50,000 and 32. The density of water is typically taken
which it will be sold for P400. Find the expenses (exclusive of the P5000 as 62.3 lbm/ft3 . For engineering problems
18. How much should you put into a 10% depreciation in the first three years using investment) are P20,000. The company where greater accuracy is not required,
savings account in order to have P20,000 double declining balance. pays a total of 55% in income taxes. what is the value in kg/m3 ?
in five years? A. P4340.80 What is the after-tax present worth of this A. 999
A. P10,418.42 B. P5340.80 year's activities if the company's interest B. 997.4
B. P12,418.42 C. P6340.80 rate for investment is 10%7 C. 1000
C. P22,418.42 D. P7340.80 A. P8,181.95 D. 1.4
D. P32,418.42
REFRESHER MANUAL 2nd Edition by JAS TORDILLO
REFRESHER MANUAL 2"d Edition by JAS TORDILLO
DAY 4- EXAM ~ 4-5
4-4 DAY 4 - EXAM ENGINEERING MATHEMATHICS
33. Determine the specific gravity of A. 77,639.75 ft-lbf
carbon dioxide gas (MW = 44) at 70°C B. 87,639.75 ft-lbf 49. Air enters a jet engine at 400 m/s and
44. Five kmols of air initially at one
and 150 kPa using STP air as a C. 97,639.75 ft-lbf leaves at 850 m/s. The thrust produced is
atmosphere and 299 K are compressed
reference. D. 99,639.75 ft-lbf 46,500 N. Disregarding the small amount
isothermally to 8 atmospheres. How
A. 1.03 of fuel added during combustion, what is
much total heat is removed during the
B. 1.93 39. When traveling at 120 kph, a car the mass flow rate?
compression?
c. 2.53 supplies a horizontal force of 100 N to the A. 25,846.3 kJ A. 103.3 kg/s
D. 3.33 hitch of a trailer. What tractive power (in B. 28,922.9 kJ B. 123.3 kg/s
horsepower) is required for the trailer c. 39,345.6 kJ C. 153.3 kg/s
34. What is the sea level (g = 32.2 alone? D. 193.3 kg/s
2 D. 44,680.9 kJ
ft/sec ) specific weight (lbf/W) of liquids A. 3.35 hp
with densities of 65.5 lbm/fe? B. 4.47 hp 50. A 3000 kg car travels at 70 kph
45. What is the centroidal moment of
A. 32.2 C. 5.26 hp around a curve with a radius of 75 m.
inertia with respect to the x-axis of a
B. 42.2 D. 7.75 hp What is the centripetal force?
rectangle 6.0 units wide and 9.0 units
C. 65.5 A. 12,123 N
D. 76.7 tall?
40: How much energy is required to just A. 164.5 units2 B. 15,123 N •
v.apori~e a kg of water. which is originally
35. A lawn mower engine is started by
B. 264.5 units2 • C.21,123N
at 27°C and one atmosphere? 2 D. 35,123 N
C. 364.5 units
pulling a cord wrapped around a sheave. A. 2562.65 kJ
D. 464.5 units2
The sheave radius is 10.0 em. The cord B. 3562.65 kJ 51. The measure of microscopic disorder
is wrapped around the sheave two times. c. 4562.65 kJ 2
46. The acceleration in m/s of a 50-kg of a substance.
If a constant tension of 100 N is D. 5562.65 kJ A. Enthalpy
body is specified by the equation
maintained in the cord during starting, B. Entropy
a(t) = 8- 16t
what work is done? 41. What mass of nitrogen is contained in C. Internal energy
What is the instantaneous force acting on
A. 110.6'J a 65 m 3 tank if the pressure and the body at t = 4 seconds? D. None of the above
B. 125.6 J temperature are one atmosphere and
C. 160.6 J
A. 4400 N
21 °C respectively? 52. A 0.050-kg bullet attains a velocity of
D. 180 J B. 3300 N
A. 75.4 kg 82 _m/s in a huntin~ slingshot. Contact
C. 2200 N
B. 57.4 kg D.1 100N with the sling is 1/10 h of a second. What
36. A 300-pound crate is pustied 30 feet C. 54.7 kg is the average force on the marble during
at constant velocity across a warehouse D. 45.7 kg 47. Twenty five kg of sand fall contact?
floor. There is a frictional force of 80
3 continuously each second on a conveyer A. 20 N
pounds between the crate and floor. 42. A 0.90 m tank contains 6.5 kg of an belt moving horizontally at 1.0 m/s. What B. 35 N
What work is done by the frictional force ideal gas. The gas has a molecular power is required to keep the belt C. 41 N
on the .crate? weight of 44 and is at 21°C. What is the D. 62 N
A. 1200 ft-lbf moving?
pressure of the.gas?
B. 1800 ft-lbf A. 10W
A. 201.3 kPa .• 53. A golf ball dropped from a height of
C. 2400 ft-lbf B. 15W
B. 301 .3 kPa 3.0 m onto a hard surface rebounds to a
D. 3000 ft-lbf C. 25W
C. 401.3 kPa height of 2.0 m. What is the coefficient of
D. 35W
D. 501.3 kPa
48. A 3000-kg cannon fires a 12-kg restitution?
37. A solid disk flywheel (I= 300 kg.m2 ) is
projectile horizontally at 700 m/s. It takes A. 0.75
rotating with a speed of 1200 rpm. What 43. One kg of hydrogen are cooled from
0.009 seconds for t he projectile to pass B. 0.81
is its rotational kinetic energy? 450°C to 320°C in a constant volume
through the barrel. The cannon has a c. 0.92
A. 1,368 X 103 J process. The specific heat at constant
spring mechanism to absorb the recoil. D. 0.99
B. 2, 368 X 103 J volume, Cv, is 10.2 kJ/kg-K. How much What force is exerted on the recoil
C. 3,368 X 103 J heat is removed? spring? 54. Find the deflection, y, at the free end
D. 4,368 X 103 J A. 1136 kJ A. 7. 77 x 105 N of the cantilever beam. Neglect the beam
B. 1326 kJ B. 8.88 x 105 N weight.
38. A 5.0-pound is launched straight up c. 1623 kJ C . 9.33 x 105 N
3
A. FL /3EI
with an initial velocity of 1000 ft/sec. D. 2136 kJ 3
D. 10.33 x 105 N B. FL /48EI
Neglecting air friction, calculate the
kinetic energy immediately after launch. REFRESHER MANUAL 2nd Edition by JAS TORDILLO
REFRESHER MANUAL 2"d Edition by JAS TORDILLO ·
F

DAY 4- SOLUTION ~ 4-7


4-6 DAY 4 - EXAM ENGINEERING MATHEMATHICS
C. FL~/12EI
D. FL3/24EI
) DAY 4 - SOLUTION B. 1952.50 J
DAY 4- ANSWER KEY
c. 2452.50 J *
55. A flywheel is brought to a standstill D. 5452.50 J
from 500 r_pm in 10 seconds. What was 1. D 16. c 31. A 46. A 1. Calculate the weight, in lbf, of a 5-lbm
2. A 17. A 32.. B 47. C 2
its average angular acceleration in 33.8 48. c
object in a gravitational field of 30 ftlsec . PE = mgh = 50(9.81)(5) = 2452.50 J
3. 0 18.8
rad/sec2 durin~ tbat period? 4. C 19. c 34. C 49. A A. 1.661bf
A. - 2.23 rad/s 5.A 20.8 35. 8 50.8 B. 2.66 1bf 5. The surviving fraction, x, of a
B. - 3.23 rad/s2 c. 3.661bf radioactive isotope is given by x = e·0 · .
0091
6. c 2l. C 36. C 51. 8
C. - 4.23 rad/s2 7. 8 22.A 37. 8 52.C D. 4.661bf * For what value of t will the surviving
D. - 5.23 rad/s 2 8. 8 23.A 38. A 53. 8
percentage be 9%?
9. 8 24.A 39.8 54. A
A. 267 *
56. From the previous problem, How far
lO.C 25.8
11. B 26. A
40. A
41. A
55. D
56. A
(Sibm{ 30 ft J B. 276
(in radians) did the flywheel travel? F = ma- \ sec2 c. 357
12. 8 27. A 42. C 57. B
A. 262 rad gc - _ Ibm- ft
13. A 28. C 43. B 58. D 32 2 D. 897
B. 322 rad 14. A 29. D 44. A 59. C lbf- sec2
C. 455 rad 15. C 30. 8 45. c 60. 0
= 4.66 lbf
x =0.09 =e·o.oost
D. 622 rad
ln(0.09) = In (e'0 ·0091 )
57. A turntable starts from rest and 2. A fighter plane with a mass of 5000 - 2.40 = -0.0091
2
accelerates uniformly at 2.3 rad/sec • Ibm travels at 30,000 ft/sec. What is its t = 267.55
How many revolutions will it take before a kinetic energy in ft-lbf?
rotational speed of 45 rpm i$ attained? A. 6.98 X 1010 ft-lbf *
A. 0 .66 rev B. 7.98 X 1010 ft-lbf 6. Evaluate the limits lim = Jx- 5
10 x~co 4X+5
B. 0.77 rev C. 8.98 X 10 ft-lbf
C. 0.88 rev D. 9.98 X 1010 ft-lbf A. 1/3
0 0.99 rev B. Y:z
2 .C. '!.. *
58. What type of curve is generated by a
Ek= mv
2
=
(sooo lbmX3o,ooo !!_)
sec
D. 18
5 5
point which moves in uniform circular 3- - 3--
motion about an axis, while traveling with 2gc (2{ 32.2 Ibm- ft ) . 3x- 5 .x co
hm - - = 11m - - = - --
5
5 4+ ·--
a constant speed, v, parallel to the axis? \ lbf- sec 2 x~co4x+5 x~4+ --
A . a cycloid 10 x co
= 6.98 X 10 ft-lbf 3-0 3
B. an epicycloid
C. a hypocycloid 4+0 4
3. A 10-kg block hangs from a cable.
D. a helix
What is the tension in the cable?
2
(Standard gravity equals 9.81 m/s . ) 7. W hat is the hydraulic radius of a 6-cm
59. What is a possible outcome of an diameter pipe fi lled to depth of 2 em?
A. 9.81 N
experiment called? A. 0.90 em
B. 19.8 N
A . a sample space 8.1.12 em*
C. 18.9 N
B. a random point C. 2.11 em
D.98.1N*
C. an event D. 3.11 em
D. a finite set F = mg = (10 kg) (9.81 ~)
60. In probability theory, what is the set of = 98.1 N ( -i }c:ABC)= arcco{~) = 70.53°
all possible outcomes of an experiment
called? 4. A 50 kg block is raised vertically 5 0 = LABC = (2{70.53°)
A. a set of random events meters. What is the change in potential
B. a fuzzy set energy?
= 141.06° = 2.462 radians
C. a cumulative distribution A . 1452.50 J
D. a sample space
REFRESHER MANUAL 2nd Edition by JAS TORDILLO
REFRESHER MANUAl 2"d Edition by JAS TORDILLO
4-8 DAY 4- SOLUTION
DAY 4 - SOLUTION 14- 9
ENGINEERING MATHEMATHICS
The area in flow and arc length are f(x) = X: - 3x.
f'(x) = 3~- 3
A = ..!. r 2 (ill -sin ill) yellow ball from the second box in one
2 draw from each bowl?
The slope at x = 3 is A. 3/50*
= (±}3)2(2.462 rad -sin(2.462 rad)) = =
f(3) 3(4)2 -3 48-3 = 45 B. 1/5
=8.251 cm2 c. 3/10
11 . A circle is expressed parametrically D. 1/10
by the equations 14. Eight people are on a sinking boat.
S = fi2l =(3 in)(2.64SI ra~)_= 7.386 em = =
x 10 cos 9 and y 10 sin 9 There are five life jackets. How many p{red and yellow} = p{red}p{yellow}
Express the derivative dy/dx as a function combinations of survivors are there?
The hydraulic radius is A. 56* 2X3) 6 3
ofx andy. = ( lo lo = 100 = 50
A 8.251 A. y/x B. 51
rh =- = - - = l.l2cm
s 7.386 B. -x/y * C. 45
C.x/y D. 30 18. How much should you put into a 10%
D. x-y savings account in order to have P20,000
8. What is minimum distance between
C(8,4) = _8_1- - 8.7.6.5.4.3.2.1 in five years?
the
(8-5)51- (3.2.1Xs.4.3.2.l) A. P-10,418.42
y = 3x .+ 4 and the origin (0,0)? dx = -LOsine = -y B. P12,418.42 *
A. 0.95 de =56
C. P22,418.42
B.1.26*
dy =JOcose= x D. P32,418.42
c. 2.25 de 15. A pianist knows five pieces but will
D. 5.25 dy have enough stage time to play only
three of them. How many different
P =s(1 + i)-n =(2o,ooo)(1 + o.10r5
Equation: 3x - y + 4 = 0
dy _ jJft_ =-X f y
dx- dx programs can be arranged?
=P12,418.42
de
A. 24 19. Maintenance costs for a machine are
d _ IAx+ By+CI _ (3Xo)+(-!Xo)+ 4 B. 45 P500 each year. What is the present
C.60*
- JA2 +B2 - ~(3)2+(-1)2 12. What is the partial derivative azJax of
D. 90
worth of these maintenance costs over a
12 year period if the interest rate is 8%?
the following function?
= 1.26 z = 4~ - 7f + xy + 6y- 10 A. P1,768
A. 4x + y P(4,3) = ~51_=~- B. P2,768
9. Find the angle between the lines. (5-3) 2.1 -60
B. 8x + y * C. P3,768 *
y =- 0.752x + 3 C. 4~ -7 D. P5,768
y = +0.752x- 6 D. 8x- 7y2 + x + 6y 16. A box contains six white balls, three
A. 13.48° red balls and four green balls. What is the
B. 23.48° " Oz. probability of getting either a white ball or p = R[(l+if -I]
C. 33.48° - = 8>1- o+ y + o- o = 8x + y a red ball in one draw from the bowl? i(l + tf
Ox
D. 38.48° A. 6/13 .
= 500 [ (1 + o.o8j2 - 1 J
.;

13. Find the derivative dy/dx of B. 3/ 13 p


121 = arctan [ m2 - ml ] f(x,y) = 2x2 + xy + 2y3 = 0 c. 9/13 * o.os{t + o.osj2
A. -(4x + y)/(x + 6y2 ) " D. 3/5
l + m1m2
B. (2x + y)/(x + 3y2)
= (500)(7.5361)
6 3 9 = P3,768.04
- [ 0.752 -{- 0.752)]
- I + (0.752X- 0.752)
C. (2x + y)/(x + 3y2) p = p{white} + p{red} = 13 + 13 =13
D. (4x + y)/(x + 3y2 ) 20. Your are currently paying P620 per
= 23.48° Of 17. One box contains five white balls. two
month to lease your office phone
-=4X+y equipment. You have three .years left on
ox red balls, and three green balls. Another the five-year lease. What would have
10. What is the slope at x = 4 of the curve
box contains three yellow and seven
f (x) = x3 - 3x? been an equivalent purchase price two
A. 25
~ =X+6y2 black balls. What is the probability of years ago? The effective interest rate per
Oy
getting a red ball from the first box and a
B. 35 month is 1%.
C. 45 * A . P17,872.12
D. 55
REFRESHER MANUAL 2nd Edition by JAS TORDILLO
REFRESHER MANUAL 2nd Edition by JAS TORDILLO
4- 10 DAY 4 - SOLUTION
DAY 4 - SOLUTION 14- 11
ENGINEERING MATHEMATHICS
B. P27,872.12 * which it will be sold for P400. Find the
C. P37,872.12 depreciation in the first three years using patent right cost to production cost, what A. P8,181.95 ~
D. P67,872.12 double declining balance. is the amortization rate in pesos per unit? B. P9,181.95
A. P4340.80 * A. P1000 C. P10,181.95
B. P5340.80 B. P1250 * D. P18,181 .95
P= A [(l+i'f'-J] C. P6340.80 C. P1500
i(l +i'f' D. P7340.80 D. P1850 The tax credit:
TC = (0.20)(5000) = P1000
::; 620 [ (1+ 0.0 1)60--..! ]· • _ (2X9ooo- 4oo) 1
0.01(1 + 0.01)60 D,- 10 = P1720 in year 1 1,000,000 = P1250 per unit P ::: - 5000 + 30,000 (0.45) - - · I
800 (1 + 0.1 O)
= (620){44.9550) = P27,872.12 _ (2X9ooo -1720) = P1456 in year 2
Dz- 10 1
26. A corporation that pays 55% of its + 1000
21. Maintenance on an old machine is profit in income taxes invests P10,000 in (I+ 0.10)1
_ (2X9ooo-<t456+tno>)
P1000 this year but is expected to D3 - 10 a project that will produce P3000 annual = -5000 + (30,000)(0.45)(0.9091)
increase by P250 each year thereafter. revenue for eight years. If the annual + (1000)(0.9091)
What is the present worth of five years of = P1164.80 in year 3
expenses are PBOO, salvage value after = P8,181.95
maintenance? Use an interest rate of eight years is P600, and 9% interest is
10%. Dep total = 1720 +1456 +1164.80 28. A ski resort installs two new ski lifts at
= P4340.80 used, what is the after-tax present worth?
A. P3506.25 Disregard depreciation. a cost of P2,000,000. The resort expects
B. P4506.25 A.- P4219.43 * annual gross revenue to increase
C. P5506.25 * 24. An asset is purchased for P9000. Its P600,000 while it incurs an annual
estimated economic life is ten years, after B. - P5219.43
D. P8506.25 C. - P6219.43 expense of P75,000 for lift operation and
which it will be sold for P400. Find the maintenance. What is the pay-back
R[(l + i)" - 1.]+A[(12+ i'f' - 1
i(l~i'f']
depreciation in the first three years using D. - P7219.43
p = period?
i(l + i'f' i (J + i)" sum- of-the-years' digits depreciation A. 2.5 years
P= - 10,000 +

10{ (1+0.10)
5-1] methods.
A. P4221.81 *
B. P5221.81 3000 [ (I+ 0·09..,. - I ] (1- 0.55)
B. 3.0 years
C. 3.8 yea..S *
0.10(1 + 0.10)5 o.o9(1 + o.o9f' D. 4.5 years
C. P6221.81
2{ (1+0.10)5-1 5 J D. P7221.81
- 8oo[ J
(l +O.o9f' - 1 (1-0.55) Pay-back period
+ 0. 102 (1 + 0.10)5 - 0. 10(1 + 0.10)5 0.09(1 + 0.09..,. . = 2,000,000 = 3_8 years
T = (~)JOXIl)= 55 +
600
600,000-75,000
= (1000)(3.7908) + (250)(6.8618) (1 + 0.09)8
= P5506.25 D, = (~~)<9000 - 400) = P1,563.63 in yr 1 = -10,000 + (3000)(5.5348)(0.45)
29. What is the uninflated present worth
of P12,000 in two years if the average
22. An asset is purchased for P9000. Its ~
- (800)(5.5348)(0.45) + (600)(0.5019) inflation rate is 8% and interest is 12%?
estimated economic life is ten years, after D2 = (; )s6oo) = P1 ,407.27 in yr 2 :; -10,000 + 7471.98 -1992.53 + 301.12 A. P5,201 .60
5 = - P4219.43
which it will be sold for P400. Find the B. P6,201.60
total depreciation for the first three years D3 = (~)(8600)
55
= $ 1250.90 in yr 3. 27. One year, a company makes a PSOOO
C. P7,201.60
using straight line. D. P8,201.60 *
investment in a historic building. The
A. P2580.00 *
B. P3580.00 Dep tolal = 1563.63 + 1407.27 + 1250.90 inve:rtment is not depreciable, but it does
qualify for a one-time 20% tax credit. In
IP = ( 12000
\2( \2 = P8201.58
C. P4580.00 = P4221.81 2
D. P5580.0Q
that same year, revenue is P50,000 and i.l r l.O&r
25. A company purchases complete and expenses (exclusive of the PSOOO
investment) are P20,000. The company 30. A P15,000 simple interest loan is
9000-400 exclusive patent rights to an invention for
Dep = = P860 each year pays a total of 55% in income taxes. taken out at 15% per annum interest rate.
10 P1,000.000. It is estimated that, once What is the after-tax present worth of this The loan matures in two years with no
DePtotal = 3 (860) = P2580.00 commercially produced, the invention will year's activities if the company's interest intermediate payments. How much will be
have a specific but limited market of 800 rate for investment is 10%? due at the end of the_second year?
23. An asset is purchased for P9000. Its units. For the purpose of allocating the
estimated economic life is ten years, after REFRESHER MANUAL 2nd Edition by JAS TOROILLO
REFRESHER MANUAL 2"d Edition by JAS TORDILLO
4- 12 DAY 4 - SOLUTION . DAY 4 - SOLUTION 14 -13
A. P17,250 ENGINEERING MATHEMATHICS
J
B. P19,500 * R* = 8314
kmoi.K floor. There is a frictional force of 80
C. P22,500 horsepower) is required for the trailer
J pounds between the crate and floor. alone?
D. P30,000 * 8314 ------
R
R= - = kmoiK
kg =189.0J/kg-K What work is done by the frictional force A. 3.35 hp
The interest each year is MW 44- - on the crate? B. 4.47 hp *
I = (0.15)(15,000) =P2250 kmol A. 1200 ft-lbf c. 5.26 hp
The total amount due in two years is 5 B. 1800 ft-lbf D. 7.75 hp_
p (1.50 x 10 )Pa c. 2400 ft-lbf"
S = 15,000+ (2)(2250) P19,500 = (laON112akmX,ooo~-)
= (189.0k~K}70+273)<
D. 3000 ft-lbf
31 . P6000 is borrowed for 90 days at
p = RT p =Fv = . '\. h km
18% per annum simple interest. How = 2.31 kg/m 3
WfricUon = Frx d = (80 lbf) (30ft)
= 2400 ft-lbf
(6o~X6o
mrn
minJIOoo~)
h kW
much will be due at the end of 90 days?
A. P6270 *
37. A solid disk flywheel (I= 300 kg.m 2 ) is
B. P7260
C. P8350
SG = 32.!_ = 1.93
1.20 rotating with a speed of 1200 rpm. What P = (1.341 :~:,)3.33 kW)
D. P9950 is its rotational. kinetic energy? = 4.47 hp
34. What is the sea level (g = 32.2
2
A- 1,368 X 1033 J
Amount due ftlsec ) specific weight (lbf/ft3) of liquids B. 2,368 X 10 J *
40. How much energy is required to just
with densities of 65.5 lbm/ft3? C. 3,368 X 103 J
vaporize a kg of water which Is originally
= 6000 + (0.18) (.2Q_)(6000) A. 32.2 D. 4,368 X 103 J at 27°C and one atmosphere?
360 B. 42.2
A. 2562.65 kJ *
= P6270 c. 65.5 * ( 1200 rpm)C21t) B. 3562.65 kJ
,D. 76.7
32. The density of water is typically taken w= = 125.66 rad/s C. 4562.65 kJ
y = gp = g d~nsity in lbmtft3 60 s - D. 5562.65 kJ

l
3
as 62.3 lbm/ft . For engineering problems min
where greater accuracy is not required, gc

~loo2 =(~X300kgm2X125.66r:df
what is the value in kg/m 3 ? Q. = me (T2 - T1)
65.5 lb~ E=

132.2_!b~-=-!!... 4.187~ Jrl00-27°c)


A. 999
B. 997.4 * = 32.2-ft- ft = (Loki
= 2,368 X 103 J
C. 1000 ( sec 2 \ kg.°C}
D. 1.4 lbf -sec 2 = 305.65 kJ
38. A 5.0-paund is launched straight up
= 65.5 lbf/ft3
~~} = 2257 kJ

with an initial velocity of 1000 ft/sec.
35.31 3 Qr9 =mhrg = (1 kg{2257
Neglecting air friction, calculate the

2.205~
1
P = 62 31bm ft3]
m 35. A lawn mower engine is started by
kinetic energy immediately after launch. Q, = Q. + Q, = 305.65 + 2257
pulling a cord w~apped around a sheave.
[ . •' A. 77,639.75 ft·lbf* = 2562.65 kJ
The sheave radius is 10.0 em. The cord
B. 87,639.75 ft-lbf
is wrapped around the sheave two times.
C. 97,639.75 ft-lbf 41 . What mass of nitrogen is contained in
= (62.3~
3 If a constant tension of 100 N is
yl6.01 kg-ft J maintained in the cord during starting,
D. 99,639.75 ft-lbf a 65 m3 tank if the pressure and
3 tt A
3 m -Ibm what work is done?
2 (Sibm{I000·-~-- )
2 temperature are one atmosphere and
21°C respectively?
~=
3 A.110.6J
= 997.4 kg/m Ekinetic = \ sec
B. 125.6 J * A. 75.4 kg*
2gc (2 { 32.2 lbm-ft
. . -· -- ) B. 57.4 kg
33. Determine the specific gravity of C. 160.6 J
D. 180 J sec2-lbf C. 54.7 kg
carbon dioxide gas (MW = 44) at 70°C
and 150 kPa using STP air as a . = 77,639.75 ft-lbf D. 45.7 kg
reference. T=Fr=100(0.10) =10N-m
A. 1.03 39. When traveling at 120 kph, a car I T = 21°C + 273 = 294 K
B. 1.93 * w
= T a= (10)(2)(2n rad) supplies a horizontal force of 100 N to the
C. 2.53 = 125.66 J hitch of a trailer. What tractive power (in I R = 297 J/kg.K
D. 3.33
36. A 300-pound crate is pushed 30 feet
at constant velocity across a warehouse REFRESHER MANUAL 2nd Edition by JAS TORDILLO
REFRESHER MANUAL 2"d Edition by JAS TORDILLO.
I -~

DAY 4 - SOLUTION 14- 15


4- 14 DAY 4 - SOLUTION ENGINEERING MATHEMATHICS
C. 39,345.6 kJ
(1 atm { 1.013 x 105 Pa X6sm3) D. 44,680.9 kJ
m = pV = \ atm P = Fv = (25 N) (t.o~) (10 kph)(10oo-r.kmn-·)
[~~]
_
RT (297 kg.K
J )(2941<) Vt - =19.44m/s
Q • nR"Tin =25W
3600~
= 75.4 kg 48. A 3000-kg cannon fires a 12- kg 2
42. A 0.90 m 3 tanl< contains 6.5 kg of an Q=5 kmol(8.314~x299
kmoLK
1
K)I..J atm)
'\ 8 atm
projectile horizontally at 700 m/s. It takes mv2 (3000kg) 19.44.!:!!)
0.009 seconds for the projectile to pass Fe= _ t = S
ideal gas. The ga~ has a molecular r 75m
Q = - 25, 846.27 kJ through the barrel. The cannon has a
weight of 44 and is at 21°C. What is the
spring mechanism to absorb the recoil. = 15,123.45 N
pressure of the gas? What force is exerted on the recoil
A. 201.3 kPa 45. What is the centroidal moment of
inertia with respect to the x-axis of a spring? 51. The measure of microscopic disorder
B. 301.3 kPa
rectangle 6.0 units wide and 9.0 units A. 7.77 X 105 N of a substance.
C. 401.3 kPa *
tall? B. 8.88 X 105 N A. Enthalpy
D. 501.3 kPa
· A. 164.5 units2 c. 9.33 X 105 N * B. Entropy*
R• 8314.3 ---~ -- ·B. 26~.5 units2 D. 10.33 x 105 N C. Internal energy
R = --= kmoi.K 2
C. 364.5 units * D. None of the above
(MW) 44 kg D. 464.5 units2
kmol (12 kg{ 700·rTl)
~=
52. A 0 .050-kg bullet attains a velocity of
= 189 J/kg.K F= \ s 82 m/s in a huntin!t slingshot. Contact
3 6t 0.009 s
lex=
bh
J2 = (6X9t
12
= 364.5 units4 with the sling is 1/10 of a second. What
T = 21°C + 273 = 294K = 9.33 X 105 N is the average force on the marble during
contact?
(6.5{189 J )(294K) 46. The acceleration in m/s2 of a 50-kg 49. Air enters a jet engine at 400 m/s and A. 20 N
mRT \ kg.K
p= - - = -~____:~:_____,.. body is specified by the equation leaves at 850 m/s. The thrust produced is B. 35 N
v (o.9om3XIOoo{;a) a(t) = 8- 16t
What is the instantaneous force acting on
46,500 N. Disregarding the small amount
of fuel added during combustion, what is
C. 41 N *
D.62N
= 401 .3 kPa the body at t = 4 seconds? the mass flow rate?
A. 4400 N *
43. One kg of hydrogen are cooled from B. 3300 N
A. 1 03.3 kg/s *
B. 123.3 kg/s
F = mt.v _ ( o.oso kgx82 ~)
t.t - I s
450°C to 320°C in a constant volume C. 2200 N C. 153.3 kg/s 5
process. The specific heat at constant D.1100N D. 193.3 kg/s . lo
volume, c.,, is 10.2 kJ/kg-K. How much F = 41 N
2
heat is removed? a= 8- (16)(6) =- 88 m/s
F 46500 N 53. A golf ball dropped from a height of
m = -=-----:
A. 1136 kJ
B. 1326 kJ * F = ma = (50 kg) (- 88 ~) ... l.l.v 850-400~
s
3.0 r'n onto a hard surface rebounds to a
height of 2.0 m. What is the coefficient of
C. 1623 kJ
D.2136kJ F= -4400N = 103.3 kg/s restitution?
A. 0.75
Q = mc.,(T2- T,) 47. Twenty five kg of sand fall 50. A 3000 kg car travels at 70 kph B. 0.81 *
continuously each second on a conveyer around a curve with a radius of 75 m. C. 0.92
= (1 kg) (10.2~Y
kg.K)\
45o 0
c
-32o 0
c)
belt moving horizontally at 1.0 m/s. What What is the centripetal force? D. 0.99
power is required to keep the belt A. 12,123 N
= 1326 kJ moving? B.15,123 N *
A.10W C. 21,123 N ..!.mv2 =mgh
2
44. Five kmols of air initially at one B. 15W D. 35,123 N

.b{9.81 ~}3m)
atmosphere and 299 K are compressed C. 25 W*
isothermally to 8 atmospheres. How D. 35W v = J2Qh =
much total heat is removed during the
compression? F=mt.v= (2s~X1.o:) = -7.67 m/s
A. 25,846.3 kJ *
B. 28,922.9 kJ =25 N REFRESHER MANUAL 2nd Edition by JAS TORDILLO

REFRESHER MANUAL2"d Edition by JAS TORDILLO


DAY 5- EXAM ~5-1
4-16 DAY 4 - SOLUTION
ENGINEERING MATHEMATHICS
v' = J2gh' = J(iX9.81)(2.o) e= "it(ro+coo)=G)(toXs2.3+0)
= 6.26 m/s C . CPM
DAY 5- EXAM
v2 = v2 =·a = 261.8 rad D. CPM, RME and PME

e= 6.26-0 =0.81 57. A turntable starts from rest and 1. For violating RA 8495, the penalty is 7. Maximum plant capacity In which a
0- (- 7.67)
~
accelerates uniformly at 2.3 rad/sec2 • not less than P50,000 nor more than licensed ME is allowed to operate, tend
tiow many revolutions will it take before a or maintain
54. Find the defle~tioQ, y, at the free end rotational speed of 45 rpm is attained? A. P100,000 A. 300 kw
of the cantilever bea?h ~/Ne.glect the beam A. 0.66 rev B. P200,000 B. 2000 kw
weight. B. 0.77 rev* C. P150,000 c. 500 kw
A. FL3/3EI* C. 0.88 rev D. P300,000 D. 6000kw
B. FL3/48EI D 0.99 rev
3 2. To be qualified to become a member
C. FL /12EI 8. For a mechanical plant with capacity of
D. FL3/24EI of the Board of Mechanical Engineering, 100 kw or over but not more than 300 kw,
rev )( rad)
( 45 - - 2n : - the minimum age is _ _ . the personnel required must be at least
- min rev = 4.71 rad/sec A. 35
w- sec one:
.!.(FLXL) = J..FL2 60--- B.40 A. CPM
2 2 min
C. 4'5 B. PME
FlJ
e=- D.32 C. RME
2
2EI co - ro6
_ (4.71)2- (of = 4.82 rad D. CPM, RME or PME

= FL
2
(~L)= FL
3 e = ~- (2X2.3) 3. Maximum plant capacity in which a
CPM is allowed to operate, tend or
y 2EI 3 3EI
9. The PRC accredited national
maintain: association for mechanical engineers:
_ 4.82 rad = 0.768 revolution . A. 200 kw A. ASME
55. A flywheel is brought to a standstill n- rad
27t--- B. 500 kw B. PAMEE
from 500 rpm in 10 seconds. What was rev C. 300 kw C. PSME
its average angular acceleration in D. 100 kw D. PICE
rad/sec durin~ that period?
2
58. What type of curve is generated by a
A. - 2.23 rad/s point which moves in uniform circular 4. Section 22 of Republic Act No. 8495 10. The new Mechanical Engineering
B. - 3.23 rad/s2 motion about an axis, while traveling with "The Philippine Mechanical Engineering Law was signed last~---·
C. - 4.23 rad/s2 a constant speed, v, parallel to the axis? Act of 1998" is about: A. February 12,1998
D. - 5.23 rad/s 2 * A. a cycloid A . oath B. February 6, 1998
B. an epicycloid B. Issuance of Certificates of Registration C. January 18, 1998
re_v )( n: rad) C. a hypocycloid and Professional License D. February 1.4, 1999
( 500 mm 2 rev D. a helix* C. Re-examination ·
wo= D. Seal of a Professional Mechanical 11 . The new Mechanical Engineering
60 s
min 59. What is a possible outcome of a[} Engineer Law is a House Bill No. 9806 was finally
= 52.3 rad/s experiment called? passed by the House Representatives
5.' The law that governs the practice of last._,__ _.,--~
A. a sample space mechanical engineering is called: A. February 3, 1998
= co-roo = 0-52.3 A. CA No.194
0 B. a random point B. January 28,1998
t 10 C. an event* B. RA No.7920 C. January 18, 1998
=- 5.23 rad/s2 D. a finite set C. RA No. 8495 D. February 12, 1998
D. RA No. 8995
56. From the previous problem, How far 60. In probability theory, wha£ is the set of 12. The new Mechanical Engineering
(in radians) did the flywheel travel? all possible outcomes of an experiment 6. To be responsible and in- charge in the Law is a Senate Bill No. 1793 was finally
A. 262 rad • called? preparation of plans and designs of a passed by the Senate last._ _ __
B. 322 rad A. a set of random events mechanical plant: A. February 14, 1999
C. 455 rad B. a fuzzy set A. PME B. January 12, 1998
D. 622 rad C. a cumulative distribution 8 . RME
D. a sample space *
REFRESHER MANUAL 2nd Edition by JAS TORDILLO
REFRESHER MANUAL2"d Edition by JAS TORDI~LO
DAY 5- EXAM ~5-3
s-2 DAY 5- EXAM ENGINEERING MATHEMATHICS
C. February 3,1998 19. The Board of Mechanical Engineering
D. January 28, 1998 is composed of _ _ members to be A. two A. Qualifications of Applicants for
appointed by the President of the B. three Mechanical engineer
13. To teach professional subjects in Philippines. C. four B. Qualifications of Applicants for CPM
mechanical engineering course unless he A. one D. five C. Qualifications of applicants for PME
is a duly licensed: B. three D. None of the above
A. PME C. two 25. One of the qualifications of applicants
B. Master's degree ttl ME D. five for Professional Mechanical Engineer be: 31. The new Mechanical Engineering
C. any of these " • , He is competent to practice, as attested Law is a consolidation of the Senate Bill
D. Doctorate degree holder in ME 20.' One of the following is not a correct to by at least _ Professional and House Bill Nos.
qualification of Members of the Board: Mechanical Engineers. A. 1792 and 8806
14. To be in responsible charge of the A. Natural born citizen and resident of the A. one B. 1893 and 9912
construction, erection, installation, Philippines. B. two C. 1993 and 9907
alteration, or of the performance of a B. Must be at least 35 years of age. C. three D. 1793 and 9806
mechanical engineering service in c: A PME with a valid professional D. many
connection with the manufacture, sale, license and an active practitioner for not 32. Republic Act 8495 known as the
supply or distribution of any mechani(fal less than 10 years. 26. , Section 34 in the Mechanical Mechanical Engineering Act of 1998 was
works, unless he is a duly registered : D. Has been convicted of any offense
Engineering Law is about: approved last:
A. CPM and moral turpitude. A. Preparation of Plans A. Feb 12,1998
B. Graduate of ME B. Personnel Required In Mechanical B. Feb 3,1998
C. PME or RME 21 . A member of the board shall hold C. Jan 26, 1998
Plant
D. Doctor of Education office for alarm of years from D. Feb 25,1998
C. Field of Action
the date of his appointment D. Renewal of license
15. 100 kw or over but not more than 300 A. three 33. At section 35 of ME Law, pressurized
kw. the personnel required in one CPM or B. two 27. Section 24 in the Mechanical pipes will have a working pressure of
one RME or one PME. Provided that · C. one Engineering law is about: not less than?
every mechanical work in this category D. four A. Seal of a Professional Mechanical A. 150 kPa
operating on more than_ s~ift every Engineer B. 95 kPa
twenty-four hours. 22. No member of the Board shall serve
B. Revocation and Suspension of C. 100 kPa
A. three for more than _ _ regular terms.
Certificate D. 70 kPa
B. two A. one C. Integration and Accreditation of ME
C. four B. two D. Grounds for Suspension 34. The PSME was established in what
D one C. three year?,
D. none of the above 28.. Section 42 in the Mechanical A. 1945
16. The Mechanical Engineering Law engineering law is about: B. 1970
ha's how many articles? 23. Which of the following is/are the
powers and duties of the Board ?
A. Penalties c. 1952
A. two B. Oath D. 1998
B. three A. To supervise the registration, licensure
C. Title
C five and practice of mechanical engineers 35. PSME is one of the accredited
D. Statement of Policy
D. six in the Phils.
B. To issue the certificate of registration
. professional organization in
29. The mechanical engineering law is I Philippines with a Certificate No. of:
the

17. What are the contents in Article II? to successful examinees. composed of _ _ _ sections. A. 001
A. Board of Mechanical Engineering C. Approve, issue, limit or revoke
temporary license to practice mechanical
A. 68 B. 004
B. Statement of Policy
engineering.
B.48 c. 002
C. Term of Office C. 54 0.010
D. Examination, Registration and License D. All of these. D. 35 36. The exact period when the
18. What are the contents in Article Ill?
A. Board of Mechanical Engineering
24. One of the qualifications of applicants
for Professional Mechanical Engineer
I
Mechanical Engineering Law begun and
30. Section 15 in the Mechanical held in Manila during the third regular
Engineering Law is about: session of the tenth congress.
B. Practice of the Profession must be: He has specific record of a total A. July 28,1997
C. Examination, Registration and License of or more I of active
D. Term of Office mechanical engineering practice. REFRESHER MANUAL 2nd Edition by JAS TORDILLO
REFRESHER MANUAL 2"d Edition by JAS TORDILLO
r

DAY 5- EXAM ~s-s


S-4 DAY 5 - EXAM ENGINEERING MATHEMATHICS
B. Feb 4, 1998 A. master's degree in ME only
C. August 20, 1997 B. doctorate degree in ME only C. two ME 53. Which of the following is NOT a
D. Feb 21, 1998 C. master's or doctorate degree in ME D. one CPM qualification of the Members of the
D. NOTA Board?
37. What are the contents in Article I? 48. The Rules and Regulations A. Must not, .tor a period of 5 consecutive
A. ME Laws .. 43. In section 41, a government or private Implementing the Philippine Mechanical years be a member of the faculty of any
B. Title. Statement of Policy and institution where a position requires a Engineering Act of 1998 are formulated institution
Definition of Terms master's degree holder, a holder of and issued pursuant to-what article of RA B. Has never been convicted of any
C. Contents of ME L~w , ,.,. license shall be eligible 8495? offense
D. Board of ME the position. A. Article V C. A Profes::;ional Mechanical Engineer
A. PME B. Article I D. An active practitioner for not less than
38. What are the contents in Article V? B. RME C. Article V 1 10 years prior to his appointment
A. Final and Conclusion C. CPM D. Article If
B. Penal and Concluding Provisions D. PME, RM E or CPM 54. One of the most important
C. Penalties 49. OTEC means: qualifications for Professional Mechanical
D. Transitory Provisions 44. In section 6, a member of the board A. Organization of Technical Engineers Engineer is to have a specific record of a
shall hold office for a term o~3 years Countrywide total of years or more of active
39. What are the contents in Article IV? from the date of his appointment and no B. .Ocean Thermal Energy Conversion mechanical engineering practice.
A. Practice of the Profession member of the board shall serve for C. Ocean Thermal Energy Conservation A.2
B. Personnel Required in Mechanical more than regular terms. D. Ocean Technical Engineers B.5
Plant A. one Corporation C. 3
C. Preparation of Plans B. three D.4
D. Posting of Certificates C. two 50. What is meant by the term Board?
D. four A. Chairman of the Board 55. The seal of a PME is In a form of a
40. Which of the following Ware required B. Chairman of the Committee gear with a 40 mm diameter outer circle
to secure a Temporary/special Permit 45. In section 6, any vacancy occurring C. Board of Mechanical Engineering 20 mm diameter inner circle. Inside the
from the Board of Mechanical within the term of a member due to some D. PRC Board lower circular band shall be inscribed, the
Engineering: cause, shall be filled by appointment by words
A. Mechanical Engineers frqm other the President of the Philippines .and such 51. In the guidelines for applicants in A. Mechanical Engineer
countries called in for consultation member appointed shall serve for the Professional Mechanical Engineer, the B. Official Seal
B. Foreigners employed as , technical unexpired portion of the term vacated following papers must be submitted, (1). C. Name of the Professional
officers without prejudice to serve for a maximum Affidavit of the applicant. (;2) Certificate of D. Registration number
C. Foreigners employed as consultants in of _ _ full terms. experience, (3). List of designs and (4).
such installation A. one Certificate of Competence under oath by 56. What is meant by Commission
D. All of these B. three a Chairman?
C. two A.-=R
::-:M
-::-:E::--- · A. PRC Commissioner
41 . In section 18, to pass the examination D. four B. PME B. Chairman of the Board
a candidate must obtain with no C. CPM C. Chairman of the CHED
rating below _ _ percent in any of the 46. What is the minimum pipe working D. Manager D. Chairman of the CPE
subjects and obtain an average of pressure to be considered as mechanical
percent on all subjects. equipment? 52. Which of the following refers to as a 57. An important guideline for applicants
A. 70%,50% A. less than 70 kPa practice of Mechanical Engineering? in Professional Mechanical Engineer
B. 50%,70% B. 70 kPa A. Project studies of any mechanical Examination is to require the applicant to
c. 75%,60% c. 60 kPa projects present himself before at least two
D. 50%,75% D. 40 kPa B. All of these members of the board for
C. Management of the installation of interviews.
42. In section 41, where a position 47. One of the most important mechanical projects A. three
requires a Professional Mechanical qualification for PME, that he is . D. Management of the sale of mechanical B. single
Engineer license, a holder of a _ _ __ competent to practice as attested .by at parts · C. four
with a registered mechanical engineer's least--------- D. two
license shall be considered for the A. two PME
position. B. one PME REFRESHER MANUAL 2nd Edition by JAS TORDILLO
REFRESHER MANUAL2"d Edition by JAS TORDILLO
s-6 DAY 5- EXAM DAY 5 - EXAM Is -7
58. In the evaluation of the experience of
ENGINEERING MATHEMATHICS
64. The official Seal of the Board is
applicants in PME examination, the represented by a drawing of:
following should be considered: design A. Gear
70. In the implementing rules and tS: -fhe seal of a Professional
regulations, what is the content in section Mechanical-Engineer is in the form of a:
experience, maintenance and operation, B. Hero's Steam Engine
31? A. kettle
fabrication and shop practice and C. spherical kettle
A. Full Computerization of Examination B. gear
D. internal cylinder
B. Renewal of Professional License C. pressure vessel
A. management
C. Qualifications of the Board D. machine
B. operation 65. In section 28, to pass the
D. Transitory Provisions
C. employment ~: · t· examination, a candidate for PME, RME
D. installation and CPM must obtain an average of 77. The seal of a PME is in a form of a
71. What is the content in Rule 11? gear with a ~0 mm diameter outer circle
___,,.-----, percent in all subjects, with no
59. In the Meetings of the Board, in the rating below _ _ percent in any of the
A. Statement of Policy and 20 mm diameter inner circle. A
B. Effectivity rectangular band placed horizontally
absence of the chairman, which of the subjects.
C. Grounds for the suspension across the center of the gear is the
members will preside? A. 70%,7.0%
D. Official Seal of the Board
A. two members B:50%, 70%
B. senior member C.-70%,.60%
A. Registration Number
72. The title of Resolution No. 16 series B. Name of the Professional
C. commissioner D. 70%,50%
of 1998 is: C. Official Seal
D. assistant commissioner
66. Which of the following is/are grounds A. 'Code of Ethics of Mechanical D. Company Name
for the revocation of Professional Engineers
60. It is the unification of all mechanical
B. Philippine Mechanical Engineering 78. In the Code of Ethics for Mechanical
engineers including CPM into one Certificates?
national organization of mechanical A. Use of any fraud in obtaining Code Engineers in the Philippines, Rule No. 5
engineers. certificate C. Rules and Regulations implementing states that 'The mechanical engineer
RA 9495 shall honor and respect the supreme
A. engineering firm B. Negligence
8 . engineering management C. all of the these D. Mechanical Engineering Act of 1998 authority of the state, the rule of law, the
C. consultation D. violation of the policies of the board obligations and privileges of citizens
73. Which of the following is NOT recognized and guaranteed by the
D. integration
considered a mechanical equipment? Constitution, of the Philippines". This
67. The Board of Mechanical Engineering
61. In Section 7 of the Implementing rules is composed of:
A. Xerox machine statement is for our relation with
B. fired pressure vessel A. the state
and regulations, a member of the board, A. one chairman and a member
C. Internal combustion engines B. labor
shall hold office for a term of _ _ years B. three members
D. crane C. the community
from the date of his appointment. C. one chairman and two members
A. two D. five members D. employers
74. What is the content in section 43?
B. four
A. Seal of a Professional Mechanical 79. In the Code of Ethics for Mechanical
C. three 68. Which of the following is NOT a
qualification of the Members of the Engineer Engineers in the Philippines, Rule No. 14
D. six
Board? B. Code of Ethics states that "He shall strive to improve the
62. What are the contents in Rule V? A resident of the Philippines C. Teaching of Mechanical Engineering skills of his workmen, pay them a fair
A. Licensure examination for the practice B. holder of the degree of BSME Subjects living wage, and instruct them on how to
D. Report of the Board prevent accidents in their place of work".
of mechanical engineering C. Must be at least 35 years old
B. Administrative proceedings D. must be a national officer of the PSME This statement is for our relations with
C. Miscellaneous provisions 75. The Rules and Regulations
D. Practice of Mechanical Engineering 69. The seal of a PME is in a form of a Implementing the Philippine Mechanical A. labor
gear with a 40 mm diameter outer circle Engineering Act of 1998 are formulated B. client
63. In the implementing rules and and 20 mm diameter inner circle. Inside and issued pursuant to what section of C. safety
regulations, what is the content in section in the upper circular band shall be RA 8495? D. associates
24? A. Definition ofTerms inscribed the words _ _ _ _ __ A. Section 5
B. Issuance of Certificates of Registration A. Professional Mechanical Engineer B. Section 49 80. In the Code of Ethics for Mechanical
C. Section 10 Engineers in the Philippines, Rule No. 18
C. Scope of Examination B. Registered Professional
D. Section 43 states that 'The mechanical engineer is
D. Renewal of Professional License C. Mechanical Engineer
D. Professional Engineer entitled to a just and fair compensation
REFRESHER MANUAL 2nd Edition by JAS TORDILLO
REFRESHER MANUAL 2nd Edition by JAS TORDILLO
s- s DAY 5 - SOLUTION
for his services." This statement is for our
DAY 5 - SOLUTION 5-9
ENGINEERING MATHEMATHICS
I
relations with _ __
DAY 5 :- SOLUTION
7. Maximum plant capacity in which a 13. To teach professional subjects in
A. clients and employers licensed ME is allowed to operate, tend
1. For violating RA 8495, the penalty is mechanical engineering course unless he
B. employers only or maintain is a duly licensed:
C. colleagues and associates not less than P50,000 nor more than
A. 300 kw A. PME
D. salary and wage B. 2000 kw* B. Master's degree in ME
A. P100,000
... B. P200,000 * C. 500 kw C. any ofthese"
~
D. 6000 kw
"' . C . P150,000
D. P300,000
D. Doctorate degree holder in ME

0 8. For a mechanical plant with capacity of 14. To be. in responsible charge of the
DAY 5- ANSWER KEY 2. To be qualified to become a member 100 kw or over but not more than 300 kw, construction, erection, • installation,
1. 8 16. c 31. 0 46. 8 61. c 76. 8 of the Board of Mechanical Engineering, the personnel required must be at least alteration, or of the performance of a
2.A 17. A32. A 47.A 62. A 77.8 the minimum age is _ _ . one: mechanical engineering service in
3. C 18. C33. D 48. A 63. C 78. A
A . 35 * A. CPM* connection with the manufacture, sale,
4. B 19. 834. C 49. 8 64. B 79. A B..40 B. PME supply or distribution of any mechanical
5. C 20. 035. B 50. C 65. 0 80. A C. RME
C.'45 works, unless he is a duly registered :
6. A 21. A36. A 51. 8 66 C.
7. B 22. B37. 8 52. B 67. C D. 32 D. CPM, RME or PME A. CPM
8. A 23, D 38. B 53. A 68. 0 B. Graduate of ME
9. C 24. C 39. A 54. 0 69. 8 3. Maximum plant capacity in which a 9. The PRC accredited national C. PME or RME *
10. A 25. B 40. 0 55. A 70. A CPM is allowed to operate, tend or association for mechanical engineers: D. Doctor of Education
11. B 26. B 41. B 56. A 71. 0 maintain: A. ASME
12. C 27. A 42. C 57. D 72. C 8. PAMEE 15. 100 kw or over but not more than 300
13. C 28. A 43. A 58. 0 73. A
A. 200 kw
B. 500 kw C. PSME * kw, the personnel required in one CPM or
14. C 29. B 44. C 59. B 74. A
1l I 15. 0 30. A 45. c 60. 0 75. D c. 300 kw* D. PICE one RME or ohe PME. Provided that
D. 100 kw every mechanical work in this category
10. The new Mechanical Engineering operating on more than_ shift every
4. Section 22 of Republic Act No. 8495 Law was signed last..,....,.._ _ twenty-four hours.
"The Philippine Mechanical Engineering A. February 12,1998 * A. three
Act of 1998" is about: B. February 6, 1998 B. two
A. oath C. JClnuary 18, 1998 C. four
B. Issuance of Certificates of D. February 1.4, 1999 D. one*
Registration and Professional
License* 11 . The new Mechanical Engineering 16. The Mechanical Engineering law
C. Re-examination Law is a House Bill No. 98Q6 was finally ha's how many articles?
D. Seal of a Professional Mechanica passed by the House Representatives A. two
Engineer last::-:--~::--:"7 B. three
A. February 3, 1998 C. five*
5. The law that governs the practice o B. January 28,1998 * D. six
mechanical engineering is called: C. January 18. 1998
A. CA No.194 D. February 12, 1998 17. What are the contents in Article II?
B. RA No.7920 A. Board of Mech~nical Engineering *
C. RA No. 8495 * 12. The new Mechanical Engineering B. Statement of Policy
D. RA No. 8995 Law is a Senate Bill No. 1793 was finally C. Term of Office
passed by the Senate last._ _ __ D. Examin ation, Registration and License
6. To be responsible and in-charge in the A. February 14, 1999
preparation of plans and designs of a B. January 12. 1998 18. What are the contents in Article Ill ?
mechanical plant: C. February 3,1998 * A. Board of Mechanical Engineering
A. PME* D. January 28, 1998 B. Practice of the Profession
B. RME
C.CPM
D. CPM, RME and PME
I REFRESHER MANUAL 2nd Edition by JAS TORDILLO
REFRESHER MANUAL 2"d Edition by JAS TORDILLO
s - 10 DAY 5 - SOLUTION
C. Examination, Registration and 24. One of the qualifications of applicants
DAY 5 - SOLUTION 5 -11
ENGINEERING MATHEMATHICS
I
License* for Professional Mechanical Engineer.
D. Term of Office must be: He has specific record of a total 30. Section 15 in the Mechanical held in Manila during the third regular
of or more I of active Engineering Law is about: session of the tenth congress.
19. The Board of Mechanical Engineering mechanical engineering practice. A. Qualifications of Applicants for A. July 28,1997 *
is composed of _ _ members to be A. two Mechanical engineer * B. Feb 4, 1998
appointed by the ,President of the B. three B. Qualifications of Applicants for CPM C. August 20, 1997
Philippines. C. four* C. Qualifications of applicants for PME D. Feb 21,1998
A. one .. \.~ D. five D. None of the above
B. three* -•· " 37. What are the contents in Article I?
~
C. two 25. One of the qualifications of applicants 31 . The new Mechanical Engineering A ME Laws
D. five for Professional Mechanical Engineer be: Law is a consolidation of the Senate Bill B. Title, Statement of Policy and
He is competent to practice, as attested and House Bill Nos. Definition of Terms *
20. One of the following is not a correct to by at least _ Professional A. 1792 and 8806 C. Contents of ME Law
qualification of Members of the Board: Mechanical Engineers. B. 1893 and 9912 D. Board of ME
A. Natural born citizen and resident of the A. one C. 1993 and 9907
Philippines. B. two* D. 1793 and 9806 * 38. What are the contents in Article V?
B. Must be at least 35 years of age. C. three A. Final and Conclusion
C. A PME with a valid professional D. many 32.' Reput)lic Act 8495 known as the B. Penal and Concluding Provisions *
license and an active practitioner for not Mechanical Engineering Act of 1998 was C. Penalties
less than 10 years. 26. Section 34 in the Mechanical approved last: D. Transitory Provisions
D. Has been convicted of any offense Engineering Law is about: A. Feb 12,1998"
and moral turpitude. * A. Preparation of Plans B. Feb 3,1998 39. What are the contents in Article IV?
B. Personnel Required In Mechanical C. Jan 26, 1998 A. Practice of the Profession *
21. A member of the board shall hold Plant* D. Feb25,1998 B. Personnel Required in Mechanical
office for alarm of years from C. Field of Action· Plant
the date of his appointment. D. Renewal of license 33. At section 35 of ME Law, pressurized C. Preparation of Plans
A. three* pipes will have a working pressure of D. Posting of Certificates
B. two 27. Section 24 in the Mechanical not less than?
C. one Engineering law is about: A. 150 kPa · 40. Which of the following Ware required
D. four A. Seal of a Professional Mechanical B. 95 kPa to secure a Temporary/special Permit
Engineer* c_ 100 kPa from the Board of Mechanical
22. No member of the Board shall serve B. Revocation and Suspension of D. 70 kPa * Engineering:
for more than _ _ regular terms. Certificate A. Mechanical Engineers from other
A. one C. Integration and Accreditation of ME 34. The PSME was established in what countries called in for consultation
B. two* D. Grounds for Suspension - year?, B. Foreigners employed as technical
C. three A. 1945 officers
D. none of the above 28. Section 42 in the Mechanical B. 1970 C. Foreigners employed as consultants in
engineering law is about: C.1952 * such installation
23. Which of the following is/are the A. Penalties * D. 1998 D. All of these *
powers and duties of the Board ? B. Oath
A. To supervise the registration, licensure C. Title 35. PSME is one of the. accredited 41. In section 18, to pass the examination
and practice of mechanical engineers D. Statement of Policy professional organization in the a candidate must obtain with no
in the Phils. Philippines with a Certificate No. of: rating below _ _ percent in any of the
B. To issue the certificate of registration 29. The mechanical engineering law is A. 001 subjects and obtain an average of
to successful examinees. composed of ___ sections. B. 004 * percent on all subjects.
C. Approve, issue, limit or revoke A. 68 c. 002 A. 70%,50%
temporary license to practice mechanical B. 48 * D. 010 B. 50%,70% *
engineering. C. 54 c. 75%,60%
0 : All of these. * D.35 36. The exact period when the D. 50%,75%
Mechanical Engineering Law begun and
REFRESHER MANUAL 2nd Edition by JAS TORDILLO
REFRESHER MANUAL 2nd Edition by JAS TORDILLO
s- 12 DAY 5 - SOLUTION DAY 5 - SOLUTION Is -13
42. In section 41, where a position 47. One of the most important ENGINEERING MATHEMATHICS
requires a Professional Mechanical qualification for PME, that he is
Engineer license, a holder of a -:---:--- competent to practice as attested by at C. Management of the installation of members of the board for
with a registered mechanical engineer's least mechanical projects interviews.
license shall be considered for the A. tw-o-:::Pc::-M:::E:-*:-- D. Management of the sale of mechanical A. three
position. B. one PME parts B. single
A. master's degree in ME only C. two ME C. four
B. doctorate degree..in ME only D. one CPM 53. Which of the following is NOT a D. two*
C. master's or doctocat,e degree in ME* qualification of the Members of the
D. NOTA ' 48. The Rules and Regulations Board? 58. In the evaluation of the experience of
Implementing the Philippine Mechanical A. Must not, for a period of 5 applicants in PME examination, the
43. In section 41, a government or private Engineering Act of 19ga are formulated consecutive years be a member of the following should be considered: design
institution where a position requires a and issued pursuant to-what article of RA faculty of any institution * experience, maintenance and operation,
master's degree holder, a holder of 8495? B. Has never been convicted of any fabrication and shop practice and
.,.,.---=-- license shall be eligible A. Article V * offense
the position. B. Article I C. A Pmfessional Mechanical Engineer A. management
A. PME * C. Artic)e V1 D. An active practitioner for not less than B. operation
B. RME D. Article If 10 years prior tQ his appointment C. employment
C.CPM D. installation *
D. PME, RME or CPM 49. OTEC means: 54. One of the most important
A. Organization of Technical Engineers qualifications for Professional Mechanical 59. In the Meetings of the Board, in the
44. In section 6, a member of the board Countrywide Engineer is to have a specific record of a absence of the chairman, which of the
·shall hold office for a term of 3 years B. Ocean Thermal Energy Conversion* total of years or more of active members will preside?
from the date of his appointment and no C. Ocean Thermal Energy Conservation mechanical engineering practice. A. two members
member of the board shall serve for D. Ocean Technical Engineers A. 2 B. senior member*
more than regular terms. Corporation B. 5 C. commissioner
A. one c. 3 D. assistant commissioner
B. three 50. What is meant by the term Board? D.4*
C. two* A. Chairman of the Board 60. It is the unification of all mechanical
D. four B. Chairman of the Committee 55. The seal of a PME is in a form of a engineers including CPM into one
C. Board of Mechanical Engineering* gear with a 40 mm diameter outer circle national organization of mechanical
45. 111 section 6, any vacancy occurring D. PRC Board 20 mm diameter inner circle. Inside the engineers.
within the term of a member due to some lower circular band shall be inscribed, the A engineering firm
cause, shall be filled by appointment by 51. In the guidelines for applicants in words B. engineering management
the President of the Philippines .and such Professional Mechanical Engineer, the A. Mechanical Engineer* C. consultation
member appointed shall serve for the following papers must be submitted, ( 1). B. Official Seal D. integration *
unexpired portion of the term vacated Affidavit of the applicant, (2) Certificate of C. Name of the Professional
without prejudice to serve for a maximum experience, (3). List of designs and (4). D. Registration number 61. In Section 7 ofthe Implementing rules
of _ _ full terms. Certificate of Competence under oath by and regulations, a member of the board,
A. one a _ _ _ _. 56. What is meant by Commission shall hold office for a term of _ _ years
B. three A. RME Chairman? from the date of his appointment.
C. two* B. PME * A. PRC Commissioner* A. two
D. four C.CPM B. Chairman of the Board B. four
D. Manager C. Chairman of the CHED C. three*
46. What is the mmrmum pipe working D. Chairman of the CPE D. six
pressure to be considered as mechanical 52. Which of the following refers to as a
equipment? practice of Mechanical Engineering? 57. An important guideline for applicants 62. What are the contents in Rule V?
A. less than 70 kPa A. Project studies of any mechanical in Professional Mechanical Engineer A. Licensure examination for the
B. 70 kPa * projects Examination is to require the applicant to practice of mechanical engineering *
C. 60 kPa B. All of these * present himself before at least two B. Administrative proceedings
D. 40 kPa

REFRESHER MANUAL 2nd Edition by JAS TORDILLO


REFRESHER MANUAL 2"d Edition by JAS TORDILLO
s - 14 DAY 5 - SOLUTION
DAY 5 - SOLUTION
ENGINEERING MATHEMATHICS
\s -1s ·
C. Miscellaneous provisions 69. The seal of a PME is in a form of a
D. Practice of Mechanical Engineering gear with a 40 mm diameter outer circle Engineering Act of 1998 are formulated B. client
and 20 mm diameter inner circle. Inside and issued pursuant to \Yhat section of C. safety
63. In the implementing rules and in the upper circular band shall be RA 8495? D. associates
regulations, what is the content in section inscribed the w o r d s - - - - - - - A. Section 5
24? A. Definition of Terms A. Professional Mechanical Engineer B. Section 49 80. In the Code of Ethics for Mechanical
B. Issuance of Certificates of Registration B. Registered Professional * C. Section 10 Engineers in the Philippines, Rule No. 18
C. Scope of Examin~ion * C. Mechanical Engineer D. Section 43 * states that ''The mechanical engineer is
D. Renewal of Profession~ license D. Professional Engineer entitled to a just and fair compensation
76. The seal of a Professional for his services." This statement is for our
64. The official Seal of the Board is 70. In the implementing rules and Mechanical-Engineer is in the form of a: relations with _ __.
represented by a drawing of: regulations, what is the content in section A. kettle A. clients and employers *
A Gear 31? B. gear* B. employers only
B. Hero's Steam Engine * A. Full Computerization of C. pressure vessel C. colleagues and associates
C. spherical kettle Examination * D. machine D. salary and wage
D. internal cylinder B. Renewal of Professional License
c: Qualifications of the Board 77. The seal of·a PME is in a form of a
65. In section 28, to pass the D. Transitory Provisions gear with a 40 mm diameter outer circle
examination, a candidate for PME, RME and 20 mm diameter inner circle. A.
and CPM must obtain an ·average of 71 . What is the content in Rule 11? rectangular band placed horizontally
----,,.--..,....- percent in all subjects, with no A. Statement of Policy across the center of the gear is the
rating below _ _ percent in any of the B. Effectivity
subjects. C. Grounds for the suspension A. Registration Number
A. 70%,70% D. Official Seal of the Board * B. Name of the Professional *
B. 50%, 70% C. Official Seal
C. 70%,60% 72. The title of Resolution No. 16 series D. Company Name
D. 70%,50% * of 1998 is:
A. Code of Ethics of Mechanical. 78. In the Code of Ethics for Mechanical
66. Which of the following is/are grounds Engineers Engineers in the Philippines, Rule No. 5
for the revocation of Professional B. Philippine Mechanical Engineering states that "The mechanical engineer
Certificates? Code shall honor and respect the supreme
A Use of any fraud in obtaining C. Rules and Regulations authority of the state, the rule of law, the
certificate implementing RA 9495 * obligations and privileges of citizens
B. Negligence D. Mechanical Engineering Act of 1998 recognized and guaranteed by the
C. all of the these * Constitution, of the Philippines". This
D. violation of the policies of the board 73. Which of · the following is NOT statement is for our relation with _ __
considered a mechanical equipment? A. the state *
67. The Board of Mechanical Engineering A. Xerox machine * B. labor
is composed. of: B. fired pressure vessel C. the community
A. one chairman and a member C. Internal combustion engines D. employers
B. three members D. crane
C. one chairman and two members * 79. In the Code of Ethics for Mechanical
D. five members 74. What is the content in section 43? Engineers in the Philippines, Rule No. 14
A. Seal of a Professional Mechanical states that "He shall strive to improve the
68. Which of the following is NOT a Engineer* skills of his workmen, pay them a fair
qualification of the Members of the B. Code of Ethics living wage, and instruct them on how to
Board? C. Teaching of Mechanical Engineering prevent accidents in their place of work".
A. resident of the Philippines Subjects This statement is for our relations with
B. holder of the degree of BSME D. Report of the Board
C. Must be at least 35 years old
A. labor*
D. must be a national officer of the 175. The Rules and Regulations
PSME* Implementing the Philippine Mechanical REFRESHER MANUAL 2nd Edition by JAS TOROILLO
REFRESHER MANUAL 2nd Edition bv JAS TORDILLO
DAY 6- EXAM ~6-1
ENGINEERING MATHEMATHICS

C. save
DAY 6- EXAM D. investment
1. The amount of money paid for the use 7. The decrease in value of a property
of borrowed capital or income produced due to the gradual extraction of its
by money which has been loaned. contents .
...., •\.,Ut-
~
A. Interest A. Depreciation
B. income B. appreciation
C. percentage C. property loss
D. release amount D. depletion
2. The quantity of a certain commodity 8. A PLOT tower and a monument stand
that is offered for sale at a given price at on a level plane. The angles of
a given place and time. depression of the top and bottom of the
A. supply monument viewed from the top of the
B. goods PLOT .tower are 13 deg and 35,
C. demand respectively. The height of the tower is 50
D. bond meters. Find the height of the monument.
A. 16.5 m
3. On a negotiable paper is the difference B. 71.5 m
between the present worth and the worth C33.5m
of the paper at some time in the future. D. 95.5 m
A. differential amount
B. discount 9. A mechanical engineer was granted a
C. allowance loan of P20,000 by her employer with an
D. appraisal interest at 6% fo'r 180 days on the
principal collected In advance. If
4. The analysis and evaluation of the discounted at once, find the proceeds on
factors that will affect the economic the note.
success of engineering projects to the A. P20,000
end that recommendation can ·be made B. P14,000
which will insure the best use of capital. C. P18,800
A. economic analysis D. P1,300
B. break-even analysis
C. capital evaluation
. D. engineering economy
10. Mr. Suave borrowed P50,000 from
SSS, in the form of calamity loan, with
interest at 8%, ·compounded quarterly,
5. The process of determining the value payable in equal quarterly installments for
of certain property for specific reasons. 10 years. Find the quarterly payments.
A. depletion A. P4,827.80
B. determination B. P2,827.80
C. appraisal C. P3,827.80
D. estimation D. P1,827.80

6. Funds supplied and used by owners of 11 . Alexander Michael owes P25,000 due
an enterprise in the expectation that a in one year and P75,000 due in four
profit will be earned. · years. He agrees to pay P50,000 today
A. equity capital and the balance in 2 years. How much
B. credit must he pay at the end of two years if
REFRESHER MANUAL 2nd Edition by JAS TORDILLO
I
DAY 6 -EXAM ~6-3
6-2 DAY 6 - EXAM
ENGINEERING MATHEMATHICS
money is worth 5% compounded semi- 17. A rubber ball was dropped from a
annually? height of 36 m, and each time it strikes
A. another P50,000
21. What is the maximum profit of I B. work
the ground it rebounds to a height of 2/3 product y per day? C. weight
B. only P39,026.30 of the distance from which it fell. Find the A. P100 D. section modulus
C. only P25,000 total distance traveled by the ball before if B. P1500
D. only P30,000.00 comes to rest. C.P800 28. The coefficient of friction for dry
t
A. 72 m D.P2400 surfaces:
12. In problem," No. 11, what is the B. 150m A. depends on the materials and the
effective rater per y~ar?, C. 120m 22. A statement of equality between two finish condition of the surface
A. 10% D. 180m ratios: B. depends ·only on the finish condition of
B. 5.06% A. valuation the surfaces
C. 5% Problems 18 to 21 C. does not depend on the materials
B. power factor
D. 3.06% A certain company manufacture two 0:. depends on the composition of the
C. theorem
products, x and y, and each of these D. proportion materials only
13. A light rail transit travels between two products must be processed on two
terminals 1 km apart in a minimum time different machines. Product X requires 1 23. The side opposite the right angle of a 29. Output over input is:
of 1 minute. If the LRT cart accelerates minute of work time per unit on machine right triangle: · A. efficiency
and decelerate.s at 3.4 m per sec2 , 1 and 4 minutes of work time on machine B. bond
A. quadrilateral
starting from rest at the first terminal and 2. Product y requires two minutes of work B. hypotenuse C. rate of return
coming to stop at the second terminal, time per unit on machine 1 and 3 minutes C. apothem D. annuity
find the maximum speed. of work time per unit on machine 2. Each D. median
A. 66 kph day, 100 minutes are available on 30. An instrument for measuring specific
B. 60 kph machine 1 and 200 minutes are available 24. A plane closed curve, all points of gravity of fluids:
C. 55 kph on machine 2. To satisfy certain which are the same distance from a point A. flowmeter
D. 20 kph customers, the company must produce at within, called the center is: B. clinometer
least 6 units per day of product x and at A. chord C. hygrometer
14. In problem No. 13, how long does it least 12 units of product y. If the profit of B. arc D. hydrometer
travel at this top speed? each unit of product x is PSO.OO and the C. circle
A. 60 sec profit of each unit of product y is P60.00. D. radius 31. The decrease in value of property
B. 39 sec due to passage of time:
C. 49 sec • 18. How many units of product x should 25. Two triangles are congruent if two A. devaluation
D. 30 sec be ·produced daily in order to maximize angles and the included side of one are B. salvage value
the company profits? equal respectively to two angles and the C. depletion
15. An Aerolift airplane can fly at an air A. 6 units D. depreciation
included side of the other.
speed. of 300 mph. If there is a wind B. 20 units
A. axiom
blowing toward the East at 50 mph, what C. 10 units B. corollary 32. The difference between the book
should be the plane's compass reading in D. 40 units value and the actual lower resale value
C. postulate
order for its course to be 30 deg? D. theorem is:
A. 8.3 deg 19. How many units of product y should A . salvage value
B. 21.7 deg be produced daily in order to maximize 26. The volume of a circular cylinder is B. fixed cost
C. 30 deg the company profits? equal to the product of its base and C. resale value
D. 38.3 deg A. 10 units altitude: D. sunk cost
B. 20 units A. postulate
16. In problem No. 15, what will be the C. 12 units B. theorem 33. It occurs when a unique product or
plane's ground speed if its flies this D. 40 units C. corollary service is available only from a single
course? supplier and entry of all other possible
D. axiom
A. 188 mph 20. What is the maximum profit of suppliers prevented:
B. 311 mph product x per day?
·c. 300 mph A. P200
27. The dimension of Acceleration x A. monopoly
Mass is the same unit as that of: B. profitability
D. 322 mph B. P1000 A. length
C. P500
D.P2000 REFRESHER MANUAL 2nd Edition by JAS TORDILLO
REFRESHER MANUAL2"d Edition by JAS TORDILLO
DAY 6 - EXAM ~ 6-5
6-4 DAY 6 - EXAM ENGINEERING MATHEMATHICS
C. competition 40. The maximum moment of a beam
D. inventory supported at both ends and carrying a 46. The digits of a measurement which a A. 40 kph
uniformly distributed load is located at scientist reads and estimate on a scale B. 60 kph
34. The pressure reading of 35 psi in KPa A. either of the two ends are called: C. 80 kph
is: B. center A. accuracy D. 90 kph
A. 427.3 C. anywhere in the span B. precision
B. 273.4 v •
D. at the left end C. significant figures 53. A motorcycle starts from rest and
c 724 . ,_ · , t:t D. experimentation reaches a velocity of 50 m/s in 10 sec. If
D. 342.72 41. A cantilever beam having a uniformly its acceleration remains the same, what
increasing load toward the fixed end: 47. Galvanized iron is a term referring to will its velocity be 8 sec later?
35. What is the area of a triangle with A. has uniform shear iron coated with: A. 90 m/s
sides of 5, 7 and 10? B. has a reaction equal to the load A. aluminum B. 100 m/s
A. 16.248 C. will have maximum bending moment B. magnesium C: 180 m/s
B. 25.248 midway to the beam C. tin D: 220 m/s
c. 18.248 D. has a reaction equal to one half of the D. zinc
D. 30.248 load 54. A ball is moving at 20 m/s when it is
48. The sum of the sides of a polygon: struck by a bat and moves off in the
36. The radius of a circle inscribed in a 42. A chemist of a distillery experimented A. circumference opposite direction at 40 m/s. If the impact
triangle with sides of 5, 7 and 10 is · on two alcohol solutions of different B. square lasted 0.020 sec, find the ball's
A. 1.774 strengths, 30% alcohol and 60% alcohol, C. perimeter acceleration during the impact in m/s2 .
B. 1.744 respectively. How many cubic meters of D. hexagon A. - 1000
C. 1.477 each strength must be used in order to B. -5000
D. 1.747 produce a mixture of 50 cubic meters that 49. A tax on imports: c. -2000
contain 40% alcohol? A. import tariffs D. -3000
37 The moment of inertia of a section 2 A. 25, 25 B. value added tax
inches wide and 2 feet high about an axis B. 30,20 C. import quote 55. A car travels from 20 to 50 kph in 2
1 foot above the bottom edge of the C. 23 2/3, 26 1/3 D. capital account sec? At the same acceleration, how long
4 D. 33 1/3, 16 2/3
section, in inches , is: will it take the car to go from 20 to 45
A. 1834 50. A line that meets a plane but is not kph?
B. 9214 43. If a train passes as many telegraph perpendicular to it, in relation to the A. 1.66 sec
c. 384 poles in one minute as it goes miles per plane: B. 1.88 sec
D. 2304 hour, how far apart are the poles? A. parallel C. 2.22 sec
A. 18 feet B. coplanar D. 3.55 sec
38. A beam supported at both ends and B. 88 feet C. collinear 2
carrying a uniformly distributed load: C. 38 feet D. oblique 56. A car has an acceleration of 5 m/s .
A. has its maximum bending moment at D. 96 feet How far does it go to reach a velocity of
the support 51. An airplane whose velocity relative to 30 m/s if it starts from at rest?
B. has its maximum shear at the center of 44. The ratio of the annual revenues to the air is at constant 900 kph has a A. 60m
the beam the annual expenses: constant tailwind of 200 kph. How long B. 90m
C. has its maximum shear at the supports A. benefit-to-cost ratio will it take an airplane to cover 3000 km C. 130m
D. has uniform shear throughout the B. rate of return relative to the ground? D. 240m
length _of the beam C. income ratio A. 2. 72 hours
D. benefit ratio B. 3.52 hours 57. The brakes of a certain truck can
39. What is the form of the moment C. 2 .15 hours produce an acceleration of 5 m/s2 . How
diagram of a beam supported at both 45. An instrument for measuring high D. 4.5 hours far does the truck travel during the time
ends and carrying a uniformly distributed temperature gases: the brakes applied from a velocity of 40
load? A. atmometer 52. A truck travels at 11 0 kph for 2 hours m/s?
A . hyperbola B. odometer at 70 kph for the next 2 hours, and finally A. 220m
B. rectangle C. altimeter at 90 kph for 1 hour. What is the car's B. 120m
C. parabola D. pyrometer average velocity for the entire journey?
D. triangle
REFRESHER MANUAL 2nd Edition by JAS TORDILLO
REFRESHER MANUAL 2"d Edition by JAS TORDILLO
6-6 DAY 6 - SOLUTION
DAY 6 - SOLUTION 16-7
ENGINEERtNG MATHEMATHICS .
C. 90m DAY 6 - SOLUTION
D. 50 m 7. The decrease in value of a property 10. Mr. Suave borrowed P50,000 from
1. The amount of money paid for the use due to the gradual extraction of its SSS, in the form of calamity loan, with
58. A truck starts from rest with an contents. interest at 8%, compounded quarterly,
acceleration of 5 m/s2 . W hat is its velocity of borrowed capital or income produced ·
A. Depreciation payable in equal quarterly installments for
by money which has been loaned.
after it has gone 500 p1? B. appreciation 10 years. Find the quarterly payments.
A. 50.7 m/s A. interest*
C. property loss A : P4,827.80
B. income
B. 70.7 m/s D. depletion * B. P2,827.80
\ ···l . ',.,. C . percentage
C . 90 m/s C. P3,827.80
D. release amount
D. 120 m/s 8. A PLOT tower and a monument stand D. P1,827.80 *
2. The quantity of a certain commodity on a level plane. The angles of
59. An airplane must have a velocity of depression of the top and bottom of the n ::; 10 years = 40 quarters
that is offered for sale at a given price at
70 m/s in order to take off. W hat must the
airplane's acceleration be if it is to take
off from a runway 700 meters long in
a given place and time.
A. supply •
monument viewed from the top of the
PLOT tower are 13 deg and 35, i =.-;-
. 80!.
=,2% per quarter
B. goods respectively. The height of the tower is 50
m/sec2 ? meters. Find the height of the monument.
A. 2.5 C. demand
A. 16.5 m
P = R[{l+i)n-1]
B. 3.5
D. bond
B. 71 .5 m i(l +ir
C . 2.0 c 33.5 m *
D. '5.0
3. On a negotiable paper is the difference
between the present worth and the worth D. 95.5 m 50 000
0
=R [ (1 + 0.02 )40-1 ]
40
of the paper at some time in the future. 0.02(1 + 0.02)
60. The brakes of a truck whose initial Y=50 - H
A. differential amount R = 1,827.78
velocity is 40 m/s are applied, and the
2
truck receives an acceleration of -3 m/s . B. discount *
C. allowance Tan 3 5o = 50 11 . Alexander Michael owes P25,000 due
How far will it have gone when its velocity
D. appraisal X in one year and P75,000 due in four
has decreased to 10 m/s? x = 71.4 m years. He agrees to pay P50,000 today
A. 100m and the balance in 2 years. How much
4. The analysis and evaluation of the
B. 189m must he pay at the end of two years if
factors that will affect the economic
C. 250m Tan 13° ::; Y...
success of engineeri ng projects to the X money is worth 5% compounded semi-
D 290m annually?
end that recommendation can be made
Tan 13° = SO- H A. another P50,000
which will insure the best use of capital. 71.4
A. economic analysis B. only P39,02f;.30 *
50- H::; 16.48

ra DAY 6- ANSWER KEY


l.A 16. 0 31. 0 46. C
) B. break-even analysis
C. capital eval uation
D. engineering economy*
H = 33.51 m

9. A mechanical engineer was granted a


C. only P25,000 ·
D. only P30,000.00

i = 5% compounded SA
2. A 17. 0 32. 0 47.0 loan of P20,000 by her employer with an
5. The process of determining the value = 5.0625% per annum
3. 6 18. B 33. A 48. C interest at 6% for 180 days on the
4. 0 19. 0 34. 0 49. A of certain property for specific reasons.
principal collected in advance. If
5. c 20. 6 35. A SO. 0 A. depletion 50000+ _ _ A
discounted at once, find the proceeds on
6.A 21.0 36. C 51. A B. determination ' (1 + 0.0506)2
37. 0 52.0 the note.
7.0 22. 0 C. appraisal *
8. c 23. 6 38. C 53. A A. P20,000 = 25,000 + _7_5,:.. _00:. . :. 0
D. estimation
9.C 24. c 39. C 54.0 B. P14,000
10. 0 25. 0 40. 6 SS.A C. P18,800 * (1 + 0.0506j (1 + 0.0505)4
41. 8 56. B
6. Funds supplied and used by owners of 50,000 + 0.906A
11. 8 26. 6 D. P1,300
12.8 27. c 42. 0 57. A an enterprise in the expectation that a = 23,795.93 + 61,561 .85
13. A 28. A 43. 8 58. 8 profit will be earned. A = P39,026.24
Interest = 20,000(0.06) = P1,200
14. C 29. A 44. A 59. B A . equity capital * Proceeds = 20,000-1 ,200::; P18,800
(Jjl 15. 8 30. 0 45. 0 60. C B. credit 12. In problem No. 11, what is the
C. save effective rater per year? ·
D. investment
REFRESHER MANUAL 2nd Edition by JAS TORDILLO
REFRESHER MANUAL 2"d Edition by JAS TORDILLO
6- s DAY 6 - SOLUn:ON
DAY 6 - SOLUTION 6- 9
ENGINEERING MATHEMATHICS
I
A. 10% 15. An Aerolift airplane can fly at an air
B. 5.06% * speed of 300 mph. If there is a wind 20. What is the maximum profit of
Problems 18 to 21
c. 5% blowing toward the East at 50 mph, what A certain company manufacture two product x per day?
D. 3.06% should be the plane's compass reading in A. P200
products, x and y, and each of these
order for its course to be 30 deg? products must be processed on two B. P1000"
;:n
.1= ( 1+ in)m -I = ( 1+ 0.05)2
2 - 1 = s.o62s% A. 8.3 deg different machines. Product x requires 1 C. P500
B. 21.7 deg * minute of work time per unit on machine D. P2000
C. 30 deg 1 and 4 minutes of work time on machine
13. A light rail transit ti"avel.s between two D. 38.3 deg Profit= 20(50) = P1000
2. Product y requires two minutes of work
terminals 1 km apart in a minimum time
time per unit on machine 1 and 3 minutes
of 1 minute. If the LRT cart accelerates e
Let = compass reading of work time per unit on machine 2. Each 21. What is the maximum profit of
and decelerates at 3.4 m per sec2 , 50 300 product y per day?
day, 100 minutes are available on
starting from rest at the first terminal and sinl3 = sin60° A. P100
machine 1 and 200 minutes are available
coming to stop at the second terminal,
find the maximum speed.
13 = 8.30 on machine 2. To satisfy certain s: P15dO
customers, the company must produce at C. P800
A. 66 kph *
e + 13 = 30° least 6 units per day of product x and at D. P2400 *
B. 60 kph
C. 55 kph
e = 30°-13 = 30°- 8.3°= 21.7° least 12 units of product y. If the profit of
each unit of product x is P50.00 and the Profit= 60(40) = P2400
D. 20 kph
16. In problem No. 15, what will be the profit of each unit of product y is P60.00.
plane's ground speed if its flies this 22. A statement of equality between two
Vz = maximum speed ratios:
2 course? 18. How many units of product x should
a= 3.4 m/s A. valuation
A. 188 mph be produced daily in order to maximize
a= v2 -VI B. 311 mph the company profits? B. power factor
t C. theorem
C. 300 mph A. 6 units
3.4 = v2 -o D. 322 mph* B. 20 units* D. proportion *
t C. 10 units
v2 = 3.4t Eq. 1 Let G = plane's ground speed D. 40 units 23. The side opposite the right angle of a
S = V1t + }S af ~ = 180-8.3- 60 = 111.7° right triangle:
s = 0 + }S (3.4)f 1X+ 2y = 100 Eq. 1 A. quadrilateral
S=1 .7tz Eq.2 4x + 3y = 200 Eq. 2 B. hypotenuse "
G 2 = 502 + 3002 - 2(50)(300) cos 111.7°
C. apothem
G = 321.86 mph D. median
Distance = speed x time· by elimination:
1000- 2s = v2 (60- 2t) 17. A rubber ball was dropped from a 4x + 8y = 400
1000 = 2(1 .7f) = 3.4t(60-2t) -4x - 3y = -200 24. A plane closed curve, all points of
height of 36 m, and each time it strikes which are the same distance from a point
1000 = 3.4t2 = 204t- 6.8t2 the ground it rebounds to a height of 2/3 5y = 200
3.4f- 204t + 1000 = 0 y = 40 units within, called the center is:
of the distance from which it fell. Find the
t = 5.38 sec A. chord
total distance traveled by the ball before if
X+ 2y = 100 B. arc
comes to rest.
Maximum Speed, V2 = 3.4(5.38) X= 100- 2(40) C. circle*
A. 72 m
= 18.292 m/sec = 65.85 kph x = 20 units D. radius
B. 150m
C. 120m 25. Two triangles are congruent if two
14. In problem No. 13, how long does it D.180 m * 19. How many units of product y should
travel at this top speed? be produced daily in order to maximize angles and the included side of one are
A. 60 sec the company profits? equal respectively to two angles and the
r = 2/3 , a = 2(36) = 72 included side of the other.
B. 39 sec A . 10 units
C. 49 sec* S:;: _a__ 36 B. 20 units A. axiom
D. 30 sec 1- r C. 12 units B. corollary
72 D. 40 units* C. postulate
60- 2t = 60- 2(5.38) S= - - 3 6 D. theorem*
2
= 49.24 sec 1- -
3
S =180m REFRESHER MANUAL 2nd Edition by JAS TORDIL~O
REFRESHER MANUAL 2nd Edition by JAS TORDillO
6- 10 DAY 6 - SOLUTION
DAY 6 - SOLUTION 6-11
ENGINEERING MATHEMATHICS
I
26. The volume of a circu lar cylinder is 33. It occurs when a unique product or
equal to the product of its base and service is available only from a single A. 1834 strengths, 30% alcohol and 60% alcohol,
altitude: supplier and entry of all other possible B. 9214 respectively. How many cubic meters of
A. postulate suppliers prevented: C.384 each strength must be used in order to
B. theorem • A. monopoly * D. 2304 * produce a mixture of 50 cubic meters that
C. corollary B. profitability contain 40% alcohol?
D. axiom C. competition bh3
1=- A. 25,25
D. inventory 12 B. 30,20
27 The dimension of Acceleration· x 24 c. 23 2/3, 26 1/3
Mass is the same unit as that of: 34. The pressure reading of 35 psi in KPa 1 = 2( }3 =2304 in4 D. 331/3, 16 2/3 "
12
A. length is:
B. work A. 427.3 x+y=50
B. 273.4 38. A beam supported at both ends and
C. weight* y = 50-x
C. 724 carrying a uniformly distributed load:
D. section modulus
D. 342.72 * . A. has its maximum bending moment at
0.30x + 0.60y = 0.40(50)
the support ·
28. The coefficient of friction for dry 101.325)
Pressure= 35 ( - - + 101.325 B. has its maximum shear at the center of
=
0.30x + 0.60(50 - x) 0.40(50)
surfaces: . 14.7 0.30x + 30- 0.60x = 20
A. depends orr the materials and the the beam'
Pressure = 342.57 kPa abs C. ·has its maximum shear at the 0.30x = 10
finish condition of the surface * x = 33.33 m 3
supports* 3
B. depends only on the finish condition of 35. What is the area of a triangle with y =50- 33.33 = 16.67 m
the surfaces
D. has uniform shear throughout the
sides of 5, 7 and 10? length of the beam
C. does not depend on the materials 43. If a train passes as many telegraph
A. 16.248 *
D. depends on the composition of the poles in one minute as it goes miles per
B. 25.248 39. What is the form of the moment
materials on ly C. 18.248 hour, how far apart are the poles?
diagram of a beam supported at both
D. 30.248 ends and carrying a uniformly distributed
A. 18 feet
29. Output over input is: B. 88 feet*
A. efficiency* load?
C. 38 feet
s= 5+7+10= 11 A. hyperbola
B. bond D. 96 feet
2 B. rectangle
C. rate of return
D. annuity Area = Jrs('s---aX"'s---b"X-s--c" ) C. parabola *
D. triangle
=
Let x distance between poles
n = number of poles passed by train in 1
30. An instrument for measuring specific Area = Jn(ll- sXn- 7XII -I o) min
40. The maximum moment of a beam
gravity of fluids: Area= 16.248 = no. of miles traveled per hour
supported at both ends and carrying a
A. flowmeter
36. The radius of a circle inscribed in a uniformly distributed load is located at szson = no. of feet per min traveled
B. clinometer 60
C. hygrometer triangle with sides of 5, 7 and 10 is: A. either of the two ends
B. center * time= 1 min
D. hydrometer* A. 1.774
B. 1.744 C. anywhere in the span
D. at the left end Distance = rate x time
31. The decrease in value of property .c. 1.477.
due to passage of time: D. 1.747 nx = 5280n (l)
41. A cantilever beam having a uniformly 60
A. devaluation
increasing load toward the fixed end: x = 88 feet
B. salvage value s = 11
A. has uniform shear
C. depletion At!.
R= - B. has a reaction equal to the load * 44. The ratio of the annual revenues to
D. depreciation * s C. will have maximum bending moment the annual expenses:
16.248 = 1.477 midway to the beam A. benefit-to-cost ratio *
32. The difference between the book R= -,,
D. has a reaction equal to one half of the B. rate of return
value and the actual lower resale value
load C. income ratio
is: 37. The moment of inertia of a section 2
A. salvage value D. benefit ratio
inches wide and 2 feet high about an axis 42. A chemist of a distillery experimented
B. fixed cost 1 foot above the bottom edge of the on two alcohol solutions of different .
C. resale value section, in inches4 , is:
D. sunk cost*, REFR~SHER MANUAL 2nd Edition by JAS TORDILLO
REFRESHER MANUAL2"d Edition by JAS TORDILLO
DAY 6 - SOLUTION 16 - 13
6 - 12 DAY 6 - SOLUTION ENGINEERING MATHEMATHICS
45. An instrument for measuring high I Solving the ground speed of the plane: will it take the car to go from 20 to 45
temperature gases: v = 900 + 200 = 11 00 kph
A. atmometer
kph?
A. 1.66 sec *
v = -has= J2(s) (500) = 70.71 m/s
t = ~=
B. odometer 3000
= 2.72 hours B. 1.88 sec
C. alttmeter v 1100 C. 2.22 sec _59. An airplane must have a velocity of
to'
D. pyrometer" D. 3.55 sec 70 m/s in order to take off. What must the
52. A truck travels at 110 kph for 2 hours airplane's acceleration be if it is to take
46. The digits of a m'eas~ment which a at 70 kph for the next 2 hours, and finally off from a runway 700 meters long in
scientist reads and estima-te on a scale at 90 kph for 1 hour. W hat is the car's v - v o - 50 - 20 = 15 kph/s m/sec2?
a = - t- - 2
are called: average velocity for the entire journey? A 2.5
A. accuracy A. 40 kph v- v0 _ 45- 20 = 1.66sec B. 3.5*
t = -a- - 15 c .. 2.0
B. precision B. 60 kph
C. significant figures* C. 80 kph D.• 5.0
D. experimentation o . 90 kph* 56. A car has an acceleration of 5 m/s2.

47. Galvanized iron is a term referring to


.
y = S1+S2 +S3 = 11 0(2)+70(2) + 90(1)
How far does it go to reach a velocity of
30 m/s if lt starts from at rest?
a = V 2 _ (70)2
2S - 2(700) = 3.5 m/s2
iron coated with: It +t2 +t3 2 + 2+ 1 A. 60 m
A. aluminum B. 90 m*
450 60. The brakes of a truck whose initial
B. magnesium v = - = 90 kph C. 130m
velocity is 40 m/s are applied, and the
C. tin 5 D. 240m
D. zinc* truck receives an acceleration of -3 m/s2 .
53. A motorcycle starts from rest and How far will it have gone when its velocity
48. The sum of the sides of a polygon: reaches a velocity of 50 m/s in 10 sec. If t= ::: = 30 = 6 s has decreased to 10 m/s?
its acceleration remains the same, what
a 5 A. 100m
A. circumference S = Y:. af = Y:. (5)(6)2 = 90 m
B. square will its velocity be 8 sec later? B. 189m
C. perimeter* A. 90 m/s* C. 250m *
57. The brakes of a certain truck can D. 290m
B. 100 m/s
D. hexagon produce an acceleration of 5 m/s2. How
C. 180 ,m/s
far does the truck travel during the time y 2 = V 0 2 + 2aS
49. A tax on imports: D. 220 m/s
the brakes applied from a velocity of 40
A. import tariffs " y2 - Vo2
B. value added tax v 50 2
m/s?
A. 220m*
s = = (10)2 .-(40)2
- =250m
a = - =- = 5 m/s
C import quote t 10 B. 120m
D. capital accou nt
C . 90m
V =Yo + at = 50+ 5(8) ·= 90 m/s D. 50m
50 A line that meets a plane but is not
perpendicular to it, in relation to the 54. A ball Is moving at 20 m/s w hen iUs
= v
plane:
A . parallel
struck by a bat and moves off in the
opposite direction at 40 m/s. If the impact
t
a=s
40
= 8s
B. coplanar lasted 0.020 sec, find the ball's
2 S = Vat + Y:. at2 = 40(8) - Y:. (5)(8)2
C. collinear acceleration during the impact in m/s .
D. oblique* = 220m
A. -1000
B. -5000
51 An airplane whose velocity relative to 58. A truck starts from rest with an
c. -2000 acceleration of 5 m/s2 .'What is its velocity
the air is at constant 900 kph has a
D. - 3000 *
constant tailwind of 200 kph. How long after it has gone 500 m?
will it take an airplane to cover 3000 km A. 50.7 m/s
relative to the ground? _ v-v 0 -~= -60 =-3000m/s2 B. 70.7 m/s*
a - - t- - 0.02 0.02
A. 2.72 hours* C. 90 m/s
B. 3.52 hours D. 120 m/s
55. A car travels from 20 to 50 kph in 2
C. 2.15 hours
sec? At the same acceleration, how long REFRESHER MANUAL 2nd Edition by JAS TORDILLO
D. 4.5 hours

REFRESHER MANUAL 2nd Edition by JAS TORDILLO


DAY 7- EXAM ~7-1
ENGINEERING MATHEMATHICS

having already been fenced by the


DAY' 7- EXAM adjoining. One of the fenced sides is 40
m long the other 15 m longer than the
1. A boy tied an 80-gram stone to a string side to be fenced. If the total length of the
l which he rotated to from a circular motion three sides is 125 m, the height of the
with a diameter of 1000 mm. Compute for fence required is 2436 mm, and the
.·"' ·...... ~ the pull exerted on the string by the stone height and length of each hollow block
if it got loose leaving at a velocity of 25 are 203 mm and 406 mm, respectively.
m/s.
A. 50 N 4. Determine the length of side to be
B. 100 N fenced by hollow blocks.
C. 80 N A. 35m
D. 250 N B. 15m
C. 40m
2. A plane drppped a bomb at an D. 55m
elevation of 1000 m from the ground
intended to hit a target which elevation is 5. Find the area of one hollow block in sq.
200 m from the ground. If the plane was meter.
flying at a velocity of 300 kph, at what A. 0.082418
distance from the target must the bomb B. 1.24561
be dropped to hit the target? Wind C. 0.189234
velocity and atmospheric pressure to be D. 2.92560
disregarded.
A. 1065m 6. Find the area of the side to be fenced
B. 1650 m in sq: meter.
C. 1560 m A. 95.26
D. 1605 m B. 75.26
C. 85.26
3. A man standing on a 48.5 m building D. 65.26
high, has an eyesight height of 1.5 m
from the top of the building, took a 7. Find the number of hollow blocks that
depression reading from the top of have to be purchased, allowing an
another nearby building and nearest wall, additional 10% for possible breakages.
which are 50 deg and 80 deg Disregard effects of post on the required
respectively. Find the height of another number of hollow blocks.
building in meters. The man is standing A. 758
at the edge of the building and both B. 1055
buildings lie on the same horizontal C. 995
plane. D. 1138
A. 59.5 m
B. 49.5 m 8. For a loan acquired six years ago, a
C. 69.5 m man paid out the amount of P75,000. The
D. 39.5 m interest was computed at 18%
compounded annually. How much was
Problems 4 to 7 the borrowed amount?
A mason was asked to compute the A. P24,782.36
number of hollow blocks which have to B. P26,782.36
be purchased to fence one side of a c. P25,782.36
triangle lot which the two other sides D. P27,782.36

REFRESHER MANUAL 2nd Edition by JAS TORDILLO


DAY 7 - EXAM 17 -3
7-2\ DAY 7- EXAM ENGINEERING MATHEMATHICS
9. A man purchased a .car with a cash C. 22 days
price of P350,000. He was able to D. 30 days 18. Compute the annual deprecation of How much will the company deposit each
negotiate with the seller to allow him to Offer B. year if interest to be earned is computed
pay only a down payment of 20% and the 14. A flywheel of a puncher is to be A. P44,944 at 15%?
balance payable in equal to 48 end of the brought to a complete stop in 8 seconds B. P64,944 A. P278,900
month installment at' 1.5% interest per from a speed of 60 revolutions per C. P54,944 B. P156,009
month. How much is the monthly minute. Compute the angular D. P74,944 C. P197,009
payment? , , acceleration of the flywheel in rev per D. P125,909
A. P2,225.00 ' sec2. 19. Compute the annual cost of Offer B.
B. P8,225.00 A. -0.125 A. P275,744 . 24. For having been loyal, trustworthy
C. P4,225.00 B. 0.125 B. P175,744 and efficient, the company has offered a
D. P9,225.00 C. -0.512 C. P215,744 supervisor a yearly gratuity pay of
D. 0.512 D. P144,744 P20,000 for 10 years with the first
10. In problem No. 9, on the day he paid payment to be made one year after his
the 201h installment. he decided to pay the 15. In prpblem No. 14, compute for the 20. Which offer will you recommend to be retirement. The supervisor, instead,
remaining balance. What is the remainin~ number of turns the flywheel will still purcj1ased? requested that he be paid a lump sum on
balance that he paid after paying the 20 make before coming to stop. A.. Offer A the date of his retirement less interest
installment? · A. 2 turns B. either Offer A or Offer B that the company would have earned if
A. P270,000.00 B. 4 turns C. Offer B the gratuity is to be paid on yearly basis.
B. P156,297.25 C. 3 turns D. none of the two offers If interest is 15%, what is the equivalent
C. P186,927.25 D. 9 turns lump sum that he could get?
D. P116,927.25 21 . A marketing company established a A. P100,375.37
Problems 16 to 20 program to replace the cars of its sales B. P137,375.37
11 . A mechanical engineer who was For its proposed expansion, and ice plant representatives at the end of every 5 C. P107,375.37
awarded a P450,000 contract to install company is selecting form two offers of years. If the present price of the D. P237,375.37
the machineries of an oil mill failed to ice cans. The data on the offers are as approved type of car is P520,000 with a
finish the work on time. As provided for in follows: For Offer A, 1.8 mm thick, total resale value at the end of 5 years of 40% 25. From an oblique triangle in which side
the contract, he has to pay a daily penalty cost is P720,000, annual maintenance is of its present value. How much is the a = 6 em, side b = 9 em and angle C = 32
equivalent to one-fourth of one percent of P60,000 and a life of 12 years. For Offer resale value at the end of 5 years? degree. Solve for angle A.
the contract price for the first ten days of B, 1.6 mm thick, total cost is P640,000, A. P500,000 A. 19.1 deg
the delay, one-half percent per day for annual maintenance is P90,000 and a B. P240,000 B. 39.1 deg
the next ten days and one percent per title of 8 years. For their replacements, C. P420,000 C. 29.1 deg
day for everyday thereafter. Determine the company is putting up a sinking fund D. P208,000 D. 109 deg
the penalty .for the first ten days. to earn 16% . interest compounded
A. P11 ,250.00 annually. If the money to purchase the 22. In problem No. 21 , how much money 26. In problem No. 25, solve for angle B.
B. P31,250.00 ice cans is to be borrowed at 20% annual must the company accumulate for 5 A. 39 deg
C. P21 ,250.00 interest and the tax on the first cost is yea rs if inflation annually is 10%. Resale B. 108.9 deg
D. P41 ,250.00 2%. value will also appreciate at 10% yearly. C. 75 deg
A. P520,000 D. 180 deg
12. In problem No. 11 , determine the 16. Compute the annual depreciation of B. P502,480
penalty for the next ten days. Offer A. C. P512,560 27. The Philippine Long Distance
A. P11,500 A. P12,339 D. P310,000 Telephone Company has 183 m ·straight
B. P33,500 B. P33,339 vertical tower. There are two cables in
C. P22,500 C. P23,339 23. A fabrication company purchased an the same vertical plane anchored at two
D. P44,500 D. P43,339 adjacent lot for its expansion program points on a level with the base of the
where it hopes to put up a building tower. The angles made by the cables
13. In problem No. 11, determine how 17. Compute the annual cost of Offer A. projected to cost P4M when it will be with the horizontal are 44 deg and 58 deg
many days was the completion of the A. P190,739 constructed 10 years after. To provide for respectively. Find the lengths of the
contract delayed if the total penalty paid B. P318,739 the required capital expenses, it plans to cables.
was P60,750? C. P241,739 put up a sinking fund for the purpose. A. 236m and 251m
A. 16 days D. P369,739
REFRESHER MANUAL 2nd Edition by JAS TORDILLO
B. 26 days
REFRESHER MANUAL 2"d Edition by JAS TORDILLO
7-41 DAY 7- EXAM DAY 7- EXAM ~7 - 5
B. 263 m'and 215m decelerate at a constant rate of 4 m per
ENGINEERING MATHEMATHICS
C. 245 m and 263 m sec per. In what time will Bert overtake
D. 289m and 315m loti? Treasurer and an Auditor, if no member C. P1000
A. 6.164 sec can hold more than one position? D. P50
28. In problem No. 27, if the cables are B. 10. 164 sec A. 160,160ways
on the same side of, the tower, find the C.164 sec B. 360,360 ways 43. The time required for an elevator to
distance between t):leir anchor points. C. 260,260 ways lift a weight varies directly with the weight
D. 12.164 sec
A. 60m D. 460,460 ways and the distance through which it is to be
.....
-

B. 300m 34. In problem No. 33, find the distance lifted and inversely as the power if the
C. 75m traveled by Bert in overtaking loti? 39. Chemist of a distillery experimented motor. If it takes 30 seconds for a 10 HP
D. 450 m A. 110m on two alcohol solutions of different motor to lift 100 lbs through 50 ft, what is
B. 185m strength 35% alcohol and 50% alcohol, the size of motor is required to lift 800 lbs
29. In problem No. 27, if the cables are C. 150m respectively. How many cubic meters of in 40 seconds through 40 ft?
on the opposite sides of the tower, find D. 225m each strength must he use in order to A. 20 HP
the distance between their anchor points. produce a mixture of 60 cubic meters that B. 48 HP
A. 75 m 35 .- Ed ,who was then 26 years old contain 40% alcohol? C. 30 HP
B. 250m planned to retire at the age of 56 years A. 10, 50 D. 68 HP
C. 90m and so he purchased an annuity. He 8. 40,20
D. 304m agreed to pay equal annual payments, C. 20, 30 44. Any method of repaying a debt, the
the payment was made on his 26th D.30, 30 principal and interest included, usually by
30. Using a powerful air gun, a steel ball birthday and the last was made on his 55 a series of equal payments at equal
is shot vertically upward with a velocity of birthday. At present, Ed is receiving a 40. The initial cost of a paint sand mill, intervals of time.
80 m/s, followed by another shot after 5 yearly income of p120,000 per which he including its installation, is P800,000. The A. annuity
seconds. The second ball met the first started receiving on his 56th birthday. BIR approved life of this machine is 10 B. amortization
ball 150 meters from the ground. Find the Ccmpute for the annual amount that he years for depreciation. The estimated C. perpetuity
maximum height reached by the first ball. paid for the annuity if interest rate is 10%. salvage value of the mill is P50,000, and D. compou!"'d
A. 150m A. P2,295.10 the cost of dismantling is estimated to be
B. 326.2 m B. P6,295.10 P15,000. Using straight line depreciation, 45. Funds supplied by others on which a
C. 240m C. P4,295.1 0 what is the annual depreciation charge? fixed rate of interest must be paid and
D. 362.2 m D. P7 ,295. 10 A. P75,000 the debt be repaid at a specific time.
B. P85,000 A. borrowed capital
31 . In problem No. 30, solve for the time 36. A unit welding machine cost P45,000 C. P76,500 B. first cost
when the first ball reached its maximum with an estimated life of 5 years. Its D. P90,000 C. discount
height. salvage value is P2500. Find its D. bond
A. 5.994 sec depreciation rate using straight 41. In problem No. 40, what is the book
B. 9.149 sec depreciation. value of the machine at the end of six 46. The length of time during which the
C. 8.155 sec A. 18.9% .- years?
A. P400,000
property may be operated at a profit.
A . economic life
D. 9.941 sec B. 12.1%
C. 16.7% B. P241,000 B. physical life
32. In problem No. 30, find the initial D. 10.2% C. P341,000 C. bank life
velocity of the second ball. D. P200,000 D. life time
A. 31 .27 m/s 37. In problem no. 36, assuming that you
B. 91 .27 m/s will deposit the money to a bank giving 42. A provincial chapter of PSME held a 47. The length of time during which it is
C. 61 .27 m/s 8.5%, solve for the deposit. lottery to raise funds for their capable of performing the function for
D. 99.27 m/s A. P7,172.54 organization, with P10,000 top prize and which it was designed and manufactured.
B. P8,500.00 with 2,000 tickets printed and sold. What A. economic life
33. Two cars left the building for Bataan C. P5,172.54 is the mathematical expectation of a B. pro life -
to attend the meeting. Bert in a Toyota D. P9,500.00 member if he bought 10 tickets? C. physical life
and loti in a Mercedez Benz have a A. P2000 D. no. of periods
velocity of 30 m/s both in the same Bert 138. How many ways can a PSME B. P100
is 76 meters behind l oti when the brakes Chapter with 15 directors choose a
was applied to the Benz. causing it to President, a V-President, .a Secretary, a REFRESHER MANUAL 2nd Edition by JAS TORDILLO
REFRESHER MANUAL 2 nd Edition by JAS TORDILLO
"~r ··

1-6 1 DAY 7- EXAM DAY 7 - SOLUTION 17- 7


48. The value of the variable for which 54. A car towing a trailer is accelerating ENGINEERING MATHEMATHICS
on a level road. The car exerts a force on
the costs for the alternative will be equal.
the trailer whose magnitude is kg to a height of 100 m. If the efficiency
A . break-even analysis
A. the same as that of the force the trailer of the hoist is 80 percent, find the time DAY 7 - SOLUTION
B. break-even graph needed .
C. break-even point exerts on the car
B. the same as that of the force the trailer A 57.1·sec 1. A boy tied an 80-gram stone to a string
D. break- even business t B. 78.2 sec which he rotated to from a circular motion
exerts on the road
C. 80.4 sec with a diameter of 1000 mm. Compute for
49. Products that ar~ -~rectly used by C. the same as that of the force the road the pull exerted on the string by the stone
exerts on the road D. 90 sec
people to satisfy their wants. • if it got loose leaving at a velocity of 25
A. producers goods D. greater than that of the force the trailer
60. A horse has a power output of 1. 1 kW m/s.
B. consumer goods exerts on the car
when it pulls a car with a force of 500 N. A. 50 N
C. supply What is the car's velocity? 8 .100 N *
55. In Newton's third law of motion the
D. demand A 2.2 m/s C. 80 N
action and reaction forces
B. 4.5 m/s D. 250 N
50. The sum of the first cost and the A. act on the same object
B. act on different objects C. 3.5 m/s
present worth of all cost of any property r = D/2 = 1000/2 = 500 mm = 0.500 m
C. do not necessarily have the same D. 5.5 m/s
of replacement, operation and
maintenance for a long time or forever. magnitude and do not necessarily have
Force or pull exerted by the string
A. annual cost the same line of action
B total cost
D. have the same magnitude but do not = mv2 = (o.osXzs)Z
C. capitalized cost have necessarily have the same line of
action
~'-1=========;..,--' = 100 N
r 0.5
D. expenses DAY 7- ANSWER KEY
1. 8 16. c 31. C 46. A
51. Relative to the force needed to keep 56. A jumper of weight w presses down 2. A 17. c 32. C 47.C 2. A plane dropped a bomb at an
a box moving at constant velocity across on the floor with a force of magnitude F, 3. D 18. A 33.A 48.C elevation of 1000 m from the ground
a floor, to start the box movjng usually and the jumper. leaves the floor as a 4.A 19. A 34.6 49. 6 intended to hit a target which elevation is
result. The force the floor exerted on the 5. A 20. A 35. D 50. C 200 m from the ground. If the plane was
needs 6. C 21.0 36. A 51. C
A less force jumper must have had a magnitude flying at a velocity of 300 kph, at what
7. 0 22.6 37. A 52. B
A. equal to wand less than F distance from the target must the bomb
B. the same force 8.0 23. c 38. B 53. D
C. more force B. equal to wand equal to F 9. 6 24. A 39. 6 54. A be dropped to hit the target? Wind
D. any of the above, depending on the C. more than wand equal to F 10. C 25. 6 40.C 55. 6 velocity and atmospheric pressure to be
natures of the surface in contact D. more than wand more than F 11. A 26. 6 41.c 56.c disregarded.
12. c 27. 6 42. D 57. A A. 1065m *
13. 6 28. c
52. When two surfaces are in contact, the 57. A 3000-lb car ascends a 12° slope at 43. B 58. 0 B. 1650 m
14. A 29. 0 44. 6 59. A
frictional force between them depends on 30 mph. If the overall efficiency is 80 C. 1560 m
which one or more of the following? percent, what is the power output of the ~I 15. B 30. B 45. A 60. A
D. 1605 m
A. the normal force pressing one surface car's engine?
agamst the other A. 62.3 hp y = -(1000 - 200) m =-BOOm
B. all of these B. 67.7 hp = =
Vo 300 kph 83.33 m/s
C. whether the surfaces are stationary or C. 77.7 hp e = 0°
in relative motion D. 87.7 hp 2
gx
D . whether a lubncant is used or not
58. What velocity does a 10-kg object y = X tan e0 - - -cos 2 6
y2
2
0 2
53. Compared with her mass and weight have when its kinetic energy is 10 J?
9.8lx
on Earth, an astronaut on Venus, where A. 4.4 m/s
-BOO= x tan e o- ( . 3) cos 2 0 o
2 83 3 2
2
the acceleration of gravity is 8.8 m/s , has B. 2.4 m/s
A. less mass and less weight C. 3.4 m/s
x = 1064.2 m
B. less mass and the same weight D. 1.4m/s
C. less mass and more weight 3. A man standing on a 48.5 m building
D. the same mass and less weight 59. A hoist powered by a 15-kW motor is
high, has an eyesight height of 1.5 m
used to raise an elevator loaded with
. workers and having a total mass of 700
REFRESHER·MANUAL 2nd Edition by JAS TORDILLO
REFRESHER MANUAL 2"d Edition by JAS TORDILLO I
1- s I DAY 7 - SOLUTION DAY 7 - SOLUTION 17- 9
from the top of the building, took a 5. Find the area of one hollow block in sq.
ENGINEERING MATHEMATHICS
depression reading from the top of meter.
balance payable in equal to 48 end of the 12. In problem No. 11, determine the
another nearby building and nearest wall, A. 0.082418 *
month installment at 1.5% interest per penalty for the next ten days.
which are 50 deg and 80 deg B. 1.24561
month. How much is the monthly A. P11,500
. respectively. Find the height of another c. 0.189234
payment? B. P33,500
building in meters. The man is standing D. 2.92560
A. P2,225.00 C. P22,500 *
at the edge of th~ building and both
B. P8,225.00 * D. P44,500
buildings lie on fh~ same horizontal Area= 0.203(0.406) = 0.082418 m
C. P4,225.00
plane. '~
D. P9,225.00 Penalty (11 - 20 days)
A. 59.5 m 6. Find the area of the side to be fenced
B. 49.5 m
C. 69.5 m
in sq. meter.
A. 95.26
p= R[(I + iP - I] = 0.005 X P450,000 X 10
= P22,500
i(1+if
D. 39.5 m * B. 75.26
13. In problem No. 11, determine how
c. 85.26 * 48
h = 48.5 + 1.5 =50 m D. 65.26 350,000(0.80) = R [ (I+ 0 · 015) -I ] many days was the completion of the
y =50- H 0.015(1 + 0.015)4 8 contract delayed if the total penalty paid
was P60,750?
=
A 35 m x 2.436 m 85.26 m2 = R =S,225.00
A. 16 days
0 50 B. 26 days*
Tan 80 - - 7. Find the number of hollow blocks that 10. In ~roblem No. 9, on the day he paid
X 1 C. 22 days
x = 8.816 m have to be purchased, allowing an the 20 installment, he decided to pay the
D. 30 days
additional 10% for possible breakages. remaining balance. What is the remain i n~
Tan 50° = "j_ Disregard effects of post on the required balance that he paid after paying the 201
X Let D = no. of days delayed after 20 days
number of hollow blocks. installment?
0 _ 50-H A. 758 A. P270,000.00
T an 50 - - - 11,250 + 22,500 + 0.01(450,000)D
8.816 B. 1055 B. P156,297.25
= 60,750
50- H = 10.50 C.995 C. P186,927.25 *
D = 6 days
H = 39.49 m 0.1138 * D. P116,927.25

A= 8225 [(1 + 0.015)28 -1] Total no. of days delayed


Problems 4 to 7 No. of hollow blocks
0.015(1 +0.0 15) = 10 + 10 + 6 = 26 days
A mason was asked to compute the = 85.26 (1.10)
number of hollow blocks which have to 0.082418 A= 186,927.25 14. A flywheel of a puncher is to be
be purchased to fence one side of a = 1137.93 say 1138 brought to a complete stop in 8 seconds
triangle lot which the two other sides 11. A mechanical engineer who was from a speed of 60 revolutions per
having already been fenced by the 8. For a loan acquired six years ago, a awarded a P450,000 contract to install minute. Compute the angular
adjoining. One of the fenced sides is 40 man paid out the amount of P75,000. The the machineries of an oil mill failed to acceleration of the flywheel in rev per
m long the other 15 m longer than the interest was computed at 18% finish the work on time. As provided for in sec2.
s1de to be fenced. If the total length of the compounded annually. How much was the contract, he has to pay a daily penalty A. -0.125*
three sides is 125 m, the height of the the borrowed amount? equivalent to one-fourth of one percent of B. 0.125
fence required is 2436 mm, and the A. P24,782.36 the contract price for the first ten days of C. -0.512
height and length of each hollow block B. P26,782.36 the delay, one-half percen t per day for D. 0.512
are 203 mm and 406 mm, respectively. C. P25,782.36 the next ten days and one percent per
D. P27,782.36 * day for everyday thereafter. Determine w1 = 60 rev/sec
4. Determine the length of side to be
S = P(1 + i)"
the penalty for the first ten days. W2 =0
fenced by hollow blocks. A. P11,250.00 *
A. 35m* 6 B. P31 ,250.00
75,000 = P (1 + 0.18) ID2- IDj
B. 15m C. P21,250.00 a=---
P = P27,782.36 t
C. 40m D. P41,250.00
D. 55 m Penalty (1-10 days) a= O-l = -0.125 rev/sec2
9. A man purchased a car with a cash 8
x+(x+15)+40=125 = 0.0025 X P450,000 X 10
price of P350,000. He was able to
x =35m = P11,259
negotiate with the seller to allow him to
pay only a down payment of 20% and the REFRESHER MANUAL 2 nd Edition by JAS TORDILLO
REFRESHER MANUAL 2"d Edition by JAS TORDILLO
-- - -- - - - - - -- m

7 -1o I DAY 7 - SOLUTION DAY 7 - SOLUTION 17 -11


15. In problem No. 14, compute for the Annual Cost =Annual Dep + Interest on
ENGINEERING MATHEMATHICS
number of turns the flywheel will still Capital + OM +Tax
sv = 0.40(520,000) the gratuity is to be paid on yearly basis.
make before coming to stop. = 23,339 + 720,000(0.20) + 60,000 + SV = P208,000
A. 2 turns 0.02(720,000) If interest is 15%, what is the equivalent
B. 4 turns" = P241,739.00 lump sum that he could get?
C. 3 turns ~-
22. In problem No. 21, how much money A. P1 00,375.37 *
D. 9 turns must 'the company accumulate for 5 B. P137,375.37
·; 18. Compute the annual deprecation of
'!' .._"~-,
years if inflation annually is 10%. Resale C. P107,375.37
Offer B.
value wi ll also appreciate at 10% yearly . D. P237,375.37
wl = w12 + 28a • ' "' A. P44,944 *
A. P520,000
0 = 12 + 28 (-0.125)
e= 4 rev or '4 turns
B. P64,944
C. P54,944
D. P74,944
B. P502,480 *
C. P512,560
D. P310,000
P = R [ (! + r-J
I

Problems 16 to 20
For its proposed expansion, and ice plant
company is selecting form two offers of
Annual Dep = FC- SV
(l+il'-_1_
s = 520,000(1 + 0.1 0)5
- 208,000(1 + 0.10)5
P'=20,000 [(1+0.15)
10
0.15(1 +0. 15)10
-J]
ice cans. The data on the offers are as s= 502,479.12
p = 100,375.37
follows: For Offer A, 1.8 mm thick, total 640,000 -0 = P44,943.53
cost is P720,ooo: annual maintenance is
P6~000 and a life of 12 years. For Offer
= (1+0.16~-1 23. A fabrication company purchased an
adjacent lot for its expansion program 25. From an oblique triangle in which side
B, 1.6 mm thick, total cost is P640,000,
- 0.16 where it hopes to put up a building a = 6 em, side b = 9 em and angle C = 32
projected to cost P4M when it will be degree. Solve for angle A.
annual maintenance is P90,000 and a
title of 8 years. For their replacements, 19. Compute the annual cost of Offer B. constructed 10 years after. To provide for A. 19.1 deg
A. P275,744 * the required capital expenses, it plans to B. 39.1 deg *
the company is putting up a sinking fund
to earn 16% interest compounded B. P175,744 put up a si nking fund for the purpose. C. 29.1 deg
annually. If the money to purchase the C. P215,744 How much will the company deposit each D. 109 deg
ice cans is to be borrowed at 20% annual D. P144,744 year if interest to be earned is computed 2 2
interest and the tax on the first cost is at 15%? · c = a + b2 - 2abcos C
2%. Annual Cost = Annual Dep + Interest on A. P278,900 c2 = 62 + 92 - 2(6)(9) cos 32°
Capital +OM +Tax B. P156,009 c = 5.04 em
16. Compute the annual depreciation of = 44,944 + 640,000(0.20) + 90,000 +
0.02(640,000)
C. P197,009 * -a- = -c-
Offer A. D. P125,909 sinA sinC
= P275,744.00

r-I]
A. P12,339 6 5.04
B. P33,339
C. P23,339 * 20. Which offer will you recommend to be
purchased?
S=R[(I + sinA = sin32°
A=39.11°
D. P43,339
A. Offer A* ~

B. either Offer A or Offer B 26. In problem No. 25, solve for angle B.
4,000,000 = R [(1 + 0.15')10 _ 1] A.· 39 deg
Annual Dep = _ __ C. Offer B 0.15
D. none of the two offers B. 108.9 deg *
R = 197,008.25 C . 75 deg
21. A marketing company established a D. 180 deg
= 720,000-0 = P23,338.60
program to replace the cars of its sales 24. For having been loyal, trustworthy
(1 + 0.16) 1 ~ ~! and efficient, the company has offered a B=180-3239.1
. . 0.16 representatives at the end of every 5
years. If the present JJrice of the supervisor a yearly gratuity pay of B = 108.9°
approved type of car is P520,000 with a P20,000 for 10 years with the first
17. Compute the annual cost of Offer A. resale value at the end of 5 years of 40% payment to be made one year after his 27. The Phi lippine Long Distance
A. P190,739 of its present value. How much is the retirement. The supervisor, instead, Telephone Company has 183 m straight
B. P318,739 resale value at the end of 5 years? requested that he be paid a lump sum on vertical tower. There are two cables in
C. P241,739 * A. P500,000 the date of his retirement less interest the same vertical plane anchored at two
D. P369,739 B. P240,000 that the company would have earned if points on a level with the base of the
C. P420,000
D. P208,000 * REFRESHER MANUAL 2nd Edition by JAS TORDILLO
REFRESHER MANUAL 2"d Edition by JAS TORDILLO
1 -12 I DAY 7 - SOLUTION
DAY 7 - SOLUTION 17-13
ENGINEERING MATHEMATHICS
tower. The angles made by the cables ball150 meters from the ground. Find the
with the horizontal are 44 deg and 58 deg sec per. In what time will Bert overtake
respectively. Find the lengths of the
cables.
maximum height reached by the first ball.
A. 150m
B. 326.2 m *
Toti?
A. 6.164 sec* R[('+f-']= Rr
A. 236m and 251 m C. 240m B. 10. 164 sec
B. 263 m and 215 m<* D. 362.2 m C.164 sec R[(1-0.10}30-1 ]= 120,00
C. 245 m and 263 m D.12.164sec 0.10 0.10
D. 289m and 315'm._ Vi= V, 2 + 2gS
R = 7,295.09
0 = (80)2 + 2(-9.81 )S, Sr =distance traveled by Toti
s, = 326.2 m Sa = distance traveled by Bert
Sin 44° = ~ T =time traveled by Bert and Toti 36. A unit welding machine cost P45,000
a with an estimated life of 5 years. Its
a= 263.4 m . 31. In problem No. 30, solve for the time
when the first ball reached its maximum Sr = 30t + Y, (-4)t2 Eq. 1 salvage value is P2500. Find its
height. Sa = 30t Eq. 2 depreciation rate using straight
Sin 58° = ~ A. 5.994 sec
deprecia~ion .
b Sa= 76 + Sr A. 18.9% *
B. 9.149 sec
b = 215.7 m 30t = 76 + 30t + Yz (-4)t2 B. 12.1 %
C. 8.155 sec*
D. 9.941 sec t = 6.164 sec c. 16.7%
28. In problem No. 27, if the cables are D. 10.2%
on the same side of the tower, find the
d1stance between their anchor points.
_ v - v1
2-
g- -
34. In problem No. 33, find the distance
FC - SV
traveled by Bert in overtaking Toti? Annual Dep = - - -
A. 60m t1 A. 110m n
B. 300m B. 185m*
-9.81 = 0-80 = 45,000 - 2500 = 8500
C. 75 m * t} C. 150m
D. 450 m 5
t, = 8.155 sec
D. 225m
Depreciation Rate
x=A-B _ Annua!Dep 8500
32. In problem No. 30, find the initial Sa= 30t = 30(6.164) = 184.92 m .
183 183 - FC = 45,000
x=----- velocity of the second ball.
tan440 tan58° 35. Ed who was then 26 years old = 0.1889 =18.89%
A. 31.27 m/s
X= 189.5-114.35 B. 91.27 m/s planned to retire at the age of 56 years
and so he purchased an annuity. He 37. In problem no. 36, assuming that you
x=75.1m c. 61.27 m/s * agreed to pay equal annual payments, will deposit the money to a bank giving
D. 99.27 m/s
the payment was made on his 26 111 8.5%, solve for the deposit.
29 In problem No. 27, if the cables are
on the opposite sides of the tower, find birthday and the last was made on his 55 A. P7,172.54 *
s2 = 326.2- 150 = 176.2 m
birthday. At present, Ed is receiving a B. P8,500.00
the distance between their anchor points. v2 = o yearly income of p120,000 per which he C. P5,172.54
A. 75 m
B. 250m started receiving on his 561h birthday. D. P9,500.00
s2 = V2t2 + % gtl ·
C. 90 m 176.2 = 0 +% (-9.81)t/ Compute for the annual amount that he
D. 304m* h= 5.994 sec paid for the annuity if interest rate is 10%.
A. P2,295.10 .
Annual Deposit = FC - SV
t = 8.155 + 5.994-5 = 9.149 sec
B. P6,295.1 0 (!.+i'f_:. ~
x=A+B
183 183 150 = V(9.149) + Yz (-9.81)(9.149)2 C. P4,295.10
- 45,000- 2,500
x=----- D. P7,295.10 *
tan440 tanss0 V = 61.27 m/s
- (1+ o.o8s)S -I
X= 189.5- 114.35
33. Two cars left the building for Bataan R =annual payment of annuity 0.085
x = 303.85 m PA =future value of annuity
to attend the meeting. Bert in a Toyota = P7,172.54
and Toti in a Mercedez Benz have a Pp = present value of perpetuity
30. Usmg a powerful air gun, a steel ball 38. How many ways can a PSME
velocity of 30 m/s both in the same Bert
is shot vertically upward with a velocity of PA = Pp Chapter with 15 directors choose a
is 76 meters behind Toti when the brakes
80 m/s, followed by another shot after 5 President, a V-President, a Secretary, a
was applied to the Benz, causing it to
seconds. The second ball met the first
decelerate at a constant rate of 4 m per REFRESHER MANUAL .2nd Edition by JAS TORDILLO

REFRESHER MANUAL 2"d Edition by JAS TORDILLO


7 -14 I DAY 7 - SOLUTION
DAY 7 - SOLUTION 7 -15
ENGINEERING MATHEMATHICS
I
Treasurer and an Auditor, if no member 41. In problem No. 40, what is the book
can hold more than one position? value of the machine at the end of six TtPJ = T2P2 B. consumer goods *
A. 160,160 ways years? C. supply
wp 1 ~D 2
D. demand
B. 360,360 ways • A. P400,000
C. 260,260 ways B. P241,000 30(10) 40f'2
D. 460,460 ways C. P341,000 * 100(50) = 800(40) 50. The sum of the first cost and the
. D. P200,000 P2 =48 HP present worth of all cost of any property
of replacement, operation and
No. of ways= {n h!
_ r)' '! t
Book Value (6 yrs) = FC- Total Dep 44. Any method of repaying a debt, the maintenance for a long time or forever.
= {800,000)- (76,500 X 6) principal and interest included, usually by A. annual cost
_ _15_! = 360,360 ways = 341,000 a series of equal payments at equal B. total cost
- (15 - 5) intervals of time. C. capitalized cost*
42. A provincial chapter of PSME held a A. an1'ft.lity D. expenses
39. Chemist of a distillery experimented lottery to raise funds for their B. amortization *
on two alcohol solutions of different organization, with P1 0,000 top prize and C. perpetuity 51 . Relative to the force needed to keep
strength 35% alcohol and 50% alcohol, with 2,000 tickets printed and sold. What D. compound a box moving at constant velocity across
respectively. How many cubic meters of is the mathematical expectation of a a floor, to start the box moving usually
each strength must he use in order to member if he bought 10 tickets? 45. Funds supplied by others on which a needs
produce a mixture of 60 cubic meters that A. P2000 fixed rate of interest must be paid and A. less force
contain 40% alcohol? B. P100 the debt !:>e repaid at a specific time. B. the same force
A. 10, 50 C. P1000 A. borrowed capital * C. more force*
B. 40,20 * D. P50 * B. first cost D. any of the above, depending on the
C. 20, 30 C. discount natures of the surface in contact
D. 30,30 D. bond
Prob= ~ 52. When two surfaces are in contact, the
2000
X+ y = 60 46. The length of time during which the frictional force between them depends on
Mathematical Expectation
y = 60 -x Eq. 1 property may be operated at a profit. which one or more of the following?
= (__.!.Q_)
2000
P1 0,000 =P50.00 A. economic life * A. the normal force pressing one surface
0.35 x 0.50(60- x) = 0.40(60) B. physical life against the other
0.35 + 30- 0.50x = 24 C. bank life B. all of these*
0.15x = 6 43. The time required for an elevator to D. lifetime C. whether the surfaces are stationary or
x = 40 m3 of 35% alcohol lift a weight varies directly with the weight in relative motion
y = 60 - 40 = 20 m3 of 50% alcohol and the distance through which it is to be 4 7. The length of time during which it is D. whether a lubricant is used or not
lifted and inversely as the power if the capable of performing the function for
40. The initial cost of a paint sand mill, motor. If it takes .30 seconds for a 10 HP which it was designed and manufactured. 53. Compared with her mass and weight
including its installation, is P800,000. The motor to lift 100 lbs through 50 ft, what is A. economic life on Earth, an astronaut on Venus, where
2
SIR approved life of this machine is 10 the size of motor is required to lift 800 .lbs B. pro life the acceleration of gravity is 8.8 m/s , has
years for depreciation. The estimated in 40 seconds through 40 ft? C. physical life* A. less mass and less weight
salvage value of the mill is P50,000, and A. 20 HP D. no. of periods B. less mass and the same weight
the cost of dismantling is estimated to be B. 48 HP" C. less mass and more weight
P15,000. Using straight line depreciation, C. 30 HP 48. The value of the variable for which D. the same mass and less weight *
what is the annual depreciation charge? D. 68 HP the costs for the alternative will be equal.
A. P75,000 A. break-even analysis 54. A car towing a trailer is accelerating
B. P85,000 TaWD B. break-even graph on a level road: The car exerts a force on
C. P76,500"' p C. break-even point* the trailer whose magnitude is
D. P90,000 D. break-even business A. the same as that of the force the
T=k WD trailer exerts on the car*
p
FC-SV 49. Products that are directly used by B. the same as that of the force the trailer
Annual Dep = - - n - k= TP people to satisfy their wants. exerts on the road
WD A. producers goods
- (800,000 + 15,000)- 50,000 = 76,500
- 10 REFRESHER MANUAL 2nd Edition by JAS TORDILLO

REFRESHER MANUAL 2"d Edition by JAS TORDILLO


DAY 8- EXAM ~8-1
7- 16 I DAY 7 - SOLUTION ENGINEERING MATHEMATHICS
C. the same as that of the force the road C. 3.4 m/s
exerts on the road D. 1.4 m/s * 6. If the specific weight of liquid is 583.5
D. greater than that of the force the trailer DAY 8- EXAM lbf/cubid foot, what is the specific volume
exerts on the car
v = r(KE) =F(IO) = 1.4 m/s 1. Starting from rest, an airplane acquires
of the liquid?
A. 1.0657 cm 3 /g
55. In Newton's t~ird law of motion the m 10
a take-off velocity of 150 mph on a 2-mile B. 0.6748 cm 3/g
action and reaction forces run-away. The acceleration is assumed C. 0.9504 cm /g
3
A. act on the sall'le object 59. A hoist powered by a 15-kW motor is
used to raise an elevator loaded with
to be uniform. Calculate the time required D. 0.5321 cm 3/g
B. act on different'Obj~cts * to reach take-off speed.
C. do not necessarily have the same workers and having a total mass of 700
A. 80 sec 7. Which of the following is not a unit of
magnitude and do not necessarily have kg to a height of 100 m. If the efficiency
B. 116 sec pressure?
the same line of action of the hoist is 80 percent, find the time
needed.
C. 96 sec A. Pa
D. have the same magnitude but do not D. 132 sec B. kg/m-s
2
have necessarily have the same line of A. 57.1 sec*
B. 78.2 sec C. Bars
action 2 Water drips from a faucet at the rate of 2
C. 80.4 sec D. kg/m
4 drops per second. Determine the
56. A jumper of weight w presses down D. 90 sec
distance between two successive drops 1 8. A cylinder weighs 150 lbf. Its cross-
on the floor with a force of magnitude F, sec after the first one has fallen. sectional area is 40 in2 . When the
and the jumper leaves the floor as a Pout = P;n(Eff) = 15(0.8) A. 4.0 ft cylinder stands vertically on one end,
result. The force the floor exerted on the = 12 kw = 12,000 W B. 6.03 ft what pressure does the cylinder exert on
jumper must have had a magnitude p = W = FS c. 5.2 ft the floor?
A. equal to wand less than F t t D. 7.04 ft A. 14.1 kPa
B. equal to wand equal to F
t = FS = mgh = 700(9.8) (100} B. 63.2 kPa
C. more than wand equal to F * 3. An iron block weighs 10.5 N and has a C. 25.8 kPa
p p 12000
D. more than wand more than F volume of 400 cubic centimeters. What is D. 89.7 kPa
= 57. 16 sec
the density of the block?
57. A 3000-lb car ascends a 12° slope at A. 2478 kg/m 3 9. What is the flow rate through a pipe 4
60. A horse has a power output of 1.1 kW
30 mph. If the overall efficiency is 80 B. 2889 kg/m
3
inches in diameter carrying water at a
when it pulls a car with a force of 500 N.
percent, what is the power output of the C. 2676 kg/m
3
velocity of 11 ft/s?
What is the car's velocity?
car's engine? D. 3015 kg/m
3 3
A. 726 cm /s
A. 2·.2 m/s*
A. 62.3 hp* B. 19184 cm /s
3
B. 4.5 m/s,
B. 67.7 hp 4. If the density of. a gas is 0.003 slugs C. 993 cm3 /s
C. 3.5 m/s
C. 77.7 hp per cubic ft, what is the specific weight of D. 27183 cm 3/s
D. 87. 7 hp
D. 5.5 m/s
the gas?
3
A. 9.04 N/m 10. A car having a mass of 1000 kg is
F = w sin 9 = 3000 sin 12° = 623.7 lb v =
p
1= 1100
= 5oo
= 2.2 m/s B. 76.3 N/m3 driven around a curve with a velocity of
V = 30 mph = 44 ft/sec C. 15.2 N/m
3
20 m/s, radius is 10 m. Find the
D. 98.1 N/m3 centrifugal force in Newtons?
P = F V = 623.7(44) A. 40,000
= 27,444.3 ft-lb/sec 5. The specific gravity of mercury relative B. 30,000
p = 27,444.3= 49.9 hp to water is 13.55. What is the specific c. 36,500
550 weight of mercury? The specific weight of D. 20,000
At 80 percent efficiency, water is 62.4 lbf per cubic foot.
3
49 9 A. 82.2 kN/m 11. A highway curve of radius 40 meters
p = ·
0.80
=
62.3 hp B. 132.9 kN/m 3 is designed for a maximum car speed of
3
C. 102.3 kN/m 72 kph. If the coefficient of friction
D. 150.9 kN/m3 between tires and the road is 0.50, what
58. What velocity does a 10-kg object
is the angle, from the horizontal that the
have when its kinetic energy is 10 J?
road should be banked?
A. 4.4 m/s
A. 11 deg
B. 2.4 m/s
REFRESHER MANUAL 2nd Edition by JAS TORDILLO
REFRESHER MANUAL 2"d Edition by JAS TORDILLO
DAY 8- EXAM ~8-3
s -2 DAY 8 .- EXAM ENGINEERING MATHEMATHICS
B. 19 deg the horizontal. It has an initial velocity of
C. 15 deg 45 m/s. How long will it take before B. P70,000 28. A car is traveling on a horizontal
D. 23 deg the projectile hits the ground? C. P65,000 unbanked circular track of radius r.
A. 4.1 sec D. P66,550 Coefficient of friction between the tires
12. A car moving at 70 kph has a mass of B. 7.94 sec and the track is 0.30. Determine the cars
1700 kg. What force is necessary to C. 5.8 sec 23. An individual possesses a promissory velocity while traveling at a radius of 34
decelerate it at-a rate of 40 cm/s? D. 9.53 sec note due in three years at a maturity m, without skidding.
A. 680N ' value of P10,000. At an interest rate of A. 15 m/s
B. 1150 N 18. A motorist on a highway curve of 6%, what proceeds will he obtain by B. 14 m/s
C. 980 N radius 80 meters is traveling at 20 m/s. discounting the note at the present date? C. 12 m/s
D. 1365 N His speed in the tangential direction A. P6,389.91 D. 10 m/s
increases at the rate of 3 m/s2 • What is B. P8,396.19
13. A machine is capable of accelerating the motorist total acceleration? C. P6,936.33 29. A certain steel tape is known to be
a 1 kg mass at 1 m/s2 for 1 meter. The A. 5.83 m/s2 D. P9,638.21 100 ft long at a temperature of 70°F.
machine runs at 60 rpm. What is the B. 1.38 m/s2 When the tape is at a temperature of
power output of the machine? C. 3.58 m/s2 24. The selling, price of a TV set is 10°F. what tape reading corresponds to a
6
A. 1 cal D. 1.03 m/s2 double that of its net cost. If the TV set distance Of 90 ft? k for steel = 6.5 X 10'
B. 1 w ·sold to a customer at a profit . of 25% of per °F.
C. 1 J 19. If a deposit now of P15,249.13 will the net cost, how much discount given to A. 90.035 ft
D. 1 kw become P300,000 in 18 years, what is the customer? B. 91 .025 ft
the rate of interest compounded A. 37.5% C. 92.045 ft
14. A power of 6 kw is supplied to the annually? B. 50.0% D. 90.039 ft
motor of a crane. The motor has an A. 18% C. 42.5%
efficiency of 90%. With what constant B. 14% D. 60.0% 30. What is the simplified equivalent
2
speed does the crane lift an 800 lbf C. 16% expression of (cof 9)(sin2 9) = 1/csc e.
weight? D.12% 25. A man borrowed P280,000 and A.1
A. 0.09 m/s agrees to pay the loan at the end of each B.O
B. 0.98 m/s 20. For a loan acquired six years ago, a month for 48 months at 1.5% interest per C. cos 9
C. 0.32 m/s man paid out the amount of P750,000. month. What is the monthly payment? D. sin 2 9
D. 1.52 m/s The interest was computed at 18% A. P2,582.00 .
compounded annually. How much was B. P5,822.00 31 . How far will an object in a standard
15. If a particle's position is given by the the borrowed amount? C. P5,228.00 gravitational field drop in 13 sec, starting
expression. x(t) = 3.4t3 - 5.4t meters, A. P282,770 D. P,8,225.00 from rest and neglecting air friction?
what is the acceleration of the particle B. P277,820 A. 2721 ft
aftef t = 5 seconds? C. P272,780 26. You borrowed the amount of P10,000 B. 2595 ft
A. 3.40 m/s2 D. P360,577 for 120 days at 30% per annum simple C. 2661 ft
B. 25.5 m/s2 interest. How much will be due at the end D. 2275 ft
C. 18.1 m/s2 21. On January 1, 1991, Justine of 120 days?
2 borrowed P50,000 from Dave and
D. 102 m/s A. P10,100 32. The velocity of a particle at time t is:
discharged the debt on Dec. 31, 1995 B. P11,600 v(t) = 12t4 = 7ft. What total distance is
16. A car starts from rest and has a with the payment of P70,000. What was C. P11,000 traveled between t =0.2 and t =0.3?
2 interest rate, compounded annually, on
constant acceleration of 3 ft/s . What is D. P12,000 A. 4.22
the average velocity during the first 10 this loan? B. 3.85
seconds of motion? A. 6.69% 27. How much should you put into a 10% C. 3.25
A. 16 ft/s B. 9.66% savings account in order to have P50,000 D. 2.84
B. 14 ft/s C. 6.96% . in eight years?
C. 15 ft/s D. 12.9% A. P23,500.73 33. The position of an object as a
function of time is described by x = 4t +
3
D. 13 ft/s B. P23,325.37
2e -
17. A projectile is launched upward from
level ground at an angle of 60 deg with
I22. Find the sum of a deposit of P50,000
now after three years using 10% simple
interest.
C. P24,325.37
D. P24,000.00 I
t + 3. What is the acceleration of the
object at t =2?

A. P60,000 REFRESHER MANUAL 2nd Edition by JAS TORDILLO

REFRESHER MANUAl2"d Edition by JAS TORDlllO


DAY 8- EXAM ~8-5
8-4 DAY 8- EXAM ENGINEERING MATHEMATHI,CS
A 52 deposited to yield a total of P10,000 in 10
B. 64 years? • remarnrng balance will be paid on depreciation for the second year using
C. 56 A P1 458 installment basis with a monthly payment the sum-of-the-years digit method.
D. 66 B. P3875 of P12,000 for 60 months. Determine the A P17,672.00
C. P2550 interest rate compounded annually. B. P18,276.00
34. The diameter of-steel rod is 76.2 mm. D. P4564 A. 19.56% C. P17,850.00
Determine the el~ngation of the rod per B. 16.45% D. P19,636.00
meter length. The ·force applied is 8000 40. The quantity of a certain commodity c. 18.00%
kN. ' that is offered for sale at a certain price at D. 14.35% 51. It is a quantity of substance that is
A 8.48. mm a given place and time: homogenous throughout
B. 12.85 mm A. stocks 46. An item is purchased for P1 00,000. A. State
C. 9.35 mm B. demand Annual cost is P18,000. Using 10%, what B. Phase
D. 15.65 mm C. goods is the capitalized cost of the perpetual C. Datum
D. supply service? D. Property
35. The copper bar is 90 em long at 86°F. A. P220,000
What is the increase in its length when 41. The place where buyers and sellers B. P265,000 52. The amount of energy required to
the bar is heated to 95°F? The linear exchange goods: C. P250,000 convert saturated liquid to saturated
expansion coefficient for copper is 1.7 x A demand D. P280,000 vapor or from saturated vapor to
10'5 per deg C. B. market saturated liquid with the temperature that
A. 2.12 X 10'5 m C. business 47. A parent on the day the child is born remains constant
B. 5.25 x 10·5 m D. buy and sell section wishes to determine what lump sum A. Latent heat
C. 3.22 x 10'5 m would you have to be paid into an B. Sensible heat
D. 7.65 x 10·5 m 42. The amount of P50,000 is deposited account bearing interest at 5% C. Specific heat
in a bank. How much money are you compounded annually, in order to D. Internal energy
36. A car travels 100 miles to city A in 2 going to withdraw after 8 years at 8% withdraw P20,000 eac.h on the child's
hours, then travels 200 miles to city B in compounded annually? 18th, 19th, 20th and 21 51 birthdays. 53. A thermodynamic property that was
3 hours. What is the average speed of A. P82,546.00 A. P35,941 .73 first introduced by Clausius in 1865 and
the car for the trip? B. P92.546.00 B. P30,941.73 could be defined as the measure of the
A. 45 mph C. P85,456.00 C. P33,941.73 irreversibility of the system or substance.
B 58.3 mph D. P97,856.00 D. P25,941 .73 A. Enthalpy
C. 66.7 mph B. Internal energy
D. 60 mph 43. These are products or services that 48. How much must you invest today in C. Entropy
are desired by human and will be order to accumulate P20,000 at 8% after D. Temperature
37. How long it will take a sum of money purchased if money is available after the 6 years?
to double at a 5% annual interest required necessities have been obtained. A. P20,004.50 54. The measure of the unavailable
percentage rate? A. demand B. P15,305.60 energy in an irreversible process.
A 14.2 years B. utilities C. P18,450.80 A Enthalpy
B. 18.4 years C. necessities D. P12,603.40 B. Internal energy
C. 15.9 years D. luxuries C. Entropy
D. 19.3 years 49. A bank offers 2% effective monthly D. Temperature
44. These are the products or services interest. What is the effective annual
38. It is defined to be the capacity of a that are required to support human life interest rate? 55. Thermodynamic properties, which
commodity to satisfy human want: and activities that will be purchased in A. 26.82% depend of the mass.
A. discount somewhat the same quantity even B. 24.65% A. Extensive properties
B. utility through the price varies considerably: c. 25.28% B. Intensive properties
C. luxuries A. necessities D. 22.45% C. Specific properties
D. necessity B. producer goods D. Specific heat
C. luxuries 50. An asset is purchased for P120,000
39. Funds are deposited in a savings D. utilities and it can be sold for P12,000. Its 156. A classification of temperature and
account at an interest rate of 8% per estimated life is 10 years. Find the tota l volume as properties.
annum compounded semi-annually. What 145. A car was b-ought at P549,492.13
is the initial amount that must be with 14% down payment and the REFRESHER MANUAL 2nd Edition by JAS TORDILLO
REFRESHER MANUAL 2"d Edition by JAS TORDILLO
B-6 DAY 8 - EXAM
DAY 8 - SOLUTION 8-7
ENGINEERING MATHEMATHICS
I
A. Intensive properties
B. Extensive properties
C. Specific properties
I ra
1''-;-::::::=====
) DAY 8- SOLUTION
3. An iron block weighs 10.5 N and has a
volume of 400 cubic centimeters. What is
D. None of the above DAY 8- ANSWER KEY the density of the block?
1. Starting from rest, an airplane acquires 3
1. c 16. c 31. A 46.0 A. 2478 kg/m
a take-off velocity of 150 mph on a 2-mile 3
57. That portion of the universe, an atom, 2. 0 17.8 32. 0 47. B B. 2889 kg/m
run-away. The acceleration is assumed 3
a certain quantity of matter, or a certain 3.C 18. A 33.A 48.0 C. 2676 kg/m *
volume in space that one wishes to 4. c 19.A 34. A 49. A to be uniform. Calculate the time required D. 3015 kg/m
3

5. B 20. B 35.0 50. A to reach take-off speed.


study. ·
6.A 2l.C 36.0 51. B A. 80 sec
A. Property 7. 0 22.C 37. A 52. A
. mass
B. 116 sec Density = - - -
B. System 8.C 23. B 38. B 53. C volume
C. Fluid 9. 0 24.A 39.0 54. C C. 96 sec*
D. State 10. A 25.0 40. 0 55. A D. 132 sec -10.5 kg
11. B 26. C 41. B 56.0
;; 9.81
58. Descriptive characteristics used to 12. A 27. B 42.8 57.8 v, = 0 400 em3x _ ...m3
__...
13. B 28. 0 43.0 58.C v2 = 150 mph= 220 fUsee
express the behavior of the system or
14. 0 29. A 44. A 59. B I I-
(IOo)3cmJ
substance. S = 2 mile= 10,560 ft 3
45. A 60. 'C = 2675.84 kg/m
A. Datum
1
G 15. 0 30. A
I)
vl = v,2 + 2aS
B. State 4. If the density of a gas is 0.003 slugs
C. Properties (220)2 = 0 + 2a(1 0,560) per cubic ft, what is the specific weight of
2
D. System a= 2.29 fUs
the gas?
3
A. 9.04 N/m
59. A combination of processes taking a s = v,t +% (2.29)t2 B. 76.3 N/m3
10,560 = 0 +% (2.29)f
system through a succession of states C. 15.2 N/m 3 *
and returning to its initial state. t = 96 sec D. 98.1 N/m3
A. Process
B. Cycle 2. Water drips from a faucet at the rate of
Weight
C. State 4 drops per second. Determine the
='0.003 slug( · b) ~x 9.81~2
distance between two successive drops 1
32 21
D. Datum
sec after the first one has fallen. , , slug 2.21b 5
60. When two bodies, isolated from other A. 4.0 ft = 0.43075 N
environment, are in thermal equilibrium
with a third body, the two are in
equilibrium with each other.
B. 6.03 ft
c. 5.2 ft
D. 7.04 ft *
Volume = 1 ft3 ( m:
3.28 ft 3
J; 0.028338 m
3

A. first Law of Thermodynamics .


B. Second Law of Thermodynamics t = time between drops Specifi c weight = weight
...• = Y. sec per drop volume
C. Zeroth law
D. Amagat's Law t, = 1 sec 0.43075 N
h=1-Y.=o/.sec
=
0.028338 m3

s, =v,t, +% at1
2
2
= 15.2 ~
= 0 +% (32.2)(1 ) m3
s, = 16.1 ft
5. The specific gravity of mercury relative
S2 = v,t2 +% atl to water is 13.55. What is the specific
2
= 0 +% (32.2)(0.75) weight of mercury? The specific weight of
s2 = 9.056 water is 62.4 lbf per cubic foot
3
A. 82.2 kN/m
s = s,- s2 = 16.1-9.056 B. 132.9 kN/m *
3

s = 7.043 ft
I

REFRESHER MANUAL 2nd Edition by JAS TORDILLO

REFRESHER MANUALZ"d Edition by JAS TORDILLO


DAY 8 - SOLUTION Is- 9
8- 8 DAY 8 - SOLUTION ENGINEERING MATHEMATHICS
C. 102.3 kN/m cylinder stands vertically on one end,
D. 150.9 kN/m
3 what pressure does the cylinder exert on B. 19 deg * speed does the crane lift an 800 lbf
the floor? C. 15 deg weight?
3 A . 0.09 m/s
Vwatar = 62.4 lb/ft3 = 9.81 kN/m A. 14.1 kPa D. 23 deg
B. 63.2 kPa B. 0.98 m/s
VHG = S.G. X Vwater C. 25.8 kPa * V = 72 kph = 20 m/s C. 0.32 m/s
= 13.55 (9.81) D. 89.7 kPa Tan a= 0.50 D. 1.52 m/s *
a= 26.56° (frictional angle)
= 132.92 k~ v2 F = 800 lbf = 363.63 kg= 3567.21 kN
m Pressure= force
area Tan (9 +a)=-
gR
6 If the specific weight of liquid is 583.5 = Efficiency = Po = FxV
lbf/cubid foot, what is the specific volume ~ = 3.751bflin2 x IOJ.325 kPa Tan (9 + 26.56) = (2o)2 F\n F\n
of the liquid? 40 14.7 psi 9.81(40) 0.90 = 3567.21 v
A. 1.0657 cm3 /g * = 25.85 kPa e = 18.99° say 19 deg 6000
3
B. 0.6748 cm /g V=1.514m/s
C. 0.9504 cm3/g 9. W hat is the flow rate through a pipe 4 12. A car moving at 70 kph has a mass of
D. 0.5321 cm 3/g inches in diameter carrying water at a 1700 kg. What force. is necessary to 15. If a particle's position is given by the
velocity of 11 ftls? decelerate it at a rate of 40 cm/s? expression. x(t) = 3.4e - 5.4t meters,
Specific volume 3
A. 726 cm /s A. 680 N * what is the acceleration of the particle
B. 19184 cm3 /s B. 1150 N after t = 5 seconds?
I I tt3
= --=-=0.017094- C. 993 cm3/s C. 980 N A. 3.40 m/s2
density 58.5 lbf D. 1365N
D. 27183 cm 3/s * B. 25.5 m/s2
Specific volume C. 18.1.m/s 2
F =rna D. 102 m/s2 *
= 0.017094 tt3 X 2.2lb D = 4 in= 10.16 em
= 1700(0.40)
lbf lOOOg V = 11 ft/s = 132 in/s = 335.28 cm/s
= 680 N Distance = x(t) ::: 3.4tl- 5.4t
(2.54~ cm
3
1728 in3 Q=AV
x--x~~=--
ft3 in3 13. A machine is capable of accelerating Velocity= x'(t) = 3.4(3)f- 5.4
= ~(10.16)2(335.28) a 1 kg mass at 1 m/s2 for 1 meter. The
3
=1.065 ~ = 27,182.3 cm4/sec
machine runs at 60 rpm. What is the
power output of the machine?
Acceleration = x"(t) = 3.4(3)(2)t
g = 3.4(3)(2)(5)
A. 1 cal = 102 m/s2
7. Which of the following is not a unit of
10. A car having a mass of 1000 kg is
driven around a curve with a velocity of
B.1 w·
pressure? C. 1 J 16. A car starts from rest and has a
20 m/s, radius is 10 m. Find the
A Pa centrifugal force in Newtons?
D. 1 kw constant acceleration of 3 ftls 2. What is
B. kg/m-s2 A. 40,000 *
the average velocity during the first 10
C. Bars B. 30,000 Frequency, f = 60 re.v x~ =1 rev seconds of motion?
D. kg/m 2 * c. 36,500 mtn 60 sec sec A . 16ftls
B. 14ftls
D. 20,000 . I
T1me, t
= -
f
=-I =
I
1 sec c. 15 ft/s*
Pressure = force D. 13ftls
area
mv2 (lOOOX2o)2 =40,000 N F = 2
ma = ( 1 kg)( 1 m/s ) =1 N
Fe::: R= 10 Power= FxD = (1 NXI m) = 1 W V1:::0
A. Pa - is a unit of pressure
B. Bars - is a unit of pressure (1 bar =
t I sec _ v2 -v1
a---
11. A highway curve of radius 40 meters t
100 kPa)
is designed for a maximum car speed of 14. A power of 6 kw is supplied to the
C. kg/m-s2 - is equal to pascal, a unit of
72 kph. If the coefficient of friction motor of a crane. The motor has an
v2 -o
pressure 3= - -
between tires and the road is 0.50, what efficiency of 90%. With what constant 10
D. kg/m2 - is equal to mass per unit area.
is the angle, from the horizontal that the V2 = 30 ftls
8. A cylinder weighs 150 lbf. Its cross-~ road should be banked?
REFRESHER MANUAL 2nd Edition by JAS TORDILLO
sectional area is 40 in2 When the A. 11 deg
REFRESHER MANUAL2"d Edition by JAS TORDILLO
~ -------------=------------~------------------------------~,-~~-------------------------------- .........................

s- 1o DAY 8 - SOLUTION
DAY 3 - SOLUTION Is -11
ENGINEERING MATHEMATHICS
Average velocity 20. For a loan acquired six years ago, a
= Vt+V2 =0+30=lSft/s man paid out the amount of P750,000. S = P(1 + i}" B. P11 ,600
2 2 The interest was computed at 18% 10,000 = P( 1 + 0 .06)3 C. P11,000 *
compounded annually. How much was p = 8,396.19 D. P12,000
17. A projectile is launched upward from the borrowed amou nt?
level ground at an angle of 60 deg with A. P282,770 24. The selling, price of a TV set is S =P(1 + ni}
the horizontal. It has an initial velocity of B. P277 ,820 * double that of its net cost. If the TV set 120
45 m/s. How long will it take before C. P272,780 sold to a customer at a profit of 25% of s = 10,000 (1 + 360 (0.30))
the projectile hits the ground? D. P360.577 the net cost, how much discount given to
A. 4.1 sec the customer? s = 11,000
B. 7.94 sec* S =
P(1 + i}" A. 37.5%*
C. 5.8 sec 750,000 = P(1 + 0.18}6 B. 50.0% 27. How much should you put into a 10%
p = 277,823.60 C. 42.5% savings account in order to have P50,000
D. 9.53 sec
D. 60.0% in eight years?
y = 0 (same horizontal plane) 21. On January 1, 1991, Justine A. P23,500.73
borrowed P50,000 from Dave and S = selling price B. P23,325.37 *
discharged the debt on Dec. 31, 1995 C. P24,325.37
y = Vo sin 9 t- Y2 gf N =net cost
with the payment of P70,000. What was D. P24,000.00
0 = 45(sin 60°)t + Y. (9.8 1 )f P = principal or capital
4.905t = 38.971 interest rate, compounded annually, on .
S = P(1 + i}"
=
t 7.94 sec this loan? S = 2N Eq. 1
50,000 = P(1 +0.1 0) 8
A. 6.69%
B. 9.66% P = 1.25N Eq. 2 p = 23,325.37
18. A motorist on a highway curve of
radius 80 meters is traveling at 20 m/s. c. 6.96%*
D. 12.9% 28. A car is traveling on a horizontal
His speed in the tangential direction Discount = S- P _ 2N- 1.25N unbanked circular track of radius r.
increases at the rate of 3 m/s2 . What is S - 2N
the motorist total acceleration? S = P(1 + i}" Coefficient of friction between the tires
2 70,000 = 50,000(1 + i}5 = 0.375 = 37.5% and the track is 0.30. Determine the cars
A. 5.83 m/s *
2 i = 0.06961 = 6.96% velocity while traveling at a radius of 34
B. 1.38 m/s 25. A man borrowed P280,000 and m, without skidding.
C. 3.58 m/s2 agrees to pay the loan at the end of each
D. 1.03 m/s2 22. Find the sum of a deposit of P50,000 A. 15 m/s
month for 48 months at 1.5% interest per B. 14 m/s
now after three years using 1 0% simple
interest. month. What is the monthly payment? C. 12 m/s

an=
v2 =202
R so
= 5 ml sz A. P60,000
A. P2,582.00
B. P5,822.00
0 . 10 m/s *
B. P70,000
C. P5,228.00
C. P65,000 * v2
a1 = Ja? +a~ I D. P66,550 .
.- D. P8,225.00 * f= -
gR

S = P(1 + ni)
=R[ (1 + i}'l - 1]
v2
ar = J(3)2+(5)2 =5.83 m/s2 s = 50,000[1 + 30(0.10)] p
0.30 = 9.81(34)
i(1 + iJ'l
s = 65,000 v = 10 m/s
19. If a deposit now of P15,249.1 3 will
become P300,000 in 18 years, what is 23. An individual possesses a· promissory 280,000 = R [ (1 + 0.015)48 - 1 ]
29. A certain steel tape is known to be
the rate of interest compounded note due in three years at a maturity 0.015(1 + 0.0 15)48
100 ft long at a ·temperature of 70°F.
annually? value of P10,000. At an interest rate of R = 8,225.00 When the tape is at a temperature of
A.18% * 6%, what proceeds will he obtain by 10°F, w hat tape reading corresponds to a
B. 14% discounting the note at the present date? 26. You borrowed the amount of P10,000 distance of 90ft? k for steel = 6.5 x 10-e
C. 16% A. P6,389.91 for 120 days at 30% per annum simple per °F.
D. 12% B. P8,396.19 * interest. How much will t>e due at the end A. 90.035 ft *
C. P6,936.33 of 120 days? B. 91.025 ft
S = P(1 +if D. P9,638.21 A. P10,100
300,000 = 1~.249.13( 1 + i)
18

i = 0.18 = 18% per annum REFRESHER MANUAL 2nd Edition by JAS TORDILLO
REFRESHER MANUAL 2"d Edition by JAS TORDILLO
,..

DAY 8 - SOLUTION Is -13


s - 12 DAY 8 - SOLUTION ENGINEERING MATHEMATHICS
C. 92.045 ft Integrate V(t):
D. 90.039 ft

5
5
S = -12t- + 71nt I8:~ A. 2.12 X 10'5 m
B. 5.25 x 10·5 m
deposited to yield a total of P10,000 in 10
years?
Y=kUT C. 3.22 x 10'5 m A. P1458
= 6.5 X 10'6 (90)(70-10) 5 5
12( 0.3 - 0.2 ) D. 7.65 X 10'5 m * B. P3875
= 0.0351 S= + 7(1n0.3 -ln0.2) C. P2550
5 Ll.T = 95 - 86 = 9°F = 5°C D. P4564 *
LI.L = Y + L =0.0351 + 90 s = 2.843 Y = klt.T = 1.7 X 10' 5 {0.90)(5)
=90.0351 ft Y = 7.65 x 10·5 m .
1= -8% = 40/, o per sem1-annua
. I
33. The position of an object as a
2
30. What is the simplified equivalent function of time is described by x = 4e + 36. A car travels 100 miles to city A in 2 n = 10 years = 20 semi.-annuals
expression of (cof 9)(sin 2 9) = 1/csc2 e. 2t2 - t + 3. What is the acceleration of the hours, then travels 200 miles to city 8 in
A.1 * object at t = 2? 3 hours. What is the average speed of S = P(1 + i)"
B. 0 A . 52* the car for the trip? 10,000 = P(1 + 0 .04)
20
C. cose B. 64 A. 45 mph p =4563.87
2
D. sin 9 c. 56 B. 58.3 mph
D. 66 C. 66.7 mph 40. The quantity of a certain commodity
1
2 D. 60 mph* that is offered for sale at a certain price at
(cof6)(sin 6) + - - 3
Distance, x = 4t + 2t - t + 3 2
cos 2 e a given place a~d time:
Velocity , x' = 4(3)f + 2(2)t -1 t:,x A. stocks
= -t.l
=( c~s: e)sin2 e+ sin 2 e
Acceleration, x" = 4(3)(2)t + 2(2) Average speed, t.V
B. demand
s1n e = 4(3)(2)(2) + 2(2) 100+200 C. goods
=52
= cos 2
e + sin e
2 2+3 D. supply *
=1 34. The diameter of steel rod is 76.2 mm. = 60 mph
41 . The place where buyers and sellers
Determine the elongation of the rod per
37. How long it will take a sum of money exchange goods:
31. How far will an object in a standard meter length. The force applied is 8000
to double at a 5% annual interest A. demand
gravitational field drop in 13 sec, starting kN.
from rest and neglecting air friction? percentage rate? B. market*
A. 8.48. mm* C. business
A. 2721 ft" B. 12.85 mm A. 14.2 years*
D. buy and sell section
B. 2595 ft C. 9.35 mm B. 18.4 years
C. 2661 ft D. 1.5.65 rpm C. 15.9 years
D. 2275 ft 42. The amount of P50,000 is deposited
D. 19.3 years
in a bank. How much money are you
v, = 0
F = 8000 kN =8000 x 103 N going to withdraw after 8 years at 8%
E = 30,000,000 psi for steel S = P(1 + i)"
s = v,t +% gt2 compounded annually?
= 206,786 X 106 Pa 2P = P(1 + 0.05)"
= 0 +% (32.2)(13)2 A. P82,546.00
In 2 = n In 1.05
= 2720.9 ft n = 14.2 years
B. P92.546.00 *
y = £!:.. C. P85,456.00
AE 38. It is defined to be the capacity of a D. P97,856.00
32. The velocity of a particle at time t is:
4 3
v(t) = 12t = 7/t. What total distance is = (sooox Jo )0) commodity to satisfy human want:
S = P(1 + i)"
traveled between t = 0.2 and t = 0.3? A. discount •
A. 4.22 ~ (0.0762 f( 206,786xl o6) B. utility*
s = 50,000(1 + 0.08)8
B. 3.85 C. luxuries
s = 92,546.51
C. 3.25 Y = 8.48 x 10-3m= 8.48 mm D. necessity
43. These are products or services that
D. 2.84 *
35. The copper bar is 90 em long at 86°F. are desired by human and w ill be
39. Funds are deposited in a savings
What is the increase in its length when purchased if money is available after the
account at an interest rate of 8% per
V(t) = 12t + '!._
4
the bar is heated to 95°F? The linear annum compounded semi-annually. What requi red necessities have been obtained.
t
expansion coefficient for copper is 1.7 x is the initial amount that must be A. demand
10·5 per deg C. · ·
REFRESHER MANUAL 2nd Edition by JAS TORDILLO
REFRESHER MANUAL 2"d Edition by JAS TORDILLO
s- 14 DAY 8 - SOl-UTION
DAY 8 - SOLUTION Is -15
ENGINEERING MATHEMATHICS
B. utilities
C. necessities Capitalized cost = FC + OM depreciation for the second year using
D. luxuries • I 55. Thermodynamic properties, which
the sum-of-the-years digit method.
= 100,000 + 18,000 A. P17,672.00 * depend of the mass.
44. These are the products or services 0.10 B. P18,276.00 A. Extensive properties *
that are required to support human life = 280,000 C. P17,850.00 B. Intensive properties
and activities that will be purchased in D. P19,636.00 C. Specific properties
somewhat the same quantity even 47. A parent on the day the child is born D. Specific heat
through the price varies considerably: wishes to determine what lump sum 10
A. necessities • would you have to be paid into an SYD = - (10+1)=55 56. A classification of temperature and
B. producer goods 2 total volume as properties.
account bearing interest at 5%
C. luxuries compounded annually, in order to A. Intensive properties
Dep2 = (FC - SV) ( -n-1)
-
D. utilities withdraw P20,000 each on the child's SYD B. Extensive properties
18th, 19th, 20th and 21"1 birthdays. C. Specific properties
45. A car was bought at P549,492.13
with 14% down payment and the
A. P35,941.73
B. P30,941.73 •
= (120.000-12.000)c~~~) D. None of the above *

remaining balance will be paid on C. P33,941.73 = 17,672.72 57. That portion of the universe, an atom,
installment basis with a monthly payment D. P25,941.73 a certain quantity of matter, or a certain
of P12,000 for 60 months. Determine the 51. It is a quantity of substance that is volume in space that one wishes to
interest rate compounded annually. homogenous throughout. study.
A.19.56% * 2o,ooJ (1 + o.o5)4 -1 J A. State A. Property
B. 16.45%
c. 18.00%
p = 1o.os(1 + o.o5)4 B. Phase* B. System •
C. Fluid
(1+0.05)17' C. Datum
D. 14.35% D. Property D. State
p = 30,941 .73
52. The amount of energy required to 58. Descriptive characteristics used to
P=R [ (I+if - i ] 48. How much must you invest today in express the behavior of the system or
convert saturated liquid to saturated
l(l+i'fl order to accumulate P20,000 at 8% after substance.
vapor or from saturated vapor to
6 years?
472,563.23 = 12,000 [(I +i)liO -I]
i(l + ;)6°
A. P20,004.50
B. F?15,305.60
C. P.18,450.80
saturated liquid with the temperature that
remains constant.
A. Latent heat •
A. Datum
B. State
C. Properties *
B. Sensible heat · D. System
by trial & error, try i = 1.5% pe'r month D. P12,6d3.40 •
C. Specific heat
15 60
472,563.23 = 12,000 [ (I+ o.o ) -I ] S = P(1 + !)" D. Internal energy 59. A combination of processes taking a
system through a succession of states
O.o15(1 + 0.015)60 20,000 = P(1 + 0.08) 6
and returning to its initial state.
.; 53. A thermodynamic property that was
472,563.23 = 472,563.23 p = 12,603.39 A. Process
first introduced by Clausius in 1865 and
could be defmed as the measure of the B. Cycle*
Solving for the effective rate per year: 49. A bank offers 2% effective monthly C. State
irreversibility of the system or substance.
i = (1 + 0.015)12 -1 interest. What is the effective annual
A. Enthalpy D. Datum
i = 0.1956 interest rate?
A. 26.82% * -- ""' C:l B. Internal energy
= 19.56% per annum (compounded 1e net amount C. Entropy * 60. When two bodies, isolated from other
annually) B. 24.65% environment, are in thermal equilibrium
f the air in the D. Temperature
c. 25.28% with a third body, the two are in
•eriod?
46. An item is purchased for P100,000. D. 22.45% equilibrium with each other.
54. The measure of the unavailable
Annual cost is P18,000. Using 10%, what 12 energy in an irreversible process. A. First Law of Thermodynamics
is the capitalized cost of the perpetual i=(1+0.02) -1 B. Second Law of Thermodynamics
i = 26.82% A. Enthalpy
service? B. Internal energy C. Zeroth Law*
A. P220,000 D. Amagat's Law
50. An asset is purchased for P120,000 C. Entropy •
B. P265,000 D. Temperature
C. P250,000 and it can be sold for P12,000. Its
D. P280,000 * estimated life is 10 years. Find the REFRESHER MANUAL 2nd Edition by JAS TORDILLO
REFRESHER MANUAL2"d Edition by JAS TORDILLO
DAY 9- EXAM ~ 9-1
ENGINEERING MATHEMATHICS

DAY 9- EXAM A. tan e


B. sec e
1. Find the value of sine 750°
c. cos e
D. sin 9
A. -0.5
B. +0.342
7. A winch lifted a mass of 2600 kg
C. +0.5
through a height of 30 rn in 40 seconds. If
D. -0.342
the efficiency of the winch is 75%,
calculate the energy consumed in kw-hr.
2. The derivative with respect to y of the A. 0.2834
function f(y) = 2 ifY is: B. 0.3824
C. 0.3284
A. 3.y·2/3 D. 0.8324
3
B. 6y213 8. Cast iron weighs 490 lb per cu. Ft. The
C. 6y2 weight of a cast iron block 20" x 20" x 12"
-2/3 is"
D. _Y _ A. 1360 kg
3 B. 490 kg
c. 618 kg
3. If .§!. is the simple constant, what is the D. 250 kg
derivative of y = X2 ?
A. ax-x 9. Solve x for the given equation,
B. x to the a-1 power 9.2 x 10· 4 = e·9.2x .
C. ax A. 0.1122
D. ax to the a-1 power B. 0.7599
C. 0.3251
4. What is the angle between two vectors D. 0 .9577
A and B?
A = 2i + 1Oj + 4k 10. Log (x)"' 1 =
B = 20i - 4j + 4k A. log x
A. 85.98 deg B. n log x
B. 61.50 deg C. (n-1) log x
C. 74.49 deg D. x log (n- 1)
D. 42.25 deg
11. The path of a projectile is a:
5. g = 32.2 ft/sec2 . How is it expressed in A. ellipse
Sl? B. half circle
A. 9.81 m/sec2 C. parabola
B. 9.08 m/sec2 D. hyperbola
C. 9.86 m/sec2
D. 9.91 m/sec2 12. The equation 9x2+16y2+54x- 64y= -1

6. The expression l
[sec~I -cassin2e e)ta~ e
describes:
A. an ellipse
B. a hyperbola
c: a circle
D. a sphere
simplifies to:

REFRESHER MANUAL 2nd Edition by JAS TORDILLO


9-2 . DAY 9 - EXAM DAY 9- EXAM ~9-3
13. The first derivative with respect to y of 19. Water is pouring into a pool. After n ENGINEERING MATHEMATHICS
the function d(y) = zJIO is - - hours, there are 2n + Fn
gallons in the 26. Find the sum of even integers C. utilization factor
pool. At what rate is the water pouring between 1 to 31. D. efficiency
A.4JIO
B.O
=
into the pool when n 4 hours? A 210
A. o/. gph B.240 33. An ideal gas contained in a vessel of
C. .flO B. 1 gph C. 220 unknown volume at a pressure of 1 atm.
D. 6.32
c. 9/4 gph D. 242 The gas is released and allowed to
D. 7/4 gph expand into a previously evacuated bulb
14. In the Sl system of measurement, the 27. Find the sum of odd integers from 1 whose volume is 0.75 liter. Once
20. A winch lifted a mass of 2600 kg to 41 . equ ilibrium is has been reached, the
base unit for pressure is:
through a height of 30 m in 40 seconds. A 361 temperature remains .the same white the
A. Joule
Calculate the input power in kw if the B 431 pressure is recorded as 400 mm of Hg.
B. Watt
efficiency of the winch is 75%. C. 421 What is the unknown volume of the first
C. Newton
A. 19.9 D 441 bulb?
D. Pascal
B. 28.4 A. 0.661i
C. 25.5
15. Find the derivative of f(x) = [x to the 28. What is the interior angle (in radians) B. 0.831i
D. 35.2 of a dodecagon? C. 0.721i
2"d power- (x-1) to the 2"d power] to the
3rd power? A. 2.361 D. 0.951i
21. Find the 91h term of the progression: B. 2.9
A. 2x - 2(x-1)
B. 3[x to the 2"d power - (x-1) to the 2"d
3, 6, 12, 24 ... c 2.61 8 34. In a diesel engine, the duration
A. 768 D. 2.75 between the time of injection and the time
power][Jt - (x-1) 2 ]
B. 1536
c. 6[x to the 2"d power - (x-1) to the 2"d c. 945
of ignition is called:
power][x-(x-1 )]2 29. The no. of sides of a regu lar octagon A. explosion period
D. 3072 is: B. delay period
D. 6[x to the 2"d power - (x-1) to the 2"d
power] 2 A.6 C . period of ignition
22. Sin 2e is equal to: B. 8 D. P.re-ignition period
A. 2 CoseSine C. 7
16. W hat is the name for a vector that
represent the sum of the two vectors?
B. sine Cose D. 15 35. A condenser vacuum gage reads 710
A. moment
c. %sine 2 mm Hg when the barometer stands at
D. 14 - cos e 30. Which of the following is a prime 755 mm Hg. Determine the absolute
B. scalar
C. torque number? Pressure in the condenser.
D. resultant
23. Find the slope of the equation >t = y A.4 A. 5.2 kPa
when x = 1. B. 12 B. 5.9 kPa
A.2 c. a C. 5.6 kPa
17. An oblique equilateral parallelogram:
8.6 D. 17 D. 6.2 kPa
A. square
C.4
B. rhombus ,;
D. 1 31. Oil . flow through a 30 tubes 36. In the design of pulley, key and shaft.
C. rectangle
D. trapezoid condenser with a velocity of 1.75 m/s. A. Key is made the weaker link
24. A diagram which shows only the The internal diameter of tube is 20 mm B. Pulley is made weaker.
forces acting on the body. an oil density is 0.90 gm/ml. Find the C. Shaft is made weaker.
18. Box A has 4 white balls, 4 blue balls
A. free body diagram volume flow in liters per second. D. All three are designed for same length.
and 6 orange balls. Box B has 2 white
B. force triangle A. 16.5
balls, 6 blue balls and 6 orange balls. If
C. cash flow diagram B. 11.6 37. A solid disk flywheel weighing 10,000
one ball is drawn from each box, what is
D. motion diagram C. 15.6 kg and having a radius of gyration of 1.5
the probability that one of the two balls
will be white? D. 9.4 m , rotating with a speed of 90 rpm. What
25. Determine the curve : is the rotational KE?
A. 0.985
B. 0.437
9Jt + 6y2 + 2x + 3y + 9 =0. 32. The ratio of real power to apparent A. 666 kJ
A. ellipse power is: B. 888 kJ
C. 0.743
B. parabola A. power factor C. 777 kJ
D. 0.347
C. hyperbola B. load factor D. 999 kJ
D. circle
REFRESHER MANUAL 2nd Edition by JAS TORDILLO
REFRESHER MANUAL2"d Edition by JAS TORDILLO
9-4 DAY 9 - EXAM DAY 9- EXAM ~9-5
ENGINEERING MATHEMATHICS
38. The output of the shaft is 2,000 kw 44. Convert 30 mm of water into N/m .
and a fuel rate of 22.4 lbs/mn. What is A. 196
C. State B. State
the overall thermal efficiency of the B. 310
D. Temperature C. Properties
machine? (HHVoffuel is 18,000 Btu/lb) C. 294
D. System
A. 19.2% D.470
52. It is a branch of science that deals
B. 28.2%
with energy, it~ conversion from one form 58. Thermodynamic properties that are
c 22.4% 45. 100 Btu per minute, is how many kJ
to another, and the movement of energy dependent of the mass.
D 35.1% per minute. ·
from one location to another. A. Extensive properties
A. 105
A. Physics B. Intensive properties
39. If the outside diameter of a B.418
B. Heat transfer C. Specific properties
locomotive wheel minus its steel tire is C. 150
C. Thermodynamics D. All of the above
62.378 inches and the inside diameter of D.470
D. Hydraulics
the steel tire at 65°F is 62.263 inches, to
59. Is a physical system that does npt
what temperature (Fahrenheit) must the 46. The specific heat of ice at constant
53. It is the capacity of a system or interact or exchange energy with its
tire be heated to just fit the wheel? (k for pressure in Sl.
substance to do an effect. surroundings.
steel = 0.0000065 in per in-deg F). A. 4.18
A. Work A. isolated system
A. 325 B. 335
B. Torque B. control volume
B. 349 C. 2.09
C. Energy C. control space
C. 249 D. 144
D. Power D. isolated area
D. 225
47. Boiling temperature of water is
54. It is the product of the component of a 60. The focused volume in space from
40. If one end of the manometer is open nearest to:
force in the direction of motion, and the which the substance flows. Ex. Turbine,
to the atmosphere, it is called: A. 212 deg C
distance through which the point of pumps, heater, etc.
A. open manometer B. zero deg C
application of the force moves during its A. control volume
B. all of these C. 32 deg C
action. B. control space
C. closed manometer D. 100 deg C
A. Heat C. defined volume
D. differential manometer
B. Work D. moving elements
48. One kw is how many horsepower?
C. Energy
41. One day a celcius thermometer and a A. 0.736
D. All of the above
Fahrenheit thermometer registered B. 1000
exactly the same numerical value of the C. 0.746
55. When a certain mass of a fluid in a
temperature. What was the temperature D. 1.34 * DAY 9- ANSWER KEY
that day?· · particular state passes through a series 1. c 16.0 31. A 46. C
of processes and returns to its initial 2. A 17. B 32. A 47. 0
A. 35 49. A liter of water is how many grams?
state. 3. 0 18.0 33.8 48. 0
B. -40 A. 1
A. Cycle 4.A 19. c 34.0 49. C
C. -36 B. 4.187 5. A 20. c 35. 8 50.0
C. 1000 B. Process
D.40 6.A 21. A 36. A 51. C
C. State
D. 9.81 7. A 22.. A 37.0 52.(
42. In a steady flow system, the mass of D. Datum 8.( 23.A 38. 8 53. c
working substance is: 50. The specific gravity of the substance 9. B 24.A 39.8 54. 8
56. A system whose mass does not cross 10. C 25. A 40. A 55. A
A. always conserved is 1.05. Determine the specific weight of
the substance in lbs per cu. ft. Density of its boundaries. 11. C 26. B 41. 8 56. B
B. constant 42. A 57. C
water is 1000 kg/m 3 . A. Open system 12. A 27.0
C. not equal 13. 8 28. C 43. A 58. A
B. Closed system
D. infinity A. 42.4 14.0 29.8 44. C 59. A
C. Isolated system
B. 55.6 15.0 30.0 45. A 60. A
D. None of the above
43. Express one ton of refrigeration 1n c. 50.2
kcal/min: D. 65.5
57. Descriptive characteristics used to
A. 50.4
B. 14.7 51. Refers to the thermal condition of the express the behavior of the system or
C. 335 system or substance. substance.
D. 2.516 A. Process A. Datum
B. Datum REFRESHER MANUAL 2nd Edition by JAS TORDILLO
REFRESHER MANUAL 2"d Edition by JAS TORDILLO
9-6 DAY 9- SOLUTION
DAY 9. - SOLUTION 19-7
ENGINEERING MATHEMATHICS
= N + B~ - 2AB cos 9
DAY 9- SOLUTION (22.8)2 = (10.95)2 + (20.78)2 B. 490 kg
- 2(10.95)(20.78) cos e c. 618 kg* C. JIO
1. Find the value of sine 750°. e = 85.98° D. 250 kg D. 6.32
A. -0.5
B. +0.342 5. g = 32.2 ftlsec2 . How is Jt expressed in
c. +0.5 * Sl?
Weight = density x volume d(y) = zJIO = o
D. -0.342 A. 9.81 m/sec2 *
= 490 lb ( 20 20 .!2)tt3x ~
ft3 12 12 12 2.2 lb 14. In the Sl system of measurement, the
B. 9.08 m/sec2
Using calculator: sine 750° = +0.5 C. 9.86 m/se~ =618.6 kg base unit for pressure is:
A. Joule
D. 9.91 m/se~
2. The derivative with respect to y of the 9. Solve x for the given equation, B. Watt
9.2 X 10'4 =e' 9' 2x •
function f(y) = 2 ifY is:
C. Newton

[sec~1- sin2 a)tanel A. 0.1122 D. Pascal*


2 -2/3 * 6. The expression B. 0.7599 *
A .-y cose 15. Find the derivative of f(x) = [x to the
3 C. 0.3251
B. 6y213 D. 0.9577 2nd power - (x-1) to the 2nd power] to the
2 simplifies to: · 3'd power?
c. 6y
-2 /3 A. tan e • 9.2 X 10' 4 =e - 9·2x
A. 2x - 2(x-1)
D. _y_.- B. sece B. 3[x to the 2nd power- (x-1) to the 2nd
In 9.2 x 10·4 = -9.2x In e
3 C. cos a power][~- (x-1) 2 ]
X = 0.7599
C. 6[x to the 2nd power - (x-1) to the 2"d
f(y) = 2 ifY =z(y)113 D. sine
1 power][x-(x-1 )]2
10. Log (xt' =
2 D. 6[x to the 2"d power- (x-1) to the 2"d
2y 3
f(y)= - -
2
seca(l-sin e)tan a]=[~(cos 2
e)tan a]
A. log x
B. n log x
power] 2 *
3 [ cosa cose C. (n-1) log x *
f(x) = [xl - (x-1)2] 3
3. If g is the simple constant, what is the
D. x log (n-1)
f(x) = 3[~- (x-1 n [2x- 2(x-1))
f(x) = 3[x2- (x- 1) 2 ~2 [2x- 2x + 2))
derivative of y = x• ? = tan e Log (x)n., =(n - 1) log x f( x) = 6[~ - (x - 1)2]
A. ax-x
B. x to the a-1 power 7. A winch lifted a mass of 2600 kg 11. The path of a projectile is a:
16. What is the name for a vector that
C. ax through a height of 30 m in 40 seconds. If A. ellipse
represent the sum of the two vectors?
D. ax to the a-1 power " the efficiency of the winch is 75%, B. half circle A. moment
calculate the energy consumed in kw-hr. C. parabola *
B. scalar
y =~· A. 0.2834 * D. hyperbola
C. torque
y' = ax•·1 B. 0.3824 D. resultant*
C. 0.3284 12. The equation 9~+1 6y 2 +54x- 64y= -1
4. What is the angle between two vectors D. 0.8324 describes:
17. An oblique equilateral parallelogram:
A and B? A. an ellipse *
A. square
A = 2i + 1OJ + 4k B. a hyperbola
B. rhombus*
Efficiency= Wo = Po(t)
B = 20i - 4j + 4k C. a circle
~n ~n C. rectangle
A. 85.98 deg * D. a sphere D. trapezoid
B. 61.50 deg
C. 74.49 deg 26 hr 13. The first derivative with respect to y of
oo{9.81X3o)kw x (40sec)3 oosec 18. Box A has 4 white balls, 4 blue balls
- 40(1000) 6
D. 42.25 deg
0.75= VVjn the function d(y) = 2M is _ _. and 6 orange balls. Box B has 2 white
balls, 6 blue balls and 6 orange balls. If
A= J22 + 102 + 42 =10.95 I W1n = 0.2834 kw-hr A.4M one ball is drawn from each box, what is
B. 0 * the probability th.at one of the two balls
B= bo2 + (- 4J2 + 42 = 20.78 8. Cast iron weighs 490 lb per cu. Ft. The
weight of a cast iron block 20" x 20" x 12" 0
will be white?
A. 0.985
x= J(zo-if+(-4-10)1 +(4-4}2 = 22.8 is: REFRESHER MANUAL 2 nd Edition by JAS TORDILLO
0 A. 1360 kg
REFRESHER MANUAL 2"d Edition by JAS TORDILLO
"' -'"~

9-8 DAY 9 - SOLUTION


DAY 9 - SOLUTION 19- 9
ENGINEERING MATHEMATHICS
B. 0.437 21. Find the ?Jf' term of the progression:
C. 0.743 3, 6, 12. 24 ... d = 2 (even) a=2 L = 30 30. Which of the following is a prime
D. 0.347" A. 768" number?
B. 1536 L =a+ (n -1)d A4
First draw white, second draw not white: C. 945 30 = 2 + (n- 1)2 B. 12
4 12 48 D. 3072 n = 15 C.8
P1 : - X- = -
14 [4 196 0.17*
6
r= -=2 a=3 n=9
S= ~(a+L)=.!2(2+30)
First draw not white, second draw is
2 2
3 s = 240 Prime number "' is any positive integer
white: 1 1
L =ar"" = 3(2t = 768 that is not one( 1), has a factor of 1 and
10 2 20 the number itself.
P2 = - X - = - 27. Find the sum of odd integers from 1
14 14 196 22. Sin 2e is equal to: to 41.
A. 2 Cos9Sin9 " A 361 The prime number in the choices is only
Total Probability = P1 + P2 B. sine Cose B.431 17.
48 20 68 C. %sine C. 421
= -+- = -
196 196 196 D. 14-cos2 e D. 441 * 31. Oil flow through a 30 tubes
= 0.347 condenser with a velocity of 1, 75 m/s.
Sin= 2e = 2 CoseSine, 1,3,5,7, .. ....... 39,41 The internal diameter of tube is 20 mm
19. Water is pouring into a pool. After n an oil density is 0.90 gm/ml. Find the
23. Find the slope of the equation X: =y volume flow in liters per second.
hours, there are 2n + Fn
gallons in the when x = 1.
a=1 d = 2(odd) L=41
A.16.5 *
pool. At what rate is the water pouring A.2" ' B. 11.6
L =a+ (n- 1)d
into the pool when n =4 hours? B.6 C. 15.6
41"'1+(n-1)2
A % gph C.4 D. 9.4
n = 21
B. 1 gph D. 1
c. 9/4 gph" S = ~(a+L)=~(1+41) Q=AV
D. 7/4 gph Y= x2 2 2
s =441 Q= 2: (0.020)2(1.75)
y = 2x =2(1) =2 4
Q = 2n + Fn 28. What is the interior angle (in radians) . -4 m3 1000 li
24. A diagram which shows only the Q = 5.4977 x 10 - x -- x 30 tubes
Q' = 2 + _ 1_ = 2+-1-= 2. forces acting on the body. of a dodecagon? sec m3
2-ln 2J4 4 A. free body diagram * A. 2.361 Q"' 16.5 li/sec
B. force triangle B. 2.9
20. A winch lifted a mass of 2600 kg C. cash flow diagram c. 2.618 * 32. The ratio of real power to apparent
through a height of 30 m in 40 seconds. D. motion diagram D. 2.75 power is:
Calculate the input power in kw if the A. power factor "
efficiency of the winch is 75%. 25. Determine the curve : n = 12 (dodecagon) B. load factor
A. 19.9 9-:t: + 6j + 2x + 3y + 9 = 0. C. utilization factor
B. 28.4 A. ellipse" e= (n - 2)180 = (12-2)180 D. efficiency
c. 25.5" B. parabola n 12
D. 35.2 C. hyperbola e = 150 deg = 2.618 rad 33. An ideal gas contained in a vessel of
D. circle unknown volume at a pressure of 1 atm.
Efficiency = Po 29. The no. of sides of a regular octagon I The gas is released and allowed to
· Pin 26. Find the sum of even integers is: expand into a previously evacuated bulb
between 1 to 31. A6 whose volume is 0.75 liter. Once
2600(9.81X3o)kw
A. 210 B. 8 * equilibrium is has been reached, the
0.75 = 40(1000) B. 240" C. 7 temperature remains the same while the
Pin C.220 D. 15 pressure is recorded as 400 mm of Hg.
W1n = 25.506 kw 0 . 242 What is the unknown volume of the first
bulb?
. 2,4,6, ......... .. ..... .... 28, 30
REFRESHER MANUAL 2nd Edition by JAS TORDILLO
REFRESHER MANUAL 2"d Edition by JAS TORDILLO
DAY 9 - SOLUTION 19 -11
9 - 10 DAY 9 - SOLUTION ENGINEERING MATHEMATHICS
A. 0.661i C. 777 kJ
B. 0.83 li • D. 999 kJ * 41. One day a Celcius thermometer and 45. 100 Btu per minute, is how many kJ
C. 0.721i a Fahrenheit thermometer registered per minute.
D. 0.951i w = 90 rpm = 9.424 rad/sec exactly the same numerical value of the A. 105 *
temperature. What was the temperature B.418
V2=0.75+V, KE = ~ mk2 (w)2 that day? C. 150
=
P2 400 mm Hg =0.5263 atm KE = ~ (10,000)(1.5)2 (9.424)2 A. 35 0 . 470
KE = 999,132 J = 999.1 kJ B. -40 *
P,V, =PN2 C. -36
100 Btu x 1.055 kJ =105.5 kJ/min
(1)V, = (0.5263)(0.75 + V,) 38. The output of the shaft is 2,000 kw D. 40 min Btu
V1 = 0.39474 + 0.5263V, and a fuel rate of 22.4 lbs/mn. What is
0.4737V,:: 0.39474 the overall thermal efficiency of the OF :: oc 46. \he specific heat of ice at constant
v,:: 0.8331i machine? (HHV of fuel is 18,000 Btu/lb)
°F ::; .2. °C + 32 pressure in Sl.
A. 19.2% 5 A. 4.18
34. ·In a diesel engine, the duration B. 28.2% * B. 335
between the time of injection and the time c. 22.4% oc = 9 0c + 160 c. 2.09 *
of ignition is called: D. 35.1% 5 D. 144
A. explosion period 4°C::; -160
B. delay period Po= 2,000 kw °C:: -40 = °F Specific heat of ice
C. period of ignition P;o = mrOh = 22.4(18,000)
D. pre-ignition period • = 403 200 Btu x 1.055 kJ x ~ . 42. In a steady flow system, the mass of = '2.093 ~=0.5~
' min Btu 60sec working substance is: kg-°C lb - °F
35. A condenser vacuum gage reads 710 P;0 = 7089.6 kw A. always conserved *
mm Hg when the barometer stands at B. constant 47. Boiling temperature of water is
755 mm Hg. Determine the absolute p 2000 C. not equal nearest to:
Pressure in the condenser. Efficiency = p.- 7089.6
0- -- D. infinity A. 212 deg C
A. 5.2 kPa 10 B. zero deg C
B. 5.9 kPa * = 0.2821 = 28.2% 43. Express one ton of refrigeration in C. 32 deg C
C 5.6 kPa kcal/min: 0 . .100 deg C *
D. 6.2 kPa 39. If the outside diameter of a A. 50.4 *
locomotive wheel minus its steel tire is B. 14.7 48. One kw is how many horsepower?
P(abs) = P(g) + P(atm) 62.378 inches and the inside diameter of C. 335 A. 0.736
the steel tire at 65°F is 62.263 inches, to D. 2.516 B. 1000
P(abs) = -710 + 7-55 what temperature (Fahrenheit) must the c. 0.746
: 45 mm Hg x 101.325 kPa tire be heated to just fit the wheel? (k for One ton of refrigeration D. 1.34 *
760 mmHg steel = 0.0000065 in per in-deg F). = 200 Btu x 0.252 kcal = 50 .4 kcal
A. 325 min Btu min 0.746 kw = 1 hp
P(abs) = 5.99 kPa
B. 349 * 1
2 kw = - -hp = 1.34 hp
C.249 44. Convert 30 mm of water into N/m 0.746
36. In the design of pulley, key and shaft.
D.225 A. 196
A. Key is made the weaker link *
B. Pulley is made weaker. B. 310 49. A liter of water is how many grams?
Y = kl(T2- T,) c. 294 * A.1
C. Shaft is made weaker.
62.378 - 62.263:: D. 470 B. 4.187
D. All three are designed for same length.
(0.0000065J(62.263)(T2- 65) c. 1000 *
T2 = 349.1 F Weight density of water, D. 9.81
37. A solid disk flywheel weighing 10,000
kg and having a radius of gyration of 1.5 V = 9.81 kN/m 3 = 9810 N/m3 1 liter= 1 kg= 1000 grams
40. If one end of ttle manometer is open
m, rotating with a speed of 90 rpm. What
to the atmosphere, it is called: P = yh = 9810(0.030) = 294.3 N/m
2
50. The specific gravity of the substance
is the rotational KE?
A. open manometer* is 1.05. Determine the specific weight of
A. 666 kJ
B. all of these
B. 888 kJ
C. closed manometer
D. differential manometer REFRESHER MANUAL 2nd Edition by JAS TORDILLO
REFRESHER MANUAL 2"d Edition by JAS TORDILLO
9-12 DAY 9- SOLUTION
DAY 10- EXAM 10-1
ENGINEERING MATHEMATHICS
I
the substance in lbs per cu. ft. Density of 56. A system whose mass does not cross
3 its boundaries. B. 55
water is 1000 kg/m .
A. Open system
DAY 10- EXAM c. 53
A. 42.4
B. 55.6 B. Closed system * D. 50
1. Which of the following is a binominal?
c. 50.2 C. Isolated system
A. 3x 8. What do you call a polygon with 11
D. 65.5 * D. None of the above
B. (X+ y)'l>. sides?
C. 3x + 2y + 5z A. undecagon
Specific weight (substance) 57. Descriptive characteristics used to
express the behavior of the system or D. 7x + 9y + z B. pentedecagon
= S. G. x specific weight (water) C. dodecagon
= 1.05 (62.4) substance.
2. A real number does not include a D. hexadecagon
A. Datum
= 65.52 .!!:_ B. State
fractional part is:
ft3 A. irrational 9. In finding the distance between two
C. Properties *
51. Refers to the thermal condition of the B. integer points P,(x,, y,) and P2(><2, Y2), the most
D. System
system or substance. C. rational direct procedure ls to use:
A. Process 58. Thermodynamic properties that are D. surd A. the translation of the axes
B. Datum dependent of the mass. B. the derivative
C. State"' 3. A fraction with a numerator that is I C. the pythagorean theorem
A. Extensive properties *
D. Temperature greater than the denominator is: D. the slope of the line
B. Intensive properties
A. proper fraction
C. Specific properties
52. It is a branch of science that deals
D. All of the above
B. improper fraction 10. One root of x3 - 8x -3 = 0 is:
with energy, its conversion from one form C. geometric mean A. 2
to another, and the movement of energy 59. Is a physical system that does not D. terminating fraction B.4
from one location to another. interact or exchange energy with its C. 3
A. Physics 4. In quadratic formula, if b2 - 4ac is D. 5
surroundings.
B. Heat transfer greater than one, the roots are:
A. isolated system *
C. Thermodynamics * B. control volume A. real and equal 11. The value of Tan (A + B), where
D. Hydraulics C. control space
B. real and unequal tan A= 1/3 and Tan B = Y. is:
D. isolated area
C. real and imaginary A. 7/12
53. It is the capacity of a system or D. imaginary B. J/11
substance to do an effect. 60. The focused volume in space from c. 1/11
A. Work which the substance flows. Ex. Turbine, 5. Factor the expression: 6l- 2
y - 15. D. 7/13
B. Torque A. (3/ + 5)(2y2- 3)
pumps, heater, etc.
C. Energy* B. (2y2- 5)(3y2 - 3) 12. If a right circular cone is cut parallel
A. control volume *
D. Power C. (3y2- 5)(2y2 + 3) with the axis of symmetry, you would
B. control space
C. defined volume
D. (2y2 + 5)(3f - 3) reveal:
54. It is the product of the component of a D. moving elements A. circle
force in the direction of motion, and the 6. To find the angles of a triangle, given B. ellipse
distance through which the point of only the lengths of the sides, one would C. hyperbola
application of the force moves during its use: D. parabola
action. A. the taw of cosine
A. Heat B. the taw of sines 13. If log. 10 = 0.250, log,o a equals:
B. Work* C. the taw of tangents A.4
C. Energy D. orthogonal functions B.2
D. All of the above C. 0.50
55. When a certain mass of a fluid in a 7. Study the series of numbers to D. 10,000
particular state passes through a series discover the SYSTEM in which they are
of processes and returns to its initial arranged. For the series 1, 5, 14, 30, _ , 14. If the first derivative of the equation of
state. 91 ; the fifth term is: a curve is a constant, the curve is a:
A. Cycle * A. 59 A. circle
B. Process
C. State REFRESHER MANUAL 2nd Edition by JAS TORDILLO
D. Datum
REFRESHER MANUAL 2"d Edition by JAS TORDILLO
10-2 DAY 10 - EXAM
DAY 10- EXAM 110-3
ENGINEERING MATHEMATHICS
B. parabola 20. An asset is purchased for
C. hyperbola P500,000.00. The salvage value in 25 25. A merchant puts in his P2000.00 to 30. Find the nominal rate which if
D. straigh\ line years is P100,000.00. What are the small business for a period of six years. converted quarterly could be used
depreciation in the first three years using With a given interest rate on the instead of 12% compounded semi-
15. Angle between 90 deg and 180 deg straight line method? investment of 15% per year, annually.
has: A. P48,000.00 compounded annually, how much will he A. 11.82%
A . a positive sine and cosine B. P32,000.00 collect at the end of the sixth year? B. 9.38%
B. a negative cotangent and cosecant C. P24,000.00 A. P4,400.00 C. 10.7%
C. a negative secant and tangent D. P16,000.00 B. P4,200.00 D. 8.46%
D. all its trigonometric functions negative C. P4,390.00
21. A house and lot can be acquired a D. P4,626.00 31. Fifteen years ago P1000 was
16. A bank charges 12% simple interest dow"n payment of P500,000 and yearly deposited in a bank account, and today it
on a P300 loan. How much will be repaid payment of P100,000.00 at the end of 26. A deposit of P1000 is made in a bank is worth P2370. The bank pays interest
if the loan is paid back in ·one lump sum .each year for a period of 10 years, account that pays 8% interest semi-annually. What was the interest rate
after three years? starting at the end of 5 years from the compounded annually. Approximately paid on this account?
A. P408 date of purchase. If money is worth 14% how much money will be in the account A .4.9%
B.P551 compounded annually, what is the cash after 10 years? B. 5.0%
c. P415 I price of the property? A. P2160 c. 5.9%
D. P450 A. P810,100 B. P1060 0.3.8%
B. P801 ,900 C. P2345
17. If P5000 shall accumulate for 10 C. P808,835 D.P1925 32. A product has a current selling price
years at 8% compounded quarterly. Find D. P805,902 of P325.00. If its selling price is expected
the compounded interest at the end of 10 27. An interest rate is quoted as being to decline at the rate of 10% per annum
years? 22. How much must be deposited at 6% 7%% compounded quarterly. What is the because of obsolescence, what will be its
A P6,000.00 each year beginning on January 1, year 1 effective annual interest rate? selling price four years hence?
B. P6,040.20 in order to accumulate P5000.00 on the A . 21 .81% A. P213.23
C. P6,004 30 date of the last deposit, January 1, year B. 7.22% B. P302.75
D. P6,010.50 6? C. 7.71% C. P202.75
A. P751 D. 15.7% D. P156.00
18. An asset is purchased for P9000. Its B. P715
estimated life is 10 years, after which it C. P717 28. When will an investment of P5,000 33. PSOOO.OO is borrowed for 75 days at
will be sold for P1000.00. Find the book D. P725 double if the effective rate is 8% per 16% per annum simple interest. How
value during the third year if the sum-of- annum? much will be due at the end of 75 days?
the-years digit (SOYD) depreciation is 23. P1500 was deposited in a bank A. 6.2 years A. P2,750.20
used. account, 20 years ago. Today it is worth B. 8.40 years B. P5,666.67
A. P6100.00 P3000.00. Interest paid on the account is C. 7.45 years C. P5, 166.66
B. P5072.00 semi-annua lly. Determine the interest D. 9.01 years D. P6,666.67
C. P4500.00 rate paid on th is account?
D. P4800.00 A. 3% 29. A steel drum manufacturer incurs a 34. Mr. Robles plans to deposit PSOO.OO
B. 3.5% yearly fixed operating cost of at the end of each month for 10 years at
19. A sum of P1000.00 is invested now C.2.9% P200,000.00. Each drum manufactured 12% annual interest, compounded
and left for eight years, at which time the D.4% cost P160.00 to produce and .sells for monthly. The amount that will be
principal is withdrawn. The interest that P200.00 What is the manufacturer's available in two years is:
has accrued is left for another eight 24. If the nominal interest rate is 3%. break-even sales volume in drums per A. P13,987
years. If the effective annual interest rate How much is P5000.00 worth in 10 years year? B. P13,875
is 5%, what will be the withdrawal amount continuously compounded account? A. 1250 C. P13,678
at the end of the 16th year? A. P5750.00 B. 1000 D. P13,487
A. P706 B. P7500.00 C.2500
B. P774 C. P6750.00 D. 5000
C. P500 D. P6350.00
D. P799
REFRESHER MANUAL 2nd Edition by JAS TORDILLO

REFRESHER MANUAL 2"d Edition by ~AS TORDILLO


. DAY 10- EXAM 110-5
10-4 DAY 10 - EXAM ENGINEERING MATHEMATHICS .
35. A machine costs P20,000 today. If The ore of a gold mine in Mountain
Province contains on the average, 1
A. 1.3 sec C. 500 lbs
inflation is 6% per year and interest is B. 3.5 sec
ounce of gold per ton of ore and D. 1000 lbs
10% per year, what will be the C. 2.1 sec
approximate future value of the machine, processing cost is P1800 per ton and
D. 5.5 sec 51. The work done in pushing a fluid .
adjusted for inflation, in 5 years? recovers 85% of the gold. God is sold at
P3,000 per ounce. across a boundary, usually into or out of
A P36,300 47. Two cars A and B are traveling along the system is known as:
B P42,000 the same route. Car A is traveling at 4.5 A. Internal energy
C. P28,500 41 . Determine the profit per ton of ore in
m/s and has a mass of 1150 kg. Car B is B. Flow energy
D. P43,100 Diwalwal gold mine.
traveling at 6. 7 m/s and has a mass of C. Work due to surface tension
A. P240 1300 kg. If car B gently bumps into car A
B. P1,240 D. Kinetic energy
36. A quadrilateral whose opposite sides and their bumpers lock together, what will
are parallel: C . P850
be their common velocity? 52. It is defined as a process that once it
A. triangle D. P2,000
A.O has taken place can be reversed and in
B. parallelogram B. 11.33 m/s so doing leave no change in either
C. trapezoid 42. Determine the profit per ton of ore in
Mountain Province gold mine. C. 15.53 m/s system or surroundings.
D. median D. 5.67 m/s A. Irreversible
A . P750
B. P1250 B. Reversible process
37. In the Sl system of measurement, the
48. A body moves so that during the first C. Nonflow process
base unit for mass is: C. P900
part of its motion its distance S in inches D. Steady-flow process
A kg D. P1800
from the starting point . is given by the
B. joule expression: S = 6.8e - 10.8t (t in 53. The energy of a system undergoing
C. Newton 43. Which of the two gold mines is a
seconds). The acceleration in seconds process can be increased or decreased
D. pascal better processing method.
wou ld be at t = 3 seconds: by exchange with surroundings and
A. Diwalwal gold mine
B. Mountain Province gold mine A. 172.8 in/sec2 converted from one form to another
38. The mass of air in a room 3 x 5 x 20 B. 61.2 in/sec2
C. Either of the two is better within that system.
m is known to be 350 kg. Find its density C. 122.4 in/sec2
D. None of the two is better A. The above statement is incorrect
in kg/m 3. D. 22.4 in/sec2 B. The above statement is correct
A. 1.167 C. ·The above statement is incomplete
B. 1.617 44. In the design of a floor, it is found out
that the floor is supported on 2 inches x 8 49. A pendulum with a concentrated D. • Thete is no such condition for a
c. 1.176 mass (m) is suspended vertically inside a system
D. 1.716 inches guijo floor joist$ 0.3 m on centers.
If each joists is 12 ft (3.9 m ) long, using stationary railroad freight car by means of
21 pieces of guijo joists and cost P29 per a rigid weightless connecting rod. If the 54. ·It is a system where energy and mass
39. How many cubic feet is 100 gal of
board sheet, determine the cost of guijo connecting rod is pivoted where it cross its boundaries.
liquid? attaches to the box car, compute the
joists. A. Transient flow system
A. 74.80 angle that the rod makes with the vertical B. Closed system
B. 13.37 A. P6,720
as a result of a constant horizontal C. Open system
c 1.337 B. P9,744
acceleration of 2 ft/sec2 of the box car. D. Isolated system
D. 133.7 C. P8,955
A. 3.55 deg
D. P7,944
B. 6.33 deg 55. A substance existing in the gaseous
40. How many horsepower is 746 C. 4.66 deg
45. From a box containing 6 red balls, 8 phase but relatively near its saturation
kilowatt? D. 7.55 deg temperature is known as:
A. 500 hp white balls and 10 blue balls, one ball is
drawn at random. Determine the A. Gas
B. 1 hp 50. A 10 inches diameter pulley in belt B. Water
C. 100 hp probability that it is red or white.
driven with a net torque of 250 ft-lbs. The C. Steam
D. 1000 hp A.%
ratio of tensions in the tight to slack sides D. Vapor
B. 1/3
of the belt is 4 to 1. What is the maximum
Problems 41 to 43 C.7/12
tension in the belt? 56. The ones defining the physical
The ore of a gold mine in Diwalwal, I D. o/. A. 250 lbs condition of tlie fluid are called:
contains on the average, 1.2 ounce of
gold per ton of ore. The method of 46. A ball is dropped from a height of 60
B. BOO ibs A. Thermodynamic properties
processing cost P2000 per ton and meters above the ground. How long does REFRESHER MANUAL 2nd Edition by JAS TORDILLO
recovers 90% of the gold. it take to hit the ground?

REFRESHER MANUAL 2"d Edition by JAS TORDILLO


10-6 DAY 10 - EXAM DAY 10 - SOLUTION 110-7
B. State properties ENGINEERING MATHEMATHICS
C. Cycle properties (ri
D. Substance properties
) 7. Study the series of numbers to
DAY 10 - SOLUTION discover the SYSTEM in which they are
DAY 10- ANSWER KEY
57. A theory stating that it is impossible 1. B 16. A 31. C 46. 8 arranged. For the series 1, 5, 14, 30, _ ,
1. Which of the following is a binominal?
by any procedure, no matter how 2. B 17. B 32.A 47. 0 91; the fifth term is:
3. B 18.8 33. c 48. c A. 3x A. 59
idealized, to reduced any system to an
4. B 19. A 34. 0 49. A B. (x + y)'" * B. 55*
absolute zero temperature in finite 5. ( 20.A 35.0 50. 8 C. 3x + 2y + 5z c. 53
number of operations. 6. A 21. c 36.8 51.8 D. 7x + 9y + z D. 50
A. Nerst heat theory 7. B 22. c 37. A 52. B
B. Kinetic heat theory 8.A 23. B 38. A 53. B
C. Kelvin-Plank theory 9. c 24. c 39. B 54. C Binominal is an expression of two terms. 1 1, 5, 14, 30, 25;, 91
D. Clausius theory 10. C 25. D 40. 0 55. D
11. B 26. A 41. B 56. B 2. A real number does not include a 22 32 42 52 62
12. c 27. c 42.A 57.A fractional part is:
58. When there is no transfer of the 13. A 28. D 43. A 58. C A. irrational X = 30 + 52 = 30 + 25 = 55
working substance during the process, it 14. 0 29. D 44.8 59. B B. integer*
is termed as: 15. C 30. A 45.C 60. A
A. Transient flow ~ C. rational
D. surd
8. What do you call a polygon with 11
B. Steady-flow sides?
C. Nonflow A. undecagon *
D. Continuous flow 3. A fraction with a numerator that is B. pentedecagon
greater than the denominator is: C. dodecagon
A. proper fraction D. hexadecagon
59. An apparatus in which by an
B. improper fraction *
obstruction in its through-flow reduces
the pressure of the flow; or it is a device
C. geometric mean 9. In finding the distance between two
D. terminating fraction points P1(X1, Y1) and Pz(Xz, Yz), the most
that is used to reduce the pressure of the
fluid with the increase of its velocity. direct procedure is to use:
A. Condenser 4. In quadratic formula, if b2 - 4ac is A. the translation of the axes
B. Throttling device greater than one, the roots are: B. the derivative
C. Nozzle A. real and equal c : the pythagorean theorem *
D. Diffuser B. real and unequal * D. ihe slcpe of the line
C. real and imaginary
60. A n apparatus that condenses a D. imaginary 10. One root of x3 - 8x -3 = 0 is:
substance from its vapor phase to its 2 A.2
liquid phase by extracting heat from the 5. Factor the expression: 6l- y - 15. B. 4
substance. A. (3/ + 5)(2/- 3) c.
3*
A. Condenser B. (2/- 5)(3/ - 3) D. 5
B. Boiler c. (3y2- 5)(2y 2 + 3) *
2
C. Nozzle D. (2/ + 5)(3y - 3) Try 2: 23 - 8(2) - 3 = 0
D. Diffuser - 11< 0
Trial and Error Method:
2
6l-l - 15 = (3/- 5)(2y + 3) Try 3: 3 3 -8(3)- 3 = 0
0 =0
6. To find the angles of a triangle, given
only the lengths of the sides, one would
I Therefore, one of the root is 3.

use: 11. The value of Tan (A + B), where


A. the law of cosine * tan A= 1/3 and Tan B = Y. is:
B. the law of sines A. 7/12
C. the law of tangents B. 7/11 *
D. orthogonal functions c. 1/11
D. 7/13
REFRESHER MANUAL 2nd Edition by JAS TORDILLO
REFRESHER MANUAL 2"d Edition by JAS TORDILLO
---

10-s DAY 10- SOLUTION


DAY 10- SOLUTION 10-9
ENGINEERING MATHEMATHICS
I .
TanA+ TanB Therefore, there is a negative secant and
Tan (A+ B)= l-TanATanB tangent. years is P100,000.00. What are the
Dep, = (FC-SV) (-n-) depreciation in the first three years using
1 1 SYD
·+ 16. A bank charges 12% simple interest straight line method?
= 3 4_ on a P300 loan. How much will be repaid
(9ooo - loooG~) A. P48,000.00 *
1-H~J if the loan is paid back in one lump sum
after three years?
= B. P32,000.00
C. P24,000.00
4+3 A. P408 * :: P1454.54 D. P16,000.00
B.P551
= .J.i_ =..?.
12-1
12
11 C. P415
D. P450 Dept2l = (FC-SV) n-1)
(SYD-
FC-SV
Annual Dep :: - - -
n
.
12. If a right circular cone ts cut parallel
with the axis of symmetry, you would
IssS === 408.00
P(1+i}"
300[1 + 3(0.12)) = (9000-IOOOt~~
1
)
- 500,000-100,000
25
= 16,000
reveal: = P1309.09
A. circle 2
17. If P5000 shall accumulate for 10 Dept3l = (FC-SV) (n- ) Depreciation in the first 3 years
B. ellipse
years at 8% compounded quarterly. Find SYD = 3(16,000) = P48,000.00
c. hyperbola * the compounded interest at the end of 10
=(9000-1000{' 5~
D. parabola 0 2
years? ) 21 . A house and lot can be acquired a
A. P6,000.00 down payment of P500,000 and yearly
13. If log. 10 = 0.250, log,o a equals: = P1163.63
B. P6,040.20 * payment of P100,000.00 at the end of
A. 4*
C. P6,004.30 each year for a period of 10 years,
B.2 Book Value = FC -Total depreciation
D. P6,010.50 starting at the end of 5 years from the
c. 0.50 =9000- (1454.54 + 1309.09 + 1163.63) date of purchase. If money is worth 14%
D. 10,000 = P5072.74 compounded annually, what is the cash
i = S% = 2% per quarter price of the property?
Loa. 10 = 0.250 4
19. A sum of P1000.00 is invested now A. P810,100
a·0~5 = 10 n = 10(4) = 40 quarters and left for eight years, at which time the B. P801,900
a= 10,000 principal is withdrawn. The interest that C. P808,S35 *
S = P(1 +i}" has accrued is left for another eight D. P£05,902
log,o a= log,o (10,000} = 4 s = 5000(1 .+ 0.02)40 years. If the effective annual interest rate
s :: 11,040.198 is 5%, what will be the withdrawal amount P = cash price of the house and lot
14. If the first derivative of the equation of at the end of the 161h year?
a curve is a constant, the curve is a:
A. circle
Interest= S- P
:: 11,040.198-5,000
A. P706 *
_ vl
J
100,00J (1+0.14)10 - 1
0.14(1+0.14)10
B. P774
B. parabola = 6 ,040.198 C. P500
P- \4 + 500,000
C. hyperbola (1+0.14}
D. P799
D. straight line * 18. An asset is purchased for P9000. Its =
p 808,835.92
estimated life is 10 years, after which it S=P(1+i)"
15. Angle between 90 deg and 180 deg will be sold for P1000.00. Find the book s, = 1000(1 + 0.05)8 22. How much must be deposited at 6%
has: value during the third year if the sum-of- each year beginning on January 1, year 1
the-years ·digit (SOYD} depreciation is
s, = 1477.45 in order to accumulate P5000.00 on the
A. a positive sine and cosine
B. a negative cotangent and cosecant used. S2 = withd rawal amount at the end of the date of the last deposit, January 1, year
C. a negative secant and tangent* A. P6100.00 161h year 6?
D. all its trigonometric functions negative B. P5072.00 * s2 = (1477.45 - 1ooo)(1 + o.o5)8 A. P751
C. P4500.00 B. P715
D. P4800.00
s2 = 705.41 C. P717 *
Try 120° deg:
Sin 120° = 0.866 20. An asset is purchased for D. P725
Cos 120° = -0.5 P500,000.00. The salvage value in 25
Tan 120° = -1.732 SYD:: .!:(n+ l)=.!.Q(lO+ l)= 55
Sec 120° = -2
2 2
REFRESHER MANUAL 2nd Edition by JAS TORDILLO
REFRESHER MANUAL 2"d Edition by JAS TORDILLO
10-12 DAY 10 - SOLUTION
DAY 10 - SOLUTION 110-13
S= R[(1 + I]r- mass
Density = volume
ENGINEERING MATHEMATHICS

42. Determine the profit per ton of ore in


Mountain Province gold mine.
46. A ball is dropped from a height of 60
meters above the ground. How lon9 does
A. P750 *
= 500 [(1 + 0.01)24 -I] - 350
- 3(5X2o) B. P1250
it take to hit the ground?
A. 1.3 sec
0.01 C. P900 B. 3.5 sec*
= 1.167 kg/m3
s = 13,486.73 D. P1800 C. 2.1 sec
39. How many cubic feet is 100 gal of D. 5.5 sec
35. A machine costs P20,000 today. If liquid? Profit/ton of ore
inflation is 6% ·per year and interest is A. 74.80 = 1.0(0.85)(P3000)- P1800 S = V1t + Y:z gf
10% per year, what will be the B. 13.37 * = P750.00 60 = 0 + Y:z (9.81 )f
approximate Mure value of the machine, c. 1.337 t = 3.49 sec
adjusted for inflation, in 5 years? D. 133.7 43. Which of the two gold mines is a
A. P36,300 better processing method. 47. Two cars A and Bare traveling along
3
B. P42,000 100 gal x _ft_ = 13.368 tt3 A. Diwalwal gold mine* the same route. Car A is traveling at 4.5
C. P28,500 7.48 gal B. Mountain Province gold mine m/s and has a mass of 1150 kg . Car B is
D. P43,100 * C. Either of the two is better traveling at 6.7 m/s and has a mass of
40. How many horsepower is 746 D. None of the two is better 1300 kg. If car B gently bumps into car A
Value of machine without inflation: kilowatt? and their bumpers lock together, what will
=20,000(1 +0.10)5 A. 500 hp 44. In the design of a floor, it is found out be their common velocity?
= 32,210.2 B. 1 hp that the floor is supported on 2 inches x 8 A. 0
c. 100 hp inches guijo floor joists 0.3 m on centers. B. 11.33 m/s
Value of machine with inflation: D.1000 hp * If each joists is 12 ft (3.9 m ) long, using C. 15.53 m/s
= 32,210.2(1 + 0.06)5 21 pieces of guijo joists and cost P29 per D. 5.67 m/s *
= P43,104.51 =
1 hp 0.746 kw board sheet, determine the cost of guijo
1000 hp = 746 kw joists . m,V, + mN2 = (m1 + m2)Vf
36. A quadrilateral whose opposite sides A. P6,720 1150(4.5) + 1300(6.7) = (1150 + 1300)Vf
are parallel: ' Problems 41 to 43 B. P9,744 * Vf = 5.667 m/s
A. triangle The ore of a gold mine in Diwalwal, C. P8,955
B. parallelogram * contains on the average, 1.2 ounce of D. P7,944 48. ·A body moves so that during the first
C. trapezoid gold per ton of ore. The method of part of its motion its distance S in inches
D. median processing cost P2000 per ton and Cost = total bd. ft x price/bd. ft from the starting point is given by the
expression: S = 6.8tl - 10.8t (t in
1~
recovers 90% of the gold.
37. lp the Sl system of measurement, the
= ( 2x x12}1(P29) seconds). The acceleration in seconds
The ore of a gold mine in Mountain
base unit for mass is: Province contains on the average, 1 would be at t = 3 seconds:
= P9744
A. kg* ounce of gold per ton of ore and A. 172.8 in/sec2
B. joule processing cost is P1800 per ton and B. 61.2 in/sec2
45. From a box containing 6 red balls, 8
C. Newton recovers 85% of the gold. God is sold at C. 122.4 in/sec2 *
white balls and 10 blue balls, one ball is
D. pascal P3,000 per ounce. D. 22.4 in/sec2
drawn at random. Determine the
probability that it is red or white.
38. The mass of air in a room 3 x 5 x 20 41 . Determine the profit per ton of ore in Distance, s = 6.8e- 10.8t
A.Y.
m is known to be 350 k9. Find its density Diwalwal gold mine. B. 1/3
in kg/m3 . A. P240 Velocity, S' =6.8(3)f- 10.8
C.7/12 *
A.1.167* B. P1,240 * D.:Y.
B. 1.617 C. P850 Acceleration, S" = 6.8(3)(2)t
C. 1.176 D. P2,000 Prob = 6 + 8 = .!i = _2_ = 122.4 in/sec2
24 24 12
D. 1.716
Profit = Income - expenses 49. A pendulum with a concentrated
Profit/ton of ore mass (m) is suspended vertically inside a
= 1.2(0.90)(P3000)- P2000 stationary railroad freight car by means of
= P1,240.00 REFRESHER MANUAL 2nd Edition by JAS TORDillO
REFRESHER MANUAL2"d Edition by JAS TORDILLO
10-14 DAY 10 - SOLUTION
DAY 10 - SOLUTION 110-15
ENGINEERING MATHEMATHICS
a rigid weightless connecting rod. If the so doing leave no change in either
connecting rod is pivoted where it system or surroundings. 58. When there is no transfer of the
attaches to the box car, compute the A. Irreversible working substance during the process, it
angle that the rod makes with the vertical B. Reversible process * is termed as:
as a result of a constant horizontal C. Nonflow process A. Transient flow
acceleration of 2 ft/sec2 of the box car. D. Steady-flow process B. Steady-flow
A. 3.55 deg * C. Nonflow *
B. 6.33 deg 53. The energy of a system undergoing D. Continuous flow
C. 4.66 deg process can be increased or decreased
D. 7.55 deg by exchange with surroundings and 59. An apparatus in which by an
converted from one form to another obstruction in its through-flow reduces
within that system. the pressure of the flow; or it is a device
Tan 9 = £_
w A. The above statement is incorrect that is used to reduce the pressure of the
B. The above statement is correct* fluid with the increase of its velocity.
Tan 9 = rna
mg C. The above statement is incomplete A Condenser
D. There is no such condition for a B. Throttling device *
Tan 9 = ~ system C. Nozzle
9 D. Diffuser
Tan 9 = _2_ 54. It is a system where energy and mass
32.2 cross its boundaries. 60. An apparatus that condenses a
e = 3.55° A. Transient flow system substance from its vapor phase to its
B. Closed system liquid phase by extracting heat from the
50. A 10 inches diameter pulley in belt c. Open system * substance.
driven with a net torque of 250 ft-lbs. The D. Isolated system A. Condenser *
ratio of tensions in the tight to slack sides B. Boiler
of the belt is 4 to 1. What is the maximum 55. A substance existing in the gaseous C. Nozzle
tension in the belt? · phase but relatively near its saturation D. Diffuser
A. 250 lbs temperature is known as:
B. 800 lbs * A. Gas
C. 500 lbs B. Water
D. 1000 lbs C. Steam
D. Vapor*
£L =4 56. The ones defining the physical
~
condition of the fluid are called:
F2 = 0.25F, A. Thermodynamic properties
D B. State properties *
T=(F,-F2)-
2 C. Cycle properties
10 D. Substance properties
250(12) = (F,- 0.25F,)-
2
F1 = 800 lbs 57. A theory stating that it is impossible
by any procedure, no matter how
51. The work done in pushing a fluid idealized, to reduced any system to an
across a boundary, usually into or out of absolute zero temperature in finite
the system is known as: number of operations.
A. Internal energy A. Nerst heat theory *
B. Flow energy * B. Kinetic heat theory
C. Work due to surface tension C. Kelvin-Plank theory
D. Kinetic energy D. Clausius theory

52. It is defined as a process that once it


has taken place can be reversed and in REFRESHER MANUAL 2nd Edition by JAS TORDILLO
REFRESHER MANUAL 2nd Edition by JAS TORDILLO
DAY 11 - EXAM 11- 1 I
ENGINEERING MATHEMATHICS
voltage used by each if the three are
DAY 11 -EXAM connected in parallel to the terminals of a
12 volt battery.
1 A current of 10 amperes passing A. 12 volts
through a resistance of 5 ohms, what is B. 6 volts
the power in the resistor? C. 10 volts
A. 0.33 hp D. 4 volts
B. 0.57 hp
c 0.55 hp 7. A solid copper wire conductor at 50°C
D. 0.67 hp has the following characteristics.
Resistivity of 1.77 x 10·8 ohm-meter, Wire
2. What is the power required to transfer diameter of 0.4 inch and wire total length
110,000 coulombs of charge through a of 10,000 m. What is the resistance of the
potential rise of 24 volts in half hour? wire conductor?
A 5.5 kw A. 2.18 ohm
B. 2.2 kw B. 5.5 ohm
C. 4.5 kw C. 3.25 ohm
D.1.46kw D . .6.55 ohm

3. How long must a current of 15 8. A single phrase inductive load of 7500


amperes pass through a 10 ohm resistor kw has a lagging power factor of 0.80.
until a charge of 2505 coulombs passes Determine the reactive power of a
through? capacitor to be connected in parallel with
A. 167 min the load to raise the power factor to 0.90.
B. 334 min A. 1993 KVA
C. 2.783 min B. 1999 KVA
D. 3.789 min C. 1997 KVA
D. "2000 KVA
4. Ten coulombs of charge pass through
awire in 2 seconds. What is the average 9. An electric motor has a trade mark
current flowing? label indicating 2 hp, 240 V, 15 amp.
A. 5 amp Calculate the motor power factor.
B. 9 amp Assume t:notor efficiency to be 85% . .
C. 7 amp A. 0.987
D. 11 amp B. 0.657
C. 0.867
5. What is the equivalent resistance, in D. 0.487
ohms, of the circuit where there are 1 of
20 ohms and 1 of• 10 ohms connected in 10. A circuit has a resistance of 20 ohms
series followed by 3 resistors in parallel and the current flows at 12 amperes.
of 10 ohm each connected in series to Determine the voltage.
the circuit? A. 110 volts
A. 55.5 ohms B. 220 volts
B. 33.3 ohms C. 150 volts
C. 50.5 ohms D. 240 volts
D. 60.0 ohms

I11. What is the average power dissipated


6. Three coils of wire have_ resist~nce of by an electric heater with resistance of
5, 10 and 15 ohms, respectively. Find the 100 drawing a current of 30 sin (30t) A?

REFRESHER MANUAL 2nd Edition by JAS TORDILLO


~

11-2 DAY 11 -EXAM


DAY 11 - EXAM 111 -3
ENGINEERING MATHEMATHICS
A. 30 kw A. watt
B. 90 kw B. volt
C. 1200W 32. An ideal transformer has a primary
C. 60 kw C. ampere coils turns of 750 and V2 is 240 volts.
D. 100 kw D. none of these
D. ohm How many turns can a secondary coil
26. Are devices which serve to open or make if the primary voltage is 1200 volts?
12. A charge of 12 coulombs pass 19. The unit of electrical resistance is: A. 150 turns
closed the electrical circuit.
through a wire in 2 seconds. Determine A farad B. 200 turns
the average current flowing. A. plugs .
B. henry C. 140 turns
A12A B. outlets
C. ampere D. 120 turns
B. 6A C. switches
D. ohm
C. 9A D. receptacles
33. The ratio of power in watts and volt is:
D. 3A 20. The unit of electrical current flow is:
27. In a circuit, three resistors of 10, 15 A ampere
A ampere B. power factor
13. How long must a current of 2A pass and 20 ohms are connected in series.
B. weber C. load factor
Find the potential at the source if the
through a 20 ohm resistor until a charge C. coulomb D. coulombs
of 1800 coulombs pass through? current flowing is 4 amp?
D. volt
A 15 min A. 90V
B. 135 V 34. Which of the following rules does not
B. 9min 21. The ratio of real power to apparent apply to a parallel circuit?
C. 12 min power is: C. 180V
D.45V A. Total current flow is the sum of the
D. 6min A power factor current in each path.
B. load factor B. The applied voltage is the same
28. A 25 hp engine drives a DC
14. If three resistors of 5, 10, 15 ohms C. utilization factor across each path.
generator, if the generator has an
respectively are connected in parallel. D. plant factor C. There is more than one path for circuit
efficiency of 85%. How much does it
What is the equivalent resistance of the flow.
combination? deliver?
22. In a series circuit, the total resistance D. The total resistance is the sum of the
A. 30 ohms is: A. 21 HP
B. 25 HP individual resistors
B. 2.73 ohms A. the sum of reciprocals of all
C. 2.07 ohms resistances C. 24 HP
D. 29 HP 35. In an electric transformer the no. of
D. none of these B. the average of all resistances turns in the primary winding, and in the
C. the sum of all resistances
29. A battery is a group of cells secondary winding has a no. of turns 5
15. Three 100-ohms resistors are D. smaller than the smallest resistance times the no. of turns in the primary. If the
connected in series-parallel. What is the connected in:
A. series secondary voltage is 240 volts with a coil
equivalent resistance? 23. In a parallel circuit, the total resistance of 10 ohms. what is the
A. 300 ohms B. series-parallel
resistance is: current flowing through the secondary
B. 150 ohms A smaller than the smallest resistance
C. parallel
D. any of these coil?
C. 67 ohms B. the sum of all resistances A 24A
D. none of these C. larger than the largest resistance
30. The number of cycles per second of B. 18A
?· the ~eciprocal of all resistances
an AC voltage is known as: C . 12A
16. Electric current in a wire is a flow of: D. 26A
A atoms A frequency
24. In a parallel circuit with unequal
B. free electrons B. phase annie
resistance on each branch, 36. What total current is drawn by a
C. v alence electrons A the voltage drop across each branch is
C. waveform
D. alternation circuit composed of a 10 ohm resistor in
D. bound electrons the same series with two 10 ohm parallel
B. the current on each branch is the sme 31 . The ratio of real power to apparent combination. The supply voltage is 120
17. In a series circuit with resistors, the C. the total resistance is the sum of all R
power: V.
current is: D. none of these
A. power factor A.4A
A the same in each resistor
B. load factor B. 10A
B. largest in the smallest resistor 25. A toaster takes 10A from a 120 V
C. utilization factor C. 6A
C. largest in the largest resistor line. The power used is:
D. plant factor D. SA
D. different in each resistor A 1300 W
B. 120W
18. The unit of electrical pressure is: REFRESHER MANUAL 2nd Edition by JAS TORDILLO
REFRESHER MANUAL 2"d Edition by JAS TORDILLO
11-4 DAY 11 -EXAM
DAY 11 - SOLUTION
ENGINEERING MATHEMATHICS
Ill- 5

37. With a current of 5 amperes passing 43. What is the equivalent inductance, in
through a resistance of 10 ohms, what is henries, of the circuit where there are 1 of c. 12A
the dissipated power in the resistor? 15 H and 1 of 5 H inductors connected in D. 20A
DAY 11 - SOLUTION
A. 3.0 hp series followed by 2 inductors of 10 H
B. 330 hp 1. A current of 10 amperes passing
each connected in series to the circuit? 48. On a rotational surface area, how
c. 0.250 hp A. 25 H many mils in a 0.005 inch diameter?
through a resistance of 5 ohms, what is
D. 0.335 hp B. 5 H the power in the resistor?
A. 1000 mils
C. 20 H A. 0.33 hp
B. 5 mils
38. What is the average power dissipated D. 23.75 H B. 0.57 hp
C. 50 mils
by an electric heater with resistance of 50 C. 0.55 hp
D. 500 mils
drawing a current of 20 sin (30t) A? 44. Which of the statements is not true? D. 0.67 hp *
A. 10 kw In a simple parallel circuit with only one 49. Two coils connected in parallel to a 6 2
B. 14.14 kw active source and three resistive loads. Power= 12 R = (10) (5)
volt battery. One resistance is 17 and the
C. 20 kw Rsub1, Rsub2 and Rsub3. = 500 watts = 0.67 hp
other is 5. Find the current in each.
D.O A. the reciprocal of the equivalent A. 1.2 A & 0.353 A
2. What is the power required to transfer
resistance is the sum of the reciprocals of B. 1.1A & 0.453 'A
39. Six coulombs of charge pass through the individual resistance. 110,000 coulom bs of charge through a
C. 1.0A & 0.553 A
a wire in 2 seconds. W hat is the average potential rise of 24 volts in h alf hour?
B. The total current is the sum of the D. 0.9A & 0.653 A
current flowing? length currents. A. 5.5 kw
A:6A C. The equivalent resistance is the sum B. 2.2 kw
50. What is the resistance of the
B. 1.6A of the reciprocals of the individual secondary side if the turns ration is 20.1?
C. 4.5 kw
C. 3A resistance. Where the primary has a resistance of D. 1.46 kw *
D. 4.8A D. The voltage drop is the same across 2000 ohms?
all legs. Q =I xt
A.3
40. A circuit draws 5000 kVA w ith a 110,000 coulombs = I x 1800 sec
B 5
power factor of 0. 72. What size capacitor 45. A solid copper conductor at 20 deg C I= 61.11 amps
C. 7
is required to increase the power factor to has the following characteristics: D. y.
0.86? The 60 Hz line voltage is 220 V Resistivity = 1.77 x 10 to the -8th power Power= E I= 24 (61.11)
(rms). Diameter= 0.20 in = 1466.67 watts= 1.467 kw
A. 0.07 F Length = 5000 m frj
B. 0.04 F 3. How long must a current of 15
What is the resistance of the conductor?
C. 14 F A. 1'8.26 [,} DAY 11- ANSWER KEY amperes pass through a 10 ohm resistor
1.0 16.8 31. A 46. A until a charge of 2505 coulombs passes
D. 160 F B. 12.32 0 · • 2. 0 17. A 32. A 47. B
C. 0.01740 through?
3. c 18.8 33. A 48. 8
41 . How long must a current of 5 D. 4.37 0 34. D 49. A
A. 167 min
4.A 19.0
amperes pass through a 10 ohm resistor S. B 20. A 3S.A SO.B B. 334 min
until a charge of 1200 coulombs passes 46. In eiectrjcal circuits, the potential 6.A 21. A 36.0 C. 2.783 min*
through? difference across terminals of circuit 7.A 22. c 37.0 D. 3.789 m in
A. 3min 8.A 23. A 38. c
elements is called:
B. 4min 9.0 24. A 39. c Q =I xt
A. voltage
10. 0 25. C 40.A 2505 = 15 x time
C. 1 min B. current in amperes 11. B 26. C 41.8
D. 2 min C. resistance in ohms time = 167 sec = 2.783 min
12. B 27. C 42. c
D. power in watts 13. A 28. A 43. A
42. What is the power required to .transfer 14. B 29. A 44. c 4. Ten coulombs of charge pass through
97,000 coulombs of charge through a 45.0 a wire in 2 seconds. What is the average
47. A 20 volts source is connected to the ~I 15. B 30. A
current flowing?
potential rise of 50 volts in one hour? terminals A and 8 Rsub1. A 10 amp flows
A. 0.5 kw to Rsub1. If 30V source is connected in f:!.. 5 amp*
B. 0.9 kw terminals of A and B Rsub1. What is the B. 9 amp
C. 1.3 kw amount of current flow through the C. 7 amp
D. 2.85 kw Rsub1? D: 11 amp
A. 17A
B. 15A REFRESHER MANUAL 2nd Edition by JAS TORDILLO
REFRESHER MANUAL 2"d Edition by JAS TORDILLO
DAY "11 - SOLUTION ~ 11-7
11-6 DAY 11 - SOLUTION ENGINEERING MATHEMATHICS
Q =I X t
10=1x2 _ -
A- 1t (o.4l. n x25.4mm
---x - lm
- - )
2
A. 110 volts C. 2.07 ohms
I= 5 amps 4 in IOOOmm
B. 220 volts D. none of these
A = 0.081 X 10'3 m2 C. 150 volts
5 . What is the equivalent resistance, in
D. 240 volts * 1 I 1 I
ohms, of the circuit where there are 1 of -=-+-+-
20 ohms and 1 of 10 ohms connected in Resistance = 1. 77 x 10-a I 0000 RT R1 R2 R3
V= I R
series followed by 3 resistors in parallel 0.081 X 10-3 I I 1 I
V = 12 (20) = 240 volts -=-+-+-
of 10 ohm each connected in series to RT 5 10 15
the circuit? = 2.18 ohm
11. What is the average power dissipated Rr = 2. 73 ohms
A. 55.5 ohms
8. A single phrase inductive load of 7500 by an electric heater with resistance of
B. 33.3 ohms * 15. Three 100-ohms resistors are
kw has a lagging power factqr of 0.80. 100 drawing a current of 30 sin (30t) A?
C. 50.5 ohms connected in series-parallel. What is the
Detenmine the reactive power of a A. 30 kw
D. 60.0 ohms equivalent resistance?
capacitor to be connected in parallel with B. 90 kw*
Rr = R, + R2 + -..,.-- - - c ---,- the load to raise the power factor to 0.90. C. 60 kw A. 300 ohms
1 1 D. 100 kw B. 150 ohms*
+ + A. 1993 KVA *
RJ R4 R 5 B. 1999 KVA C. 67 ohms
2 2
C. 1997 'rWA Power= 1 R = (30) (100) D. none of these
Rr=20+10+ =33.3ohms = 90,000 w =90 kw
1 1 1 D. 2000 'rWA
+ +
10 10 10 R2R3
Cos e, = 0.80 12. A charge of 12 coulombs pass Rr= R , + R ·+RJ
-
2
6. Three coils of wire have resistance of e, = 36.87° through a wire in 2 seconds. Determine
100(100) =ISO ohms
5, 10 and 15 ohms, respectively. Find the the average current flowing.
= 100+ 100+100
voltage used by each if the three are Q, =7500 tan 36.87° =5625 kw A. 12A
connected in parallel to the tenminals of a B. 6A *
Cos e2 = 0.90 C. 9A 16. Electric current in a wire is a flow of:
12 volt battery.
e2 = 25.84° D. 3A A. atoms
A. 12 volts*
B. free electrons *
B. 6 volts
Q =I X t C. valence electrons
C. 10 volts 02 = 7500 tqn 25.84° = 3632 kw
12 =I X 2 D. bound electrons
D. 4 volts
Po~er (capacitor)= Q 1 - 02 I= 6 amps
= 5625-3632 17. In a series circuit with resistors, the
The voltage is equal to 12 volts the
= 1993 kw = 1993 kv 13. ~ow long must a current of 2A pass current is:
connection is parallel. In parallel
through a 20 ohm resistor until a charge A. the same in each resistor*
connection the voltage across are equal
9. An electric motor has a trade mark of 1800 coulombs pass through? B. largest in the smallest resistor
to the terminal.
label indicating 2 hp, 240 V, 15 amp. A. 15 min* C. largest in the largest resistor
Calculate the motor power factor. B. 9 min D. different in each resistor
7. A solid copper wire conductor at 50°C
has the following characteristics. Assume motor efficiency to be 85%. C. 12 min
D. 6 min 18. The unit of electrical pressure is:
Resistivity of 1.77 x 10·8 ohm-meter, Wire A. 0.987
B. 0.657 A. watt
diameter of 0.4 inch and wire total length B. volt *
of 10,000 m. What is the resistance of the C. 0.867 Q =I X t
D. 0.487 * 1800=2xt C. ampere
wire conductor?
time = 900 sec = 15 min D. ohm
A. 2.18 ohm*
B. 5.5 ohm Power Factor= cos e
C. 3.25 ohm 2hp 746w 14. If three resistors of 5, 10, 15 ohms 19. The unit of electrical resistance is:
D. 6.55 ohm
- X-- respectively are connected in parallel. A. farad
= .!:.. = 0.85 hp = 0.487 What is the equivalent resistance of the B. henry
Vi 240(15) combination? C. ampere
. L · D. ohm*
Res1stance = p- A. 30 ohms
A 10. A circuit has a resistance of 20 ohms B. 2. 73 ohms *
L = 10,000 m and the current flows at 12 amperes.
Determine the voltage. REFRESHER MANUAL 2nd Edition by JAS TORDILLO
REFRESHER MANUAL 2"d Edition by JAS TORDILLO
11-8 DAY 11 - SOLUTION
DAY 11 - SOLUTION
ENGINEERING MATHEMATHICS
Ill- 9
20. The unit of electrical current flow is: 27. In a circuit, three resistors of 10, 15
A. ampere* and 20 ohms are connected in series. c. 140 turns A.4A
B. weber Find the potential at the source if the D. 120 turns B. 10A
C coulomb current flowing is 4 amp? C.6A
D. volt A. 90V Vt NJ o. aA~
B. 135 V v2 = N2
21. The ratio of real power to apparent c. 180 v * R = IO(to) =sn
power is: D. 45V 1200 750
10+10
A. power factor* 240 = N2 X

Rr = 5 + 10 = 150
B. load factor Rr = 10 + 15 + 20 = 45 ohms N2 = 150 turns
C. utilization factor V=IRr
D. plant factor V = I Rr = 4 (45) = 180 volts 33. The ratio of power in watts and volt is: 120 = 1(15)
A. ampere* I = 8A
22. In a series circuit, the total resistance 28. A 25 hp engine drives a DC B. power factor
is: generator, if the generator has an C. load factor 37. With a current of 5 amperes passing
A. the sum of reciprocals of all efficiency of 85%. How much does it D. coulombs through a resistance of 10 ohms, what is
resistances deliver? the dissipated power in the resistor?
B. the average of all resistances A. 21 HP* 34. Which of the following rules does not A. 3.0 hp
C. the sum of all resistances * B. 25 HP apply to a parallel circuit? B. 330 hp
D. smaller than the smallest resistance C. 24 HP A. Total current flow is the sum of the c. 0.250 hp
D. 29 HP current in each path. D. 0.335 hp *
23. In a parallel circuit, the total B. The applied voltage is the same
resistance is: Generator Output= 25 x 0.84 = 21 HP across each path. Power= 12 R = (5)2(10)
A. smaller than the smallest C. There is more than one path for circuit = 250 watts = 0.335 hp
resistance* 29. A battery is a group of cells flow.
B. the sum of all resistances connected in: D. The total resistance is the sum of 38. What is the average power dissipated
C. larger than the largest resistance A. series* the individual resistors * by an electric heater with resistance of 50
D. the reciprocal of all resistances B. series-parallel drawing a current of 20 sin (3ot) A?
C. parallel 35. In an electric transformer the no. of A. 10 kw
24. In a parallel circuit with unequal D. any of these turns in the primary winding, and in the B. 14.14 kw
resistance on each branch, secondary winding has a no. of turhs 5 c. 20 kw*
A. the voltage drop across each 30.·The ()Umber of cycles per second of times the no. ofturns In the primary. lfthe D.O
branch is the same* an AC voltage is known as: secopdary voltage is 240 volts with a coil
B. the current on each branch is the sme A. frequency * resistance of 10 ohms, what is the 2
Power= 12 R = (20) (50)
C. the total resistance is the sum of all R B. phase angle current flowing through the secondary = 20,000 watts= 20 kw
D. none of these C. waveform coil?
D. alternation A. 24A * 39. Six coulombs of charge pass through
25. A toaster takes 1OA from a 120 V B. 18A a wire in 2 seconds. What is the average
line. The power used is: 31. The ratio of real power to apparent C. 12A current flowing?
A. 1300W power: D. 26A A.6A
B. 120 W A. power factor * B. 1.6A
C.1200W* B. load factor V=IR C.3A*
D. none of these C. utilization factor 240 = 1(10) D. 4.8A
D. plant factor 1=24A
Power= E I= 120 (10) = 1200 W Charge, Q = I x t
32. An ideal transformer has a primary 36. What total current is drawn by a 6= lx2
26. Are devices which serve to open or coils turns of 750 and V2 is 240 volts. circuit composed of a 10 ohm resistor in 1=3A
closed the electrical circuits. How many turns can a secondary coil series with two 10 ohm parallel
A. plugs make if the primary voltage is 1200 volts? combination. The supply voltage is 120 40. A circuit draws 5000 kVA with a
B. outlets A. 150 turns* v. power factor of 0.72. What size capacitor
C. switches • B. 200 turns
D. receptacles REFRESHER MANUAL 2nd Edition by JAS TORDILLO
REFRESHER MANUAL2"d Edition by JAS TORDILLO
11-10 DAY 11 -SOLUTION
DAY U - SOLUTION 111 -11
ENGINEERING MATHEMATHICS
is required to increase the power factor to 15 H and 1 of 5 H inductors connected in
0.86? The 60 Hz line voltage is 220 V series followed by 2 inductors of 10 H C. resistance in ohms 50. What is the resistance of fhe
(rms). each connected in series to the circuit? D. power in watts secondary side if the turns ration is 20.1?
A.0.07F* A. 25 H * Where the primary has a resistance of
B. 0.04 F B. 5 H 47. A 20 volts source is connected to the 2000 ohms?
C. 14 F C. 20 H terminals A and 8 Rsub 1. A 10 amp flows A. 3
D. 160 F D. 23.75 H to Rsub1 . If 30V sour~ is connected in B. 5 *
terminals of A and B Rsub1 . What is the c. 7
Cos e, = 0.72 (9 1 = 43.94°) R 3 = 10(10) = SH amount of current flow through the D. y.
= =
Cos 62 0.86 (62 30.68°) 10+ 10 Rsub1?
A. 17A
~ =(~J
Rtotal = R, + R2 + R3 2
P(real) = P(apparent) cos 9, =15+5+5 B. 15A * = a2 a = turns ratio
= (5000)(0.72) = 3600 kVA = 25H C. 12A Z2 N2
= 3.6 x 106 Watts D. 20A 2
44. W hich of the statements is not true? 2000 =(20) = (20)2
llP(reactive) = P(real) (tan 9 1 - tan 92J In a simple parallel circuit with only one P =VI= 20 (10) = 200 w z2 1
= (3.6 x 106 )(tan 43.94°- tan 30.68 ) active source and three res istive loads. Zz;; 5
= 1.3x 106 W Rsub 1, Rsub2 and Rsub3. P = 12 R
A the reciprocal of the equivalent 200 = 102 R
llP(reactive) = 2rrfC(VIIne)2 resistance is the sum of the reciprocals of R=20
1.3 X 106 = 2rr(6Q) C (220)2 the individual resistance.
C = 0.07 F B. The total cu rrent is the sum of the V = IR
length currents. 30 =I (20)
41 . How long must a current of 5 C. The equiva lent resistance- is the I= 15A
amperes pass through a 10 ohm resistor sum of the reciprocals of the
until a charge of 1200 coulombs passes individual resistance. * 48. On a rotational surface area, how
through? D. The voltage drop is the same across many mils in a 0.005 inch diameter?
A . 3min all legs. A. 1000 mils
B. 4min * B. 5 mils *
C. 1 min 45. A solid copper conductor at 20 deg C C. 50 mils
D. 2 min has the following characteristics: D. 500 mils
Resistivity = 1.77 x 10 to the -8111 power
Charge, Q = I x t
1200 = 5 X t
Diameter= 0.20 in
Length = 5000 m 0.00~ in x ~ =5 mils
t = 240 sec= 4 min 0.00 1in
What is the resistance of the conductor?
A 18.26 n
42. What is the power required to transfer B. 12.32 0 49. Two coils connected in parallel to a 6
97,000 coulombs of charge through a C. 0.0174 0 volt battery. One resistance is 17 and the
potential rise of 50 volts in one hour? o. 4.37 n * other is 5. Find the current in each.
A . 0.5 kw A. 1.2 A & 0.353 A *
B. 0.9 kw B. 1.1A & 0.453 A
c. 1.3 kw • R -- PAL = !.77xi0-8~ C. 1.0A & 0.553 A
D. 2.85 kw 7t ( 0.2 )2 D. 0.9A & 0.653 A
4 39.37
Coulombs = Amp x Sec = 4.36680 v = v, = v2= 6 volts (parallel)
97,000 =Amp X 3600 V 6
Amp = 26.944A I,= - 1 =-= 0.353A
46. In electrical circuits, the potential R1 17
Power= El = 50(26.944) difference across terminals of circuit
= 1,347 W = 1.347 kw v 6
elements is called: lz = - 2 =-= 1.2A
A. voltage* R2 5
43. What is the equivalent inductance, in B. current in amperes
henries, of the circuit where there are 1 of REFRESHER MANUAL 2nd Edition by JAS TORDILLO
REFRESHER MANUAL 2nd Edition by JAS TORDILLO
DAY 12- EXAM
ENGINEERING MATHEMATHICS
112-1
DAY 12- EXAM 7. An angle between 90 degree and 180
degree has:
A. A positive sine and cosine
1. To find the angles of a triangle, given B. a negative cotangent and cosecant
only the lengths of the sides, one would C. a negative secant and tangent
use:
D. all its trigonometric functions negative
A. the law of cosines
B. the law of sines
8. Solve for x in the equation:
C. the law of tangents 1/xl- 1/4a2 ;:: 0.
D. the inverse-square law A. +2, -2
B. +1,-1
2. The sine of 840 degree equals: C. +2a, -2a
A. -cos 30 degree D. +a, -a
B. sin 30 degree
C. -cos 60 degree 9. The dimensions of a covered box are
D. sine (2rr + 120 degree) such that it is 1 inch taller than it is wide
and 1 inch longer than it is tall. The sum
3. To cut a right circular cone in such a of the volume and surface area is 628.
way as to reveal a parabola, it must be What are the dimensions of the box in
cut: inches?
A. perpendicular to the axis of symmetry A. 6x 7x8
B. at any acute angle to the axis of B. 1 X 2 X 3
symmetry C. 2x 3x4
C. parallel to the axis of symmetry D. 2 X 4 X 6
D. none of these
10. Find the value of the seventeenth
4. Evaluate the limit x2 - 1/x·- 1, where x term of 68, 56, 44, 32, ...... 17th term .
approaches to one. A.41
A.2 B. 14
8. 0 c. -124
C. none of these D. -142
D. 1
314 11. Find the value of the eighth term of
5. If x ;:: 8. x equals: 1/243, 1/81, 1/27, ...... ..... 81hterm.
A.6 A 1/9
B. -9 B. 1/3
C. 9 C. 9
D. 16 D. 3
x314 =8 12. The identity sec2 A- sec2 A sin2 A is
x=8<113
equal to:
X: 16
A. sin A
B. sec A
6. If log. 10;:: 0.250, log1o x equals: C. zero
A. 4 D. one
B. 2
C. 1,000 13. Simplify the identity:
D. 0.50 (1 + tan2 A)/(1 + cot2 A)
A.1
REFRESHER MANUAL 2nd Edition by JAS TORDILLO
12-2 DAY 12 - EXAM
DAY 12- EXAM 112-3
ENGINEERING MATHEMATHICS
B. tan" A 20. Find the dy/dx of y =cos (2x- 3).
C. sin A A. sin (2x- 3) B. sum A 9.33kw
D. tan A B. 2 cos (2x- 3) C. resultant B. 10 kw
C. 2 sin (2x- 3) D. phase angle C. 5.97 kw
14. Find the value of sin 2A if sin A is D. -2 sin (2x -3) D. 12 kw
equal to 12/13 and angle A terminates is 27. Which of the following is not a vector
the first quadrant. 21. A wire 2 ft long is to be cut into two quantity? 32. What is the output torque of a 20-hp
A. 0.35 pieces. One piece is then used to form a A. velocity motor running at 1800 rpm?
B. 0.79 circle and the other piece is used to form B. acceleration A. 25.6 ft-lb
c. 0.71 a square. How much wire should be used C. speed B. 58.4 ft-lb
D. 1.10 to form the square if the sum of the areas D. displacement C. 42.8 ft-lb
enclosed by the circle and the square is
D. 85.4 ft-lb
15. The point (5, 2) is the midpoint of the to be a minimum? 28. Body A traveling to the right with a
segment o( a line connecting A(-3, -5) A.1.12ft velocity of 30 fps, collides with Body B, 33. A large flywheel, weighing 1288 lb,
and B(x, y). Find point B. B. 0.88 ft which is traveling to the left with a has a radius of gyration of 12 in.
A. 13,9 c. 1.00 ft velocity of 10 fps. Determine the velocity Determine its kinetic energy if it is
B. -2,7 D. 0.66 ft of body A after Impact ifWA = 300 lbs, Wa rotating with an angular velocity of 60
c . 5,-9 = 150 lbs, and eccentricity e = 0.80. rpm.
D. -1,-3 22. Evaluate the definite integral of xdx A. 40 fps to the left A. 789 ft-lb
range from -1 to 7. B. 18 fps to the right B. 1155 ft-lb
16. Find the slope of the line connecting A.6 C. 28 fps to the left c. 998 ft-lb
(4, y) and (-6, 13). B.24 D. 0 fps to the right D. 3512 ft-lb
A. 10/(y- 13) C.12
B. (y- 13)/10 D.30 29. A man weighing 180 lb jumps off a 34. What velocity will a weight attain if it
c. (13- y)/-2 clock onto a stationary boat weighing 350 falls a distance of 100 ft on the moon?
D. 13/2 23. Find the area of the region bounded lb. The horizontal distance component of (g = 5.47 ftls2)
by y = '1?!4 andy= x/2 + 2. the man as he leaves the dock is 10 fps. A. 80.1 fps
17. What is the equation of a line that A2 Assuming that he does not fall out on of B. 45.6 fps
passes to the point (-2, 1) and a slope of B. 15 the boat, find his velocity after landing. C. 70.1 fps
o/.? C. 9 A. 1.2 fps D. 33.1 fps
A 4x + 3y =2 D. 20 B. 5.6 fps
B. X+ 2y = -3 c. 3.4 fps 35. A cyclist leans toward the center of
C. 3x-4y = -10 24. A bag contains 3 white balls and 5
D. 7.2 fps curvature when maneuvering a turn.
D. 3x + 2y = -4 red balls. If two balls are drawn at
Determine the banking angle measured
random, find the probability that one ball
30. To overcome rolling resistance, a from the vertical, if the radius of the turn
18. Determine the equation of the line is white and the other Is red.
locomotive exerts a force of 20,000 lb is 30 ft and the velocity of the cycle is 15
that is perpendicular to the line segment A. 15/56 while pulling a train of cars over a level fps.
(-4, 2) and (1, -7) and passes through the B. y. road of 30 mph. Determine the A. 11.2 degree
point x = 4 andy= -3. C.15/28 horsepower that is developed by the B. 14.3 degree
A. 5x - 9y - 4 7 = 0 D. 225/784 engine. C. 13.1 degree
B. 2x + 3y + 1 = 0 A.200 D. 15.3 degree
C. 9x- 5y + 40 = 0 25. If the product of the slopes of any two B 1100
D.x+5y-10=0 straight lines is negative 1, one of these C. 600 36. A 10-lb weight, fastened to a 2-ft
lines are said to be: D. 1600 cord, is whirled in a vertical circle with an
19. What is the derivative of y = square A. parallel
angular velocity of 100 rpm. Determine
root of 7x? B. perpendicular 31. How many kilowatts of.electricity must the tension in the cord when the weight is
A.~ ~7) 'I? C. skew be supplied to run a motor that is rated at at the top of the circle.
B. 7x D. non-intersecting
10 hp of electrical energy is transformed A. 691b
C. 7x·~ into mechanical energy with an efficiency B. 58.11b
D. % (7)112 X-112 26. The vector which represents the sum
of 80%?
of a group of force vectors is called the:
A . magnitude REFRESHER MANUAL 2nd Edition by JAS TORDILLO
REFRESHER MANUAL 2"d Edition by JAS TORDILLO
12-4 DAY 12 -EXAM
DAY 12 - EXAM 112- 5
ENGINEERING MATHEMATHICS
C. 481b C. 12.5 fps
D. 77.11b D. 8.34 fps C. forces and motion C. Clausius statement
D. matter D. Nernst heat theorem
37. A passenger on an elevator weighs 43. One gram of ice at 0°C is placed on a
180 lb. What is the force between him container containing 2,000.00 cu. meter 49. Momentum is a properly related to 54. The third law of thermodynamics is
and the floor of the elevator if the elevator of water at 0°C. Assuming no heat loss, the object's _ _. also known as:
is accelerating upward at the rate of 5 what will happen? A. mass and acceleration A Clausius heat theorem
ft/s2? A. ice will melt and become water , B. motion and weight B. Joule's theorem
A 180 lb B. some part of ice will not change C. weight and velocity C. Kinetic theory of energy
8 . 2081b C. the volume of ice will not change D. motion and mass D. Nernst heat theorem
c. 195 lb D. water will become ice
D. 2801b 50. Assume three force vectors intersect 55. Properties that define the thermal and
44. What is the atmospheric pressure on at a single point. energy conditions of the fluid.
38. How many feet of water are a planet where the absolute pressure is F1= 2i + 7j- k A Specific properties
equivalent to 25 psi? 101 kPa and the gage is 9 kPa? F2 = -i + 4j + 2k B. State properties
A 57.7 ft A 11 0 kPa F3 = i + 3j + 4k C. Transport properties
8 . 77.1 ft B. 92 kPa What is the magnitude of the resultant D. Specific properties
C. 67.7 ft C. 105 kPa force vector?
D. 97.9 ft D. 88 kPa A. 15 56. Properties that measures the
B. 12 diffusion within the fluid resulting from
39. The gage on a condenser shows a 45. A flywheel of radius 18 inches is C. 14 molecular activities.
vacuum of 24 inches of mercury. What is rotating at the rate of 1500 rpm. How fast D. 16.6 A State properties
the absolute pressure in pounds per does a point on the rim travel in ft/sec? B. Transport properties
square foot? A 118 ft/s 51. It is a reversible adiabatic process or C. Thermodynamic
A. 417 B. 236 ftls D. Specific properties
an internally reversible constant entropy
8 . 781 C. 187 ftls process or change of state of a
C. 0.694 D. 326 ftls 57. The sum of internal energy and the
substance or system.
D. 950 displacement energy or flow work of an
A. lsochoric process
46. A ball is thrown 56 ft upward with an B. Isentropic process open system is said to be:
40. The pressure 8 ft below the free initial velocity of 60 ftls. Determine the A. Entropy
C. Polytropic process
surface of a liquid is 2.4 psi. What is the velocity at the maximum height. B. Internal energy
D. Isothermal process
specific gravity of the liquid? A. 6.12 ftls C. Enthalpy
A. 0.469 B. 2.12 ft/s D. Thermal conductivity
52. It is a process in which no heat
8. 0.946 C. 2.61 ftls transfer occurs from the system to the
C. O.p94 D. 0 ftls surroundings or from the surroundings to 58. A process that is completely
D. 1.096 controlled and yields a maximum amount
the system.
47. Determine the super elevation of tbe A. Reversible process of useful work.
41. A rock weighs 90 lb in air and 62 lb in outer rail of a 5 ft wide railroad track on a A. Irreversible process
B. Isentropic process
water. Find its volume. 10 deg curve. (A 10 deg curve is one in C. Adiabatic process B. Adiabatic process
A . 1.119 cu. ft which a chord 100 ft long subtends an C. Quasi-static process
D. lsochoric process
8. 0.669 cu. ft angle of 10 deg at the center). Assume D. Reversible process
C. 0.889 cu. ft velocity of 60 mph. 53. The second law of thermodynamics
D. 0.449 cu. ft A 1.93 ft that it is impossible for a self-acting 59. A constant-pressure regenerative
B. 3.91 ft machine unaided by any external agency power cycle is called:
42. A person can swing a bucket of paint C. 0.91 ft A. Dual cycle
to transfer heat from one body to another
in a vertical plane without spilling its D. 2.53 ft at higher temperature. This second law B. Stirling cycle
contents. What would be the minimum
statement is known as: C. Ericson cycle
tangential velocity necessary to avoid 48. Kinematics is the study of _ _ D. Otto cycle
A. Kelvin Statement
spilling the paint? Assume an effective without reference to the forces that 8 . Kelvin-Plank ·statement
radius of 26 in. causes the body to move.
A. 15.6 fps . A. motion
8. 9.92 fps B. force REFRESHER MANUAL 2nd Edition by JAS TORDILLO
REFRESHER MANUAl2"d Edition by JAS TORDillO
i2- 6 DAY 12 - SOLUTION DAY 12 - SOLUTION 112-7
60. It is the model cycle or the theoretical ENGINEERING MATHEMATHICS
prototype cycle for reciprocat ing spark- DAY 12 - SOLUTION
ignition engines. 6. If log. 10 = 0.250, log1o x equals: of the volume and surface area is 628.
A. Carnot cycle 1. To find the angles of a triangle, given A. 4 * What are the dimensions of the box in
B. Otto cycle only t he lengths of the sides, one would B. 2 inches?
C. Diesel cycle use: C. 1,000 A. 6 X7X8 *
D. Dual cycle A. the law of cosi nes * D. 0:50 B. 1 X 2 X 3
B. the law of sines C. 2 X 3 X 4
C. the Jaw of tangents loa. 10 = 0.250 D. 2x4 x 6
X~25 = 10
r1 DAY 12- ANSWER KEY
I
) D. the inverse-square law

2. The sine of 840 degree equals:


X = 10,000 a= width
b =length= c + 1 =(a+ 1) + 1 =a+ 2
l.A 16. B 31. A 46.0 A. -cos 30 degree log1o x = log1o 10,000 = 4 c = height = a + 1
2. 0 17. c 32.8 47.A B. sin 30 degree
3. 0 18. A 33.A 48. A
4. A 19.0 34.0 49. 0
C. - cos 60 degree 7. An angle between 90 degree and 180 V+SA = 628
5. 0 20.0 35. C 50. A D. sine (2rr + 120 degree) • degree has: abc + (2ab + 2ac + 2bc) = 628
6. A 21. A 36. B 51. B A. A positive sine and cosine
7.C 22. B 37. B 52. C 3. To cut a right circular cone in such a B. a negative cotangent and cosecant by trial & error, try choice A (a = 6, c , 7 &
s.c 23. c
9.A 24. c
38. A 53. C
39. A 54.0
way as to reveal a parabola, it must be C. a negative secant and tangent* b 8)=
cut: D. all its trigonometric functions negative
10. C 25. B 40. c 55.0 A . perpendicular to the axis of symmetry 6(8)(7) + 2(6)(8) + 2(6)(7) + 2(8)(7) = 628
11. c 26. c 41. 0 56. B
B. at any acute angle to the ax is of 336 + 96 + 84 + 112 = 628
12. 0 27. c 42.0 57. c
symmetry Sin 9 = !.~+ Csc9= !.~+ 628 = 628
13. B 28. B 43. c 58.0 + +
14. c 2.9. c 44. B 59. C C. parallel to the axis of symmetry Therefore, a= 6, b = 8 & c = 7. ·
-Jjl 15. A 30. 0 45. B 60. B D. none of these * Cos9= =~ ­ Sec9= !.~-
+ 10. Find the value of the seventeenth
To cut a right circular cone in such a way term of 68, 56, 44, 32, ...... 17"' term.
as to reveal a parabola, it must be cut Tan 9 = !. ~ - Cot 9= = ~ ­ A. 41
+
parallel to the lateral side. B. 14
Therefore, an angle between 90 and 180 c. -124 *
4. Evaluate the lif')'lit X: - 1/x - 1, where x degree has negative secant and tangent. D. -142
approaches to one.
A. 2 * 8. Solve for x in th.e equation: d =56 - 68 = -12
B.O fiX: - 1/4a2 = 0. n = 17
C . none of these A. +2, -2 a= 68
D. 1 B. +1 ,-1
. x2 - 1 • (X+ lXx -1} C. +2a, ~2a * L=a+(n-1)d
Ilm - - = 11m
D. +a, -a = 68 + (17 - 1)(-12)
x~l X -I X_.) X- I
= -124

lim (x+ l)= (l +l)=2 I I


- --= 0 11. Find the value of the eighth term of
2
x_.l x 4a2 1/243, 1/81, 1/27, .. ......... 8111 term.
I A. 1/9
5. If x314 = 8, x equals:
A.6
?'= 4a 2 B. 1/3
c . .9 *
B. '-9 X: =4a2 0.3
c. 9 x =±2a
0 .16 *
x314 =8
9. The dimensions of a covered box are
such that it is 1 inch taller than it is wide
I
r= Yst =3
Y243 n=8 a= 1/243

X= 8413 and 1 inch longer than it is tall. The sum


X= 16
REFRESHER MANUAL 2"0 EdHion by JAS TORDILLO
REFRESHER MANUAL 2"d Editio n by JAS TORDILLO
12-10 DAY 12 - SOLUTION
DAY 12 - SOLUTION 112-11
ENGINEERING MATHEMATHICS
23. Find the area of the region bounded C. skew
by y = x2/4 and y = x/2 + 2. D. non-intersecting
A2
C. 3.4 fps • c. 42.8 ft-lb
D. 7.2 fps D. 85.4 ft-lb
B. 15 26. The vector which represents the sum
c. 9. of a group of force vectors is called the: Mt=MF Power = 21TTN
D. 20 A magnitude (mV), + (mV)2 = (m, + m2)Vf 20(33,000) = 2TTT(1800)
B. sum 180(10) + 0 = (180 + 350)Vf T = 58.35 ft-lb
C. resultant * Vf = 3.4 ft/sec·
A= J(Yu- YL)jx D. phase angle 33. A large flywheel, weighing 1288 lb.

A = J(~
-2
+ 2- X
4
2
}x 27. Which of the following is not a vector
quantity?
A. velocity
30. To overcome rolling resistance, a
locomotive exerts a force of 20,000 lb
while pulling a train of cars over a level
has a radius of gyration of 12
Determine its kinetic energy if it
rotating with an angular velocity of
in.
is
60
road of 30 mph. Determine the rpm.
4 B. acceleration
x2 x3 horsepower that is developed by the A. 789 ft-lb "
C. speed* engine. B. 1155 ft-lb
A= [ 4+2X-J2] -2 D. displacement
A.200 C. 998 ft-lb
B 1100 D. 3512 ft-lb.
4)2 +2(4)- (4)3] 28. Body A traveling to the right with a
A= [( C. 600
velocity of 30 fps, collides with Body B,
4 12 0.1600* r = 12 in= 1 ft
which is traveling to the left with a w = 60 rpm = 6.283 rad/sec
velocity of 10 fps. Determine the velocity V = 30 mph = 44 fps
-[(-2)2 +2(-2)- (-2)3] I = m~ =
12881b (I)2tt 2 = 40 lb- ft- s2
of body A after impact ifWA = 300 lbs, We
4 12 = 150 lbs, and eccentricity e = 0.80. 32.2 7s2 .
Power = Force x Velocity
A= (4 + 8- 5.33)- (1 - 4 + 0.666) A 40 fps to the left
= 20,000(44) ft- b X~ KE = Y.lwl
1
B. 18 fps to the right*
A = 9 sq. units
C. 28 fps to the left
sec 550 = y. (40)(6.283)2
= 1600 hp = 789.57 ft-lb
24. A bag contains 3 white balls and 5 D. 0 fps to the right
red balls. If two balls are drawn at 34.. What velocity will a weight attain if it
.31. How many kilowatts of electricity must
random, find the probability that one ball mAVA + maVa = mAVA' + maVa'
be supplied to run a motor that is rated at falls a distance of 100 ft on the moon?
is white and the other is red.
A. 15/56
300+(30) + 150(10) = 300W + 150 Va'
10,500 = 300VA' + 150 Va' Eq. 1
10 hp of electrical energy is transformed =
(g 5.47'ft/s2)
into mechanical energy with an efficiency A. 80.1 fps
B. Y. B. 45.6 fps
of80%?
C.15/28 * e= Vs'-VA'
A. 9.33 kw* C. 70.1 fps
D. 225/784 VA-VB D. 33.1 fps *
B. 10 kw
0.80 = Vs'-VA' C. 5.97 kw
First white then red:
3 -=
p,: -X 5 -IS
30-10 D. 12 kw v = J2gh =J2(5.47)100
Va' - VA'= 16
8 7 56 Va' =VA'+ 16 Eq. 2 . p = 33.08 ft/sec
First red then white: Effmotor = _.2.
- 5 3 15 Pin 35. A cyclist leans toward the center of
P2- - X - = - 10,500 = 300VA' + 150(W + 16)
1Ohp 0_.7~6 kw curvature when maneuvering a turn.
8 7 56 10,500 = 300VA' + 150VA' + 2400
Determine the banking angle measured
VA, = 18 ft/sec to the right 0.80 = hp from the vertical, if the radius of the turn
Total Probability= P1 + P2 Pin is 30 ft and the velocity of the cycle is 15
15 15 30 15 29. A man weighing 180 lb jumps off a Power supplied, Pin = 9.325 kw
= -56+-=-=-
56 56 28 clock onto a stationary boat weighing 350
fps.
A. 11.2 degree
lb. The horizontal distance component of 32. What is the output torque of a 20-hp B. 14.3 degree
25. If the product of the slopes of any two the man as he leaves the dock is 10 fps. motor running at 1800 rpm? C. 13.1 degree*
straight lines is negative 1, one of these Assuming that he does not fall out on of A. 25.6·ft-lb D. 15.3 degree
lines are said to be: the boat, find his velocity after landing. B. 58.4 ft-lb *
A . parallel A. 1.2 fps
B. perpendicular • B. 5.6 fps REFRESHER MANUAL 2nd Edition by JAS TORDILLO
REFRESHER MANUAL 2"d Edition by JAS TORDILLO
,.. "

t>AY 12 - SOLUTION 112-13


12-12 DAY 12 - SOLUTION ENGINEERING MATHEMATHICS
v2 p = yh
Tan 9 = - 25(144) = 62.4(h) 42. A person can swing a bucket of paint V =·2nRN
gr h= 57.7 ft in a vertical plane without spilling its
v =.2n(.!!X tsoo)
Tan9 = ~
32.2{30)
39. The gage on a condenser shows a
contents. What would be the minimum
tangential velocity necessary to avoid
12 60
v acuum of 24 inches of mercury. What is V = 235.62 ft/sec
9=13.1° spilling the paint? Assume an effective
the absolute pressure in pounds per radius of 26 in.
square foot? 46. A ball is thrown 56 ft upward with an
36. A 10-lb weight, fastened to a 2-ft A. 15.6 tps initial velocity of 60 ft/s. Determine the
A. 417 * B. 9.92 fps
cord, is whirled in a vertical circle with an B. 781 velocity at the maximum height.
angular velocity of 100 rpm. Determine C. 12.5 fps A. 6.12 ft/s
c. 0.694 D. 8.34 fps *
the tension in the cord w hen the weight is D. 950 B. 2.12 ft/s
at the top of the circle. C. 2.E)1 ft/s
A. 691b Fc=W D. 0 ft/s *
P(atm) = 29.92 in Hg
B. 58.11b * P(g) = -24 in Hg mV2
C . 481b R=mg 47. Determine the super elevation of the
D. 77.1 lb P(abs) = P(g) + P(atm) outer rail of a 5 ft wide railroad track on a
v2 10 deg curve. (A 10 deg curve is one in
= -24 + 29.92 w = 32.2
which a chord 100 ft long subtends an
=2rr(2) ( 60
100) =20.94 ft/ sec ~2
V = 2rrRN
= 5.92 in Hg (
14 7
· psi
29.92 in Hg
J V = 8.35 ft/sec
angle of 10 deg at the center). Assume
velocity of 60 mph.
Fe= wv2 =(lOX20.94f =68 _1 lbs = 2.9(1 44) 43. One gram of ice at 0°C is placed on a A.1.93ft*
gr 32.2(2) =4 17.6 lbs/ff B. 3.91 ft
Z:Fv = 0
T + 10 - Fe= 0
.
40. The pressure 8 ft below the free
container containing 2,000.00 cu. meter
of water at 0°C. Assuming no heat loss, c. 0.91 ft
D. 2.53 ft
what will happen?
T = Fc- 10 = 68.1 - 10 = 58. 1 1bs surface of a liquid is 2.4 psi. What is the A. ice will melt and become water
specific gravity of the liquid? B. some part of ice will not change V =60 mph = 88 ft/sec
37. A passenger on an elevator weighs A. 0.469 C. the volume of lee will not change * S 10. so =so-
180 lb. What is the force between him B. 0.946 D. water will become ice r
and the floor of the elevator if the elevator c. 0.694 * R = fJ73.7 ,ft
is accelerating upward at the rate of 5 D. 1.096 44. What is the atmospheric pressure on v2
ft/s2? a planet where the absolute pressure is Tan9= -
A. 180 lb PL = YL hL gR
101 kPa and the gage is 9 kPa?
B. 2081b * 2.4(144) = YL (8)
c. 1951b YL = 43.2 lbs/ft3
A. 110 kPa
B. 92 kPa *
Tan 9 = (ss)2
32.2(5 73.7)
D. 280 lb 43 2
S.G. = A=
yw 62.4
• =0.693 C. 105 kPa 9 = 22.74°
D. BB kPa
Z:Fv = 0 sin 22.74° = ~
T- REF -W = 0 5
41. A rock weighs 90 lb in air and 62 lb in P(abs) = P(g) + P(atm)
T= REF +W 101 = 9 + P(atm) h = 1.933 ft
water. Find its volume.
T= Wa+W= 180(5)+ 180 A. 1.1 19 cu.ft P(atm) = 101-9 = 92 kPa
48 . Kinematics is the study of _ _
g 32.2 - B. 0.669 cu. ft
without reference to the forces that
T =207.95 lbs C. 0.889 cu. ft 45. A flywheel of radius 18 inches is
rotating at the rate of 1500 rpm. How fast causes the body to move.
D. 0.449 cu. ft *
does a point on the rim travel in ft/sec? A . motion *
38. How many feet of water are
Bouyant force, Fa= 90 - 62 = 28 lbs A. 118 ft/s B. force
equivalent to 25 psi?
B. 236 ft/s * C. forces and motion
A . 57.7 ft*
c. 187 ft/s D. matter
B. 77.1 ft Fa= V X p
C. 67.7 ft 28 = V X 62.4 D. 326 ft/s
D. 97.9 ft v = 0.449 ft3
REFRESHER MANUAL 2nd Edition by JAS TORDILLO
REFRESHER MANUAL 2"d Edition by JAS TORDILLO
13-2 DAY 13- EXAM DAY 13 :. EXAM 113-3
11 . A bank charges 1 Y. % per month on
ENGINEERING MATHEMATHICS
interest is 5%, the capitalized cost of the
the unpaid balance for purchases made dam, including maintenance, is:
with its credit card. This is equivalent to year's free maintenance. In the second· C. P29,050
A. P100,000
what effective annual interest rate? B. P215,000
year the maintenance is estimated at D. P52,050
P20. In the subsequent years the
A. 1.5% C. P200,000
B. 18% maintenance cost will increase P20 per 24. A company is considering buying a
D. P300,000
year. How much would need to be set- new piece of machinery. The initial cost
c. 12%
D. 19.5% aside now at 5% interest to pay the of the machine is P80,000 and its
17. A piece of property is purchased for
maintenance costs on the tractor for the salvage value is P20,000 at the end of 20
P10,000 and yields a 1?1000 yearly profit.
first six years of ownership? years. Annual operating cost is P18,000.
12. The principal sum .E. is invested at a If the property is sold after 5 years, what
nominal interest rate r. compounded m
A. P101.52 , 1 What is the capitalized cost of the
is the minimum price to break-even, with
B. P239.36 machine based on 10% interest rate?
times a year, for rr years. The interest at 6%?
C. P164.74 A. P60,000
accumulated amount at the end of this A. P5,000
period will be: D. P284.13 B. P230,000
A. P(1 +rim)'
B. P7,745
C. P6,554 . .
21. A manufacturing company buys a
C. P80,000
D. P270,000
B. P(1 + r)" D. P8,314
C. P(1 + r)mr machine for P50,000. It estimates the
D. P(1 + r/m)mn machine's useful life is 20 years and that 25. A company is considering buying a
18. A steam boiler is purchased on the
it can then be sold for P5000. Using new piece of machinery. The initial cost
basis of guaranteed performance. A test
straight line depreciation, what is the of the machine is P80,000 and its
13. An annuity is defined as indicates that the operating cost will be
A. Earned interest due at the end of each P300 more per annual depreciation charge? salvage value is P20,000 at the end of 20
year than the
interest period manufacturer guaranteed. I the expected
A. P2000 years. Annual operating cost is P18,000.
B. P2500 Using sum-of-the-years digit
B. Cost of producing a product or life of the boiler is 20 years and money is
rendering a service C. P2250 depreciation, what would be the book
worth 8%, how much should the
D. P2750 value after two years?
C. Amount of interest earned by a unit of purchaser deduct from the purchase
principal in a unit of time A. P21,000
price to compensate for the extra
22. Company A has faxed expenses B. P59,000
D. A series of equal payments occurring operating cost?
P15,00 per year and each unit of product C. P42,000
at equal periods of time A. P2945
has a P0.002 variable cost. Company B D. P69,000
B. P4102
has fixed expenses of P5000 per year
14. The present sum is equivalent to a C. P3320
series of P1000 annual end-of-year D. P5520 and can produce the same unit of product 26. These are product or services that
at a P0.05 variable cost. At what number are required to support human life and
payments, if a total of 10 payments are
made and interest is 6%? of units of annual production will activities, that will be purchased in
19. Annual maintenance costs for a
A. 1?6,250 Company A if they have the same overall somewhat the same quantity even
particular section of highway pavement
B. P9,400 cost as Company B? though the price varies considerably.
are P2000. The placement of a new
C. P7,360 A. 158,333 units A. utilities
surface wou ld reduce the annual
D. P10,600 B. 300,000 units B. luxuries
maintenance cost to P500 per year for
C. 208,333 units ,. C. necessities
the first five years and to P1 000 per year
D. 400,000 untis D. producer goods
15. A woman made ten annual end-of- for the next five years. The annual
year purchases of P1000 of common maintenance after ten years would again
23. A company Is considering buying a 27. We may classify an interest rate,
stock. At the end of the tenth year she be P2000. If maintenance costs are only
new piece of machinery. The initial cost which specifies the ac.tual rate on interest
sold all the stock for P12,000. What saving, what maximum investment can
of the machine is P80,000 and its on the principal for one year as:
interest rate did she. obtain on her be justified for the new surface? Assume
investment? salvage value is P20,000 at the end of 20 A. nominal rate
interest at 4%.
A. 2% years. Annual operating cost is P18,000. B. exact interest rate
A. P5,500
B. 4% What is the equivalent uniform annual C. rate of return
B. P10,000
cost of the machine based on 10% D. effective rate
C. 8% C. P7,1 70
D.14% interest rate?
D. P10,340
A. P27,050 28. A formal organization of producers
B. P38,050 within an industry forming a perfect
16. A dam was constructed for P200,000.120. A man buys a small garden tractor.
collision purposely formed to increase
The annual maintenance cost is P5000. If There will be no maintenance cost in the
first year as the tractor is sold with one REFRESHER MANUAL 2nd EdlHon by JAS TORDILLO
REFRESHER MANUAL 2"d Edition by JAS TORDILLO

13-4 DAY 13 - EXAM DAY 13- EXAM ~13-5
ENGINEERING MATHEMATHICS
profit and block newcomers from the interest compounded annually. In how
industry is called: many years will the amount be P4M? 40. TOERC company manufactures CD B. P350,000
A. Oligopoly A. 4 years
I tapes has a capacity of 20,000 units in C. P250,000
B. Corporation B. 10.6 years one year. Material cost to produce one D. P400,000
C. Monopoly C. 8.2 years unit is P20.00 and a variable cost of
D. Cartel D. 12.5 years P5.00 each unit produced. Fixed annual 45. ·what interest rate, compou nded
cost of the company amount to P200,000 monthly is equivalent to 10% effective
29 A market whereby there is only one 35. If P1 000 is deposited in a savings which includes salaries and taxes. rate?
buyer of an item for which there are no account and left for 12 years. What is the Determine the number of units produced A. 10.75%
goods substitute. Cjccumulated amount if Interest is 8% and sold annually in order to break-even. B. 9.68%
A. Monopoly compounded quarterly? CD price is P40.00 each. C. 9.75%
B. Monopsony A. more than P2500 A. 13,300 units D. 9.57%
C. Oligopoly B. more than P3000 B. 13,334 units
D. Oligopsony C. less than P2000 C. 13,333 units 46. A flywheel is brought from rest up to a
D. exactly equal to P3000 D. 13,999 units speed of 1500 rpm in 1 minute. What
30 The difference between the present
is the average angular acceleration?
worth and the worth at some future time 36. A bank-offers 0.75% effective monthly 41. Corporation is a form of bu~iness A. 2.617 rad/sec2
is: interest. What is the effective and annual ownership in which it can hold title to B. 157 rad/sec
A discount interest rate? property , sue and be used in its own C. 333 rad/sec
B. credit A. 9.1% interest. Which of the following is not a D. 5.23 rad/sec2
C. profit B. 10.5% form of ownership?
D. debit C. 9.4% A. partnership
i 47. What is the coefficient of performance
l D. 12.1% B. company of a Carnot refrigeration cycle operating
31. The original amount P if borrowed will C. private individual between -10 deg F and -190 deg F?
have a future sum of F at an interest rate 37. Find the nominal rate, which if D. single proprietorship A. 1.5
of i%. What is the amount of money paid converted quarterly could be used B. 1.06
for the use of borrowed money? instead of 11 .88% compounded monthly.
A. present amount
42. The length of time, usually in no. of c. -0.5
A.1 0% years, that a certain investment can be D. 0.5
B. overhead cost B. 12% can be recovered.
C. interest C. 11% A. break-even point 48. If the pressures at the beginning and
D. rate of return D. 14% B. return on investment end of compression in an Otto cycle
C. pay-back period engine are 1 kg/sq. em and 8 kg/sq. em,
32. If you deposit an amount of P5000 at 38. A businessman needs an additional D. maturity period respectively, the air standard efficiency
8% simple interest. What is the exact capital of P1 00,000 10 years hence. will be nearly:
simple interest for a period of 18 months? What is the annual deposit in order to 43. Which of the following has the least A. 57%
A. P600 met the required capital? Assume interest effective annual interest rate? B. 50%
B. P5650 at 5%. A. 12% compounded quarterly C.40%
C. P900 A. P10,000 B. 11.5% compounded monthly D. 45%
D.6909 B. P12,500 C. 11.7% compounded semi"annually
C. P12,850 D. 12.2% compounded annually 49. What is the latent beat of fusion
;33. A businessman nees P300,000 in I D. P7,950 required to turn ice to liquid?
years. What amount of money will he 44. A debt of ~ pesos, with interest rate of A. 2256 kJ/kg
invest now at 7% interest compounded 39. A motorcycle can be acquired for 7% compounded annually will be retired B. 335 kJ/kg
annually? P5000 down payment and a monthly at the end of 10 years through the C. 970 Btu/Ibm
A. P126,800 payment of P3000 for 3 years. Using 3% accumulation of deposit in the sinking D. 166 Btullb
B. P186,825 effective monthly rate, what is the cash fund invested at 6% compounded semi-
C. P162,825 price of the motorcycle? annually. The deposit in the fund sinking 50. The ratio of the weight of the water
D. P205,660 A. P30,000
every end of six months is P21,962.68. vapor mixed with a kg of dry air to the
B. P68,500
What is the value of~? weight of saturated vapor at the same
34. Mr. Bacus invested a time deposit of C. P65,000
A. P300,000 temperature:
P1M to a bank earning 14% annual D. P70,500
. REFRESHER MANUAL 2nd Edition by JAS TORDILLO
REFRESHER MANUAL 2"d Edition by JAS TORDILLO
13 - 6 DAY 13 - EXAM
DAY 13 - SOLUTION 113 ·
1
ENGINEERING MATHEMATHICS
A. relative humidity B. P4,481 .76
B. humidity ratio C. P8,414.67 A. 6.42 m ·
C. saturation ratio D. P4,418.67 B. 3.46 m
DAY 13 - SOLUTION
D. specific density C. 2.64 m
56. A pump discharges 50 ton of water D. 5.32 m 1. A shot is fired at an angle of 30 deg
51 . If $500 is invested at the end of each with the horizontal and a velocity of 120
per hour to a height of 8 m. The overall
year for six years, at an effective annual m/s. Calculate the range of the projectile.
efficiency of the pumping system is being
A. 12.71 m
interest rate of 7%, what is the total dollar 69%. Calculate the energy consumed by (d )
B. 0.789 mile "
amount available upon the deposit of the the pump in 2 hours, expressed in MJ.
sixth payment? A. 12.68 MJ DAY 13- ANSWER KEY c. 387.57 ft
A. $4260 B. 12.38 MJ
1. 8 16. 0 31.C 46. A D. 423.74 yd
2. C 17. 8 32.A 47.A
B. $3000 C. 11.68 MJ 3. A lB. A 33. 8 48.0
c. $3210 D. 11.38 MJ 4.8 19. 0 34. B 49. B vJsin29
R=...,:,t.___
D. $3577 5. c 20.8 35. A 50. C g
57. What is the density of the stone if the 6. c 21. c 36.C 51.0
52. If P25,000 is invested now, P35,000 stone weighs 19.9 lbf (88 N) in air and 7. 0 22. c 37.8 52.A R = (120)2 sin 2(30)
two years from now, and P45,000 four 12.4 lbf (55 N) in water? 8.C 23.A 38.0 53. 0
39.0 54.0
9.81
9.C 24.0
years from now (all at 4%), what will be
the total amount in ten years?
A. 2655 kg/m 3
B. 2620 kg/m 3
10. A 25. D 40. 8 55. c = =
R 1271.2 m 0.790 mile
11. D 26. C 41.8 56. D
A. P141,850 C. 271 o kg/m 3 12.0 27. D 42. C 57. A 2. Ethylene glycol at 60 deg C, with a
B. P141,000 D. 2700 kg/m 3 13. D 28. D 43. C 58. D velocity of 4 cm/s enters a 2.5 em ID
C. P141,500 14. c 29.8 44. A 59. C tube. At 60 deg C the viscosity is 4.75 x
D. P142,000 58. Steam with an enthalpy of 800 15. B 30. A 45. D 60. C 10~ m2/s, determine the Reynolds
kcallkg enters nozzle at a velocity of 80 G Number.
53. Find the nominal rate which if m/sec. Find the velocity of the steam at A. 150
converted quarterly could be used the exit of the nozzle if its enthalpy is 8 . 2100
instead of 12% compounded semi- reduced to 750 kcal/kg, assuming the c. 210"
annually. nozzle is horizontal and disregarding heat D. 1200
A. 10.58% losses. Take g = 9.81 m/s2 and J DV
B. 11 .28% constant = 427 kg-m/kcal. Reynolds Number, NR =-
jJ.
c. 9.38% A. 561 .24 m/s
D. 11 .82% B. 52.41 m/s - 0.025(0.04)
C. 142.5 m/s - 4.75xlo-6
54. A sum of money is deposited now in D. 652.14 m/s = 210
a savings account. The effective annual
interest rate is 12% and interest is 59. A perfect venturi with throat diameter 3. Determine the mean velocity of an
compounded monthly. How much money of 1.8 inches is placed horizontally in a engine oil at 60 deg C flowing at 0.01
must be deposited to yield P500.00 at the pipe with a 5 in inside diameter. Eighty kg/s through a square duct 1 em x 1 em
end of 11 months? (80) Ibm of water flow through the pipe in cross section. At 60 deg C density is
A. P153.00 each second. What is the difference 864 kg/m 3 .
B. P446.00 between the pipe and venturi throat static A. 0.1157 m/s *
C. P144.00 pressure? B. 0.2263 m/s
D. P451 .00 A. 29.9 lbf/in 2 C. 0.1517 m/s
B. 5020 lbf/in2 D. 0.2630 m/s
55. The maintenance cost of a machine C. 34.81bf/in2 '
this year is expected to be P1 ,000.00. D. 72.3 lbf/in2 Q=AV
The cost will increase P200.00 each year
for the subsequent 9 years. The interest 60. Determine the friction head loss for ~ = (o.olXO.Ol)V
864
is 15% compounded annually. What is fully develocr.ed laminar flow of ethylene V = 0.11574 m/s
the approximate worth of maintenance for glycol at 40 C (p = 1101 kg/m 3 ) through a
the machine over the full ten years? 5 em diameter, 50 m long pipe, if friction
A. P4,814.67 factor is 0.242 and a flowrate of 0.1 kg/s.
REFRESHER MANUAL 2nd Edition by JAS TORDILLO

REFRESHER MANUAL 2"d Edition by JAS TORDILLO


DAY 13- SOLUTION
. DAY 13 - SOLUTION 113-9
13-8 ~NGINEERING MATHEMATHICS
4. A water main with 24 inches inside Q=AV
diameter carries a flow of 20 cu. ftlsec. If
bh3, 30(60~ = 540,000 cm4 P = R[{l+i)n -1]
the friction factor is 0.02 and the pump is -- 41t(2)2
i2 (25) 1=
12 = 12
85% efficient, how much horsepower is i(1 + i)"
required to pump the water thrq_ugh
10,000 ft of pipeline?
= 0.545415 .!tx 7.48 Gal x 60 sec k=
{T
v"A =
540,000
(3oX6o)
= !7.32 em
p = 1000[ (l+iJO -1 ]
sec · f0 min 0.06(1 + 0.06)10 = 7,360
A. 140
B. 168 * = 244.78 GPM
11 . A bank charges 1 Yz % per month on
C. 156 the unpaid balance for purchases made
D. 250 7. The value of the coefficient of viscosity 15. A woman made ten annual end-of-
with its credit card. This is equivalent to year purchases of P1 000 of common
of air at 19.2 deg C is 1.828 x 10-4
poises (g/cm-sec). The equivalent value, what effective annual interest rate? stock. At the end of the tenth year she
D = 24 in= 2ft A. 1.5%
expressed in pounds per foot-second, is sold all the stock for P12,000. What
closest to: B.. 18% interest rate did she obtain on her
Q =Av c. 12%
A. 1.23 X 10' 2 investment?
20 = ~(2)2V B. 1.02 X 10'3 0.19.5% * A.2%
4 3 8.4%*
C. 5.57 X 10"
V =6.36 ft/sec D. 1.23 X 10'5 * i=(1 +ip)m -1 =(1 +0.015)12 -1 . C.8%
= 0.1956 = 19.56% 0.14%
-4 g 2.2 lbs 2.54cm 12 in
tLV 2 I .828x 10 - - x -- x - - x - - 12. The principal sum E is invested ·at a
hL= - - cm - s 1000 g in ft
2g0 nominal interest rate r. compounded !!l S=R[(l+ir-1]
= 1.226 X !0-5 ~ times a year, for !l years. The
h = (o.o2Xto,oooX6.J6)2 = 62 _ tt ft -sec
8 accumulated amount at the end of this
L

'lp
2(32.2)2
PL = QA.hL = (20X62.4X62.8) ft - lb X~
0.85 sec 550
8. A fluid flows at a constant velocity in a
pipe. The fluid completely fills ·the pipe,
period will be:
A. P(1 + r/m)'
B. P(1 + r)"
12,000 [(I -l]
=1000 +i~IO
PL = 167.65 hp and the Reynolds number is such that the C. P(1 + r)mr by trial and error, try i = 4%:
0 . P(1 + r/m)mn *
0~0 = 1,000[(1+0.04)10 - 1]
flow is laminar. If all other parameters
5. The theoretical head required to push remain unchanged and the viscosity of 12
the fluid is decreased a significant 13. An annuity is defined as ' 0.04
water through and orifice at 15 m/sec is
closest to: amount, one would generally expect the A. Earned interest due at the end of each 12,000 = 12,000
A. 9.8m flow to: interest period
B. 12.7 m A. not change B. Cost of producing a product or. 16. A dam was constructed for P200,000.
C.11.5m* 8. become more laminar rendering a service The annual maintenance cost is P5000. If
D. 22'.9 m C. become turbulent" C. Amount of interest earned by a unit of interest is 5%, the capitalized cost of the
D. increase principal in a unit oftime dam, including maintenance, is:
D. A series of equal payments A . P100,000
y2
h=- 9. An instrument used for measuring occurring at equal periods of time * 8. P215,000
2g specific gravity of fluids: C. P200,000
A . hygrometer 14. The present sum is equivalent to a
h =
(15~ =11.467 m
2(9.8 1)
B. flowmeter series of P1000 annual end:-ef-year
D. P300,000 *
C. hydrometer* payments, if a total of 10 payments are
D. psychrometer made and interest is 6%? Capitalized Cost =FC + OM
6. Water is flowing through a 2 inches A. P6,250 i
diameter pipe at a rate of 25 ftlsec. The 10. A rectangular shape has a cross B. P9,400 5000
rate of flow, in US gallon per minute, is section 30 em wide and 60 em in height. C. P7,360 * = 200,000 +
0.05
closest to: The least radius of gyration, in em, for D. P10,600
A. 200
= P300,000
this shape is closest to:
B.310 A.17.3*
c. 245 * B. 20
D. 770 C. 15 REFRESHER MANUAL 2nd Edition by JAS TORDILLO
D. 25
REFRESHER MANUAL 2"d.Edition by JAS TORDILLO
13- 10 DAY 13 - SOLUTION
DAY 13 - SOLUTION 13 -11
ENGINEERING MATHEMATHICS
I
17. A piece of property is purchased for be P2000. If maintenance costs are only
P10,000 and yields a P1000 yearly profit. saving, what maximum investment can machine's useful. life is 20 years and that Annual Depreciation
If the property is sold after 5 years, what be justified for the new surface?'Assume it can then be sold for P5000. Using
is the minimum price to break-even, with interest at 4%.
= 80,000-20,000 =1047.577
straight line depreciation, what is the (1+0.10)20 -1
interest at 6%? A. P5,500 annual depreciation charge?
A P5,000 B. P10,000 A.P2000 0.10
B. P7,745 * C. P7,170 B. P2500 Interest on Capital
C. P6,554 D. P10,340 * C. P2250 * = (80,000 X 0.1 0) = 8,000.00
D. P8,314 D. P2750 Annual operating cost
. . FC-Sv
= 18,000
Let F = minimum price of the property to p = (2000-500) [ (1 + 0.04f -1 ] Annual Deprec1at1on = - - -
0.04(1 + 0.04f n Total annual cost= Annual Dep +Interest
break-even
= 50,000-5000 on Cap+ OM
10,000(1 + 0.06) 5 (2ooo- 10oof12'_o.o4p-:.LJ 20 = 1047.57 + 8000 + 18,000

= F + 1000[(1+0.06)5
0.06
-l] +
10.04(1 + 0.04)5
(1+ 0.04)5
= P2250.00 = 27,047.57

22. Company A has fixed expenses 24. A company is considering buying a


p = 6677.73-3659.07 P15,00 per year and each unit of product new piece of machinery. The initial cost
13,382.25 = F + 5637.09 p = 10,336.80
F = 7,745.16 has a P0.002 variable cost. Company B of the machine is P80,000 and its
has fixed expenses of P5000 per year salvage value is P20,000 at the end of 20
20. A man buys a small garden tractor.
18. A steam boiler is purchased on the and can produce the same unit ofproduct years. Annual operating cost is P18,000.
There will be no maintenance cost in the at a P0.05 variable cost. At what number What is the capitalized cost of the
basis of guaranteed performance. A test first year as the tractor is sold with one
indicates that the operating cost will be of units of annual production will machine based on 10% interest rate?
year's free maintenance. In the second Company A if they have the same overall A. P60,000 .
P300 more per year than the year the maintenance is estimated at B. P230,000
manufacturer guaranteed. I the expected cost as Company B?
P20. In the subsequent years the A. 158,333 units C. P80,000
life of the boiler is 20 years and money is maintenance cost will increase P20 per D. P270,000 *
worth 8%, how much should the B. 300,000 units
year. How much would need to be set- C. 208,333 units *
purchaser deduct from the purchase aside now at 5% interest to pay the
price to compensate for · the extra maintenance costs on the tractor for the
D. 400,000 untis = =
Capitalizep cost FC OM + FC- sv
operating cost? ·i (l+if - 1
first six years of ownership?
A. P2945* Cost A = Cost B
A. P101 .52 = 80,000 + 18,000 + 80,000-20,000
B . P4102 15,000 + 0.002x = 5,000 + 0.05x
B. P239.36 * 0.10 (1+0.10)20 -I
C. P3320 0.048x = 10,000
C. P164.74 = P270,475.77
D. P5520 x = 208,333.33 units
D. P284.13
23. A company is considering buying a 25. A company is considering buying a
P=R[(1+ir - 1]
R =0, A = 20, n =6 new piece of machinery. The initial cost new piece of machinery. The initial cost
i(1+ir of the machine is P80,000 and its of the machine is P80,000 and its
p = R[(l+ir - 1]
salvage value is P20,000 at the end of 20 salvage value is P20,000 at the end of 20
i(l+ir years. Annual operating cost is P18,000.
P = 300 [ (1 + o.o8)2° - 1 J years . Annual operating cost is P18,000.
What is the equivalent uniform annual Using sum-of-the-years digit
o.o8(1 + o.o8)20 +A[(l+if-J _n_J cost of the machine based on 10% depreciation, what would be the book
p = 2945.44 i2{J +if i(l + ir interest rate? value after two years?
P=O A. P27,050* A. P21,000
19. Annual maintenance costs for a B. P38,050 B. P59,000
particular section of highway pavement + 20 [ (1 + o.o5)6 -1 6 J C. P29,050 C. P42,000
are P2000. The placement of a new o.os2(1 + o.os)6 o.o5(1 + o.os'f D. P52,050 · D. P69,000 *
surface would reduce the annual
maintenance cost to P500 per year for
=
p 20(1 01.51 - 89.545)
20
the first five years and to P1000 per year
p = 239.35 SYD = ~(n+ 1) = (20+ 1) =210
2 2
for the next five years. The annual
21 . A manufacturing company buys a REFRESHER MANUAL 2nd Edition by JAS TORDILLO
maintenance after ten years would again
machine for P50,000. It estimates the
REFRESHER MANUAL 2"d Edition by JAS TORDILLO
13-12 DAY 13 - SOLUTION CAY 13 - SOLUTION
ENGINEERING MATHEMATHICS
113-13
C. Oligopoly

r
Dep, = (FC-SV) (-n-) • D. Oligopsony
SYD C. 8.2 years

= (80.000-20,000{ ~)
\210
30. The difference between the present
worth and the worth at some future time
D. 12.5 years

S = P(1 + i)"
(I+ i; -i=(l+ 0.~~88r -1
is: In= 0.11998 = 11.99% say 12%
= 5714.28 4,000,000 = 1,000,000(1 + 0.14)"
A. discount * In 4 = n In (1.14) compounded quarterly
Dep2 = (FC-SV) ( -n-1) B. credit n = 10.58 yrs
SYD C. profit 38. A businessman needs an additional
D. debit capital of P100,000 10 years hence.
35. If P1 000 is deposited in a savings
=(80,000- 20,ooof 20 - 1) = 5428.57
account and left for 12 years. What is the
What is the annual deposit in order to
\ 210 31 . The original amount P if borrowed will met the required capital? Assume interest
accumulated amount if interest is 8%
= 5428.57 have a future sum of F at an interest rate at5%.
compounded quarterly?
of i%. What is the amount of money paid A. P10,000
A. more than P2500 *
for the use of borrowed money? B. P12,500
BV2 = FC- (Dep, + Dep2) B. more than P3000
A. present amount C. P12,850
= 80,000- (5714.28 + 5428.57 C. less than P2000
B. overhead cost D. P7,950 *
= 68,857.15 D. exactly equal to P3000
C. interest*
26. These are product or services that
are required to support human life and
activities, that will be purchased in
D. rate of return

32. If you deposit an amount of P5000 at


i = 8%/4 = 2% per quarter
n- 12 years= 48 quarters
S = R[ (1 + r-
I]

8% simple interest. What is the exact S = P(1 + i)"


somewhat the same quantity even simple interest for a period of 18 months? 100,000 = R[(l +O.OsjO - 1]
though the price varies considerably. A. P600 *
=1000(1 + 0.02)48 . 0.05
A. utilities = 2587.07
B. P5650 R = 7950.45
B. luxuries C. P900
C. necessities * 36. A bank offers 0.75% effective monthly
D.6909
D. producer goods interest. What is the effective and annual 39. A motorcycle can be acquired for
interest rate? P5000 down payment and a monthly
I= Pni A. 9.1%
27. We may classify an interest rate, payment of P3000 for 3 years. Using 3%
which specifies ·the actual rate on interest I = (5000) c~ )(o.os) B. 10.5%
c. 9.4% *
effective monthly rate, what is the cash
on the principal for one year as: price of the motorcycle?
A. nominal rate I= 600.00 0.12.1% A. P30,000
B. exact interest rate B. P68,500
C. rate of return 33. A businessman nees P300,000 in I C. P65,000
i = (l+ip)m -I
D. effective rate* years. What amount of money will he D. P70,500 *
invest now at 7% interest compounded = (1 + 0.0075) 12-1
28. A formal organization of producers annually? = 0.0938 = 9.38% Cash price
within an industry forming a perfect A. P126,800 6
collision purposely formed to increase B. P186,825 * 37. Find the nominal rate, which if = 30oo[ (l+O.o3 -l ]+sooo
)3
profit and block newcomers from the C. P162,825 converted quarterly could be used O.D3(1 + 0.03 )36
industry is called: D. P205,660 instead of 11.88% compounded monthly. Cash price= 70,496.75
A. Oligopoly
A. 10%
B. Corporation S = P(1 + i)" 8.12% *
c. 11% 40. TOERC company manufactures CD
C. Monopoly 300,000 = P(1 + 0.07)7 tapes has a capacity of 20,000 units in
D. Cartel* p = 186,824.92 D. 14%
one year. Material cost to produce one
unit is P20.00 and a variable cost of
29. A market whereby there is only one 34. Mr. Bacus invested a time deposit of Effective rate (quarterly) = Effective rate
(monthly) P5.00 each unit produced. Fixed annual
buyer of an item for which there are no P1M to a bank earning 14% annual cost of the company amount to P200,000
goods substitute. interest compounded annually. In how which includes ·salaries and taxes.
A. Monopoly many years will the amount be P4M? Determine the number of units produced
B. Monopsony * A. 4 years
B. 10.6 years * REFRESHER MANUAL 2nd Edition by JAS TORDILLO
REFRESHER MANUAL 2"d Edition by JAS TORDILLO
13-14 DAY 13 - SOLUTION DAY 13 - SOLUTION 113-15
and sold annually in order to break-even. 44. A debt of~ pesos, with interest rate of ENGINEERING MATHEMATHICS ·
CD price is P40.00 each. 7% compounded annually will be retired
A. 13,300 units at the end of 10 years through the 47. What is the coefficient of performance B. humidity ratio
B. 13,334 units* accumulation of deposit in the sinking of a Carnot refrigeration cycle operating C. saturation ratio *
C. 13,333 units fund invested at 6% compounded semi- between -10 deg F and -190 deg F? D, specific density
A.1.5 *
D. 13,999 units annually. The deposit in the fund sinking
B. 1.06
every end of six months is P21,962.68. 51. If $500 is invested at the end of each
Let x =no. of cd tapes produced and sold What is the value of~?
c. -0.5 year for six years, at an effective annual
D. 0.5
A. P300,000 * interest rate of 7%, what is the total dollar
Income= Expenses B. P350,000 amount available upon the deposit of the
40x =20x + 5x + 200,000 C. P250,000 COP= ___:!1. sixth payment?
x = 13,333.33 say 13,334 units D. P400,000 TH -TL A. $4260

41. Corporation is a form of business


ownership in which it can hold title to
x(1 +0.07) 10 = 21,962.68 [ (I+O~~:o -I] -190+460
= -10-(- 190)
B. $3000
C. $3210
D. $3577 *
property ; sue and be used in its own X= 3QQ,QQQ.QQ = 1.5
interest. Which of the following is not a
form of ownership?. 45. What interest rate, compounded 48. If the pressures at the beginning and S = R (l+i)"
. -I]
A. partnership [ I
monthly is equivalent to 10% effective end of compression in an Otto cycle
B. company* rate? engine are 1 kg/sq. em and 8 kg/sq. em,
C. private individual
6
A. 10.75% respectively, the air standard efficiency S O.Q7 -1] = P3576.64
= 50{ (1+0.07)
D. single proprietorship B. 9.68% will be nearly:
c. 9.75% A. 57%
42. The length of time, usually in no. of D. 9.57% * B. 50%
years, that a certain investment can be C.40% 52. If P25,000 is invested now, P35,000
can be recovered. D. 45% * two years from now, and P45,000 four
. . )m years from now (all at 4%), what will be
A. break-even point
B. return on investment
1= (1+ m'n -I the total amount in ten years?
C. pay-back period *
. )'2 f'1 =(~)K A. P141,850 *
B. P141,000
D. maturity period 0.10 = (I+~~ -1
PJ v2
C. 'P141,5,00

~
D. P142,000
43. Which of the following has the least
effective annual interest rate?
in = 9.56% compounded monthly
fK: =(F1 )}k =(.!)){.4 =4.416
V2 P1 I s;, 45,000(1 + 0.04)6
A. 12% compounded quarterly 46. A flywheel is brought from rest up to a
1 + 35,000(1,+0.04}8 +25,000(1 +0.04) 10
B. ~ 1~5% compounded monthly speed of 1500 rpm in 1 minute. What e = 1- - - = 1----;-~
C. 11 .7% compounded semi-annually* is the aver~ge anpular acceleration? r~-l 4.4161.4- 1
D. 12.2% compounded annually A. 2.617 rad/sec * s = 56,939.35 + 47,899.92 + 37,006.10
e = 1 - 0.552 = 0.448 = 44.8% S = P141,845.37
B. 157 rad/sec
C. 333 rad/sec
A. i = 1+( 0 ·~ 2 r -1=12.55% D. 5.23 rad/sec2
49. What is the latent beat of fusion 53. Find the nominal rate which if

r
required to turn ice to liquid? converted quarterly could be used
a_-ro2
-- -col
- A. 2256 kJ/kg instead of 12% compounded semi-
01 5 t B. 335 kJ/kg " annually.
B. i = (t+ · ; -1=12.12% C. 970 Btu/Ibm A. 10.58%
1 ~00 -0 D. 166 Btu/lb B. 11.28%
= _QQ
(t + 0 · ~17 r -1=12.04%
C. 9.38%
C. i= 60 50. The ratio of the weight of the water D. 11.82% *
= 0.4 166 rev x 27t rad vapor mixed with a kg of dry air to the
D. i =12.2%
sec 2 rev weight of saturated vapor at the same· Effective rate quarterly = Effective rate
temperature: semi-annually
Therefore, the least effective annual = 2.617 rad/sec2
A. relative humidity
interest rate is 12.04%.
REFRESHER MANUAL 2nd Edition by JAS TORDILLO
REFRESHER MANUAL 2"d Edition by JAS TORDILLO
DAY 13 - SOLUTION
DAY 13 - SOLUTION 113 -17
13: 16 ENGINEERING MATHEMATHICS
. J4 - 1 = (1+-2-
(1+4
In 0. 12 )2
-1 P =1 oo{ (1.15)1 o-110 ] 58. Steam with an enthalpy •of 800
kcallkg enters nozzle at a velocity of $0
~P = Specific Weight x ~h
0.1 5(1.15) _ 62.4(81.73 -1.372) = 34.82 lb/in2
in = 11.82% compounded quarterly m/sec. Find the velocity of the steam at - 144
20 (1.15)10 -1 10 ] the exit of the nozzle tf its enthalpy is
54. A sum of money is deposited now in + { 0.152 (1.15)1°- 0.15(1.15)10 reduced to 750 kcallkg, assuming the
60. Determine the friction head loss for
a savings account. The effective annual nozzle is horizontal and disregarding heat
P = P8,414.67 fully develotfed laminar flow of ethylene
interest rate is 12% and interest is losses. Take g = 9.81 m/s2 and J
glycol at 40 C (p = 1101 kg/m3 ) through a
compounded monthly. How much money constant = 427 kg-mlkcal.
56. A pump discharges 50 ton of water A. 561 .24 m/s 5 em diameter, 50 m long pipe, if friction
must be deposited to yield P500.00at the factor is 0.242 and a flowrate of 0.1 kg/s.
per hour to a height of 8 m. The overall B. 52.41 m/s
end of 11 months? A. 6.42 m
efficiency of the pumping system is being C. 142.5 m/s
A. ~153 . 00 B. 3.46 m
69%. Calculate the energy consumed by D. 652'.14 m/s *
B. P446.00 C. 2.64m *
the pump in 2 hours, expressed in MJ.
C. P144.00 D. 5.32 m
A. 12.68 MJ
D. P451 .00 *
B. 12.38 MJ h, + v? = h2 + vi
C. 11.68 MJ 2g(J) 2g(J) Q=AxV
Solving for the effective rate per month:
x ~ = 2:(o.os)2m 2 (V)
D. 11.38 MJ *
(80) 2 v? 1 kg
. . )m - 1
In
800 + 2(9.81)(427) = 750 + 2(9.81)(427) s 1101kg 4
I = (1+-;-
Q = 50 Mton x 1000 kg x_!!_x~ =0.4626 m/s
v2 = 652.14 m/s Velocity
. J12 hr Mton kg 1000 li 2
0.12 = (1+~1; -1 = 50 m3 /hr 59. A perfect venturi with throat diameter
Frictional head loss, hf = fLV
2gD
QwH . of 1.8 inches is placed horizontally in a
Energy = - - xt1me
i
.Jl. =0.94888% per month Eff pipe with a 5 in inside diameter. Eighty hf = (0.242) (50) (0.4626)
2
= 2 .64 m
3
12
S = P(1 + i)"
500 = P(1 + 0.0094888)
11 (
m x
SO-h-;- kN)
9.81 m3 (8m)
(80) Ibm of water flow through the pipe
each second. What is the difference
between the pipe and venturi throat static
2(9.81) (0.05)

P = P450.67 Energy = x 2 hrs pressure?


0.69
A. 29.9 lbffin2
55. The maintenance cost of a machine
= 11,373.9 kJ
B. 5020 lbf/in 2
this year is expected to be P1,000.00.
Energy = 11.37 MJ
C. 34.8 lbf/in2 *
The cost will increase P200.00 each year D. 72,3 lbf/in2
57. What is the density of the stone if the
for the subsequent 9 years. The interest
stone weighs 19.9 lbf (88 N) in air and
is 15% compounded annually. What is 80/
12.4 lbf (55 N) in water? _ Q _ 7 62.4 = 9.4 ftls
the a'pproximate worth of maintenance for
the machine over the full ten years?
A. 2655 kg/m *
B. 262b kg/m3
3
Vs - A- rr ( 5 )2
A. P4,814.67 4 12
C. 2710 kg/m 3
B. P4,481 .76
D. 2700 kg/m3 8
C. P8,414.67 *
D. P4,418.67
Q %2 4 = 72.549 ftls
'EFv = 0 Vu . = A= rr(l.8)2
P= Fe+Ww-WA = 0 4 12

R[(l+i)n - l] +A[(l+i)'' -1
i(l +i) 11 i 2 (l+i)n
_n_]
i(l+i}"
Fe= WA - Ww = 19.9-12.4 = 7.51bf
~B = V X dw
7.5 = V X 62.4 hs =
y2 (9.4)
2g = 2(32.2)
2
= 1.372 ft

v = 0 .1202 ft3 2
Where R = P1000.00 and y2 =81 .73ft
(72.549)
A = P200.00 Density of stone, d, = mass h1.8 = 2g = 2(32.2)
volume
- 19.91bf kg 35.28ft3
- x--x--
0.1202ft3 2.2lb m3
REFRESHER MANUAL 2nd Edition by JAS TORDILLO
= 2655 kg/m 3
REFRESHER MANUAL2"d Edition by JAS TORDILLO
DAY 14- EXAM
ENGINEERING MATHEMATHICS
114-1
DAY 14- EXAM 7. A rectangular array of numbers
forming m rows and n columns.
A . determinants
1. Justine is 12 years old and Tresha is 3
B. pascal triangle
years old. In how many years will Justine C. elements
be twice as old as Tresha?
D. matrix
A. 6
B. 18
8. Which of the following is NOT a
C.9 property of a binomial expansion of (x +
D. 24 y)"?
A power of x is decreasing.
2. What is the expression of three or B. power of y is increasing
more terms?
C. sum of exponents in each term = n
A. polynominal D. number of terms= n -1
B. multi
C. monomial
9. A number which can be expressed as
D. trinomial a quotient of two integers is called:
A . irrational number
3. The difference between highest score B. imaginary number
and the lowest score in the distribution. C. rational number
A deviation D. real number
B. median ·
C. range
10. The absolute value of a non-zero
D. mode
number is
A . always zero
4. Convergence series is a sequence of
B. always negative
decreasing numbers or when the C. always positive
succeeding number is the
D. sometimes zero and sometimes
preceding term. positive
A. greater than
B. equal to
11. What is the additive identity element?
C. less than AO
D. negative B. -1
c. 1
5. A statement that one mathematical D. infinity
expression is greater than or less than

- another is called:
A. absolute condition
B. inequality
C. non-absolute condition
12. What is the multiplicative identity
element?
AO
B.- 1
D. conditional expression c. 1
D. infinity
6. The part of the theorem which is
assumed to be true.
13. A Prime number is an integer greater
A. corollary than I Which has,
B. postulates A 1 as its only positive divisor
C. hypothesis
B. itself as its only positive divisor
D. conclusion

REFRESHER MANUAL 2nd Edition by JAS TORDILLO


14-2 DAY 14- EXAM DAY 14- EXAM 114-3
ENGINEERING MATHEMATHICS
C. 1 and itself as Its only positive divisors B. means
D. 1 and it additive inverse as its only pos C. mode 27. If the roots of an eq uation are zero, C. directrix
divisor D. standard deviation then they are classified as D. axis of the parabola
A. conditional solutions
14. Indicate the false statement: 21. What is the allowable error in B. hypergolic solutions 34. The equation whose roots are the
A. the objects in a set are called its measuring the edge of the tube that is C. extraneous solutions reciprocals of the roots of the equation,
elements intended to hold cubic meter, if the error D. trivial solutions 2x2 - 3x- 5 = 0.
B. every number is either rational or of the computed volume is not to exceed
irrational A. 2x2 - 5x - 3 = 0
0.02 cubic meter? Assume volume of a 28. If the first derivative of a function is a B. 5x2 - 2x- 3 = 0
C. the additive inverse of number "a" is cube equal to 8. constant, then the function is C. 5x2 + 3x - 2 = 0
1/a A. 0.0170 m A. sinusoidal D. 3x2 - 5x - 2 = 0
10. the negative of zero is zero. B. 0. 1700 m B. linear
C. 0.07190 m C. logarithmic 35. Which of the following is true?
15. If the discriminant of a quadratic D. 0.0017 m D. quadratic A. sin (-9) =sine 9
equation is less than zero, the equation
29. The cords of an ellipse, which pass B. cos (-9 ) =cos 9
has: 22. Find the equation of the directrix of through the center, are known as C. tan (-9) =tan 9
A: no real root the parabola y2 = 16x.
B: two real roots D. esc (-9 ) = esc 9
A. X= -4
A. asymptote
C. one real root B. X =4 B. radical axes 36. At maximum point the value of y" is
D. none of the above C. X= -8 C. major axes A. zero
D. X =8 D. diameters B. positive
16. Considered as the counting numbers
A. integers C. infinitive
23. The difference between an 30. If sin A = 3/5 and A is in quadrant II
• B. rational numbers D. negative
approximate value of a quantity and its while cos B = 7/25 and B is in
C. irrational numbers exact value or true value is: quadrant 1, find sin (A + B).
D. natural numbers 37. The derivative of In cos x is
A. absolute error A. -3/5 A. sec x
B., one B. 3/5
17. A prime number has exactly how I C. relative error
B. -secx
C. 4/5 C. -tan x
many divisors? D. change D. :y.
A.1 D. tan x
B.3 24. The sum of the interior angles of a 31. A horizontal line has a slope of 38. Find the area bounded by the curve
C. 2 polygon is MO degrees. Find the number A. negative defined by the equation x2 = By and its
D. 4 of sides. B. positive latus rectum.
A. 11
C. infinity A. 11/3
18. Which of the following statements is B. 8
NOT true? c.5 D. zero B. 16/3
C. 32/3
A. The sum of even numbers·is even 0.6 32. The hypotenuse of a right triangle is
B. The difference of even numbers is D. 22/3
34 em. Find the length of two legs, if one
even, 25. MCMXCIV is a Roman numeral
leg is 14 em longer than the other. 39. The altitude of the sides of a triangle
C The product of even numbers is even equivalent to:
A. 15 and 29 intersect at the point known as:
D. The quotient of even numbers is even. A. 1974
B. 18 and 32 A. incenter
B.11964
C. 16 and 30 B. circumcenter
19. The numbers between two geometric C. 1984
D. 17 and 3 1 C. orthocenter
terms D~ 1994
A. means D. centroid
33. The line passing through the focus
B. geometric means 26. Three forces 20N, 30 N and 40 N are
and is perpendicular to the directrix of a 40. When two planes intersect with each
C. arithmetic means in equilibrium. Find the angle between
parabola. other, the amount of divergence petween
D. median the 30 N and 40 N forces. A. tangent line the two planes is expressed by
A. 28.96 deg
B. latus rectum measuring the _ _.
20. A number that occurs most frequent B. 30.15 deg
in a group of numbers. A. dihedral angle
C. 40 deg
A. median D. 25.97 deg REFRESHER MANUAL 2nd Edition by JAS TORDILLO
REFRESHER MANUAL 2"d Edition by JAS TORDILLO
14.4 DAY 14- EXAM
DAY 14- EXAM 14-5 I
ENGINEERING MATHEMATHICS
B. reflex angle B. 0.1 X0 cubic inch
C. polyhedral angle C. 0.3x3 cubic inch A. 3x + 3 C. 33.21
D. plane angle D. 0.030 cubic inch B. 6x- 3 D. 13.23
C. 3x-3
41. A person draws 3 balls in succession 47. A plane is headed due east with D. 6x + 3 1
from a box containing 5 red balls, 6 airspeed of 240 mph. If a wind at 40 mph 59. What is the integral of - - with
· 3x+4
yellow balls and 7 green balls. Find the is blowing from the north, find the ground 54. What is the equation of the normal to respect to x and evaluate the result from
probability of drawing the balls in the speed of the plane. the curve y = 3x2 - 2x+ 7 at the point x = 1 to
order red, yellow and green. A:281 mph (1, 8)?
A. 0.3894 X= 3.
B. 200 mph A. X+ 4y- 33 = 0
B. 0.0894
A . 0.206
C. 274 mph B. X + 2y + 30 = 0
C. 0.03489 B. 0.406
D. 243 mph C. X - 4y - 33 = 0 C. 0.306
D. 0.04289 D. X- 2y + 30 = 0 D. 0.506
48. Find the length of the vector (2.4. 4).
42. For a particular experiment you need A. 7.00 55. Jojo bought a second hand betamax 60. Find the slope of the curve l? + y2-
5 liters of a 10% solution. You find 7% B. 8.50 VCR and then sold it to _Rudy at a profit of 6x + 10y + 5 = 0 at point (1, 0).
and 12% solution on the shelves. How C. 6.00 40%. Rudy then sold the VCR to Noel at
much of the 7% solution should you mix
A. 1/3
D. 5.18 a profit of 20%. If Noel paid P2856 more
with the appropriate amount of the 12%
B. 2/5
than it cost to Jojo, how much did Joho C.%
solution to get 5 liters of a 10% solution. 49. Find the 1OOth term of the sequence, paid the unit? D. 1/5
A. 1.5 1.01, 1.00, 0.99, .... A. P4000 I
B. 3 A. 0.02 B. P4200
c. 2 B. 0.05 C. P4100
D. 2.5 C. 0.03 D. P4300 rt3
D. 0.04
43. A circle with a radius of 6 has half of DAY 14- ANSWER KEY
56. Find the area bounded by the
its area removed by cutting a border of 1.A 16.0 3L 0 46.0
50. If a regular polygon has 27 diagonals, parabola x2 = 16(y - 1) and its latus 2.A 17.0 32.C 47.0
uniform width. Find the width of the then it is a: rectum. 3.C 18.0 33.0 48. c
border. A. nonagon A. 56.27 4.C 19. B 34. C 49.A
A. 2.2 B. hexagon B. 42.67 5. B , 20. C 35. B 50. A
B. 1.76 C. p·entagon C. 46.27 6.C 21.0 36.0 51. A
c. 1.35 D. heptagon D. 52.67 7.0 22.A 37. c 52. c
D. 3.75 8.0 23.A 38. c 53. 0
51. Integrate x cos (2lt + 7) dx 9.C 24.( 39. C 54. A
57. Evaluate the integral of (31- 1)3 dt. 10. c 25.0 40.A 55. B
44. The roots of a quadratic equation are A. V. sin(2Jt + 7) + c 41.0 56. B
11. A 26. A
1/3 and Y. .What is the equation? B. sin (2x2 + 7) + c A. ..!...(3t -1)4 + C 12. c 27.0 42. C 57. A
=
A. 12Jt .:.. 7x + 1 0 C. Y. cos(2Jt + 7) + c 12 13. C 28. B 43. B 58. A
B. 12Jt- 7x- 1 = 0 D. sin (Jt + 7) + c 14. c 29.0 44. A 59. A
C.12lt+7x+1=0 B. .!_ (3t -1)4 + C 15. A 30. A 45. B 60. B
D. 12Jt + 7x-1 = 0
3
52. A box is to be constructed from a
piece of zinc 20 sq. in by cutting equal c . .!.(3t-1)4 + c
45. A line, which is perpendicular to the squares from each corner and turning up 4
x-axis, has a slope equal t o _ . the zinc to form the side. What is the
A.1 volume of the largest box that can be so D. _.!._(3t-1)3 +C
B. infinity constructed? 12
c. 0 A. 599.95 cu. in
D. -1 B. 579.50 cu. in 58. Find the area bounded by the
C. 592.59 cu . in parabola, l? = 4y and y = 4.
46. Find the approximate change in (he D. 622.49 cu. in A. 21 .33
volume of a cube of side "x" inches B. 31.32
caused by increasing its side by 1%. 153. What is the derivative of the function
3
A. 0.02x cubic inch with respect to x of (x + 1 )3 - 0. REFRESHER MANUAL 2nd .Edition by JAS TORDILLO
REFRESHER MANUAL 2"d Edition by JAS TORDILLO
14-6 DAY 14 - SOLUTION DAY 14 - SOLUTION 114- 7
C. hypothesis*
ENGINEERING MATHEMATHICS
DAY 14 - SOLUTION D. conclusion
C. 1 and itself as Its only positive C. arithmetic means
1. Justine is 12 years old and Tresha is 3 7. A rectangular array of numbers divisors* D. median
years old. In how many years will Justine forming m rows and n columns. D. 1 and it additive inverse as its only pes
be twice as old as Tresha? A. determinants divisor · 20. A number that occurs most frequent
A. 6 * B. pascal triangle in a group of numbers.
B. 18 C. elements 14. Indicate the false statement: A. median
C. 9 D. matrix* A. the objects in a set are called its B. means
D. 24 elements C. mode*
8. Which of the following is NOT. a B. every number is either rational or D. standard deviation
Let x = no. of years the age of Justine be property of a binomial expansion of (x + irrational
twice, as Tresha y)n? C. the additive inverse of number "a" 21. What is the allowable error 'in
A. power of x is decreasing. is 1/a * measuring the edge of the tube that is
12 +X= 2(3 +X) B. power of y is increasing 10. the negative of zero is zero\ intended to hold cubic meter, if the error
x = 6 years C. sum of exponents in each term = n of the computed volume is not to exceed

2. What is the expression of three or


=
D. number of terms n -1 * 15. If the discriminant of a quadratic
equation is less than zero, the equation
0.02 cubic meter? Assume volume of a
cube equal to 8.
more terms? 9. A number which can be expressed as has: A. 0.0170 m
A. polynominal * a quotient of two integers is called: A. no real root * B. o. 1700 m
B. multi A. irrational number B. two real roots C. 0.07190 m
C. monomial B. imaginary number C. one real root D. 0.0017 m *
D. trinomial C. rational number * D. none of the above
D. real number Let x = be the side of a cube
3. The difference between highest score 16. Considered as the counting numbers
and the lowest score in the distribution. 10. The absolute value of a non-zero A. integers V=i'
A deviation number is B. rational numbers 8 = x3
B. median A. always zero C. irrational numbers x=2
c. range* B. always negative D. natural numbers *
D. mode C. always positive* V= x3
D. ·sometimes zero and sometimes 17. A prime number has exactly how dV= 3x2 dx
4. Convergence series is a sequence of positive many divisors? 0.02 = 3(2)2 dx
decreasing numbers or when the A.1 dx = 0.00166
succeeding number is the 11. What is the additive identity element? 8. 3
preceding term. A. 0* c. 2 22. Find the equation of the directrix of
A . greater than B. -1 D. 4 * the parabola y2 = 16x.
B. equal to c. 1 A. X= -4 *
C. less than * D. infinity 18. Which of the following statements is B. X =4
D. negative NOT true? C. X =-8
12. What is the multiplicative identity A. The sum of even numbers is even D. X= 8
5. A statement that one mathematical element? B. The difference of even numbers is
expression is greater than or less than A.O even, I= 16x
another is called: B. - 1 C The product of even numbers is even 4a = 16
A. absolute condition c. 1 * D. The quotient of even numbers is a=4
B . inequality* D. infinity even.* directrix, x = -4
C. non-absolute condition
D. conditional expression 13. A Prime number is an integer greater 19. The numbers between two geometric 23. The difference between an
than I Which has, terms approximate value of a quantity and its
6. The part of the theorem which is A. 1 as its only positive divisor A. means exact value or true value is:
assumed to be true. B. itself as its only positive divisor B. geometric means * A. absolute error*
A. corollary 1;3., one
B. postulates
REFRESHER MANUAL 2nd Edition by JAS TORDILLO
REFRESHER MANUAL 2"d Edition by JAS TORDILLO
DAY 14 - SOLUTION \ 14-9
ENGINEERING MATHEMATHICS
14-8 DAY 14- SOLUTION
C. relat1ve error 29. The cords of an ellipse, which pass 33. The line passing through the focus sinx d
D. change
dy= - - X
through the center, are known as and is perpendicular to the directrix of a cosx
parabola.
24. The sum of the interior angles of a A. asymptote A. tangent line dy =-tanx
polygon is 540 degrees. Find the number B. radical axes B. latus rectum
dx
of sides C. major axes C. directrix
A 11 38. Find the area bounded by the curve
D. diameters • D. axis of the parabola *
B. 8 defined by the equation ~ = By and its
C. 5 * 30. If sin A = 3/5 and A is in quadrant II 34. The equation whose roots are the latus rectum.
D. 6 while cos B = 7/25 and B is in reciprocals of the roots of the equation, A . 11/3
quadrant 1 find sin (A + B). 2x2 - 3x - 5 = 0. B. 16/3
S=(n-2)180 A. -3/5 * A. Y -5x -3=0 c. 32/3 *
540 = (n- 2)180 B. 3/5 B. 5~ - 2x - 3 = 0 D. 22/3
n = 5 s1des C. 4/5 c. 5x 2
+ 3x- 2 =o• ~ =By
D. y. D. 3x2 - 5x- 2 =0
25. MCMXCIV is a Roman numeral
equivalent to· Sin(A +B)= sin 'A cos B +cos A sin B 2~- 3x- 5 = 0 4a = 8

ls3y~ 2s7) + (-4X


5 2")
A 1974 (2x- 5)(x + 1) = 0 a=2
B 11964 = 25 5
c 1984 21 96
x = -and x = -1 base = 4a = 4(2) = 8
D. 1994 • =--- 2 height= a= 2
125 125
1=-1, V=5, X=10, L=50, -75 - 3 Reciprocal:
c = 1'oo,
D = 500, M = 1000
"'-=-
125 5 2 Area = 3. base x height
x = -and x =-I 3
5 2 (, 32
M+CM +XC +IV 31 A horizontal line has a slope of Area= -\sX2)=-
1000 + 900 + 90 + 4 = 1994 A. negat1ve
B positive
(x-~)x+l)= O 3 3

26. Three forces 20N, 30 N and 40 N are C. infinity 5x2 + 3x - 2 =0 39. The altitude of the sides of a triangle
1n equilibrium Find the angle between D. zero* intersect at the point known as:
the 30 N and 40 N forces 35. Which of the following is true? A. incenter
A. 28.96 deg * 32. The hypotenuse of a right triangle is A. sin'(-9) =sine 9 B. circumcenter
B. 30.15 deg 34 em Find the length of two legs, if one e. cos ( -9 ) = cos 9 • C. orthocenter *
C 40 deg leg is 14 em longer than the other c. tan (-9) =tan 9 D. centroid
D. 25 97 deg A 15 and 29 D. esc (-9) =esc 9
B 18 and 32 40. When two planes intersect with each
202 = 402 + 302 - 2(40)(30) cos e C. 16 and 30 * 36. At maximum point the value of y" is other, the amount of divergence between
e =28.955.0 D. 17 and 31 A. zero the two planes is expressed by
B. positive measuring the _ _.
27 If the roots of an equation are zero, b =a+ 14 C. infinitive A. dihedral angle *
then they are classified as D. negative * B. reflex angle
A. conditional solut1ons 342 = a2 + b2 C. polyhedral angle ·
B hypergolic solutions 342 = a2 +(a+ 14)2 37. The derivative of In cos xis D. plane angle
2
C. extraneous solutions 1156 = a + a2 + 28a + 196 A. sec x
D. trivial solutions * 2
2a + 28a + 960 0
2
= B. -secx 41. A person draws 3 balls in succession
a + 14a + 480 = 0 C. -tan x* from a box containing 5 red balls, 6
28. If the first derivative of a function is a (a-16)(a-30)=0 D. tan x yellow balls and 7 green balls. Find the
constant, then the function is a= 16 probability of drawing the balls in the
A. sinusoidal =
b 16 + 14-= 30 y =In cos x 1 order red, yellow and green.
B. linear*
C logarithmic REFRESHER MANUAL 2nd Edition by JAS TORDILLO
D quadratic

REFRESHER MANUAL 2nd Edition.bv JAS.TORDII Ln


"''iW ~-

14-10 DAY 14 - SOLUTION DAY 14 - SOLUTION 114-11


A. 0.3894 x = 1/3 and x = Y. ENGINEERING MATHEMATHICS
B. 0.0894
C. 0.03489
D. 0.04289 *
(x-~)(x-±)=o 49. Find the 100'" term of the sequence,
1.01, 1.00, 0.99, .. ' .
volume of the largest box that can be so
constructed?

Prob = (~)(~)(}_) = 0.04289


-'X -J)=o
(3 x
3
4x
4
A. 0.02 *
B. 0.05
C. 0.03
A. 599.95 cu. in
B. 579.50 cu . in
C. 592.59 cu. In •
18 >17 16 12x2 - 3x - 4x + 1 = 0 D. 0.04 D. 622.49 cu. in
12x2 -7x + 1 = 0
42. For a particular experiment you need d = a2- a1 = 1.00- 1.01 = -0.01 V(x) = (20- 2x)2 x
5 liters of a 10% solution. You find 7% 45. A line, which is perpendicular to the n = 100 V'(x) = (20- 2x)2 ·+ x(2)(20- 2x)(-2)
and 12% solution on the shelves. How x-axis, has a slope equal to_ .
(20- 2x)2 - 4x(20- 2x)
much of the 7% solution should you mix A. 1 L=a1+(n-1)d = (20 - 2x) (20 - 2x - 4x)
B. infinity *
with the appropriate amount of the 12%
c. o
L = 1.01 + (100 - 1)(-0.01) = (20 - 2x) (20 - 6x)
solution to get 5 liters of a 10% solution. L = 0.02 = 4(10- x)(10- 3x)
A. 1.5 D. -1
B. 3 50. If a regular polygon has 27 diagonals, Equating V'(x) = 0
C. 2 * 46. Find the approximate change in the
then it is a: 4(10-x)(10-3x) = 0
D. 2.5 volume of a cube of side "x" inches
A. nonagon • 10 - x = 0 10 - 3x = 0
caused by increasing its side by 1 %.
3 B. hexagon X= 10 X = 10/3
Let x = liters of 7% solution A. 0.02x cubic inch C. pentagon
3
8 . 0.1x cubic inch D. heptagon For V maximum at x = 1013
0.07(x) + 0.12(5 - x) = 5(0.1 0) C. 0.3x3 cubic inch
0.07x + 0.6- 0.12x = 0.5
0.05x = 0.10
D. 0.03x3 cubic inch *
D= ~(n-3) v = [20-2C3°JTC~)
x=2 v1 =x3 2
X -+ (1 +0.01)X = 1.01X V = 592.59 cu. in
3
v2 = (1 .01x) = 1.03x
3 27 = ~(n-3)
43. A circle with a radius of 6 has half of 2 53. What is the derivative of the function
its area removed by cutting a border of 54= n 2 - 3n with respect to x of (x + 1)3 - x3 .
uniform width . Find the width of the AV = V2-V1 n2 - 3n- 54= 0
= 0.03x3 - x3 A. 3x + 3
border. 3 (n - 9)(n+ 6) = 0 B. 6x- 3
A. 2.2 = 0.03x n = 9 (nonagon) C. 3x- 3
B. 1.76 * D. 6x + 3 *
C. 1.35 47. A plane is headed due east with 51 . Integrate x cos (2x2 + 7) dx
D. 3.75 airspeed of 240 mph. If a wind at 40 mph A. y. sin(2x2 + 7) + c *
is blowing from the north, find the ground y = (x +1 )3 - x3
B. sin (2x2 + 7) + c y' = 3(x + 1)2 - 3~
As = area big circle speed of the plane. C. Y. cos(2x2 + 7) + c
A. 281 mph y' = 3x2 + 6x + 3 - 3x2
As = are small circle D. sin (x2 + 7) + c y' = 6x + 3
B. 200 mph
A( shaded) = Ae -As

.!.n(6) 2= 2
n:(6) 2
m
C. 274 mph
D. 243 mph * y = f x cos(2x 2 + 7)dx 54. What is the equation of the normal to
the curve y = 3x2 - 2x+ 7 at the point

±f
-
2 2 (1, 8)?
r = 4.24 V = J240 2 + 40 2 =243.3 mph y = cos(2x + 7)4xdx A. X + 4y - 33 = 0 *
t = R- r=6-4.24 = 1.76 B. X + 2y + 30 = 0
I . J
48. Find the length of the vector (2,4, 4). y= -stn{2x -+7)+C C. X - 4y - 33 = 0
4
44. The roots of a quadratic equation are A. 7.00 D. X- 2y + 30 = 0
1/3 andY. .What is the equation? 8. 8.50 52. A box is to be constructed from a y = 3~- 2x + 7
A. 12x2 -7x + 1 :: 0 * c. 6.00 * piece of zinc 20 sq. in by cutting equal
B.12x2 - 7x-1 = 0 D. 5.18 dy
squares from each corner and turning up slope - = 6x - 2
C. 12x2 + 7x + 1 = 0 ' dx
the zinc to form' the side. What is the
D. 12x2 + 7x- 1 = 0 2 2 2 m = 6(1) - 2 = 4
, L= J2 +4 +4 =6
REFRESHER MANUAL 2nd Edition by JAS TORDILLO
REFRESHER MANUAL 2"d Edition by JAS TORDILLO
~-.- ~"

14-12 DAY 14 - SOLUTION


DAY 14- SOLUTION 14-13 I
ENGINEERING MATHEMATHICS
I
Slope of normal line, mn = A = .!.§_((5-1)-(1-l)jYz
4 3 B. 0.406
y- Y1 = mn (X- X1) C. 0.306
A = .!.§_(4)%
y- 8 -- - -I (X- 1) 3 D. 0.506
4 A = 42.67
4y- 32 = -x + 1 3 3
3
X+ 4y - 33 = 0

55. Jojo bought a second hand betamax


57. Evaluate the integral of (3t - 1 )3 dt.

A. _!_(3t-1)4 +C *
f
I
- I - dx "' -If --=-ln(3x
Jx + 4
3dx I
3 3x + 4 3
1
+4) 1
1

VCR and then sold it to Rudy at a profit of 12


= ~[~n(3 · 3+ 4)-1n(3 · l + 4)}
40%. Rudy then sold the VCR to Noel at
a profit of 20%. If Noel paid P2856 more
B. ..!.(3t-1)4 +C
3 _!_[(1n(13)-ln(7)] = 0.206
than it cost to Jojo, how much did Joho 3
4
paid the unit? C . .!.(3t -1) + C
A. P4000 4 60. Find the slope of the curve x?- + / -
B. P4200 * 3 6x + 10y + 5 = 0 at point (1, 0).
C. P4100 D. ...!..(3t-I) +C
12 A. 1/3
D. P4300 B. 215 *
C.%
Let x = Jojo's amount paid for the unit J{3t -1 )3 dt = _!_ f{3t - I)3 (3dt) D. 1/5
1.4 x =selling price to Rudy 3
( 1.2)( 1.4x) = selling price to Noel x2 + y2 - 6x + 1Oy + 5 =0
= ...!_ (3t - I)4 + C 2x + 2y y' - 6 + 10 y' = 0
(1.2)(1.4x) = x + 2856 12
X + y y' - 3 + 5 y' = 0
X= P4200
58. Find the area bounded by the - 3-- -
Y. -
x
56. Find the area bounded by the parabola, x?- = 4y andy= 4. 5+ y
parabola x?- = 16(y - 1) and its latus A. 21.33*
rectum. B. 31 .32 at Point (1, 0)
A. 56.27 c. a3.21 3-1
B. 42.67 * D. 13.23 y'=m= - -
5+0
c. 46.27
D. 52.67 4
= -2· orm =slope=-2
LR = 4a = 16
Area = J 2xdy
y'
5 5
0
Vertex (0, 1) and a = 4
5 Where: x = ,f4Y
A = 2J xdy 4

1 Area = J 2./4Ydy
5 0
A = 2 f .j16(y - I) dy
2(2)~y)X ]
4
I Area = = 21.33 sq. units

I
5 2 0
A.= 2(4) (y -I)Yi dy 1
59. What is the integral of - - with

r
I 3x +4
respect to x and evaluate the result from
- 8(y -1)3/2
A- - x = 1 to
X I x= 3.
A. 0.206 * REFRESHER MANUAL 2nd Edition by JAS TORDILLO
REFRESHER MANUAL 2"d Edition by JAS TORDILLO
DAY 15- EXAM
ENGINEERING MATHEMATHICS
115-1
DAY 15- EXAM x3 x2
C. - - - + X
3 2
1. The unit prefix nano is opposite to: x4 x2
A. Mega D. - - -+X+C
4 2
B. Tera
C. Giga
7. The only point of inflection on the
D. Hexa 3
curve representing the equation y = x +
2. If the eccentricity is less than one, then
Y! - 3 is at x equals:
A. -2/3
the curve is:
B. O
A. parabola
C. -1/3
B. hyperbola
D. 2/3
C. ellipse
D. circle
8. Naperian logarithm have a base of:
3. If the product of the slopes of any two
A. 3.1416
B. 10
straight lines is negative 1, one of these
lines are said to be:
c. 2.171828
A. parallel D. 2.71828
B. perpendicular 1130
9. (5.743) equals:
C. skewed
A. 1.03
D. non-intersecting
B. 1.05
4. Ivory soap floats in water because:
C. 1.04
D. 1.06
A. all matter has mass
B. the density of ivory is unity
10. If y =cos x , dy/dx =?
C. the specific gravity of ivory soap is
A , sinx
greater than that of water .
D. the specific gravity of ivory soap is
B. 1/sec x
less than that of water
C. -tan x cos x
D. sec x sin x
5. What pressure is a column of water
11. About how long will it take for
100 em high equivalent to?
P10,000 invested at 5% per year,
A. 9810 dyne/cm2
compounded annually, to double in
B. 0.1 bar
value?
C. 9810 N/m2
A. 5 years
D.0.1atm
B. 14.2 years
C. 9.8 years
6. The integral of y = x3 - x + 1 is:
D. 19.8 years
A. 3>?-- 1 + c
12. If P200 is deposited in a savings
x4 x2 account at the beginnings of each of 15
B. - - - + l+C
3 2 years and the account draws interest at
7% per year, compounded annually, the
value of the account at the end of 15
years will be most nearly.
A. P5000
B. P6000
REFRESHER MANUAL 2nd Edition by JAS TORDILLO
1s-2 DAY15-EXAM
DAY 15- EXAM 115-3
ENGINEERING MATHEMATHICS
C. P5400 I C. P9000
D. P6900 D. P4224 A. P2665 A. unchanged
B. P50442 B. minimum
13. How many months at an interest rate 19. P1000 is borrowed for one year at an C. P8552 C. doubled
of 1% per month does money have to be interest rate of 1% per month. If this D. P82000 D. maximum
invested before it will double in value? same sum of money is borrowed for the
A. 59 months same period at an interest rate of 12% 24. What present sum would be needed 30. Which of the following is/are short-
B. 70 months per annum, the saving in interest charges to provide for annual end-of-year range forces?
C 62 months would be: payments of P15 each, forever? Assume A. Gravitational force
D. 76 months A. PO.OO interest is 8%. B. weak nuclear force
B. P7.00 A. P120.00 C. electromagnetic force
14. A department store charges one and C. P3.00 B. P150.50 D. strong nuclear force
one-half percent interest per month on D. P14.0 C. P137.50
credit purchases. This is. equivalent to a D. P187.50 31. What happens to the mechanical
nominal annual interest rate of: 20. A company deposits P1000 every advantage of a lever if its effort distance
A.1 .5% year for ten years in a bank. The 25. Given a sum of money Q that will be increases?
B. 15% company makes no deposits during the received six years from now. At 5% A. increases
C.18% subsequent five years. If the bank pays annual interest the present worth now of B. remains the same
IJ. 19.6% 8% interest, the amount in the account at Q is P60.00. At this same Interest rate, C. decreases
the end of 15 years is nearest to: would be the va lue of Q ten years from D. becomes zero·
15. A bank pays one percent interest on A. P10,800 now?
savings account four times a year. The B. P16,200 A. P60.00 32. What kind of energy is present
effective annual interest rate is: C. P15,000 B. P90.00 whenever a body is at a distance from
A. 1.00% D. P21,286 C. P76.58 the ground?
B. 4.00% D. P97.74 A. elastic potential energy
C. 1.04% 21 . P25,000 is deposited in a savings B. electromagnetic potential energy
D. 4.06% account that pays 5% interest, 26. What law states that, within the C. electric potential energy
compounded semi-annually. Equal elastic limit, strain is directly proportional D. gravitational potential energy
16. What interest rate, compounded annual withdrawals are to be made from to stress?
quarterly, is equivalent to a 9.31% the account, beginning one year from A. Ohm's Law 33: An object's acceleration as-it starts to
effective interest rate? now and continuing forever. The B. Hooke's Law fall is:
A . 2.25% maximum amount of the equal withdrawal C. Lenz's Law A. equal tog
B. 4.66% is closest to: D. Coulomb's Law B. less than g
c. 2.33 A. P625 C. greater than g
D. 9.00% B. P1250 27. Wh ich of the following is a measure D. zero
C. P1000 of the inertia of a body?
17. An individual wishes to deposit a D. P1265 A. weight 34. How is pressure related to depth?
certain quantity of money now so that at B. volume A. directly proportional
the end of five years he will have P500. 22. What present sum would need to be C. density B. no effect
With interest at 4% per year, put in a savings account now to provide a D. mass C. inversely proportional
compounded semi-annually, how much P1000 annual withdrawal for 50 years, if D. equal to one another
must be deposited now? interest is 6%? The present sum is closet 28. What are the three fundame ntal
A. P340.30 to: quantities? 35. Which of the following is a scalar
B. P410.17 A. P1000 A. volume , weight, time quantity?
C. P400.00 B. P25,000 B. density, mass, volume A. acceleration
D. P416.95 C. P15,762 C. length, mass, time B. distance
D. P37,500 C. displacement
D. speed, distance, time
18. The present worth of an obligation of D. velocity
P10,000 due in 10 years if money is 23. What amount of money deposited 50 29. What is the speed of a projectile if it is
worth 9% is nearest to: years ago at 8% interest would now at the top of its path?
A. P10,000 provide a perpetual payment of P10,000
B. P7500 per year? The amount is nearest to: REFRESHER MANUAL 2nd Edition by J~S TORDILLO
REFRESHER MANUAL 2"d Edition by JAS TORDILLO
- - - - -- - -- --- - - - --- - -- -- - -

15-4 DAY 15- EXAM DAY 15- EXAM


ENGINEERING MATHEMATH'ICS
115-5
36. Wflich of the following would float on C. glass
water? D. iron C.45
A iron 55. Find the eccentricity of an ellipse
D. 75° when the length of the latus rectum is 2/3
B. wood 43. What is the device used to convert
of the length of the major axis.
C. stone mechanical energy to electrical energy? 50. A bullet is fired from a gun at an A. 0.58
D. mercury A. generator angle of 40°. What is the range if its B. 0.78
B. heat pump velocity is 300 m/s? g = 10 m/s2 . C. 0.68
37. What is Boyle's law in - equation C. motor A. 192.9 m D. 0.98
form? D. rotor B. 8863 m
A VN2 = P2P1 C. 229.8 m 56. Determine the coordinates of the
B. V1N2 = P,/P2 44. A force of 2d N acts on an object for 5 D. 12000 m point which is three-fifths of the way from
C. V1P2 = k seconds. What is the impulse?
the point (2, 5) to the point (-3, 5).
D. V1N2 = P2/P, A. 0.25 s/N 51 . A carpenter's ladder which is 25 feet A. (-1, 1)
B. 15 N-s long leans against a wall. The top of the B. (-1, -2)
38. What law stp.tes that there is C. 4 N/s ladder is sliding against the wall at the C. (-2, -1)
decrease in pressure at the side of a D. 100 N-s rate of 1 fps. Find the rate at which the D. (1, 1)
moving stream of fluid?
bottom of the ladder is sliding when the
A Bernoulli's Law 45. What would be the strain produced bottom is 7 feet from the base of the wall. 57. Find the equation of a line which
B. Archimedes Law on a wire of length 100 em and an A. 3.43 fps passes through the point (2, 1) and
C. Pascal's Law increase in length of 0.5 em? B. 5.67 fps perpendicular to the line whose equation
D. Hooke's Law A. 0.05 C. 4.33 fps is y = 4x + 3.
B. 100 D. 694 fps A. X -4y + 6 = 0
39. The interactions responsible for C. 0.005
B. X +4y -6 = 0
holding together electrons to their atom D. 200 52. Water from the filtering facility is C. y -4x + 6 = 0
as well as for combining atoms and pouring into a swimming pool. After n D. y- 4x + 6 = 0
molecules to produce larger bodies are: 46. What determines the pitch of sound?
hours, there are · n + ...J n gallons in the
A. strong nuclear A. amplitude
pool. At what rate is the water pouring 58. What is the diameter of a circle with
B. weak nuclear B. wavelength
into the pool when n = 16 hours? the following equation:
C. electromagnetic C. frequency
D. gravitational D. speed
A. Y. gph · x2 + l- 6x + 4y- 12 = 0
B. 1 gph A: 8
C. 9/8 gph B. 5
40. Which of the following processes 47. A car's- speed increases from 30 m/s
D. 7/6 gph C. 6
occurs when the heat passes from place to 40 m/s in 5 sec. What is its
D. 10
to another, from molecule to molecule? acceleration?
53. A box is to be constructed from. a
A. convection A. -2m/s2
piece of zinc 20 sq. in by cutting equal 59. The equation of a line that intercepts
B. conduction B. 2 m/s 2
squares from each corner and turning up the x-axis at x = 4 and the y-axis at
C. radiation C. -14 m/s2
the zinc to form the side. What is the y = -6 is:
D. expansion D. 14 m/s2 volume of the largest box that can be so A. 3x + 2y = 12
constructed? B. 3x - 2y = 12
41. Which of the following, is the 48. An object dropped from the roof of
A. 599.95 CU. in C. 2x-3y=12
temperature at which the gas would no the building hits the ground 3 sec later.
B. 579.50 cu . in D. 2x - 37 = 12
longer exert pressure? How high is the building? (g = 10 m/S2)
C. 592.59 cu . in
A Kelvin temperature A. 45m
D. 622.49 cu. in 60. Find the value of x:
B. absolute zero B. 13 m
logs (2x- 5) = log4 6
C. Celsius scale temperature C. 30m
54. What is the derivative of the function A. 7.57
D. absolute temperature D. 3.33 m with respect to x of (x + 1)3 - x3 . B. 9.66
A. 3x + 3 C. 8.45
42. Which of the following substances 49. At what angle does a projectile go B. 6x- 3 D. 10
has the greatest specific heat? farthest when fired? C. 3x - 3
A. aluminum A . 30° D. 6x + 3
B. alcohol B. 60°
REFRESHER MANUAL 2nd Edition by JAS TORDILLO
REFRESHER MANUAL 2"d Edition by JAS TORDILLO
15-6 .DAY 15 -SOLUTION DAY 15 - SOLUTION 115-7
ENGINE~RING MATHEMATHICS . .
DAY 15- SOLUTION
ld x4
D. ---+X+C
x2 * 11. About how long will it take for
P10,000 invested at 5% per year,
1. The unit prefix nano is opposite to:
DAY 15- ANSWER KEY 4 2 compounded annually, to double in
A. Mega
1. c 16.0 31. A 46. C value?
2. c 17. B 32. 0 47. B
B. Tera
3. B lB. 0 33. A 48. A C. Giga *
D. Hexa
y= J(x 3 -x+ t) A . 5 years
B. 14.2 years*
4.0 19. B 34. A 49. C
s.c 20.0 35.8 so. 8 x4 x2 C. 9.8 years
6. 0 21.0 36.8 51. A 2. If the eccentricity is less than one, then y= ---+X+C D. 19.8 years
7.C 22. c 37.0 52. C
4 2
the curve is:
8. 0 23. A 38. A 53. C S = P(1 + i)"
A. parabola
9. 0 24.0 39. C 54.0 7. The only point of inflection on the 20,000 = 10,000(1 + 0 .05)"
B. hyperbola 3
10. C 25. 0 40.8 55. A curve representing the equation y = x + ln2=nln1.05
11. 8 26. 8 41.8 56. A C. ellipse* x2 - 3 is at x equals:
12. C 27. 0 42. B 57. B D. circle n = 14.2 y rs
A. -2/3
13. B 28. C 43. A 58. D
14. C 29. B 44. 0 59. B
B. 0 12. If P200 is deposited in a savings
3. If the product of the slopes of any two
15.0 30.0 45. C 60. A C. -1/3 * account at the beginn ings of each of 15
(1 straight lines is negative 1, one of these
D. 2/3 years and the account draws interest at
lines are said to be:
A. parallel 7% per year, compounded annua lly, th.e
y=x3 +x2 -3
B. perpendicular * y' = 3x2 + 2x
value of the account at the end of 15
C. skewed years will be most nearly.
y" = 6x + 2 = 0 A. P5000
D. non-intersecting
6x = -2 B.P6000
4. Ivory soap floats in water because:
X= -1/3 C. P5400*
A. all matter has mass. D. P6900
8. Naperian logarithm have a base of:
B. the density of ivory is unity
A. 3.1416
C. the specific gravity of ivory soap is
B. 10 P .= R+R[(l+if-1]
greater than that of water
C. 2.171828 i(l+if
D. the specific gravity of ivory soap is
less than that of water*
D. 2.71828 *
,
5. What pressure is a column of water
9. (5.143)
1130
equals: p = 200 '+ 200 [_,_(t_+_0.0_71_4~-t-'--)]
14
A. 1.03 O.D7(1+0.07)
100 em high equivalent to?
B. 1.05
A. 9810 dyne/cm2 p = 1949.09
C. 1.04
B. 0.1 bar
C. 9810 Nfm 2 *
D. 1.06 * s = 1949.09(1 + 0.07)15 = 5377.61
D. 0.1 atm 1130 13. How many months at an interest rate
(5.743) = 1.0599
2 of 1% per month does money have to be
Pressure= yh = 9810(1) = 9810 N/m
10. If y =cos x, dy/dx =? invested before it will double in value?
3 A. sin x A. 59 months
6. The integral of y = x - x + 1 is:
B. 1/sec x B. 70 months *
C. -tan x cos x * C. 62 months
A.3~-1+c
D. sec x sin x D. 76 months
x4 x2 .
B. - - - + l+ C
3 2 y =COS X S = P(1 + i)"
y' =sin x 2P = P(1 + 0.01)"
sinx In 2 = n In 1.01
x3 x2 tan x = - -
C. - - - + X n = 69.66 months
3 2 cosx
y' = -tan x cos x
REFRESHER MANUAL 2nd Edition by JAS TORDILLO
REFRESHER MANUAL 2"d Edition by JAS TORDILLO
1s-s DAY 15- SOLUTION DAY 15 - SOLUTION 115-9
14. A department store charges one and C. P400.00 ENG-INEERING MATHEMATHICS
one-half percent interest per month on D. P416.95
provide a perpetual payment of P10,000
credit purchases. This is equivalent to a
nominal annual interest rate of: n =5 yrs =10 semi-annuals s = 1000 [(1+ o.osyo -I ](1 + o.o8)5 per year? The amount is nearest to:
0.08 A. P2665 *
A. 1.5%
B. 15% . annu aJ
s = 21 ,285.51 B. P50442
C. 18% *
.1 = -
4%- = 20"
,o per semt- C. P8552
2 D. P82000
D. 19.6% 21 . P25,000 is deposited in a savings
account that pays 5% interest,
S = P(1 +it
i = 1.5% per month 500 = P(1 + 0.02) 10
compounded semi-annually. Equal P= ~
annual withdrawals are to be made from i
p = 410.17
nominal, In= 1.5(12) the account, beginning one year from p = 10,000
= 18% compounded monthly now and continuing forever. The 0.08 = 125,000
18. The present worth of an obligation of
P10,000 due in 10 years if money is maximum amount of the equal withdrawal
Po= 125,000
15. A bank pays one percent interest on worth 9% is nearest to:
is closest to: (l + o.o8)SO = 2665.15
savings account four times a year. The A. P625
A. P10,000
effective annual interest rate is: B. P1250
B. P7500 24. What present sum would be needed
A. 1.00% C. P1000
C. P9000 ~
B. 4.00% D. P1265 * to provide for annual end-of-year
D. P4224 * payments of P15 each, forever? Assume
C. 1.04%
D. 4.06% * S = P(1 + i)n i = (I+ o.~sr -I = 5.0625% per annum interest is 8%.
A. P120.00
10,000 = P(1 + 0.09) 10
ip = 1% per quarter R B. P150.50
p =4224.10 P=-
m = 4 (quarterly) i C. P137.50
D. P187.50 *
4 19. P1 000 is borrowed for one year at an R
Effective annual rate, i = (1+0.01) - 1 25,000= - -
interest rate of 1% per month. If this 0.0506
:: 0.0406 :: 4.06%
same sum of money is borrowed for the R = 1265.62 P=~=~
same period at an interest rate of 12% i 0.08
16. What interest rate, compounded per annum, the saving in interest charges p:;: 187.50
22. What present sum would need to be
quarterly, is equivalent to a 9.31% would be: - put 1n a savings account now to provide a
effective interest rate? A. PO.OO P1 000 annual withdrawal for 50 years, if
25~ <?iven a sum of money Q that will be
A 2.25% B. P7.00 * received six years from now. At 5%
interest is 6%? The present sum is closet
B. 4.66% C. P3.00 annual interest the present worth now of
to:
c. 2.33 D. P14.0 A. P1000 Q is P60.00. At this same interest rate,
D. ~.00% * B. P25,000 would be the value of Q ten years from

;r
now? ·
s1 :: 1000(1 + 0.01 )12 = 1126.82 C. P15,762 *
= 1oooc1 + o.12)1 = 1120 A. P60.00
i= ()+ - I
s2 D. P37,500
B. P90.00

0.0931 =(I+ i; r_,


in = 0.09 = 9% compounded quarterly
Savings= 1126.82-1120 = P6.82

20. A company deposits P1000 every


year for ten years in a bank. The
company makes no deposits during the
p:: R[(l+i)" ~ IJ
1(1 + i)
p = 1000 [ (I + 0.06)50 -I ]
.
C. P76.58
D. P97.74 *

p = Q = 60.00
s = 60(1 + 0.05)10
subsequent five years. If the bank pays s = 97.73
0.06(1 + 0.06)50
17. An individual wishes to deposit a 8% interest, the amount in the account at
certain quantity of money now so that at the end of 15 years is nearest to: p = 15,761 .86 26. What law states that, within the
the end of five years he will have PSOO. A. P10,800 elastic limit, strain is directly proportional
With interest at 4% per year, B. P16,200 23. What amount of money deposited 50 to stress?
compounded semi-annually, how much C. P15,000 years ago at 8% interest would now A. Ohm's Law
must be deposited now? D. P21,286 * B. Hooke's Law *
A. P340.30
B. P410.17 * REFRESHER M,ANUAL 2nd Edition by JAS TORDILLO
REFRESHER MANUAL 2"d Edition by JAS TORDILLO
15-10 DAY 15 - SOLUTION
DAY 15 - SOLUTION 115-11
ENGINEERING MATHEMATHICS
C. Lenz's Law C. greater than g
D. Coulomb's Law D. zero
C. radiation C. frequency *
D. expansion D. speed
27. Which of the following is a measure 34. How is pressure related to depth?
of the inertia of a body? A. directly proportional* 41 . Which of the following , is the 47. A car's - speed increases from 30 m/s
A weight B. no effect to 40 m/s in 5 sec. What is its
temperature at which the gas would no
B. volume C. inversely proportional longer exert pressure? acceleration?
C. density D. equal to one another A. Kelvin temperature A -2m/s2
D. mass* 2
B. absolute zero * B. 2 m/s *
35. Which of the following is a scalar C. Celsius scale temperature C. -14 m/s
2
28. What are the three fundamental quantity? D. 14 m/s 2
D. absolute temperature
quantities? A acceleration
A volume, weight, time B. distance* 42. Which of the following substances
B. density, mass, volume C. displacement has the greatest specific heat?
a- -v2-
- v1
-
C. length, mass, time* t
D. velocity
A. aluminum
D. speed, distance, time a = 40- 30 = 2 m 1s2
B. alcohol*
36. Which of the following would float on 5
C. glass
29. What is the speed of a projectile if it is water?
D. iron
at the top of its path? A iron 48. An object dropped from the roof of
A. unchanged B. wood" the building hits the ground 3 sec later.
43. What is the device used to convert 2
B. minimum* C. stone How high is the building? (g = 10 m/S )
mechanical energy to electrical energy?
C. doubled D. mercury A. 45 m *
A. generator *
D. "1aximum B.13 m
B. heat pump
37. What is Boyle's law in - e,quation C. 30m
30. Which of the following is/are short-
C. motor
form? D. 3.33 m
range forces? D. rotor
A. VN2 = P2P1
A. Gravitational force B. V1N2 = P1/P2 44. A force of 20 N acts on an object for 5
s = v,t + v. gt2 2
B. weak nuclear force C. V1P2 = k seconds. What is the impulse?
s = 0 + y. (10)(3)
C. electromagnetic force =
D. V1Nz P2/P1 * S-= 45 m
D. strong nuclear force *
38. What law states that there is
A 0.25 s/N
B. 15 N-s .
49. At what angle does a projectile go
31 . What happens to the mechanical
C. 4 N/s
decrease in pressure at the side of a farthest when fired?
D. 100 N-s *
advantage of a lever if its effort distance moving stream of fluid? A 30°
increases? A. Bernoulli's Law * B. 60°
I= F X t
A. increases * B. Archimedes Law
I= 20(5) = 100 N-s
c. 45° *
B. remains the same C. Pascal's Law D. 75°
C. decreases D. Hooke's Law
45. What would be the strain produced
D. becomes zero
on a wire of length 100 em and an vJsin29
39. The interactions responsible for R = ----"---
increase in length of 0.5 em?
32. What kind of energy is present holding together electrons to their atom 9
A. 0.05
whenever a body is at a distance from as well as for combining atoms and
B. 100
Using e = 30°, assuming Vo = 300 m/s
the ground? molecules to produce larger bodies are:
A. elastic potential energy A. strong nuclear
c. 0.005 * R = (300)2 sin2(30) = 7945 . 19 m
D.200 9.81
B. electromagnetic potential energy B. weak nuclear
C. electric potential energy C. electromagnetic*
D. gravitational potential energy * D. gravitational ..22
Strain = Y... = = 0.005 Using e = 45°, assuming Vo = 300 m/s
L 100
R = (3oo)2 s in2(45) =9174.31 m
33. An object's acceleration as-it starts to 40. Which of the following processes 9.81
fall is: occurs when the heat passes from place 46. What determines the pitch of sound?
A. amplitude Therefore, at 45° the projectile go farthest
A. equal to g * to another, from molecule to molecule? when fired.
B. less than g A. convection B. wavelength
B. conduction * REFRESHER MANUAL 2nd Edition by JAS TORDILLO
REFRESHER MANUAL2"d Edition by JAS TORDillO
15-12 DAY 15 -SOLUTION
DAY 15 - SOLUTION 15 -13
ENGINEERING MATHEMATHICS
I
50. A bullet is fired from a gun at an 1
angle of 40°. What is the range if its V'= 1 + I
2vn B. 0.78 57. Find the equation of a line which
velocity is 300 m/s? g = 10 m/s2 . passes through the point (2, 1) and
A. 192.9 m C. 0.68
1 9 D. 0.98 perpendicular to the line whose equation
B. 8863 m * v· 1
= 1 + 2Jl6 = 1 +-
s 8 gph is y = 4x + 3.
C 229 8 m
A. x-4y + 6 = 0
D 12000 m LR = 2b2 B. X + 4y - 6 = 0 *
53. A box is to be constructed from a
a C. y -4x + 6 = 0
piece of zinc 20 sq. in by cutting equal
R= vg sin29 "' (300)2 sln2(40) squares from each corner and turning up 2 D. y - 4x + 6 = 0

R = 8863.3 m
9 10 the zinc to form the side. What is the
volume of the largest box that can be so
LR
3
= - (length of the major axis)
m1 = slope of given line = 4
2
constructed? ~ = 3.(2a) mz = slope of perpendicular line = _l
51 . A carpenter's ladder which is 25 feet A. 599.95 cu. in a 3 4
long leans against a wall. The top of the B. 579.50 cu. in
ladder is sliding against the wall at the C. 592.59 cu. in * b2 = 3. a 2 Using point-slope form:
D. 622.49 cu. in 3
rate of 1 fps. Find the rate at which the y-y1 = m(X-X1)
bottom of the ladder is sliding when the
c2 = a2- b2
I
V(x) = (20 - 2x)2 X y - 1 = - - (X - 2)
bottom is 7 feet from the base of the wall.
A. 3.43fps * V'(x) = (20 - 2x)2 + x(2)(20- 2x)(-2)
c2 = a2 - 3. a 2 4
3
B. 5.67 fps = (20 - 2x)2 - 4x(20 - 2x) 4y -4 = -x + 2
C. 4.33 fps = (20 - 2x) (20 - 2x - 4x) c2= .!..a2 X +4y-6 = 0
D. 694 fps = (20 - 2x) {20 - 6x) 3
=
4(10-x)(10-3x) . .
eccentnctty, e = - = 0.58
c 58. What is the diameter of a circle with
the following equation:
dy a
-
dt
= -1 fps Equate V'(x) = 0 x2 + 1- 6x + 4y - 12 = 0
4(10-x)(10-3x) = 0 A. 8
252 = l + x2
10- x = o 10- 3x = 0 56. Determine the coordinates of the B. 5
Y= J2s 2-x 2 X =
10 X 10/3= point which is three-fifths of the way from
the point (2, 5) to the point (-3, 5).
c .. 6
D. 10 *
A. (-1, 1) *
when x=7, y =? For V maximum at x = 10/3
8.(-1,-2) x2 + 1- 6x + 4y - 12 = 0
y = hs2 - 1 2 = 24 ft
0 = 2ydy + 2xdx.
v = [2o-{'~Jrcn c. (-2, -1 )
D. (1, 1)
x2- 6x + 9 +I+ 4y + 4 = 12 + 9 + 4
(x- 3)2 + (y + 2)2 = 5
2

d)( dy
)(-;-y-
V = 592.59 cu. in
X = X1 + k (Y2 - Y1)
dt dt radius, r = 5
54. What is the derivative of the functioll k = P,P =L diameter, d = 1o
dx = -y dy "'-24 (-1) = 3.43 fps with respect to x of (x + 1)3 - x3 . P1P2 5
dt X dt 7 59. The equation of a line that intercepts
A. 3x + 3
52. Water from the filtering facility is
B. 6x- 3
C. 3x-3
X = 2+ 0}-3-3) the x-axis at x = 4 and the y-axis at
y = -6 is:
pouring into a swimming pool. After n A. 3x + 2y = 12
D. 6x + 3 * X = -1
hours, there are n + .J n gallons in the
Y = Y1 + k(y2 - Y1l B. 3x - 2y = 12 *
pool. At what rate is the water pouring Y =(X +1) 3 - 0 C.2x -3y = 12
into the pool when n = 16 hours? y' = 3(x + 1)2 - 3x2 D. 2x -37 = 12
A. y, gph
y' = 3x2 + 6x + 3 - 3x2
B. 1 gph
c. 9/8 gph.
y' = 6x + 3 y =-5 + ( ~}s + s) Using the Intercept Form:

D. 7/6 gph ~+r. =


55. Find the eccentricity of an ellipse y=1 a b
when the length of the latus rectum is 2/3
V=n+Jn of the length of the major axis. p (-1, 1)
A. 0.58" REFRESHER MANUAL 2nd Edition by JAS TORDILLO
REFRESHER MANUAL 2"d Edition by JAS TORDILLO
~

15 ·14 DAY 15 - SOLUTION


DAY 16 - EXAM 16·1 I
ENGINEERING MATHEMATHICS
~+_1_ = 1 7. If five coins are tossed, how many
4 -6 DAY 16- EXAM
6x • 4y = 24 possible ways are there of getting four
3x - 2y = 12 tails.
1. Justine's average in 5 subjects is 89. A. 1/3
Disregarding the lowest grade his B. 5
60. Find the value of x:
average is 90. What is Justine's lowest C. 1/16
logs (2x- 5) = log 4 6
grade? D. 5/32
A. 7.57 *
A. 85
B. 9.66
B. 87 8. On the number system, find x if the
c 8.45
c. 86 point (x, 1) is equidistant from (3, 1) and
D. 10
D. 89 {9, 1).
Jogs (2x- 5) = Jog4 6 A. 4
2. Determine the arithmetic mean of 95, B. 5
let y = Jog4 6
92 & 87. ' c. 7
4y = 6 A. 90.66 D. 6
y In 4 = In 6
B. 137
C. 91 .33 9. On the number system from negative
y = 1.292 D. 247 5 to positive 3, find x so that x will be
Jogs (2x- 5\ = 1.292 three times from negative five.
3. Find the geometric mean of 7, 9 & 12. A. X = 1
2x- 5 = 61.292 =
10.124
A. 9.1 B. X = -1
2x = 15.124
X = 7.562
B. 9.33 C. X = -3
C. 9.8 D. X = 0
D. 14
10. A rectangle with sides parallel to the
4. If five coins are tossed simultaneously, axis has vertices at (5, 1), (·3, 1) and (5,
find the probability that they will just have 4}. Find the coordinates of the fourth
two heads. vertex.
A. 5/32 A." {4, -:~)
B. 5/16 B. (-3, -4)
C. 7/32 c. (-3, 5)
D. 1/16 D. (-3, 4)

5. If five coins are tossed simultaneously, 11. A triangle is defined by the points
find the probability of getting two tails. A(5, 2.) & 8(-2, 2). Find the third point C
A.Yl in order to have a right triangle.
B. Y. A. (-2, 6)
c. 1/3 B. (2, 6)
D. 1/5 c. (6, -2)
D. (-6, 2)
6. If five coins are tossed simultaneously,
find the probability of getting at least four 12. A triangle is defined by the points (2,
tails. 6), (·2, 2) & (5, 2). Find the area of the
A. Y. triangle.
B. 3/16 A. 28 sq. units
C. 4/5 B. 12 sq. units
D. 6/31 C . 14 sq. units
D. 10 sq. units ·

REFRESHER MANUAL 2nd Edition by JAS TORDILLO

REFRESHER MANUAL 2"d Edition by JAS TORDILLO


16-2 DAY 16 - EXAM DAY 16- EXAM 116-3
ENGINEERING MATHEMATHICS
13. A triangle Is defined by the points (0, A. sign is negative
0). (3. 1) and (-3, 1). Determine what B. cosine is positive
26. What do you call the largest integer B. 9 sides
type of triangle? C. tangent is positive
which is a factor of each of the given C. 8 sides
A. vertical D. cotangent is negative
D. 10 sides
numbers?
B. equilateral
A. Least common factor
C. angular 20. What kind of fraction with a
B. Greatest common multiple 32. What type of polygon if the number
D. isosceles numerator ·that is greater than the
C. Least common multiple of diagonals is 54?
denominator?
D. Greatest common factor A . dodecagon
14. Point (5, 4) is the center of a circle A. improper
B. octagon
that is tangent to the y-axis. What is the B. proper
27. Justine can paint a cabinet alone in 5 C . pentedecagon
point of tangency? C. mixed D. nonagon
working days and Bonbon can do it in 10
A. (4, 0) D. decimal
days alone. How long will it take Justine
B. (0, 4) 33. Each interior angle of a regular
and Bonbon to do the job working
c. (5, oi 21. The center of the circle inscribed
together? heptagon is nearest to:
D. (0, 5) inside the triangle and it is the
A. 3 2/3 days A. 130 deg
intersection of t he three angle bisectors
B. 2 1/3 days B. 145.9 deg
15. If two point in a number line have of the triangle.
C. 3 1/3 days C. 138 deg
coordinates -2 and 10. Find the A. incircle
D. 2 2/3 d'ays D. 128.6 deg
coordinate of a point that is three times B. increment
as far from 10 as from -2. C. median
28. A stone is 'placed in a cylinder vessel 34. Justine was told to get 20 cans of
A. 7 D. incenter
of water whose radius is 10 em, the water milk in a store. Justine can carry only
B. 1
rises 4 em. What is the volume of the three cans at a time. How many trips
c. 4 22. What do you call a radical expressing
stone if it is completely submerged? would Justine have to make?
D. O an irrational number?
Qensity of stone 2,350 kg/cubic m. A. 6 trips
A. surd
16. If two points in a number line have B. radix A. 3141 cm 3 B. 7.6 trips
B. 1526 cm 3 C. 7 trips
coordinates 1 and 10, find the coordinate C. complex number
C. 1256 cm3 D. 6.7 trips
of a point that is twice as far from 1 as D. index
D. 1625 cm 3
from 10. 35.. W hich of the following is the v alue of
A. 7 23. XYZ factory employs 100 workers. If
3 out of 4 workers are men, how many
29. A square with side S has a perimeter x2 + 1/x21 when x + 1/x = 10/3?
B. 4 A. 9
equal to the circumference of a circle
c. 3 men workers are employed in the
w ith radius R. Which of the followi ng is B. 3
D. 3 factory?
A. 60 true? c. 70/9
A. 2S = TTR D. 82/9
17. Find the equation of the line that is B. 75
B. S = 2TTR
tangent to the curve at point (3, -5) and c. 65 C. 4S = TTR 36. A rectangular 80 m perimeter lot is to
parallel to y-axis. D. 90
D. s = TTR contain 300 m2 floor area. What should
A. X = 3 be its dimension if the length is three
B. y = ·7 24. Justine's factory has 60 workers. If 4
30. How long is each side of a regular times its width?
C. X = -7 out of 5 workers are married, how many
A. 10m x 30m ·
hexagon inscribed in a circle of radius 15
D. y = 3 workers are not married?
B. 5m x 15 m
A. 12 workers em?
A. 10 em C. 15m x 45 m
18. A line that has an equation of y = 0 is· B. 48 workers
B. 15 em D. 10m x25 m
what?. C. 24 workers
A. y-axis D. 60 workers C. 12 em
D. 20 em 37. What do you call a surd that contains
B. point of origin
at least one rational number, example
C. x-axis 25. An equilateral triangle has an altitude
31. What is the number of the sides of 3 .fi.?
D. between x-axis and y-axis of 10v'3 mm. long. Find its area in mm2•
the polygon if the sum of the angles is
A. 20~3 A. pure surd
I
19. Which Is true regarding the signs of B. 100~3
the natural functions for angles between C. 50...J3
.
1440 degrees?
A. 6 sides
B. s urd

90 degrees and 180 degrees? D. 200~3 REFRESHER M ANUAL 2nd Edition by JAS TORDILLO
REFRESHER MANUAL 2"d Edition by JAS TORDILLO
~ --"
- -- ~--
--

16-4 DAY16-EXAM DAY 16- EXAM 116-5


C. binomial surd C. quadrantal ENGINEERING MATHEMATHICS
D. mixed surd D. co-terminal
B. 15 A. capital
38. The ratio of two numbers is 5 : 6. If 44. Two angles are C. 18 B. funds
if they
D.24 C. assets
the smaller number is 70, what is the have the same terminal side when placed
larger number? in standard position. D. liabilities
A 58 51. Solve angle A of an oblique triangle
A. coincide
B 80 with vertices ABC, if a= 25, b = 16 and 57. A form of business organization in
B. coterminal
c 68 C = 94 degrees and 6 minutes. which a person conducts his business
C. right
D. 84 A 50 deg and 40 min alone and entirely for his own profit,
D. supplementary
B. 55 deg and 32 min being solely responsible for all its
39. A sequence 1, 3, 6, 10, 15, 21, .... is C. 45 deg and 35 min activities and liabilities:
45. Find the equation of the line whose
known what? D. 54 deg and 30 min A . sole proprietorship
slope is -3 and the x-intercept is 5.
A. triangular numbers A. 3x - y = 5 B. partnership
B. square numbers B. 3x+y = 15 52. A man finds the angle of elevation of C. enterpreneurship
C. harmonic C. y = 3x + 15 the top of a tower to be 30 degrees. He D. corporation
D geometric walks 85 m nearer the tower and finds its
D. y = -3x + 5
angle of elevation to be 60 degrees. 58. The exclusive right of a company to
40. A sequence 1, 8, 27, 64, ...... is W hat is the height of the tower? provide a specific product/services in a
46. Find the equation of the line whose
known as what? A. 76.31 m given region of the country.
y-intercept is -2 and slope 1.
A. oblong numbers A. y = x- 2 B. 78.16 m A. Permit
B. cubic numbers C. 79.31 m B. Franchise
B. y =-X+ 2
C. Gnomons numbers C. y = -2x + 1 D. 73.61 m C. Agreement
D. pentagonal numbers D. y = 2x -1 D. Sales Memo
53. 38.5 to the x power= 68.5 to the x -2
41 . A hot air balloon rising straight up 47. Find C so that the line y = 4x + 3 is power, solve for x using logarithms. 59. A system used to record historical
from a level field is track by a range tangent to the curve y = x?- + C.
A 2 .70 financial . transactions, the resultant
finder 500 ft from the point of lift off. At B. 2.10 records of which are used for product
A. 6
the moment the lift finder's elevation is C. -2.10 costing, satisfaction of statutory
B.5
45 deg, the angle is increasing at the rate D. -2.02 requirements, reporting of profit for
C. 7
of 0.14 rad/min. How fast is the balloon D.4 income • tax purposes, and general
rising? 54. At one side of a road is a pole 17 ft company management.
A. 120ft/min 48. Find the equation of the line if the line high fixed on top of a wall, 15 ft high. On A. Sales system
B 130ft/min the other side of the road, at a point on B. Inv estment layout
y - 4x = C is tangent to the curve y = x?- +
C. ·110ft/min 7. the ground directly opposite, the flagstaff C. Bookkeeping system
D. 140ft/min A. 2x = 2y - 3 and the wall subtend equal angles. Find D. Business record
B. 2x = y-2 the width of the road.
42. Going against the wind, a domestic C: X= 4y + 3 A. 30ft 60. The worth of property which is equal
plane can travel 5/8 of the distance in D. 4x = y -3 B. 40ft to the original cost less depreciation:
one hour than it is going with the wind. If C. 60ft A . Book value
the plane can fly 300 miles per hour in 49. The product of two numbers is D. 25ft B. First cost
calm wind, what is the velocity of the increased by 4 is D. If one of the number C. Net sales
wind? is P, find the other number. 55. Which of the following types of flow D. Gross Margin
A . 69.3 mph A. (D + 4)/P meters is most accurate?
B. 89.3'mph B. D/P -4 A. venturi tube
C. 73.3 mph C. (D -4)/P B. pitot tube
D. 93.3 mph D. P/D -4 C. flow nozzle
D. foam type
43. What do you call an angle whose 50. Paul is 10 years old and his sister is
terminal side coincides with an axis? one year old. What will be the age of 56. Cash money and credit necessary to
A . reflex Paul be twice as old as his sister? establish and operate an enterprise:
B. right · A.B REFRESHER MANUAL 2nd Edition by JAS TORDILLO
REFRESHER MANUAL 2"d Edition by JAS TORDILLO
16-6 DAY 16 -SOLUTION DAY 16 - SOLUTION 116-7
ENGINEERING MATHEMATHICS
DAY 16 - SOLUTION
~ ) No. of ways each coin w ill fall = 2 (H or T) 8. On the number system, find x if the
DAY 16- ANSWER KEY 1. Justine's average in 5 subjects is 89. No. of ways for 5 tosses can fall = 2 5 = 32 point (x, 1) is equidistant from (3, 1) and
l.A 16.A 31. D 46. A Disregarding the lowest grade his C(5 2) = S! = ~ = 10 (9, 1).
2. c 17.A 32. A 47. C average is 90. What is Justine's lowest ' (5-2) !2! 3!2! A. 4
3.A 18. c 33.0 48.0 grade? B. 5
No. of ways of getting 2 heads (H):
4. B 19. D 34. c 49. c A. 85* c. 7
5. B 20.A 35.0 so.c P=.!.Q=5 D. 6 *
6. B 21. D
7. B 22.A
36. A
37. D
51. D
52.0
B. 87
C. 86
32 16
8.0 23. B 38.0 53. c D. 89 9-3
5. If five coins are tossed simultaneously, X=-+3=6
9.A 24. A 39. A 54. C 2
10.0 25. B 40. B 55. B S = sum of 5 subjects find the probability of getting two tails.
A.Y,
11. A 26.0 41.0 56. A L = lowest grade 9. On the number system from negative
12. c 27. c 42. A 57. A Average of 5 subjects: B. %*
13.0 28. c 43. C 58. B C. 1/3 5 to positive 3, find x so that x will be
14. B 29. A 44. B 59. C = 89= ~ D. 1/5 three times from negative five.
~I 15.8 30. B 45.8 60.A 5 A. X = 1*
s = 89(5) = 445 B. X = -1

Average of 4 subjects:
Prob = (21)(21)_- _!_4 C. X
D. X = 0
= -3

S - L = 90
4 6. If five coins are tossed simultaneously, 3a +a = 3- (-5) = 8
find the probability of getting at least four 4a = 8
445-L = a=2
90 tails.
4 A. y. x=3 - 2=1
L = 85 B. 3/16 *
C. 4/5 10. A rectangle with sides parallel to the
2. Determine t_he arithmetic mean of 95, D. 6/31 axis has vertices at (5, 1), (-3, 1) and (5,
92 & 87. 4). Find the coordinates of the fourth
A. 90.66 No. of ways each coin will fall = 2 (H or T) vertex.
B. 137 No. of ways for 5 tosses can fall = 2 5 = 32 A. (4, -3)
c. 91.33 * B. (-3, -4)
D. 247 Cr = C(5, 4) + C(5, 5) c. (-3, 5)
. . 95+92+87 5! D. (-3, 4) *
Anthmet1c mean = = 91.33 - - +1 =6
3 (5-4) !4!
11. A triangle is defined by the points
3. Find the geometric mean of 7, 9 & 12. P= j__=2_ A(5, 2) & B(-2, 2). Find the th ird point C
A. 9.1" 32 16 in order to have a right triangle.
B. 9.33 A. (-2, 6) *
C. 9.8 7. If five coins are tossed , how many B. (2, 6)
D. 14 possible ways are there of getting four C. (6, -2)
tails. D. (-6, 2)
A. 1/3
Geometric mean= ?}7(9) (12) = 9.109 B. 5 * 12. A triangle is defined by the points (2,
c. 1/16 6), (-2, 2) & (5, 2). Find the area of the
4. If five coins are tossed simultaneously, D. 5/32 triangle.
find the probability that they will just have A. 28 sq. units
two heads.
c (5 ' 4 ) = S!
(5 - 4) !4!
=5 8. 12 sq. units
A. 5/32 C. 14 sq. units*
B. 5/16 * D. 10 sq. units
C. 7/32
D. 1/16 REFRESHER MANUAL 2nd Edition by JAS TORDILLO
REFRESHER MANUAL 2nd Edition by JAS TORDILLo"
16- s DAY 16 - SOLUTION DAY 16 - SOLUTION 116-9
x=a-2 = 3-2=1 ENGINEERING MATHEMATHICS
D1 = J(2-5)2 +(6-2) 2 =5
16. If two points in a number line have C . mixed A · 20"3
D2 = J(-2-5) 2 +(2-2)2 =7 coordinates 1 and 10, find the coord inate D. decimal B. 100--13 *
of a point that is twice as far from 1 as c. 50--13
D3 = J(2- -2)2 + (6- 2)2 = 5.65 21 . The center of the circle inscribed D. 200--13
from 10.
A. 7* inside the triangle and it is the
S= 5+7+5.65
- --- - 8.825 B. 4 intersection of the three angle bisectors
2
c. 3 of the triangle. Tan 60o = J0./3
X
D. 3 A. incircle
A= Js.825(8.825- sX8.825- 1Xs.s2s- s.6s) B. increment 10
X:::

C. median Area ::; Y2 bh = 1/2 (2x)(h)


A::: j8.825(3.825Xt.825X3. 175) a + 2a ::: 10 - 1 = 9 Area = Y2 (2)(10)(10)V3
3a = 9 D. incenter *
A= 13.98 sq. units Area ::: 100 V3
a=3
22. What do you call a radical expressing
13. A triangle is defined by the points (0, an irrational number? 26. What do you call the largest integer
0), (3, 1) and (-3, 1). Determine what
x = 2a + 1
X=2(3) + 1 A. surd * which is a factor of each of the given
type of triangle? · B. radix numbers?
x=7
A. vertical C. complex number A. Least common factor
B. equilateral D. index B. Greatest common multiple
17. Find the equation of the line that is
C. angular C. Least common multiple
tangent to the curve at point (3, -5) and
D. isosceles * 23. XYZ factory employs 100 workers. If D. Greatest common factor"
parallel to y-axis.
A. X= 3* 3 out of 4 workers are men, how many
x= ~=M B. y = -7 men workers are employed in the
factory?
27. Justine can paint a cabinet alone in 5
working days and Bonbon can do it in 10
C. X = -7
y ::: J_32 +12 = JIO D. y = 3 A. 60 days alone. How long will it take Justine
and Bonbon to do the job working
B. 75 *
z = J(-3-3)2+(1- 1)2 =6 18. A line that has an equation of y = 0 is C . 65 together?
D. 90 A . 3 2/3 days
what?
B. ·2 1/3 days
since x = y, therefore the triangle is an A y-axis
B. point of origin 3:4 = x: 100 C. •3 1/3 ,days *
isosceles. D. 2 2/3 days
C. x-axis * 3 X
-=-
14. Point (5, 4) is the center of a circle D. between x-axis andy-axis 4 100
D = No. of days if Bonbon and Justin
that is tangent to the y-axis. What is the X= 75
19. Which is true regarding the signs of working together
point of tangency?
A. (4,'0) the natural functions for angles between 24. Justine's factory has 60 workers. If 4
B. (0, 4) * 90 degrees and 180 degrees? out of 5 workers are married, how many .!. = ~+2.
D 5 10
C. (5, 0) A sign is negative workers are not married?
D. (0, 5) B. cosine is positive A~ 12 wo rkers * 1 2+1 3
C. tangent is positive · o= 10 =10
B. 48 workers
15. If two point in a number line have D. cotangent is negative * C. 24 workers
coordinates -2 and 10. Find the D. 60 workers D ::: !£ ::: 3 1/3 days
3
coordinate of a point that is three times Sine-++
as far from 10 as from -2. Tan e-+- 4 X 28. A stone is placed in a cylinder vessel
A. 7 Cos 9-+- -= - of water w hose radius is 10 em, the water
5 60
8.1 * Cote+- rises 4 em. What is the volume of the
x = 48 workers married
C. 4 stone if it · is completely submerged?
y = 60 - 48 = 12 workers not married
D. 0 20. What kind of fraction with a Density of stone 2,350 kg/cubic m.
numerator that is greater than the A. 3141 cm3
25. An equilateral triangle has an altitude
a+3a=10-0~=12 denominator? 8. 1526 cm3
A. improper * of 10v'3 mm long. Find its area in mm2
4a=12
a=3 B. proper REFRESHER MANUAL 2nd Edition by JAS TORDILLO
REFRESHER MANUAL 2nd Edition by JAS TORDILLO
""W'

16-10 DAY 16 -SOLUTION DAY 16 - SOLUTION 116-11


C. 1256 em• * C. pentedecagon ENGINEERING MATHEMATHICS
D. 1625 cm 3 D. nonagon
B. 80
x2 + 1 10
C. 68
Volume of stone= ~(20)2(4) D = ~(n-3) X 3 D. 84 *
4 2 3~ -10x + 3 = 0
= 1256.63 cm 3
54 = ~(n-3) 5
-=-
70
2
29. A square with side S has a perimeter X = - (-10)± J(-10)2 -4(3X3) 6 X
108 = n2.- 3n X: 84
equal to the circumference of a circle 2 2(3)
n - 3n - 108 = 0
with radius R. Which of the following is (n- 12)(n + 9) = 0 x = 3 and x = 1/3
true? 39. A sequence 1, 3, 6, 10, 15, 21, .... is
n = 12 sides (dodecagon) known what?
A. 25 = TTR *
B. S = 2TTR
~ + __!__ =32 + _!.. _ 81 + I _ 82 A. triangular numbers *
33. Each interior .angle of a regular · x2 32 --9--9
C. 4S = TTR B. square numbers
heptagon is nearest to: C . harmonic
D. S = TTR A. 130 deg 36. A rectangu lar 80 m perimeter lot is to D. geometric
B. 145.9 deg contain 300 m2 floor area. What should
Perimeter= Circumference C. 138 deg
4S = 2nR be its dimension if the length is three 40. A sequence 1, 8, 27, 64, .. , ... is
D. 128.6 deg * times its width? known as what?
2S = nR
A. 10mx30m* A. oblong numbers
n = 7 sides for heptagon B. 5 m x 15m B. cubic numbers *
30. How long is each side of a regular
hexagon inscribed in a circle of radius 15 8 = (n-2}180 C. 15mx45m C. Gnomons numbers
em? n D. 10m x 25m D. pentagonal numbers
A. 10 em 8= (7-2)180 = 128.570
B. 15 em* 7 L=3W 41. A hot air balloon rising straight up
C. 12 em P = 2L + 2W from a level field is track by a range
D. 20 em 34. Justine was told to get 20 cans of 80 = 2(3W) + 2W finder 500 ft from the point of lift off. At
milk in a store. Justine can carry only 80 = 8W the moment the lift finder's elevation is
three cans at a time. How many trips W= 10m 45 .deg, the angle is increasing at the rate
8 = 360 =600
6 would Justine have to make? L=3(10)=30m of 0.14 rad/min. How fast is the balloon
A. 6 trips rising?
sin 30o = ~ B. 7.6 trips 37. What do you call a surd that contains
at least one rational number, example
A. 120 ft/min
B. 130 ft/min
15 c. 7 trips* C. 110ft/min
x = 15 em D. 6.7 trips 3fi.?
A. pure surd D. 140 ft/min "
31. What is the number of the sides of B. surd
. 20 cans
the polygon if the sum of the angles is No. of tnps = --- C. binomial surd Tan 8 = 'f..
1440 degrees? 3~.~ ~. D. mixed surd * X
A. 6 sides trip
Sec2 8 d9 = dyfdt
B. 9 sides = 6.67 trips say 7 trips
C. 8 sides
J3 + fi ~ pure surd dt X
D. 10 sides* 35. Which of the following is the value of 3+ fi ~ binomial surd Sec2 45°(0.1 4)= dyfdt
X:+ 1/~ when x + 1/x 10/3? = fi ~surd
X
S = (n- 2)180 A. 9 dy/dt = 140ft/min
1440 = (n- 2)180 B. 3 3 fi ~surd
n = 10 sides C. 70/9 42. Going against the wind, a. domestic
D. 82/9 * 38. The ratio of two numbers is 5 : 6. If plane can travel 5/8 of the distance in
32. What type of polygon if the number the smaller number is 70, what is the one hour than it is going with the wind. If
of diagonals is 54? larger number? the plane can fly. 300 miles per hour in
X+ _!_ _ 10
A. dodecagon * x-3 A. 58
B. octagon
REFRESHER MANUAL 2nd Edition by JAS TORDILLO
REFRESHER MANUAL2"d Edition by JAS TORDILLO
...

16-12 DAY 16 - SOLUTION DAY 16 - SOLUTION 116-13


calm wind, what is the velocity of the Y- Y1 = m(x- x1) ENGINEERING MATHEMATHICS
wind? y- 0 =- 3(x- 5)
A. 69.3 mph • y = -3x + 15 =
y 2' + 7 = 4 + 7 =11 Using cosine law:
3x + y = 15 2 2 2
B. 89.3 mph c = a + b - 2ab cos C
C. 73.3 mph 11 -4(2) = c 2 2
c = 25 + 162 -2(25)(16) cos 94°6'
D. 93.3 mph 46. Find the equation of the line whose C=3 c = 30.63 units
y-intercept is -2 and slope 1.
Distance = velocity x time A. y = x- 2 * y-4x = 3 Using sine Law:
Time = _d_is_ta_n_c_e
=
B. y -x + 2 4x = y- 3 a c
time
C. y = -2x + 1 --= --
sinA sinC
D. y = 2x- 1 49. The product of two numbers is
Let x = velocity of wind 25 30,63
increased by 4 is D. If one of the number
300 mph = plane's normal speed
m = 1 and b = -2 (y-intercept) is P, find the other number. sinA = sin94°6'
300 - x= plane's velocity against the wind
A. (0 + 4)/P
300 + x = plane's velocity with the wind Sin A= 0.8141
Y = mx + b B. 0/P -4
y = distance travelled with the wind A= 54.5° = 54° 30'
y=x-2 C. (0- 4)/P *
5/8y = distance travelled against the wind
D. P/D-4
47. Find C so that the line y = 4x + 3 is 52. A man finds the angle of elevation of
Time( going against the wind) the top of a tower to be 30 degrees. He
tangent to the curve y = x? + C. Let x = the other number
= T ime (going with the wind) walks 85 m nearer the tower and finds its
A.6 P = one of the number
5 angle of elevation to be 60 degrees.
8y y B.S
xP + 4 =D ·What is the height of the tower?
--=--
300- X 300+ X
c. 7* A. 76.31 m
D.4 D- 4
187.5 + 0.625x = 300- x x= - - B. 78.16m
1.625x = 112.5 P C. 79.31 m
y = 4x + 3
x = 69.23 miles per hour
y' =m =4(slope) D. 73.61 m*
50. Paul is 10 years old and his sister is
43. What do you call an angle whose one year old. What will be the age of h=c sin30°
y ~; + c Paul be twice as old as his sister?
terminal side coincides with an axis?
y' = 2x"' m A.8
A. reflex Solving for c:
2x = 4 B. 15
B. right c 85
x=2 C.18 *
C. quadrantal •
y :;: 4(2) + 3 = 11 D.24 sin I 20° = sin30°
D. co-terminal
2
y=x +C
44. Two angles a re if they 10 + x =age of Paul x years hence as to' c = 147.22 m
have the same terminal side when placed
2
11 = 2 + c double the age of his sister
C=11-4=7
in standard position. 1 + x = age of his sister x years hence as h = (147.22) sin 30°
A. coincide to half the age of Paul h = 73.61 m
48. Find the equation of the line if the line
B. coterminal * 2
y - 4x = C is tangent to the curve y = x +
C. right 10+x=2(1+x) 53. 38.5 to the x power = 68.5 to the x -2
7. X =8 power, solve for x using logarithms.
D. supplementary
A. 2x = 2y - 3 A 2 .70
Paul's age= 10 + x = 10 + 8
45. Find the equation of the line whose
B . 2x y- 2= =
18 years old B. 2.10
C. X= 4y + 3 c. -2.10.
slope is -3 and the x-intercept is 5.
A . 3x -y = 5
=
D. 4x y- 3 *
51 . Solve angle A of an oblique triangle D. -2.02
B . 3x + y = 15 * with vertices ABC, if a= 25, b = 16 and
C. y = 3x + 15
y -4x = C
y = 4x + C
C = 94 degrees and 6 minutes. 3a.s• = 6.s•·2
D. y = -3x + 5 A. 50 deg and 40 min log 3s.s• = log 6.s•·2
m = 4 (slope)
B. 55 deg and 32 min X log 38.5 = (X • 2) log 6.5
y =x? +7 C. 45 deg and 35 min
m =- 3 log38.5 X : X•
y' = m = 2x 2
D. 54 deg and 30 min * log6.5
2x = 4
x-intercept 5 is P(S,O)
x=2
REFRESHER MANUAL 2nc1 Edition by JAS TPRDILLO
REFRESHER MANUAL 2"d Edition by JAS TORDlLLO
16 - 14
DAY 17- EXAM 117 -1
DAY 16 - SOLUTION ENGINEERING MATHEMATICS
1.95x = x -2 57. A form of business organization in
0.95x = -2 which a person conducts his business 6. What is the present worth of two P1000
X : ·2.10 alone and entirely for his own profit.
DAY 17- EXAM payments at the end of third year and fifth
being solely responsible for all its year? The annual interest rate is 8%.
54. At one side of a road is a pole 17ft activities and liabilities: 1. The amount of P10,000 is borrowed at 30% A. P1744
high fixed on top of a wall, 15ft high. On A. sole proprietorship • per annum simple interest. How much will be B. P1384
the other side of the road, at a point on .B. partnership due at the end of 130 days? C. P1474 '
the ground directly opposite, the flagstaff C. enterpreneurship A. P1 1,830 D. P1 224
and the wall subtend equal angles. Find D. corporation B. P1 1,083
the width of the road. C. P11 ,028 7. A sinking fund consists of 15 annual
A. 30ft 58. The exclusive right of a company to D. P11,038 deposits of P1 ,000 each, with interest earned
B. 40 fl. . provide a specific productlservices in a at the rate of 4% compounded annually. What
c. 60 ft . given region of the country. 2. How much interests are you going to pay is the principal in the fund at its terminal date?
D. 25ft A. Permit after 45 days if you borrowed P50,000 in a A. P25, 115.15
B. Franchise • lending investor with an interest rate of 5% per B. P29,546.37
15 C. Agreement month. C. P28,105.25
IanO e - Eq. 1 D. Sales Memo A. P3770 D. P20,023.59
X
B. P3570
32 C. P3075
tan 20 = - Eq. 2 59. A system used to record historical 8. You borrowed the amount of P10,000 in a
)(
financial transactions, the resultant D. P3750 bank with an interest rate of 30%
r~cords of which are used for product compounded monthly. How much would you
2tanB 32 costing, satisfaction of statutory 3. How much should you put into a 10% have to pay after 2 years?
savings account in order to have P100,000 in A. P18,087
J-tan 2 B =-; requirements, reporting of profit for
five years? B. P17,987
income tax purposes, and general
A. P62,092.20
_ 2('s)
_x_ = -32 company management. C. 17,032
B. P112,598.60 D. P17,825
t-{';f X
A. Sales system
B. Investment layout C. P62,902.20
30 C. Bookkeeping system • D. P85,836.98 9. An initial savings deposit of P200,000 in a
X 32 D. Business record certain bank earns 60% interest per annum
xl - 22S =-;- 4. A company invests P200,000 today to be compouP!ded monthly. If the earnings from
s:! repaid in five years in one lump sum at 18% the deposit are subject to 10% tax, what would
60. The worth of property which is equal
2 compounded annually. How much profit in be the net value of the deposit after one year?
to the original cost less depreciation:
~=32 A. Book value •
present day pesos is realized over 5 years? A. P359, 171
2
x - 225 A. P257,551
B. First cost B. P159,171
30xl = 32xl - 720.0 C. Net sales B. P247,896 .C. P343,254
2x2 :. 7200 C. P227,681 D. P325,896
D. Gross Margin
X= 60 ft D. P457,551
10. A bank is advertising 10% accounts that
55. Which of the following types of flow 5. A firm borrows P5000 for 5 years at 16%. yields 10.47% annually. How often is the
meters is most accurate? At the end of 5 years, it renews the loan for an interest compounded?
A. venturi tube amount due pius P2000 more for 2 years at A. quarterly
B. pltot tube " 16%. What is the lump sum due? B: daily
C. flow nozzle A. P17,822 C. semi-annually
D. foam type B. P16,282 D. monthly
C. P17,383
56. Cash money and credit necessary to D. P16,822 11. A man borrowed the amount of P20,000
establish and operate an enterprise: with an interest of 30%. How much interest is
A. capital" he going to pay at the end of 22 months?
B. funds
C. assets
D. liabilities
REFRESHER MANUAL 2N° Edition by JAS TORDILLO

REFRESHER MANUAL 2 nd Edition by JAS TOROILLO '·


17-2/ DAY 17- EXAM DAY 17- EXAM 117-3
A. P32, P12,220 ENGINEERING MATHEMATICS
B. P31,533 C. P12,062
C. P12,353 Salaries and Wages 27. Assets that lack physical substance but
D. P12,602
D. P32,296 ..... ... .... ..... .... P30,000 per month consists of valuable rights, privileges, or
Others .... ....... ....... P15,000 per mo11th advantages; examples are patents,
17. You obtain a loan of P1 .5M at the rate
12. If P700 is invested at the end of each 18% compounded annually in oraer to build Unit Price ...... .. ..... P90 per block franchises, organizational expense and
year for 10 ~ears, at an annual interest rate of house. The payment is made at the first d~, A. 8712 goodwill.
8%, what is the tota l peso amount available of the month. How much you pay monthly t B. 7812 A. current assets
upon the deposit of the tenth payment? C. 1378 B. intangible assets
amortize the loan within a period of ten years·
A. P4,700 D. 6817 C. fixed assets
A. P25,752
8. P1 0,410 B. P25,673 D. gross assets
C. P10,014 22. The bank offers 0.75% effective monthly
C. P25,400
D. P10,140. D. P25,501 interest. What is the effective annual·rate? 28. Exclusive privilege to use a certain
A. 3.89% process in manufacturing:
13. How much money must you invest today B. 8.93% A. patent
18. A machine cost P120,000 with a
in order to withdraw P5000 per year for 10 C. 9.01% B. franchise
estimated life of ten yea rs . It salvage val~
years if the interest rate is 14%? D. 9.38% C. goodwill
after ten years is estimated to be P25,0C\
A. P26,008 D. expense
Determine the depreciation rate if the interes
B. P26,081 rate is 10%. 23. Assets that cannot be converted into cash
C. P26,808 A. 6.98% within the accounting period, such as land, 29. Permit granted to a business to handle a
D. P26,800 B. 4.82% buildings, machinery, and equipment that are given product or to operate within a given
C. 5.62% to be used in business operations: territory or along a certain route:
14. A series of year end payments extending D. 4.97%
A. fixed assets A. patent
• over ten years are as follows: PSOOO for 'the B. used assets B. goodwill
first year, P10,000 for the second year, and 19. An asset is purchased for P15D,OOO. ~ C. current assets C. franchise
P20,000 for each year from the third to the estimated life is 10 years, after wh ich it will r.: D. intangible assets D. expense
tenth. Find the equivalent annual worth of sold for P20,000. Find the depreciation for ill:
these payments if the annual interest is 8%. 24. Payment of goods or services before they 30. Factor in business which causes a
second year using the sum-of-the-years dig
A. P16,634 are-received: company to earn a higher than normal rate of
method.
B. P16,652 A. P21,273 A. current payment return due to managereal skill, popular
C. P16,365 8, P21,723 B. fixed payment acceptance of the product or some other
D. P16,562 C. P20;732 C. prepaid expense favorable circumstances:
D. late payment A. patent
D. P21,836
15. What uniform annual amount should be B. goodwill
deposited each year in order to accumulate 20. The hire of any person to do whatev< 25. Payment of services or goods before they C. cleanliness
P1 M at the end of the 1Oth annual deposit if work he is skilled in doing: ' are rendered: D. clearness
money earns 10% interest? A. advance payment
A. Promotion cost
A. P62,745 B. Salary cost B. prepaid income 31. Certain costs such as initial incorporation
B. P64,634 C. current payment fee and the cost of legal services in
C. Material cost
C. P67,045 D. Labor cost D. expense connection with the formation of the
D. P68,078 corporation.
26. Cash and goods that can be readily A. material expense
21. Cqmpute the number of blocks that an ia:
16. A company needs P60,000 in five years converted into cash: B. labor cost
plant must be able to sell per year to brea)
to buy a new equipment. In order to A. fixed assets C. equity cost
even, based on the following data:
accumulate this sum, a sinking fund consisting Cost of electricity per block B. intangible assets D. organizational expense
of three annual payments is established now. C. equity
.. .. ..... .............. P20.00
For tax purposes, no further payments will be D. current assets 32. Funds in cash or security form held for
Tax to be paid per block
made after three years . What payments are income producing purpose:
.............. .... ..... P2.50
necessary if money is worth 18% per annum? Real Estate tax A. investments
A. P12,620 .... ..... ............... P48,000 per year

REFRESHER MANUAL 2ND Edition by JAS TORDILLO


REFRESHER MANUAL 2N° Edition by JAS TORDILLO
17-4j DAY 17- EXAM DAY 17- EXAM 117-5
B. expense B. acid test ratio
ENGINEERING MATHEMATICS
C. assets C. current asset turnover
A. dividends paid A. 21 .6 in
D. peso D. current ratio B. invested capital B. 8.4 in
C. license to start business C. 11.6 in
33. Accumulated net earnings of a 40. A more vigorous measurement of the D. revenues D. 18.4 in
corporation, also called reinvested earnings: company's ability to service short term debt:
A. retained earnings A. current ratio 47. What is the acid test ratio? 53. Steam with an enthalpy of 900 cal/g enters
B. fixed deposit B. rate of return A. the ratio of owners equity to total current nozzle at a velocity of 90 m/sec. Find the
C. net 1ncome C. gross margin ratio liabilities velocity of the steam at the exit of the nozzle if
D. gross income D. acid test ratio B. the ratio of all assets to total liabilities its enthalpy is reduced to 700 caVg, assuming
C. · the ratio of current assets (exclusive the nozzle is horizontal and disregarding heat
34. Net losses in excess of retained earnings 41. Accounting book in which are recorded 2
inventory) to total current liabilities losses. Take g = 9.81 m/s and J
which are deductions in the owners equity: the credits and debits of commercial
D. the ratio of profit after taxes to equity constant = 427 kg-m/kcal.
A. deficit transactions:
A. .3297.55 m/s
B. debts A. ledger 48. Decrease in value of a physical property B. 2297.55 m/s
C. gain B. journal due to passage of time: C. 1297.55 m/s
D. gross margin C. balance A. inflation D. 997.55 m/s
D. clerks book B. recession
35. Cost of goods available for sale during C. depletion 54. 200 g of water are mixed with 230 g of
the fiscal period minus the inventory of 42. Additional cost to produce one more unit
D. depreciation alcohol (d = 790 kg/cu.m). What is the
finished goods available at the end of fiscal of product: specific volume of its resulting mixtures,
period: A. marginal cost 49. The cost of things that are neither labor assuming that the·fluids mix completely?
A. cost of goods sold B. sunk cost nor materials: A.1. 14cm3/g
B. liabilities C. expense
A. money B. 1.41 cm 3/g
C. material cost D. operating cost B. damage C. 2.1 4 cm3/g
D. organizational cost C. expenses D. 3.14 cm3/g
43. Lowering in prices of commodities:
D. asset
36. Difference between the net revenues A. inflation
55. Water flows with a mean velocity of 3.2
realized from the sale of goods and their cost: B. deflation 50. Type of ownership in business where m/s inside a circular pipe of 4 em radius. The
A. net capital C. bonus individuals exercise and enjoy the right in their bulk mean temperature of water is 60°C at this
D. discount
B. gross margin
C. expense
own interest: = 3
temperature p 985 kg/m and viscosity =
A. Equitable 4.71 x 10_. kg/m-s. What is the Reynolds
D. accounts 44. What is an annuity? B. Private Number?
A. the future worth of a present amount
C. Public A. Re = 2.35 x 105
37. Net Income divided by total capital: B. an annual repayment of a loan
D. Pure B. Re = 3.35 x 105
A. ROI C. a series of uniform amounts over a period
C. Re = 4.35 x 105
B. ROR of time 51. A steel block weighs 10 N and has volume D. Re = 5.35 x 105
C. interest D. a lump sum at the end of the year
of 300 cm3 . What is the density of the block in
D. asset rate kg/m3? 56. An oil at 50°C (p = 890 kg/m 3 ) flow into a 3
45. What must two investments with same 3
A. 2398 kg/m em diameter horizontal pipe with a velocity of
38. Rate of return on total assets divided by present worth and unequal lives have?
B. 3398 kg/m3 1.75 m/s and exits through a nozzle of 0.75
rate of return on sales: A. identical salvage values
C. 4398 kg/m3 em diameter. If the pressure drop is 600
A. income turnover B. different salvage values kN/m 2, what must be the exit velocity.
D. 5398 kg/m 3
B. asset turnover C. ·identical equivalent uniform annual cash A. 16.76 m/s
C. expenses rate flows 52. A 30 inches long rod floats vertically in B. 26.76 m/s
D. rate of income D. different equivalent uniform annual cash
water. It has a 2.5 sq. inch cross section and C. 36.76 m/s
flows has a specific gravity of 0. 72. What length, L D. 46.76 m/s
39. General ability of a company to meet its that is floating?
short term indebtedness: 46. Which of the following does not affect
A. rate of return owners equity?

REFRESHER MANUAL 2N° Edition by JAS TORDILLO


REFRESHER MANUAL 2N° Edition by JAS TORDILLO
17-6 \ DAY 17 - EXAM
DAY 17 - SOLUTION 117-7
ENGINEER:tNG MATHEMATICS
57. Water flows through a pipe of cross
sectional area of 20 cm 2 . At end section the lrj ) DAY 17 - SOLUTION C. P227,681
2
cross sectional area is 4 cm . The pressure D. P457,551
difference between the two sections is 202.6 DAY 17- ANSWER KEY
l. 8 16. c 31. 0 46. c 1. The amount of P10,000 is borrowed at 30% S = P(1 + i)"
kPa. How many liters of water will flow out of per annum simple interest. How much will be s = 200,000( 1 + 0.18)5
2.0 l7.C 32.A 47.C
the pipe in 120 seconds assuming the pipe is 33. A 48.0 due at the end of 130 days?
horizontal?
3.A 18.0
4.A 19. A 34. A 49. C A. P11 ,830
s = 457,551 .55
A 586.2 liters
B. 786.2 liters
5.0 20. 0
6. c 21. A
35.A 50.8
36. B 51. B
B. P11,083 * Profit = S - P
37.8 52. 8
C. P11,028 = 457,551 .55 - 200,000
C. 886.2 liters 7.0 22.0
B. A 23. A 38. i! 53. c D. P11 ,038 Profit =257,551 .55
D. 986.2 liters
9.C 24. c 39.0 54. A
10.0 25. 8 40. 0 55.0 S = P(1 + ni) 5. A firm borrows P5000 for 5 years at 16%.
58. Determine the friction head loss for fully 11. C 26. 0 41.A 56.C S = 10,000[1+ ffi<o.Jo)J At the end of 5 years, it renews the loan for an
developed laminar flow of a liquid at 40°C {p = 42. A 57.0
12. 0 27. 8 S = P11 ,083.33 amount due plus P2000 more for 2 years at
1202 kg/m 3 ) through a 8 em diameter, 75 m 13. B 28. A 43.8 58. A
long pipe, if friction factor is 0.195 and a flow 14. 8 29. C 44. C 59.8 16%. What is the lump sum due?
15. A 30. 8 45. 0 60. 8 2. How much interests are you going to pay A. P17,822
rate of 2 kg/s. (); after 45 days if you borrowed P50,000 in a B. P16,282
A. 1.02 m
lending investor with an interest rate of 5% per C. P17,383
B. 1.20 m month.
C. 2.02 m D. P16,822 *
A. P3770
D. 3.20 m B. P3570 S = P(1 +i)"
59. Air at atmospheric pressure and with a
C. P3075 s = 50,000(1 + 0.16f + 2000{1 + 0.16)2
mean velocity of 1.0 m/s, flows inside a
D. P3750 * s = 14,131.09 + 2,691.20
square section duct of side b = 4 em. If the air s = 16,822.29
I = Pni
temperature is 350°K, determine the Reynolds
Number. {at 350°K viscosity= 20.76 x 10'
6
=
I 50,00om) (0.05) =P3750 6. What is the present worth of two P1000
m2 /s). payments at the end of third year and fifth
A. Re = 1296.8 3. How much should you put into a 10% year? The annual interest rate is 8%.
B. Re = 1926.8
A: P1744
savings account in order to have P100,000 in
C. Re = 1629.8 five years? B. P1384
C. P1474 *
D. Re = 1962.8 A. P62,092.20 *
B. P112,598.60 D. P1224
60. If the 'absolute pressure at the bottom of C. P62,902.20
the ocean is 300 kPa, how deep is the water D. P85,836.98 p = s
(Hi)"
at this point?
A. 16.66 m p = _s_ P=~+~
(1+1)"
B. 19.66 m (HO.OB)' (1+0.08)5
C. 29.66 m p = (1+0.10)5
100,000
D. 39. 66 m = 62,092.13 p = 793.83 + 680.58
p = 1474.41
4. A company invests P200,000 today to be
repaid in five years In one lump sum at 18% 7. A sinking fund consists of 15 annual
deposits of P1 ,000 each, with interest earned
compounded annually. How much profit in
present day pesos is realized over 5 years? at the rate of 4% compounded annually. What
A. P257,551 * is the principal in the fund at its terminal date?
A. P25,115.15
B. P247,896
B. P29,546.37

REFRESHER MANUAL 2N° Edition by JAS TORDILLO


REFRESHER MANUAL2N° Edition by JAS TORDILLO
17- sl DAY 17 - SOLUTION
DAY 17 - SOLUTION 17-9
ENGINEERING MATHEMATICS
I
C. P28, 105.25 Tax = 159,171.26 x 0 .1 0
D. P20,023.59 * Tax = 15,917.12 8%, what is the total peso amount available p = R [Ct+il" -
l(t+i)"
1]
upon the deposit of the tenth payment?
S = R [(l+i~" -1] Net Value = S- Tax
Net Value = 359,171.26- 15,171.26
A. P4,700
B. P10,410
10
111,739.34 :: R [<t+0.08) - 1]
R_ o.o8(t+o.o8J'o
Net Value = 343,254.13 C. P10,014
s = 1ooo r(l+0.04)1S
0.04
-11 D. P10,140 *
- 16,652.45
10. A bank is advertising 10% accounts that 15. What uniform annual amount should be
s = 20,023.58
yields 10.47% annually. How often is the S =R [(Hi);'-I] deposited each year in order to accumulate
interest compounded? P1M at the end of the 10th annual deposit if
A. quarterly
8. You borrowed the amount of P10,000 in a
bank with an interest rate of 30% B. daily S :: 700 [(1+0.08)10-
0.08
I] money earns 10% interest?
A. P62,745 *
compounded monthly. How much would you C. semi-annually B. P64,634
have to pay after 2 years?
D. monthly* s = 10,140.60 C. P67,045
A. P18,087 * D. P68,078
B. P17,987 i = (l+~f- 1 13. How much money must yciu invest today
in order to withdraw P5000 per year for 10 S = R [(t+l~" -1]
o.1o41 = (1+ ~ f- 1
C. 17,032 0 0
D. P17,825 years if the interest rate is 14%?
1.1047 = (1+ ~ f
0 0 A. P26,008 10
B. P26,081 * 1,000,000 :: R [(1+0.10) -1]
i = ~ = 2 .5% per month 0.10
12 C. P26,808
Try m =12
n = 2(12) = 24 months
D. P26,800 R = 62,745.39

1.1o47 = (1+ ~~ot


0
S = P(1 + i)"
p :: R [(t+iJ"-
l(t+i)"
1] 16. A company needs P60,000 in five years
to buy a new equipment. In order to
s = 10,000(1 + 0.025)
24
= 18,087.25 1.1047 = 1.1047
accumulate this sum, a sinking fund consisting
Therefore, m =12 (interest is compounded
p = 5000 [(t+O.l4-)' 0
-1]
0.14(1+0.14)'0 of three annual payments is established now.
9. An initial savings deposit of P200,000 in a
monthly). For tax purposes, no further payments will be
certain bank earns 60% .interest per annum
compounded monthly. If the earnings from p = 26,080.58 made after three years. What payments are
the deposit are subject to 10% tax, what would 11 . A man borrowed the amount of P20,000 necessary if money is worth 18% per annum?
be the net value of the deposit after one year? with an interest of 30%. How much interest is 14. A series of year end payments extending A. P12,620
he going to pay at the end of 22 months? over ten years are as follows: P5000 for the B. P12,220
A. P359,171
A. P32,353 first year, P10,000 for the second year, and C. P12,062*
B. P159,171
B. P31,533 P20,000 for each year from the third to the D. P12;602
C. P343,254 *
D. P325,896 C. P12,353 * tenth. Find the equivalent annual worth of
D. P32,296 ... these payments if the annual interest is 8%. s
p = (t+i)"
6 A. P16,634
i = 0% = 2 .5% per month
12 n = ~ = 1.833 yr B. P16,652 *
P=~
12 C. P16,365 (1+0.18)'
n = 1 year = 12 months D. P16,562
S = P(1 + i)"
s = 20,000( 1 + 0.30)
1 833 p = 26,226.55
S = P(1 + i)" '
P~+~+
s = 200,000(1 + 0.05)
12 s = 32,353.86 (1+0.08)' (1 + 0.08)2
p = R [(Hi)n-1]
s = 359,171.26 20 000 [ (1+0.08)' -1
0.08(1+0.08)'(1+0.08) 2
] 1(1+i)"
I = S- P '
26,226.55 :: :: R [(1+0.18)" -1]
I= S-P I = 32,353.86 - 20,000 = 12,353.86
p = 4,629.63 + 8,573.38 + 98,536.33 R = .12,062.21 o.18(t+o.1sl'
I = 359,171.26 - 200,000
I = 159,171.26 12. If P700 is invested at the end of each p = 111 ,739.34
year for 10 years, at an annual interest rate of
Tax = Interest x Tax rate
REFRESHER MANUAL 2N° Edition by JAS TORDILLO
REFRESHER MANUAL 2ND Edition by JAS TORDILLO
17-10 I DAY 17 - SOLUTION DAY 17 - SOLUTION 117-11
ENGINEERING MATHEMATICS
17. You obtain a loan of P 1.5M at the rate of SYD = ~2 (n + 1) = !£
2
(10 + 1) = 55
18% compounded annually in order to build a 23. Assets that cannot be converted into cash 29. Permit granted to a business to handle a
house. The payment is made at the first day
of the m onth. How much you pay monthly to Dep2 (FC - SV) G;;) 10 1
within the accounting period, such as land, given product or to operate within a given
buildings, machinery, and equipment that are territory or along a certain route:
amortize the loan within a period often years? Dep2 (150,000 - 20,000) ( - ) to be used in business operations: A. patent
A. P25,752 . 55 A. fixed assets * B. goodwill
B. P25,673 Dep2 21 ,272.73
B. used assets C. franchise *
C. P25,400 *
20. The hire of any person to do whatever
C. current assets D. expense
D. P25,501 D. intangible assets
work he is skilled in doing:
30. Factor in business which causes a
i = (1 + ipr-
1
2
A Promotion cost
B. Salary cost
24. Payment of goods or services before they company to earn a higher than normal rate of
are received: return due to managereal skill, popular
o.18 = (1+iS - 1 C. Material cost
acceptance of the product or some other
A current payment
ip = 0.013888 D. Labor cost *
B. fixed payment favorable circumstances:
C. prepaid expense * A. patent
1,500,000 = R + R r0.013888(1
(1+0.013888)u• - 1
+0.013888)
] 21. Compute the number of blocks that an ice
D. late payment B. goodwill *
119 plant must be able to sell per year to break
C. cleanliness
even, based on the following data:
R = 25,399.91 Cost of electricity per block
25. Payment of services or goods before they D. clearness
are rendered:
....................... P20.00
18. A machine cost P120,000 w ith an A advance payment 31 . Certain costs such as initial incorporation
Tax to be paid per block
estimated life of ten years. It salvage value B. prepaid income * fee and the cost of legal services in
.. ..... .. .............. . P2.50
after ten years is estimated to be P25,000. C. current payment connection with the formation of the
Real Estate tax
Determine the depreciation rate if the interest D. expense corporation.
.......... .... ... ... .... P48,000 per year
rate is 10%. A. material expense
Salaries and Wages
A 6.98% 26. Cash and goods that can be readily B. labor cost
.. .............. ..... ...... P30,000 per month
B. 4.82% converted into cash: C. equity cost
Others .. ...... ... .... .. .. .. . :. P15, 000 per month
C. 5.62% A fiXed assets D. organizational expense *
Unit Price ................. ... P90 per block
D. 4.97% * A. 8712 * B. intangible assets
C. equity 32. Funds in cash or secwity form held for
B. 7812
PC- SV D. current assets * income producing purpose:
AnnuaiDep = (l+O"-' C. 1378
A. investments •
i D . 6817
27. Assets that lack physical substance but B. expense
120.000-25,000 consists of valuable rights, privileges, or C. assets
Annual Dep = = 5,960.81 Let x = no. of blocks produced and sold per
(1+0.1.0)10_ 1
advantages; examples are patents, D. peso
0.10 year
franchises, organizational expense and
Income = Expenses
goodwill. 33. Accumulated net earnings of
Depreciation Rate = AnnuaL dep 90x = 20x + 2.50x + 48,000 + 30,000(12)
PC A current assets corporation, also called reinvested earnings:
+ 15,000(12)
B. intangible assets • A. retained earnings *
x = 8711 .11 say 8712 blocks per year
=~ C. fixed assets B. fixed deposit
120.000
D. gross assets C. net income
= 0.04967 = 4.967% 22. The bank offers 0.75% effective monthly
D. gross income
interest. What is the effective annual rate?
19. An asset is purchased for P150,000. Its 28. Exclusive privilege to use a certain
A 3.89% 34. Net losses in excess of retained earnin
estimated life is 10 years, after which it will be process in manufacturing:
B. 8.93% which are deductions in the owners equity:
sold for P20,000. Find the depreciation for the C. 9.01%
A. patent •
second year using the sum-of-the-years digit B. franchise A. deficit'*
D. 9.38% * B. debts
method. C. goodwill
A. P21,273 * D. expense C. gain
= (1 + ip)m - 1 D. gross margin
B. P21,723
C. P20,732
= (1 + 0.0075)12 -1 = 0.0938
= 9.38% effective annual rate
D. P21,836
REFRESHER MANUAL 2N° Edition by JAS TORDILLO
REFRESHER MAN\.!AL2N° Edition by JAS TORDillO
17-12 I DAY 17 - SOLUTION
35. Cost of goods available for sale during 42. Additional cost to produce one more unit
DAY 17 - SOLUTION 17 -13
ENGINEERING MATHEMATICS
I
the fiscal period minus the inventory of of product:
finished goods available at the end of fiscal A. marginal cost * 49. The cost of things that are neither labor 53. Steam with an' enthalpy of 900 cal/g enters
period: B. sunk cost nor materials: nozzle at a velocity of 90 m/sec. Find the
A. cost of goods sold ~* C. expense A. money velocity of the steam at the exit of the node if
B. liabilities D. operating cost B. damage its enthalpy is reduced to 700 cal/g, assuming
C. material cost C. expenses * the nozzle is horizontal and disregarding heat
D. organizational cost 43. Lowering in prices of commodities: D. asset losses. Take g = 9.81 m/s2 and J
A. inflation constant = 427 kg-mlkcal.
36. Difference between the net revenues B. deflation * 50. Type of ownership in business where A. 3297.55 m/s
realized from the sale of goods and their cost: C. bonus individuals exercise and enjoy the right in their B. 2297.55 m/s
A net capital D. discount own interest: C. 1297.55 m/s *
B. gross margin * A. Equitable D. 997.55 rnls
C. expense 44. What is an annuity? B. Private* v2 2
D. accounts A. the future worth of a present amount C. Public h +_I_ =h +l
D. Pure 1 2g(J) 2 2g(J)
B. an annual repayment of a loan
37. Net income divided by total capital: C. a series of uniform amounts over a 90 2
A ROI period of time * 51. A steel block weighs 10 Nand has volume 900 + ( ' = 700 + ___yJ_
of 300 cm 3. What is the density of the block in 2{9.81) (427) 2(9.81)(427)
B. ROR * D. a lump sum at the end of the year
C. interest I
3
kg/m ? v2 = 1297.55 m/s
D. asset rate 45. What must two investments with same A. 2398 kg/m 3
present worth and unequal lives have? B. 3398 kg/m 3 * 54. 200 g of water are mixed with 230 g of
38. Rate of return on total assets divided by A. identical salvage values C. 4398 kg/m 3 alcohol (d = 790 kg/cu.m). What is the
3 specific volume of its resulting mixtures,
rate of return on sales: B. different salvage values D. 5398 kg/m
assuming that the fluids mix completely?
A. income turnover C. identical equivalent uniform annual cash
A.1.14 cm 3/g *
B. asset turnover* flows Density = mass B. 1.41 cm3/g
C. expenses rate D. different equivalent uniform annual volume
D. rate of income cash flows*
C. 2.14 cm3/g
. 10 N kg (100)3em 3 D. 3.14 cm3/g
Dens11y = ---x--X-'----::--
39. General ability of a company to meet its 46. Which of the following does not affect 300 cm3 9.81N m3
Density = 3397.9 kg/m3 Tstal mqss = 200 + 230 = 430 grams
short term indebtedness: owners equity?
A rate of return A. dividends paid 0.200)
Total volume = ( - - + (0.230)
--
B. acid test ratio B. invested capital 52. A 30 inches long rod floats vertically in 1000 790
C. current asset turnover C. license to start business * water. It has a 2.5 sq. inch cross section and = 0.00049114 m3 = 491.14 cm3
D. current ratio * D. revenues has a specific gravity of 0.72. What length, L
. volume 491.14
that is floating? Spec1fic volume = - - =- -
A. 21.6 in mass 430
40. A more vigorous measurement of the 47. What is the acid test ratio?
A. the ratio of owners equity to total current B. 8.4 in* = 1.14 cm3/g
company's ability to service short term debt
A current ratio liabilities C. 11 .6 in
D. 18.4in 55. Water flows with a mean velocity of 3.2
B. rate of return B. the ratio of all assets to total liabilities
m/s inside a circular pipe of 4 em radius. The
C. gross margin ratio C. the ratio of current assets (exclusive
W =Fa bulk mean temperature of water is 60°C at this
D. acid test ratio * inventory) to total current liabilities * 3
VxdR=V,xdw temperature p = 985 kg/m and viscosity =
D. the ratio of profit after taxes to equity 4
A(30)dR = A(y)dw . 4.71 x 10' kg/m-s. What is the Reynolds
41. Accounting book in which are recorded
48. Decrease in value of a physical property Number?
the credits and debits of commercial
transactions: due to passage of time: y = 30( : : ) = 30 (Sp. Gravity) =
A. Re 2.35 x 105
5
A. inflation B. Re = 3.35 x 10
A. ledger*
B. journal B. recession y = 30(0.72) =
21.6 in =
C. Re 4.35 x 10
5

C. balance C. depletion L = 30 - y = 30- 21.6 = 8.4 in D. R.e = 5.35 x 105 *


D. clerks book D. depreciation *

REFRESHER MANUAL 2N° Edition by JAS TORDILLO


REFRESHER MANUAL 2N° Edition by JAS TORDILLO
17-14 I DAY 17- SOLUTION DAY 17 - SOLUTION 117 -15
Re = pvD = 985(3.2) (0.08) Q, = 02 ENGINEERING MATHEMATICS
JJ 4.71xto-4 20V1 = 4V2
Hydraulic Diameter for square duct:
Re = 5.35 x 105 v2= 5V, Eq. 2
4b2 4(4)2
D= - = - - = 4cm
3 Substitute Eq. 2 to Eq. 1: 4b 4{4)
56. An oil at 50°C (p = 890 kg/m flow into a 3
)
(5V,) - vl = 405.3
2
em diameter horizontal pipe with a velocity of Re = DV = 0.04{1.0)
v, = 4.109 m/sec
1.75 m/s and exits through a nozzle of 0. 75 J.l ~0. 76 X 10- 6
em diameter. If the pressure drop is 600 20 3
2 Q= ( )(4.109) = 0.0082189 m /sec
kN/m , what must be the exit velocity. 1002 Re = 1926.8
A. 16.76 m/s
B. 26.76 m/s After120 seconds: 60. If the absolute pressure at the bottom of·
C. 36.76 m/s • Volume= 0.0082189 x 120 = 0 .986268 m 3 the ocean is 300 kPa, how deep is the water
D. 46.76 m/s
= 986.2 liters at this point?
A 16.66 m
Bernoulli's Equation;
3 3 58. Detennine the friction head loss for fully 8.19.66 m *
y = 890 kg/m = 8. 73 kN/m
developed laminar flow of a liquid at 40°C (p = C. 29.66 m
P, - Pz = 600 kN/m 2 3
1202 kg/m ) through a 8 em diameter, 75 m D. 39.66 m
P1 V12 P2 V 2 2
long pipe, if friction factor is 0. 195 and a flow
-+ -+Z t
y 2g
=-y
+ - +Zz
2g rate of 2 kg/s. P(abs) = P(g) + P(atm)
A. 1.02 m • 300 = P(g) + 101.325 kPa
V22 = [PI
- -- -
y
P2+ -
2
V1+ (Z - Z ) 2g
2g
1 2
J B. 1.20 m P(gauge) = 198.675 kPa
C. 2.02 m
D. 3.20 m Density = SG x density of water
v 2 = [ 6oo + {1.75)2 + oJ2(9.8 1) SG of sea water =1.03
2
8. 73 2(9.81) Q=A xV 198·675 . 19.66 m
Hei ht = =
v2 = 36.76 m/s kg m3 n 2 g 1.03(9.8 1)
2 - x - - = -(o.o8)2m (V)
s 1202kg 4
57. Water flows through a pipe of cross Velocity= 0.331 m/s
sectional area of 20 cm 2 . At end section the 2
cross sectional area is 4 cm 2 . The pressure Frictional head loss, hf = fLV
difference between the two sections is 202.6 2gD
kPa. How many liters of water will flow out of 2
hf= (0. 195) (75) (0.331) = 1.02 m
the pipe in 120 seconds assuming the pipe is
2(9.81) (0.08)
horizontal?
A. 586.2 liters
B. 786.2 liters 59. Air at atmospheric pressure and with a
C. 886.2 liters mean velocity of 1.0 m/s, flows inside a
D. 986.2 liters • square section duct of side b = 4 em. If t he air
Bernoulli's Equation: temperature is 350°K, determine the Reynolds
Number. (at 350°K viscosity= 20.76 x 10~
P V12 P2 V/ m2/s).
-1+ - + Z t = -+-+Zz
y 2g y 2g A. Re = 1296.8
PI-P2- v22 -v~ 2 (Z
- ---
z)
+ 2- I
=
B. Re 1926.8 •
C. Re = 1629.8
y 2g
D. Re = 1962.8
2
{202.6) = v2 - Vt 2 +
0
9.81 2(9.8 1)
V/- V/ = 405.3 Eq. 1

REFRESHER MANUAL 2ND Edition by JAS TORDILLO


REFRESHER MANUAL 2ND Edition by JAS TORDILLO
DAY 18 - EXAM\ 18-1
ENGINEERING MATHEMATICS
6. A body weig hing 180 lb starts from
DAY 18- EXAM rest and slides down a plane at an angle
of 30° with the horizontal for which the
1. A horizontal platform with a diameter coefficient of friction f = 0.3. How far will
of 6 m revolves about its center at 20 it move when t = 4 sec?
rpm . Find the tangential speed, in m/s of A. 68.11 ft
a point at the edge of the platform. B. 11.68 ft
A. 6.34 c. 45.88 ft
B. 6.46 D. 61 .88 ft
C. 6.28
D. 6.12 7. A force of 400 lbf acts on a block at an
angle of 30° with the horizontal. The
2. The radius of the earth is 3960 mi. block is pushed 5 ft horizontally. What is
The gravitational acceleration of the the work done by this force?
2
earth's surface is 32.16 fps • What is the A. 2439 J
velocity of escape from the earth in mils? B. 2259 J
A. 4.91 c. 2689 J
B. 7.83 D. 2349 J
c. 8.62
D. 9.36 8. Trailer travels at 85 mi/hr around a
banked highway curve with a radius of
3. A train running at 60 kph decelerated 1000 ft. What banking angie is
at 2m/min 2 for 14 minutes. Find the necessary such that friction f = 0.3 to
distance travelled, in km within this resist the centrifugal force?
period. A. 6°
A. 12.2 B. 10°
B. 13.8 C. go
C. 13.2 D. 12°
D. 12.8
9 . Given the component velocities, what
4. A ·18 in long _rod floats vertically in is the resultant acceleration when t = 4?
water. It has a 5 in2 cross section and a Vx = t4 - t
specific gravity of 0.80. What length L is vy =6 -e
floating? A. 240
A. 14.4 in B. 270
B. 6.2 in c. 259
C. 14.9 in D. 310
D. 3.6 in
10. An elevator weighing 5000 lb attains
5. A car is traveling on a horizontal an upward velocity of 20 fps in 4 sec with
unbanked circular frack of radius r. uniform acceleration. What is the tension
Coefficient of friction between the tires in the supporting cable when elevator
and the track is 0.30. Determine the cars resistance is 20 lb per ton.
velocity while traveling at a radius of 34 A. 5826.41b
m, without skidding. B. 5676.81b
A. 15 m/s C. 5776.41b
B. 14 m/s D. 5576.21b
C. 12 m/s
D. 10 m/s

REFRESHER MANUAL 2ND Edition by JAS TORDILLO


18-21 DAY 18- EXAM DAY 18 - EXAM 118-3
ENGINEERING MATHEMATICS
11 . A 10 kg block was dropped 3 meters atm). What minimum volume of tank is
high. What is the velocity of the block necessary to transport the gas at 90°F 22. A perfect venture with throat elevation without accelerating, what is the
upon reaching the ground? and a maximum pressure of 10 atm? diameter of 2 in is placed horizontally in a balance reading?
A. 7.67 m/s A . 141 liters pipe with a 6 in pipe diameter. What is A. p
B. 7.76 m/s B. 60 liters the difference in static pressure if water B. P+2W
C. 6.77 m/s C. 152 liters flows 100 lb per second? C. P+W
D. 6.67 m/s D. 56 liters A. 247.2 kPa D. P - W
B. 242.7kPa
12. 150 g of water are mixed with 100 g 18. Air is compressed in a diesel engine C. 274.3 kPa 28. The vector which represents the sum
of alcohol (p = 790 kg/m 3 ). What is the from an initial pressure of 13 psia and a D. 224.7 kPa of a group of force vectors is called:
specific volume of the mixture assuming temperature of 120°F to one-twelfth of its A. magnitude
the fluids mix completely? initial volume. Calculate the final 23. Wliat is the water pressure if B. sum
A. 1.1 cm 3/g temperature assuming the compression manometer is 0.8 m Hg? Mercury is 13.6 C. resultant
B. 1.2 cm /~
3
to be adiabatic. times heavier than water. D. component
C. 0.95 em /g A. 1010°F A. 106.73 kPa
D. 1.3 cm3 /g B. 980°F B. 110.55 kPa 29. Which of the following is not a vector
C. 987°F C. 107.99 kPa quantity?
13. Absolute viscosity is essentially D. 1560°F D. 120.22 kPa A. velocity
independent of pressure and is primarily B. acceleration
dependent on: 19. A closed vessel contains air at a 24. Water is flowing in a pipe with radius C. speed
A. Temperature pressure of 200. kN/m 2 gauge and a of 25.4 em and a velocity of 10 m/s. The D. displacement
B. Specific gravity temperature of 32°C. The air is heated to viscosity of water is 1.131 Pa-s. What is
C. Density 60°C with the atmospheric pressure of the Reynold's Number? 30. A hoist with a 100 hp engine is
D. Velocity 750 mm Hg. What is the final gauge A. 4492 capable of lifting a 10,000 lb load to a
pressure? B. 4429 height of 20 feet in 10 seconds. What is
14. An instrument for measuring specific A. 337.54 C. 2449 the efficiency of this machine?
gravity of fluids. B. 127.54 D. 2944 A. 34.6%
A . Hygometer C. 227.54 B. 36.4%
B. Hydrometer D. 427.54 25. What is the density of a stone in C. 46.3%
C. Flowmeter kg/m 3 that weighs 90 N in air and 50 N in D. 43.6%
D. Psychrometer 20. ,A shot is fired at an angle of 45° with water?
the horizontal and velocity of 300 fps. A. 2259 31. An object is placed 3 feet from the
15. The sum of the pressure head, Calculate, to the nearest, the range of the B. 2502 center of horizontally rotating platform.
elevation head and the velocity head projectile. c. 2205 The coefficient of friction is 0.30. The
remains constant, this is known as: A. 350ft D. 2270 object will begin to slide off when the
A. Boyles law - B. 1200 m platform speed is nearest to:
B. Bernoulli's Theorem C. 400 mi 26. For a system to be in equilibrium, the A. 12 rpm
C. Terrecelli's law D. 932 yards sum of the external forces acting on the B. 22 rpm
D. Archimedes Principle system must be: C. 17 rpm
21. Steam with an enthalpy of 900 A. equal to unity D. 33 rpm
16. A liquid whose temperature is lower kcal!kg enters a nozzle at a velocity of 50 B. indeterminate
than the saturation temperature m/s. Find the velocity of the steam at the C. a maximum 32. Two 'barges, one weighing 10 tons,
corresponding to the existing pressure: exit of the nozzle if its enthalpy is D. zero the other weighing 20 tons are connected
A . compressed liquid reduced to 600 kcal/kg, assuming the by a cable in quiet water. Initially the
B. subcooled liquid nozzle is horizontal and disregarding heat 27. An air tight closed pox of weight Pis barges are at rest 100 feet apart. The
C. saturated liquid losses. suspended from a swing balance. A bird cable is reeled in until the barges are 50
D. pure liquid A. 1685 m/s of weight W is placed on the .floor of the ft apart. If the friction is negligible,
B. 1568 m/s box, and balance reads W + P. If the bird calculate the distance moved by the 10
17. A company specializes in the C. 1658 m/s flies around the box at a constant ton barge.
shipment of pressurized gaseous D. 1586 m/s A. 33.33 ft
materials. An order is received for 500
liters of a particular gas at (32°F and 1 REFRESHER MANUAL 2ND Edition by JAS TORDILLO
REFRESHER MANUAL 2ND Edition by JAS TORDILLO
18-4j DAY18-EXAM DAY 18- EXAM 118-5
ENGINEERING MATHEMATICS
B. 20ft 38. An open topped cylindrical water
C. 50ft tank has a horizontal circular base 10 ft in coolant is allowed to rise 10°F. The solely by raising the higher temperature
D. 16.66 ft diameter. When filled to a height of 8 water flow rate in liters per minute is: by 150 deg F and raising the low
feet, the force in lbs exerted on its base is A. 76 temperature by 100 deg F. Which of the
33. A projectile weighing 1 00 lbs strikes nearest to: B. 83 following statements is true?
the concrete wall of a fort with an impact A. 3,900 c. 34.5 A. thermodynamic efficiency decreased
velocity of 1200 ft/sec. The projectile B. 10,000 D. 54.3 B. thermodynamic efficiency increased
comes to rest in 0.01 second, having C. 7,800 C. less heat rejected in the system
penetrated the 8-foot thick wall to a D. 39,200 44. The maximum thermal efficiency that D. less heat added in the system
distance of 6 feet. What is the average can be obtained in an ideal reversible
force exerted on the wall by the 39. A gas bubble rising from the ocean heat engine operating between 1540 deg 50. Name the process that has no heat
projectile? floor is 1 inch in diameter at a depth of SO F and 340 deg F is closest to: transfer.
A. 3.73 x 105 lbs feet. Given that sp. gravity of sea water A. 100% A. isentropic
5
B. 3.33 X 10 lbs is 1.03, the buoyant force in lbs being B. 78% B. quasistatic
C. 2.37 x 105 1bs exerted on the bubble at this instant is c. 60% C. isothermal
D. 3.371bs nearest to: ' D. 40% D. polytropic
A. 0.014
34. A projectile· launced at 2000 ft/sec B. O.D76 45. Second law limitations on the 51 . A company pays 3.5% interest
from A at an angle of 30 deg with the c. 0.020 maximum horsepower output from any quarterly. What is the effective annual
horizontal, impacts at the same elevation D. 0.14 power unit burning 1,000,000 BTU/hr of interest rate?
at B. Assume no in-flight propulsion and fuel with high and low temperature A.. 10.75%
no aerodynamic drag. Horizontal 40. In fluid flow, if the fluid travels extremes of 1540 deg F and 40 deg F is: B. 14.750(o
distance in miles between A and B is parallel to the adjacent layers and the A. 98 C. 17.45%
nearest to: paths of individual particles do not cross, B. 1140 D. 24.75%
A. 7.9 the flow is said to be: C. 295
B. 15.9 A. laminar D. 3830 52. A savings certificate that costs P5000
c. 10.2 B. critical now will pay P8000 in five years. What is
D. 20.3 C. turbulent 46. Work or energy can be a function of the interest rate?
D. dynamic all of the following except: A. 9.85%
35. Kin'ematic viscosity may be A. force and distance B. 11.85%
expressed as: 41. Normal boiling point of liquid oxygen B. power and time C. 18.55%
A. ft2/sec is 90°K. What is this temperature in deg C. torque and angular rotation D. 25.55%
B. slugs/sec R? . D. force and time
C. sec2/ft A. -330 53. If P20,000 is invested now, P30,000
D. sec2/cm B. 162 47. A hig h velocity flow of gas at 800 two years from now, and P40,000 four
c. -183 ft/sec possesses kinetic energy nearest years from now (all at 5%), what will be
36. An open chamber rests on the ocean D. 168 to which of the following? the total amount in ten years?
floor in 160 ft of seawater (SG = 1.03). A. 1.03 BTU/Ib A. P110,505.37
What air pressure in psig must be 42. 55,000 gallons of water passes B. 9.95 BTU/Ib B. P130,505.37
maintained inside to exclude water? through a heat exchanger and absorbs C. 4.10 BTU!Ib C. P230,505.37
A. 45.6 28,000,000 BTU. The exit temperature is D. 12.8 BTU/Ib D. P330,505.37
B. 71.4 110 deg F. The entrance water
C. 60.9 temperature in deg F is nearest to: 48. Power may be expressed in units of: 54. What is the capitalized cost of a
D. 93.2 A. 49 A. ft-lbs project that will cost P20,000,000 now
B. 68 B. HP-hrs and will require P3,000,000 in
37. What head in feet of air, at ambient C. 56 C. BTU/hr maintenance annually? The effective
conditions of 14.7 psia and 68°F, is D. 73 D. kw-hr annual interest rate is 15%.
equivalent to 2 psia? A. P30,000,000
A. 146 43. The mass flow rate of Freon 12 49. A Carnot engine operating between B. P40,000,000
B. 288 through a heat exchanger is 10 lbs/min. 70 deg F and 2000 deg F is modified
C. 395 Enthalpy of Freon entry is 102 BTU/Ib
D. 3830 and of Freon exit is 26 BTU/lb. Water REFRESHER MANUAL 2ND Edition by JAS TORDILLO
REFRESHER MANUAL 2ND Edition by JAS TORDILLO
18-61 DAY 18- EXAM DAY 18 - SOLUTION 118-7
C. P50,000,000
ENGINEERING MATHEMATICS
cost of P0.52 per unit. What is the break-
D. P60,000,000 even point? 4. A 18 in long rod floats vertically in
A. 222,223 units DAY 18 - SOLUTION water. It has a 5 in2 cross section and a
55. What is the present worth of B. 242,223 units specific gravity of 0.80. What length L is
P60,000.00 due in 5 years if money is C. 252,223 units 1. A horizontal platform with a diameter floating?
worth 15% and is compounded semi- D. 222,222 units of 6 m revolves about its center at 20
annually?
A 14.4 in
rpm . Find the tangential speed, in m/s of B. 6.2 in
A P19,111.63 a point at the edge of the platform. C. 14.9 in
B P29, 111.63
C. P39,111.63
'd A. 6.34 D. 3.6 in*
B. 6.46
DAY 18- ANSWER KEY
D. P49,111.63
1.C 16. B 31. C 46.0
c. 6.28 * W =Fe
2. A 17.0 32. A 47. 0 D. 6.12
56. A sum of money is deposited now in
3.8 18.A 33. A 48. C A(L)Prod = A(L-X)Pwater
a savings account. The effective annual 4.0 19.( 34. D 49.A V = TTDN
interest rate is 12% and interest is 5. 0 20. D 35. A 50. A L Prod = L- X
compounded monthly. How much money
must be deposited to yield P1 ,500.00 at
6. D 21. D
7. D 22. A
36. B
37. D
51. B
52. A
V = rr(6) GD = 6.28 m/s Pwacer

the end of 10 months? 8.C 23. A 38.D 53.8 18(0.8) = 18- X


A P1351.99 9.( 24. A 39. C 54. 8 2. The radius of the earth is 3960 mi. x= 3.6 in
10. A 25. A 40. A 55. 8 The gravitational acceleration of the
B. P1551.99
ll.A 26. D 41. B 56. A earth's surface is 32.16 fps 2 What is the
C. P2351.99 5. A car is traveling on a horizontal
12. A 27. C 42. A 57. A
D. P3351.99 velocity of escape from the earth in mils? unbanked circular track of radius r.
13. A 28. C 43. c 58. c
14. 8 29. C 44. C 59. B A. 4.91 * Coefficient of friction between the tires
57. Find the compound amount if P7500 15. 8 30.8 45. C 60. A B. 7.83 and the track is 0.30. Determine the cars
is invested at 12% compounded quarterly G c. 8.62 velocity while traveling at a radius of 34
for 7 years and 6 months. D. 9.36 m, without skidding. ·
A. P18,204.47 A. 15 m/s
B. P28,204.47 W =Fe B. 14 m/s
C. P38,204.47 C. 12 m/s
D. P48,204.47 mg = mv• D. 10 m/s *
R

58. How much money must be invested 2 f = v•


V = gR = (32 ..16)(3960 X 5280) gR
today in order to withdraw P12,000 per V ~ 25.931.2 fUsee = 4.91 mi/sec
year for 12 years if the interest rate is
v•
12%? 3 . A train running at 60 kph decelerated 0.30 = 9.81(34)
A P44,332.49 at 2m/min 2 for 14 minutes.
B. P54,332.49
C. P74,332.49
... Find the
distance travelled, in km within this V = 10 m/s
period.
D. P94,332.49 6. A body weighing 180 lb starts from
A. 12.2
B. 13.8 * rest and slides down a plane at an angle
59. How long will it take a sum of money of 30° with the horizontal for which the
C. 13.2
to double at 10% annual percentage coefficient of friction f = 0.3. How far will
D. 12.8
rate?
it move when t = 4 sec?
A. 5.27 years
B. 7.27 years
v1 = 60 kph = 1000 m/min A 68.11 ft
B. 11.68 ft
C. 9.27 years
D. 12.27 years
S = V1t + Y. at2 c. 45.88 ft
s = (1000)(14) + y. (-2)(14)2 D. 61.88 ft *
S = 13.804 m = 13.8 km
60. In a small factory with a capacity of I:Fx = 0
500,000 units per year with a 75% REF+ Ft -Wx = 0
efficiency, the annual income is PSOO,OOO
and a fixed cost of P180,000 and variable REFRESHER MANUAL 2ND Edition by JAS TORDILLO
REFRESHER MANUAL 2ND Edition by JAS TORDILLO .
18- sl DAY 18- SOLUTION
DAY 18 - SOLUTION 118-9
A. 240 ENGINEERING MATHEMATICS
~ + Wcos 30° f- W sin 30° = 0 B. 270
g
c. 259. 12. 150 g of water are mixed with 100 g C. saturated liquid
D. 310 of alcohol (p = 790 kg/m 3 ). What is the D. pure liquid
~ + cos 30° (0.3) - sin 30° = 0
32.2 spec!fic volume of the mixture assu.rning
v. = t4 - t the fluids mix completely? 17. A company specializes in the
a = 7.734 ft/sec2 v; = a. = 4t3 - 1 = 4(4) 3 - 1 = 255 A. 1.1 cm 3/g • shipment of pressurized gaseous
B. 1.2 cm /~
3
materials. An order is received for 500
S = V1t + Y. at2 vy = 6- e C. 0.95 em /g liters of a particular gas at (32°F and 1
s = 0 + 1/2 (7.734)(4)2 Vy' = ay = -3f = -3(4) 2 = -48 D. 1.3 cm 3/g atm). What minimum volume of tank is
S=61.872ft necessary to transport the gas at 90°F
ar = .Jai+af, Vatc = md!c
Pale
= ~
790
m3 and a maximum pressure of 10 atm?
7. A force of 400 lbf acts on a block at an A. 141 liters
angle of 30° with the horizontal. The B. 60 liters
block is pushed 5 ft horizontally. What is .J
= (255)2+ ( 48) 2 Vwater = ~
Pwatar
= o1000
.15o m3
C. 152 liters
the work done by this force? I ar = 259.47 D. 56 liters •
A. 2439 J 0.100 0.150

10. An elevator weighing 5000 lb attains v : : :; --+--


790 1000
= P2 V2
B. 2259 J 150+100
P1 V1
an upward velocity of 20 fps in 4 sec with T1 T2
0.00000110 ~ x 1,000,000 cm3
C. 2689 J 3
D. 2349 J * uniform acceleration. What is the tension 6 g m- (1)(500) (l0)V
in the supporting cable when elevator --- = --- 2

resistance is 20 lb per ton. v = 1.1 cm 3/g 32+460 90+460


W = FxD
W = 400cos 30° x 5 A. 5826.4 lb *
B. 5676.81b 13. Absolute viscosity is essentially Vz = 55.90 liters
C. 5776.41b independent of pressure and is primarily
W = 1732 ft-lb X~ dependent. on: 18. Air is compressed in a diesel engine
118[t-lb D. 5576.21b
w = 2348.73 J A. Temperature • from an initial pressure of 13 psia and a
tFv = 0 B. Specific gravity temperature of 120°F to one-twelfth of its
8. Trailer travels at 85 mi/hr around a T - W - R - REF = 0 C. Density initial volume. Calculate the final
banked highway curve with a radius of T ·= W+ R +REF D. Velocity temperature assuming the compression
1000 ft. What banking angle is to be adiabatic.
14. An instrument for measuring specific A. 1010°F *
necessary such that friction f = 0.3 to a = ~ = 5 ft/sec2
resist the centrifugal force? 4 gravity of fluids. B. 980°F
A. 60 w = 5000 lbs A. Hygometer C. 987°F
B. 10° B. Aydrometer • D. 1560°F
~
201
C. go* R = b x5000 lb x = 50 lbs C. Flowmeter
ton 2000 lbs
D. 12° D. Psychrometer ~ = (!:!.)k-1
REF = soooc5l
32.2
=· 776.4 lbs 15. The sum of the pressure head,
T1
T
Vz
( V )1.4-l
V = 85 mi/hr = 124.67 ft/sec
Tan a = 0.3 elevation head and the velocity head 120:460 = &_~,
a = 16.7° T = 5000 + 50 + 776.4 = 5826.4 lbs remains constant, this is known as:
A. Boyles law T 2 = 1567.11°R = 1107.11°F
yl 11. A 10 kg block was dropped 3 meters B. Bernoulli's Theorem *
tan (9 +a) high. What is the velocity of the block 19. A closed vessel contains air at a
gR C. Terrecelli's law
upon reaching the ground? 2
D. Archimedes Principle pressure of 200 kN/m gauge and a
tan(9 + 16.7) = (124.67),
A. 7.67 m/s * temperature of 32°C. The air is heated to
= . 32.2(1ooo) B. 7.76 m/s 16. A liquid·whose temperature is lower 60°C with the atm9spheric pressure of
9 9 060 C. 6.77 m/s than the saturation temperature 750 mm Hg. What is the final gauge
D. 6.67 m/s corresponding to the existing pressure: pressure?
9. Given the component velocities, what A. compressed liquid A. 337.54
is the resultant acceleration when t = 4? vi = v, 2 + 2gh B. subcooled liquid * B. 127.54
v. = t4 - t vi = o + 2(9.81 )(3)
vy = 6- e v3 = 7.67 m/s
REFRESHER MANUAL 2ND Edition by JAS TORDILLO
REFRESHER MANUAL 2ND Edition by JAS TORDILLO
18-10 I DAY 18 -SOLUTION DAY 18 - SOLUTION 18-11
ENGINEERING MATHEMATICS
I
c. 227.54. pipe with a 6 in pipe diameter. What is
D. 427.54 the difference in static pressure if water 25. What is the density of a stone in 30. A hoist with a 100 hp engine is
flows 100 lb per second? kgfm 3 that weighs 90 N in air and 50 N in capable of lifting a 10,000 lb lo(ld to a
Pa1m = 750 mm Hg = 100 kPa A. 247.2 kPa * water? height of 20 feet in 10 seconds. What is
B. 242.7kPa A. 2259 * the efficiency of this machine?
!2=~ C. 274.3 kPa B. 2502 A. 34.6%
T1 Tz
D. 224.7 kPa C. 2205 B. 36.4% *
~=~ D. 2270 c. 46.3%
32+273 60+273 p,_ P• = v,>-v,• + (Z2- Z1) D. 43.6%
y 29
Fe = V XPw
P2 = 327.54 kPa abs = 227.54 kPag 90 -50 = V(9810) P;n = 100 hp
Q =~ = p
100
62.4
= 1.602564 ft /sec3
V = 0.004077 m3
20. A shot is fired at an angle of 45° with p = ~ = 10,0001bx20/t X hp
0 time 10sec
the horizontal and velocity of 300 fps. ~ = 8.16 ft/sec •• ssoft-lb

Calculate, to the nearest, the range of the v1 - _


::~-
Q _
1
!!(~)·
4 12
Density d :;;: ~
' v = 0.004077
•·•' = 2250 kg/m
3
= 36.36 hp sec
projectile.
A. 350ft
v2 = = .!!. =
1 602
' , = 73.46 ft/sec 26. For a system to be in equilibrium, the E!f =~=
P;1
· ~X 100 =
100
36.36%
B. 1200 m Az Hi\) sum of the external forces acting on the
C. 400 mi system must be:
31. An object is placed 3 feet from the
D. 932 yards * P,_ Pz = (73.46) 2 -(8.16) 2 + O A. equal to unity
center of horizontally rotating platform.
62.4 2(32.2) B. indeterminate
The coefficient of friction is 0.30. The
R = v,fsin28 C. a maximum object will begin to slide off when the
.9 ~P = 5164.26 lb/W = 35.86 psi D. zero*
platform speed is nearest to:
= 247.2 kPa
R = (300) 2 sin (2x•S) A. 12 rpm
27. An air tight closed box of weight P is
32.2
23. What is the water pressure if suspended from a swing balance. A bird B. 22 rpm
manometer is 0.8 m Hg? Mercury is 13.6 of weight W is placed on· the floor of the
C. 17 rpm*
R = 2795.03 ft = 931.7 yards times heavier than water. box, and balance reads W + P. If the bird
D. 33 rpm
21. Steam with an enthalpy of 900 A: 106.73 kPa * flies around the box at a constant
B.. 110.95 kPa . elevation without accelerating, what is the Fr = Fe
kcal/kg enters a nozzle at a velocity of 50
C. 107.99 kPa balance reading?
m/s. Find the velocity of the steam at the f(mg) = mv•
exit of the nozzle if its enthalpy is D. 120.22 kPa A. p R
reduced to 600 kcal/kg, assuming the B.• P +2W
P :;;: YL h = SG(9.81 )h
0.30(m)(32.2) = mv•
nozzle is horizontal and disregarding heat C. P +W*
p = 13.6(9.81 )(0.8) D. P-W 3
losses. V = 5.38 ft/sec = 322.8 ft/min
A. 1685 m/s
P = 106.73 kPa
B. 1568 m/s 28. The vector which represents the sum
24. Water is flowing in a pipe with radius of a group of force vectors is called: V = 21TRN
C. 1658 m/s 322.8 = 21T(3)N
of 25.4 em and a velocity of 10 m/s. The A. magnitude
D. 1586 m/s * N = 17.12 rpm
viscosity of water is 1.131 Pa-s. What is B. sum
the Reynold's Number? C. resultant*
Joules constant: 427 kg-m = kcal 32. Two barges, one weighing 10 tons,
A. 4492 * D. component
B. 4429 the other weighing 20 tons are connected
h
I
+ _y]_=
2g(427)
h 2 + _!L_
29(.27) C. 2449 29. Which of the following is not a vector by a cable in quiet water. Initially the
D. 2944 quantity? barges are at rest 100 feet apart. The
·A. velocity cable is reeled in until the barges are 50
900 + = (600) + _..!]_
(SO)'
2(9.81)(427) 2(9.81)(.27) D = 2(25.4) = 50.8 em = 0.508 em B. acceleration ft apart. If the friction is negligible,
C. speed* calculate the distance moved by the 10
Vz = 1586.13 m/s IJ = 1.1
31
= 0.001131 m2/sec D. displacement ton barge.
1000 A. 33.33 ft *
22. A perfect venture with throat
diameter of 2 in is placed horizontally in a
I _ ov _ o.soa(1o) _
NR - -; - Q.OOi"ljl - 4491.6 REFRESHER MANUAL 2ND Edition by JAS TORDILLO
REFRESHER MANUAL 2ND Edition by JAS TORDILLO
18 - 141 DAY 18 - SOLUTION DAY 18 - SOLUTION 18-15
ENGINEERING MATHEMATICS
I
45. Second law limitations on the temperature by 100 deg F. Which of the
maximum horsepower output from any following statements is true? 1.6 = (1 +i) 56. A sum of money is deposited now in
power unit burning 1,000,000 BTU/hr of A. ther modynamic efficiency 1 + i = 1.09856 a savings account. The effective annual
fuel with high and low temperature decreased * i = 0.09856 = 9.85% interest rate is 12% and interest is·
extremes of 1540 deg F and 40 deg F is : B. thermodynamic efficiency increased compounded monthly. How much money
A. 98 C. less heat rejected in the system 53. If P20,000 is invested now, P30,000 must be deposited to yield P1,500.00 at
B. 1140 D. less heat added in the system two years from now, and P40,000 four the end of 10 months?
c. 295 . years from now (all at 5%). what will be A. P1351.99 *
D. 3830 -_ -----
TH-TL 2000-70
------- the total amount in ten years? B. P1551 .99
Eff1 :::;
TH 2000+ 460
A. P11 0,505.37 C. P2351 .99
_ Btu hp
QA - 1 ,OQQ,QQQ -h X ---;m; - 392.93 hp
_
B. P130,505.37 * D. P3351 .99
T 25451iT = 0.784 = 78.4%
C . P230,505.37
D. P330,505.37 Solving for the effective rate per month:
w 1540-40 Eft = (2ooo+lso)-(7 0 Hoo)
392.93 :::; 1540+460
2
(2000 + 1 50) + 460 . .•n )m
(!+-;; -J
8
= 0.758 = 75.8% s 40,000(1+0.05)6 + 30,000{1+0.05)
=
10
I=

w= 294.7 hp + 20,000(1 + 0.05) .


Therefore, the efficiency decreases. s = 53,603.82 + 44,323.66 + 32,577.89 . )12
46. Work or energy can be a function of S = P130,505.37 0.12 = (1+ ~; -1
all of the following except: 50. Name the process that has no heat
A. force and distance transfer. 54. What is the capitalized cost of a
project that will cost P20,000,000 now
i
...!!...
12
=0.94888% per month
B. power and time A. isentropic *
C. torque and angular rotation B. quasistatic and will require P3,000,000 in S =
P(1 + i)"
11
D. force and t ime • C. isothermal maintenance annually? The effective 1500 = P(1 +0.0094888)
D. polytropic annual interest rate is 15%. P = P1351 .99
47. A high velocity flow of gas at 800 A. P30,000,000
ft/sec possesses kinetic energy nearest 51 . A company pays 3.5% interest B. P40,000,000 * 57. Find the compound amount if P7500
to which of the following? quarterly. W hat is the effective annual C. P50,000,000 is invested at 12% compounded quarterly
A. 1.03 BTU/Ib interest rate? D. P60,000,000 for 7 years and 6 months.
B. 9.95 BTU/Ib A·1 0 .75% A. P18,204.47 *
C. 4.10 BTU/Ib B. · 14.75~o * B. P28,204.47
Capitalized Cost = FC + OM
D. 12.8 BTU/Ib * c. 17.45% i C. P38,204.47
D. 24.75% D. P48,204.47
= 20,000,000 + 3,000,000
KE = wv> 0.15
2g in = nominal interest rate = 3. 5% x 4 S = P(1 + it
= 14% compounded quarterly · = P 40,000,000 n =7.5 x 4 =30 quarters
KE = (BOO)' = 9937.88 ft-lbllb i = effective annual interest rate
55. What is the present worth of i = g_
= 3% per quarter
2{32.2) .
4
=9937.88 /Hb X ~ 4 P60,000.00 due in 5 years if money is
(l+mln)m- (!+4
KE
tb 778 f t -tb = I =
0. 14) .,. I worth 15% and is compounded semi- s = 7500(1 + 0 .03)
30

= 12.77 Btullb
annually? S = P18,204.47
= 14.75% per annum A. P19,111 .63
48. Power may be expressed in units of: 58. How much money must be invested
B. P29,111.63 *
A. ft-lbs 52. A savings certificate that costs PSOOO today in order to withdraw P12,000 per
C. P39,11 1.63
B. HP-hrs now w ill pay P8000 in five years. What is year for 12 years if the interest rate is
D. P49,111 .63
C. BTU/hr t he interest rate? 12%?
D. kw-hr A. 9.85% *
B. 11.85%
S = P(1 + i)" A. P44,332.49
B. P54,332.49
49. A Carnot engine operating between C. 18.55% 60,ooo = ·p(l+ o.;sf C. P74,332.49 *
70 deg F and 2000 deg F is modified D. 25.55% D. P94,332.49
solely by raising the higher temperature P = P29,111 .63
by 150 deg F and raising the low S = P(1 + i)"
8000 = 5000(1 + i)5 REFRESHER MANUAL 2ND Edition by JAS TORDILLO
REFRESHER MANUAL 2ND Edition by JAS TORDILLO
19-21 DAY 19- EXAM DAY 19- EXAM
ENGINEERING MATHEMATICS
119-3
14. Integrate: (t•H)dt c. y.
t 1 +12t+~ D. -1/4
A. (1/2) ln(e + 12t + 2) + c 27. Find the second derivative of 34. Find the partial derivative with respect
B. (1/3) ln(t3 + 12t + 2)2 + C y = ..fX2 + )(2 to x of the function:
21. In triangle ABC, AB = 8 m and BC =
c. (1/2) ce + 12t + 2) 2
+ c 20 m. One possible dimension of CA is:
A. -6-x?- 150-x?-y- 10xy + 3y + 5
. D. (1/3) ce + 12t + 2)2
+ c A. 13
B. -2x'3
C. 6x-4
A. 300xy - 1Oy
B. 150~ -10x
B. 9 D. 6x3 C. 150y- 10y
15. The radius of the sphere increases at C. 11
the rate of 3 em per second from zero D. 300xy -10x + 3
D. 7
initially. How fast is the volume 28. Where does the ~oint of inflection
increasing after 2 seconds? occur in the equation: x - 3x. 35. As x increases uniformly at the rate
22. Determine the sum of the infinite A. first quadrant of 0.001 ftlsec, at what rate is the
A. 4327t cm3/sec geometric series of 1 - 1/5 + 1/25.
B. 6327t cm 3/sec B. ongm expression (1 + x)2 increasing when x =
A. 5/7 4ft?
C. 532n cm 3/sec C. third quadrant
B. 4/7 D. y-axis A. 0.01
D. 8327t cm 3/sec C. 5/6 B. 0.001
D. 4/5 c. 0.10
29. The value of e raise to the negative 5
16. There are seven arithmetic means
power is equal to: D. 1.01
between 3 and 35. Find the sum of all 23. Polygons are classified according to A . 0.0067379
the terms. their: B. 0.0063779 36. What is the value of each interior
A. 169
B. 167
c. 171
D. 173
A. diagonals
B. angles
C. sides
D. vertices
c. 148.41 30
D. 0.0076397 .
angle of a regular pentagon?
A. !!!
B. i

17. The logarithms of the quotient and


the product of two numbers are
24. An object moves along a straight line
such that, after t minutes, its distance
30. A quadrilateral with no side parallel:
A. trapezoid
B. rhombus
C. rhomboid
C. i
D. .
!?!

0.352182518 and 1.556302501 , from its starting point is S = 20t + 5/(t + 1) D. trapezium 37. How many cubic meters is 150
respectively. Find the first number. meters. At what speed, in m!min wi ll it be gallons of liquid?
A. 11 moiling at the end of 4 minutes? A. 0.58761
31 . What is the slope of the curve
B. 9 A. 39.8 B. 0.57871
C. 10 B. 29.8
X:+ l- 6x + 10y + 5 = 0 at (1,0)?
c. 0.56781
D. 12 A. 2/5 D. 0.56871
C. 49.8 B. -2/5
D. 19.8 C. 512
18. Differentiate: 7-x?- + 16x. D. -5/2 38. Determine the center of the curve:
A . 14x - 16 25. A cone was formed by rolling a thin 2-x?- + 2y2- 8x + 4y- 40 = 0.
B. 14x + 16 s heet of metal in the form of a sector of a A. (2, 1)
32. What is the equivalent expression for
C. 7x + 16 circle 72 em in diameter with a central B. (2, -1)
sin 2x?
D. none of these angle of 150 degrees. Find the volume of A. % sin x cos x
c. (-2, -1)
the cone in cc. D. (-2, 1)
B. 2sin x
19. What is the length of the latus rectum A. 7733 sec x
o.fthe curve: x2 = -12y? B. 7744 C. -2 sin x 39. Factor the expression: 2-x?- + 12x +
A. 12 c. 7722 D. - 2 sin X cos X 16 as completely as possible.
B. 3 D. 7711 A. 2(x- 2)(x- 4)
C. -3 33. Given the function f(x) = x to the 3 B. 2(x + 2)(x- 4)
D. -12 26. If n is the number of sides of a rd power - 12x + 1, find the value of the C. 2(x- 2)(x + 4)
polygon, then the number of diagonals of first derivative at x = 1. D. 2(x + 4)(x + 2)
20. Find the slope of the line whose a polygon is expressed as: A. 9
parametric equations are x = 4t + 6 and y A. n! B. 12 40. Body or space bounded by surface
= t -1 . B. (n/2)(n - 3) c. -9 every point of which is equidistant from a
A . -4 C. (n-1)! D. 3 point within.
B. 4 D. (n/3)(n- 2) A. circle
REFRESHER MANUAL 2ND Edition by JAS TORDILLO
REFRESHER MANUAL 2ND Edition by JAS TORDILLO
19·41 DAY 19- EXAM DAY 19 - EXAM 119 -5
ENGINEERING MATHEMATICS
B. ellipsoid 47. A line segment joining two points on
C. spheroid a circle is called: What is the deformation of the spring in B.1.75in
D. sphere A. arc C. 1.95 in 3
em due to the body on top?
B. sector 3
A . 2.1 em D. 2.47 in
41 . The slope of the line is Y. . The C. tangent B. 3. 1 em
slope of the second line is -213. Both D. chord C. 4.1 em 59. Find the polar section modulus of a
lines intercept at the point (3, 1). What is hollow shaft with an outside diameter of 5
D. 5.1 em
the acute angle between the lines? 48. All circles having the same center inches and inside diameter of 3 inches.
A. 60.3° but with unequal radii are called : A 51.36 in3
54. A steel rod 25 mm in diameter and
B. 75.4° A. encircle 1000 mm long has an allowable B. 41.36 in
3

C. 67.2° B. concyclic C . 31 .36 in3


elongation not to exceed 3 mm, find the
D. 90.0° C. tangent circles D. 21.36 in
3
allowable load in kN.
D. concentric circles A. 604.5 kN
42. The position of an object as a B. 504.5 kN 60. A 16 ft horizontal beam is subjected
=
function of time is described by x 4t3 + 49. The esc of 960 deg is equal to: C. 404.5 kN to a load of 500 lbs located at its center.
2t2 - t + 3. What is the ;;~cceleration of A.1 D. 304.5 kN The dimension of the beam is 3 x 5
the object at t = 2? B _'!!_ inches respectively and its unit weight is
A. 52 . 2
55. Find the section modulus in cubic 100 lb/ft. Find its flexural stress in psi.
B. 64 C. y.
2-/3 inch of a shaft with diameter 2.25 inches. A. 2992 psi
C. 56
D. 66
D. --3 A 1.11 B. 3255 psi
B. 2.22
c. 4992 psi
50. A quadrilateral with two and only two D. 5982 psi
43. The velocity of a particle at time t is: sides of which are parallel is called: C. 3.33
v(t) = 12t4 + 7/t. What total distance is A . parallelogram D. 4.44
travelled between t = 0.2 and t = 0.3?
A. 2.84
B. quadrilateral (Cj=======;-1
C. trapezoid 56. What is the decrease in lateral DAY 19-ANSWER KEY
B. 3.85 dimension due to an axial tensile force of
D. rhombus 1. A 16.C 31.A 46. B
C. 3.25 40,000 lbs in a bar 2.5 inches on each 2. c 17. 8 32. 8 47. 0
D. 2.48 side and 5 feet long? The material is 3. A 18.8 33. c 48. 0
51. Compute for the torsional deflection
in degrees .pf a 100 mm diameter, 1.75 steel (E = 3.0 x 107 psi, J.l = 0.3). 4.8 19.A 34. A 49. D
44. The value of (4 to the 2"d power) A 5.133 x 10_. in/in S. A 20.C 35. A 50. C
meter long shaft subjected to a twist 6. B 21. A 36. D 51. D
square root is equal to: B. 4 .133 x 10" 4 in/in
moment of 5.5 x 106 N-mm. The 7. A 22. c 37. c 52.8
A. 2 C. 3.133 x 10"4 in/in
torsional modulus of elasticity is 85000 8. 0 23.( 38. 8 53. B
B. 3 4
D. 2.133 X 1Q- in/in
N/mm2 . 9. 8 24.0 39. D 54. D
C. 16
A. 0.22 deg 10. A 25. D 40. D 55. A
D. 4 57. A wire of length 1.90 m has a 11. A 26.8 41. A 56. D
B. 0.33 deg
12.8 27.( 42. A 57. C
c. 0.55 deg percentage strain of 0.032% when loaded
43.A 58. A
45. What is the simplified equivalent D. 0.66 deg with tensile force. Determine the 13. D 28. B
expression of (cof 8)(sin2e) + 1/csc2e. extension of the wire in mm.
14. B 29. A 44. 0 59.0
A.O
B. cos e
52. What diameter of bolt is required if
the bolt is subjected to a tensile load of
A. 0.108mm
B. 0.408 mm
c: 15. A 30. D 45. D 60. 0

c. sin2e 24,000 lb and a torque (for tightening) of C. 0.608 mm


D. 1 7,000 in-lb and maximum normal stress D. 1.208 mm
is not to exceed 30,000 psi?
46. A angle greater than a straight angle A. % inch 58. Determine the polar section modulus,
and less than two straight angles are B. 1 inch Zp (in3 ) of a shaft delivering 15 Hp at 300
called: C. 1 Y. inch rpm. The diameter of the shaft is 2.0 in
A. right angle -D. 2 inches and allowable shear stress of 4500 psi.
B. reflex angle Use a factor of safety of 5.
C. obtuse angle 53. A body weighing 1,019.36 kg was put A . 1.57 in3
D. acute angle on top of a 3205 N/cm helical spring.
•.REFRESHER MANUAL 2ND E~ifion by JAS TORDILLO
REFRESHER MANUAL 2ND Edition by JAS TORDILLO
19- 8 I DAY 19 - SOLUTION DAY 19 -SOLUTION 119-9
ENGINEERING MATHEMATICS
C. 7x + 16 C. 5/6 *
D. none of these D. 4/5
h=~ U~ing calculator: e"" = 0.006737946

Y = 7x?- + 16x s = _;__ h = .J(36)Z- (15) 2 = 32.73 30. A quadrilateral with no side parallel:
1-r
y' = 14x + 16 1 A. trapezoid
a=1, r = J. =-; V = !(rrr2 )h B. rhombus
1 5 3
19. What is the length of the latus rectum C. rhomboid
of the curve: X: = -12y? S =2.... -5 V = ~ (rr(15)2 )(32.73) = 7710.91 cm3 D. trapezium *
A. 12 * t+i- 6
B. 3 31. What is the slope of the curve
26. If n is the number of sides of a
C. -3 23. Polygons are classified according to
polygon, then the number of diagonals of
x2 + f-
6x + 10y + 5 = 0 at (1,0)?
D. -12 their: A. 2/5 *
A. diagonals a·polygon is expressed as:
B. -2/5
= 4a = 12 A. n!
Length of latus rectum B. angles
B. (n/2)(n - 3) *
c. 5/2
C. sides* D. -5/2
20. Find the slope of the line whose D. vertices C. (n -1)1
D. (n/3)(n- 2)
parametric equations are x = 4t + 6 and y x2 + f- 6x + 10y + 5 = 0
= t-1. 24. An object moves along a straight line 2x + 2yy' - 6 + 1Oy' = 0
27. Find the second derivative of
A. -4 s uch that, after t minutes, its distance
Y =..(Xi+ x·z
B. 4 from Its starting point is S = 20t + 5/(t + 1)
A. -6x23 y' =~ at(1 0)
c. y.. • meters. At what speed, in m/min will it be '
2y+10 '
6-2(1) 4 2
D. -1/4 moving at the end of 4 minutes? B. -2x' y = = 2(0)+10 ='iii = 5
A. 39.8 C. &x,. *
y = t -1 B. 29.8 D. 6~
32. What is the equivalent expression for
dy = 1 c. 49.8 2 2 sin 2x?
D. 19.8 * Y = .fXi + x· = x + x· A . % sin x cos x
X = 4t + 6 y' = 1 + (-2)x'3 B. 2sin z •
dx = 4 s = 20t + ~ y" = (-2)(-3)x.. = 6x'4 sec .x
t +1 C. -2sin x
dy - ; 28. Where does the point of inflection D. -2 sin X cos X
slope, -;; - 4 5
~ ·= 20 .!...- - 3
dt (t+l)Z occur in the equation: x - 3x.
A. first quaprant Sin 2x = 2 sin x cos x
21 . In triangle ABC, AB = 8 m and BC =
~ = 20 ---2 = 19.8m/min
5 B. origin "
20 m. One possible dimension of CA is: where: cos x
1
= -sec:c
dt (4+1) C . 'third quadrant -
A. 13 *
D. y-axis
B. 9 25. A cone was formed by rolling a thin
c. 11 Sin2x = ~
sheet of metal in the form of a sector of a y = ~-3x sec z
D. 7 circle 72 em in diameter with a central y' = 3x2- 3
The possible solution of th is triangle is by
angle of 150 degrees. Find the volume of
the cone in cc.
y• =
6x = 0 33. Given the function f(x) = x to the 3
rd power - 12x + 1, find the value of the
X = 0, Y = 0
using the theorem. The difference of the A. 7733 first derivative at x = 1.
two sides is less than the third side. B. 7744 A. 9
Therefore, point of inflection occur at the
C. 7722 origin. B. 12
BC - AB < CA D. 7711 * c. -9 .
20 - 8 = 12 < CA 29. The value of e raise to the negative 5 D. 3
Therefore, CA should be greater than 12. S = R9
z =-
72 ( lSOx-
")
180
power is equal to:
f(x) = ~ -12x + 1
S = 94.24 em A. 0.0067379 *
The answer is 13.
B. 0.0063779 f(x)' = 3x?- - 12
c. 148.4130 f(x)' = 3(1)2 - 12 = -9
22. Determine the sum of the infinite S=C=2rrr
94.24 = 2m D. 0.0076397
geometric series of 1 -1/5 + 1/25.
A. 5/7 r = 15 em
B. 4/7 REFRESHER MANUAL 2ND Edition by JAS TORDILLO
REFRESHER MANUAL 2ND Edition by JAS TORDILLO
19-10 I DAY 19- SOLUTION DAY 19 - SOLUTION 119-11
34. Find the partial derivative with respect 38. Determine the center of the curve : ENGINEERING MATHEMATICS
to x of the fu nction: 2i + 2-j- 8x + 4y- 40 = 0.
2t" - t + 3. What is the acceleration of
150l'!y- 10xy + 3y + 5 A. (2, 1)
the object at t = 2?
A. 300xy - 1 Oy * B. (2, -1) *
B. 150x"- 10x A. 52 *
C. (-2, -1)
c. 150y- 10y B. 64 46. A angle greater than a straight angle
D. (-2. 1)
C. 56 and less than two straight angles are
D. 300xy- 10x + 3
D. 66 called:
2x2 + 2yl- 8x + 4y - 40 = 0 A. right angle
Z = 150l'!y- 10xy + 3y + 5 x22 + -1 - 4x + 2y
2
-20 = 0 X = 4t3 + 2f - t + 3 B. reflex angle •
x -4x + 4 + y + 2y + 1 = 20 + 4 + 1
2 2 2 x' = 12f + 4t - 1 C. obtuse angle
!!!. = 150(2)xy - lOy (X - 2) + (y + 1) ::: 5
x• = 24(2) + 4 = 52 D. acute angle
dx
r = 5, C(2, -1)
!!!.
dx
= 300xy - lOy
43. The velocity of a particle at time t is: 47. A line segment joining two points on
39. Factor the expression: 2x2 + 12x +
v(t) = 12t4 + 7/t. What total distance is a circle is called:
35. As x increases uniformly at the rate
of 0.001 ft/sec, at w hat rate is the
16 as completely as possible.
A. 2(x- 2)(x- 4)
=
travelled between t 0.2 and t 0.3? = A. arc
A. 2.84 * B. sector
express ion (1 + x)2 increasing when x = B. 2(x + 2)(x- 4)
4ft?
B. 3.85 C . tangent
C. 2(x- 2)(X + 4)
A. 0.01 * D. 2(x + 4)(X + 2) *
c. 3.25 D. chord*
D. 2.48
B. 0.001
48. All circles having the same center
C. 0. 10
D. 1.01
2l'! + 12x + 16 = 2(i + 6x + 8)
= 2(x + 4)(x + 2)
v(t) = 12t4 + 7/t but with unequal radii are called:
A. encircle
f~
03
y = (1 + x) 2 40. Body or space bounded by surface de
= f.0.2 (12t4 + Z.)
t
dt B. concyclic
C. tangent circles
every point of which is equidistant from a D. concentric circles •
ctr
~ = 2(1 + x)dt' point w ithin. S = m • + 7 ln tl&~

*
de

= 2(1 + 4)(0.001) ::; 0.01


A. circle
B. ellipsoid
C. spheroid
D. sphere*
s
s

= 2.8428

44. The value of (4 to the 2nd power)


49. The esc of 960 deg is equal to:
A. 1
B - -13
. 2
36. What is the val ue of each interior
square root is equal to:
c. y,
angle of a regular pentagon? 41. The slope of the line is Y, . The -2-/3 *
~ A. 2 0• • 3
A. s slope of the second line is -2/3. Both
B. ~
B. i lines intercept at the point (3, 1). W hat is
C . 16 CSC 960° = X
C. i the acute angle between the lines?
D. 4 01 1
A. 60.3° * - -: X
s *
D. !!! s in960
B. 75.4°
C. 67.2° (4 to the 2nd power) square root
2-/3
n = 5 (regular pentagon) D. 90.0° =.fil =4 Using Calculator: x = -1.1547 = 3

e = ~5 mz - ml 45. What is the simplified equivalent 50. A quadrilateral with two and only two
Tan 9 = t+m 2 m,
expression of (cotl O)(sin20) + 1/csc29. sides of which are parallel is called: ·
37. How many cubic meters is 150 A. ' 0 A . parallelogram
gallons of liquid?
m1 = -2/3 , m2 = Y, B. cos 9 B. quadrilateral
A. 0.58761 1 -2 C. sin2 9 C. trapezoid *
B. 0.57871 2-s D. 1 * ' D. rhombus
c. 0.56781 * Tan 9 = t+{i)(~)
D. 0.56871 (cof 9)(sin2 8) + _ 1_ 51 . Compute for the torsional deflection
8 = 60.25 deg csc'B in degrees Of a 100 mm diameter, ·1 .75
150 al x 3 '
g
7854
gal
li x~
1000 Ll
= 0.56781 m 3
42. The position of an obj ect as a
=
28
(c~s2
sm 9
)(sin28) + sinz 8 meter long shaft subjected to a twist
moment of 5.5 x 106 N-mm. The
function of time is described by x = 4e +
REFRESHER MANUAL 2ND Edition by JAS TORDILLO
REFRESHER MANUAL 2ND Edition by JAS TORDILLO
19-12 I DAY 19 - SOLUTION DAY 19 - SOLUTION 119- 13
torsional modulus of elasticity is 85000 elongation not to exceed 3 mm, find the ENGINEERING MATHEMATICS
2
N/mm . allowable load in kN.
60. A 16 ft horizontal beam is subjected
A. 0.22 deg A. 604.5 kN 3 x 101= 6400 to a load of 500 lbs located at its. center.
B. 0.33 deg B. 504.5 kN strain
C. 0.55 deg Strain= 2.133 x 10""' in/in The dimension of the beam is 3 x 5
C. 404.5 kN
inches respectively and its unit weight is
D. 0.66 deg* D. 304.5 kN *
57. A wire of length 1.90 m has a 100 lb/ft. Find its flexural stress in psi.
e= TL A. 2992 psi
.IG percentage strain of 0.032% when loaded
y = .!:.!:_ B. 3255 psi
AE with tensile force. Determine the
C. 4992 psi
5.5xJ0 6 N-mm\'1750 mm) Since E is not given, use E = 30 x 106 psi extension of the wire in mm.
( }- 180 deg A. 0.108 mm D. 5982 psi*
= x----- for steel
4 4 B. 0.408 mm
11 (IOo) mm (s5,000 N ) 11 rad
32 mm2 6
E = 30 x 10 psi= 206,786 Mpa C. 0.608 mm *
8 = 0.66 deg D. 1.208 mm
Substitute:
52. What diameter of bolt is required if Strain= l Load } ·- I 1 ! ~
0.003 = F(l.O)
the bolt is subjected to a tensile load of L
~ (0.025)2(206,786xro 6 )
24,000 lb and a torque (for tightening) of
7,000 in-lb and maximum normal stress
is not to exceed 30,000 psi?
A. o/. inch
F = 304,517.5 N = 304.5 kN
0.032 = _Y_
100 1.90
y = 6.08 x 10-4m= 0.608 mm
•. T.,."
55. Find the section modulus in cubic
B. 1 inch*
C. 1 ~inch
inch of a shaft with diameter 2.25 inches. 58. Determine the polar section modulus,
Zp (in3 ) of a shaft delivering 15 Hp at 300
!· .......... ' .. ;.; 'l . . . l
D. 2 inche A.1.11 * rpm. The diameter of the shaft is 2.0 in
F B. 2.22 and allowable shear stress of 4500 psi.
s, = - c. 3.33 Use a factor of safety of 5.
A
D. 4 .44 A. 1.57 in 3 * Moment ~
30 000 = 24,000 B. 1.75 in3
• 1! d2 3 3
Z' = 11D = 11(2.25) = l.ll C. 1.95 in3
4
32 32 D. 2.47 in
3
d = 1.0 inch bolt
rrD 3
56. What is the ·decrease in lateral Zp=-
53 A body weighing 1,019.36 kg was put 16
dimension due to an axial tensile force of
on top of a 3205 N/cm helical spring.
What is the deformation of the spring in
40,000 lbs in a bar 2.5 inches on each
side and 5 feet long? The material is
Zp' = ~t(l)J =I .57 in3 R1 = R2 -- 100(16)+ 500
2
= 1050 lb
em due to the body on top? 16
A. 2.1 em = 7
steel (E 3.0 x 10 psi, J.l = 0.3).
80
A. 5.133 X 10-4 in/in 59. Find the polar section modulus of a
A1 = °C8l + ZSO(B) = 5200 ft-lb
B.3.1cm* 2
B. 4.133 X 10'4 in/in hollow shaft with an outside diameter of 5
C. 4.1 em
C. 3.133 x 10·~ in/in inches and inside diameter of 3 inches. A2 = -5200 ft-lb
D. 5.1 em
D. 2.133 x 10-4 in/in"' A 51 .36 in3
3 800 8
F = 1019.36 kg = 10,000 N B. 41.36 in M = moment =
2
( ) + ZSO(B)
Force C. 31.36 in3
Spring constant, k = F Stress D. 21.36 in3 *
y Area M = 5,200 ft-lb = 62,400 in-lb

y =
F
k=
10,000
3,205
40,000
= 2.5(2.5)
Zp = 32
11
(o~ -D~)
1
~ 32
11
- (s4 -34) 1 c =distance from the farthest-fiber to the
~ 21.36 i 3neutral axis
y = 3. 12 em = 6400 psi -,o/
/2
7i c=-
h 5
=- = 2.5 In
.
. . stress
54. A steel rod 25 mm in diameter and 1 Modulus of elast1c1ty, E = ---.- 2 2
1000 mm long has an allowable stram

REFRESHER MANUAL 2ND Edition by JAS TORDILLO


REFRESHER MANUAL 2N° Edition by JAS TORDILLO
19 -14 J DAY 19- SOLUTION DAY 20 - EXAM 120- 1
ENGINEERING MATHEMATHICS
I = moment of 1nert1a
bh3 7. In a pile of logs, each layer contains
=- for a rectangular cross section DAY 20- EXAM one or more log than the layer above and
12
top contains just one log. If there are 105
3 1. The reciprocal of 5 is:
1= 3(S) = 31.25 ln 4 logs in the pile, how many layers are
12 A. 0.1 there?
B. 0.4 A. 10
SF = Me = 62.400(2.5) =4992 lb/in2 c. 0.2 B. 14
I 31.25 D. 0.5 c. 12
D. 15
2. The digits of measurement which a
scientist reads and estimates on a scale 8. A and B working together can finish
are called: painting a house in six days. A, working
A. accuracy alone, can finish it in five days less than
B. precision B. How long will it take B to finish the
C. significant figures work alone?
D. experimentation A. 10 days
B. 12 days
3. Simplify: am/n C. 11 days
A. 'V(iiii D. 15 days
B. am/n
c. am•n
D. am•n 9. simplify:
A. . a .
('var
B. a 1tn
4 . Factor the expression '1? + 7x + 12 as C. an/1
completely as possible:
D. a/n
A. (x + 4)(x + 2)
B. (x + 4)(X + 3)
C. (x + 3)(x + 2)
10. Solve for x: 2'1?- f- 14 =0 but
y=x-1 .
D. (X + 2)(x + 5)
=
A. x -5 and x = 3
5. Factor the· expression:
=
B. x -6 and x 2 =
:i!>(l- 2'1? - 12x.
=
C . x = 5 and x -3
D. X = 6 and X = -2
A. 2x(x - 3)(x + 2)
B. 2x(x + 3)(x- 2)
11 . A manufacturing firm maintains one
C. 2x(x- 4).(x - 2)
.; product assembly line to produce signal
D. 2x(x + 3)(x- 4)
generators. Weekly demand for the
generator is 35 units. The line operates
6. If A can do the job in x days and B in y
for 7 hours per day, 5 days per week.
days, how long will they finish the job
What is the maximum production time per
working together?
unit in hours required of the line to meet
A . (x + y)/xy days
the demand?
B. x + y days
A . 1 hour
C. (xy)/(x + y) days
B. 3 hours
D. (x + y)/2 days
C. 2 hours
D. 4 hours

12. Which number has three significant


digits?
A. 0.0011
REFRESHER MANUAL 2nd Edition by JAS TORDILLO
REFRESHER MANUAL 2N° Edition by JAS TORDILLO
20- 2 DAY 20 - EXAM DAY. 20 - EXAM 120 -3
B. 0.01111 20. Factor the expression
, ENGINEERING MATHEMATHICS
c. 1.1111 (1- a to the 8 th power) as completely as
D. 0.0111 possible. A. 35 per sec, how many potatoes he put into
A ( 1+a to the 4th power) x B. 45 · the basket in 76 seconds?
13. An urn contains 6 black balls and 4 (1 +a2 )(1+a)(1 -a) C. 40 A. 15 potatoes
white balls. What is the probability of 8, (1+a to the 4th power) x D. 50 B. 21 potatoes
getting 1 black and 1 white ball in two . (1- a to the 4th power) C. 19 potatoes
consecutive draws from the urn? C. (1+ a to the 4th power) x 27. A certain two digit number is four D. 25 potatoes
A. 0.25 (1 + a2 )(1- a2 ) times the sum of its digits, if 36 be added
B. 0.53 D. (1 + a2 )(1 - a 2) x to the number, the digits will be reversed. 32. A man bought 23 chickens for
c. 0.33 (1 -a to the 4th power) Find the number. P29.00. The cocks cost P3.00 each, the
D. 0.76 A. 48 hens P1 .50 each, and the chicks at P0.50
21. In a quadratic equation, if the quantity B. 74 each. How many cocks, hens and chicks
14. If A to the '!. power equals 18.8, A is under the radical is equal to zero, the c. 64 did he buy?
equal to: roots are: D. 84 A 3, 4, 13
A. 35 A. imaginary numbers B. 4, 6, 12
B. 50 B. complex and unequal 28. A and B working together, can finish c. 5, 5, 13
C. 45 C. real and unequal a piece of work in 6 days. A working D. 5, 7, 12
D. 60 D. real and equal alone can finish it in 10 days. How long
w ill it take B to do the work alone? 33. At what time between 3 PM and 4 PM
15. The value of (3 to 2.5 power) square 22. Solve for x and y: 3x - y - 6 = o· A. 15 days will the hands of the clock be opposite to
is equal to: 9x- y -12 = 0. B. 10 days each other?
A. 730 A. X= 3; y = 1 C. 12 days A. 3:15.PM
B. 150 B x=1;y=-3 D. 8 days B. 3:45PM
c. 243 C. X= 2; y = 2 C. 3:35PM
D. 93 D. x _=4;y=2 29. How much of a 40% solution of D. 3:49PM
alcohol should be mixed to an 80%
16. 10 to the negative 12th power is the 23. Factor the expression (x4 - y4) as solution to give 150 liters of a 50% 34. A and B together weigh 242 pounds.
va lue of the prefix: completely as possible: solution? They balance when A is seated 5 feet
A. micro \ A. (X + y)~~ + 2X¥, + y2) A. 127.51iters from the fulcrum on one side of a lever
B. tera B. (x2
+ y )(~ - y ) B. 112.5 liters and B is seated 6 feet from the fulcrum
C. nano C. (lt + f')(x + y)(x- Yi C . 122.5 1iters on the other side. Find the weight of A.
D. pico D. (1 + ~)(1 + y)(1 - y ) D. 37.5 liters A. 821bs ·
B. 110 lbs
17. The product of (a - b)2: 24. What is the sum of the following finite 30. Two planes fly toward each other C. 100 lbs
2 2
A. a + b sequence of terms? from points 2500 km apart, at the rates of D. 132 lbs
B. a2 - 2ab + b2 18, 25, 32, 39, ........ .. ......... , 67. 600 kph and 400 kph . When will they
C. a2 - b 2 A 234 J
meet if they start at the same instant? 35. A group of students rented a
D. a2 + 2ab + b2 B. 181 A. 2 hr convention hall for P240.00. If there
C. 213 B. 3 hr were 4 students less, the share of each
18. The arithmetic mean of 2 and 5 is: D. 340 C. 2 ~ hr would ·have been P2.00 more. How
A. 3.5 D. 1 hr many students were in the group?
B. 7 25. Factor the expression: 16- 10x + ~­ A. 40
C. 3.16 A (8 + x)(2- x) 31. Twenty five potatoes are placed on B. 24
D. 10 B. (X-8)(x-2) the ground 4ft apart in a straight row. In c. 30
C. (8 - X)(2 + x) line with potatoes and 10ft from the first D. 18
19. The geometric mean of 3 and 12 is: D. (X + B)(x + 2) one is a basket. A runner starting from
A. 18 the basket picks up the potatoes and 36. Resolve into Partial Fractions:
B. 7.5 26. A father is 24 years older than his carries them one at a time to the basket. (8x- 8) I (x3 -2~- 8x).
C. 15 son. In 8 years he will be twice as old as If he runs at an average rate of 5 yards A. 1/x + 1/(x- 4)- 2/(x + 2}
D. 6 his son. What is ·the present age of the B. 2/x + 1/(x- 4) + 1/(x + 2)
father?
REFRESHER MANUAL 2nd Edition by JAS TORDILLO
REFRESHER MANUAl2"d Edition by JAS TORDillO
20-4 DAY 20 - EXAM DAY 20 - EXAM 120 -5
C. 1/x- 1/(x- 4) + 2/(x + 2) C: 25X'Y ENGINEERING MATHEMATHICS
D. 25x l
D. 2/x- 1/(x- 4)- 1/(x + 2) 4

B. (3 + 4m){2 - m~) C. sinking fund


C. (3- 4m)(2 - m J
2
37. From a box containing 8 red balls, 10 43. Find the last term of the progression: D. sum of the years digit
white balls and 12 blue balls, one ball is 10, -2, -14, ... ... to 17th term. D. (3 -4m)(2 + m)
drawn at random. Determine the A. -182 56. What do you call an increase in the
probability that is white: B. -1462 50. Which of the following is a prime value of a capital asset?
A. Y. c. -220 number? A. capital expenditure
B. y. D. -1530 A. 100 B. capital stock
c. 1/3 B. 101 C. capital gain
D. 215 44. Find the sum of the integers between c. 99 D. profit
2 and 100 which are divisible by 3. D. 105
38. From a box containing 8 red balls, 10 A. 970 57. What is the value which a
white balls and 12 blue balls, one ball is B. 1683 51. Funds supplied by others on which a disinterested third party, different from
drawn at random. Determine the C. 1100 fixed rate of interest must be paid and the the buyer and seller, will determine in
probability that is white or blue: D. 1985 debt be repaid at a specific place and order to establish price acceptable to
A. 0.933 time . . both parties?
B. 0.733 45. Find the last term of the progression: A. Borrowed capital A. franchise value
C. 0.833 2, 6, 18, ................ 6'lh terms. B. Investment B. goodwill value
D. 0.333 A. 335 C. Salvage value C. fair value
B. 555 D. Interest D. market value
39. From a box containing 8 red balls, 10 C. 486
white balls and 12 blue ball. If two balls D. 728 52. What is an annuity? 58. Liquid assets such as cash and other
is drawn at random, determine the A. A series of equal payments at equal assets that can be converted quickly into
probability that one is red and the other is 46. Find the sum of the roots of: 2~ + time periods. cash, such as accounts receivables and
blue: 5x + 5 = 0. B. The cost of manufacturing a product. merchandize are called:
A. 0.110 A. 5/2 C. An investment that yields an equal A. land and buildings
B. 0.330 B. -5/2 amount of interest each year. B. current assets
c. 0.220 C. 2/5 D. The overhead cost per unit of C. total assets
D. 0.440 D. -2/5 production. D. fixed assets

40. From a box containing .8 red balls, 10 47. · Find •the value of x in the equation: 53. Estimated value at the end of the 59. What is the recorded current value of
white balls and 12 blue balls. If one ball is (5x+6) 112 = x+2 useful life: an asset?
drawn at random, determine the A. 2 and 1 A. salvage value A. salvage value
probability that is not blue: B. 2 and -1 B. economic life B. present worth
A. 0.60 C. -2 and -3 C. compounded annually C. book value
B. 0.45 D. -2 and 3 D. balance sheet D. scrap value
C. 0.50
D. 0.35 48. The force F of attraction between two 54. Consists of the actual counting or 60. What is the method of depreciation
electric charges varies inversely as the determination of the actual quantity of the where a fixed sum of money is regularly
41 . Determine the 7 th term of the square of the distance d between them. materials on hands as of a given date: deposited at compound interest in a real
expansion (2y - 11)11 If the force is 20 grams when they are A. physical inventory or imaginary fund in 'order to accumulate
A. 14,784 xV 150 mm apart, find the force when they B. technological assessment an amount equal to the depreciation of an
B. 11,784xV are 75 mm apart. C. material update asset at the end of the asset's estimated
C. 985 x6 i A. 30 grams D. material count life?
D. 765 x6 y7 B. 60 grams A. sinking fund method
C. 45 grams 55. A depreciation model where the B. SYD method
42. Find the term involving l
in the . D. 80 grams value of an asset decreases at a C. straight line method
expansion of (x + y( decreasing rate: D. declining balance
A. 35x4 y3 49. Factor the given expression: A. straight line
B. 35xV 6-3m2 - am· + 4m3 . B. declining balance
A. (3 + 4m)(2 + m2 )
REFRESHER MANUAL 2nd Edition by JAS TORDILLO
REFRESHER MANUAL 2"d Edition by JAS TORDILLO
CAY 20 - SOLUTION 120- 7
20-6 DAY 20- SOLUTION ENGINEERING MATHEMATHICS
DAY 20 - SOLUTION Let D = no. of days A and 8 cab finish (B- 15)(8- 2) = 0
(c-1
1. The reciprocal of 5 is:
the job working together 8 = 15, 8 2 =
DAY 20- ANSWER KEY A. 0.1 1 1 1 y+x Therefore, worker 8 can finished the
1. c 16. 0 31.1\ 46.8 -D = -X +-=-
B. 0.4 y xy
work alone in 15 days.
2. C 17. B 32.( 47.8
c. 0.2 *
3. A 18. A 33. 0 48 0
34. 0 49. c D. 0.5 D = .2..
4. B 19.0 x+y 9. Simplify: ('Vri'/
5. A 20. A 35. B 50. B
A. a*
6. c 21. 0 36. A 51. A Reciprocal of 5 = ~5 = 0.20 7. In a pile of logs, each layer contains B. a11n
7. 8 22. B 37. C 52. A
8. 0 23. c 38. B 53. A
one or more log than the layer above ·and C. an/1
9. A 24.0 39. C 54. A 2. T he digits of measurement which a top contains just one log. lfthere are 105 D. a/n
10. A 25.8 40. A 55.8 scientist reads and estimates on a scale logs in the pile, how many layers are
11. A 26. C 41. A 56. C are called: there? n 1

12. D 27. A 42.8 57. ( A. accuracy A. 10 ('\Ia) = (a")" = a


13. B 28. A 43. A 58. B B. precision B. 14 *
14. B 29. B 44. B 59. C C. significant figures * c. 12 10. Solve for x : 2x,l - i - 14 = 0 but
15. c 30. c 45. C 60. A y = x -1.
1 D. experimentation D. 15
=
A. x -5 and x 3 * =
3. Simplify: amln a = 1 d = 1 S = 105 B. X = -6 and X = 2
C. x = 5 and x = -3
A. VQ.lii*
B. am/n S = ~2 [Za + (n- l ) d] D. x =6 and x = -2
C. am•n
D. amxn 2x
2
-i - 14 = 0
105
.
= ~2 [2(1) + (n- 1)(1) ] 2-x,l-(x -1) 2 -1 4 = 0
4. Factor the expression x,2 + 7x + 12 as x,2 + 2x - 15 = o
210 = 2n+n -n
2
completely as possible: (X + 5)(x - 3) = 0
A. (X+ 4)(x + 2) n2 +n-210 =0 X = -5, X =3
B. (x + 4)(x + 3) * (n- 14)(n + 15) = 0
n = 14 and n = - 15 11 . A manufacturing firm maintains one
C. (x + 3)(x + 2)
product assembly line to produce signal
D.' (X+ 2)(x + 5)
8. A and B working together can finish generators. Weekly demand for the
' generator is 35 units. The line operates
Factoring by trial and error method: painting a house in six days. A, working
x,2 + 7x + 12 = (x + 4)((x + 3) alone, can finish it in five days less than for 7 hours per day, 5 days per week.
B. How long will it take B to finish the What is the maximum production time per
unit in hours reqtJired of the line to meet
5. Factor the expression:
2x~- 2x2 - 12x.
... work alone?
A. 10 days the demand?
A. 2x(x- 3)(x + 2) * B. 12 days A. 1 hour •
... B. 2x(x + 3)(x- 2) C. 11 days B. 3 hours
C. 2x(x- 4)(x- 2) D. 15 days* C. 2 hours
D. 4 hours
D. 2x(x + 3)(x- 4)
D = 6 days A =8 - 5
Production time
2x3 - 2-x,l- 1ix = 2x(x_2 - x - 6) oM w eek 5 days 7 hours
= 2x(x - 3)(x + 2) .!. = .!. +.!.
D A 8
= ---x--x---
35 units week day
=1 hour/unit
6. If A can do the job in x days and B in ~=-1-+.!_
days, how long will they finish the jo 6 8 -5 8
12. Which number has three significant
working together? 2 digits?
A. (x + y)/xy days B - 178 + 30 = 0
A. 0.0011
B. x + y days B. 0.0111 1
c. (xy)/(x + y) days * REFRESHER MANUAL 2nd Edition by JAS TORDILLO
D. (x + y)/2 days
REFRESHER MANUAL 2"d Edition by JAS TORDILLO
20 -8 DAY 20 - SOLUTION DAY 20 - SOLUTION 120- 9
ENGINEERING MATHEMATHICS
c. 1.1111 C. nano
D. 0.0111 * D. pico * 22. Solve for x and y: 3x - y - 6 = 0; 16- 10x +X" = X"- 10x + 16
2 9x- y -12 = 0. = (x - 8)(x - 2)
A. 2 significant digits 17. The J?roduct of (a- b) :
A. X = 3; y = 1
B. 4 significant digits A. a2 + b2 . B x=1;y=-3* 26. A father is 24 years older than his
C. 5 significant digits B. a2 - 2ab + b 2 •
C . X= 2; y = 2 son. In 8 years he will be twice as old as
D. 3 significant digits C. a2 - b2 D. X = 4; y = 2 his son. What is the present age of the
D. a2 + 2ab + b2
father?
13. An urn contains 6 black balls and 4 3x - y - 6 = 0 + y = 3x - 6 A. 35
white balls. What is the probability of 18. The arithmetic mean of 2 and 5 is: 9x - y - 12 = 0 B. 45
getting 1 black and 1 white ball in two A. 3.5 * 9x - 3x + 6 - 12 = 0 c. 40 ..
consecutive draws from the urn? B. 7 x = 1,y=-3 D. 50
A. 0.25 C. 3.16
B. 0.53 * D. 10 23. Factor the expression (x4 ll as
- Let F = present age of the father
c. 0.33 completely as possible:
D. 0.76 Arithmetic mean = z;s= 3.5 A. (x + y)l-1- + 2xy + /)
S =
F+8 =
present age of the son
age of father in 8 years
B. (X2 + y2)(X2 - y") , S+8 = age of son in 8 years
First draw black ball and second draw C. (X2 + y2)(x + y)(X -~f) •
19. The geometric mean of 3 and 12 is:
white:
P1 =~X~=~
A. 18 . D. (1 + x")(1 + y)(1 - y") F = S + 24 + S = F - 24
10 9 90 B. 7.5
c. 15 (x4 - l> = (x2 + /)(x2 - y2) F+8 =
2(S + 8)
First draw white ball and second draw D. s• = (x2 + y2 )(x + y)(x- y) F + 8 = 2[(F -24) + 8]
P2=~x~ =~ F = 40
10 9 90 24. What is the sum of the following finite
Geometric mean = '..}3(12) = 6
sequence of terms? 27. A certain two digit number is four
p = p1 + p2
20. Factor the expression 18, 25, 32, 39, ....... ............. 67. times the sum of its digits, if 36 be added
(1- a to the B th power) as completely as A. 234 to the number, the digits will be reversed.
p = ~+~=~ = 0.5333 B. 181 Find the number.
90 90 90 possible.
A. (1+a to the 4th power) x c. 213 A. 48 *
14. If A to the o/. power equals 18.8, A is · (1 +a2 )(1+a)(1 -a) • D. 340 * B. 74
equal to: B. · ( 1+a to the 4th power) x C. 64
A. 35 (1- a to the 4th power) d = 25-18 = 7; a = 18; L = 67 D. 84
B. 50* C. (1+ a to the 4th power) x
C. 45 (1 + a2 )(1- a2 ) L = a + (n - 1)d Let x = units digits
D. 60 D. (1 + a2)(1- a2 ) x 67 = 18+(n-1)7 y =tens digits
(1 -a to the 4th power) n=8 x+y = sum of the digits
A314 = 18.8 1Oy + x = represents the number
413
A = 18.8 = 49.99 (1 -a to the 8th power) S = !!.[a+L]
2
1Ox + y = reversed number
= (1 - a8)
15. The value of (3 to 2.5 power) square = (1 - a4 ) ( 1 + a4 ) 10y +X = 4(x + y) Eq. 1
is equal to:
s = .'![18 + 67]
10y +X+ 36 = 10x + yEq. 2
= (1 + a4)(1 - a2 )(1 + a2 ) 2
A. 730 =- (1 + a4 )(1 + a2)(1 + a)(1 -a) 9x-9y = 36
B. 150 s = 340 x- y = 4
c. 243. 21. In a quadratic equation, if the quantity X = y +4 Eq.3
D. 93 under the radical is equal to zero, the 25. Factor the expression: 16 - 1Ox+~-
A. (8 + x)(2 - x) Substitute to Eq. 1
roots are:
(32 5) 2 = 35 = 243 A. imaginary numbers B. (x - S)(x - 2) * 10y+y+4 = 4(y+4)+4y
B. complex and unequal C. (8- x)(2 + X) 11y+4 = 8y+16
16. 10 to the negative 12 th power is the C. real and unequal D. (x + 8)(x + 2) 3y = 12
value of the prefix:· D. real and equal * y =4
A. micro
B. tera REFRESHER MANUAL 2nd Edition by JAS TORDILLO
REFRESHER MANUAL2"d Edition by JAS TORDillO
DAY 20 - SOLUTION. I20- 11
20-10 DAY 20 - SOLUTION ENGINEERING MATHEMATHICS
x=4+4=S S = s,
+ Sz
2500 = 600t + 400t By trial and Error method: xy = 240
The number is 4(10) + S = 4S X = 5, y = 5, Z = 13 (x- 4)(y + 2) = 240
t = 2soo = 2.5 hours x2 - 4x - 4SO = 0
1000
2S. A and B working together, can finish 33. At what time between 3 PM and 4 PM (x- 24)(x + 20) = 0
a piece of work in 6 days. A working 31 . Twenty five potatoes are placed on will the hands of the clock be opposite to X =24
alone can finish 1t in 10 days. How long the ground 4 ft apart in a straight row. In each other?
will it take B to do the work alone? line with potatoes and 10 ft from the first A. 3:15PM 36. Resolve into Partial Fractions:
A. 15 days • one is a basket. A runner starting from B. 3:45PM (8x- 8) I (x3 - 2x2 - Sx).
B. 10 days the basket picks up the potatoes and C. 3:35PM A. 1/x + 1/(x- 4)- 2/(x + 2) *
C. 12 days carries them one at a time to· the basket. D. 3:49PM* B. 2/x + 1/(x- 4) + 1/(x + 2)
D S days If he runs at an average rate of 5 yards C. 1/x- 1/(x - 4) + 2/(x + 2)
per sec, how many potatoes he put into Let x = no. of minutes traveled by . the D. ,2/x- 1/(x- 4)- 1/(x + 2)
;,=;,+,;. the basket in 76 seconds? minute hand
D A 8 ax-a _ ax-a _ ax-a
A. 15 potatoes • x 3 -2x 2 -8x - x(x2 -2x-8) - x(x-+)(x+2)
B. 21 potatoes X = .!... + 30 + 15
;.=..:.+.: 12 . •
6 10 8 C. 19 potatoes ~= ~+~+..£__
D. 25 potatoes x = 49.09 minutes x(x-4)(x+Z) x x-4 x+2
B = 15 days
Let n = no. of potatoes Time: 3:49.09 PM Sx- 8 = A(x2 - 2x- S) + B(x2 + 2x) +
29. How much of a 40% solution of
Velocity = 5 yards per sec = 15 fUsee C(x2 -4x)
alcohol should be mixed to an SO% Total distance traveled in 76 seconds, 34. A and B together weigh 242 pounds. Sx - 8 = Ax2 - A(2x) -A(B) + Bx2 +
solution to give 150 liters of a 50% s = 15(76) = 1140 ft . B(2x) + Cx2- C(4x)
They balance when A is seated 5 feet
solution? 8x- 8 = x2 (A + B + C) +
from the fulcrum on one side of a lever
A. 127.51iters a, = 20ft; a2 = 2S ft; a3 = 36ft; ... x(-2A + 28- 4C)- SA
and B is seated 6 feet from the fulcrum
B. 112.5 liters* d = 2S -20 = s on the other side. Find the weight of A. -SA = -8
C. 122.5 liters A. S2 1bs A = 1
D. 37 .5 liters
S = ~ [2a + (n - 1)d] B. 110 lbs
Let x = liters of 40% solution
2
C. 100 1bs =
8 -2A + 2B - 4C
D. 132 lbs • S = -2(1) + 2B -4C
y = liters of SO% solution 1140 = ~ [2(20)
2
+ (n- 1)(8)] 2B - 4C = 10
A + B = 242 Eq. 1 B - 2C = 5 Eq. 1
X + y = 150 o+ y = 150 - X 2
n + 4n - 2S5 = 0 A(S). = B(6) Eq. 2
(n-15)(n+19) = 0 A+B+C = 0
0.40x + O.SOy = 150(0.50) n = 15; n = -19 (absurd) 1+B+C=O
Solving for A:
0.40x + O.S0(150- x) = 150(0.05) B = -C-1 Eq. 2
A = 132 1bs
x = 112.5 liters of 40% solution alcohol 32. A man bought 23 chickens for -C-1 -2C = 5
30. Two planes fly toward each other
P29.00. ·The cocks cost P3.00 each, the 35. A group of students rented a c = -2
hens P1 .50 each, and the chicks at P0.50 convention hall for P240.00. If there B = -(-2)- 1 = 1
from points 2500 km apart, at the rates of each. How many cocks, hens and chicl<s were 4 students less, the share of each
600 kph and 400 kph. When will they did he buy? Sx- 8 1 1 2
would have been P2.00 more. How . = -+-----
meet if they start at the same instant? A. 3, 4, 13 many students were in the group? x(x-4)(x+2) x x-4 x+2
A. 2 hr B. 4, 6, 12 A. 40
B. 3 hr c. 5, 5,13 * B. 24* 37.. From a box containing 8 red balls, 10
c. 2 Y. hr * D. 5, 7, 12 c. 30 white balls and 12 blue balls, one ball is
D. 1 hr drawn at random. Determine · the
D. 1S
Letx = no. ofcocks probability that is white:
Distance = Speed x time y = no. ofhens
s, = distance traveled by plane A Let x = no. of students in the group A.Y.
z = no. of chicks y = share per student B. Y2
S 2 = distance traveled by plane B
S = total distance traveled by plane A X+ y + z = 23 . Eq. 1
and B 3x + 1.5y + 0.50z = 29 Eq. 2 REFRESHER MANUAL 2nd Edition by JAS TORDILLO
REFRESHER MANUAL 2"d Edition by JAS TORDILLO
DAY 20 - SOLUTION 120 - 13
20-12 DAY 20- SOLUTION ENGINEERING MATHEMATHICS
c. 1/3 * A. 14,784 x"y" *
D. 2/5 B. 11.784
C. 985 x6 l
x:v B. 1683 * 48. The force F of attraction between two
6 7 C. 1100 electric charges varies inversely as the
p = ~30 = 1/3 D. 765 x y D. 1985 square of the distance d between them.
If the force is 20 grams when they are
The exponent of the given binomials is a1 = 3, d = 3, L = 99 150 mm apart, find the force when they
38. From a box containing 8 red balls. 10
n = 11. Only choices A and B has an are 75 mm apart.
white balls and 12 blue balls, one ball is
exponent of 11 , therefore the answer will L = a1 + (n -1)d A. 30 grams
drawn at random. Determine the
either A orB only. 99 = 3 + ( n -1 )3 B. 60 grams
probability that is white or blue:
A. 0.933 n = 33 C . 45 grams
r th term D. 80 grams*
B. 0.733 * = (n)(n-l)(n-2) .....(n-r+2) x<n-r+1)y(r-t)
c. 0.833 (r - 1)! S = iCa 1 + L) 1
D. 0.333 r=11 , n=11 F ad>
p = 10+12 = 0.733 r th term
s = E.(3
2
+ 99) = 1683
1
30
= (11)(10)(9)(8)(7)(6) (Zx)ll-7+1 1-1
F = kdi'
0-1~ y 45. Find the last term of the progression:
39. From a box containing 8 red balls, 10
white balls and 12 blue ball. If two balls
r .th term = 14,784 x
5
l 2, 6, 18, ................ 6thterms. k = Fd 2
A. 335
is drawn at random , determine the 42. Find the term involving y4 in the B. 555 F1d12 = Fzdl
probability that one is red and the other is expansion of (x + y}'. c. 486" 20(150)2 = F2(75)2
blue: A. 35x4 l D. 728 F2 = 80 grams
A. 0.110 3 4
B. 35x y *
B. 0.330 C. 25x4y3 L = ar"·1 49. Factor the given expression:
c. 0.220 * D. 25x
4
l a1 = 2, n =6, r = 6/2 = 3 6-3m 2 - 8m +4m3 .
D. 0.440 A. (3 + 4m)(2 + m 2 )
The term involves l
is only choices C L = 2(3)6 " 1 = 486 B. (3 + 4m)(2- m2 )
Probability that is red then blue: and D. The possible answer is C, since C. (3 • 4m)(2 • m2 ) *
p 1 = .!.x.!!.=~ the sum of the exponents is 7. 46. Find the sum of the roots of: 2x2 + 2
D. (3 - 4m)(2 + m )
30 29 870
5x + 5 = 0.
2 3
Probability that is blue then red: x' = (n)(n-l)(n-2) .....(n-r+l) xn-ryr A. 5/2 6-3m - 8m +4m
p 12 8 96 r! B. -5/2 * = (6 - 3m2) - (8m -4m3 )
2 = 3ijXzg = 87ij C. 2/5 = 3(2 - m2) - 4m(2 - m2)
r = 4, n = 7 D. "-215 = (2- m2 )(3- 4m)
Pr = P1 + Pz x' = 7(6)(5)(4) x 7-4y4
4! b
Sum of the roots = X1 + Xz = 50. Which of the following is a prime
P = ~x~ ·=~ = 0.22 x' = 35x y 3 4 a
number?
T 870 870 870 a = 2, b = 5
A. 100
Sum of the roots B. 101 *
40. From a box containing 8 red balls, 10 43. Find the last term of the progression:
white balls and 12 blue balls. If one ball is 10, -2, -14, ...... to 17th term. C. 99
drawn at random, determine the A. -182 * 47. Find the value of x in the equation: D. 105
B. -1462 (5x+6)112 = x+2
probability that is not blue:
c. -220 A. 2 and 1 Prime number is an integer that is greater
A. 0.60 *
B. 0.45 D. -1530 B. 2 and -1 * than 1 and is divisible by 1.
a = 10, d = -2-10 = -12, n = 17 C. -2 and -3
C. 0.50
D. 0.35 L = a1 + (n -1)d D. -2 and 3 51. Funds supplied by others on which a
L = 10 + (17 -1)(-12) fixed rate of interest must be paid and the
= 0.60 L = -182 ,Jsx + 6 = x + 2 debt be repaid at a specific place and
p = 30
8+10
x2-x-2 = 0 time.
x = 2,x= -1 A. Borrowed capital *
41 . Determine the 7 th term of the 144. Find the ~um of the_ i~tegers between
expansion (2y -11)H 2 and 100 which are diVISible by 3. REFRESHER MANUAL 2nd Edition by JAS TORDILLO
A. 970
REFRESHER MANUAL 2"d Edition by JAS TORDILLO
20-14 DAY 20- SOLUTION
B Investment
C. Salvage value 58. Liquid assets such as cash and other
D Interest assets that can be corverted quickly into
cash, such as accounts receivables and
52 What is an annuity? merchandize are called:
A. A series of equal payments at equal A. land anq buildings
time periods. • B. current assets *
B The cost of manufacturing a product. . C. total assets
C An 1nvestment that yields an equal D. fixed assets
amount of interest each year.
D. The overhead cost per unit of 59. What is the recorded current value of
PART 2
production. an asset?
A. salvage value
53. Estimated value at the end of the B. present worth
useful life: C. book value*
A. salvage value * D. scrap value
B. economic life
C. compounded annually 60. What is the method of depreciation
D. balance sheet where a fixed sum of money is regularly
deposited at compound interest in a real
54. Consists of the actual counting or or imaginary fund in order to accumulate " POWER PLANT ENGINEERING
determination of the actual quantity of the an amount equal to the depreciation of an
materials on hands as of a given date:
A. physical inventory •
asset at the end of the asset's estimated
life?
&
B. technological assessment A. sinking fund method *
C material update B. SYD method
C. straight line method
INDUSTRIAL ENGINEERING
D. matenal count
D. declining balance
55 A depreciation model where the
value of an asset decreases at a
decreasing rate·
A straight line
B. declining balance *
C sinking fund
D. sum of the years digit

56. What do you call an increase in the


value of a capital asset?
A. capital expenditure
B. capital stock
C. capital gain *
D. profit

57. What is the value IJ!Ihich a


disinterested third party, different from
the buyer and seller, will determine in
order to establish price acceptable to
both parties?
A. franchise value
B. goodwill value
C. fair value *
D. market value
REFRESHER MANUAL2"d Edition by JAS TORDILLO
Day 21 - EXAM I 21 - 1
Power and Industrial Plant Engineering
B. 0.024
DAY 21 - EXAM c. 0.022
D. 0.026
1. Methane gas CH4, burns in air and
release 192 kcal/mol. Assuming a 90% 5. An inventor proposes to have
efficient heat transfer, what mass in kg developed a small power plant that
of ice at -17.8 deg C can be converted operates at 70% efficiency. It operates
to water at 37.8 deg C by burning 500 between temperature extremes of 600°C
liters of methane measured at 20.56 deg and 50°C. Your analysis shows that the
C and 1 atm. maximum possible efficiency is:
A. 15 g A. 56
B. 28 g B. 67
C. 20 g c. 63
D. 130 g D. 72
2. A hydro-electric generating station is 6. The term "nominal" size in turbine
supplied from a reservoir of capacity pipe is:
3
6,000,000 m at a head of 170 m. A. outside diameter
Assume hydraulic efficiency of 80% and B. pipe wall structures
electrical efficiency of 90%. The fall in C. inside diameter
the reservoir level after a load of 15 D. approximate size
MW has been supplied for 3 hrs, ·if the
catchment area is 2.5 km 2 is nearest to 7. Heat transfer decreases with
(in em): increased
A. 5.39 A. conductivity
B. 4.83 B. R-factor
c. 5.98 C. U-factor
D. 4.32 D. molecular activity

3. If the actual draft required for a 8. A river 60 m wide and 2 m deep flows
furnace is 6.239 em of water and the at 2 m/s. a hydro-plant develops a
fractional losses in the stack are 15% of pressure of 300 kPa gage just before
the theoretical draft, calculate the the turbine, what maximum power in
required height in m. Assume that the MW is possible:
flue gas have an average temperature A. 40
of 149 deg C and the molecular weight B. 56
of 30. Assume air temperature of 21 c. 48
deg C. D. 72
A. 215
B. 200 9. The actual head, neglecting kinetic
C. 220 energy, in which the pump work against
D. 210 A. total dynamic head
B. pressure head
4. Water flows through a 10-cm C. suction head
diameter, 100 m long connecting two D. static head
reservoirs with an elevation difference of
40 m. the average velocity is 6 m/s. 10. A 1.5 m pipe stands vertically with
Neglecting minor losses, the friction 0.08 m top diameter and 0.04 bottom
factor is: · diameter. If the water flow rate is 0.020
A. 0.020

REFRESHER MANUAL 2nd Edition by JAS TORDILLO


21 - 2 I Day 21 - EXAM Day 21 - EXAM I 21 - 3
Power and Industrial Plant Engineering
m"/s, fi nd the pressure difference at the. B. decrease
top and bottom. C. liquefies A. 0.1656 At 1.03 Mpa: V9 = 0.18905
A. 104 kPa D. increase B. 0.3565 . ht = 768.50 hrg = 2010.7
B. 220 kPa C. 0.4235 sr = 2.1512 =
sr9 4.4252
C. 300 kPa 16. The power intended to be always D. 0.2565 A. 25.5
D. 901 kPa available even under emergency B. 19.49
conditions. 22. In pipe specification, schedule is C. 12.15
11 . Which of the following statement is A. dump power used, when the pipe is specified as D. 29.20
true for a vapor dome drawn on a T-S B. pri'me power "schedule 80" the pipe corresponds to
diagram? C. firm power the: 26. The primary effect of reheating is to:
A. The bell-shaped curve indicates the D. reserve power A. extra standard weight A. increase the efficiency
saturation points for a constant pressure B. internal processes B. decrease the pump work
B. The region under the bell-shaped 17. In a completely turbulent flow the C. allowable stress C . decrease the heat requirement
curve indicates superheating head loss: D. old standard weight D. decrease or eliminate moisture
C. The bell-shaped curve indicates the A. increases with velocity squared condensation in the turbine
saturation points for various pressures B. increase with velocity 23. A graphical representation between
D. The left side of the bell-shaped curve C. decrease with wall roughness discharge and time is known as: 27. A pressure rise of 500. kPa is
indicates saturated vapor D. increase with flow rate A. hectograph needed across · a pump in a pipe
B. monograph transporting 0.2 m3 /s of water. If the
12. Ninety kilograms of ice at 0°C 18. Steam enters a turbine in a 20-cm C. hydrograph pump is 85% efficient, the power
completely melted. Find the entropy pipe at 600°C and 6 Mpa (v = 0.06525 D. !apograph needed In kw would be:
changed in kJ/K. (Take latent heat of m3 /kg). It exist from a 5-cm pipe at 20 A. 118
ice as 320kJ/kg). = = 3
kPa with x 1 (V9 7.649 m /kg). What 24. A vessel of 0.058 m3 capacity is B.85
A. 0 is the velocity ratio? well-insulated and is divided equally by c. 100
· B. 85 A. 1875 a rigid conducting diaphragm. Initially D. 65
C. 45 B. 1210 both halves contain air at a pressure of
D. 105 C. 1640 137.8 kPa and 413.4 kPa and 28. The chemical formula for butane:
D. 320 temperature of 27°C and 177°C A. CzHs
13. A 2000 kw diesel engine un it uses 1 respectively. What is the increase of B. C4H1o
bbl of oil per 525 kw-hr produced. Oil is 19. The thermal effic iency of gas-vapor enthalpy of the system in kJ/K? . C. C3Hs
25°API. Efficiency of generator is 93% cycle as compared to steam or gas A. 1.002 D. C1oH1e
and mechanical efficiency of engine is turbine is: B. 0.00173
80%. What is the thermal efficiency of A. greater than c. 0.5080 29. A miXture of 14.7 psia and 68°F that
the engine based on indicated power B. lower than D. 0.1080 is 30% by weight COz and 70% by
(%)? C. less than weight Nz has a partial pressure of COz
A. 31 .7 D. equal to 25. A venturi meter is placed in a line in psia that is nearest to:
B. 29.67 carrying dry saturated steam at 1.03 A. 2.14
C . 39.6 20. Find the power of a rotating shaft Mpa to enable estimates of steam flow B. 8.83
D. 31 .6 with a torque of 188 N-m and rotating at rate to be rn.ade. The inlet and throat c. 3.15
1350 rpm. diameter of the venture are 22.88 em D. 7.88
14. The radius of cross-sectional area A. 35.6 HP and 17.78 em respective ly. It is found
of flow to the wetted perimeter. B. 36.6 HP out that between these two positions the 30. The change in thermodynamic
A. hydraulic head C. 111.55 HP steam pressure falls by 0.04 Mpa. availability is equal to:
B. hydraulic mean dept D. 156.75 HP Assuming the flow is isentropic. What is A. the quantity delta S
C. hydraulic radius the rate steam flow in kg/s? B. maximum possible work output
D. hydraulic garment 21. One hundred eighty grams saturated C. actual work output ·
water at temperature 95°C undergoes Properties of steam: · D. the change in enthalpy
15. If the exhaust pressure is lowered vaporization process at constant 3
At 0.99 Mpa V9 = 0.1963 m /kg
or the boiler pressure raised, the pressuce. Determine the change in ht = 760.88 hg = 2777.7 31. The process where heat energy is
moisture content of steam. volume in m3 • Properties of 95°C (V1 = St = 2.1345 Stg = 4.4555 transferred to a thermal energy storage
A. vaporize 1.0397 and V9 = 1981.9). device:

REFRESHER MANUAL 2nd Edition by JAS TORDILLO REFRESHER MANUAL 2nd Edition by JAS TORDILLO
- - -- -- - - - ---

21 - 4 I Day 21 - EXAM Day 21 - EXAM I 21 - 5


A adiabatic cooling 38. W hat differential pressure, in Power and Industrial Plant Engineering
B. isentropic storage pascals, exists at the bottom of a 3.0 m 44. Ice cubes were added to a glass of 49. A 2.1 kg of refrigerant is circulated
C. isothermal cooling vertical wall if the temperature inside is water and stirred. Moistu re starts to per hour in a refrigeration system·,
D regeneration 20°C and outside it is -20°C. Assume condense at the outer surface of- the whose refrigerating effect is 290 kJ per
equal pressures at the top. glass. what is the temperature of the kg. Compute the heat that has to be
32. Which is not a viscosity rating? A. 15 moisture at the outer surface?
0 removed from the system per hour.
A . API B. 8 A critical temperature A. 309 kJ/hr
B Redwood C. 12 B. surface temperature B. 609 kJ/hr
c. ssu D. 6 C. dew point temperature C. 509 kJ/hr
D. Centipoise D. saturation temperatu re D. 709 kJ/hr
39. For saturated air, the value of dry-
33. Given gaseous fuel CaH1a and the bulb, wet bulb and dew point
45. The dew point temperature of the 50. A centrifugal pump has a constant
volumetric products of combustion: temperature is: products of combustion is the saturation speed of 1500 rpm and has a head of
COz = 10.01%, CO = 0.85%, Nz = 85%, A . proportional temperature that corresponds to the 75 m. What is the effect on the head of
0 2 = 14%. Determine the air-fuel ratio. B. the same partial pressure of the in the the pump if the impeller diameter is
A. 18.0 C. inversely proportional products of combustion. reduced from 280 mm to 230 mm?
B. 20.1 D. unity A. sulfur dioxide A. 25.6 m
c. 15.0 B. water vapor B. 65.6 m
D. 24.0 40. If H; is the indicated horsepower and
H 8 is the indicated brake horsepower of
C. nitrogen c. 50.6m
D. carbon dioxide D. 72.6 m
34. In turbulent flow in a pipe we know a compressor, then what is mechanical
the: efficiency, Em, equal to?
46. The size of the reciprocating pump
A. Reynolds number is much greater A. Em= He/H; is stamped on the builder's plate as 3 x ~ )
than 10,000 B. Em= H,l Hs
4 x 6. The diameter of the liquid
B. Fluid particles move in straight line C. Em= Hs- H; cylinder is:
DAY 21- ANSWER KEY
C Viscous stresses dominate D. Em= H;- Hs 1.8 16.C 31.046.C
A. 3inches 2. A 17. A 32. B 47. 0
D Shear stress varies linearly with B. Sinches 3. C 18. A 33. 8 48. A
radius 41 . The least number of compressor/s 4. A 19. A 34.049. B
C. 4inches
if) a multistage system is: 5. C 20. A 35. A 50. C
D. 6 inches
3"5. The water products is in its vapor A . four 6. 0 21 . B 36. C
state: El two' 7. B 22. D 37. D
47. A carnot refrigerator operates 8. 0 23. C 38. 0
A. lower heating value C. three
between two reservoirs. One reservoir 9, B 24. B 39. 8
B. higher heating value D. one 10.A25.B 40.8
is at a higher temperature, T, and the
C. gross calorific value other is at a cooler temperature, t. What 11. C26. D 41. B
D latent heat 42. A refrigerant control that guards the 12.027. A 42.0
is the coefficient of performance, COP, 13.C28.8 43.C
compressor from overloading brought
of the refrigerator? 14.C29.C 44.C
36. Given the velocity of 10 m/s. about by an increased loads in the
A. T/t 15. B 30. B 45. 8
Compute the velocity head in m. evaporator due to defrosting, warm
B. 1 - tiT (I
A. 6.2 products and other is called:
A. safety valve
c. tiT -1
B. 8.0 D. U(T - t)
C. 5.1 B. suction valve
D. 3.5 C. solenoid valve
48. Air is compressed in a reversible
D. expansion valve
. compressor at an initial temperature and
37. Air standard efficiency of a diesel
43. Latent heat of air is a function of the pressure of 28 deg C and 95 kPa,
engine depends on:
dew point temperature and the sensible respectively. Air leaves the compressor
A. speeds
heat of air is a function of the _ _ __ at 650 kPa. Compute the compressor
B. torque
A. wet-bulb temperature work per unit mass.
C. fuel A. -221.34 kJ/kg
D. compression ratio B. critical point
C. dry-bulb temperature B. -234.34 kJ/kg
D. dew point temperature c. -228.34 kJ/kg
D. -324.34 kJ/kg
REFRESHER MANUAL 2nd Edition by JAS TORDILLO REFRESHER MANUAL 2nd Edition by JAS TORDILLO
21 • 6 1 Day 21 - Solution Day 21 - Solution 1 21 • 7
P·o wer and Industrial Plant Engineering
3. If the actual draft required for a
DAY 21 - SOLUTION furnace is 6.239 em of water and the operates at 70% efficiency. It operates C. suction head
fractional losses in the stack are 15% of between temperature extremes of 600°C D. static head
1. Methane gas CH4, burns in air and the theoretical draft, calculate the and 50°C. Your analysis shows that the
release 192 kcal/mol. Assuming a 90% required height in m. Assume that the maximum possible efficiency is: 10. A 1.5 m pipe stands vertically with
efficient heat transfer, what mass in kg flue gas have an average temperature A. 56 0.08 m top diameter and 0.04 bottom
of 1ce at -17.8 deg C can be converted of 149 deg C and the molecular weight diameter. If the water flow rate is 0.020
B. 67
to water at 37.8 deg C by burning 500 of 30. Assume air temperature of 21 c. 63 * m3 /s, find the pressure difference at the
liters of methane measured at 20.56 deg deg C. D. 72 top and bottom.
C and 1 atm. A. 215 A. 104 kPa *
A. 15 g B. 200 Max cycle elf= TH- TL · = (600 + 273) - (50 + 273) B. 220 kPa
B. 28 g * c. 220 * TH (600+273) C. 300 kPa
C. 20 g D. 210 = 0.63 = 63% D. 901 kPa
D. 130 g
hw = total draft
PV = nRT hw = 6.239 + 0.15hw
6. The term "nominal" size in turbine
pipe is:
F\
-
vl P2
+ - + ZJ = - = - + Z2
v?
(10 1.325)(0.5) = n (8.3143)(20.56 + hw = 7.34 em water A. outside diameter w 2g w 2g
273)
n =0.021 mol of CH4
hw = 0.0734 (9.81) = 0.72 kPa = 720 Pa B. pipe wall structures
C. inside diameter t.P=i='1 - P2 =
v22 v21
;g +(Z2-ZI)r
l
3
Density of air= 1.2 kgim at 21°C and 1 D. approximate size •
[
Heat Supplied by methane = Heat atm pressure Q 0.020
absorbed by ice 7. Heat transfer decreases with v1=-= =3.97m/s
kcal 4.187 kJ Solving for the density of flue gas: A1 1t (0.080) 2
192 -· X - - x0.021 mol x0.90 increased 4
mol kcal R = 8.3143 = o. 277 A . conductivity v2 = o.o2o
= m[2093(0 -- 17.B) + 335 +4.187(37.8-0)jkJ 30 B. R-factor *
kg
d =-p-= 101.325 =0.867 k /mJ C. U-factor ~ (0.04) 2 = 15.91 m/s
m =0.0286 kg =28.6 grams 9 Rg T
9 0.277(149 + 273) g
D. molecular activity
4
hw = H(da- dg) 6P =p - p =[(1 5.91)2 - (3.97)2 + (0- 1.5)}.8 I
2. A hydro-electric generating station is 2
supplied from a reservoir of capacity
2.2!. H(l .2- 0.867)
. 9:81
8. A river 60 m wide and 2 m deep flows 2(9.81)
at 2 m/s. a hydro-plant develops a
6·,ooo.ooo m 3 at a head of 170 m. H= 22Q m
pressure of 300 kPa gage just before = 103.96 kPa
Assume hydraulic efficiency of 80% and
the turbine, what maximum power in
electrical efficiency of 90%. The fall in 4. Water 'flows through a 10-cm
the reservoir level after a load of 15 diameter, 100 m long connecting two MW is possible: 11 . Which of the following statement is
MW has been supplied for 3 hrs, if the reservoirs with an elevation difference of A. 40 true for a vapor dome drawn on a T-S
catchment area is 2.5 km 2 is nearest to 40 m. the average velocity is 6 m/s. B. 56 diagram?
Neglecting minor losses, the friction C. 48 A. The bell-shaped curve indicates the
(in em):
D. 72 * saturation points for a constant pressure
A. 5.39 * factor is:
B. 4.83 A. 0.020 * 3 B. The region under the bell-shaped
Q = A V = 60 (2) (2) = 240 m /sec curve indicates superheating
C. 5.98 B. 0.024
D. 4.32 C. 0.022 Head=~= 300
=30.581 m C. The bell-shaped curve indicates
w 9.81
D. 0.026 the saturation points for various
Power Output = QwH nhne pressures*
Water Power = QwH D. The left side of .the bell-shaped curve
15,000 = Q (9.81) (170) (0.80) (0.90) By Darcy's Equation:
= 240(9.81)(30.581)
Q = 12.492 m3/s tl.V 2 indicates saturated vapor
hf ~--
2g0
= 72,000 kw = 72 MW
In 3 hours, volume of water consumed: 40 - 1(100) ( 6) 2 12. Ninety kilograms of ice at 0°C
9. The actual head, neglecting kinetic
= 12.492 (3) (3600) = 134,914 m3 2(9.81 ) ( 0 . 1) completely melted. Find the entropy
f = 0.0218 energy, in which the pump work against:
changed in kJ/K. (Take latent heat of
A . total dynamic head
Volume = Area x height ice as 320kJ/kg).
134,914 = (2.5 x 106 ) height I 5. An inventor proposes to have B. pressure head *
A.O
height= 0.0539 m = 5.39 em developed a small power plant that B. 85
REFRESHER MANUAL 2nd Edition by JAS TORDILLO REFRESHER MANUAL 2nd Edition by JAS TORDILLO
21 - 8 I Day 21 - Solution
Day 21 - Solution 1 21 - 9
C. 45 C. liquefies Power and Industrial Plant Engineering
D. 105 • D. increase
C. 111 .55 HP 6S = 6SA + 6Se
D. 156.75 HP · Tr and
68 =3 = 320(90) =105.5~ 16. The power intended to be always ~SA =mACvlnTA
T 0+273 OK av ailable even under emergency
conditions . 1350 Tr
P = 2TTTN = 2n(188) - - ~Sa= msCv In Ts
13. A 2000 kw diesel engine unit uses 1 A. dump power 60
bbl of oil per 525 kw-hr produced. Oil is B. prime power = 26,577.87 w = 35.6 hp 137 J0.058)
C. firm power • PA VA _ '\.--i-. = 0.0464 kg
25°API. Efficiency of generator is 93% mA "' RTA - 0.287(27- 273)
and mechanical efficiency of engine is D. reserve power 21 . One hundred eighty grams saturated
80%. What is the thermal efficiency of water at temperature 95°C undergoes 413.4 0.058)
. .
( 2
the engine based on indicated power 17. In a completely turbulent flow the vaporization process at constant Pe Va _ =o.0928 kg
head loss: me = RTe - 0.287(177 + 273)
(%)? pressure. Detenmine the change in
A. 31.7 A. increases with velocity squared • volume in m 3 . Properties of 95°C (Vr =
B. 29.67 B. increase with velocity 1.0397 and Vg = 1981.9). · Solving for the equilibrium temperature:
c. 39.6 . C. decrease with wall roughness A. 0.1656
D. 31 .6 D. increase with flow rate B. 0.3565 • 0.0464(0.718)(TF- 27)
c. 0.4235 = 0.0928(0.718)(177- TF)
1 bbl = 42 gal 18. Steam enters a turbine in a 20-cm D. 0.2565 TF = 127°C(400 °K)
pipe at 600°C and 6 Mpa (v = 0.06525
3
~SA =0.0464(0.718)1n
400
= 0.009584 kJ /kg
m /kg). It exist from a 5-cm pipe at 20 300
Solving for the density: 6V=m(Vg-Vr) . 400
kPa with x = 1 (Vg = 7.649 m3 /kg). What = 0.180(1981 .9 -1 .0397) X 10'
3 ~SA =0.0928(0.719)1n =-0.007848 kJ/kg
0 API =~- 1 3 1 .5 is the velocity ratio?
300
SG
.'\. ' 1875 .
=0.3565 m 3
68 = 0.009584 + (-0.007848)
25=~-1 3 1.5 = 0.001736 kJ/kg
SG 15 . 6 B. 1210 22. In pipe specification, schedule is
SG1s s = 0.904 C. 1640 used, when the pipe is specified as
D. 320 25. A venturi meter is placed in a line
Density= 0.904 kg/li "schedule 80' the pipe corresponds to carrying dry saturated steam at 1.03
the: Mpa to enable estimates of steam flow
mF = fuel consumed Continuity Equation: A. extra standard weight rate to be made. The inlet and throat
. = 42(3.7854)(0.0904) = 143.724 kg p1AN1 = pzAN2 B. internal processes diameter of the venture are 22.88 em
Or = 41,130 = 139.6(25) = 44,620 kJ/kg C. allowable stress and 17.78 em respectively. It is found
A, V1 = A 2 V2
Indicated Work = 525
=705.645kw- hr D. old standard weight • out that between these two positions the
VI V2
0.93 X 0.80 steam pressure falls by 0.04 Mpa.
= V2 =(£L) (~) = (20)
2 2
= 705.645(3600) Velocliy ratio ( 7.649 ) 23 . .A graphical representation between Assuming the flow is isentropic. What is
= 2,540,323 kJ V1 D2 vt 5 0 .06525 discharge and time is known as: the rate steam flow in kg/s?
2 540 323 = 1875 A. hectograph
Indicated thennal eft = · • = 39.6% B. monograph Properties of steam:
143. 724( 44,620}
C. hydrograph • 3
19. The thermal efficiency of gas-vapor At 0.99 Mpa V 9 = 0.1963 m /kg
14. The radius of cross-sectional area cycle as compared to steam or gas D. topograph hr = 760.88 hg = 2777.7
of flow to the wetted perimeter. turbine is: St = 2.1345 Stg = 4.4555
A. hydraulic head A. greater than * 24. A vessel of 0.058 m3 capacity is
B. lower than well-insulated and is divided equally by At 1.03 Mpa: Vg = 0.18905
B. hydraulic mean dept
C. hydraulic radius • C. less than a rigid conducting diaphragm. Initially hr = 768.50 htg = 2010.7
D. hydraulic garment · D. equal to both halves contain air at a pressure of sr=2.1512 Srg = 4.4252
137.8 kPa and 413.4 kPa and
15. If the exhaust pressure is lowered 20. Find the power of a rotating shaft temperature of 27°C and 177°C A . 25.5
or the boiler pressure raised, the with a torque of 188 N-m and rotating at respectively. What is the increase of B. 19.49 •
moisture content of steam. 1350 rpm. enthalpy of the system in kJ/K? c. 12.15
A. vaporize A. 35.6 HP * A. 1.002 D. 29.20
B. decrease • B. 36.6 HP B. 0.00173 *
c. 0.5080
D. 0.1080
REFRESHER MANUAL 2nd Edition by JAS TORDIUO
REFRESHER MAN"UAL2nd Edition by JAS TORDILLO
21 -10 I Day 21 -Solution Day 21 - Solution I 21 - 11
Power and Industrial Plant Engineering
hl-h 2 = v;- _.1
-v2 29. A mixture of 14.7 psia and 68uF that
2 is 30% by weight C02 and 70% by From actual reaction: 100 mots of dry 38. What differential pressure. in
weight N2 has a partial pressure of C02 pascals, exists at the bottom of a 3.0 m
where : m = AV or V = mv = 4mv · gas:
in psia that is nearest to:

n
v A •d2 vertical wall if the temperature inside is
A. 2.14 20°C and outside it is -20°C. Assume

<~·[[:lr -[:;
ACsH1s + B(Oz + 3.67 Nz) = 10.01
B. 8.83 C02 + 0.85 CO + 85N2 + 4.14 Oz + equal pressures at the top.
h, -h, c. 3.15 * CH20 A. 15
D. 7.88 B. 8
Solving for h2: Nitrogeri Balance: c. 12
s, = S2 = (sf+ x sfg)2
(2.1512 +4.4252) = 2.1 345 + Xz(4.4555)
Pco2 = ( N~~
2 )rr 3.76 (2) B ::' 85 (2)
B = 22.6
Carbon Balance:
D. 6 *

Pressure =density x height


Xz = 0. 9969 100 kg of gas mixture: A (8) = 10.01 + 0.85 . m P kg 2 N
Denslty=-=- x 9.81 m/ s .....
A = 1.3575 V RTm3 2m (Pa)
hz = hf + x hfg Mass: Mol
Actual A/F = 22.6(1 + 3.76)28.97 =IO. I kg air [_I_ - ...!.._]
=760.88 + 0.9969(2777.77- 760.88) kg fuel
t.P = P1 - P2 =
= 2771.5 C02 =30 kg 30/40 = 0.682 mol
1.3575[8(12)+ 18(1)] Tl T2

=
h, 768.5 + 2010.7 = 2779.2 N2 =70 kg 70/28 = 2.5 mol 6P= 3(101.325) [ I _ _1_ ](9.81)= 5.6 Pa
Nr = 3.182 mol 34. In turbulent flow in a pipe we know 0.287 - 20 + 273 20 + 273
the:
(2779.2 - 1771.5) kJ XI 000 ..:!._
kg kJ Pco2 =(
0 682
·
3.182
)14.7 = 3.15psia A . Reynolds number is much greater
than 10,000
39. For saturated air, the value of dry-
bu lb, wet bulb and dew point
2 2 2
= m (8)[(. 0. 1963) -(0. 18905 ) ] B. Fluid particles move in straight line temperature is:
n2 1778 2 0.2288 2 30. The change in thermodynamic C. V iscous stresses dominate A . proportional
m = 19.3 kg/s availability is equal to: D. Shear stress varies linearly with B. the same*
A. the quantity delta S radius* C. inversely proportional
26. The primary effect of reheating is to: B. maximum possible work output* D. unity
A. increase the efficiency C. actual work output 35. The water products is in its vapor
B. decrease the pump work D.- the change in enthalpy state: 40. If H; is the indicated horsepower and
C. decrease the heat requirement A. lower heating value* He is the indicated brake horsepower of
D. decrease or eliminate moisture 31 . The process where heat energy is B. higher heating value a compressor, then what is mechanical
condensation in the turbine * transferred to a thermal energy storage C. gross calorific value efficiency, Em, equal to?
device: D. latent heat A. Em= Hs/H;
27 A pressure rise of 500 kPa is A . adiabatic cooling B. Em = H; I He*
needed across a pump in a pipe B. isentropic storage 36. Given the velocity of 10 m/s. C. Em= He- H;
transporting 0.2 m 3/s of water. If the C. isothermal cooling Compute the velocity head in m. D. Em = H; - Hs
pump is 85% efficient, the power D. regeneration * A . 6.2 Power Output lnd Power H·
Em= -- - - - =- -- - =_t
needed in kw would be: • B. 8.0 Power Input Brake Power He
A. 118 * 32. Which is not a viscosity rating? c. 5.1. 41. The least number of compressor/s
B. 85 A . 0API D. 3.5 in a multistage system is:
C. 100 B. Redwood*
H= 102 A. four
D. 65 C. SSU
2(9. I) = 5.1 m B. two*
D. Centipoise 8
C. three
Qwh 500(0.~ ) = 117.65 kw D. one
Power ~ eff ~ 0.85 33. Given gaseous fuel CsH1s and the 37. Air standard efficiency of a diesel
volumetric products of combustion: engine depends on: 42. A refrigerant control that guards the
28. The chemical formula for butane: C02 = 10.01%, CO= 0.85%, N2 = 85%, A . speeds compressor from overloading brought
A. CzHs 02 = 14%. Determine the air-fuel ratio. 8 . torque about by an increased loads in the
B. C4H1o" A. 18.0 C. fuel evaporator due to defrosting, warm
C. C3Hs B. 20.1 * D. compression ratio * products and other is called:
D. C,oH,s C. 15.0
A. safety valve
D. 24.0
REFRESHER MANUAL 2nd Edition by JAS TORDILLO REFRESHER MANUAL 2nd Edition by JAS TORDILLO
21 -12 I Day 21 -Solution Day 22 - EXAM I 22 - 1
B. suction valve Power and Industrial Plant Engineering
TL =t-
C solenOid valve COP= TH- TL T -t principally from the natural salts present
D. expansion valve* DAY 22- EXAM in the crude oil.
43. Latent heat of air is a function of the 48. Air is compressed in a reversible A. carbon test
compressor at an initial temperature and 1. Class of fuel oil which are topped B. moisture test
dew point temperature and the sensible crude petroleums or or viscous
• heat of air is a function of the _ _ __ pressure of 28 deg C and 95 kPa, C. ash test
residuums obtained in refinery D. carbon test
A. wet-bulb temperature respectively. Air leaves the compressor
at 650 kPa Compute the compressor operations:
B. critical potnt A. blended fuels
C. dry-bulb temperature* work per unit mass 8. The contamination of various
A. -221 .34 kJ/kg • B. residual oils amounts of water and impurities out of
D . dew point temperature
B. -234.34 kJ/kg C. fuel oils the boiler.
C. -228.34 kJ/kg D. gaseous fuels A. priming
44 Ice cubes were added to a glass of
D. -324.34 kJ/kg B. steam purity
water and stirred. Moisture starts to 2. It is a measure of resistance of oil to C. spray
[(P2P )k~l -I]
condense at the outer surface of the
flow:
glass, what is the temperature of the We= -kmRT, A. viscosity
D. carryover

l
k-1 1
m01sture at the outer surface?
B. fluid flow 9. The liquid contamination in the steam,
A. critical temperature
C.' specific gravity expressed in percentage, by weight, of

~
B. surface temperature
C. dew point temperature*
we=_ 1.4(0.287) (~+ ~31 (650)1.4-1
·1:4 _1 D. density the mixture.
1.4-1 95
D. saturation temperature A. steam qual(ty
3. The lowest temperature at which oil B. steam quantity
We= -221.34 kJ/kg
will flow under prescribed conditions: C. liquid quality
45. The dew point temperature of the
A. flash point D. wet quality
products of combustiOn IS the saturation 49. A 2.1 kg of refrigerant is circulated
B. pour point
temperature that corresponds to the per hour in a refngeration system,
C. distillation temp 10. The solids impurity in the steam,
partial pressure of the in the whose refrigerating effect is 290 kJ per
D. fire point expressed in parts per million (ppm) of
products of combustion. kg. Compute the heat that has to be
A. sulfur dioxide removed from the system per hour. impurity.
4. The degree API gravity of pure water A. steam spray
B. water vapor • A 309 kJ/hr at 60 degree F:
B. 609 kJ/hr * B. steam purity
C. nitrogen
C. 509 kJ/hr
A. 10 C. steam quantity
D carbon dioxide
B. 20 D. steam buffles
D. 709 kJ/hr
46. The s1ze of the reciprocating pump
c . 15
D. 40 11. Type of carryover that will occur
is stamped on the builder's plate as 3 x QA = m(h1 - h4)
under conditions of high water level or
4 x 6. The diameter of the liquid QA = 2. 1(290) = 609 kJ/hr
5. The range of fuel oils is approximately severe surging in the boiler drum.
cylinder 1s:
A. 3 inches 50. A centrifugal pump has a constant -~_,AP I. A. priming ca rryover
A. 10 to 40 B. spray carryover
B 5 1nches speed of 1500 rpm and has a head of
B. 20 to 30 C. foamover
C. 4 inches • 75 m What is the effect on the head of
C. 10to30 D. steam sampling
D. 6 1nches the pump if the impeller diameter is
D. 10 to 20
reduced from 280 mm to 230 mm?
12. A device use to determine the
47. A carnot refngerator operates A. 25.6 m 6. This is to detect to free sulfur or quality of steam.
between two reservoirs. One reservoir B 65.6 m corrosive sulfur compounds in residual A. throttling calorimeter
is at a higher temperature, T, and the C. 50.6 m *
fuels. B. steam valve
other IS at a cooler temperature. t. What D. 72.6 m A. ash test C . steam vent
~=(~)2
1s the coefficient of performance, COP, B. corrosion test D. steam nozzles
of the refrigerator? C. sulfur test
h2 d2

r
A T/t D. carbon test 13. First step in purification of steam
~: = ( ~~~
B. 1 -tiT
and necessary in water tube boilers.
c. tiT -1 7. This is to determine the A. primary separation
D. t/(T -t) * h2 = 50.6 m noncombustible impurities, which come B. steam washing
REFRESHER MANUAL 2nd Edition by JAS TORDILLO REFRESHER MANUAL 2nd Edition by JAS TORDILLO
,c:-

22 - 2 I Day 22 - EXAM Day 22 - EXAM I 22 - 3


Power and Industrial Plant Engineering
C. steam cleaning B. single shaft
D steam drying C. multiple shaft A. water rate 33. When ammonia is water free it is
D. triple shaft B. air rate known as anhydrous ammonia and
14. Consist of a separating and throttling C. power rate when mixed with water it is called
calorimeter, of high and low range. 20. If no fixed speed relationship exists D. fuel rate
A. electric calorimeter between the several shafts, all except A. water ammonia
B. universal calorimeter the power coupling shaft are known as: 27. The ratio of the highest main B. aqua ammonia
C throttling calorimeter A. multiple shaft compressor discharge pressure to the C. liquid ammonia
D steam calorimeter B. floating shaft lowest main compressor inlet pressure. D. moist ammonia
C. series-row A. pressure ratio
15. A process of removing small D. double-row B. cycle temp ratio 34. Recommended working pressures
amounts of moisture from a relatively C. cycle property ratio for design if using ammonia, of the high
large volume of steam, is essentially a 21. The ratio of the heat equivalent of D. cycle atmospheric pressure side is 250 psig and for the low side
filtration process. the net output power to the total heat rat _ _ psig.
A. heating supplied. 28. The ratio of maximum pressure in A. 150
B. ventilating A. mechanical efficiency the cycle to the atmospheric pressure. B. 100
C. drying B. heat efficiency A. cycle work level C. 225
D. washing C. thermal efficiency B. cycle pressure level D. 75
D. power efficiency C. cycle pressure ratio
16. A heat exchanger which recovers D. compre.ssion ratio 35. Which of the following refrigerants.
waste heat from the cycle and transfers 22. The fraction of recoverable waste has the lowest toxicity.
it to the fluid at a point where the heat actually restored to the cycle. 29. The ratio of the actual heat transfer A. ammonia
pressure is higher and the temperature A. regenerator effectiveness in a regenerator to that theoretically B. carbon dioxide
is lower. B. turbine efficiency possible. C. freon 12
A. mtercooler C. compressor efficiency · A. operating variable D. freon 11
B. regenerator D. cycle effectiveness B. regenerator effectiveness
C. aftercooler C. regenerator ratio 36. Type of refrigerant that exists as a
D. reheater 23. The ratio of the compressor power D. back-pressure ratio liquid at normal atmospheric
to gross turbine power. temperature and pressure and is also
17. When a heater is used to increase A. back-work ratio 30. Heat exchanger used to cool the called vacuum refrigerants.
the temperature between expansion B. work ratio exhaust gases in a closed or semiclosed A. freon 11
stages to approximate isothermal C. pressure ratio cycle. B. freon 22
expansion: D. power ratio A. intercooler C. freon 12
A. reheater B. heater D. methyl chloride
B. heater 24. The ratio of the output power to the C. aftercooler
C. after heater total installed turbine power. D. precooler 37. It is a higher pressure member of
D. intercooler A. cycle efficiency the Freon group which is suitable for
B. power ratio 31 . Piping used for ammonia operation at lower temperatures. It is
18. A patented device combining C. cycle pressure flitio refrigerants: relatively non toxic.
expansion and compression functions in D. work ratio A. copper A. freon 11
a single rotor permitting higher cycle B. steel B. freon 22
temperatures. 25. The quantity of a working medium C. bronze C. freon 17
A. cyrix entering the first sta'ge of compression D. copper D. carbon dioxide
B. comprex per unit output energy.
C chamber A. water rate 32. Most extensively used refrigerant, 38. Type of refrigerants that is
D. exhaust valve B. substance rate particularly in industrial and commercial noncorrosive and inert, has no odor, is
C. air rate refrigeration. nonirritating and essentially nontoxic.
19. When all rotors are coupled together D. higher rate A. ammonia A. freon 22
on a common axis and route with fixed B.. freon 12 B. carbon dioxide
speed reli3tionships. 26. The quantity of a specified fuel C. freon 11 C. air
A. double shaft consumed per unit of output energy. . D. water D. sulfur dioxide

REFRESHER MANUAL 2nd Edition by JAS TORDILLO REFRESHER MANUAL 2nd Edition by JAS TORDILLO
22 - 4 I Day 22 - EXAM Day 22 • Solution I 22 - 5
Power and Industrial Plant Engineering
39. For comfort cooling, the amount of B. arrangement of nozzles
a1r recirculated and only enough outside C. fineness of spray C. ventilation
air is introduced to provide for ventilation D. all of these D. industrial cleaning DAY 22- SOLUTION
requirements, approximately from
..,---__,...,.-cu ft per minute per occupant. 45. Air washers are rated at._,--__ 1. Class of fuel oil which are topped
A. 10 to15 cu ft of air per min per sq ft of cross- !rj crude petroleums or or viscous
)
B. 20 to25 sectional area through the spray residuums obtained in refinery
C. 15 to 25 chamber. DAY 22- ANSWER KEY operations:
D. 25 to 35 A. 200 to 500 1. 8 16.8 31. 846.A A. blended fuels
2. A 17. A 32. A47. A
B. 50to150 B. residual oils *
3. 8 18. 8 33. 8 48. 0
40. For comfort cooling, inside dry bulb C. 400 to 500 4. A 19. 8 34. A49. A C. fuel oils
temperature to be maintained at D. 100 to 350 5. A 20.8 35. C50. C D. gaseous fuels
breathing line should not exceed 6. 8 21 . C 36. A
.,....----,.--below outside design dry-bulb 46. The addition of moisture to air being 7. C 22. A 37. 8 2. It is a measure of resistance of oil to
8. D 23. A 38. 8
temperature . circulated is called: flow:
9. A 24. D 39. A
A. 10 to 15 deg F A. humidification 10.8 25. c 40.A A. viscosity *
B. 20 to 35 deg F B. dehumidification 11.A 26.0 41. 8 B. fluid flow
C. 15 to 25 deg F C. cooling 12.A 27. A 42. D C. specific gravity
D. 25 to 35 deg F D. drying 13.A 28. 8 43.C D. density
14.8 29. 8 44.0
15.C 30. C 45.C
41 . For comfort cooling, another 47. The removal of moisture to air being 3. The lowest temperature at which oil
frequently used rule states that inside circulated is: ~ will flow under prescribed conditions:
dry bulb temperature should be midway A. dehumidification A. flash point
between and outside B. heating B. pour point •
temperature. C. washing C. distillation temp
A. 50 deg F D. drying _D. fire point
B. 70 deg F
C. 60 deg F 48. The maximum air velocity 4. The degree API gravity of pure water
D. 90 deg F comfortable to human beings at rest is at 60 degree F:
approximately ft per min. A. 10 *
42. The temperature corresponding to A. 25 B. 20
saturation (100% RH) for a given weight B. 45 C. 15 ·
of vapor. C . 75 D. 40
A. dry bulb D. 50
B. wet bulb 5. The range of fuel oils is approximately
C. saturation 49. An atmospheric impurities of _ _ _ API.
D. dew point particles of 0.2 to 1 micron, resulting A. 10 to 40 *
from reactions such as distillation, B. 20 to 30
43. The actual amount of moisture oxidation of metal fumes, and pure C. 10 to 30
contained in 1 cu . ft of the mixture chemical reactions. D. 10 to 20
divided by the amount which 1 cu ft of A. fumes
the mixture would hold at the same B. smokes 6. This is to detect to free sulfur or
temperature if saturated. C. dust corrosive sulfur compounds in residual
A. specific humidity D. bacteria fuels.
B. humidity A. ash test
C. degree of humidity 50. The displacement of vitiated air from B. corrosion test*
D. saturated humidity an inhabited room and its replacement C. sulfur test
with fresh air. D. carbon test
44. The ability of an air washers to cool A. air conditioning
air depends on which of the following? B. drying 7. This is to determine the
A. water pressure at nozzles noncombustible impurities, which come

REFRESHER MANUAL 2nd Edition by JAS TORDILLO REFRESHER MANUAL 2nd Edition by JAS TORDILLO
22 - 6 I Day 22 - Solution Day 22 - Solution 1 22 - 7
Power and Industrial Plant Engineering
principally from the natural salts present C. steam cleaning
m the crude oil. D. steam drying B. single shaft* 26. The quantity of a specified fuel
A. carbon test C. multiple shaft consumed per unit of output energy.
B. moisture test 14. Consist of a separating and throttling D. triple shaft A. water rate
C. ash test • calorimeter, of high and low range. B. air rate
D. carbon test A. electric calorimeter 20. If no fixed speed relationsh ip exists C. power rate
B. universal calorimeter * between the several shafts. all except D. fuel rate *
8 The contamination of various C. throttling calorimeter the power coupling shaft are known as:
amounts of water and impurities out of D. steam calorimeter A. multiple shaft 27. The ratio of the highest main
the boiler. B. floating shaft* compressor discharge pressure to the
A prim1ng 15. A process of removing small C. series-row lowest main compressor inlet pressure.
B. steam purity amounts of moisture from a relatively D. double-row A. pressure ratio *
C. spray large volume of steam, is essentially a B. cycle temp ratio
D. carryover* filtration process. 21 . The ratio of the heat equivalent of C. cycle property ratio
A. heating the net output power to the total heat rat D. cycle atmospheric pressure
9. The liquid contamination in the steam, B. ventilating supplied.
expressed in percentage, by weight, of C. drying* A. mechanical efficiency 28. The ratio of maximum pressure in
the mixture. D. washing B. heat efficiency the cycle to the atmospheric pressure.
A. steam quality * C. thermal efficiency* A. cycle work level
B steam quantity 16. A heat exchanger which recovers D. power efficiency B. cycle pressure level *
C. liquid quality waste heat from the cycle and transfers C. cycle pressure ratio
D. wet quality it to the fluid at a point where the 22. The fraction of recoverable waste D. compression ratiCJ
pressure is higher and the temperature heat actually restored to the cycle.
10. The solids impurity in the steam, is lower. A. regenerator effectiveness * 29. The ratio of the actual heat transfer
expressed in parts per million (ppm) of A. intercooler B. turbine efficiency in a regenerator to that theoretically
impurity. B. regenerator * C. compressor efficiency possible.
A. steam spray C. aftercooler D. cycle effectiveness A. operating variable
B. steam purity * D. reheater B. regenerator effectiveness *
C. steam quantity 23. The ratio of the compressor power C. regenerator ratio
D. steam buffles 17. When a heater is used to increase to gross turbine power. D. back-pressure ratio
the temperature between expansion A. back-work ratio *
11. Type of carryover that will occur stages to approximate isothermal B. work ratio 30. Heat exchanger used to cool the
under conditions of high water level or expansion: C. pressure ratio exhaust gases in a closed or semiclosed
severe surging in the boiler drum. A. reheater * D. power ratio cycle.
A. priming carryover • B. heater A. intercooler
B. spray carryover C. after heater 24. The ratio of the output power to the B. heater
C foamover D. intercooler total installed turbine power. C. aftercooler *
D. steam sampling A. cycle efficiency D. precooler
18. A patented device combining B. power ratio
12. A device use to determine the expansion and compression functions in C. cycle pressure ratio 31 . Piping used for ammonia
quality of steam. a single rotor permitting higher cycle D. work ratio * refrigerants:
A. throttling calorimeter* temperatures. A. copper
B. steam valve A. cyrix 25. The quantity of a working medium B. steel*
C. steam vent B. comprex* entering the first stage of compression C. bronze
D. steam nozzles C. chamber per unit output energy. D. copper
D. exhaust valve A. water rate
13. First step in purification of steam B. substance rate 32. Most extensively used refrigerant,
and necessary in water tube boilers. 19. When all rotors are coupled together C. air rate* particularly in industrial and commercial
A. primary separation * on a common .axis and route with fixed D. higher rate refrigeration.
B. steam washing speed relationships. A. ammonia*
A. double shaft B. freon 12

REFRESHER MANUAL 2nd Edition by JAS TORDILLO REFRESHER MANUAL 2nd Edition by JAS TORDILLO
22 - 8 1 Day 22 - Solution Day 22 • Solution 1 22 - 9
Power and Industrial Plant Engineering
C. freon 11 C. air
D. water D. sulfur dioxide 44. The ability of an air washers to cool 50. The displacement of vitiated air from
air depends on which of the following? an Inhabited room and its replacement
33. When ammonia is water free it is 39. For comfort cooling, the amount of A water pressure at nozzles with fresh air.
known as anhydrous ammonia and air recirculated and only enough outside B. arrangement of nozzles A air conditioning
when mixed with water it is called air is introduced to provide for ventilation C. fineness of spray B. drying
requirements, approximately from D. all ot'these * C. ventilation *
A. water ammonia _ _ _ cu ft per minute per occupant. D. industrial cleaning
B. aqua ammonia * A. 10 to15 * 45. Air washers are rated at._,--__
C. liquid ammonia B. 20 to25 cu ft of air per min per sq ft of cross-
D. moist ammonia C. 15 to 25 sectional area through the spray
D. 25 to 35 chamber.
34. Recommended working pressures A 200 to 500
for design if using ammonia, of the high 40. For comfort cooling, inside dry bulb B. 50 to 150
side is 250 psig and for the low side temperature to be maintained at C. 400 to 500 *
_ _ psig. breathing line should not exceed D. 100 to 350
A. 150 * ____ below outside design dry-bulb
B. 100 temperature. 46. The addition of moisture to air being
c. 225 A. 10 to 15 deg F * circulated is called:
D. 75 B. 20 to 35 deg F A. humidification*
C. 15 to 25 deg F B. dehumidification
35. Which of the following refrigerants D. 25 to 35 deg F C. cooling
has the lowest toxicity. D. drying
A. ammonia 41. For comfort cooling, another
B. carbon dioxide frequently used rule states that inside 47. The removal of moisture to. air being
C. freon 12 * dry bulb temperature should be midway circulated is:
D. freon 11 between _ and outside A. dehumidification *
temperature. B. heating
36. Type of refrigerant that exists as a A 50 deg F C. washing
liquid at nonnal atmospheric B: 70 deg F * D. drying
temperature and pressure and is also C.' 60 d!:!g F
called vacuum refrigerants. D. 90 deg F 48. The maximum air velocity
A. freon 11 * comfortable to human beings at rest is
B. freon 22 42. The temperature corresponding to approximately ft rer min.
C. freon 12 saturation (100% RH) for a given weight A 25
D. methyl chloride of vapor. B. 45
A dry bulb C. 75
37. It is a higher pressure member of B. wet bulb D. 50*
the Freon group which is suitable for C. saturation
operation at lower temperatures. It is D. dew point* 49. An atmospheric impurities of
relatively non toxic. particles of 0.2 to 1 micron, resulting
A freon 11 43. The actual amount of moisture from reactions such as distillation,
B. freon 22 * contained in 1 cu. ft of the mixture oxidation of metal fumes, and pure
C. freon 17 divided by the amount which 1 cu ft of chemical reactions.
D. carbon dioxide the mixture would hold at the same A. fumes*
temperature if saturated. B. smokes
38. Type of refrigerants that is A. specific humidity C. dust
noncorrosive and inert. has no odor, is B. humidity D. bacteria
nonirritating and essentially nontoxic. C. degree of humidity *
A. freon 22 D. saturated humidity
B. carbon dioxide *

REFRESHER MANUAL 2nd Edition by JAS TORDILLO REFRESHER MANUAL 2nd Edition by JAS TORDILLO
Day 23 - EXAM I 23 - 1
Power and Industrial Plant Engineering
C. two
DAY 23- EXAM D. 4
1. Any boiler which does exceed any of
7. Steam generators should be mounted
the following: 405 mm inside diameter over a suitable foundations or concrete
and 1065 mm overall length of outside pad of not less than _ _thick and with
to outside heads at center. sufficient base area.
A. portable boiler A. 150 mm
B. miniature boiler B. 255 mm
C. locomotive C. 195 mm
D. power boiler D. 305 mm
2. A closed vessel intended for use in 8. When new boilers are installed in
heating water or for application of heat either existing or new buildings, a
to generate steam or other vapor to be minimum height of at least _ _ shalf
used externally to itself. be provided between the top of the
A. boiler boiler proper and ceiling.
B. boiler or steam generator A. 2130 mm
C. steam generator B. 900 mm
D. none of these C. 1560 mm
D. 746 mm
3. A vessel in which pressure is
obtained from an external sources, or 9. When feeding hot water, feed pump
from an indirect application of heat. should at least be _ _ below hot well
A. unfired pressu re vessel to prevent vapor lock.
B. pressure vessel A. 550 mm
C. power boiler B. 1220 mm
D. low pressure boiler C. 990 mm
D. 3050 mm
4. Any boiler or unfired pressure vessel . '
constructed, installed, placed in 10. A steel catwalk or platform at least
operation but subject to annual _ _ wide and provided with standard
inspection. · handrails and toeboard on either side
A. existing installations shall be installed across the tops of
B. new boilers adjacent boilers.
C. hot water boiler A . 255 mm
D. reinstalled boilers B. 455 mm
C. 355 mm
5. A boiler in which that has been D. 555mm
inspected and declared unsafe to
operate or disqualified stamped and 11 . For power boilers, when the tensile
marked indicating its rejection. strength of steel is not known, it shall be
A. second hand taken as _ _N/mm2 and 310 N/mm 2 for
B. reinstalled wrought iron.
C. condemned A. 379
D. unfired B. 450
C. 398
6. No part of the steam generator D. 521
should be closer than _ _ meter from
any walls of the building.
A. one
B. three
23 - 2 I Day 23 - EXAM Day 23 - EXAM I 23 - 3
12. For power boilers, the resistance to surface or the generating capacity
Power and Industrial Plant Engineering
crushing of mild steel shall be taken at exceeds 910 kg/hr. it shall have _ _ C. 111
_ _N/mm2 of cross sectional area. 28. Each miniature boiler shall be
safety valves. D. 145 equipped with _ __
A. 655 for
A one
B. 855 determination of water level.
B. two 23. A boi ler of which both the location
c. 755 A. pressure gage
C. two or more and ownership have been.changed after
D. 955 B. safety valve
D. three o r five primary use. C. water gage glass
A. internal boiler D. fusible plug
13. The lowest factor of safety 18. For miniature boilers, the lowest B. secondhand boiler
permissible on existing installations shall permissible water level shall be at a C. surplus boiler 29. Each miniature boiler shall be
be _ _excepting for horizontal return point _ _ of the height of the shell, D. external boiler equipped with a steam gage having a
tubular boilers having continuous lap except where the boiler is equipped with dial range not less than _ __ times
seams more than 3650 mm in length internal furnace. 24. A boiler removed from its original and not more than twice the maximum
where the factor of safety shall be. A. 1/2 setting and re erected at the same allowable working pressure.
A. 4.5, 9 B. 1/3 location or erected at a location without
B. 9, 4
A one
C . 1/4 change of ownership. B. 3
C. 5.5, 8 D. 2/5 A. PSME boiler C. 1.5
D. 8, 5
B. reinstalled boiler D. 4
19. Where ground water or surface C. fusion boi ler
14. Reinstalled or secondhand boilers water is used for feedwater, water D. unknown boiler 30. The maximum allowable working
shall have a minimum factor of safety of hardness of 0-10 ppm (parts per million)
pressure of a non-code steel or wrought
_ _ when the longitudinal seams are and alkalinity of be 25. As a general requirements, iron heating boiler of welded
of lap riveted construction, and a considered. smokestacks should be of sufficient construction shall not exceed _ _
minimum factor of safety Of _ _ when A. pH 10 to pH 11 capacity to hand le flue gases, self- A. 2 Mpa
the longitudinal seams are of butt and B. pH 9 to pH 10 supporting or guyed to withstand a wind B. 1 atm
double strap construction. C. pH 6 to pH 10 load 160 kph and rise at least _ _ mm C. 2 gage pressure
A. 6,5 D. pH 8 to pH 10 above the eaves of any building within a D. 1 bar
B. 5, 6 radius of 50 meters.
c. 4, 6 20. A closed vessel in which steam or A. 5000 3:1. A vessel permanently connected to
D. 6,4 other vapor is generated at a pressure B. 3000 a_ system by inlet and outlet pipes for
of more than 1.055 kg/crri2 gage by the C. 4000 storage' of liquid refrigerant.
15. The age limit of a horizontal return, direct application of heat. D. 10,000 A. liquid receiver
flue or cylinder boiler having a A power boiler B. accumu lator
longitudinal lap joint and operating at a B. boiler 26. As a general requirements , no C. container
pressure in excess of 0.345 Mpa shall C. fusion boiler smokestacks shoLJid be closer than D. duct
be _ _. D. external boiler _ _ mm from any exposed woodwork
A. 10 yrs or framing. 32. A device having a predetermined
B. 20 yrs 21. The heating surface area of A. 305 temperature fusible member for the
C. 15 yrs miniature boiler is 1.85 m2 and the B. 503 relief of pressures.
D. 30 yrs maximum allowable working pressure is C. 450 A. gage glass
_ _ kg/cm2 .
D. 1200 B. fusible plug
16. For power boilers, a reasonable A. 7.03 C. pressure gage
time for replacement shall be given at B. 9.85 27. Each miniature boiler, shall be D. stop valve
the discretion of the inspector not to C. 8.05 equipped with a sealed, spring-loaded,
exceed _ _year. D. 14.7 pop type safety valve not less than 33. A shut-off valve other than a valve
A. 1 _ _ pipe size, connected directly to for controlling the flow of refrigerant.
B. 4 22. Low pressure heating boiler is a the boiler. A. stop valve
c. 2 boiler. operated at pressure not A . .10.2 mm B. c heck valve
D. 3 exceeding 1.055 kg/cm 2 gage steam or B. 15.6 mm C. pressure valve
water temperature not exceeding C. 12.7 mm D. relief valve ·
17. For power boilers, each boiler shall _ _degC.
D. 25.4 mm
have ill least one safety valve and if it A. 100
has more than 46.5 sq. m of heating B. 121
23 - 4 I Day 23 - EXAM Day 23 - EXAM I 23 - 5
Power and Industrial Plant Engineering
34. A refrigeration system in which the 40. The carbon dioxide concentrated
refrigerant gas evolved in the when measured 910 mm above the floor 53. It is the reading of a liquid
evaporator is taken up in an absorber shall not exceed _ _ ppm. 47. Suitable means shall be provided manometer at the suction of the pump.
and released in a generator upon the A. 50 for the collection and disposal of A. suction lift
application of heat. B. 100 condensate from the equipment. The B. suction head
A absorption C. 75 condensate drain shall be at least _ _ C . static head
B flooded D. 150 nominal pipe size and shall be copper, D. total head
C brine spray galvanized steel, or other corrosion-
D cascade 41. A refrigeration system having two or resistant material. 54. Used to measure the pressure of
more refrigerant circuits. each with a A . 19 mm water discharging from a nozzle.
35. A refrigeration system in which only pressure-imposing element, condenser B. 25mm A. manometer
part of the refrigerant passing over the and evaporator, where the evaporator of C. 23 mm B. Pitot tube
heat transfer separated from the vapor one of the circuit cools the condenser of D. 90 mm C. Piezometer
and recirculated. another circuit. D. flow meter
A. flooded A. brine 48. An indirect refrigerating system
B. steam jet B. cascade employing water as the circulating 55. T~e level with respect to the pump,
C. mechanical C. mechanical system. of the body of water from which it takes
D. vapor D. none of these A. chilled water suction when the pump is not in
B. brine spray operation.
36. The temperature of the air to be 42. A s ubstance which produces a C. vapor A. static water level
used for comfort cooling shall be refrigerating effect by its absorption of D. heating system B. pumping water level
maintained at degree C. heat while expanding or evaporating. C. drawdown
A. 10- 15 A. refrigerant 49. This is the ratio of the actual weight D. dynamic level
B. 20-23.3 B. water of moisture to the weight of saturated
C. 15-20 C. air water vapor of mixture. 56. The vertical difference between the
D. 21.1 D. brine system A. humidity pumping water level and static water
B. density level.
37. The humidity of the air to be used 43. Any liquid cooled by the refrigerant C. specific weight A. total head
for comfort cooling shall be maintained and used for the transmission of heat D . specific gravity B. static head
at relative humidity. without a change in its state. C. · pump level
A . 55 to 60% A. mercury 50. A vessel in which vaporized D. · drawJdown
B. 60 to 65% B. water spray refrigerant is liquefied by the removal of
C. 50 to 55% C. brine heat. 57. An underground formation that
D. 70 to 75% D. water cooler A. condenser contains sufficient saturated permeable
B. evaporator material to yield significant quantities of
38. Refrigerant pipi ng crossing an open 44. The chemica, name of refrigerant C. compr~ssor water.
space which affords passageway in any 12: D. brine tank A. aquifer
building shall not be less than _ _ A. carbon dioxide B. wet pit
above the floor unless against the B. dichlorodifluoromethane 5 1. Science of water at rest. C. cavitation
ceiling of such space. C. dichloromethane A. hydrostatic D. NPSH
A 2290 mm D. methyl chloride B. hydrokinetics
B. 2920 mm C. hydraulic 58. Is the rate of flow of liquid measure
C 2570 mm 45. The chemical symbol of ammonia: D. hydrodynamic per unit of time.
D. 3450 mm A. SO A. capacity
B. NH3 52. Science of the force exerted by B. pump work
39. When refrigerant is added to a C. H20 water in motion. C. velocity
system, it shall be charge into the _ _ D. C4H1o A. hydrostatics D. pump head
pressure side of the system. B. hydrodynamics
A high 46. The chemical symbol of Freon 12: C. hydrokinetics 59. The reading of a pressure gage at
B. low A. CHCI2F D. hydraulic the discharge of the pump plus velocity
C. low and high B. CCbF2 head at the point of gage attachment.
D. low or high C. C3Hs A. total discharge head
D. C02 .
-"""'~'£ ~

23 - 6 I Day 23 - EXAM Day 23 - EXAM I 23 - 7


Power and Industrial Plant Engineering
B. total head 66.
Piping supports shall be placed
C. NPSH ____ mm intervals or less, 74. A fitting with a number of branches C. Class B
D. suction head A. 3000 in line connecting to smaller pipes. D. Class D
B. 4000 A. manifold
60. A type of valve installed between c. 4500 B. small pipe 81. An organized in-charge with the
the suction of pump and water mains to D. 6000 C. major pipe mission of fire prevention, fire
prevent any return of water to mains D. valve protection.
when pump is stopped. 67. Piping color for water. A. fire service
A. globe valve A. white 75. A large radius bend in a pipe line to B. fire building
B. suction valve B. green absorb longitudinal expansion in the line C. fire hose
c. discharge valve C. silver-gray due to heat. D. fire storage
D. check valve D. black A. expansion loop
B. expansion joint 82. Materials and their packaging which
61. Valve designed to open 68. Piping color for acids and alkalis: C. compression joint wil l neither ignite nor support
automatically to relieve excess A. violet D. compression loop combustion.
pressure. B. light blue A. non-combustibles
A. check valve C. light orange 76. For fire protection purposes, is an B. combustibles
B. relief valve D. yellow integrated system of one or more water C. moderate
C. gate valve supplies for fire use, underground and D. none of these
D. medium valve 69. Piping color for electricity: overhead piping design.
A. brown A. sprinkler system 83. The term designates commodities,
62. is a hollow product of B. black B. container system. packaging or storage aids which will not
round or any cross section having a C. safety red C. storage system ignite, burn or liberate flammable gases
continuous periphery. D. light orange D. fire protection system when heated to a temperature of 749
A. pipe degree for five minute. ·
B. tube 70. Piping color for air: 77. Class of fire involving ordinary A. non-combustibles
C. valve A. light blue combustible materials such as wood, B. combustibles
D. coupling B. yellow cloth, paper, rubber and plastics. C. medium
C. white A. ClassA D. high
63. A large pipe or drum into which D. orange B. Class D
each of a group of boilers is connected. C. Class B 84. Materials either of which will
A. muffler 71. Piping color for communication: D. Class E contribute fuel to fire.
B. header A. silver-gray A. moderate
C. manifold B. white 78. Class of fire in flammable liquids B. extra
D. expansion loop C. yellow and gases. C. combustibles
D. black A. Class A D. high
64. Medium pressure, when applied to B. Class C
valves and fittings, implies they are 72. Piping color for hazardous services: C. Class B 85. Type of fire extinguishers that is
suitable for a working pressure of from A. safety red D. Class E used by the occupants of a building
____ kPa. B. safety yellow primarily for immediate use of small
A. 862 to 1207 C. orange 79. Class of fire involving combustible fires.
B. 1207 to 1506 D. no color metals, such as magnesium, sodium, A. portable
C. 955 to 1345 titanium and other similar metals: B. outdoor
D. 1345to1750 73. Which of the following statements A. Class A C. indoor
for piping system is correct? B. Class C D. miniature
65. All piping shall be run to A. all piping to headers shall come from C. Class B
building walls. below rack. D. Class D 86. The amount of variation permitted in
A. opposite B. all piping from headers shall go up the part of total variation allowed in a
B. parallel above rack. 80. Class of fire involving energized · given dimension,
C. inclined C. All piping shall be reasonably electrical equipment. A. tolerance
D. slope cleaned before installation. A. Class A B. sensitivity
D. All of these B. Class C C. allowance
23 - 8 I Day 23 - EXAM Day 23 - Solution I 23 - 9
Power and Industrial Plant Engineering
D accuracy B. psychrometer
C. water meter the axis of flow and free to rotate about
87. The m1n1mum clearance space D. flow meter an axis set in jeweled bearings.
DAY 23 - SOLUTION
intended between the mating parts and A anemometers
represents the condition of tightest 94. A temperature measuring instrument B. electrical meters 1. Any boiler which does.exceed any of
possible fit. in which the voltage, generated at the. C . pilot tube the following: 405 mm inside diameter
A allowance junction of two dissimilar metal wires D. flow-nozzle and 1065 mm overall length of outside
B clearance indicates the degree of temperature. to outside heads at center.
C. space A thermocouple pyrometers 100. An instrument consist of a A portable boiler
D correction B. mercury thermometers rectangular head graduated in degrees B. miniature boiler *
C. electrical thermometer along a semici rcle , w ith a blade pivoted C. locomotive
88. The most generally used graduated D. oxygen bomb on the center pin, any angle from 0 to D. power boiler
measuring instrument for approximately 180 deg can be set.
determining linear dimensions: 95. A temperature measuring instrument A. protractor 2. A closed vessel intended for use in
A calipers in which there is an expansion or 'B . dividing head heating water or for application of hea
B. gages contraction of a liquid corresponding to C. calipers to generate steam or other vapor to be
C. level the changes in temperature. D. angle gage block used externally to itself.
D. rules A pyrometer A boiler

89. Directly reading calipers which are


B. mercury thermometer
C. liquid-filled glass
ra ) B. boiler or steam generator*
C. steam generator
accurate up to the thousandth of a D. industrial thermometer DAY 23 -ANSWER KEY D. none of these
centimeter. 11. A 16.A 31.A 46. 8
2 . 8 17. c

~
32. 8 47. A
A . vernier 96. Temperature measuring instrument 48.A
3. A vessel in which pres·sure is
. A 18. 8 33. A
B slide which the temperature is determined by . A 19. A 34.A 49. A obtained from an external sources, o
C. tapered matching the luminosity of the hot body. 5. C 20. A 35. A 50. A from an indirect application of heat.
D. dial A. radiation pyrometer ~ · A 21 . A 36. 6 51. A A. unfired p ressure vessel *
7. 0 22. 6 37. A 52. 6 B. pressure v essel
B. optical pyrometer 8. A 23. 8 38. A 53. A
90. A tool for checking the flatness of C . bourdon 19. B 24. 6 39. 6 54.6 C. power boiler
plane surfaces to tenths-of-thousands of D. electronic 10.8 25 A 40. B 55. A D. low pressure boiler
a centimeter. 11 .A 26. A 41. 6 56.0
A . dial indicator 12.A 27. C 42. A 57. A 4'. Any. boiler or unfired pressure vessel
97. Most useful close-tolerance 58. A
B. planimeter 13.A 28. C 43. C constructed, installed, placed in
measuring devices for quick and 14.A 29. C 44. 8 59. A
C. level accurate measurement to the thousands 15.0 30. 0 45. 8 60. D operation but subject to annual
D. trammels part of a centimeter. inspection.
A micrometer calipers ~ A. existing installations •
91 . Device used to determine whether B. dial B. new boilers
the part has been made to the tolerance C. balancer (cj ) C . hot water boiler
required. D. ultrasonic D. reinstalled boi lers
A gag~ DAY 23- ANSWER KEY
61 . B 76. A 91 . A 5. A boiler in which that has been
B. inside micrometer 98. This is used for speed measurement 62.8 77. A 92. A
C. level with indicating dials calibrated 63.8 78.C 93.A
inspected and declared unsafe to
D. none of these throughout the range 700 to 14 000 rpm 64. A 79. 0 94.A operate or disqualified stamped and
and especially valuable where it is 65. 6 80.6 95.C marked indicating its rejection.
92. Instrument that measures speed. 66. A 81 . A 96.6 A. second hand
convenient to make a connection or 67. B 82. 6 97. A
A tachometer contact with the rotating shaft 68.A 83.A 98. A
B. reinstalled
B. speedometer A stroboscope 69.0 84. A 99. A C. condemned *
C. counter B. counter and timer 70. A 85. A 100. A D. unfired
D. balancer C. speedometer 71 . B 86. A
72. B 87. A 6. No part of the steam generator
D. humeter 73. D 88. 0
93. Instrument to measure the relative should be closer than _ _ meter from
74. A 89. 0
humidity of the atmospheric air which is ~9. Instruments for measuring the flow 75. A 90. 6 any walls of the building.
important as comfort factor. A. one*
A. humeter
of gas or ai r consists of a set of rotating
vanes placed at an angle of 45 deg to ~ B. three
23 -10 1 Day 23- Solution Day 23 - Solution 1 23 - 11
C. two
Power and Industrial Plant Engineering
B. 855
D. 4 C. 755 B. two
D. 955 C. two o r more • 23. A boiler of which both the location
7. Steam generators should be mounted D. three o r five and ownership have been changed after
over a suitable foundations or concrete 13. The lowest factor of safety primary use.
pad of not less than _ _thick and with permissible on existing installations shall 18. For miniature boilers, the lowest A. internal boiler
sufficient base area. be _ _excepting for horizontal return permissible water level shall be at a B. secondhand boiler •
A 150 mm tubular boilers having continuous lap point _ _ of the height of the shell, C . surplus boiler
B. 255 mm seams more than 3650 mm in length except w here the boiler is equipped with D. external boiler
C. 195 mm where the factor of safety shall be. internal furnace.
D. 305 mm * A. 4.5, 9 * A. 1/2 24. A boiler removed from its original
B. 9, 4 B. 1/3 * setting and re erected at the same
8. When new boilers are installed in C. 5.5, 8 c. 1/4 location or erected at a location without
either existing or new buildings, a D. 8, 5 change of ownership.
D. 2/5
minimum height of at least _ _ shall A. PSME boiler
be provided between the top of the 14. Reinstalled or secondhand boilers 19. Where ground water or surface B. reinstalled boiler*
boiler proper and ceiling. shall have a minimum factor of safety of water is used for feedwater, water C. fusion boiler
A. 2130 mm * _ _ when the longitudinal seams are hardness of 0-10 ppm (parts per million) D. unknown boi ler
B. 900 mm of lap riveted construction, and a and alkalinity of be
C. 1560 mm minimum factor of safety Of _ _ when 25. As a general requirements,
considered.
D. 746 mm the longitudinal seams are of butt and A. pH 1 0 to pH 11 * smokestacks should be of sufficient
double strap construction. B. pH 9 ~o pH 10 capacity to handle flue gases, self-
9. When feeding hot water, feed pump A. 6,5. C. pH 6 to pH 10 supporting or guyed to withstand a wind
should at least be _ _ below hot well B. 5, 6 load 160 kph and rise at least _ _mm
D. pH8topH10
to prevent vapor lock. c. 4, 6 above the eaves of any building within a
A. 550 mm D. 6,4 radius of 50 meters.
20. A closed vessel in which steam or
B. 1220 mm * other vapor is generated at a pressure A. 5000 *
C. 990 mm 15. The age limit of a horizontal return, 2
B. 3000
of more than 1.055 kg/cm gage by the
D. 3050 mm flue or cylinder boiler having a C. 4000
direct application of heat.
longitudinal lap joint and operating at a A. power boiler * D. 10,QOO
10. A steel catwalk or platform at least pressure in excess of 0.345 Mpa shall B. boiler
_ _ wide and provided with standard be_ 26. As a general requirements, no
C. fusion boiler
handrails and toeboard on either side A 10yrs smokestacks should be closer than
D. external boiler
shall be installed across the tops of B. 20 yrs _ _ mm from any exposed woodwork
adjacent boilers. · C. 15 yrs or framing.
21. The heating surface area of
A 255 mm D. 30 yrs * miniature boiler is 1.85 m2 and the A. 305 •
B. 455 mm * maximum allowable working pressure is B. 503
C. 355 mm 16. For power boilers, a reasonable _ _kg/cm 2 . c. 450
D. 555 mm time for replacement shall be given at D. 1200
A. 7.03 *
the discretion of the inspector not to
B. 9.85
11 . For power boilers, when the tensile exceed _ _year. 27. Each miniature boiler, shall be
C. 8.05
strength of steel is not known, it shall be A. 1 * equipped with a sealed, spring-loaded,
D. 14.7
taken as _ _N/mm2 and 31 0 N/mm2 for B. 4 pop type safety valve not less than
wrought iron. C. 2 22. Low pressure heating boiler is a _ _ pipe size, connected directly ·to
A. 379 * D. 3 boiler operated at pressure not the boiler.
B. 450
exceeding 1.055 kg/cm 2 gage steam or A 10.2 mm
c. 398 17. For power boilers, each boiler shall water temperature not exceeding B. 15.6 mm
D. 521 have at least one safety valve and if it _ _degC. C. 12.7 mm *
has more than 46.5 sq. m of heating D. 25.4 mm
A 100
12. For power boilers, the resistance to surface or the generating capacity
B. 121 *
crushing of mild steel shall be taken at exceeds 91 0 kg/hr, it shall have _ _
_ _N/mm2 of cross sectional area. safety valves.
c. 111
D. 145
A. 655 * A one
23 -12 1Day 23 - Solution Day 23- Solution 123 -13
Power and Industrial Plant Engineering
28. Each miniature boiler shall be and released in a generator upon the
equipped with _ __ for application of heat. A. 50 47. Suitable means shall be provided
determination of water level. A. absorption * B. 100 * for the collection and disposal of
A. pressure gage B. flooded C. 75 condensate from the equipment. The
B. safety valve C. brine spray D . 150 COJ;Jdensate drain shall be at least
C. water gage g lass * D. cascade nominal pipe size and shall be ·copper,
D. fusible plug 41 . A refrigeration system having two or galvanized steel, or other corrosior,-
35. A refrigeration system in which only more refrigerant circuits, each with a resistant material.
29. Each miniature boiler shall be part of the refrigerant passing over the pressu re-imposing element, condenser A. 19 mm *
equipped with a steam gage having a hea~ transfer separated from the vapor and evaporator, where the evaporator of B. 25 mm
dial range not less than _ _ _ times and recirculated. one of the circuit cools the condenser of C. 23 mm
and not more than twice the maximum A. flooded* another circuit. D. 90 mm
allowable working pressure. B. steam jet A. brine
A. one C. mechanical B. cascade* 48. An indirect refrigerating system
B. 3 D. vapor C. mechanical employing water as the circulating
c. 1.5 * D. none of these system.
D. 4 36. The temperature of the air to be A. chilled water *
used for comfort cooling shall be 42. A substance which produces a B. brine spray
30. The maximum allowable wo rking rr]aintained at degree C. ·refrigerating effect by its absorption of C. vapor
pressure of a non-code steel or wrought A. 10-15 heat while expanding or evaporating. D. heating system
iron heating boiler of welded B. 20-23.3 * A. refrigerant *
construction shall not exceed c. 15-20 B. water 49. This is the ratio of·the actual weight
A. 2 Mpa D. 21.1 C. air of moisture to the weight of saturated
B. 1 atm D. brine system water vapor of mixture.
C. 2 gage pressure 37. The humidity of the air to be used A. humidity*
D. 1 bar • for comfort cooling shall be maintained 43. Any liquid cooled by the refrigerant '8. density
at relative humidity. and used for the transmission of heat C. specific weight
31. A vessel permanently connected to A. 55 to 60% • w ithout a change in its state. D. specific gravity
a system by inlet and outlet pipes for B. 60 to 65% A. mercury
storage of liquid refrigerant. C . 50 to 55% B. water spray 50. A vessel in which vaporized
A. l iquid receiver* D. 70 to 75% C. brine* refrigerant is liquefied by the removal of
B. accumulator D . water cooler heat.
C. container 38. Refrigerant piping crossing an open A. condenser*
D. duct space which affords passageway in any 44. The chemical name of refrigerant • B. evaporator
building s hall not be less than _ _ 12: C. compressor
32. A device having a predetermined above the floor unless against the A . carbon dioxide D. brine tank
temperature fusible member for the ceiling of such space. B. dichlorodifluoromethane *
relief of pressures. A. 2290 mm * C. dichloromethane 51. Science of water at rest.
A. gage glass B. 2920 mm D. methyl chloride A. hydrostatic *
B. f us ible plug • C. 2570 mm B. hydrokinetics
C. pressure gage D. 3450 mm 45 . The chemical symbol of ammonia: c., hydraulic
D. stop valve A. so D. hydrodynamic
39. When refrigerant is added to a B. NH3 *
33. A shut-off valve other than a valve system, it shall be charge into the _ _ C. H20 52. Science of the force exerted by
for controlling the flow of refrigerant. pressure side of the system. D. C4H1o water in motion.
A . s top va lve* A. high A. hydrostatics
B. check valve B. low* 46. The chemical symbol of Freon 12: B. hydrodynamics *
C . pressure valve C. low and high A. CHCbF C. hydrokinetics
D. reliefvalve D. low or high B. CCI2F2 * D. hydraulic
C. C3He
34. A refrigeration system in which the 40. The carbon dioxide concentrated D. C02 53. It is the reading of a liquid
refrigerant gas evolved in the when measured 910 mm above the floor manometer at the suction of the pump.
evaporator is taken up in an absorber shall not exceed _· __ ppm.
23 -141 Day 23- Solution Day 23 - Solution 1 23 - 15
Power and Industrial Plant Engineering
A. suction lift * 60. A type of valve installed between
B. suction head the suction of pump and water mains to C. 4500 A. manifold *
C. static head prevent any return of water to mains D. 6000 B. small pipe
D. total head when pump is stopped. C. major pipe
A. globe valve 67 . Piping color for water. D. valve
54. Used to measure the pressure of B. suction valve A. white
water discharging from a nozzle. C. discharge valve B. green* 75. A large radius bend in a pipe line to
A. manometer D. check valve * C. silver-gray absorb longitudinal expansion in the line
B. Pitot tube * D. black due to heat.
C. Piezometer 61. Valve designed to open A. expansion loop *
D. flow meter automatically to relieve excess 68. Piping color for acids and alkalis: B. expansion joint
pressure. A. violet* C. compression joint
55. The level with respect to the pump, A. check valve B. light blue D. compression loop
of the body of water from which it takes B. relief valve * C. light orange
suction when the pump is not in C. gate valve D. yellow 76. For fire protection purposes, is an
operation. D. medium valve integrated s~·stem of one or more water
A. static water level * 69. Piping color for electricity: supplies for fire use, underground and
B. pumping water level 62. is a hollow product of A. brown overhead piping design.
C. draw down round or any cross section having a B. black A. sprinkler system *
D. dynamic level continuous periphery. C. safety red B. container system.
A. pipe D. light orange * C. storage system
56. The vertical difference between the B. tube* 1 D. fire protection system
pumping water level and static water C. valve 70. Piping color for air:
level. D. coupling A. light blue * 77. Class of fire involving ordinary
A. total head B. yellow combustible materials such as wood,
B. static head 63. A large pipe or drum into which C. white cloth, paper, rubber and plastics.
C. pump level each of a group of boilers is connected. D. orange A. Class A*
D. draw-down * A. muffler B. Class D
B. header* 71 . Piping color for communication: C. Class B
57 . An underground formation that C. manifold A. silver-gray D. Class E
contains sufficient saturated permeable D. expansion loop B. white*
material to yield significant quantities of C. yellow 78. Class of fire in flamt!Jable liquids
water. 64. Medium pressure, when applied to D. black and gases.
A. aquifer* valves and fittings, implies they are A. Class A
B. wet pit suitable for a working pressure of from 72. Piping color for hazarrl::lus services: B. Class C
C. cavitation ..,.---o-:,.....,- kPa. A. safety red C. Class B *
D. NPSH A. 862 to 1207 * B. safety yellow * D. Class E
B. 1207 to 1506 C. orange
58. Is the rate of flow of liquid measure C. 955 to 1345 D. no color 79. Class of fire involving combustible
per unit of time. D. 1345 to 1750 metals, such as magnesium. sodium,
A. capacity * 73. Which of the following statements titanium and other similar metals.
B. pumpwork 65. All piping shall be run to for piping system is correct? A. Class A
C. velocity building walls. A. all piping to headers shall come from B. Class C
D. pump head A opposite below rack. C. Class B
B. parallel* B. all piping from headers shall go up D. Class D *
59. The reading of a pressure gage at C. inclined above rack.
the discharge of the pump plus velocity D. slope C. All piping shall be reasonably 80. Class of fire involving energized
head at the point of gage attachment. cleaned before installation. electrical equipment.
A. total discharge head * 66. Piping supports shall be placed D. All of these * A. ClassA
B. total head ...,.--___,. mm intervals or less. B. Class C *
C. NPSH A. 3000 * 74. A fitting with a number of branches C. Class B
D. suction head B. 4000 in line connecting to smaller pipes. D. Class D
~

23 ~ 16 1 Day 23- Solution Day 23 ~Solution I 23 -17


81. An organized in-charge with the represents the condition of tightest Power and Industrial Plant Engineering
mission of fire prevention, fire possible fit.
94. A temperature measuring instrument A. anemometers *
protection. A. allowance "
in which the voltage, generated at the B. electrical meters
A. fire service * B. clearance junction of two dissimilar mf!tal wires
C . space C. pilot tube
B. fire building indicates the degree of temperature. ·
D. correction D. flow-nozzle
C. fire hose A. thermocouple pyrometers "'
D. fire storage B. mercury thermometers
88. The most generally used graduated 100. An instrument consist of a
C. electrical thermometer rectangular head graduated in degrees
82. Materials and their packaging which measuring instrument for approximately
D. oxygen bomb along a semicircle, WJth a blade pivoted
will neither ignite nor support determining linear dimensions:
A. calipers on the center pin, any angle from 0 to
combustion.
95. A temperature measuring instrument 180 deg can be set.
A. non-combustibles B. gages in which there is an expansion or
C. level A. protractor *
B. combustibles * contraction of a liquid corresponding to
D. rules" B. dividing head
C. moderate the changes in temperature. C. calipers
D. none of these A. pyrometer
89. Directly reading calipers which are D. angle gage block
accurate up to the thousandth of a B. mercury thermometer
83. The term designates commodities,
C. liquid-filled glass •
packaging or storage aids which will not centimeter.
D. industrial thermometer
ignite, burn or liberate flammable gases A. vernier
when heated to a temperature of 749 B. slide
96. Temperature measuring instrument
degree for five minute. C. tapered
D. dial* which the temperature is determined by
A. non-combustibles *
matching the luminosity of the hot body.
B. combustibles
90. A tool for checking the flatness of A radiation pyrometer
C. medium B. optical pyrometer •
D. high plane surfaces to tenths-of-thousands of
C. bourdon
a centimeter.
D. electronic
84. Materials either of which will A. dial indicator
contribute fuel to fire. B. planimeter*
97 . Most useful close-tolerance
A. moderate * C. level
measuring devices for quick and
B. extra D. trammels
accurate measurement to the thousands
C. combustibles
91 . Device used to determine whether part of a centimeter.
D. high
the part has been made to the tolerance A. micrometer calipers *
B. dial
85 .• Type of fire extinguishers that is required.
A. gages" C. balancer
used by the occupants of a building
B. inside micrometer D. ultrasonic
primarily for immediate use of small
fires. C. level
I A. portable *
a: outdoor
D. none of these 98. This is used for speed measurement
with indicating dials calibrated
92. Instrument that measures speed. throughout the range 700 to 14 000 rpm
C. in!,:loor
A. tachometer" and especially valuable where it is
D. miniature
B. speedometer convenient to make a connection or
C. counter contact with the rotating shaft.
86. The amount of variation permitted in
D. balancer A . stroboscope •
the part of total variation allowed in a
B. counter and timer
given dimension.
93. Instrument to measure the relative C. speedometer
A . tolerance *
humidity of the atmospheric air w hich is D. humeter
B. sensitivity
C. allowance important as comfort factor.
A. humeter* 99. Instruments for measuring the flow
D. accuracy
B. psychromE!lter of gas or air consists of a set of rotating
C. water meter vanes placed at an angle of 45 deg to
87. The mmtmum clearance space
D. flow meter the axis of flow and free to rotate about
intended between the mating parts and
an axis set in jeweled bearings.
Day 24 - EXAM l 24 - 1
Power and Industrial Plant Engineering

DAY 24- EXAM is the discharge of pump 2, where Qz <


p1 , what is the discharge?
A. Q,
1. If Q' is the actual discharge flow of a
B. Q, + 0 2
nozzle and Q is the theoretical C. 02
discharge flow, what will be the D. Q,-Q2
coefficient of discharge equal to?
A. Q /Q'
B. Q'/Q 7. Two pumps are connected in series,
if Q, is the discharge of pump 1 and 02
C. Q' xQ
is the discharge of pump 2, where 02 >
D. 1- Q /Q '
Q,, what is the discharge?
A. Q,
2. Two pumps are connected in parallel,
B. Q, + 02
if h, is the head of pump 1 and h2 is the C. 02
head of pump 2, where h, = h2 , what is D. Q,-Qz
the total head?
A. h,/h2
8. Using the Psychrometric Chart, what
B. h, + h2
is the enthalpy at dry bulb temperature
c. h,
of 308 deg K and RH of 40%?
D. h,- h2
A. 70.6
B. 74.6
3. Two pumps are connected in parallel, C. 71 .6
if Q 1 is the discharge of pump 1 and 02 D. 75.6
is the discharge of pump 2 , where 02 >
Q 1, what is the discharge?
9. Using the Psychrometric Chart, what
A. Q,
is the wet-bulb temperature of air
B. Q, + 0 2
contained in a room at a temperature of
c. Q2
308 deg K, RH = 40% and a pressure of
D. Q,-Q2
1 atmosphere?
~
A . 24 deg K
4. Two pumps are connected in parallel, B. 297 deg K
if Q , is the discharge of pump 1 and 0 2 C . 350 deg K
is the discharge of pump 2, where 02 < D. 290 deg K
Q, , what is the discharge?
A. Q, 10. Using the Psychrometric Chart, find
B. a,+ a2 the dew point temperature of air
C. 02 contained in a room at a temperature of
D. Q, - 02
308 deg K, RH = 40% and a pressure of
1 atmosphere.
5. Two pumps are connected in series, A. 292.5 deg K
if H, is the head of pump 1 and H2 is the B. 290.5 deg K
head of pump 2, where H2 < H 1, what is C. 288.5 deg K
the :otal head?
D. 294.5 deg K
A. H,
B. rl , + H2 11 . Using the Psychrometric Chart,
C. H2 atmospheric air at 308 deg K dry bulb
D H,- H2 and 293 deg K dew point, what is the
relative humidity?
6. Two pumps are connected in series , A. 42%
if Q, is the discharge of pump 1 and Q 2 B. 50%

REFRESHER MANUAL 2nd Edition by JAS TORDILLO


24 - 4 I Day 24 - EXAM Day 24 - Solution 124 - 5
C. otto cycle
Power and Industrial Plant Engineering
50°C. What is the amount of the heat
D. hussein cycle rejected? B. 978 m/s
A. 160.4 kJ C. 145 m/s CAY 24- SOLUTION
38. In a flow of fluids through channel B. 182.5 kJ D. 1020 m/s
wherein the velocity changes along the · C. 171.5 kJ 1. If Q' is the actual discharge flow of a
length of a channel from point to point D. 195.5 kJ 49. A high temperature source at 900°K nozzle and Q is the theoretical
on the account of changing its depth, provides 400 kJ of heat to an engine. discharge flow, what will be the
width and direction of flow. 44. An engine gains 550 kJ of heat from The engine has a net work of 220 kJ coefficient of discharge equal to?
A. constant flow a hot source at 700°C and rejected at and the temperature sink is 300°K. A. Q/Q'
B. unsteady flow 50°C. What is the amount of power Compare the thermal efficiency Et of B. Q'/Q*
C. nonuniform flow output? this engine to the thermal efficiency Ecc C. Q'xQ
D. uniform flow A. 260.4 kJ of the Carnot cycle. D. 1-QJQ'
B. 367.5 kJ A. Et = Ecc
39. Which of the following is c. 271.5 kJ B. Et < Ecc 2. Two pumps are connected in parallel,
compressed in a diesel engine? D. 495.5 kJ C. Et > Ecc if h1 is the head of pump 1 and h2 is the
A. fuel only D. Unity head of pump 2, where h1 = h2, what is
B. air only 45. The energy input to the turbine is the total head?
C. fuel or air 2800 kJ/kg and the energy at the exit is 50. In a power piping system, the A. h1/h2
D. fuel and air 1400 kJ/kg, efficiency of the engine is corresponding piping color for safety B. h1 + h2
75% and power output at full load is and proper fluid identification for other c. h1 *
40. If Y is the efficiency and E is the 6000 kw. What is the turbine flow rate fluids, including drainage pipes is D. h1- h2
maximum energy attained and ~ is the at full load in kg/kw-hr?
actuai energy, what is the formula of A. 3.43 A. green 3. Two pumps are connected in parallel,
actual energy? B. 5.97 B. violet if Q 1 is the discharge of pump 1 and 02
A. EY c. 4.33 C. safety red is the discharge of pump 2, where Q2 >
B. EN D. 9.23 D. black 01, what is the discharge?
C. Y/E A. Q1
D. Y+ E 46. A steam reciprocating pump has a B. Q1+~*

41. If Y is the efficiency and E is the


nameplate of 6 in x 4 in x 6 in. What is
the diameter of the liquid piston? ra ) c. Q2
D. 01-02
maximum energy attained and ~ is the I( 6 in' DAY 24- ANSWER KEY
actual energy, what is the formula for B. 4 in 1. 8 16. 0 31. 846. 8 4. Two pumps are connected in parallel,
the input energy? C. 12in 2. C 17. A 32. C 47. A if 01 is the discharge of pump 1 and 02
A. eY 3. 8 18. 0 33. C 48. A
D. 2 in 4.8 19.8 34.049. 8
is the discharge of pump 2, where 02 <
B. eN 5. B 20. 8 35. B 50. 0 01, what is the discharge?
C. EN 47. If y is height between elevation in 6. A 21. B 36. A A. 01
D. Y +e feet, Q is the volume of fluid in 7. C 22. A 37.A B. Q1+ Q2*
8. c 23. c 38. c
gallons/min and d is the weight of fluid in
9. B 24. 0 39. B
c. Q2
42. The latent heat of vaporization lb/gal. What is the energy E required to 10.A 25. A 40. A D. 01-02
decreases as the pressure and raise the given volume of fluid per 11.A 26. A 41. B
temperature of the liquid incr~ases , if minute. 12.C 27. B 42. A 5. Two pumps are connected in series,
both pressure and temperature A. E =Qdy 13.0 28. A 43. B if H1 is the head of pump 1 and H2 is the
increases, the latent heat vaporization B. E = Qy/d 14.A 29. A 44. 8 head of pump 2, where H2 < H1, what is
15.0 30. B 45. A
C. E = y/Qd the total head?
A.
B.
decreases
constant
D. E= (Q + y)/d G A. H1
B. H1 +Hz*
C. increases 48. In a certain process, energy entering C. H2
D. zero is 1500 kJ/kg and 150 m/s. If the D. H1- H2
energy leaving is 1200kJ/kg, determine
43. An engine gains 550 kJ of heat from the velocity at the exit? 6. Two pumps are connected in series,
a hot source at 700°C and rejected at A. 789 m/s if 01 is the discharge of pump 1 and 02

REFRESHER MANUAL 2nd Edition by JAS TORDILLO REFRESHER MANUAL 2nd ·e~ltion by JAS TOROILLO
24-61 Day 24- Solution Day 24 - Solution 124 - 7
is the discharge of pump 2, where a2 < Power and Industrial Plant Engineering
C. 38%
a,, what is the discharge? D. 65% B. wetwell 24. For supersonic and subsonic
A. a,• C. weir divergent nozzle, what is the Mach
B. Q,+ 02 12. Using the Psychrometric Chart, D. water table
C. a2 . number for hypersonic:
atmospheric air at 308 deg K dry bulb A. less than 1
D. a,-a2 and 293 deg K dew point, what is the 18. What would be the velocity at all B. greater than 1
constant pressure saturation points in the pipe of a uniform flow? C. equal to 1
7. Two pumps are connected in series, temperature? · A. unity D. extremely high values *
ifa, is the discharge of pump 1 and a2 A. 290 deg K B. steady
a
is the discharge of pump 2, where 2 > B. 303 deg K C. constant 25. It is defined as the ratio of the fluid
a,, what is the discharge? C. 293 deg K * D. equal* velocity to the velocity of a sound wave.
A. a, D. 308 deg K A. Mach Number*
B. a,+ a2 19. In a uniform flow, the velocities at all B. Reynolds Number
c. Q2* 13. Using the Psychrometric Chart, points are _ _ __ C. Newtonian Number
D. a,-a2 atmospheric air at 306 deg K dry bulb A. constant D. Velocity Factor
and 293 deg K dew point, what is the B. equal*
8. Using the Psychrometric Chart, what specific volume in m3/kg? C. increasing 26. In steady flow of fluids, velocity at a
is the enthalpy at dry bulb temperature A. 0.75 D. decreasing section at the streamlines remains
of 308 deg K and RH of 40%? B. 0.85
A . 70.6 c . 0.81 20. If A is the cross section of the flow A. uniform*
B. 74.6 D. 0.89 * and Pw is the wetted perimeter, then the B. equal
c. 71.6 * hydraulic head, Hd, is equal to: C. Ljnsteady
D. 75.6 . 14. Using the Psychrometric Chart, A. Pw/A D. constant
atmospheric air at 308 deg K dry bulb B. A/Pw*
9. Using the Psychrometric Chart, what and 293 deg K dew point, what is the C. PwxA 27. In steady flow of fluids, the diameter
is the wet-bulb temperature of air enthalpy? D. Pw-A of a pipe in any section is _ _ __
contained in a room at a temperature of A. 74 kJ/kg * A. uniform
308 deg K, RH = 40% and a pressure of B. 60 kJ/kg 21. For supersonic and subsonic B. the same*
1 atmosphere? c..78 kJ/kg divergent nozzle, what is the Mach C. constant
A . 24 deg K
B. 297 deg K"
Q. 68 kJ/kg number for supersonic:
A. less than 1
.
D. nonuniform
C. 350 deg K 15. Using the Psychrometric Chart, what B. greater than 1* 28. The flow of liquid at a constant rate
D. 290 deg K is the relative humidity of saturated air at C. equal to 1 through a conical pipe is ____.
313 deg K? D. i1:ero A. steady nonuniform *
10. Using the Psychrometric Chart, find A. zero B. steady uniform
the dew point temperature of air B. 50% 22. For supersonic and subsonic C. unsteady nonuniform
contained in a room at a temperature of c. 20% divergent nozzle, what is the Mach D. unsteady uniform
308 deg K, RH = 40% and a pressure of D. 100% * number for subsonic:
1 atmosphere. A. less than 1 * 29. In uniform flow, the velocity at any
A. 292.5 deg K * 16. What is the temperature range of air section the pipe is _ _ __
B. greater than 1
B. 290.5 deg K in an air conditioning application where C. equal to 1 A. equal*
C. 266.5 deg K in that range dry air can be considered D. zero B. unity
D. 294.5 deg K as an ideal gas?
C. steady
A. 100- 125 deg C 23. For supersonic and subsonic D. constant
11. Using the Psychrometric Chart, B. 75-100 deg C divergent nozzle, what is the Mach
atmospheric air at 306 deg K dry bulb C. so - 75 deg C number if the flow is sonic: 30. Fluids that has constant density is
and 293 deg K dew point, what is the b. 10-50degC" A. less than 1 considered as:
relative humidity? B. greater than 1 A. compressible fluids
A. 42% * 17. In hydraulics, the length of the C. equal to 1 * B. incompressible fluids *
B. 50% channel which lias water contact. D. zero C. real fluids
A. wetted perimeter * D. ideal fluids

REFRESHER MANUAL 2nd Edition by JAS TORDILLO REFRESHER MANUAL 2nd Edition by JAS TORDILLO
24 • 8 I Day 24 - Solution Day 24 - Solution I 24 - 9
Power and Industrial Plant Engineering
31 . At 1 atmosphere, the boiling point of C. otto cycle
water is 100°C. If the pressure is D. hussein cycle save. What is the amount of the heat 46. A steam reciprocating pump has a
decreased, the boiling temperature will: nameplate of 6 in x 4 in x 6 in. What is
rejected?
A . increase 38. In a flow of fluids through channel the diameter of the liquid piston?
A. 160.4 kJ
B. decrease • wherein the velocity changes along the B. 182.5 kJ * A. 6 in
C. remains the same length of a channel from point to point C. 171.5 kJ B. 4 in •
D. grater than on the account of changing its depth, C. 12 in
D . 195.5 kJ
width and direction of flow. D. 2 in
QA =t>.S(T1)
32. What is the pressure above zero? A. constant flow
550 =t>.S(700 + 273)
A. gage pressure B. unsteady flow For duplex pump:
B. vacuum pressure C. nonuniform flow • t>.S = 0.5652 kJ/K
C. absolute pressure * D. uniform flow 6 in x 4in x 6in
D. atmospheric pressure =
QR L\S(T2)
39. Which of the following is QR = 0.5652(50 + 273)
33. Thermodynamic process following compressed in a diesel engine? =
QR 182.5 kJ
the law PV" = constant. A. fuel only
A. isentropic B. air only • 44. An engine gains 550 kJ of heat from
B. adiabatic C. fuel or air a hot source at 700°C and rejected at
C. polytropic* D. fuel and air 50°C. What is the amount of power
output? 47. If y is height between elevation in
D. isobaric feet, Q is the volume of fluid in
40. If Y is the efficiency and E is the A. 260.4 kJ
B. 367.5 kJ *
gallons/min and d is the weight of fluid in
34. Thermodynamic process following maximum e nergy attained and ~ is the lb/gal. What is the energy E required to
the law PV" = constant if n is equal to 1, actual energy, what is the formula of C. 271.5 kJ
D. 495.5 kJ raise the given volume of fluid per
what is the process? actual energy? minute.
A. isentropic A. EY* A. E = Qdy*
B. adiabatic B. ElY WN = QA - QR
WN = 550 - 182.5 B. E = Qy/d
C. polytropic C. Y/E C. E = y/Qd
D. isothermal * D. Y+E WN = 367.5 kJ
D. E= (Q + y)/d
35. Thermodynamic process following 41. If Y is the efficiency and E is the 45. The energy input to the turbine is
2800 kJ/kg and the energy at the exit is 48. In a certain process, energy entering
the law PV" = constant if n is equal to 0, maximum energy attained and ~ is the is 1500 kJ/kg and 150 m/s. If the
what is the process? actual energy, what is the formula for 1400 kJ/kg, efficiency of the engine is
75% and power output at full load is energy leaving is 1200kJ/kg, determine
A . isentropic the input energy? the velocity at the exit?
B. isobaric • A. eY 6000 kw. What is the turbine flow rate
at full load in kglkw-hr? A. 789 m/s *
C. polytropic B. elY* B. 978 m/s
D. isothermal C. ElY A. 3.43 *
B. 5.97 C. 145 m/s
D. Y +e D. 1020 m/s
36. Thermodynamic process following C. 4.33
the law PV" = constant if n is equal to 42. The latent heat of vaporization D. 9.23
v2 v2
1.4, what is the process? decreases as the pressure and HI = 2(1doo) = hl + 2( 1~)
A. isentropic * temperature of the liquid increases, if Wr =m(h1 - h2l11r
B. isobaric both pressure and temperature 6000 = m(2800- 1400) 0.75 )2 y2
m = 5.714 kg/sec 1500 + ~ = 1200 + - -2
C. polytropic increases, the latent heat vaporization 2(1000) 2( 1000)
D. isothermal
Turbine flow rate
v2 = 789 m/s
A. decreases "
3600 sec
37. Which of the following cycle is for B. con$tant - 5 . 714 kg 49. A high temperature source at 900°K
- ~:; sec hr
compression ignition reciprocating C. increases Wr 6ooo kg provides 400 kJ of heat to an engine.
engine? D. zero The engine has a net work of 220 kJ
A. diesel cycle • and the temperature sink is 300°K.
B. gas cycle 43. An engine gains 550 kJ of heat from
=3.428 kg
kw-hr Compare the thermal efficiency Et of
a hot source at 700°C and rejected at

REFRESHER MANUAL 2nd Edition by JAS TORDILLO REFRESHER MANUAL 2nd Edition by JAS TORDILLO
24 - 10 1 Day 24 - Solution Day 25 - EXAM I 25 - 1
Power and Industrial Plant Engineering
this engi11e to the thermal efficiency Ecc
of the Carnot cycle. C. inches
A. Et = Ecc
DAY 25- EXAM D. feet
B. Et < Ecc *
C. Et > Ecc 1. All proposed installations, additions or 6. In the standard symbols, which of the
D. Unity alterations involving machinery or following symbols is the same as the
process shall be covered by plans and symbol for brick and stone masonry?
specifications prepared by or under the A. wood
E, = V'il = 220 =0.55 supervision of a signed and
~ 400 B. concrete
sealed. C. cast and malleable iron
Ecc = 900-300 = 0.666 A. PME
900 D. rock
B. CPM
C. RME 7. In the standard symbols, which of the
Therefore, E, < Ecx: D. PME and RME following symbols is the same as the
symbol for bronze, brass and copper?
50. In a power piping system, the 2. In the requirement for permit A. firebrick
corresponding piping color for safety application, general layout plan for each
and proper fluid Identification for other R water
floor drawn to scale not less than C. earth
fluids, including drainage pipes is ____ , in heavy lines the equipment D. sand
with super-imposed building outline in
A. green light or suppressed lines.
B. violet
8. In the standard symbols. which of the
A . 1:20 following symbols is the same as the
C. safety red B. 1:200
D. black* symbol for sound insulation?
C. 1:100 A. whit~ metal
D. 1;.1000 B. brick and stone masonry
C. rubber
3. In the requirement for permit D. wood and grain
application, the mechanical plans shall
contain the signature and 13eal of a 9. In the standard symbols, which of the
Professional Mechanical Engineer .with following is the symbol for rubber,
the foilowing: Registration number, plastic and electrical insulation? Refer
validity date, PTR and _ _ __ to the attached drawings.
A. date of birth A. symbol A
B. serial number B. symbol F
C. birthplace C. symbol C
D. Tax Identification Number D. symbols

4. Some of the requirements for 10. In .the standard symbols, which of


mechanical permit applications are the following is the symbol for earth?
General layout plan for each floor, Refer to the attached drawings.
piping plan, detailed plans of A. symbol B
foundations and _ _ _. B. symbol 0
A. detailed construction plans C. symbol J
B. location plan D. symboi T
C. flow-sheet
D. all of these 11. In the standard symbols, which of
the following is the symbol for water and
5. In the standards for drawings on other liquids? Refer to the attached
mechanical drawings, all dimensions are drawings.
in general, given in _ _ _. A. symbol E
A. millimeters B. symbol M
B. meters
REFRESHER MANUAL 2nd Edition by JAS TORDILLO REFRESHER MANUAL 2nd Edition by JAS TORDILLO
25 - 2 I Day 25 - EXAM Day 25 - EXAM I 25 - 3
Power and Industrial Plant Engineering
C. symbol H C. one person per 10 m
D. symbol R D. one person per 18 m3 least _ _ from upper surface of the be at least 1120 mm away from all
top rail to floor level. · obstruction except hand rails.
12. In the standard symbols, which of 18. As a general requirement for A. 1000mm A 200 mm •
the following is the symbol for concrete? manholes, manhole floor opening shall B. 2000 mm B. 500 mm
Refer to the attached drawings. be manhole covers of C. 1300 mm C. 400mm
A symbol L adequate strength which which need not D. 2300 mm D. 900 mm
B. symbol C be hinged.
C. symbol! A. with no 23. As a general requirement for 28. As a general requirement for stairs,
D. symbol Q B. may be provided with railings, standard railings shall have no stairway shall have a height of more
C. provided with posts not more than _ ._ apart and an than _ _ between landings, and
13. In the standard symbols, which of D. no intermediate rail halfw~y between top intermediate landings shall have
the following is the symbol for wood-with rail and the floor. , dimensions of not less than 1120 mm
grain? Refer to the attached drawings. 19. As a general requirement for floor A. 1000 mm measured in the direction of the run.
A. symbol T openings, floor openings into which B. 2500 mm A. 1120 mm
B. symbol I person can accidentally walk shall be C. 2000 mm B. 2200 mm
·C. symbol E guarded by covers securely held · in D. 3000 mm C. 1750 mm
D. symbol B place and leaving no openings more D. 2750 mm
than _ _ in width. 24. As a general requirement for
14. In the standard for drawings, all A. 25 mm railings, for pipes railings the top rails 29. As a general requirement for
borders should be at least1 0 mm from B. 75 mm and posts of metal pipe of at least machinery and equipment foundation,
the sheet edge; and all tit!e blocks shall C. 50mm _ _ diameter and intermediate rails of the weight of the machine plus the
be located at. the corner D. 150 mm metal pipe at least 25 mm diameter. weight of the foundation should be
inside borders for larger sheets. A. 20mm distributed over a sufficient soil area
A. upper right hand corner 20. As a general requirement for wall B. 30 mm which is large enough to cause a
B. lower right hand corner openings, all wall openings less than C. 25mm bearing stress within the safe bearing
C. upper left hand corner 1000 mm from floor having a height of at D. 45 mm capacity of soil with a factor of safety of
D. lower left hand corner least750 mm and width of 450 mm from
there is a drop of more than _ _ shall 25. As a general requirement for A. two
15. Standard dimensions of a little block, be solidly enclosed or guarded by fixed railings, toe boards shall be at least 150 B. five
length x width, is: rails. mm in height and shall be securely C. four
A. 150 mm x 50 mm A. 1000 mm fastened in place with not more than D. ten
B. 100 mm x 50 mm B. 3000 mm clearance above floor level.
C. 150 mm x 75 mm C. 2000 mm A. (j mm 30. Foundation bolts of specified size
D. 100 mm x 75 mm D. 4500 mm B. 50 mm should be used and surrounded by a
C. 10 mm pipe sleeve. No foundation bolts shall ·
16. As a general space requirement for 21. As a general requirement, all other D. 100 mm be less than in diameter.
work rooms shall be at least in wall opening, irrespective of their width A. 10 mm
height from floor to ceiling. shall, if their lower edge is either 80 mm 26. As a general requirement for stairs, B. 15 mm
A. 1m or less above floor level on the near side all stairs, platforms, and landings shall C. 12 mm
B. 3m or _ _ mm or above ground, or floor be of sufficient strength to sustain safely D. 30 mm
C. 2m level on the far side, be guarded by a a llve load of not less than 500 kg with a
D 2500 mm toeboard. factor safety of _ _. 31 . Any boiler which does exceed any
A. 500 mm A. ten of the following: 405 mm inside diameter
17 As a general space requirement, the B. 2000 mm B. five and 1065 mm overall length of outside
C. 1000 mm 2
maximum number of persons working or C. eight of heads at center, 1,85 m of water
2
will be working shall not exceed D. 2500 mm D. four heating surface and 7.03 kg/cm
maximum allowable working pressure.
A one person per 8 m 3 22. As a general requirement for 27. As a general requirement for stairs, A. portable boiler
B. one person per 12 m3 railings, standard railings shall be at width of stairs except service stairs, B. miniature boiler

l
shall be not less than _ _ and should

REFRESHER MANUAL 2nd Edition by JAS TORDILLo REFRESHER MANUAL 2nd Edition by JAS TORDILLO
25 - 4 I Day 25 - EXAM Day 25 - EXAM I 25 - 5
Power and Industrial Plant Engineering
C. locomotive A. 150 mm
D. power boiler B. 255mm 43. For fire tube boilers, the lowest A. one
C. 195 mm factor of safety permissible on existing B. two
32. A closed 'essel intended for use in D. 305 mm installations shall be _ _ excepting C. two or more
heating water or for application of heat for horizontal return tubular boilers D. three to five
to generate steam or other vapor to be 38. When boilers are replaced or new having continuous lap seams more than
used externally to itself. boilers are installed in either existing or 3650 mm in length where the factor of 48. For fire tube boilers, the use of
A. boiler new buildings, a minimum of at least safety shall be _ _ . weighted-lever safety valves shall be
B. boiler or steam generator _ _ shalf be provided between the A. 4.5, 9 prol'libited and type valves
C. steam generator top of the boiler proper and ceiling. B. 9, 4 shall replace these valves.
D. none of these A. 2130 mm C. 5.5, 8 A. disc type
B. 900mm D. 8,5 B. swing type
33. A vessel in which pressure is C. 1560 mm C. hammer lever type
obtained from an external sources, or D. 746 mm 44. For fire tube boi lers, reinstalled or D. direct spring-loaded type
from an indirect application of heat. secondhand boilers shall have a
A unfired pressure vessel 39. For fire tube boilers, the age limit of minimum factor of safety of __ · _ when 49. For fire tube boilers, each boiler
B. pressure vessel horizontal return tubular, flue of cylinder the longitudinal seams are of lap riveted shall have the _ _or more gage
C. power boiler boiler shall be construction, and a minimum factor of cocks, located within the range of the
D. low pressure boiler A. 10 years - - safety of _ _when the longitudinal visible length of the water glass.
B. 30 years seams are of butt _and double strap A. one
34. Any boiler or unfired pressure C. 20 years construction. B. three
vessel constructed, installed, placed in D. 45 years A. 6, 5 C. two
operation but subject to periodic B. 5, 6 D. five
inspection 40. A steel catwalk or platform at least C. 4, 6
A. existing installations _ _ wide and provided with standard D. 6, 4 50. For miniature boilers, the lowest
B. new boilers handrails and toeboard on either side permissible water level shall be at a
C. hot water boiler shall be installed across the tops of 45. For fire tube boilers, the age limit of point _ _of the height of the shell,
D. reinstalled boilers adjacent boilers. a horizontal return, flue or cylinder boiler except where the boiler is equipped with
A. 255 mm having a longitudinal lap joint and internal furnace.
35. A boiler in which that has been B. 455mm operating at a pressure in excess of A. 1/2
inspected and declared unsafe to C. 355mm 0.345 Mpa shall be _ _. B. 1/3
operate or disqualified stamped and D. 555mm A. 10 yrs C. 1/4
marked indicating its rejection by B. 20 yrs D. 2/5
qualified inspecting body. · 41. For fire tube boilers, when the C. 1'5 yrs
A. second hand tensile strength of steel is not known, it D. 30 yrs 51. Where ground water or surface
B. reinstalled . shall be taken as _ _N/mm2 and 310 water are used for feedwater, water
C. condemned N/mm2 forwrought iron. 46. For fire tube boilers, a reasonable hardness of 0-10 ppm (parts per million)
D. unfired A. 379 time for replacement shall be given at and alkalinity of_ be considered.
B. 450 the discretion of the inspector not to A. pH 10 to pH 11
36. No part of the steam generator c. 398 exceed _ _ year. B. pH 9 to pH 10
should be closer than meter from D. 521 A. 1 C. pH 6to pH 10
any walls of the buildings.- B. 4 D. pH 8 to pH 10
A. one 42. For fire tube boilers, the resistance C. 2
B. three to crushing of mild steel shall be taken D. 3 52. A closed vessel in which steam or
C. two as _ _ N/mm2 of cross sectional area. other vapor is generated at a pressure
D. 4 A. 655 47. For fire tube boilers, each boiler of more than 1.055 kg/cm 2 gage by the
B. 855 shall have at least one safety valve and direct application of heat.
37. Steam boilers should be mounted c. 755 if it has more than 46.5 sq. m of heating A. power boiler
over a suitable foundations or concrete D. 955 surface or the generating capacity B. boiler
pad of not less than _ _ thick and exceeds 910 kg/hr, it shall have _ _ C. fusion boiler
with sufficient base area. safety valves. D. external boiler

REFRESHER MANUAL 2nd Edition by JAS TORDILLO REFRESHER MANUAL 2nd Edition by JAS TORDILLO
- --- --- - ~

25 - 6 I Day 25 - EXAM
Day 25 - EXAM I 25 - 7
53. The heating surface area of _ _ mm from any exposed woodwork Power and Industrial Plant Engineering
miniature boiler is 1.85 m 2 and the or framing.
maximum allowable working pressure is C. safety valve B. flooded
A. 305
_ _ kg/cm 2 • D. gage glass C. brine spray
B. 503
A. 7.03 C. 450 D. cascade
B. 9.85 D. 1200 64. During hydrostatic test, hydrostatic
C. 8.05 test pressure should not exceed _ _ 70. A refrigeration system in which only
D. 14.7 59. Each miniature boiler, shall be times the maximum allowable working part of the refrigerant passing over the
pressu re. heat transfer separated· from the vapor
equipped with a· sealed, spring-loaded
54. Low pressure heating boiler is a A. one and recirculated.
pop type safety valve not less than
boiler operated at pressure not _pipe size, connected ·directly to
B. two A. flooded
exceeding 1.055 kg/cm 2 gage steam or C. one and a half B. steam jet
the boiler.
water temperature not exceeding _ _ A. 10.2 mm D. "three C. mechanical
deg C. B. 15.6 mm D. vapor
A. 100 C ..12.7 mm 65. A boiler test method based on the
B. 121 D. 25.4 mm amount of time it takes generated sound 71. The temperature of the air to be
C. 111 waves to pass through a material and used for comfort cooling shall be
D. 145 60. Each miniature boiler shall be back to the source after being reflected. maintained at degree C.
equipped with for A . radiographic A. 10 - 15
55. A boiler of which both the location determination of water level. B. ultrasonic thickness gauging B. 20 - 24
and ownership have been changed after A. pressure gage C. sound test c. 15 - 20
primary use. D. timing test D. 20 - 30
B. safety valve
A. internal boiler C. water gage glass
B. secondhand boiler D. fusible plug 66. A vessel -permanently connected to 72. The humidity of the air to be used for
C. surplus boiler a system by inlet and outlet pipes for comfort cooling shall be maintained at
D. external boiler 61. Each miniature boiler shall be storage of liquid refrigerant. -:--::-:::-:--:: relative humidity.
equipped with a steam gage having a A. liquid receiver A. 50 to 60%
56. A boiler removed from its original dial range not less than _ __ times B. accumulator B. 60 to 65%
setting and re erected at the same and .n ot more than twice the maximum C. container C. 40 to 55%
location or erected at a location without allowable working pressure. D. duct D. 70 to 75%
change of ownership. A. one
A . PSME boiler B. 3 67. A device having a predetermined i3. Retrigerant piping c rossing an open
B. reinstalled boiler c. 1.5 temperature fusible member for the space which affords passageway in any
C. fusion boiler D. 4 relief of pressu res. building shall not be less than _ _
D. unknown boiler A. gage glass above the floor unless against the
62. The maximum allowable working B. fusible plug ceiling of such space.
57. As a general requirements, pressure of a non-code steel or wrought C. pressure gage A. 2290 mm
smokestacks should be of sufficient iron heating boiler of welded D. stop valve B. 2920 mm
capacity to handle flue gases, self- construction shall not exceed _ _ . ' C. 2570 mm
supporting or guyed to withstand a wind A. 2 Mpa 68. A shut-off valve other than a valve D. 3450 mm
load 160 kph and rise at least _ _ _ B. 1 atm for controlling the flow of refrigerant.
mm above the eaves of any building C. 2 gage pressure A . stop valve 74. When refrigerant is added to a
within a radius of 50 meters. D. 1 bar B. check valve system, it shall be charged into the
A. 5000 C. pressure valve -....,...,......,.-- pressure side of the system.
B. 3000 63. A boiler device which automatically D. relief valve A. high
C. 4000 cut off fuel supply and/or supply B. low
D. 10,000 requisite feed-water when the surface of 69. A refrigeration system in which the C. low and high
the wall falls to the lowest safe water refrigerant gas evolved in the evaporator D. low or high
58. As a general requirements. no line. is taken up in an absorber and released
smokestacks should be closer than A. low water fuel cut off in a generator upon the application of 75. A refrigere~tion system having two or
B. blowoff heat. more refrigerant circuits, each w ith a
A. absorption pressure-imposing element, condenser
REFRESHER MANUAL 2nd Edition by JAS TORDILLO
REFRESHER MANUAL 2nd Edition by JAS TORDILLO
25 - 8 I Day 25 - EXAM
Day 25 - EXAM I 25 - 9
and evaporator, where the evaporator of B. 25 mm Power and Industrial Plant Engineering
one circuit cools the condenser of C. 23 mm
another circuit. 88. Class of fire in flammable liquids and C. combustibles
D. 90mm
gases. D. high
A. brine
A. Class A
B. cascade 82. An indirect refrigerating system B. Class C 95. Type of fire extinguishers that is
C. mechanical employing water as the circulating
D. none of these
C. Class B used by the occupants of a building
system.
D. Class E primarily for immediate use of small
A. chilled water
76. A substance which produces a fires.
B. brine spray
refrigerating effect by its absorption of 89. Class of fire involving combustible A. portable
C. vapor
metals, such as magnesium, sodium, B. outdoor
heat while expanding or evaporating. D. heating system
A. refrigerant
titanium and other similar metals. C. combustibles
A. Class A D. high
B. water 83. This is the ratio of the actual weight
B. Class C
C. air of moisture to the weight of saturated
C. Class B 96. Science of water at rest.
D. brine system water vapor of miXture.
D. Class D A. hydrostatic
A. humidity
77. Any liquid cooled by the refrigerant B. density
B. hydrokinetics
and used for the transmission of heat
90. Class of fire involving energized C. hydraulic
C. specific weight
without a change in its state. electrical equipment. D. hydrodynamic
D. specific gravity
A. mercury A. Class A
B. Class C 97. Science of the force exerted by
B. water spray 84. A vessel in which vaporized
C. brine C. Class B water in motion.
....... refrigerant is liquefied by the removal of
D. water cooler
D. Class D A. hydrostatics
heat.
A. condenser B. hydrodynamics
91 . An organization in-charge with the C. hydrokinetics
78. The chemical name of refrigerant B. evaporator
12. C. compressor
m1ss1on of fire prevention, fire D. hydraulic
protection.
A. cabon dioxide D. brine tank
A. fire service 98. It is the reading of a liquid
B. dichlorodifluoromethane
B. fire building manometer at the suction of the pump.
C. dichloromethane 85. Which of the following refrigerants
D. methyl chloride C. fire hose A . suction lift
Q.elong to Group 2?
D. fire storage B. suction head
A. propane
79. The chemical symbol of ammonia: B. carbon dioxide C. static head
92. Fire code of the Philippines: D. total head
A. SO C. R-114
B. NH3 D. ammonia A. PD No. 185
B. PD No. 1185 99. Used to measure the pressure of
C. H20
C. PD No. 2086 water discharging from a nozzle.
D. C4H1o 86. For fire protection purposes, is an
D. PD No. 8495 A. manometer
integrated system of one or more water
80. The chemical symbol for Freon 12: supplies for fire use, underground and
B. Pitot tube
93. The term designates commodities, C. Piezometer
A. CHCL2F overhead piping design.
packaging or storage aids which will not D. flowmeter
B. CCI2F2 A. sprinkler system
ignite, burn or liberate flammable gases
C. C3Hs B. container system
when heated to a temperature of 749 100. The level w ith respect to the pump
D. C02 C. storage system
degree for five minutes. of the body of water from which it takes
D. fire protection system
A. non-combustibles suction when the pump is not in
81. Suitable means shall be provided for
B. combustibles operation.
the collection and disposal of 87. Class of fire involving ordinary
condensate from the equipment. The
C. medium A. static water level .
combustible materials such as wood,
D. high B. pumping water level
condensate drain shall be at least cloth, paper, rubber and plastics.
___ nominal pipe size and shall b~ A. ClassA C. drawdown
94. Materials either of which will D. dynamic level
copper, galvanized steel, or other B. Class D
corro'sion -resistant material. contribute fuel to fire.
C . Class B
A. 19 mm A. moderate
D. Class E
B. extra
REFRESHER MANUAL 2nd Edition by JAS TORDILLO REFRESHER MANUAL 2nd Edition by JAS TORDILLO
25 - 10 1 Day 25 - Solution Day 25 - Solution 1 25 - 11
Power and Industrial Plant Engineering
DAY 25 - SOLUTION
ra: )
1. All proposed installations, additions or
C. inches
D. feet
C. symbol H
D. symbol R *
DAY 25- ANSWER KEY
1.A 16.8 31.8 46.A alterations involving machinery or
6. In the standard symbols, which of the 12. In the standard symbols, which of
2. B 17.8 32. 8 47. 8 process shall be covered by plans and
following symbols is the same as the the following is the symbol for concrete?
3. D 18. C 33. A 48. 0 specifications prepared by or under the
4. 0 19. A 34. A 49. 8 symbol for brick and stone masonry? Refer to the attached drawings.
supervision of a signed and
5. A 20. C 35. C 50. B A. wood A. symbol L •
6.C 21.9 36.A 51.A
sealed. B. concrete B. symbol C
7.A 22.A 37.0 52. A A. PME'"
C. cast and malleable iron * C. symbol!
8. B 23. C 38. A 53. A B. CPM D. rock
9. B 24. B 39. B 54. 8 C. RME
D. symbol Q
10.8 25. A 40. B 55. 8 D. PME and RME
11.0 26.0 41.A 56.8 7 . In the standard .symbols, which of the 13. In the standard symbols, which of
12.A 27. 0 42. A 57. A following symbols is the same as the the following is the symbol for wood-with
13.A 28. 0 43. A 58. A 2. In the requirement for permit
symbol for bronze, brass and copper? grain? Refer to the attached drawings.
14.8 29. 8 44. A 59. C application, general layout plan for each
A. firebrick * A. symbol T*
15.A 30. C 45. 0 60. C floor drawn to scale not less than
B. water B. symbol!
(I ____,in heavy lines the equipment
C. earth C. symbol E
with super-imposed building outline in
D. sand D. symbol B
light or suppressed lines.
8 A. 1:20 8. In the standard symbols, which of the 14. In the standard for drawings, all
B. 1:200*
DAY 25- ANSWER KEY following symbols is the same as the borders should be at least1 0 mm from
~.C 7aA ~. A
C. 1:100
symbol for sound insulation? the .sheet edge; and all title blocks shall
62.0 77.C 92. B D. 1:1000 A. white metal be located at the corner
63.A 78. B 93.A
B. brick and stone masonry * inside borders for larger sheets.
64.C 79. B 94.A 3. In the requirement for permit
65. B 80. 8 95. A C. rubber A. upper right hand corner
application, the mechanical plans shall
66.A 81.A 96.A D. wood and grain B. lower right hand corner*
67.8 82.A 97.8 contain the signature and seal of a
Professional Mechanical Engineer with C. upper left hand corner
68. A 83. A 98. A 9. In the standard symbols. which of the D. lower left hand corner
69.A 84.A 99.8 the following: Registration number,
70. A 85. 0 100. A validity date, PTR and _ _ __ following is the symbol for rubber,
71. B 86.A plastic and electrical insulation? Refer 15. Standard dimensions of a little block,
A. date of birth
72.A 87.A to the attached drawings. length x width, is:
73.A 88. C B. serial number
A. symbol A A. 150 mm x 50 mm *
74.8 89. D C. birthplace
B. symbol F * B. 100 mm x 50 mm
75. 8 90. 8 D. Tax Identification Number •
C. symbol C C. 150mmx75mm
'I D. symbols D. 100mmx75mm
4. Some of the requirements for
mechanical permit applications are
10. In the standard symbols, which of 16. As a general space requirement for
General layout plan for each floor,
the following is the symbol for earth? work rooms shall be at least in
piping plan, detailed plans of
foundations and _ __ Refer to the attached drawings. height from floor to ceiling.
A. detailed construction plans A. symbol B A. 1m
B. location plan B. symbol 0 • B. 3m*
C. flow-sheet C. symbol J C. 2m
D. all ofthese" D. symbol T D. 2500 mm

5. In the standards for drawings on 11 . In the standard symbols, which of 17. As a general space requirement, t~
mechanical drawings, all dimensions are the following is the symbol for water and maximum number of persons working or
in general, given in _ __ other liquids? Refer to the attached will be working shall not exceed
A. millimeters * drawings.
B. meters A. symbol E A. one person per 8 m3
B. symbol M B. one person per 12m3 *

REFRESHER MANUAL 2nd Edition by JAS TORDILLO REFRESHER MANUAL 2nd Edition by JAS TORDILLO
25 - 12 I Day 25 - Solution Day 25- Solution 125 -13
Power and Industrial Plant Engineering
C. one person per 10m least _ _ from upper surface of the
D. one person per 18 m3 top rail to floor level. be at least 1120 mm away from all C. locomotive
A. 1000 mm * obstruction except hand rails. D. power boiler
18. As a general requirement for B. 2000 mm A. 200 mm
manholes, manhole floor opening shall C. 1300 mm B. 500 mm 32. A closed vessel intended for use in
be manhole covers of D. 2300 C. 400mm heating water or for application of heat
adequate strength which which need not D. 900 mm * to generate steam or other vapor to be
be hinged. 23. As a general requirement for used externally to itself.
A. with no railings, standard railings shall have 28. As a general requirement for stairs, A. boiler
B. may be provided with posts not more than _ _ apart and an no stairway shall have a height of more B. boiler or steam generator*
C. provided with* intermediate rail halfway between top than _ _ between landings, and C. steam generator
D. no rail and the floor. intermediate landings shall have D. none of these
A. 1000 mm dimensions of not less than 1120 mm
19. As a general requirement for floor B. 2500 mm measured ir1 the direction of the run. 33. A vessel in which pressure is
openings, floor openings into which C. 2000 mm * A. 1120 mm obtained from an external sources, or
person can accidentally walk shall be D. 3000 mm B. 2200 mm from· an indirect application of heat. ·
guarded by covers securely held in C. 1750 mm A. unfired pressure vessel *
place and leaving no openings more 24. As a general requirement for D. 2750 mm * B. pressure vessel
than _ _ in width. railings, for pipes railings the top rails C. power boiler
A. 25 mm * and posts of metal pipe of at least 29. As a general requi rement for D. low pressure boiler
B. 75 mm _ _ diameter and intermediate rails of machinery and equipment foundation,
C. 50mm metal pipe at least 25 mm diameter. the weight of the machine plus the 34. Any boiler or unfired pressure
D. 150 mm A. 20 mm weight of the foundation should be vessel constructed, installed, placed in
B. 30 mm * distributed over a sufficient soil area operation but subject to periodic
20. As a general requirement for wall C. 25 mm which is large enough to cause a inspection
openings, all wall openings less than D. 45 mm bearing stress within the safe bearing A. existing installations *
1000 mm from floor having a height of at capacity of soil with a factor of safety of B. new boilers·
least 750 mm and width of 450 mm from 25. As a general requirement for C. hot water boiler
there is a drop of more than _ _ shall railings, toe boards shall be at least 150 A. two D. reinstalled boilers
be solidly enclosed or guarded by fixed mJl) in height and shall be securely B.' five*
~i~. . fastenea in place with not more than C. four 35. A boiler in which that has been
A. 1000 mm _ _ clearance above floor level. D. ten inspected and declared unsafe to
B. 3000 mm A. 6 mm * operate or disqualified stamped and
C. 2000 mm * B. 50 mm 30. Foundation bolts of specified size marked indicating its rejection by
D. 4500 mm C. 10 mm should be used and surrounded by a qualified inspecting body.
D. 100mm · pipe sleeve. No foundation bolts shall A. second hand
21. As a general requirement, all other be less than in diameter. B. reinstalled
wall opening, irrespective of their width 26. As a general requirement for stairs, A. 10 mm C. condemned *
shall, if th!'!ir lower edge is either 80 mm all stairs, platforms, and landings shall B. 15 mm D. unfired
or less above floor level on the near side be of sufficient strength to sustain safely C. 12mm*
or _ _ mm or above ground, or floor a live load of not less than 500 kg with a D. 30 mm 36. No part of the steam generator
level on the far side, be guarded by a factor safety of _ _. should be closer than _ _ meter from
toeboard. A. ten 31 . Any boiler which does exceed any any walls of the buildings.
A. 500 mm B. five of the following: 405 mm inside diameter A. one*
B. 2000 mm * C. eight and 1065 mm overall length of outside B. three
C. 1000 mm D. four" 2
of heads at cen(er, 1,85 m of water C. two
D. 2500 mm heating surface and 7.03 kg/cm 2 D. 4
27. As a general requirement for stairs, maximum allowable working pressure.
22. As a general requirement for width of stairs except service stairs, A. portable boiler 37. Steam boilers should be mounted
railings, standard railings shall be at shall be not less than and should B. miniature boiler * over a suitable foundations or concrete

REFRESHER MANUAL 2nd Edition by JAS TORDILLO REFRESHER MANUAL 2nd Edition by JAS TO'RDILLO
25 - 14 I Day 25 - Solution Day 25 - Solution 1 25 - 15
Power and Industrial Plant Engineering
pad of n,ot less than _ _ thick and 43. For fire tube boilers, the lowest
with sufficient base area. factor of safety permissible on existing B. two • 53. ifhe heating surface area of
A. 150 mm installations shall be _ _ excepting C. two or more miniature boiler is 1.85 m2 and the
B. 255 mm for horizontal return tubular boilers D. three to five maximum allowable working pressure is
C. 195 mm having continuous lap seams more·than _ _ kg/cm 2 .
D. 305 mm * 3650 mm in length where the factor of 48. For fire tube boilers, the use of A. 7.03 *
safety shall be _ _. weighted-lever safety valves shall be B. 9.85
38. When boilers are replaced or new A. 4.5, 9 * prohibited and type va lves C. 8.05
boilers are installed in either existing or B. 9, 4 shall replace these va lves. D. 14.7
new buildings, a minimum of at least c. 55,8 A. disc type
_ _ shall be provided between the D. 8,5 B. swing type 54. Low pressure heating boiler is a
top of the boiler proper and ceiling. C. hammer lever type boi ler operated at pressure not
A. 2130 mm * 44. For fire tube boilers, reinstalled or D. direct spring-loaded type * exceeding 1.055 kg/cm 2 gage steam or
B. 900 mm secondhand boilers shall have a water temperature not exceeding _ _
C. 1560 mm minimum factor of safety of _ _ when 49. For fire tube boilers. each boiler deg C.
D. 746 mm the longitudinal seams are of lap riveted shall have the _ _or more gage A. 100
construction, and a minimum factor of cocks, located within the range of the B. 121 *
39. For fire tube boilers, the age limit of safety of _ _when the longitudinal visible length of the water glass. C. 111
horizontal return tubular, flue of cylinder seams are' of butt and double strap A. one D. 145
boiler shall be constructiol']. B. three •
A. 10 years A. 6, 5 * C. two 55. A boiler of which both the location
B. 30 years* B. 5, 6 D. five and ownership have been changed after
C. 20 years C. 4, 6 primary use.
D. 45 years D. 6, 4 50. For miniature boilers. the lowest A. internal boiler
permissible water level shall be at a B. secondhand boiler •
40. A steel catwalk or platform at least 45. For fire tube boilers, the age limit of point _ _ of the height of the shell, C. surplus boiler
_ _ wide and provided with standard a horizontal return, flue or cylinder boiler except where the boiler is equipped with D. external boiler
handrails and toeboard on either side having· a longitudinal lap joint and internal furnace.
shall be installed across the tops of operating at a pressure in excess of A. 1/2 56. A boiler removed from its original
adjacent boilers. 0.345 Mpa shall be . B. 1/3 * setting and re erected at the same
A. 255 mm A: 10yts -- C. 1/4 location or erected at a location without
B. 455 mm * B. 20 yrs D. 2/5 change of ownership.
C. 355 mm C. 15 yrs A. PSME boiler
D. 555 mm D. 30 yrs * 51. Where ground water or surface B. reinstalled boiler •
water are used for feedwater, water C. fusion boiler
41. For fire tube boilers, when the 46. For fire tube boilers, a reasonable hardness of 0-10 ppm (parts per million) D. unknown boiler
tensile strength of steel is not known, it time for replacement shall be given at and alkalinity of __. be considered.
shall be taken as
2
N/mm and 310 the discretion of the inspector not to A. pH 10 to pH 11 * 57. As a general requirements,
N/mm2 for wrought iron. exceed _ _ year. B. pH 9 to pH 10 smokestacks should be of sufficient
A. 379 * A. 1 * C. pH 6 to pH 10 capacity to handle flue gases, self-
B. 450 B. 4 D. pH 8 to pH 10 supporting or guyed to withstand a wind
C. 398 C. 2 load 160 kph and rise at least _ __
D. 521 D. 3 52. A closed vessel in which steam or mm above the eaves of any building
other vapor is generated at a pressure within a radius of 50 meters.
42. For fire tube boilers, the resistance 47. For fire tube boilers, each boiler of more than 1.055 kg/cm 2 gage by the A. 5000 *
to crushing of mild steel shall be taken shall have at least one safety valve and direct application of heat. B. 3000
as _ _ N/mm2 of cross sectional area. if it has more than 46.5 sq. m of heating A. power boiler* C. 4000
A. 655 * surface or the generating capacity B. boiler D. 10,000
B. 855 exceeds 910 kg/hr. it shall have _ _ C . fusion boiler
C. 755 safety valves. D. external boiler 58. As a general requirements, no
D. 955 A . one smokestacks should be closer than

REFRESHER MANUAL 2nd Edition by JAS TORDILLO REFRESHER MANUAL 2nd Edition by JAS TORDILLO
25 - 16 1 Day 25 - ·Solution
Day 25- Solution 125 -17
_ _ mm from any exposed woodwork C. safety valve Power and Industrial Plant Engineering
or framing. D. gage glass
A. 305 * C. brine spray one circuit cools the condenser of
B. 503 64. During hydrostatic test, hydrostatic D . cascade another circuit.
c. 450 test pressure should not exceed _ _ A. brine
D. 1200 times the maximum allowable working 70. A refrigeration system in which only B. cascade"
pressure. part of the refrigerant passing over the C. mechanical
59. Each miniature boiler, shall be A. one heat transfer separated from the vapor D. none of these
equipped with a sealed, spring-loaded B. two and recirculated.
pop type safety valve not less than C. one and a half* A. flooded* 76. A substance which produces a
___pipe size, connected directly to D. three B. steam jet refrigerating effect by its absorption of
the boiler. C. mechanical heat while expanding or evaporating.
A. 10.2 mm 65. A boiler test method based on the D. vapor A. refrigerant "
B. 15.6 mm amount of time it takes generated sound B. water
C. 12.7 mm * waves to pass through a material and. 71. The temperature of the air to be C. air
D. 25.4 mm back to the source after being reflected. used for comfort cooling shall be D. brine system
A. radiographic maintained at degree C.
60. Each miniature boiler shall be B. ultrasonic thickness gauging * A. 10- 15 77. Any liquid cooled by the refrigerant
equipped with __ for C. sound test B. 20-24 * and used for the transmission of heat
determination of water level. D. timing test C. 15-20 without a change in its state.
A. pressure gage D. 20-30 A. mercury
B. safety valve 66. A vessel permanently connected to B. water spray
C. water gage glass * a system by inlet and outlet pipes for 72. The humidity of the air to be used for C. brine"
D. fusible plug storage of liquid refrigerant. comfort cooling shall be maintained at D. water cooler
A. liquid receiver * ____ relative humidity.
61 . Each miniature boiler shall be B. accumulator A. 50 to 60% * 78. The chemical name of refrigerant
equipped with a steam gage having a C. container B. 60'to 65% 12.
dial range not less than ___ times D. duct C. 40 to 55% A. cabon dioxide
and not more than twice the maximum D. 70 to 75% B. dichlorodifluoromethane *
allowable working pressure. 67. A device having a predetermined C . dichloromethane
A. one temperature fusible member for the 73. Refrigerant piping crossing an open D. methyl chloride
B. 3 relief of pressures. space which affords passageway in any
c. 1.5 .. A. gage glass building shall not be less than _ _ 79. The chemical symbol of ammonia:
D. 4 a, fusible plug * above the floor unless against the A. SO
C. pressure gage ceiling of such space. B. NH3 *
62. The maximum allowable working D. stop valve A. 2290 mm * C. H20
pressure of a non-code steel or wrought B. 2920 mm D. C4H1o
iron heating boiler of welded 68. A shut-off valve other than a valve C. 2570 mm
construction shall not exceed for controlling the flow of refrigerant. D. 3450 mm 80. The chemical symbol for Freon 12:
A. 2 Mpa A. stop valve* A. CHCL2F
B. 1 atm B. check valve 74. When refrigerant is added to a B. CChF2 *
C. 2 gage pressure C. pressure valve system, it shall be charged into the C. C3He
D. 1 bar* D. relief valve _ _ _ pressure side of the system. D. C02
A. high
63. A boiler device which automatically 69. A refrigeration system in which the B. low* 81. Suitable means shall be provided for
cut off fuel supply and/or supply refrigerant gas evolved in the evaporator C. low and high the collection and disposal of
is taken up in an absorber and released D. low or high condensate from the equipment. The
requisite feed-water when the surface of
in a generator upon the application of condensate drain shall be at least
the wall falls to the lowest safe water
75. A refrigeration system having two or _ __ nominal pipe size and shall be
line. · heat.
A. absorption * more refrigerant circuits, each with a copper, galvanized steel, or other
A. low water fuel cut off*
B. flooded pressure-imposing element, condenser corrosion -resistant material.
B. blow off
and evaporator, where the evaporator of A. 19 mm"
REFRESHER MANUAL 2nd Edition by JAS TORDILLO REFRESHER MANUAL 2nd Edition by JAS TORDILLO
25 - 18 I Day 25 - Solution Day 25- Solution 125 -19
B. 25 mm
Power and Industrial Plant Engineering
88. Class of fire in flammable liquids and
C. 23mm gases.
C. combustibles
D. 90mm A. ClassA D. high
B. Class C
82. An indirect refrigerating system C. Class B • 95 Type of fire extinguishers that is
employing water as the circulating D. Class E
system. used by the occupants of a building
primarily for immediate use of small
A. chilled water • 89. Class of fire involving combustible
B. brine spray
fires.
metals, such as magnesium, sodium, A. portable *
C. vapor titanium and other similar metals. B. outdoor
D. heating system A. ClassA
C. combustibles
B. Class C D. high
83. This is the ratio of the actual weight C. Class B
of moisture to the weight of saturated D. Class 0 * 96. Science of water at rest.
water vapor of mixture.
A. humidity •
A. hydrostatic *
90. Class of fire involving energized B. hydrokinetics
B. density electrical .equipment.
C. hydraulic
C. specific weight A. Class A
D. hydrodynamic
D. specific gravity B. Class C *
C. Class B 97. Science of the force exerted by
84. A vessel in which vaporized D. Class D
water in motion.
refrigerant is liquefied by the removal of
A. hydrostafics
heat. 91. An organization in-charge with the B. hydrodynamics *
A. condenser * mission of fire prevention, fire C. hydrokinetics
B. evaporator protection.
D. hydraulic
C. compressor A. fire service *
D. brine tank B. fire building 98. It is the reading of a liquid
C. fire-hose
manometer at the suction of the pump.
85. Which of the following refrigerants 0. fire storage A. suction lift * ·
belong to Group 2?
B. suction head
A. propane · 92. Fire code Qf the Philippines:
C. static head
B. carbon dioxide A. PD No. 185
D. total head
C. R-11 4 B. PO No. 1185 *
D. ammonia* C. PD No. 2086
99. ·Used to measure the pressure of
D. PD No. 8495 water discharging from a nozzle.
86. For fire protection purposes, is an
A. manometer
integrated sy.stem of one or more water 93. The tenm designates commodities,
B. Pitot tube *
supplies for fire use, underground and packaging or storage aids which will not
C. Piezometer
overhead piping design. ignite, burn or liberate flammable gases
D. flow meter
A. sprinkler system* when heated to a temperature of 749
B. container system degree for five minutes.
100. The level with respect to the pump
C. storage system A. non-combustibles •
of the body of water from which it takes
D. fire protection system B. combustibles
suction when the pump is not in
C. medium
87. Class of fire involving ordinary D. high operation.
A. static water level *
combustible materials such as wood,
B. pumping water level
cloth, paper, rubber and plastics. 94. Materials either of which will
C. draw down
A. ClassA." contribute fuel to fire.
D. dynamic level
B. Class D A. moderate *
C. Class B B. extra
D. Class E

REFRESHER MANUAL 2nd Edition by JAS TORDILLO REFRESHER MANUAL 2nd Edition by JAS TORDILtO
Day 26 - EXAM I 26 - 1
Power and Industrial Plant Engineering
C. piezometer ·
DAY 26- EXAM D. compound
1. The vertical difference between the 7. The following are classifications of
pumping water lev.el and static water pumps except:
level. A centrifugal
A total head B. reciprocating
B. static head C. radial flow
C. pump level D. rotary
D. draw-down
8. The following are types of
2. An underground formation that reciprocating pumps, except:
contains sufficient saturated permeable A piston
material to yield significant 'quantities of B. peripheral
water. C. plunger
A. aquifer D. diaphragm
B. wet pit
C. cavitation 9. The following are types of rotary
D. NPSH pumps, except:
A vane
3. Is the rate of flow of liquid measure B. screw
per unit of time. C. gear
A. capacity D. duplex
B. pump work
C. velocity 10. The following are types of
D. pump head centrifugal pumps, except:
A. hydraulic
4. The reading of a pressure gage at B, axial
the discharge of the pump plus velocity C. mixed
head at the point of gage attachment. D. radial
A. total discharge head
B. total head 11. Valve designed to open
C. NPSH automatically to relieve excess
D. suction head pressure.
A. check valve
5. A type of valve installed between the B. relief valve
suction of pump and water mains to C. gate valve
prevent any return of water to mains D. medium valve
when pump is stopped.
A. globe valve 12. is a hollow product of
B. suction valve round or any other cross section having
C. discharge valve a continuous periphery.
D. check valve A. pipe
B. tube
6. A gauge in the form of a glass one C. valve
tube leg of which is open to the D. coupling
atmosphere, or a straight tube one end
of which is open to the atmosphere. 13. A large p,ipe or drum into which
A manometer each of a group of boilers is connected.
B. differential A. muffler

REFRESHER MANUAL 2nd Edition by JAS TORDILLO


26 - 2 I Day 26 - EXAM Day 26 - EXAM I 26 - 3
Power and Industrial Plant Engineering
B. header 21. Piping color for high pressure water
C. manifold and hot water: C. 10-20 m/s B. -252°F
D. expansion loop A. silver D. 50 -70 mls C. -101°F
B. yellow D. -460°F
14. Medium pressure. when applied to C. red 28. To identify pipe flow, arrows will be
valves and fittings, implies they are D. green painted on pipes. All of the following are 34. The boiling point R-11 at 1
suitable for a working pressure of from notations, except: atmosphere is:
____ kPa.
22. Piping color for LPG, high vacuum A. arrows shall be in different color as A 79.9 °F
A 862 to 1207 and producer gas: pipe banding B. -11°F
B. 1207 to 1506 A. safety red B. arrows shall be stencil type C. -74.9°F
C. 955 to 1345 B. orange C. arrows shall be readable from floor D. 11°F
D. 1345 to 1750 C. red D. arrows shall be installed every 4500 35. The boiling point of R-22 at 1
D. no color mm. atmosphere is:
15. All piping shall be run to A. -21.4°F
building walls. 23. Which of the following statements 29. As a general requirement for piping, B. -61.4°F
A. opposite for piping system is correct? all piping __ and above shall be C. -41.4°F
B. parallel A. all piping to headers shall come from flanged while smaller sizes can be D. 70°F
C. inclined below rack screwed .
D. slope B. all piping from headers shall do up A. 50 mm 36. The boiling point of R- 134 at 1
above rack B. 63.5 mm atmosphere is:
16. Piping supports shall be placed C. all piping shall be · reasonably C. 60 mm A . -61.1°F
_ _ mm intervals or less. cleaned before installation D. 93.5 mm B. 25°F
A. 3000 D. all of these c. 50°F
B. 4000 30. As a general requirement for piping, D. -15.08°F
C. 4500 24. A fitting with a number of branches piping supports shall be placed on a
D. 6000 in line connecting to smaller pipes. 3000 mm intervals or less and a 37. The chemical name of Refrigerant
A manifold minimum factor of safety of -:--::-:-- 12 is:
17. Piping color for boiled feed water: B. small pipes for working pressure applied shall be A. dichlorodifluoromethane
A. green C. major pipe used. B-. trochlorodifluoromethane
B. yellow D. valve A. two C. dich,loromethane
C. red B. four D. methyl chloride
D. black 25. A large radius bend in a pipe line to C. three
absorb longitudinal expansion in the line D. five 38. The chemical name of R-22 is:
18. Piping color for acids: due to heat. A. dichloromethane
A. ye llow A. expansion loop 31. What is the freezing point pf R-22 at B. chlorodifluoromethane
B. blue B. expansion joint 1 atmospheric? C. monochlorodiflluoromethane
C. orange C. compression joint A. -256 °F D. ethane chloride
D. red D. compression loop B. 150 °F •
C. -252 °F 39. The chemicalname Of R-134a is:
19. Piping color for oxygen and oil: 26. Color bands on pipes to identify D. 32 °F A chlorotrifluoromethane
A. green pipe color is painted every _ ____ B. hydrocarbon
B. black A. 2000 mm 32. What is the freezing point_of R-134 C. tetrafluoroethane
C. white B. 4000 mm at 1 atmospheric? D. trochloroethane
D. orange C. 2500 mm A. -91 .9°F
D. 4500 mm B. -141 .9°F 40. Group B refrigerants are _ _ __
20. Piping color for low-pressure air: C. 41 .1°F A. toxic
A. green 27. In practice, the average fluid flow D. -121°F B. lethal
B. yellow velocity for water is _ __ C. harmful
C. white A. 1.5- 3m/s 33. What is the freezing point of R-12 at D. all of the above
D. orange. B. 15 -30 m/s 1 atmosphere?
A. -41°F

REFRESHER MANUAL 2nd Edition by JAS TORDILLO REFRESHER MANUAL 2nd Edition by JAS TORDILLO
26 - 4 I Day 26 - EXAM Day 26 • EXAM I 26 • 5
Power and Industrial Plant En~ineering
41 . Which of the following is NOT an oil Presently being used as a standard
suitable for use with R-134a? refrigerant in vehicular air conditioning. C. -33uF 60. Refrigerant 22 has good solubility in
A. Mineral oil-based lubricant A. R-134a D. -150°F oil. Leaks may be detected by.
B. Polyolester B. R-717 A. soap solution
C. Alkyl benzene C. R-502 54. Refrigerant ammonia corrodes B. electronic detector
D. Polyalkylene glycol D. R-123 copper and bronze. It does not corrode C. halide torch
to: D. all of these
42. Which of the following is an HFC 48. A type of refrigerant which is A. stainless
refrigerant? colorless and almost odorless B. steel 61 . The chemical symbol of R-22 is:
A. R-114 refrigerant. It is nontoxic, noncorrosive, C. chromium A. CHCIF2
B. R-22 nonirritating and nonflammable. D. manganese B. CCIF2
A. R-11 C. CHF
C. R-134a B. R-507 55. Cryogenic temperatures ranges D. CHCIH2F
D. R-113 ·from .
C. R-12 A. -250 to -460°F 62. The chemical symbol of R-134a is:
43. Which of the following is an HCFC D. NH3 B. -50 to -100°F A. CFCHF2
refrigerant? B. CF3CH2F
A. R-114 49. The cylinder code color for R-22 is C. -120 to -150°P C. CCHF2
B. R-113 D. 10 to -50°F D. FCH2CI '
C. R-22 A. white
D. all of these B. yellow 56. What does the word "ozone" 63. Ammonia is refrigerant:
C. tan describe? A. R-505
44. Which of the following is a D. light green A. a thin layer in the earth's upper B. R-717
refrigerant blend? atmosphere C. R-40
A. azeotropic 50. The cylinder code color for R-134a B. a protective layer for the earth D. R-502
B. HFCs is ____. C. A filter for the sun's ultraviolet rays
C. CFCs A. sky blue D. all of the above 64. The chemical symbol of R-12 is:
D. HCFCs B. red A. CCI2F2
C. light orange 57. The service cylinders are filled with B. CFCIFs
45. What is HFC? D. white refrigerants up to _ _ full for safe limit C. CHCI2
A. hydrogenfreecarbon and to avoid bursting of cylinders. D. CHNF2
B. hydrochlorofluorocarbons 51 . Refrigerant which is a mixture of A. 70%
C. hydrochlorocarbons two or more substance. B. 9D% 65. A synthetic chemical refrigerant,
D. hydrofluorocarbons A. azeotropic C. 80% which is. considered to be low-pressure
B. zeotropic D. 100% refrigerant.
46. A type of refrigerant which is non- C. hydrocarbon A. R-11
toxic, noncorrosive, nonirritating, and D. monocarbon 58. Liquid refrigerant expands with a/an B. R-22
nonflammable. It is <;~ synthetic = --=---:-:-:-in temperature. Cylinders C. R-12
refrigerant developed for installations 52. A type of Group B refrigerants filled with cold or cool refrigerant will D. R-502
that need a low evaporating which is commonly used in industrial burst if allowed to warm up.
temperature. systems. A. decrease
A. Ammonia A. methyl chloride B. normal
B. carbon dioxide B. sulfur dioxide C. increase
C. R-500 C. R-123 D. unknown
D. R-22 D. Ammonia
59. is an HCFC used as a
47. A type of refrigerant which is 53. The boiling temperature of replacement for R-11.
nontoxic, nocorrosive, and ammonia at atmospheric temperature is A. R-707
nonflammable. It is used as a B. R-110
replacement for R-12. A refrigerant A. -28°F C. R-501
which is compatible with polyol ester oil. B. -108°F D. R-123

REFRESHER MANUAL 2nd Edition by JAS TORDILLO REFRESHER MANUAL 2nd EdiHon by JAS TORDILLO
26-61 Day 26 - Solution Day 26 - Solution 126 • 7
Power and Industrial Plant Engineering
DAY 26- SOLUTION 7. The following are classifications of
(d pumps except:
14. Medium pressure, when applied to
valves and fittings, implies they are
IDAy 26 - ANSWER KEY 1. The vertical difference between the A. centrifugal suitable for a working pressure of from
'1 . 016. A 31 . A46. 061 . A pumping water level and static water B. reciprocating
level. -=--~~kPa.
2. A
3. A
17. B
18. A
32. B47. A62. B
33. B 48. C 63. B A. total head
c. radial flow * A. 862 to 1207 *
D. rotary

~
. A 19. D 34. A 49. 0 64. A B. static head
B. 1207 to 1506
. D 20. A 35. C 50. A 65. A C. 955 to 1345
. A 21 . 9 36. 051 . 8 C. pump level 8. The following are types of D. 1345 to 1750
. C 22. B 37. A 52. D D. draw-down * reciprocating pumps, except:
. B 23. D 38. B 53. A A. piston
9. D 24. A 39. C 54. B 15. All piping shall be run to
2. A n underground formation that B. peripheral *
10. A25. A 40. 055. A building walls.
contains sufficient saturated permeable C. plunger
11. B 26. D 41 . A 56. 0 A. opposite
12. B 27. A 42. C 57. C material to yield significant quantities of D. diaphragm B. parallel *
13. B28. A 43. C58. C water.
C. inclined
14. A 29. B 44. A 59. D A. aquifer*
15. B 30, B 45. 0 60. D 9. The following are types of rotary D. slope
B. wetpit
pumps, except:
G C. cavitation
D. NPSH
A. vane 16. Piping· supports shall be placed
B. screw _ _ mm intervals or less.
C. gear A. 3000 *
3. Is the rate of flow of liquid measure D. duplex • B. 4000
per unit of time.
A. capacity *
c. 4500
10. The · following are types of D. 6000
B. pump work
centrifugal pumps, except:
C. velocity A. hydraulic * 17. Piping color for boiled feed water:
D. pump head B. axial A. green
4 . The reading of a pressure gage at
C. mixed B. yellow•
D. radial C. red
the discharge of the pump plus velocity
head at the point of gage attachment.
D, black
11. Valve designed to open
A. · total >discharge head *
B. total head
automatically to relieve excess 18. Pipihg color for acids:
pressure. A. yellow*
C. NPSH
A. check valve B. blue
D. suction head
B. relief val ve * C. orange
C. gate valve D. red
5. A type of valve 'installed between the
D. medium valve
suction of pump and water mains to
prevent any return of water to mains ' 19. Piping color for oxygen and oil:
when pump is stopped.
12. is a hollow product of A. green
A. globe valve
round or any other cross section having B. black
B. suction valve a continuous periphery. C. ·white
C. discharge valve A. pipe D. orange *
D. check valve* B. tube *
C. valve 20. Piping color for low-pressure air:
6. A gauge in the form of a glass one D. coupling A. green •
tube leg of which is open to the B. yellow
13. A large pipe or drum into which C. white
atmosphere, or a straight tube one end
of which is open to the atmosphere. each of a group of boilers is connected. D. orange
A. muffler
A. manometer*
B. differential B. header* 21. Piping color for high pressure water
C. manifold and hot water:
C. piezometer
D. expansion loop
D. compound
REFRESHER MANUAL 2nd Edition by JAS TORDILLO REFRESHER MANUAL 2nd Edition by JAS TOROILLO
r- -
26 - 8 I Day 26 - Solution

A. silver
B. yellow*
28. To identify pipe flow, arrows will be
painted on pipes. All of the following are 34. The boiling point R-11 at 1
Day 26 - Solution I 26 - 9
Power and Industrial Plant Engineering
C. Alkyl benzene
C. red notations, except: atmosphere is:
D. green D. Polyalkylene glycol
A. arrows shall be in different color A. 79.9 °F *
as pipe banding .,. B. -11°F
22. Piping color for LPG, high vacuum 42. Which of the following is an HFC
B. arrows shall be stencil type C. -74.9°F refrigerant?
and producer gas: C. arrows shall be readable from floor D. 11°F A. R-114
A. safety red D. arrows shall be installed every 4500
B. orange* B. R-22
mm. 35. The boiling point of R-22 at t
C. red
C. R-134a •
atmosphere is: D. R-113
D. no color 29. As a general requirement for piping, A. -21.4°F
all piping _ _ and above shall be B. -61.4°F 43. Which of the following is an HCFC
23. Which of the following statements flanged while smaller sizes can be C. -41.4°F * refrigerant?
for piping system is correct? screwed. D. 70°F A. R-114
A. all piping to headers shall come from A. 50mm
B. R-113
below rack B. 63.5 mm * 36. The boiling point of R-134 at 1 C. R-22 *·
B. all piping from headers shall do up C. 60 mm atmosphere is: D. all of tbese
above rack D. 93.5 mm A. -61.1°F
C. all piping shall be reasonably B. 25°F 44. Which of the following is a
cleaned before installation 30. As a general requirement for piping, C. 50°F refrigerant blend?
D. all of these * piping supports shall be placed on a D. -15.08°F * A. azeotropic *
3000 mm intervals or less and a
minimum factor of safety 9f _ _ __
B. HFCs
24. A fitting with a number of branches
37. The chemical name of Refrigerant C. CFCs
in line connecting to smaller pipes. for working pressure applied shall be 12 is:
A. manifold * used. · D. HCFCs
A. dichlorodifhioromethane *
B. small pipes A. two B. trochlorodifluoromethane 45. What is HFC?
C. major pipe B. four"
C. dichloromethane A. hydrogenfreecarbon
D. valve C. three D. methyl chloride
D. five B. hydrochlorofluorocarbons
25. A large radius bend in a pipe line to C. hydrochlorocarbons
38. The chemical name of R-22 is: D. hydrofluorocarbon~ *
absorb longitudinal expansion in the line 31 . What is the freezing point pf R-22 at
A. dichloromethane
due to heat. 1 atmosRheric?
B. chlorodifluoromethane * 46. A type of refrigerant which is non-
A. expansion loop * A. ·256 °F * C. monochlorodiflluoromethane toxic, noncorrosive, nonirritating, and
B. e')(pansion joint B. 150°F D. ethane chloride nonflammable. It is a synthetic
C. compression joint c. -252 °F
refrigerant developed for installations
D. compression loop D. 32 °F
39. The chemicalname Of R-134a is: that need a low evaporating
A. chlorotrifluoromethane temperature.
26. Color bands on pipes to identify 32. What is the freezing point of R-134 B. hydrocarbon A. Ammonia
pipe color is painted every - - - ' - - at 1 atmospheric? C. tetrafluoroethane * B. carbon dioxide
A. 2000 mm A. -91 .9°F D. trochloroethane C. R-500
B. 4000 mm B. -141.9°F * D. R-22 *
C. 2500 mm C. 41 .1°F 40. Group B refrigerants are _ _ __
D. 4500 mm * D. -121°F A. toxic 47. A type of refrigerant which is
B. lethal nontoxic, nocorrosive, and
27. In practice, the average fluid flow 33. What is the freezing point of R-12 at C. harmful nonflammable. It is used as a
velocity for water is _ _ _. 1 atmosphere? D. all of the above * replacement for R-12. A refrigerant
A. 1.5 - 3m/s * A. -41°F
which is compatible with polyol ester oil.
B. 15-30 m/s B. ·252°F * 41. Which of the following is NOT an oil Presently being usec;l as a standard
C. 10-20 m/s C. -101°F
suitable for use with R-134a? refrigerant in vehicular air conditioning.
D. 50-70 m/s D. -460°F
A. Mineral oil-based lubricant* A. R-134a *
B. Polyolester B. R-717

REFRESHER MANUAL 2nd Edition by JAS TORDILLO REFRESHER MANUAL 2nd Edition by JAS TORDILLO
26 -10 1 Day 26 - Solution Day 26 -Solution 126 - 11
C. R-502 54. Refrigerant ammonia corrodes Power and Industrial Plant Engineering
D. R-123 copper and bronze. It does not corrode C. halide torch
to: D. all of these *
48. A type of refrigerant which is A. stainless
colorless and almost odorless B. steel* 61. The chemical symbol of R-22 is:
refrigerant. It is nontoxic, noncorrosive, C. chromium A. CHCIF2"
nonirritating and nonflammable. D. manganese B. CCIF2
A. R-11 55. Cryogenic temperatures ranges C. CHF
B. R-507 from . D. CHCIH2F
C. R-12 * A. -250 to -460°F "
D. NH3 B. -50 to -100°F 62. The chemical symbol of R-134a is:
C. -120 to -150°F A. CFCHF2
49. The cylinder code color for R-22 is D. 10 to -50°F B. CF3CH2F"
C. CCHF2
A . white 56. What does the word "ozone" D. FCH2CI
B. yellow describe?
C. tan A. a thin layer in the earth's upper 63. Ammonia is refrigerant:
D. light green "' atmosphere A. R-505
B. a protective layer for the earth B. R-717 *
50. The cylinder code color for R-134a C. A filter for the sun's ultraviolet rays C. R-40
is _ __ D. all of the above * D. R-502
A. sky blue *
B. red 57. The service cylinders are filled with 64. The chemical symbol of R-12 is:
C. light orange refrigerants up to _ _ full for safe limit A. CCI2F2"
D. white and to avoid bursting of cylinders. B. CFCIFa
A 70% C. CHCh
51 . Refrigerant which is a mixture of B. 90% D. CHNF2
two or more substance. c .. 80% .
A . azeotropic D. 100% 65. A synthetic chemical refrigerant,
B. zeotropic * which is considered to be low-pressure
C. hydrocarbon 58. Liquid refrigerant expands with a/an refrigerant.
D. monocarbon ____in temperature. Cylinders A. R-11 *
filled with cold or cool refrigerant will B. R-22
52. •A type of Group B refrigerants burst if allowed to warm up. C. R-12
which is commonly used in industrial A. decrease D . .R-502
systems. B. normal
A . methyl chloride c. increase "
B. sulfur dioxide D. unknown
C. R-123
D. Ammonia • 59. is an HCFC used as a
replacement for R-11,
53. The boiling temperature of A R-707
ammonia at atmospheric temperature is B. R-110
C. R-501
A. -28°F * D. R-123"
B. -108°F
C. -33°F 60. Refrigerant 22 has good solubility in
D. -150°F oil. Leaks may be detected by. ·
A . soap solution
B. electronic detector

REFRESHER MANUAL 2nd Edition by JAS TORDILLO REFRESHER MANUAL 2nd Edition by JAS TORDILLO
Day 27 - EXAM I 27 - 1
Power and Industrial Plant Engineering
'I
At -1 0 degree C:
DAY 27- EXAM hf = 190.82 hg = 347.134
A. 14.2 TOR
1. A refrigeration cycle has a Coefficient B. 11.5 TOR
of Performance of 5. The power input to C. 12.1 TOR
the compressor is 3 kw. Compute the D. 10.2TOR
rate of heat rejection from the refrigerant
to the environment. 5. Liquid refrigerant passing through the
A. 12 kw evaporator coils is vaporized:
B. 18 kw A. due to the heat from the condenser
C . 15 kw B. due to the. heat of compression
D. 20 kw C . due to the absorbed heat by coils
D. due to heat rejected in the condenser
2. One thousand kilogram of fish at a
temperature of 10 degree C was placed 6. A refrigeration plant stored 10 metric
in a chiller box storage. Determine the tons of eggs at a temperature of 15
quantity of heat required to bring the degree C. To preserve the eggs they
temperature of the fish to its freezing have to be cooled at -5 degree C. What
temperature. (Note: Specific heat above is the refrigerator capacity in tons of
freezing = 0. 76 kcal/kg-deg (1; specific refrigeration, if it is to cooled in 15
heat below freezing = 0.41 kcallkg- hours? The specific heat of eggs above
degC; freezing temperature = -2.2 and below freezing is 0.95 kcal/kg-deg
degree C). C and 0.4 kcal/kg-deg C, respectively.
A. 6532 kcal The latent heat of fusion is 68.5 kcal/kg.
B. 7922 kcal The freezing temperature of egg is -3
C. 7229 kcal degree C.
D. 9272 kcal A. 25 tons
B. 21 tons
3. A system needs 4HP to compress C. 23 tons
vapor refrigerant. The amount of heat D. 19 tons
· per min absorbed by the cooling coils
from the warm products is 1253 kJ . 7. A ton of refrigeration is how mahy
What is the performance of the BTU per 12 hours?
refrigerator? A. 288,000
A. 6 B. 144,000
B. 8 C. 225,000
c. 7 D. 14,400
D. 9
8. What is the chemical name of R-
4. A refrigeration system operates on 727?
an ideal vapor compression using R-12 . A. ammonia
with a vaporizing temperature of minus B. Methane
10 degree C and a condensing C. brine
temperature of 30 degree C. What is D. air '
the refrigerating capacity in tons of
refrigeration if the compressor power is 9. A pressure control valve inserted at
10 kw? the suction side of the evaporator and
R-12 Properties: sometimes used to protect the
At 30 degree C: compressor to overload due to excess
=
hf 228.54 =
hg 363.56 h = 375 amount of heat absorbed by the

REFRESHER MANUAL 2nd Edition by JAS TORDILLO


27 - 2 I Day 27 - EXAM Day 27 - EXAM I 27 - 3
evaporatqr coils . What do you call this 15. What is the humidity ratio of dry air Power and Industrial Plant Engineering
device? in kg per kg?
A. relay and head of pump 2. If H,> H2 and Q, < 23. A heat transfer between a solid
A. 100
B. condenser 02, what is the discharge of the pump? surface and a moving fluid is called:
B. 0.01 A. Q, A. conduction ·
C. solenoid valve .C. 50
B. Q, +Qz B. CO!J.Veetion
D. thermostatic expansion valve D. zero
c. Q2 C. radiation
D. 02- Q, D. thermal radiation
~ 0. How many tons of ice can a 10-ton 16. Water from a shell and tube
ice plant produce in one day? condenser enters a cooling tower at 40
A. 24 19. What is the head developed if the 24. A piston cylinder contains 2 kg of
degree C. The ambient air entering is at
B. 10 pump that has a capacity of Q, and steam, which expands from state 1 with
20 degree C dry bulb and 10 degree C
c . 20 head H, connected in parallel with an internal energy of 2700 kJ/kg to state
wet bulb temperatures and leave~ at 35
D. 1 another pump with a capacity of 02 and 2 with an internal energy of 2650 kJ/kg.
degree C dry bulb with 50% RH . The
head H2, if Q, > 02? During the expansion process the
heat rejected in the condenser in liters
11. When there is an increase in A. H, system receives 30 kJ of heat.
per second. Cooling tower efficiency is
humidity ratio to air, it means: 75%.
B. H, + H2 Determine the system work.
A. humidifying · C. H2 -H, A. 70 kJ
Properties of Air:
B. dehumidifying D. H,/H2 B. 130 kJ
At 20 deg C db and 10 deg C wb:
C. cooling = =
h 29 kJ/kg v 0.837 cu.m/kg C. 100 kJ
D. reheating 20. Pumps connected in series. Which D. 230 kJ
w = 0.004 kg/kg
of the following is not true?
At 35 deg C db and 50% RH:
A. total capacity is the sum of individual 25. The change of enthalpy in going
12. Which of the following is an effect of = =
h 81 kJikg w 0.018 kg/kg
capacities from a solid phase to a vapor phase is:
increa~ing the condensing temperature? A. 5.7
A. mass flow rate per unit capacity B. 7.5
B. total power is the sum of individual A. latent heat of fusion
increases C. 6.7 powers B. latent heat of vaporization
B. the COP increases C. total head is the sum of individual C. latent heat of sublimation
D. 8.5
C. refrigerating effect per unit mass heads D. latent heat of condensation
increases 17. Water from a shell and tube D. the capacity of one pump can be not
D. the efficiency increases condenser enters a cooling tower at 40 equal to the capacity of the other 26. In the Psychrometric Chart, during
degree C. The ambient air entering is at adiabatic saturation process the relative
13. All of the following are the effects of 20 degree C dry bulb and 10 degree C 21. Water is to be raised to a height of humidity becomes:
subcooling the liquid refrigerant in a 30 meters at a rate of 10 kg/s. Heat A. constant
wet bulb temperatures and leaves at 35
vapor compression system, except: loss due to friction is 2 kw. Inlet and exit B. 100%
degree C dry bulb with 50% RH. The
A. the refrigerating effect per unit mass heat rejected in the condenser is 50Q
pipe diameters are 2 inches and 3 c. 0%
increases incMs respectively. Determine the D. saturated
kw. Find the rate of air needed to cool
motor power required to drive this pump.
B. the mass flow rate per unit capacity the water in cubic meters per second.
A. 2.943 kw 27. In a power piping system, the
increases Cooling tower efficiency is 70 %.
C. the work of compression remains the Properties of Air:
B. 4.943 kw corresponding piping color. for safety
same
C. 3.943kw and proper fluids identification for fire
At 20 deg C db and 10 deg C wb:
D. 5.943 kw fighting materials, including detection
D. the efficiency (COP) increases h = 29 kJ/kg v = 0.837 cu. m/kg
and suppression system is ___.
w = 0.004 kg/kg
14. The temperature measurement of At 35 deg C db and 50% RH:
22. A fuel pump is delivering 50 gallons A. safety yellow
. per minute of oil with a specific gravity of B. white
an ordinary thermometer when exposed h = 81 kJikg w; 0.018 kg/kg
0.75. The total head is 30 meters. Find C. safety red
to atmosphere and where the water A. 5
the motor that is capable of handling this D. grey
vapor starts to condense: B. 7
pump. Assume pump efficiency of 70%
A. dry-bulb C. 6
and motor efficiency of 80% 28. In a power piping system, the
B. dew point D. 8
A. 1 hp motor corresponding piping color for safety
C. steam point
B. 1 Y:t hp motor and proper fluid identification for acids
D. critical point 18. Two pumps are connected in series,
C. 1 Y. hp motor and alkalis is ____.
if Q, and H, are lhe discharge and head
D. · 2 hp motor A. green
of pump 1. Q2 and H2 are the discharge
B. violet
REFRESHER MANUAL 2nd Edition by JAS TORDJLLO REFRESHER MANUAL 2nd Edition by JAS TORDILLO
27 - 4 I Day 27 - EXAM Day 27 • EXAM I 27 - 5
Power and Industrial Plant Engineering
C. safety blue and a quality of 95%. The compressor
D. white discharges at a pressure of 100 psi 45. What is the power save by
(689.5 kPa), liquid ammonia leaves the C. condenser intercooling?
29. The process of treating air so as to condenser at 50 deg F (10 deg C). D. evaporator A. 1.736 kw
control simultaneously its temperature, Properties of Ammonia: B. 2.327 kw
humidity cleanliness and distribution to At 689.5 kPa (tsat = 13.3 deg C) Problems 40 to 46 C. 1.925 kw
meet the requirements to the h = 1568 s = 5.7676 A two stage compressor with first stage D. 3.285 kw
conditioned space. At 10 deg C, hf= 246.531 piston displacement of 94,390 cu. em
A . refrigeration At -28.9 deg C by interpolation per sec is driven by a motor. Motor 46. Find the heat rejected in the
B. air conditioning =
hf 68.5 =
sf 0.49366 output is 35 hp, suction temperature is intercooler in kJ/sec.
C. ducting system hg = 1424.5 sg = 6.0452 22 deg C, volumetric efficiency is 85%, A. 5.777
D. air mixing mechanical efficiency is 95%, the B. 7.777
34. Find the heat absorbed by the intercooler pressure is 30 psi gage, air c. 6.777
30. A mechanical device used in evaporator in kJ/kg. temperature in ahd out of the intercooler D. 8.777
refrigeration system for the purpose of A. 1356.7 are 105 and 44 deg C. Final discharge
increasing the pressure upon the B. 1110.2 pressure is 100' psi gage, suction 47. When air is saturated, the wet bulb
refrigerant. c. 1259.8 estimated 14.5 psi. depression is:
A. pump D. 1010.2 A. zero
B. compressor 40. Determine the volume flow at the B. units
C. expansion valve 35. Find the heat rejected to the suction in cu. m per sec. C. positive value
D. drier condenser kJ/kg. A. 0.09439 D. 100%
A. 1568.5 B. 0.0802315
31 . The actual horsepower delivered by B. 1356.5 c. 0.08439 48. Freon flows through an expansion
the engine to the drive shaft. c. 1321.5 D. 0.07230 11 valve, select the best statement:
A. indicated hp: D. 1125.5 A. The temperature increases
B. friction hp 41 . Find the mass flow rate at the B. the pressure remains constant
C. brake hp 36. Find the COP. suction in kg per sec. C. the internal energy remains constant
D. power input A. 5.25 A. 0.09471 D. the enthalpy remains constant
B. 5 B. 0.09438
32. The process of supplying or c·. 4.25 C. 0.08555 49. The evacuation of air or any other
removing air by natural or mechanical D: 4 D. 0.08167 designated gas from a duct line, pipe
means to or from any space. line, container or furnace.
A . ventilation 37. Find the horsepower per ton of 42. , What is the air power in the first A. cavitation
B. evaporation refrigeration. stage in kw? B. purge
C. air supply A. 0.987 A. 9.25 kw C. exhauster
D. natural convection B. 1.044 B. 11.2 kw D. discharging
C. 0.897 C. 10.25 kw
33. In a certain reciprocating engine, W D. 1.404 D. 15.6 kw 50. A term used to describe the melting
is brake power engine, OA is the heat of ash or the melting of ice.
input by fuel and QR Is the heat rej ected. 38. Determine the quality of.refrigerant 43. What is the air power in the second A. evaporation
How to express the thermal efficiency of upon entering to the evaporator. stage? B. vaporization
the engine? A. 10.5% A. 8.01 kw C. sublimation
A. E = QAlQR B. 12.5% B. 8.96 kw D. fusion
B. E=W/~ c. 11.5% C. 8.75 kw
C. E = Qp,IW D. 13.1% D. 9.75 kw
D. E = W/(h. - QR
39. A vessel permanently connected to 44. What is the compressor efficiency?
Problems 34 to 38 a system by inlet and outlet pipes for A. 55.5%
An ammonia refrigeration compressor storage of a liquid refrigerant. B. 88.8%
takes its suction from the evaporator, at A. oil separator c. 66.6%
a temperature of -20 deg F (-28.9 degC) B. liquid receiver D. 77.4%

REFRESHER MANUAL 2nd Edition by JAS TORDILLO REFRESHER MANUAL 2nd Edition by JAS TORDILLO
27 - 6 1 Day 27 - Solution Day 27 • Solution I 27 • 7
Power and Industrial Plant Engineering
DAY 27 - SOLUTION
(c-3 ) c. 7* have to be cooled at -5 degree C. What
D. 9 is the refrigerator capacity in tons of
1. A refrigeration cycle has a Coefficient
DAY 27 - ANSWER KEY refrigeration, if it is to cooled in 15
of Performance of 5. The power input to
1,8 16. A 31.C46.A hours? The specific heat of eggs above
2.017. 0- 32.A47.A the compressor is 3 kw. Compute the COP= QA and below freezing is 0.95 kcallkg-deg
3. C 18. A 33. 8 48. 0 rate of heat rejection from the refrigerant We C and 0.4 kcal/kg-deg C, respectively.
4. C 19. A 34.849.8 to the environment.
5. C 20. A 35. C 50. D 1253 The latent heat of fusion is 68.5 kcallkg.
A. 12 kw
6. 0 21. 8 36.A The freezing temperature of egg is -3
7. 8 22. 8 37. C
B. 18 kw * COP =__§Q_,
C. 15 kw 4(0.746) degree C.
8. 0 23. 8 38. D
D. 20 kw A. 25tons
9. 0 24. 8 39. 8
10. 825.C 40.8 B. 21 tons
COP = 7 C. 23 tons
11 .A26. B 41 .A
12. A 27. C 42. C COP= QA D. 19 tons*
13.828.8 43. 8 We 4. A refrigeration system operates on
14. 8 29. 8 44. 0 an ideal vapor compression using R-12 QA = m[Cp1(t1-tr) + hL + Cp2(tr- hll
15. 030. 8 45.A 5= QA with a vaporizing temperature of minus
10 degree C and a condensing
G 3
OA = 15kw temperature of 30 degree C. What is QA = lO(IOOO) (0.9S(IS- -3) + 68.5+ 0.4(-3- -S)~.187
15(3600)
the· refrigerating capacity in tons of
QR=Wc.+~ refrigeration if the compressor power is ~ = 66.992 kw = 19 TOR
= 3 +15 10 kw?
OR= 18 kw R-12 Properties: 7. A ton of refrigeration is how many
At 30 degree C: BTU per 12 hours?
2. One thousand kilogram of fish at a hf = 228.54 hg = 363.56 h = 375 A. 288,000
temperature of 10 degree C was placed At -10 degree C: B. 144,000*
in a chiller box storage. Determine the hf = 190.82 hg = 347.134 C. 225,000
quantity of heat required to bring the D. 14,400
temperature of the fish to its freezing A. 14.2 TOR
temperature. (Note: Specific heat above B. 11.5 TOR 8. What is the chemical name of R-
freezing• = 0. 76 kcal/kg-deg C; specific C. 12.1 TOR* 727?
heat below freezing = 0.41 kcallkg D. 10.2 TOR A. ammonia
degC; freezing temperature =-2.2 B. Methane
degree C). We= m(h2- h1) C. brine
A. 6532 kcal 10 ='m(375- 347.134) D. air*
B. 7922 kcal m = 0.3588 kg/sec
c. 7229 kcal 9. A pressure control valve inserted at
D. 9272 kcal * QA = m(h1 -h•) the suction side of the evaporator and
= 0.3588(347.134-228.54) sometimes used to protect the
~ = mCp{T1- Tf) = 42.56 kw compressor to overload due to excess
QA: 1000(0.76)(10-2.2) = 12.1 tons of refrigeration amount of heat absorbed by the
QA = 9272 kcal evaporator coils. What do you call this
5. Liquid refrigerant passing through the device?
3. A system needs 4HP to compress evaporator coils is vaporized: A. relay
vapor refrigerant. The amount of heat A. due to the heat from the condenser B. condenser
per min absorbed by the cooling coils B. due to the heat of compression C. solenoid valve
from the warm products is 1253 kJ C. due to the absorbed heat by coils* D. thermostatic expansion valve *
What is the performance of the D. due to heat rejected in the condenser
refrigerator? 10. How many tons of ice can a 10-ton
A. 6 6. A refrigeration plant stored 10 metric ice plant produce in one day?
B. 8 tons of eggs at a temperature of 15 A. 24
degree C. To preserve the eggs they
REFRESHER MANUAL 2nd Edition by JAS TORDILLO REFRESHER MANUAL 2nd Edition by JAS TORDILLO
27 - 8 1 Day 27 - Solution Day 27 - Solution 127 - 9
B. 10 *
Power and Industrial Plant Engineering
16. Water from a shell and tube
C. 20 condenser enters a cooling tower at 40 A. 5 B. total power is, the sum of individual
D. 1 degree C. The ambient air entering is at B. 7 powers
20 degree C dry bulb and 10 degree C C. 6 C. total head is the sum of individual
A 10 tons of refrigeration (TOR) capacity wet bulb temperatures and leaves at 35 D. 8* heads
plant is capable of producing 10 tons of degree C dry bulb with 50% RH. The . D. the capacity of one pump can be not
ice in 24 hours or one day. heat rejected in the condenser in liters equal to the capacity of the other
per second. Cooling tower efficiency is Eff = ta- tb
11. When there is an increase in 75%. ta -twb 21 . Water is to be raised to a height of
humidity ratio to air, it means: Properties of Air:
A. humidifying * · 0.70 = 40- tb 30 meters at a rate of 10 kg/s. Heat
At 20 deg C db and 10 deg C wb: . 40-10 loss due to friction is 2 kw. Inlet and exit
B. dehumidifying h = 29 kJ/kg v = 0.837 cu.m/kg pipe diameters are 2 inches and 3
C. cooling w = 0.004 kg/kg tb = 19°C
inches respectively. Determine the
D. reheating motor power required to drive this pump.
At 35 deg C db and 50% RH: OR = mCp(ta - tb)
500 = m(4.187)(40 -19) A 2.943 kw
12. Which of the following is an effect of = =
h 81 kJ/kg w 0.018 kg/kg
m= 5.686 kg/s B. 4.943 kw*
increasing the condensing temperature? C. 3.943kw
A. mass flow rate per unit capacity A. 5.7* 5 686
V= · = 5.686 li/sec D. 5.943 kw
increases* B. 7.5 ')
B. the COP increases· C. 6.7 M = 10 kg/s 0 = 0.01 m3/sec
C. refrigerating effect per unit mass D. 8.5 mwCpw(ta - tb) = ma(h2- ht)
increases 5.686(4. 187)(40 -1 9) = ma(81- 29) Motor Power = OyH + PL
D. the. efficiency increases 1a -tb m. = 9.615 kg/sec = (0.01 )(9.81 )(30) + 2
Eff =
1a -twb = 4.943 kw
13. All of the following are the effects of Va = maVt = 9.615(0.837)
subcooling the liquid refrigerant in a 0.70 = 40-tb v. = 8.04 m3/sec 22. A fuel pump is delivering 50 gallons
vapor compression system, except: 40-10 per minute of oil with a specific gravity of
A. the refrigerating effect per unit mass It, = 19°C 18. Two pumps are connected in series, 0.75. The total head is 30 meters. Find
increases if Ot and Ht are the discharge and head the motor that is capable of handling this
B. the mass flow rate per unit OR = mCp(ta -tb) of pump 1. 02 and H2 are the discharge pump. Assume pump efficiency of 70%
capacity Increases * 5oo = m(4.187)(4o -19) and head of pump 2. If Ht> H2 and o,< and motor efficiency of 80%
C. the work of compression remains the m= 5.686 kg/s 0 2, what is the discharge of the pump? A 1 hp motor
same A. Ot *
5.686 B. 1 Yz hp motor*
D. the efficiency (COP) increases V = · - - = 5.686 11/sec B. 0., +02 C. 1 Y. hp motor
I
C.~ D. 2 hp motor
14. The temperature measurement of D. 02-01
an ordinary thermometer when exposed 17. Water from a shell and tube 3
0 =50 gal/min= 0.0031545 m /sec
to atmosphere and where the water condenser enters a cooling tower at 40 19. What is the head developed if the
vapor starts to condense: degree C. The ambient air entering is at Qyh
pump that has a capacity of Ot and Brake Power = -
A. dry-bulb 20 degree C dry bulb and 10 degree C head H1 connected in parallel with 11
B. dew point* wet bulb temperatures and leaves at 35 another pump with a capacity of 02 and 0.0031545{9.81x0.75) (30)
C. steam point degree C dry bulb with 50% RH. The Brake Power =
head H2, if Ot > 02? 0.70
D. critical point heat rejected in the condenser is 500 A. Ht *
kw. Find the rate of air needed to cool = 0.995 kw = 1.33 hp use 1 % hp
B. H, + H2
15. What is the humidity ratio of dry air the water in cubic meters per second. motor
C. H2-Ht
in kg per kg? Cooling tower efficiency is 70 %. D. H,/H2
A. 100 Properties of Air: 23. A heat transfer be1ween a solid
B. 0.01 At 20 deg C db and 10 deg C wb: surface and a moving fluid is called:
20. Pumps connected in series. Which
C. 50 = =
h 29 kJ/kg v 0.837 cu. m/kg of the following is not true?
A. conduction
D. zero* w = 0.004 kg/kg . B. convection *
A. total capacity is the sum of C. radiation
At 35 deg C db and 50% RH: individual capacities *
h = 81 kJ!kg w = 0.018 kg/kg D. thermal radiation
REFRESHER MANUAL 2nd Edition by JAS TORDILLO REFRESHER MANUAL 2nd Edition by JAS TORDILLO
- ---. ------------------------;::--~

27 -10 1 Day 27- Solution


Day 27 - Solution 127 - 11
24. A pi;:;ton cylinder contains 2 kg of C. safety blue Power and Industrial Plant Engineering
steam, which expands from state 1 with D. white
an internal energy of 2700 kJ/kg to state and a quality of 95%. The compressor 37. Find the horsepower per ton of
2 with an internal energy of 2650 kj/kg. discharges at a pressure of 100 psi refrigeration.
29. The process .of treating air so as to
During the expansion process the (689.5 kPa), liquid ammonia leaves the A. 0.987
control simultaneously its temperature,
system receives 30 kJ of heat. condenser at 50 deg F (10 deg C). B. 1.044
humidity cleanliness and distribution to
Determine the system work. meet the requirements to the
Properties of Ammonia : c. 0.897 *
A. 70 kJ At 689.5 kPa (tsat = 13.3 deg C) D. 1.404
conditioned space.
B. 130 kJ * h=1568; s=5.7676
A. refrigeration
C. 100 kJ B. air conditioning •
At 10 deg C, hf = 246.531 QA =m(h1 - h4)
D. 230 kJ At -28.9 deg C by interpolation 3.516 =m( 1110.17)
C. ducting system
hf = 68.5 sf = 0.49366 m:: 0.003167 kg/sec
D. air mixing
W = m(U1- Uz) hg = 1424.5 sg = 6 .0452
w = 2(2700- 2650) 30. A mechanical device used in
34. Find the heat absorbed by the
0.003167(211.3) h
w = 100 kJ refrigeration system for the purpose of
evaporator in kJ/kg.
We = 0. 746 . p = 0.897 _':!!:_
increasing the pressure upon the QA ton . ton
Wsys = t.U + QA refrigerant. A. 1356.7
= 30 + 100 A. pump
B. 1110.2* 38. Determine the quality of refrigerant
Wsy• = 130 kJ C. 1259.8 upon entering to the evaporator.'
B. compressor*
D. 1010.2
C. expansion valve A. 10.5%
25. The change of enthalpy in going D. drier B. 12.5%
h, = hf + x(hfg)
from a solid phase to a vapor phase is: C. 11.5%
h, = hf + x(hg-hf)
A. latent heat of fusion 31. The actual horsepower delivered by D. 13.1 % *
B. latent heat of vaporization h, = 68.5 + 0.95( 1424.5- 68.5)
•the engine to the drive shaft.
= 1365.7 kJ/kg
c. latent heat of sublimation * A. indicated hp h3 = h4 = hf +xhfg
D. latent heat of condensation B. friction hp 245.53 = 68.5 + x(1424.5 - 68.5)
QA : h,- h4
C. brake hp * X= 0.131 = 13.1%
QA = 1356.7 = 246.531
26. In the Psychrometric Chart, during D. p~wer input
adiabatic saturation process the relative QA = 1110.17 kJ/kg 39. A vessel permanently connected to
humidity becomes: 32. The process of supplying or a system by inlet and outlet pipes for
35. Find the heat rejected to the
A. constant removin~ air by natural or mechanical storage of a liquid refrigerant.
B. 100% • condenser kJ/kg.
means to or from any space. A. oil separator
A. 1568.5
C. 0% A. ventilation * B. liquid receiver *
B. 1356.5
D. saturated B. evaporation C. condenser
C. air supply
c. 1"321.5 * D. evaporator
D. 1125.5
27. In a power piping system, the D. natural convection
corresponding piping color for safety Problems 40 to 46
QR = hz- h3 ·
and proper fluids identification for fire 33. In a certain reciprocating engine, W A two stage compressor with first stage
QR = 1568 - 246.531. = 1321.47 kJ/kg
fighting materials, including detection is brake power engine, QA is the heat piston displacement of 94,390 cu. em
and suppression system is _ _ _. input by fuel al)d QR is the heat rejected. per sec is driven by a motor. Motor
A. safety yellow 36. Find the COP. output is 35 hp, suction temperature is
How to express the thermal efficiency of
A. 5.25 * 22 deg C, volumetric efficiency is 85%,
B. white the engine?
B. 5
C. safety red * A. E = QAI/QR mechanical efficiency is 95%, the
D. grey B. E = W/QA * c. 4.25 intercooler pressure is 30 psi gage, air
C. E=QA/W D. 4 temperature fn and out of the intercooler
28. In a power piping system, the D. E = W/QA - QR are 1 05 and 44 deg C. Final discharge
corresponding piping color for safety We =h 2 -h 1 =1568-1356.7=211.3 kJ/k pressure is 100 psi gage, suction
and proper fluid identification for acids Problems 34 to 38 QA 1110.17 estimated 14.5 psi.
and alkalis is _ _ __ . An ammonia refrigeration compressor COP= ---- = - - -·· = 5.25
We 211.3
A. green takes its suction from the evaporator, at 40. Determine the volume flow at the
B. violet* a temperature of -20 deg F (-28.9 degC) suction in cu. m per sec.
A. 0.09439
REFRESHER MANUAL 2nd EditioQ by JAS TORDILLO REFRESHER MANUAL 2nd Edition by JAS TORDILLO
27- 12 1 Day 27- Solution Day 27 - Solution 127 - 13
B. 0.0802~15 * 43. What is the air power in the second Power and Industrial Plant Engineering
c. 0.08439 stage?
Air Power
D. 0.0723011 A 8.01 kw 50. A term used to describe the melting

Vo = 94390 cm 3/sec = 0.09439 m 3/sec


B. 8.96 kw*
C. 8.25 kw
~ nmRT2
n- I
[(P2Jn~l _1]
Px
of ash or the melting of ice.
A. evaporation
D. 9.75 kw B. vaporization
v,·
v,·
v,
= Vo (nv)
= o.o9439(0 85)
= 0.0802315 m3/sec

41. Find the mass flow rate at the


.r power= -nmRT2
A1
n -1
[(p )~~
_1..
Px
2
- 1l ll
1.2824(0.0947 1)

' 790.61)
308 .I I
I.Z8 · 1
UK -I
( 0.287)
1.2824 -I

]
( lOS + 273) x C. sublimation
D. fusion*

suction in kg per sec. 1.2824(0.09471) (0.287) (44 + 273) = 10. 76 kw


----~--~~--~~----~x
A. 0.09471 *
1.2824 - 1 Total Power w/out intercooler
B. 0.09438

·[(790.61)¥~;
1 = 10.25 +10.76 = 21 kw
C. 0.08555
D. 0.08167 - 1]
308. 11 Power Saved= 21 -19.27 = 1.736 kw
p, = 14.5 psi= 99.946 kPa
A ir Power = 8.96 kw 46. Find the heat rejected in the
P,V,' = mRT, intercooler in kJ/sec.
(99.946)(0.0802315) 44. What is the compressor efficiency? A. 5.777 *
= (m(0.287)(22+273) A. 55.5% B. 7.777
B. 88.8% c. 6.777
m = 0.09471 kg/sec
C. 66.6% 0. 8.777
D. 77.4% *
42. What is the air power in the first
stage in kw? QR = mCp(Tx- T,)
Brake Power= 35 x0.746 x 0.95 QR = 0.09471(1 .0)(105- 440
A. 9.25 kw
= 24.8 kw OR= 5.777 kJ/sec
B. 11 .2 kw
Total Power = 10.25 + 8.96 = 19.2 kw
C. 10.25 kw *
D. 15.6 kw Comp eft= Power outp~ =_!_~~.owe~ 47. When air is saturated, the wet bulb
Power input Brake power depression is:
Px = 30 psig = 308.11 kPa abs Comp eff = ~x 100=77.4%
A. ·zero*
24.8 B. units
P2 = 100 psig = 790.61 kPa abs
C. positive value
n-1 45. What is the power save by D. 100%
Tx = (px) n intercooling?
A. 1.736 kw*
Tl PI 48. Freon flows through an expansion
B. 2.327 kw valve, select the best statement:
n-1
44 + 273 . = ( 308 ·!J..) n'
C. 1.925kw
D. 3.285 kw
A. The temperature increases
B. the pressu re remains constant
22 + 273 99.946
C. the internal energy remains constant
n = 1.2824 Air Pow~r (no intercooler, T2 = 105°C) in D. the enthalpy remains constant*

AP·"~~~~[(;.f -1]
second stage:
49. The evacuation of air or any other
designated gas from a duct line, pipe
line, container or furnace.

i
1.28-1 ] A. cavitation
AR=. 1282Cl094X(}28W2t-2 (30ll ~ 1.28 -1
118241 9'})4~ B. purge*
C. exhauster
D. discharging
Air Power = 10.25 kw

REFRESHER MANUAL 2nd Edition by JAS TORDILLO REFRESHER MANUAL 2nd Edition by JAS TORDILLO
Day 28 - EXAM I 28 - 1
Power and Industrial Plant Engineering
C. H is inversely proportional to the
DAY 28- EXAM cube of S
D. H is directly proporti.onai to the
1. Enthalpy is measurement of square of S
A. coldness
B. heat · 7. What is likely to occur when the
C. moisture sections of the impeller of a centrifugal
D. energy pump are handling vapor and the other
are handling liquid?
2. equipments or tools which A. erosion of the pump
are powered by air. . B. excessive vibration
A. hydraulic C. high and low capacity
B. metallic D. complete failure to operate
C. pneumatic
D. compressor 8. If Hs is the sensible heat and Ht is
the total heat transferred, determine the
3. If the available Net Positive Suction sensible heat factor of the coils.
Head of pump is higher than the A. Ht- Hs
required Net Positive Suction Head, the B. Hs- Ht
pump will ____.
C. HUHs
A. not cavitate D. Hs/Ht
B. experienced cavitation
c. cavitate 9. The diesel cycle is the ideal cycle for
D. not exist A. spark-ignition engine
B. Brayton cycle
4. A water-tube condenser has a total C. compression-ignition engine
of 90 tubes . If these type of condenser D. steam-jet cycle
are two passes, then compute the
number of tubes per pas~. 10. If the maximum and m1n1mum
A. 30 tubes temperatures of a Camet and Rankine
B. 60 tubes cycles are the same, then compare the
C. 45 tubes efficiency of Rankine cycle to the
D. 180 tubes efficiency of Carne! cycle.
A. efficiency is the same
5. If Ev is the volumetric efficiency of a B. .lower efficiency
reciprocating pump,then determine the C. efficiency is higher by 10%

- percent SLIP of the pump.


A. 1/Ev
B. Ev + 1
C. 1 - Ev
D. higher efficiency

11 . What is the turbine output of a


steam power plant having a turbine work
D. 1 ..: 1/Ev of 550 kJ/kg and pump work of 5 kJ/kg if
the mass flow rate is 15 kg/s?
6. What is the relationship of the head A. 8250 kw
of a centrifugal pump, H, to the impeller B. 9500 kw
speedS? C. 7250 kw
A. H is inversely proportional to the D. 8175 kw
square if S
B. H is directfy proportional to-the cube 12. What is the power produced by a
of S steam power plaht having a turbine work

REFRESHER MANUAL 2nd Edition by JAS TORDILLO


28 - 2 I Day 28 - EXAM Day 28 - EXAM I 28 - 3
Power and Industrial Plant Engineering
of 550 kJ/kg and a pump work of 5 kJ/kg A. specific volume
if the mass flow rate is 15 kg/s? B. specific heat 23. Determine the amount of make-up A. zero
A. 8250 kw C. BTU water requirement in kg/hr? B: maybe positive, negative or zero
B. 9500 kw D. total heat A. 8.888 C. greater than zero ·
C. 7250 kw B. 7,777 D. none of the above
D. 8175 kw 19. What is the specific gravity of brine C. 6,666
which has a Baume reading of 30 D. 5,555 29. An ideal gas is compressed in a
13. The motor turns 1500 rpm with a degrees? cylinder so well insulated that there is
torque of 10 N-m. Calculate the brake A. 0.785 24. What is the percentage of make-up essentially no heat transfer. The
power of the motor if frequency is 60 hz. B. 0.587 water to the cooling water flow? temperature of the gas:
A. 1571 watts c. 0.857 A. 1.11% A. decreases
B. 1751 watts D. 0.875 B. 3.33% B. increases
C. 1517 watts C. 2.22% C. remains constant
D. 1157watts 20. If RE is the amount of heat added to D. 4.44% D. unity
the refrigerant or heat removed from the
14. When a generator nameplate rating products and W is the compressor Problems 25 to 26 30. Name the process that has no heat
is 5 MW, then what does a 5 MW rating power input, then ehat is the efficiency Water substance at 70 bar and 65 deg transfer.
refers to? of the refrigeration cycle? C enters a boiler tube of constant inside A. isothermal
A. electrical power input A. REIW diameter of 25 mm. The water leaves B. polytropic
B. thermal energy output B. 1- REIW the boiler tube at 50 bar and 700 deg K C. isentropic
C. electrical power output C. WIRE at a velocity of 100 m/s. D. adiabatic
D. mechanical power output D. RE/W-1 Water Properties:
At 70 bar and 65 deg C: 31 . The first law of thermodynamics:
15. When an electric motor nameplate Problems 21 to 24 v = 0 ..001017 cu. m/kg A. internal energy is due to molecular
rating is 10 HP, then what does a 10 HP 250,000 kg/hr of water at 35 deg C motion
rating refers to? enters a cooling tower where it is to be At 50 bar and 700 deg K: B. entropy of the universe is increased
A. electrical power input cooled to 17.5 deg C. The energy is to V = 0.06081 cu. m/kg by irreversible processes
B. mechanical power input be exchanged with atmospheric air C. energy can be either created nor
C . electrical power output entering the unit at 15 deg C and 25. Calculate the inlet velocity. destroyed
D. mechanical power output leaving the unit at 30 deg C. The air A. 1.672 m/s D. heat energy cannot be completely
enters at 30% RH and leaves at 85% B. 72.3 m/s transformed into work
16. Load factor is the ratio of _ _ __ RH. C. 7.271 m/s
load to the peak load during the same Air Properties: D. 95.1 m/s 32. Select the most efficient
peri'Od of time. At 15 deg C and 30% RH: thermodynamic cycle from the following:
A. maximum h =
23 kJ/kg w = 0.0032 kg/kg 26. Calculate the inlet volume flow in A. Brayton
B. average liters per second. B. Carnot
C. minimum At 30 deg C and 85% RH: A. 0.8207 C. Otto
D. maybe average or maybe minimum h = 89 kJ/kg w = 0.0232 kg/kg B. 0.7208 D. Diesel
C. 0.6702
17. If Cis the capacity of a pump, His 21 . Determine the range of the tower. D. 0.9280 33. How much heat must be removed
the total pumping head and e is the A. 35 deg K from 400 lbs of beef to cool from 92 deg
efficiency of the pump, then compute B. 17.5 deg K 27. An ideal gas is compressed F to 32 deg F?
the brake power of the pump. C. 25 deg K isothermally. The enthalpy change is: A. 16940 Btu of heat
A. C x H/e D. 15degK A. sometimes negative B. 11396 Btu of heat
B. 1- CxH/e B. always positive C. 28336 Btu of heat
C. C/H x e 22. Determine the rtlass flow rate of air C. indeterminate D. none of the above
D. e/CxH required to cool the e water in kg/hr. D. zero
A. 277,746 34. Enthalpy of an ideal gas is a
18. The amount of heat necessary to B. 176.~20 28. A system with paddle wheel, work is function only of: ·
bring up the temperature of a unit mass C. 250,000 irreversible, therefore, the change in its A. internal energy
of a substance through a unit degree is D. 112,587 entropy is: B. temperature

REFRESHER MANUAL 2nd Edition by JAS TORDILLO REFRESHER MANUAL 2nd Edition by JAS TORDILLO
28-41 Day28-EXAM Day 28 - EXAM I 28 - 5
Power and Industrial Plant EnQineering
C. entropy C. 0.229
D. pressure D. 0.127 application where refrigerant ammonia 50. Dry air can be approximated as
is employed. _ _% oxygen and _ _% nitrogen
Problems 35 to 36 39. Determine the brake thermal A. plastic by mole numbers. ·
Magnolia Dairy Products plan must cool efficiency. B. steel A 30/70
4000 gallons of fresh milk receive from A. 45.1% C. brass B. 21/79
the farm per day form an initial B. 34.1% D. copper C. 70/30
temperature of 80 deg F to a C. 41.1% D. 79/21
temperature of 38 deg F in 5 hours. If D. 29.1% 45. One kilogram of air is compressed
the density of milk is 8.6 lbs/gallon, adiabatically. The compression
specific gravity is 1.03, and specific heat 40. Determine the indicated thermal efficiency is 80% and the work done is
is 0.935 Btu per lb per deg F. efficiency. 300 kJ/kg. Determine the heat added to
A 67.7% air. DAY 28- ANSWER KEY
B. 42.5% A. 240 kJ 1.8 16. 8 31.C46.A
35. What must be the capacity of the 2. C 17. A 32. 8 47. D
refrigerating machines in tons? c. 51.2% B. 375 kJ 3.A 18. 8 33.048.A
A 20 D. 35.5% c. 300 kJ 4. C 19. 0 34. 849.8
B. 25 D. 0 5. .C 20. A 35. C 50. 8
c. 23 41 . A refrigeration plant stored 10 6. 0 21.8 36.8
metric tons of eggs at a temperature of 46. The heating value obtained when 7. C 22. A 37.A
D. 30 B. 0 23. 0 38. 8
15°C. To preserve the eggs by not the water in the products of combustion 9. C 24. C 39. 0
36. How much brine must be circulate if spoiling them, they have to be cooled at is in the liquid-state: 10. B 25. A 40. D
the change in temperature is 15 deg R, -8°C. W hat is the refrigerating capacity A. higher heating value 11 . A26. A 41.A
specific gravity is 1.182, and specific of the plant in tons, if it is cooled in 12 B. lower heating value 12.027.0 42.A
C. calorific value 13. A28. C 43. 8
heat if brine is 0. 729 Btu per lb per deg hou·rs? The specific heat of the eggs 14. C29. 8 44. B
F? above and below freezing are 0.95 D. average heat value 15. 030. 0 45. 0
A. 30GPM kcallkg-°C and 0.4 kcal/kg-°C,
B. 42 GPM respectively. The latent heat of fusion is 47. How many tons of ice can a 10 ton
C. 38 GPM 68.5 kcal/kg. The freezing temperature ice plant produce in a day?
D. 95 GPM is -3°C. A. 24 tons
A. 24.1 B. 20 tons
Problems 37 to 40 B. 54.1 C. 15 tons
Performance values of a 3 MW Diesel c. 34.1 D. 10 tons
generating set are as follows: D. .64.1
Fuel.rate, 1.5 barrels for 900 kwh of 25 48. A reserved carnot cycle is operating
deg API fuel 42. A refrigeration system needs 9 hp under the temperature limits -10 deg C
Generator efficiency, 92 percent to compress a liquid, 15 kw was and 288 deg K. Find the COP.
Mechanical efficiency, 81 percent extracted from the cooled space. W hat A 10.52
Voltage, 440/220, three-phase, 60 hertz is the coefficient of performance? B. 12.52
A. 2.23 C. 11.52
37. Determine the fuel consumed for B. 3.23 D. 15.52
900 kwh energy produced. c. 2.93
A. 271.2 kg D. 3.43 49. 100 grams of water at 75 deg is
B. 172.2 kg heated at constant, pressure. The water
C. 227.1 kg 43. V iscosity is a measure of: is completely vaporized. What is the
D. 122.7 kg A. volume heat added?
B. thickness A . 336 kJ
C . temperature B. 236 kJ
38. Determine the engine-fuel rate in kg D. pressure c. 296 kJ
per kw-hr. D. 196 kJ
A. 0.300 44. Which of the following material is
B. 0.277 suitable for tubing in refrigeration

REFRESHER MANUAL 2nd Edition by JAS TORDILLO REFRESHER MANUAL 2nd Edition by JAS TORDILLO
- - - - ___,=-----

28 - 6 I Day 28 - Solution Day 28 - Solution 1 28 - 7


B. H is directly proportional to the cube
Power and Industrial Plant Engineering
DAY 28 - SOLUTION of S 12. What is the power produced by a 17. If C is the capacity of a pump, H is
C. H is inversely proportional to the steam power plant having a turbine work
Enthalpy is measurement of the total pumping head and e is the
cube of S of 550 kJ/kg and a pump work of 5 kJ/kg. · efficiency of the pump, then compute
A. coldness D. H is directly proportional to the
B. heat • if the mass flow rate is 15 kg/s? ~ the brake power of the pump.
square of S * A. 8250 kw
C. moisture A. C x H/e *
D. energy B. 9500 kw B. 1- CxH/e
7. What is likely to occur when the C. 7250 kw C. C/H x e
sections of the impeller of a centrifugal D. 8175 kw* D. · e/CxH
2. equipments or tools which pump are handling vapor and the other
are powered by air. are handling liquid?
A. hydraulic Wr = Wr-WP 18. The amount of heat necessary to
A. erosion of the pump WN = 15(550) -15(5) bring up the temperature of a unit mass
B. metallic B. excessive vibration
C. pneumatic * WN = 8175 kw of a substance through a unit degree is
C. high and low capacity • A. specific volume
D. compressor D. complete failure to operate 13. The motor turns 1500 rpm with a B. specific heat*
3. If the available Net Positive Suction torque of 10 N-m. Calculate the brake C. BTU
8. If Hs is the sensible heat and Ht is power of the motor if frequency is 60 hz. D. total heat
Head of pump is higher than the the total heat transferred, determine the
required Net Positive Suction Head, the A. 1571 watts*
sensible heat factor of the coils. B. 1751 watts
pump will 19. What is the specific gravity of brine
A. Ht- Hs C. 1517 watts
A. not ca-v77
it-at:-e--:,.:-- which has a Baume reading of 30
B. Hs - Ht D. 1157 watts degrees?
B. experienced cavitation C. Ht!Hs
C. cavitate A. 0.785
D. Hs/Ht * BP = 2rrTN B. 0.587
D. not exist
c. 0.857
4. A water-tube condenser has a total
9. The diesel cycle is the ideal cycle for
A. spark-ignition engine
BP = 2rr(lof~~?) D. 0.875 *
of 90 tubes. If these type of condenser B. Brayton cycle BP = 1570.8 watts
are two passes, then compute the 0 140
C. compression-ignition engine* Baume = SG -130
number of tubes per pass. D. steam-jet cycle 14. When a generator nameplate rating
A. 30 tubes
B. 60 tubes
is 5 MW, then what does a 5 MW rating 30 = 140 -130
10. If the maximum and mmtmum refers to? · • SG,
C. 45 tubes* temperatures of a Carnot and Rankine A. electrical power inpu1 SG = 0.875
D. 180 tubes cycles are the same, then compare the B. thermal energy output
efficiency of Rankine cycle to the C. electrical power output* 20. If RE is the amount of heat added to
5. If Ev is the volumetric efficiency of a efficiency of Carnot cycle. D. mechanical power output the refrigerant or heat removed from the
reciprocating pump,then determine the A. efficiency is the same products and W is the compressor
percent SLIP of the pump. B. lower efficiency * 15. When an electric motor nameplate power input, then ehat is the efficiency
A. 1/Ev C. efficiency is higher by 10% rating is 10 HP, then what does a 10 HP of the refrigeration cycle?
B. Ev + 1 D. higher efficiency rating refers to? A. RE/W*
C. 1- Ev * A. electrical power input B. 1- REIW
D. 1- 1/Ev 11. What is the turbine output of a B. mechanical power input C. WIRE
steam power plant having a turbine work C. electrical power output D. REIW - 1
Q = 1 _ _9_ = 1- Ev
.
% shp = v;;-
V0 -
V
0
of 550 kJ/kg and pump work of 5 kJ!kg if
the mass flow rate is 15 kg/s?
D. mechanical power output*
Problems 21 to.24
A. 8250 kw * 16. Load factor is the ratio of _ _ __ 250,000 kg/hr of water at 35 deg C
B. 9500 kw load to the peak load during the same enters a cooling tower where it is to be
6. What is the relationship of the head
C. 7250 kw period of time. cooled to 17.5 deg C. The energy is to
of a centrifugal pump, H, to the impeller
D. 8175 kw A. maximum be exchanged with atmospheric air
speedS?
B. average • entering the unit at 15 deg C and
A. H is inversely proportional to the
WN = 15(550) = 8250 kw C. minimum leaving the unit· at 30 deg C. The air
square if S
D. maybe average or maybe minimum enters at 30% RH and leaves at 85%
RH.
REFRESHER MANUAL 2nd Edition by JAS TORDILLO REFRESHER MANUAL 2nd Edition by JAS TORDILLO
28 - 8 1 Day 28 - Solution Day 28 - Solution 128 - 9
Power and Industrial Plant Engineering
Air Properties: Problems 25 to 26
At 15 deg C and 30% RH: Water substance at 70 bar and 65 deg B. always positive C. 28336 Btu of heat
h = 23 kJ/kg w = 0.0032 kg/kg C enters a boiler tube of constant inside C. indeterminate D. none of the above *
diameter of 25 mm. The water leaves D. zero*
At 30 deg C and 85% RH: the boiler tube at 50 bar and 700 deg K 34. Enthalpy of an ideal gas is a
h = 89 kJ/kg w = 0.0232 kg/kg at a velocity of 100 m/s. 28. A system with paddle wheel, work is function only of:
Water Properties: irreversible, therefore, the change in its A. internal energy
21. Determine the range of the tower. At 70 bar and 65 deg C: entropy is: B. {em perature*
A. 35 deg K v = 0.001017 cu. m/kg A. zero C. entropy
B. 17.5 deg K * B. maybe positive, negative or zero D. pressure
C. 25 deg K At 50 bar and 700 deg K: C. greater than zero *
D. 15 deg K · V = 0.06081 cu. m/kg D. none of the above Problems 35 to 36
Magnolia Dairy Products plan must cool
Range = t. -tb = 35-17.5 = 17.5°C 25. Calculate the inlet velocity. 29. An ideal gas is compressed in a 4000 gallons of fresh milk receive from
A. 1.672 m/s * cylinder so well insulated that there is the farm per day form an initial
22. Determine the mass flow rate of air B. 72.3 m/s essentially no heat transfer. The temperature of 80 deg F to a
required to cool thee water in kg/hr. C. 7.271 m/s temperature of the gas: temperature of 38 deg F in 5 hours. If
A. 277,746 * D. ~5 . 1 m/s A. decreases the density of milk is 8.6 lbs/gallon,
B. 176,920 B. increases * specific gravity is 1:03, and specific heat
C. 250,000 Q C. remains constant is 0.935 Btu per lb per deg F.
D. 112,587 mass=- D. unity
v 35. What must be the capacity of the
m 1 =m2 30.· Name t he process that has no heat refrigerating machines in tons?
mwCPwt.tw = m.(h2 - h1)
250,000(4.187)(17.5)) = ma(89- 23) a,= o2 transfer. A. 20
m. = 277,547.3 kg/hr A. isothermal B. 25
v1 v2 B. polytropic c. 23 *
23. Determine the amount of make-up ~­ A,V, = A2 V~ but A,= A2 C. isentropic D. 30
water requirement in kg/hr? v1 v2 D. adiabatic *
A. 8.888 m = 4000 Gal x 8.6 lbs = 6880 lb 1hr
B. 7,777 vi= v2 31. The first law of thermodynamics:
5 hours gal
C. 6,666 VI V2 A. internal energy is due to molecular
QA = mCp6t
D. 5,555 * v, 100
-- = ---
motion
= 6880(0.935)(80 - 30)
B. entropy of the, universe is increased
0.001017 0.06081 QA = 270,177.6 Btu/hr
Make-up water required, by irreversible processes
v1 :: 1.672 m/s (1 TOR= 12,000 Btu/hr)
ms = ma(W2 -W1) C. energy can be either created nor
destroyed* QA = 22.51 TOR
= 277,746(0.0232- 0.0032) 26. Calculate the inlet volume flow in
= 5,554.92 kg/hr liters per second.
D. heat energy cannot be completely
36. How much brine must be circulate if
transformed into work
A. 0.8207 * the change in temperature is 15 deg R,
24. What is the percentage of make-up B. 0.7208 32. Select the most efficient specific gravity is 1. 182, and specific
water to the cooling water flow? C. 0.6702
thermodynamic cycle from the following: heat if brine is 0.729 Btu per lb per deg
A. 1.11% D. 0.9280
A. Brayton F?
B. 3.33%
A. 30 GPM
c. 2.22% * 01 = AN1
B. Carnot *
B. 42 GPM *
D. 4.44% c. Otto
C. 38 GPM
= 2:(0.025) 2 (1.672) D. Diesel
4 D. 95 GPM
% make-up water requirement
' = 0.00082074 m3/ sec 33. How much heat must be removed
=~xlOO = 0.82074 liter/sec from 400 lbs of beef to cool from 92 deg QA = msCPa6ts
250,000 F to 32 deg F?- 270,177.6 = ms(0.729)(15)
= 2.22% I 27. An ideal gas is compressed A. 16940 Btu of heat ms = 24,707.6 lbs/hr
isothermally. The enthalpy change is: B. 11396 Btu of heat
A. sometimes negative
REFRESHER MANUAL 2nd Edition by JAS TORDILLO REFRESHER MANUAL 2nd Edition by JAS TORDILLO
28 -10 1 Day 28- Solution
Day 28 -Solution 128 - 11
Problems 37 to 40 40. Determine the indicated thermal Power and Industrial Plant Engineering
Performance values of a 3 MW Diesel efficiency.
generating set are as follows: A 67.7% extracted 'from the cooled space. What 48. A reserved carnot cycle is operating
Fuel rate, 1.5 barre1s for 900 kwh of 25 is the coefficient of performance? under the temperature limits - 10 cjeg C
B. 42.5%
deg API fuel A. 2.23 * and 288 deg K. Find the COP.
C. 51.2%
Generator efficiency, 92 percent D. 35.5% * B. 3.23 A. 10.52 *
Mechanical efficiency, 81 percent C. 2.93 B. 12.52
D. 3.43 C. 11 .52
Voltage, 440/220, three-phase, 60 hertz Indicated Power, IP = BP = 978.26
'lm 0.82
D. 15.52
37. Determine the fuel consumed for = 1193 kw -hr COP=~= 15
=2.23
900 kwh energy produced. We 9(0.746) COP= - ~t,.__ = _:l.~i~!L = t 0.52
A. 271.2 kg~ TH-TL 288-(-10 +273)
B. 172.2 kg Indicated Thermal Eff, lltb = _fiP 43. Viscosity is a measure of:
C. 227.1 kg mQh A volume 49. 100 grams of water at 75 deg is
D. 122.7 kg B. thickness* heated at constant pressure. The water
is completely vaporized. What is the
. 200 li kg
m1 = 1.5 barrel x - - x0.904- 1139 _kJ - hr x 36005 ~ C. temperature
heat added?
D. pressure
barrel li lltb= sec hr xlOO A. 336 kJ
271.2 kg ( 44,620 ~~)
m, = 271 .2 kg B. 236 kj *
44. Which of the following material is
kg suitable for tubing in refrigeration c. 296 kJ
38. Determine the engine-fuel rate in kg D. 196 kJ
T)tb = 35.5% application where refrigerant ammonia
per kw-hr.
is employed.
A 0.300 OA = mCp.it + mhr9
41. A refrigeration plant stored 10 A. plastic
B. 0.277 * QA : 0.1 00(4.187)(100 - 75) +
metric tons of eggs at a temperature of B. steel*
C. 0.229 0.1 00(2257)
15°C. To preserve the eggs by not C. brass
D. 0.127 QA : 236.1 kJ
spoiling them, they have to be cooled at D. copper
-8°C. What is the refrigerating capacity
. 900 50. Dry air can be approximated as
Brake Power = - = 978.26kw- hr of the plant in tons, if it is cooled in 12 45. One kilogram of air is compressed
0.92 hours? The specific heat of the eggs adiabatically. The compression _ _% oxygen and _ _% nitrogen
by mole numbers.
mf = 271.2 = 0. 277 ~ above and below freezing are 0.95 efficiency is 80% and the work done is
A. 30/70
kcallkg-°C and 0.4 kcal/kg-°C, 300 kJ/kg. Determine the heat added to
BP 978.26 kw- hr
respectively. The latent heat of fusion is air. B: 21/79 *
68.5 kcal/kg. The freezing temperature A. 240 kJ C. 70/30
39. Determine the brake thermal D. 79/21
efficiency. is -3°C. B. 375 kJ
A. 45.1%
A. 24.1 ~ c. 300 kJ
B. 34.1% B. 54.1 D. 0 *
c. 41.1% C. 34.1
D. 29.1% ~ D. 64.1 46. The heating value obtained when
the water in the products of combustion
Qh = 41,130 + 139.6(25) QA = m[Ct(lt - tr) + L + Cz(tr- tz)] is in the liquid-state:
A. higher heating value *
= 44,620 kJ/kg
0.95(4.187) (IS +3)+] B. lower heating value
QA = lO(IOOO) 68.5(4.187)+ C. calorific value
BP
Brake Thermal Eff, Tltb = mfQh 12 3600 [ D. average heat value
( ) 0.4(4.187) ( - 3+8)
978.26 kJ - hr x 3600 sec 47. How many tons of ice can a 10 ton
n -
' lib-
sec kJhr xlOO QA = 84.9 kJ/sec = 24.1 TOR ice plant produce in a day?
A. 24 tons
271.2 kg (44,620 ~· )
kg 42. A refrigeration system needs 9 hp B. 20 tons
'l tb = 29.1% to compress a liquid, 15 kw was C. 15 tons
D. 10tons *

REFRESHER MANUAL 2nd Edition by JAS TORDILLO REFRESHER MANUAL 2nd Edition by JAS TORDILLO
.....
Day 29 - EXAM I 29 - 1
Power and Industrial Plant Engineering

DAY 29 - EXAM I· 7. The. method of introduc.ing the fuel


charge mto the power cylinder of an
engine. ·
1. The ratio of the brake HP to indicated A. mechanical injection
HP of an engine: · B. trunk injection
A. brake thermal eff C. suction
B. mechanical eff D. intake
C. indicated eff
D. cycle eff 8. Preferable material for oil and fuel
pipings:
2. The ratio of the heat equivalent of 1 A. steel and iron
hp-hr to the heat units actually supplied B. copper and brass
per bhp-hr, based on the higher heating C. plastic pipes
value of the fuel. D. bronze and aluminum
A. brake thermal eff
B. indicated thermal eff 9. Preferable materials for water piping
C. mechanical eft system If internal combustion engines:
D. cycle eff A. stainless steel
B. galvanized steel
3. The volume in cu ft per minute, C. cast iron
swept by the piston or pistons during the D. black iron
power strokes.
A. bore speed 10. Cooling water used in internal
B. displacement combustion engines must be
C. events noncorrosive with a pH value of _ _.
D. piston speed A. 7
B. 3
4. HP delivered by the shaft at the C. 10
output end. D. 14
A. Indicated HP
B. Brake HP 11. A mixture of aluminum powder and
C. Thermal power a metal oxide powder is ignited by a
D. Airpower special powder in crucible.
A. thermit welding
5. Air at low pressure used to force B. resistance welding
burnt gases out of the power cylinder C. arc welding
during the exhaust period and. By the D. metal welding
displacement, to furnish a supply fresh
air for the following cycle. 12. The process in which gases are
A. turbo used in combination to obtain a hot
B. exhaust flame.
C. scavenging A. hydrogen welding
D. discharge B. spray welding
C. gas welding
6. Utilizes the working medium on only D. slug welding
one side of a single piston.
A. double-acting 13. The maximum temperature
B. single-acting developed for oxy-acetylene welding is
C. dual-acting
D. single-stroke A. 1990 deg C

REFRESHER MANUAL 2nd Edition by JAS TORDILLO

J._
29 - 2 I Day 29 - EXAM Day 29 - EXAM I 29 - 3
Power and Industrial Plant Engineering
B. 3440 deg C D. filtration
C. 2001 deg C C. heat exchanger C. all of these
D. 5660 deg C 20. Reduces the dissolved solids D. weak liquor D. rectifier
content of water. Its main application is
14. Fusion process in which the metal converting brackish water to supply that 26. A condition where in the absorber 31 . Which of the following is under the
is heated to a state of fusion permitting it is suitable for potable use. (from 1000 the water-ammonia takes up ammonia low pressure?
to flow together into a solid joint. ppm to 10,000 ppm). gas until it becomes saturated and holds A. generator
A. flash welding A. electrodialysis as much gas as it can at the B. evaporator
B. metal spray welding B. coagulation temperature and pressure existing in the C. rectifier
C. laser welding C. ion exchange vessel or a liquid saturated with D. NOTA
D. electric-arc welding D. neutralization ammonia is called:
A. strong liquor 32. Which of the following are the three
15. The process of filling the pores of a 21. Where thus the percentage of B. liquid receiver circuits of fluid in the absorption
sintered product with molten metal in moisture and refrigerant vapor in the C. weak liquor refrigeration system?
order to decrease porosity or to improve absorption last rejected? . D. slight liquor A. the ammonia circuit from the
physical properties. A. analyzer genera~or
A. embedding • B. generator 27. A component in absorption · B. the strong-liquor circuit from the
B. ironing C. condenser refngeration which raises the pressure absorber to the generator
C. piercing D. rectifier of strong liquor and sends it through a C. the weak-liquor circuit from the
D. infiltration heat exchanger: generator back to the absorber
22. A component used in absorption A. pump D. all of these
16. The operation of cutting out flat refrigeration so that the vapor from the B. absorber
area to some desired shape. evaporator is coming into contact with C. generator 33. The energy which must be removed
A. blowing the weak liquor. D. expansion valve from warmer air entering the refrigerated
B. blanking A. absorber space.
C. bending B. heatexchanger 28. Serves as a heat exchanger of A. heat energy
D. swaging C. evaporator ammonia vapor from the generator and B. infiltration
D. intrcooler tend s to throw back a portion of the C. defrosting
17. A group of welding operation in water vapor that was distilled off along D. NOTA
which no-ferrous filter metal melts at a 23: A component in absorption with ammonia.
temperature below that of the metal refrigeration which serves the same A. rectifier 34. Heat which occurs from cooling the
joined but is heated above 425 deg C. purpose as the suction stroke of a B. condenser product at its entering temperature down
A. drawing compressor. C. analyzer to the temperature held in a refrigerated
B. brazing A. generator D. eV'aporating coils space.
C. electroplating B. absorber A. product heat
D. extrusion C. aftercooler 29. The process of preferentially B. heat sources
D. evaporator condensing out the water vapor is C. heat absorbed
18. A process by which zinc coating is completed in the rectifier. The vapor D. heat loss
applied to a wide variety of stee l product 24. Most systems use ammonia as a leaving in the rectifier is essentially
to provide protection against ·corrosion. refrigerant and solutions of _ __ _ 35. Process of circulating warm air over
A. polishing as the absorbing medium. A. all ammonia the coils in order to melt off the ice
B. ironing A. water B. all water vapor which causes retardant for transferring
C. galvanizing B. ammonia-water C. all strong liquor heat to its refrigerant
D. forging C. ammonia D. all weak liquor A. absorptio.n
D. refrigerant-ammonia B. defrosting
19. The downgrading of water quality 30. Essentially two pressures ·exist in C. condensation
by sewage or other wastes to the point 25. Cool solution of water-ammonia that the absorption system, which of the D. cryogenics
where it unreasonably affects water use. is not saturated with ammonia gas is a following is under the head or
A. pollution condition called: condenser pressure: 36. In a compression-ignition engine
B. floatation A. strong liquor A. generator what is being compressed is:
C. waste B. medium liquor B. condenser A. fuel

REFRESHER MANUAL 2nd Edition by JAS TORDILLO REFRESHER MANUAL 2nd Edition by JAS TORDILLO
29-41 Day29-EXAM Day 29 - EXAM I 29 - 5
B. combustjble materials B. 61 Power and Industrial Plant Engineering
C. air C. 41
moles of carbon dioxideat 100°C and
D. fuel & air D. 51
225 kPa. The partial pressure of
oxygen is most nearly:
37. Expansion valve used to regulate 42. Calculate the compression ratio
A. 90 kPa
frj )
the refrigerant flow to the evaporator in necessary to produce a compression
B. 225 kPa DAY 29 - ANSWER KEY
order to: pressure of 150 psia with a compression
C. 75 kPa 1. 8 16.8 31.846.8
A. reduce refrigerant vapor curve index of 1.30 and an initial 2.A 17.8 32.047. 0
D. 60 kPa
B. reduce liquid refrigerant compression of 13.7 psia. 3. 8 18. c 33.848. c
C. increase liquid refrigerant A. 12.2 4.8 19.A 34. A49.8
48. A turbine pipe determined as 5:C 20. A 35. B 50. 8
D. increase refrigerant vapor B. 6.29
"nominal" size refers to: 6. 8 21. 0 36. C
C. 21.0
A. inside diameter 7. A 22. A 37. 8
38. What is the pressure above zero? D. 16.6 8. 8 23. 8 38. C
B. outside diameter
A. gage pressure 9. B 24. B 39. B
C. approximate size 10.A25.0 40. A
B. vacuum pressure 43. A substance is flowing in a pipe with
D. pipe wall thickness 11.A26.A 41 . 8
C. absolute pressure an internal diameter of 8 in at 600 fpm
12.C27. A 42.8
D. atmospheric pressure velocity average. At a given section of 13. B 28. C 43. A
49. A plant has installed a single-
the pipeline the pressure is 200 psia. 14. 029. A 44. A
suction centrifugal pump with a capacity
39. A steam condenser receives 10 kgs What is the flow work of 2 cu. Ft, 15. 030. C 45. 8
of 300 gpm under 200 ft at 1200 rpm. It
per second of steam with an enthalpy of express in Btu, passing this section? (1
is proposed to install another pump with
2570 kJ/kg . Steam condenses into a A. 74.1
double suction but of the same type to
liquid and leaves with an enthalpy of B. 84.1
operate at 100 ft head and delivers 400
160 kJ/kg. Cooling water passes C. 79.2
gpm. Determine the speed of the
through the condenser with temperature D. 89.9
proposed pump.
increases from ~ 3 degree C to 24
A. BOO rpm
degree C. Calculate the cooling water 44. Given CaH,a, FIND THE AIR-FUEL
B. 875 rpm
flow rate in kgs per second. RATIO. Given the products of
C. 850 rpm
A. 533.2 combustion (volumetric Analysis) C02 =
D. 910 rpm
B. 523.2 10.01%, CO= 0.85%, N2 = 85%, 02 =
c. 518.4 4.14%
50. Detonation or pinging noise is due
D. 528.9 A. '20.1
to:
B. 15.0
A. early timing of fuel injection
40. A hydro-electric plant discharges C. 18.0
B. late timing of fuel injection
water at the rate of 0.75 cubic meter per D. 12.0
second and enters the turbine at 0.35
c. head of piston carbonized
D. valve springs weak or broken
mps with a pressure of 275 kPa. 45. The thermodynamic cycle used in
Runner inside diameter is 550 mm, thermal power plant is:
speed is 520 rpm and the turbine A. Ericson
efficiency is 88%. Find the turbine B. Rankine
speed factor. C. Brayton
A. 0.638 D. Carnot
B. 0.368
C. 0.386 46. The taste of a pure substance is
D. 0.836 defined completely if we define:
A. Pressure and Temperature
41 . A pump lifts water at a rate of 283 B. Two independent intensive properties
Ips from a lake and force it into a tank 8 C. Two intensive properties
meters above the level of the water at a D. Any two properties
pressure of 137 kPa. What is the power
required in kilowatts? 47. A mixture ·contains 4 moles of
A. 71 oxygen, 6 moles of Nitrogen, and 5

REFRESHER MANUAL 2nd Edition by JAS TORDILLO REFRESHER MANUAL 2nd Edition by JAS TOROILLO
29-61 Day 29 - Solution Day 29 - Solution 129 - 7
Power and Industrial Plant Engineering
7. The method · of introducing the fuel
DAY· 29 - SOLUTION charge into the power cylinder of an C. 2001 deg C C. waste
engine. D. 5660 deg C D. filtration
1. The ratio of the brake HP to indicated A. mechanical injection *
HP of an engine: B. trunk injection 14. Fusion process in which the metal 20. Reduces the dissolved solids
A brake thermal eff C. suction is heated to a state of fusion permitting it content of water. Its main application is
B. mechanical eff * D. intake to flow together into a solid joint. converting brackish water to supply that
C. (ndicated eff A. flash welding is suitable for potable use. {from 1000
D. cycle eff 8. Preferable material for oil and fuel B. metal spray welding ppm to 10,000 ppm).
pipings: C. laser welding A. electrodialysis *
2. The ratio of the heat equivalent of 1 A steel and iron D. electric-arc welding * B. coagulation
hp-hr to the heat units actually supplied B. copper and brass * C. ion exchange
per bhp-hr, based on the higher heating C. plastic pipes 15. The process of filling the pores of a D. neutralization
value of the fuel. D. bronze and aluminum sintered product with molten metal in
A . brake thermal eff • order to decrease porosity or to improve 21. Where thus the percentage of
B. indicated thermal eff 9. Preferable materials for water piping physical properties. moisture and refrigerant vapor in the
C. mechanical eff system if internal combustion engines: A. embedding absorption last rejected?
D. cycle eff A stainless steel B. ironing A. analyzer
B. galvanized steel * C: piercing B. generator
3. The volume in cu ft per minute, c. cast iron D. infiltration * C. condenser
swept by the piston or pistons during the D. black iron D. rectifier*
power strokes. 16. The operation of cutting out flat
A. bore speed 10. Cooling water, used in internal area to some desired shape. 22. A component used in absorption
B. displacement * combustion , engines must be A. blowing refrigeration so that the vapor from the
C. events noncorrosive with a pH value of _ _. B. blanking * evaporator is coming into contact with
D. piston speed ~ 7* . C. bending the weak liquor.
B. 3 D. swaging A. absorber *
4. HP delivered by the shaft at the C. 10 B. heatexchanger
output end. D. 14 17. A group of welding operation in C. evaporator
A Indicated HP which no-ferrous filter metal melts at a D. intrcooler
B. Brake HP * 11: A mixture of aluminum powder and temperature below that of · the metal
C. Thermal power a metal oxide powder is ignited by a joined but is heated above 425 deg C. 23. A component in absorption
D. Air power special powder in crucible. A. drawing refrigeration which serves the same
A. thermit welding * B. b'razing * purpose as the suction stroke of a
5. Air at low pressure used to force B. resistance welding C. electroplating compressor.
burnt gases out of the power cylinder C. arc welding D. extrusion A. generator
during the exhaust period and. By the D. metal welding B. absorber *
displacement, to furnish a supply fresh
18. A process by which zinc coating is C. aftercooler
air for the following cycle. 12. The process in which gases are applied to a wide variety of steel product D. evaporator
A. turbo used in combination to obtain a hot to provide protection against corrosion.
B. exhaust flame. A. polishing 24. Most systems use ammonia as a
C. scavenging * A. hydrogen welding B. ironing refrigerant and solutions of _ _ __
D. discharg,e B. spray welding C. galvanizing * as the absorbing medium.
C. gas welding * D. forging A. water
6. Utilizes the working medium on only D. slug welding B. ammonia-water *
one side of a single piston.
19. The downgrading of water quality C. ammonia
A double-acting 13. The max imum temperature D. refrigerant-ammonia
by sewage or other wastes to the point
B. single-acting * developed for oxy-acetylene welding is where it unreasonably affects water use.
C. dual-acting
A. pollution * 25. Cool solution of water-ammonia that
D. single-stroke A. 1990 deg C B. floatation is not saturated with ammonia gas is a
B. 3440 deg C *
.
condition called:

REFRESHER MANUAL 2nd Edition by JAS TORDILLO REFRESHER MANUAL 2nd Edition by JAS TORDILLO
29 - 8 I Day 29 - Solution Day 29 -Solution I 29 - 9
A. strong liquor B. condenser
Power and Industrial Plant Engineering
B. medium liquor C. all Of these *
h =~ +
.B. combustible materials
C.
D.
heatexchanger
weak liquor*
D. rectifier
C. air* v2 = 275 + (0.35)2 =28.038 m
D. fuel & air y 2g 9.81 2(9.81)
31. Which of the following is under the
26. A condition where in the absorber low pressure? e _ n:DN
the water-ammonia takes up ammonia A generator • 37. Expansion valve used to regulate
- J2gh
gas until it becomes saturated and holds B. evaporator* the refrigerant flow to the evaporator in

n:(0.550)(~~.2)
as much gas as it can at the C. rectifier order to:
temperature and pressure existing in the D. NOTA A. reduce refrigerant vapor
B. reduce liquid refrigerant • 9= -;==:.;==~~6;,;,o,b
vessel or a liquid saturated with
ammonia is called: C. increase liquid refrigerant .}2(9.81) (28 .03)
32. Which of the following are the three
A. strong liquor* circuits of fluid in the absorption D. increase refrigerant vapor e = o.638
B. liquid receiver refrigeration system?
C. weak liquor 38. What is the pressure above zero? 41. A pump lifts water at a rate of 283
A. the ammonia circuit from the
D. slight liquor generator A. gage pressure Ips from a lake and force. it into a tank 8
B. the strong-liquor circuit from the B. vacuum pressure meters above the level of the water at a
27. A component in absorption absorber to the generator C. absolute pressure * pressure of 137 kPa. What is the power
refrigeration which raises the pressure C. the weak-liquor circuit from the
D. atmospheric pressure required in kilowatts?
of strong liquor and sends it through a generator back to the absorber A. 71
heat exchanger: 39. A steam condenser receives 10 kgs B. 61 *
D. all of these *
A. pump* per second of steam with an enthalpy of c. 41
B. absorber 2570 kJ/kg. Steam condenses into a D. 51
33. The energy which must be removed
C. generator liquid and leaves with an enthalpy of
from warmer air entering the refrigerated
D. expansion valve space. 160 kJ/kg. Cooling water passes 137
A. heat energy through the condenser with temperature H = - + 8 = 21.96 m
increases from 13 degree C to 24 9.81
28. Serves as a heat exchanger of B. infiltration *
degree C . Calculate the cooling water Power= QyH
ammonia vapor from the generator and
tend s to throw back a portion of the
C. defrosting
D. NOTA flow rate in kgs per second. =
0.283(9.81 )(21.96)
A. 533.2 Power = 60.96 kw
water vapor that was distilled off along
with ammonia. 34: Heat which occurs from cooling the
B. 523.2 *
C. 518.4 42. Calculate the compression ratio
A. rectifier product at its entering temperature down
D. 528.9 necessary to produce a compression
B. condenser to the temperature held in a refrigerated pressure of 150 psia with a compression
C. analyzer* space. curve index of 1.30 and an initial
D. evaporating coils A. product heat* m2(h2'- hs) = mwCPwl'.tw
10(2570 -160)) = mw(4.187)(24 -130) compression of 13.7 psia.
B. heat sources
29. The process of preferentially C. heat absorbed =
mw 523.26 kg/sec A. 12.2
B. 6.29 *
condensing out the water vapor is D. heat loss
completed in the rectifier. The vapor 40. A hydro-electric plant discharges c. 21.0
water at the rate of 0.75 cubic meter per D. 16.6
leaving in the rectifier is essentially 35. Process of circulating warm air over
the coils in order to melt off the ice second and enters the turbine at 0.35
mps with a pressure of 275 kPa. PV" = C
A. all ammonia * which causes retardant for transferring
Runner inside diameter is 550 mm, P1Vt = P2V2"
B. all water vapor heat to its refrigerant.
speed is 520 rpm and the turbine
C. all strong liquor A absorption
efficiency is 88%. Find the turbine
.Y!. = ( p2 )Yn """
D. all weak liquor B. defrosting * V2 P1

30. Essentially two pressures exist in


the absorption system, which of the
C. condensation
D. cryogenics
speed factor.
A. 0.638 *
B. 0.368
. compression ratio, I)( = .YJ.. = (P2
v2 F1
t
following is under the head or 36. In a compression-ignition engine C. 0.386 l

condenser pressure: what is being compressed is: D. 0.836 I)( = (~)1.3


13.7
= 6.29
A. generator A. fuel

REFRESHER MANUAL 2nd Edition by JAS TORDILLO REFRESHER MANUAL 2nd Edition by JAS TORDILLO
Day 30 - EXAM I 30 - 1
29 -10 1 Day 29- Solution Power and Industrial Plant Engineering
43. A substance is flowing in a pipe with A. Pressure and Temperature location or erected at a location without
an internal diameter of 8 in at 600 fpm B. Two independent intensive DAY 30 - · EXAM change of ownership.
velocity average. At a given section of properties * A. PSME boiler
the pipeline the pressure is 200 psia'.. C. Two intensive properties 1. Where ground water or surface water B. reinstalled boiler
What is the flow work of 2 cu. Ft. - D. Any two properties are used for feedwater, water hardness C. fusion boiler
express in Btu . passing this section? of 0 -10 ppm (parts per million) and D. unknown boiler
A. 74.1 * 47. A mixture contains 4 moles of alkalinity of _ _ be considered.
B. 84.1 oxygen, 6 moles of Nitrogen, and 5 A. pH 10 to pH 11 7. As a general requirements,
c. 79.2 moles of carbon dioxide at 100°C and B. pH 9to pH 10 smokestacks should be of sufficient
D. 89.9 225 kPa. The partial pressure of C. pH 6 to pH 10 capacity to handle flue gases, self
oxygen is most nearly: D. pH 8 to pH10 supporting or guyed to withstand a wind
Work= PV A. 90 kPa load 160 kph and rise at least
B. 225 kPa
Work= 200(144)(2) ft-lb x ~ C. 75 kPa
2. A closed vessel in which steam or
other vapor is generated at a pressure
_ __ mm above the eaves of any
building within a radius of 50 meters.
778 ft -lb D. 60 kPa * of more than 1.055 kg/cm 2 gage by the A. 5000 .
Work= 74.1 Btu direct application of heat.
P02 = ( n
02 4
)P = ( -- )(225) = 60 kPa
B. 3000
nr 4+6+5 A. power boiler C. 4000
44. Given CaH1s, FIND THE AIR-FUEL B. boiler D. 10,000
RATIO. Given .the products of C. fusion boiler
combustion (volumetric Analysis) C02 = 48. A turbine pipe determined as
"nominal" size refers to: D. external boiler 8. As . a general requirements, no
10.01%, CO= 0.85%, N2 = 85%, Oz =
A. inside diameter • smokestacks should be closer than
4.14% 3. The heating surface area of _ _ _mm from any exposed
B. outside diameter
A. 20.1 *
C. approximate size * miniature boiler is 1.85 m2 and the woodworking or framing .
B. 15.0 maximum allowable working pressure is
D. pipe wall thickness A. 305
C. 18.0 ___ kg/cm 2• B. 503 .
D 12.0
49. A plant has installed a single- A. 7.03 c. 450
suction centrifugal pump with a capacity B. 9.85 D. 1200
From actual reaction. 100 mols of dry C. 8.05
gas: of 300 gpm under 200ft at 1200 rpm . It
is ~reposed to install another pump with D. 14.7 9. Each miniature boiler, shall be
ACsH1s + B(Oz +3.76 N2) = 10.01 C02
double S}Jction but of the same type to equipped with sealed, spring-loaded pop
+ 0.85 CO + 85 N2 + 4.14 02 +CH20 4. Low pressure heating boiler is a
operate at 100 ft head and delivers 400 type safety valve not less than
gpm. Determine the speed of the boi ler operated at pressure not _ _ _ pipe size, connected directly to
Nitrogen Balance: 3.76(2)B = 85 (2) exceeding 1.055 kg/cm 2 gage steam or
proposed pump. the-boiler.
B = 22.6 water temperature not exceeding
A. 800 rpm A. 10.2 mm
Carbon balance: A (8) = 10.01 +.85 _ _ _ degC.
B. 875 rpm" B. 15.6 mm
A = 1.3575 A. 100
C. 850 rpm C. 12.7 mm
Actual A /F = 22 .6(1 + 3.76)28.97 B. 121 D. 24.5 mtn
D. 910 rpm
1.3575 [8(12) + 18(1) ) C. 11 1
N,JQ; Nda; D. 145 10. Each miniature boiler shall be
= 20 . 1 kg a1r equipped with _ _ for
kg fuel HJT4=f:P4
I l 5. A boiler of which both the location determination of water level.
400 and ownership have been changed after
but 0 2 = = 200 gpm A. pressure gage
45 The thermodynamic cycle used in 2
primary use. B. safety valve
thermal power plant is: I2oo.J3oii _ N2 J20o A. internal boi ler C. water gage glass
A. Ericson 200 314 - 100 214
B. second hand boiler D. fusible plug
B. Rankine* Nz = 847 rpm C. surplus boiler
C. Brayton D. external boi ler 11. Each miniature boiler shall be
D. Carnot 50. Detonation or pinging no1se is due to:
equipped with a steam gage having a
A. early timing of fuel injection 6. A boiler removed from its original dial range not less than times
46. The taste of a pure substance is B. late timing of fuel injection * setting and re erected at the same
defined completely if we define: C. head of piston carbonized
D. valve springs weak or broken REFRESHER MANUAL 2nd Edition by JAS TORDILLO
REFRESHER MANUAL 2nd Edition by JAS 'TORDILLO
r

Day 30 - EXAM I 30 - 1
29 -10 I Day 29 - Solution Power and Industrial Plant Engineering
43. A substance is flowing in a pipe with A. Pressure and Temperature location or erected at a location without
an tnternal diameter of 8 in at 600 fpm B. Two independent intensive DAY 30- EXAM change of ownership.
velocity average At a g1ven section of properties • A. PSME boiler
the pipeline the pressure is 200 psia . C. Two intensive properties 1. Where ground water or surface water B. reinstalled boiler
What is the flow work of 2 cu Ft, - D. Any two properties are used for feedwater, water hardness C. fusion boiler
express 1n Btu, passtng this section? of 0 -10 ppm (parts per million) and D. unknown boiler
A. 74.1 * 47. A mixture contains 4 moles of alkalinity of__ be considered.
B 84.1 oxygen, 6 moles of Nitrogen, and 5 A. pH 10 to pH 11 7. As a general requirements,
C. 79.2 moles of carbon dioxide at 100°C and B. pH 9 to pH 10 smokestacks should be of sufficient
D 89.9 225 kPa. The partial pressure of C. pH6topH 10 capacity to handle flue gases, self
oxygen is most nearly: D. pH 8 to pH10 supporting or guyed to withstand a wind
Work= PV A. 90 kPa load 160 kph and rise at least
B. 225 kPa
Work= 200(144)(2) ft-lb x ~ C. 75 kPa
2. A closed vessel in which steam or _ _ _ mm above the eaves of any
other vapor is generated at a pressure building within a radius of 50 meters.
778' ft -lb D. 60 kPa * of more than 1.055 kg/cm 2 gage by the A. 5000
Work= 74.1 Btu
Po2 ; (~)P "' ( - - )(225) = 60 kPa
4 direct application of heat. B. 3000
nr 4+6+5 A. power boiler C. 4000
44 Given CeH1e, FIND THE AIR-FUEL
B. boiler D. 10,000
RATIO. Given .the products of
48. A turbtne pipe determined as C. fusion boiler
combustion (volumetric Analysis) C02 = D. external boiler
"nominal" size refers to: 8. As . a general requirements, no
10.01%, CO = 0.85%, N2 = 85%, Oz =
A. inside diameter smokestacks should be closer than
414%
B outside diameter 3. The heating surface a~ea of ___mm from any exposed
A. 20.1 •
C. approximate size * miniature boiler is 1.85 m2 and the woodworking or framing.
B. 15.0
D. pipe wall thickness maximum allowable working pressure is A. 305
C. 18.0 ___ kg/cm 2. B. 503
D 12 0
49. A plant has installed a single- A 7.03 c. 450
suct10n centnfugal pump with a capacity B. 9.85 D. 1200
From actual react1on. 100 mols of dry
gas of 300 gpm under 200 ft at 1200 rpm. It c. 8.05
D. 14.7 9. Each miniature boiler, shall be
ACeH1e + B(02 +3.76 N2) = 10.01 C02 is proposed to install another pump with
double S}Jction but of the same type to equipped with sealed, spring-loaded pop
+ 0.85 CO + 85 N2 + 4.14 Oz +CHzO
operate at 100 ft head and delivers 400 4. Low pressure heating boiler is a type safety valve not less than
gpm. Determine the speed of the boiler operated at pressure not ___pipe size, connected directly to
N1trogen Balance 3. 76(2)8 = 85 (2)
proposed pump. exceeding 1.055 kg/cm2 gage steam or the-boiler.
B = 22.6
A. 800 rpm ·water temperature not exceeding A. 10.2 mm
Carbon balance: A (8) = 10.01 +.85 _ _ _ degC.
B. 875 rpm* B. 15.6 mm
A = 1 3575
C. 850 rpm A. 100 C. 12.7 mm
Actual A /F = 22.6(1 + 3.76)28.97 B. 121 D. 24.5 mm
D. 910 rpm
1.3575[8(12)+18(1) l C. 111
N1.JQ; Ndo:; D. 145 10. Each miniature boiler shall be
= 20.1 kg a1r
kg fuel HJ/4
I
•HV4
2
equipped with for
5. A boiler of which both the location determination of water level.
400
but 0 2 ; = 200 gpm and ownership have been changed after A. pressure gage
45. The thermodynamic cycle used in 2
primary use. B. safety valve
thermal power plant is: 1200JJ00 N2J200 A. internal boiler C. water gage glass
A Ericson 200314 ~ 100214
B. second hand boiler D. fusible plug
B. Rankine* N2 = 847 rpm C. surplus boiler
C. Brayton D. external boiler 11 . Each miniature boiler shall be
D Carnot 50. Detonation or p1nging no1se is due to equipped with a steam gage having a
A. early timing of fuel injection 6. A boiler removed from its original dial range not less than times
46 The taste of a pure substance is B. late timing of fuel injection * setting and re erected at the same
defined completely if we define: C head of piston carbonized
D valve springs weak or broken REFRESHER MANUAL 2nd Edition by JAS TORDILLO
REFRESHER MANUAL 2nd Edition by JAS TORDILLO
30 - 2 I Day 30 · EXAM
Day 30 · EXAM I 30 - 3
and not mo·re than twice the maximum 17. A device having a predetermined Power and Industrial Plant Engineering
allowable working pressure. temperature fusible member for the
A. one relief of pressure. 23. Refrigerant piping crossing an open C. dichloromethane
B. 3 A. gage glass space which affords passageway in any D. methyl chloride
C. 1.5 B. fusible plug building shall not be less than -:--:--::--
D. 4 C. pressure gage above the floor unless against the 29. The chemical symbol of ammonia:
D. stop valve ceiling of s uch space. A. so
12. The maximum allowable working A. 2290 mm B. NH3
pressure of a non-code steel or wrought 18. A shot-off valve other than a valve B. 2920 mm C. H20
iron heating boiler of welded for controlling the flow of refrigerant. C. 2570 mm D. C4Ho
construction shall not exceed _ _ __ A. stop valve D. 34!;)0 mm
A. 2 Mpa B. check valve 30. The chemical symbol of Freon 12:
B. 1 atm C. pressure valve 24. When refrigerant is added to a A. CHCI2F
C. 2 gage pressure D. relief valve system, it shall be cha rged into t he B. CCI2F2
D. 1 bar ____ pressure side of the system. C. C3Hs
19. A refrigeration system in which the A. high D. C02
13. A boiler device which automatically refrigerant gas evolved in the evaporator B. low
cut off the fuel supply and/or supply is taken up In an absorber and released C. low and high 31 . Suitable means shall be provided
requisite feed-water when the sutiace of in a generator upon the application of D. low or high for the collection and disposal of
the wall falls to the lowest safe water the heat. condensate from the equipment. The
line. A. absorption 25. A refrigeration system having two or condensate drain shall be at least
A. low water fuel cut off B. flooded more refrigerant circuits, ·each w ith a _ _ _ nominal pipe size and shall be
B. blow off C. bri"e spray pressure-imposing element, condenser copper , galvanized steel, or other
C. safety valve D. cascade and evaporator, where the evaporator of corrosion-resistant metal.
D. gage glass one circuit cools the condenser of A. 19 mm
20. A refrigeration system in which only another circuit. B. 25 mm
14. During hydrostatic test, hydrostatic part of the refrigerant passing over the A. brine C. 23mm
test pressure should not exceed heat transfer separated from the vapor B. cascade D. 90 mm
--..,.-- times the maximum allowable and recirculated. C. mechanical
working pressure. A.. flooded D. none of these 32. An indirect refrigerating system
A. one B. steam jet · employing water as the circulating
B. two C. mechanical 26. A substance which produces a system.
C. one and a half D. vapor refrigerating effect by its absorption of A . chilled water
D. three heat w hile expanding or ev aporating. B. bri ne spray
21. The temperature . of the air to be A . refrigerant C. vapor
15. A boiler test method based on the used for comfort cooling shalt be B. water D. heating system
amount of time it takes generated sound mainta ined at degree C. C. air
waves to pass through a material and A. 10-15 . D. brine system 33. This is the ratio of the actual weight
back to the source after being reflected. B. 20-24 of moisture to the weight , of saturated
A. radiographic c. 15 - 20 27. Any liquid cooled by t he refrigerant water vapor of mixture.
B. ultrasonic thickness gauging D. 20 - 30 and used for the transmission of heat A . humidity
C. sound test without a change in its state. B. density
D. timing test 22. The humidity of the air to be used A. mercury C. specific weight
for comfort cooling shall be maintained B. water spray D. specific gravity
16. A vessel permanently connected to at relative humidity. C. brine '
a system by inlet and outlet pipes for A. 50 to 60% D. water cooler 34. A vessel in which vaporized
stor11ge of liquid refrigerant. B. 60 to 65% refrigerant is liquefied by the removal of
A liquid receiver · C. 40 to 55% 28. :rhe mechanical name of refrigerant heat.
B. accumulator D. 70 to 75% 12: A. condenser
C. container A. carbon dioxide B. evaporator
D. duct B. dichlorodifluoromethane C. compressor
D. brine tank
REFRESHER MANUAL 2nd Edition by JAS TORDILLO
REFRESHER MANUAL 2nd Edition by JAS TORDILLO
· --- ~-- ~-- --- - - - · - --
- - -- - ----

30 ~ 4 I Day 30 ~ EXAM
Day 30 -Solution 1 30 - 5
35. Which of the following refrigerants C. fire hose Power and Industrial Plant Engineering
belongs to Group 2? D. fire storage
A. propane C. static head
B. carbon dioxide D . total head DAY 30 -SOLUTION
42. Fire code of the Philippines:
C. R-114 A. PO No. 185
D. ammonia 49. us'ed to measure the pressure of 1. Where ground water or surface water
B. PO No. 1185
water discharging from a nozzle. are used for feedwater, water hardness
C. PD No. 2086 of 0 -10 ppm (parts per million) and
36. For fire protection purposes, is an A. manometer
D. PO No. 8495 alkalinity of be considered .
integrated system of one or more water B. Pitot tube
supplies for fire use, underground and C. Piezometer A. pH 10 to pH 11 *
43. The term designates commodities, B. pH 9 to pH 10
overhead piping design. D. flow meter
packaging or storage aids which Will not C. pH6topH10
A. sprinkler system ignite, burn or liberate flammable gases
B. container system 50. The level with respect to the pump D. pH 8 to pH10
when heated to a temperature of 749
C. storage system of the body of water from which it takes
degree for five minutes. 2. A closed vessel in which steam or
D. fire protection system suction when the pump is not in
A. non-combustibles
operation. other vapor is generated at a pressure
B. combustibles of more than 1.055 kg/cm 2 gage by the
37. Class of fire involving ordinary A. static water level
C. medium direct application of heat.
combustible materials such as wood, D. high B. pumping water level
C . draw down A. power boiler *
cloth, paper, rubber and plastics.
A. ClassA D. dynamic level B. boiler
44. Materials either of which will c. fusion boiler
B. Class D contribute fuel to fire.
C Class B D. external boiler
A. moderate
D. Class E B. extra
C. combustibles
(d ) 3. The heating surface area of
38. Class of fire in flammable liquids miniature boiler is 1.85 m2 and the
D. high DAY 30 - ANSWER KEY
and gases. 1. A 16. A 31 . A46.A maximum allowable working pressure is
2 A 17 B 32. A 47. B ___ kg/cm 2
A. Class A 45. Type of extinguishers that is used 1
B. Class C 3. A 18.A 33. A48. A A. 7.03 *
by the occupants of a building primarily 4. B 19. A 34. A >19. B B. 9.85
C. Class B for immediate use of small fires.
D. Class E
5. B 20. A
6. B 21 . B
35. 0 50. A
36. A
c. 8.05
A. portable D. 14.7
7. A 22. A 37. A
B. outdoor
8. A 23. A 38. C
39. Class of fire involving combustible C. indoor 9. C 24. B 39. 0 4. Low pressure heating boiler is a
metals, such as magnesium, sodium, D. miniature 10. C25. B 40. 8 boiler operated at pressure not
titanium and other similar metals. 11.C25. A 41 . A
exceeding 1.055 kg/cm 2 gage steam or
A. ClassA 46. Science of water at rest. '12. 027. C 42. B
13. A 28. B 43. A water temperature not exceeding
B. Class C A. hydrostatics ____ degC.
14. C 29. B 44. A
C. Class B B. hydrokinetics 15. B 30. B 45. A A. 100
D. Class D C. hydraulic
(I B. 121 ~
D. hydrodynamic C. 111
40. Class of fire involving energized
D. 145
electrical equipment. 47. Science of the force exerted by
A. Class A water in motion. 5. A boiler of which both the location
B. Class C A. hydrostatics and ownership have been changed after
C. Class B B. hydrodynamics primary use.
D. Class D C. hydrokinetics A. internal boiler
D. hydraulic B. second hand boiler *
41. An organization in-charge with the
C. surplus boiler
mission of fire prevention, fire 48. It is the reading of a liquid D. external boiler
protection. manometer at the suction of the pump.
A. fire service A. suction lift 6. A boiler removed from its original
B. fire building B. suction head setting and re erected at the same

REFRESHER MANUAL 2nd Edition by JAS TORDILLO REFRESHER MANUAL 2nd Edition by JAS TORDILLO
30 - 6 I Day 30 • Solution Day 30 - Solution 1 30 - 7
Power and Industrial Plant Engineering
. location or erected at a location without A one
change of ownership. B. 3 17 A device having a predetermined 23. Refrigerant piping crossing an open
A. PSME boiler c. 1.5 * temperature fusible member for the space which affords passageway in any
B. reinstalled boiler * D. 4 relief of pressure. building shall not be less than _ _ __
C. fusion boiler A. gage glass above the floor unless against the
D. unknown boiler 12. The maximum allowable working ceiling of such space.
B. fusible plug *
pressure of a non-code steel or wrought C pressure gage A. 2290 mm *
7 . As a general requirements, iron heating boiler of welded 0 stop valve B. 2920 mm
smokestacks should be of sufficient construction shall not exceed _ _ __ C. 2570 mm
capacity to handle flue gases, self A 2 Mpa 18. A shot-off valve other than a valve D. 3450 mm
supporting or guyed to withstand a wind B. 1 atm for controlling the flow of refrigerant.
load 160 kph and rise at least C. 2 gage pressure A. stop valve * 24. When refrigerant is added to a
___mm above the eaves of any D. 1 bar* system, it shall be charged into the
B. check valve
building within a radius of 50 meters. C. pressure valve _ _ _ pressure side of the system.
A. 5000 * 13. A boiler device which automatically D relief valve A. high
B. 3000 cut off the fuel supply and/or supply B. low*
C. 4000 requisite feed-water w~en the surface of 19. A refrigeration system in which the C. low and high
D. 10,000 the wall falls to the lowest safe water refrigerant gas evolved in the evaporator D. low or high
line. 1s taken up in an absorber and released
8. As a general requirements, no A. low water fuel cut off* in a generator upon the application of 25. A refrigeration system having two or
smokestacks should be closer than B. blow off more refrigerant circuits, each with a
the heat.
___mm from any exposed C. safety valve pressure-imposing element, condenser
A. absorption *
woodworking or framing. D. gage glass and evaporator, where the evaporator of
B. flooded
A. 305 * C. brine spray one circuit cools the condenser of
B. 503 14. During hydrostatic test, hydrostatic another circuit.
D. cascade
C. 450 test pressure should not exceed A. brine
D. 1200 ____ times the maxim om allowable 20. A refrigeration system in which only B. cascade *
working pressure. part of the refrigerant passing over the C. mechanical
9. Each miniature boiler, shall be A. one heat transfer separated from the vapor D. none of these
equipped with sealed, spring-loaded pop B. two and recirculated.
type safety valve not less than C. one and a half* A. flooded • 26. A substance which produces a
___pipe size, connected directly to D. three refrigerating effect by its absorption of
B. steam jet
the boiler. heat while expanding or evaporating.
C. mechanical
A 10.2 mm 15. A boiler test method based on the D. vapor A. refrigerant*
B. 15.6 mm amount of time it takes generated sound B. water
c. 12.7 mm * waves to pass through a material and 21. The temperature of the air to be C. air
D. 24.5 mm back to the source after being reflected. D. brine system
used for comfort cooling shall be
A. radiographic maintained at degree C.
10. Each miniature boiler shall be B. ultrasonic thickness gauging * A. 10-15 27. Any liquid cooled by the refrigerant
equipped with __ for C. sound test and used for the transmission of heat
B. 20-24 *
determination of water level. D. timing test c. 15-20 without a change in its state.
A pressure gage D. 20-30 A. mercury
B. safety valve 16. A vessel permanently connected to B. water spray
C. water gage glass* a system by inlet and outlet pipes for 22. The humidity of the air to be used C. brine *
D. fusible plug storage of liquid refrigerant. for comfort cooling shall be maintained D. water cooler
A. liquid receiver * at relative humidity.
11 . Each miniature boiler shall be B. accumulator A. 50 to 60% * 28. The mechanical name of refrigerant
equipped with a steam gage having a C. container B. 60 to 65% 12:
dial range not less than times D. duct C. 40 to 55% A carbon dioxide
and not more than twice the maximum B. dichlorodifluoromethane *
D. 70 to 75%
allowable working pressure. ·

REFRESHER MANUAL 2nd Edition by JAS TORDILLO REFRESHER MANUAL 2nd Editi'o n by JAS TORDILLO
30 • 8 I Day 30 • Solution Day 30 -Solution 130 - 9
Power and Industrial Plant Engineering
C. dichloromethane 35. Which of the following refrigerants
D. methyl chloride belongs to Group 2? C. fire hose B. suction head
A. propane D. fire storage C . static head
29. The chemical symbol of ammonia. B. carbon dioxide D. total head
A. SO C. R-114 42. Fire code of the Philippines:
B. NHJ * D. ammonia* A. PD No. 185 49. Used to measure the pressure of
C. H20 B. PO No. 1185 * water discharging from a nozzle.
D. C~Ho 36. For fire protection purposes, is an C. PD No. 2086 A. manometer
integrated system of one or more water D. PD No. 8495 B. Pitot tube *
30. The chemical symbol of Freon 12: supplies for fire use, underground and C. Piezometer
A. CHCI2F overhead piping design. 43. The term designates commodities, D. flow meter
B. CCizF2 * A. sprinkler system * packaging or storage aids which will not
C. C3He B. container system ignite, burn or lib~rate flammable gases 50. The level with respect to the pump
D. C02 C. storage system when heated to a temperature of 749 of the body of water from which it takes
D. fire protection system degree for five minutes. suction when the pump is not in
31 . Suitable means shall be provided A. non-combusti bles * operation.
for the collection and disposal of 37. Class of fire involving ordinary B. comb ustibles A. static water level *
condensate from the equipment. The combustible materials such as wood , C. medium 8 . pum ping water level
condensate drain shall be at least cloth, paper, rubber and plastics. D. high C. draw down
___ nominal pipe size and shall be A. Class A* D. dynamic level
copper, galvanized steel, or other B. Class D 44. Materials either of which will
corrosion-resistant metal. C. Class B contribute fuel to fire.
A. 19 mm * D. Class E · A. moderate *
B. 25 mm B. extra
C. 23 mm 38. Class of fire in flammable liquids C. combustibles
D. 90mm and gases. D. high
A. Class A
32. An indirect refrigerating system B. Class C 45. Type of extinguishers that is used
employing water as the circulating C. Class B * by the occupants of a building primarily
system. D.' Class E for immediate use of small fires.
A. chilled water * A. portable *
B. brine spray 39. Class of fire involving combustible B. outdoor
C. vapor metals, such as magnesium, sodi um, C. indoor
D heating system titanium and other similar metals. D. miniature
A. ClassA
33. This Is the ratio of the actual weight B. Class C 46. Science of water at rest.
of moisture to the weight of saturated C. Class 8 A. hydrostatics *
water vapor of mixture. D. Class D * 8 . hydroki netics
A. humidity * C. hydraulic
B. density 40. Class of fire involving energized D. hydrodynamic
C. specific weight electrical equipment.
D. specific gravity A. ClassA 4 7. Science of the force exerted by
B. Class C * water in motion.
34. A vessel in which vaporized C. Class B A. hydrostatics
refrigerant is liquefied by the removal of D. Class D B. hydrodynamics *
heat. C. hydrokinetics
A. condenser * 41 . An organization in-charge with the D. hydraulic
B. evaporator mission of fire prevention, fire
C. compressor protection. 48. It is the reading of a liquid
D. brine tank A. fire service * manometer at the suction of the pump.
B. fire building A. suction lift *

REFRESHER MANUAL 2nd Edition by JAS TORDILLO REFRESHER MANUAL 2nd Edition by JAS TORDILLO
Day 31 -EXAM 131-1
Power and Industrial Plant Engineering
C. sulfur dioxide
DAY 31 - EXAM D. carbon monoxide

1. One hundred twenty five percent of :.. 7. Which of the following is a type of
theoretical air was supplied for condenser?
combustion. What is the equivalent A. shell-and-tube evaporator
excess air equal to? B. evaporative
A. 25% C. 3-phase motors
B. 100% D. cooling tower
C. 50%
D. 125% 8. If the pressure exerted on a liquid is
higher than the saturation pressure
2. The relation between the Fahrenheit corresponding to its temperature, the
absolute scale and the Celcius absolute liquid is at the region.·
scale is: A. saturated liquid
A. deg R = 1.8 deg K B. superheated liquid
B. deg R = 32 deg K C. subcooled liquid
C. deg R = 3.2 deg K D. highly superheated liquid
D. degR=18degK
9. The mass and temperature of a gas
3. The sum of the internal energy and are inversely proportional at a constant
the product of pressure and specific pressure process. This is known as:
volume is known as the: A. Dalton's law
A. enthalpy B. law of buoyancy
B. entropy C. Charles law
C. flowwork D. Newton's second law
D. total work
10. In a certain hydro power plant. the
4. Which of the following power plant total head is h and the head loss due to
uses energy from uranium to produce friction is hf, then what is the available
electric power? head of the process?
A. diesel plant A. hf- h
B. geothermal plant B. h
C. nuclear plant c. h - hf
D. hydroelectric plant D. hf

5. The force, when applied to a mass of 11 . If air is compressed without heat,


one kilogram at an acceleration of one then what do you call this kind of
meter per second per second is: compression?
A. watt A. isobaric
B. Joule B. isochoric
C. Newton C. adiabatic
D. Pascal D. isothermal

6. Dry analysis is an analysis in the 12. If a gas possesses internal energy,


products of combustion which will not then it is due to its:
include _ __ A. velocity
A. water vapor B. pressure and volume values
B. carbon dioxide C. height from a certain datum
D. molecular motion·

REFRESHER MANUAL 2nd Edition by JAS TOROILLO


I
31 - 2 I Day 31 - EXAM Day 31 -EXAM 131- 3
13. Where is the position of the piston Power and Industrial Plant Engineering
A 0.08
when combustion starts in a spark- B. 26.2
ignition engine? 24. Fi.ve moles of water vapor at 100 C. Ef
C. 12.09 D. Ef/Eg
A. Y2 from the top deg C and 1 atmosphere pressure are
D. 3547
B. piston does not move compressed isobarically to form a liquid
at 100 deg D. The process is reversible 29. Identify the cycle: isentropic
C. away from the top center 19. In an isentropic compression
D. at the top center and the ideal gas laws apply. Compute compression - isobaric heat addition -
process, air is compressed from an adiabatic expansion - isometric heat
the initial volume of vapor if water vapor
initial pressure of 100 psi to 200 psi. addition.
14. In a Brayton cycle, the result of constant is 0.0821 Liter-atm per mole
What is the final volume, in cubic A. Otto
adding a regenerator is: per degree K.
inches, if the initial volume is 10 in3 . B. Diesel
A. increase in thermal efficiency A. 123 li
A 3.509 C. Garnet
B. moderate thermal efficiency B. 5.000 B. 1431i
C. 1331i D. Rankine
C. decrease in thermal efficiency C. 4.500
D. low thermal efficiency D. 6.095 D. 1531i
30. A steam generator produces 1007
25. In a reversible process, like the Btu per Ibm of heat at 100 psia from
15. During an adiabatic, internally 20. What is true about the polytropic
compression process in Otto or Diesel 14.7 psia of saturated liquid. If the heat
reversible process, what is true about exponent, n, for a perfect gas
cycle plotted on the p-V diagram. What input supplied is 800 Btu per Ibm, which
the change in entropy? undergoing an isobaric process?
does the area on the diagram of the following is true for the device?
A. It is always zero A equal to unity
represent? A. The device cannot work because the
B. It is always greater than zero B. less than unity
A. free energy change pressure is greater than zero.
C. It is always less than zero C. greater than unity
B. work done by the system B. The device can work because output
D. It is temperature-dependent · D. equal to zero
C. heat transfer is grater than input.
D. entropy change C. The device violates the first law ·of
16. A cylinder and piston arrangement 2 1. What is an isentropic process? A thermodynamics because output is
contains saturated water vapor at 110 process that is
26. Calculate the entropy, in BTU per greater than input.
deg C(s = 7.2387 kJ/kg -deg K, u = A. irreversible D. The device violates the second law
2518.1kJ/kg). The vapor is compressed Ibm per degree R, of steam at 60 psia
B. reversible . thermodynamics because input is less
in a reversible adiabatic process until with a 60% amount of vapor.
C. heat transfer greater than zero than output.
the pressure ts 1.6 Mpa (s = A. 0.4274
D. heat transfer not equal to zero
7.2374kJ/kg-deg K, u = 2950.1kJ!kg). B. 1.1577
3 c. 0.9014 31 .. In a steam Rankine cycle, saturated
Determine the work done by the system 22. A 0.1 0 m gas is heated in a closed
D. 1.2172 liquid at i4.7 psia (vf = 0.01672 cu. ft.
per kg of water. cylinder. The initial temperature of the per Ibm) is pump to a high pressure
A. -637 kJ/kg gas is 350 deg K, until after heating the
27. A vessel with a volume of 1 cu. liquid at 100 psia. How much work is
B. -4<J2kJ/kg temperature of the gas is 600 deg K.
meter contains liquid water and water required to pump one Ibm of water?
C. -509 kJ/kg What is the work done in the system per
vapor in equilibrium at 60 kPa. The A. 118· Btu
D.. -330 kJ/kg kg mass of gas? Cv of air is 0. 716.
liquid water has a mass of 1 kg. B. 0.264 Btu
A. 0 kJ/kg
Calculate the mass of the water vapor. C. 0.129 Btu
17. Air is compressed adiabatically from B. 275 kJ/kg
At 600 kPa from steamtable: D. 18.5 Btu
an Initial pressure of 200 psi and a c. 225 kJ/kg 3
temperature of 700 deg R to 300 psi. D. 350 kJ/kg Vf = 0.0011101 m /kg
3
Vg = 0.3157 m /kg 32. A pressure gauge attached to a
Find the temperature after the
A. 0.99 kg pump reads 85 kPa . If the atmospheric
compression . 23. Gas is enclosed in a cylinder with a
B. 1.89 kg pressure is 29.92 in Hg, determine the
A. 576 deg C weighted piston as the top boundary. absolute pressure of the water pumped.
B. 680 deg C The gas is heated and expands from a C. 1.57kg
D. 3.16 kg A. 19m of water
C. 590 deg C volume of 0.04 to 0.10 m3 at a constant B. 25 m of water
D. 786 deg C pressure of 200 kPa. Calculate the
28. If Eg is the enthalpy of the saturated C. 17m of water
work done by the system. D. 14m of water
18. Nitrogen is expanded isentropically. A 8 kJ vapor and Ef is the enthalpy of the
Its temperature changes from 620 deg F B. 12 kJ saturated liquid. What is the expression
for the heat of vaporization? 33. Water enters a turbine pump at 30
to 60 deg F. Find the pressure ratio, c. 10 kJ kPa (vf = 0.0010.223 cu. m per kg) and
initial over final pr.essure. D. 14 kJ A. Eg
B. Eg- Ef discharges at 2.8 Mpa. The turbine
'
REFRESHER MANUAL 2nd Edition by JAS TORDILLO REFRESHER MANUAL 2nd Edition by JAS TORDILLO
31 - 4 I Day 31 - EXAM Day 31 -EXAM 131-5
work is 380 kJ/kg. What is the Power and Industrial Plant Engineering
200 kJ/kg. Determine the thermal
percentage of turbine work output is efficiency of the cycle.
C. Re < Rc B. 4000 MW
required to operate the pump of a steam A. 33.33%
power plant? D. cannot be determined C. 3500 MW
B. 55.55%
A. 1% D. 5000 MW
C. 44.44%
8 0.75% 44. If Rc is the compression ratio of a
D. 66.66% 50. What is the volume of a rigid tank
C. 0.9% compression-ignition cycle engine and
D. 0.62% Re is the expansion ratio. Which of the that contains 15 kg of water at a
39. A turbine has an efficiency of 75% temperature of 90 deg C (P = 84.55
following is true regarding the relation of
and has a steam flow rate of 22,000 kPa, Vf = 1.041/kg, Vg = 1982Ukg), if 10
34. Heat is transferred to a heat engine the Rc Re?
kg/hr. If the available energy is 1200 kg of this water is liquid and the
at a rate of 90 kw. The rate of rejection A. Rc = Re
kJ/kg, determine the kilowatt output at remaining is in vapor form .
of waste heat to a sink is 50 kw. B. Rc > Re
full load. A. 12.892 cubic meters
Compute the thermal efficiency of the C. Rc< Re
A. 3500 B. 9.9204 cubic meters
heat engine. · D. no relation
B. 4500 C. 0.0156 cubic meters
A. 44.4% C. 4000
45. A valve that releases steam in case D. 5.6549 cubic meters
B. 66.6% D. 5500
C. 55.5% of excessive temperature ca used by
D. 77.7% low-water level.
40. The ratio of the average load to that
A. safety valve
of the rated capacity of a plant is called:
35. What is the overall efficiency of the A. output factor B. stop valve DAY 31- ANSWER KEY
turbine if the power developed is 12 C. check valve 1. A . 16. 8 31 . 846.C
B. load factor
MW at a head of 150 meters and the D. fusible plug 2. A 17. 0 32. A47. D
C. demand factor 3. A 18. C 33. 8 48. D
nozzle discharges 10 cubic meters each D. capacity factor 4. C 19. 0 34. A49.C
second? 46. A feedwater preheating device 5. C 20. D 35. 8 50. 8
A. 70.65% wh ich utilizes the heat of the flue gases. 6. A 21 . 8 36. 8
41 . If the pressure is disregarded in the .
B. 81 .55% A. feedwater heater 7. 8 22. A 37. 0
various components of a steam power 8. C 23. 8 38. D
C. 78.99% B. heater valve
plant, the pressure drop in the pump is 9. C 24. D 39. D
D. 89.23% C. economizer
_ _ _ to the pressure drop in the 10. C 25. 8 40. 8
D. heater coils 11. C 26. 8 41 . A
turbine.
36 A steam power plant has a turbine A. proportional 12. 027. D 42. 8
work of 320 kJ/kg and the pump work is 47 . Type of boiler in which the water is 13. 028. 8 43. A
B. constant 14.' A 29. 8 44. 8
2 kJ!kg. If the mass flow rate is 10 kg outside the tubes while the hot gases
C. inversely proportional 15. A 30. C 45. D
per second, compute the power pass inside the tubes.
D. unity
produced by this plant? A. water tube
A. 3200 kw B. tubulous
42. If Va is the cylinder volume at the
B. 3180 kw C. horizontal
start of compression of an Otto cycle
C. 3220 kw D. fire-tube
and Vb is the cylinder volume after the
D. 2080 kw compression. Calculate the
48. If Wt is the shaft output of the gas
compression ratio.
37. Determine the amount of heat turbine, We is the compressor work and
A. r=VbNa
Q is the heat supplied, what is the
energy rejected by 0.5 cubic meters of B. r=VaNb
water when it is cooled from 35 degree thermal efficiency equal to?
C. r= Va -Vb
c to 6 degree c. A. Q/(Wt- We)
D. r= VaNb
A. 20, 117.5 kJ B. Q/(Wt +We)
B. 45,711.5 kJ C. (Wt + Wc)/Q
43. If Rc is the compression ratio of a
C. 30,117.5 kJ D. (Wt- Wc)/Q
spark-ignition cycle engine and Re is the
D. 60,711.5 kJ expansion ratio. Which of the following
49. If the available head of the
is true regarding the relation of the Rc
38. The h~at transferred to air during a hydrostation is 280 m and · available
andRe?
water flow is 1275 m 3/s, determine the
heat addition process of an ideal Otto A. Rc = Re
hydrostation water power.
cycle is 600 kJ/kg. The heat rejected is B. Rc > Re
A. 3000 MW
REFRESHER MANUAL 2nd Edition by JAS TORDILLO REFRESHER MANUAL 2nd Edition by JAS TORDILLO
31 - 8 I Day 31 - Solution Day 31 -Solution 131-9
A. equal to unity PV= nRT
Power and industrial Plant Engineering
B. less than unity (1)V = (5)(0.0821)(100 = 273)
C. Ef B. 25 m of water
C. greater than unity v = 153.·1 li D. Ef/Eg C. 17m of water
D. equal to zero *
D. 14mofwater
25. In a reversible process, like the Hfg = hg - hf = Eg -Ef
21. What is an isentropic process? A compression process in Otto or Diesel P(abs) = P(gage) + P(atm)
process that is cycle plotted on the p-V diagram. What 29. Identify the cycle: isentropic P(abs) = 85 + 101.325
A irreversible does the area on the diagram compression - isobaric heat addition - P(abs) = 186.325 kPa abs
B. reversible * represent?
adiabatic expansion - isom!'tric heat
C. heat transfer greater than zero A free energy change addition.
H = ~ = l86.3 25 =19m of water
D. heat transfer not equal to zero B. work done by the system * y 9.81
A Otto
C. heat transfer B. Diesel*
22. A 0.10 m3 gas is heated in a closed D. entropy change
C. C.arnot 33. Water enters a turbine pump at 30
cylinder. The initial temperature of the
D. Rankine kPa (vf = 0.0010223 cu. m per kg) and
gas is 350 deg K, until after heating the 26. Calculate the entropy, in BTU per discharges at 2.8 Mpa. The turbine
temperature of the gas is 600 deg K. Ibm per degree R, of steam at 60 psia 30. A steam generator produces 1007 work is 380 kJ/kg. What is the
What is the work done in the system per with a 60% amount of vapor. Btu per Ibm of heat at 100 psi a from percentage of turbine work output is
kg mass of gas? Cv of air is 0.716. A. 0.4274 •
14.7 psia of saturated liq uid. If the heat required to operate the pump of a steam
A. 0 kJ/kg * B. 1.1577 * input supplied is 800 Btu per Ibm, which power plant? ·
B. 275 kJ/kg c. 0.9014 of the following is true for the device? A. 1%
C. 225 kJ/kg D. 1.2172 A. The device cannot work because the B. 0.75% *
D. 350 kJ/kg
pressure is greater than zero. C. 0.9%
S = Sr + (X)Stg B. The device can work because output D. 0.62%
23. Gas is enclosed in a cylinder with a = Sr + X(Sg - Sr) is grater than input.
weighted piston as the top boundary. s = 0.4274 + 0.60(1.6446- 0.4274) C. The device violates the first law of Wp = vr(P1- P4)
The gas is heated and expands from a s = 1.1577 thermodynamics because output is = 0.0010223(2800- 30)
volume of 0.04 to 0.10 m3 at a constant
greater than input. * Wp = 2.83 kJ/kg
pressure of 200 kPa. Calculate the 27. A vessel with a volume of 1 cu. D. The device violates the second law
work done by the system. meter contains liquid water and water
A 8 kJ
B. 12 kJ *
vapor in equilibrium at 60 kPa. The
thermodynamics because input is less
than output. Y'+ = 2 ·~3 = 0.00745= 0.74S'/o
liquid water has a mass of 1 kg. Wr 380
c . 10 kJ Calculate the mass ofthe water vapor. 31. In a steam Rankine cycle, saturated
D 14 kJ At 600 kPa from steamtable: 34. Heat is transferred to a heat engine
liquid at 14.7 psia (vf = 0.01672 cu. ft.
Vf = 0.0011101 m3/kg at a rate of 90 kw. The rate of rejection
per Ibm) is pump to a high pressure
W = P(V2 - V1) Vg = 0.3157 m3/kg of waste heat to a sink is 50 kw.
liquid at 100 psia. How much work is
= 200(0.10- 0.04) A. 0.99 kg Compute the thermal efficiency of the
required to pump one Ibm of water?
W = 12 kJ B. 1.89kg
A. 118 Btu heat engine.
C. 1.57 kg A. 44.4% *
B. 0.264 Btu *
24. Five moles of water vapor at 100 D. 3.16 kg* B. 66.6%
C. 0.129 Btu
deg C and 1 atmosphere pressure are c. 55.5%
D. 18.5 Btu
compressed isobarically to form a liquid VT= Vf+Vg D. 77.7%
at 100 deg D. The process is reversible VT = mf(vf) + mg(vg)
Wp = mvr(Pr- P;)
and the ideal gas laws apply. Compute 1 = (1 )(0.001101) + mg(0.3157)
the initial volume of vapor if water vapor Wp = (1)(0.016772)(100 - 14.2) X· ~ QA -QR
mg = 3.16 kg
constant is 0.0821 Liter-atm per mole 778 llt = QR
per degree K. Wp = 0.264 Btu
28. If Eg is the enthalpy of the saturated , = 90 -
A. 1231i 50 = 0.4444 = 44.44%
vapor and Ef is the enthalpy of the t 90
B. 1431i 32. A pressure gauge attached to a
saturated liquid. What is the expression
c. 1331i pump reads 85 kPa. If the atmospheric
for the heat of vaporization? 35. What is the overall efficiency of the
D. 1531i * pressure is 29.92 in Hg, determine the ·
A Eg turbine if the power developed is 12
absolute pressure of the water pumped.
B. Eg -Ef* MW at a head of ·150 meters and the
A. 19m of water*
REFRESHER MANUAL 2nc1 Edition by JAS TORDILLO REFRESHER MANUAL 2nd Edition by JAS TORDILLO
[

31 - 10 I Day 31 -Solution Day 31- Solution 131-11


Power and Industrial Plant Engineering
nozzle discharges 10 cubic meters each QA - QR
second? llt= is true regarding the relation of the Rc 49. If the available head of the
QA
A. 70.65% andRe? · hydrostation is 280 m and available
B. 81.55% * 600-200 = 0.6666 = 66 .66% A. Rc = Re * water flow is 1275 m3/s, determine the
c. 78.99% Tlt = 600 B. Rc > Re hydrostation water power.
D. 89.23% C. Re < Rc A. 3000 MW
39. A turbine has an efficiency of 75% D. cannot be determined B. 4000MW
Wr =QyH x 11 and has a steam flow rate of 22,000 C. 3500 MW*
12,000 = 10(9.81)(150) X 11 kg/hr. If the available energy is 1200 44. If Rc is the compression ratio of a D. 5000 MW
'l = 0.8155 = 81.'55% kJ/kg, determine the kilowatt output at compression-ignition cycle engine and
full load. Re is the expansion ratio. Which of the WP = QyH
36. A steam power plant has a turbine A 3500 following is true regarding the relation of WP = 1275(9.81)(280)
work of 320 kJ/kg and the pump work is B. 4500 the Rc Re? WP = 3,502,170 kw = 3,502 MW
2 kJ/kg . If the mass flow rate is 10 kg C. 4000 A. Rc = Re
per second, compute the power D. 5500 * B. Rc > Re * 50. What is the volume of a rigid tank
produced by this plant? C. Rc < Re that contains 15 kg of water at a
A. 3200 kw Wr = m(h1 - h2)nr D. no relation temperature of 90 deg C (P = 84.55
B. 3180 kw* 22 000 kPa, Vf = 1.041/kg, Vg = 1982Ukg), if 10
C. 3220 kw W = • (1200)0.75 = 5500 kw 45. A valve that releases steam in case kg of this water is liquid and the
D. 2080 kw T 3600 remaining is in vapor form.
of excessive temperature caused by
low-water level. A 12.892 cubic meters
WN =Wr-Wp 40. The ratio of the average load to that A. safety valve B. 9.9204 cubic meters *
= 10(320 -2) of the rated capacity of a plant is called: B. stop valve C. 0.0156 cubic meters
WN = 3180 kw A output factor C. check valve D. 5.6549 cubic meters
B. load factor • D. fusible plug •
37. Determine the amount of heat C. demand factor Vr = VL + Vv
energy rejected by 0.5 cubic meters of D. capacity factor 46. A feedwater preheating device Vr = mLVt + mvVg
water when it is cooled from 35 degree which utilizes the heat of the flue gases. Vr= 10(0.00104) + (15-10)(1 .982)
C to 6 degree C. 41 . If the pressure is disregarded in the A. feedwater heater Vr.= 9.9204 m
3

A. 20,117.5 kJ various components of a steam power B. heater valve


B. 45,711 .5 kJ plant, the pressure drop in the pump is C. economizer •
C. 30,117.5 kJ _ _ _ to the pressure drop in the D. heater coils
D. 60,711.5kJ* turbine.
A. proportional * 47. Type of boiler in which the water is
QR = mCpllT B. constant outside the tubes while the hot gases
C. inversely proportional pass inside the tubes.
QR = (pV)(CpllT
QR : (1QQQ){Q.5)(4.187)(35-6) D. unity A. water tube
QR: 60,71 1.5 kJ B. tubulous
42. If Va is the cylinder volume at the C. horizontal
start of compression of an Otto cycle D. fire-tube *
38. The heat transferred to air during a
and Vb is the cylinder volume after the
heat addition process of an ideal Otto
compression. Calculate the 48. If Wt is the shaft output of the gas
cycle is 600 kJ/kg. The heat rejected is
compression ratio. turbine , We is the compressor work and
200 kJ/kg. Determine the thermal
efficiency of the cycle.
A r =VbNa Q is the heat supplied, what · is the
A. 33.33% B. r= VaNb* thermal efficiency equal to?
B. 55.55% C. r=Va-Vb A. Q/(Wt- We)
D. r=VaNb B. Q/(Wt +We)
C. 44.44%
D. 66.66% * C. 0/Vt + Wc)/Q
43. If Rc is the compression ratio of a D. (Wt- Wc}/Q *
spark-ignition cycle engine and Re is the
expansion ratio. Which of the following

REFRESHER MANUAL 2nd Edition by JAS TORDILLO REFRESHER MANUAL 2nd Edition by JAS TORDILLO
Day 32 -EXAM 132-1
Power and Industrial Plant Engineering
B. occur
DAY 32- EXAM C. may occur and may not occur
D. sometimes occur and sometimes not
1. What will happen when the pressure
at any point inside a centrifugal pump 7. If the liquid temperature (thus, vapor
goes below the vapor pressure pressure) is higher than for which the
corresponding to the temperatu re of the system was designed, cavitation will
liquid?
A. turbulent flow A. .not occur
B. laminar flow B. occur
C. cavitation C. may occur and may not occur
D. priming D. sometimes occur and sometimes not
2. Cavitation is the phenomenon which 8. If the Net Positive Suction Head
occurs in a pump only. (NPSH) is greater than the required
A. centrifugal suction head of the pump, cavitation will
B. rotary
C. reciprocating A. not occur
D. centrifugal & reciprocating B. occur
C. may occur and may not occur
3. Which of the following is/are the D. sometimes occur and sometimes not
effects of cavitation?
A. pitting due to chemical reaction 9. In order to avoid cavitation, the
between gasses and metal . (NPSH of an installation should be at
B. noise accomplishing the collapse of least than the required NPSH
the vapor bubbles. of the pump.
C. all of these A. equal or greater
D. decreased in capacity and vibration B. equal or lower
of the pump. C. lower
D. equal
4. If the discharges heads are far below
the pump's calibrated head at peak 10. Net Positive Suction Head (NPSH)
efficiency, cavitation will _ _ __ can be increased by which of the
A. not occur following:
B. occur A. increasing the height of the free fluid
C. may occur and may not occur level of the supply tank ·
D. sometimes occur and sometimes not B. Reducing the temperature of the fluid
C. Using a lager pipe size in the suction
5. If the speed of a pump is higher than side
the designed speed, cavitation will D. all of these

A. not occur 11. If Hb is the pressure head


B. occur corresponding to absolute pressure on
C. may occur and may not occur the surface of the fluid, Z is the height of
D. sometimes occur and sometimes not the fluid surface above or below the
pump centerline, hf is the total friction
6. If the suction head is lower than the head loss and Hv is the pressure head
manufacturer's recommendation, corresponding to the vapor pressure of
cavitation will _ _ _ _ the liquid. What is the available NPSH
A.. not occur of the pump?

REFRESHER MANUAL 2nd Edition by JAS TORDILLO


32 - 2 I Day 32 - EXAM Day 32 - EXAM I 32 - 3
Power and Industrial Plant Engineering
A. NPSH ;= Hb - Hv - Z + hf is located 3.60 m above the gasoline
B. NPSH = Hb- Hv - Z - hf level. 22. Achieving supersonic flow, it 28. Compare the efficiency (COP) of a
C. NPSH = Hb + Hv-Z-hf A 3.67 m requires a refrigeration cycle if the condensing
D. NPSH = Hb + Hv-Z + hf B. 7.66 m A. Convergent-divergent nozzle . te. nperature is increased.
C. 4.47 m B. Convergent flow A decrease in efficiency
12. If Pb is the absolute pressure on the D. 8.66 m C. Divergent flow B. increase in efficiency
surface of the liquid, Z is the suction lift D. Critical flow C. efficiency remains the same
of the pump, hf is the total friction head 16. A certain location has a barometric D. varies directly
loss and Pv is the vapor pressure of the reading of 720 mm Hg and the 23. What is the method used to
liquid at the existing temperature. gravitational acceleration is 9.18 m/s2. evaluate all welds performed on 29. It is important to take some
Determine the corresponding NPSH of Compute the atmospheric pressure. pressure parts of boiler tube materials? moisture from the air to dehumidify it
the pump. A. 91.99 kPa A. hydrostatic test and the relative humidity reaches high
B. 94.99 kPa B. orsat analysis levels. To do this, it requires cooling the
A. NPSH= Pa -Pv +Z+hf C. 93.99 kPa
(I) C. vacuum test air
D. 95.99 kPa D. radiographic test A. at wet-bulb temperature
B. NPSH = Pa - Pv + Z- hf B. above its dew point temperature
(I) 17. A single-acting reciprocating pump, 24. A boiler testing method that is used C. at critical tempe,rature
with a diameter of 18 em and a stroke of to verify the microstructure of the boiler D. below its dew point temperature
C. NPSH= Pa +Pv +Z-hf 30 em, delivers water at a rate of 0.18
(I) tubes using optical microscopes.
cubic meter per second. What is the A. metallurgical replication 30. The relative humidity of saturated
D. NPSH=Pa+Pv+Z+hf percentage slip it delivers at 1500 rpm? B. radiographic testing air is
(I) A. 1.6% C. optical testing A. 0%
B. 5.3% D. tube sampling B. 75%
13. If cavitation will occur, the available C. 8.4% C. 25%
NPSH is insufficient. Which of the D. 10.5% 25. The capacity of a centrifugal pump, D. 100%
following is a remedy to overcome Q, is proportional to the
cavitation? 19. Regardless of its rotative speed, the impeller diameter D at same speed. 31. The heat transferred to the engine is
A. decrease the available NPSH value of the specific speed of a given A . directly at a rate of 80 MW. Compute the
B. increase the required NPSH impeller of centrifugal pump is: B. inversely thermal efficiency of the heat engine
C. increase the available NPSH A varying C. unity when the rate of waste heat rejection is
D. none of these B. ' constant D. directly and inversely 50MW.
C. unity A. 37.5%
14. Water is pumped from a pump D. increasing 26. The capacity of a centrifugal pump, B. 62.5%
where the pressure is atmospheric to a Q, is proportional to the C. 60%
height of 3 m, water temperature is 40 20. Specific speed of a centrifugal pump speed N in the same pump. D. 40%
deg C (Pv = 7.3845 kPa), loss of head is to its operating speed. A. directly
due to friction is 0.75 m. Find the A .indirectly proportional B. indirectly 32. What pr.ocess is employed when
available NPSH of the pump. B. directly proportional C. inversely the turbine in steam power plants
A. 5.8m C. inversely proportional D. inversely and -directly experience excessive moisture?
B. 7.2 m D. unity A subcooling
C. 6.2m 27. Compare the refrigerating effect per B. freezing
D. 8.8m -21. Once sonic velocity has been unit mass of refrigerant for a saturated C. supercooling
achieved in a duct or nozzle throat, the cycle and for a superheated cycle that D. reheating
15. Determine' the available suction the mass flow rate will be at its maximum produces useful cooling, for the same
head pump that is taking a gasoline at and will remain constant for all vaporizing and condensing temperature. 33. If H is the work output of a heat
204° C frqm a close tank with a pressure subsequent decreases in ambient A greater for a superheated cycle engine and Q is the total heat input of
of 585 kPag. The specific gravity of pressure. This condition is called B. lower for a superheated cycle an engine, then determine its thermal
gasoline is 0.78 and its vapor pressure A Choked flow C. greater for a saturated cycle efficiency.
is 620 kPa absolute. The loss in suction B. Overflow D. the same for saturated and A . H/Q
pipe is 0.60 m and the pump centerline C. Divergent flow superheated cycle B. Q/H
D. Constant flow

REFRESHER MANUAL 2nd Edition by JAS TORDILLO REFRESHER MANUAL 2 nd Edition by JAS TORDILLO
32 - 4 I Day 32 - EXAM Day 32 - EXAM I 32 - 5
Power and Industrial Plant Engineering
C. 1- H/Q B. saturation pressure increases
D. Q/H -1 C. saturation pressure decreases B. fuel C. 33.3
D. Saturation pressure remains the C. fuel and air D. 13.3
34. What type of fuel is used w ith same D. hydrogen
stationary power plants where gas is
available in the pipeline?
A. coal
40. What is the effect on the saturation
temperature if the pressure of the fluid is
46. What is the generated frequency of lr1
a generator?
B. solid fuels increased? DAY 32- ANSWER KEY
A. 120P/N 1. c 16.0 31. A.46. 0
C. gaseous fuels A. saturation temp Increases B. 120N/P 2.A 17.8 32. 047. B
D. liquid fuels B. saturation temp decreases C. 120/NP 3. C 18. B 33. A48. A
C. saturation temp remains the same D. NP/120 4. B 19. B 34. C49. C
35. lfWb is shaft work of an engine and D. there is no effect 5. B 20. B 35. A 50. 0
Wi is the indicated work of an engine, 6. B 21.A 36. C
47. Air enters the cylinder of an internal 7. B22. A 37. 0
then which of the following relationship 41. For steam power plants, decreasing combustion engin.e at an initial pressure 38. B
B. A 23. 0
is correct if mechanical friction is the operating pressure of the boiler will: and temperature of 95 kPa and 28 deg 9. A 24.A 39. 8
present in the engine? A. increase thermal efficiency C. A four cylinde r, single-acting, 40mm 10. 0 25. A 40. A
A. Wb is less than WI B. decrease.thermal efficiency x 55 mm, four stroke engine runs at 11 . B 26. A 41. B
B. Wb is greater than WI 12.B27.A 42. 0
C. bring thermal efficiency 2800 rpm . The mass flow rate of air per 13. C2B. A 43. A
C. Wb is equal to WI D. make thermal efficiency seconds is 0.01 kg. What is the 14. A29. 0 44.C
D. Wb is proportional to Wi volumetric efficiency of the engine? 15. c 30. 0 45.C
42. One foot of water is equal to: A. 72%
36. A diesel generating unit, with a A . 0.4138 lb/square inch B. 87%
generator efficiency of 85%, has an B. 68.3 lb/square foot C. 81%
output of 900 kw-hr. If the mass of fuel C. 0.8678 inches of Hg D. 97.7%
is 200 kg, compute the fuel rate of the D. 62.43 lb/square foot
engine in kg per kw-hr. 48. A 20 em x 36 em, twin cylinder, two
A . 0.80 43. In an ideal Otto cycle, the initial stroke cycle diesel engine running at
B. 0.40 pressure and temperature of air are 100 650 rpm . The engine rate is 112 kw.
c. 0.19 kPa and 18 degree C. Determine the Determine the engine displacement in
D. 0.95 maximum pressure in the cycle if the cubic meter/sec per brake horsepower.
maximum temp~rature in the cycle is A. 0.00163
37. What is the effect of superheating 600 degree C, and the compression B. 0.0163
the suction vapor with regards to the ratio is 8. C. 0 .16300
work of compressor per unit mass of A. 2400 kPa D. 0.0631
refrigerant circulated? B. 1890 kPa
A. decreases C. 2240 kPa 49. A 1000 kw-hr diesel generating set
B. remains the same D. 1500 kPa has a generator efficiency of 85%. If the
C. proportional mass of the fuel is 150 kg , compute the
D. increas~s 44. An engine rejects 209 kJ of heat to engine fuel rate.
the atmosphere at 50 degree C. What A . 0. 15 kg/kw-hr
38. What is the pressure present inside is the amount of heat gain if the B. 0.1175 kg/kw-hr
the casing of an impulse turbine? temperature of the heat source is 500 C . 0.127 kglkw-hr
A. vacuum degree C? D. 22.7 kg/kw-hr
B. atmospheric pressure A. 400 kJ
C. above atmospheric B. 600 kJ 50. A condenser vacuum gauge reads
D. zero C. 500 kJ 660 mm Hg when the barometer stands
D. 650 kJ at 29.92 in Hg. State the absolute
39. What is the effect on saturation pressure in the condenser in kN/m 2 .
pressure if the temperature of the fluid is 45. In a spark-ignition engine, what is A. 44.4
increased? being compressed? · B. 23.3
A. there is no effect A. air

REFRESHER MANUAL 2nd Edition by JAS TORDILLO REFRESHER MANUAl 2nd Edition by JAS TORDillO
32 - 6 I Day 32 - Solution
Day 32 - Solution 1 32 - 7
C. may occur and may not occur Power and Industrial Plant Engineering
DAY 32- SOLUTION D. sometimes occur and sometimes not
C. NPSH = Hb + Hv- Z - hf 15. Determine the available suction the
1. What will happen when the pressure 7. If the liquid temperature (thus, vapor D. NPSH = Hb + Hv- Z + hf head pump that is taking a gasoline at
at any point inside a centrifugal pump pressure) is higher than for which the 204° C from a close tank with a pressure
goes below the vapor pressure system was designed, cavitation will 12. If Pb is the absolute pressure on the of 585 kPag. The specific gravity of
corresponding to the temperature of the surface of the liquid, Z is the suction lift gasoline, is 0. 78 and its vapor pressure
liquid? A. .not occur of the pump, hf is the total friction head is 620 kPa absolute. The loss in suction
A. turbulent flow B. occur* loss and Pv is the vapor pressure of the pipe is 0.60 m and the pump centerline
B. laminar flow C. may occur and may not occur liquid at the existing temperature. is located 3.60 m above the gasoline
C. cavitation * D. sometimes occur and sometimes not Determine the corresponding NPSH of level.
D. priming the pump. A . ·3.67 m
8. If the Net Positive Suction Head B. 7.66 m
A. NPSH= Pa - Pv +Z+hf
2. Cavitation is the phenomenon which (NPSH) is greater than the required C. 4 .47 m *
Ol
occurs in a pump only. suction head of the pump, cavitation will D. 8.66 m
A. centrifugal * B. NPSH = Pa - Pv + Z- hf •
B. rotary A. not occur * (0
NPSH= pb - Pv +Z - hf
C. reciprocating B. occur · y
C. NPSH=p9 +Pv +Z - hf
D. centrifugal & reciprocating C. may occur and may not occur
Ol
D. sometimes occur and sometimes not
3. Which of the following is/are the NPSH = ( 585 + 101.325)- 620
D. NPSH= Pa +Pv +Z+hf 0.78(9.81) -3.6-0.60
effects of cavitation? 9. In order to avoid cavitation , the Ol
A. pitting due to chemical reaction (NPSH of an installation should be at
between gasses and metal . least than the required NPSH 13. If cavitation will occur, the available NPSH = 4.467 m
B. noise accon:plishing the collapse of of the pump. NPSH is insufficient. Which of the
the vapor bubbles. A. equal or greater * following is a remedy to overcome 16. A certain location has a barometric
C. all of these • B. equal or lower cavitation? reading of 720 mm Hg and the
D. decreased in capacity and vibration C. lower A. decrease the available NPSH gravitational acceleration is 9.18 m/s2.
of the pump. D. equal B. increase the required NPSH Compute the atmospheric pressure.
C. increase the available NPSH • A. 91.99 kPa
4. If the discharges heads are far below 10. Net Positive Suction Head (NPSH) D. none of these B. 94.99 kPa
the pump's calibrated head .at peak can be increased by which of the C. 93.99 kPa
efficiency, cavitation will _ _ __ following: 14. Water is pumped from a pump D. 95.99 kPa *
A. not occur A. increasing the height of the free fluid whAre the pressure is atmospheric to a
B. occur• level of the supply tank height of 3 m, water temperature is 40 10 25
C. may occur and may not occur B. Reducing the temperature of the fluid deg C (Pv = 7.3845 kPa), loss of head
720x 1.3 =95.99kPa
D. sometimes occur and sometimes not C. Using a lager pipe size In the suction
760
due to friction is 0.75 m. Find the
side available NPSH of the pump. 17. A single-acting reciprocating pump,
5. If the speed of a pump is higher than D. all of these • A. s.sm• with a diameter of 18 em and a stroke of
the designed speed, cavitation will B. 7.2 m 30 em, delivers water at a rate of 0.18
11 . If Hb is the pressure head C. 6.2m cubic meter per second. What is the
A. not occur corresponding to absolute pressure on D. 8.8m percentage slip it delivers at 1500 rpm?
B. occur• the surface of the fluid, Z is the height of
A. 1.6%
C. may occur and may not occur the fluid surface above or below the
D. sometimes occur and sometimes not NPSH = Pa - Pv +Z-hf B. 5.3%"
pump centerline, hf is the total friction
(1) C. 8.4%
head loss and Hv is the pressure head
6. If the suction head is lower than the NPSH= 101.325-7.3845 _ _ D. 10.5%
corresponding to the vapor pressure of 3 0 75
manufacturer's recommendation, the liquid. What is the available NPSH 9.81
cavitation will _ _ _ _.
A. .not occur
of the pump? Vo= ~D 2 LN=1!.(0.18)
4
2 (0.30)( 1500 )
A. NPSH = Hb - Hv - Z + hf NPSH=5.82m 4 60
B. occur* =
B. NPSH Hb - Hv- Z - hf * Vo = 0.19m3/sec

REFRESHER MANUAL 2nd Edition by JAS TORDILLO


REFRESHER MANUAL 2nd Edition by JAS TORDILLO
32-81 Day 32- Solution Day 32 - Solution 1 32 - 9
Q=0.18 ' 24. A boiler testing method that is used Power and Industrial Plant Engineering
V -Q to verify the microstructure of the boiler
0 C. at critical temperature 35. If Wb is shaft work of an engine and
%slip = - - xl 00 tubes using optical microscopes.
Vo D. below its dew point temperature* Wi is the indicated work of an engine,
A. metallurgical replication *
then which of the following relationship
. 0.19-0. 18 100 B. radiographic testing
o/coSIip = X C. optical testing· 30. The relative humidity of saturated is correct if mechanical friction is
0.19 air is present in the engine?
D. tube sampling
A. 0% A. Wb is less than Wi *
%slip = 5.3% B. 75% B. Wb is greater than Wi
25. The capacity of a centrifugal pump,
Q, is . . proportional to the C. 25% C. Wb is equal to Wi
19. Regardless of its rotative speed, the D. 100% * D. Wb is proportional to Wi
impeller diameter D at same speed.
value of the specific speed of a given
impeller of centrifugal pump is: A. directly *
31 . The heat transferred to the engine is 36. A diesel generating unit, with a
B. inversely
A. varying at a rate of 80 MW. Compute the generator efficiency of 85%, has an
B. constant" C. unity
thermal efficiency of the heat engine output of 900 kw-hr. If the mass of fuel
D. directly and inversely
C. unity when the rate of waste heat rejection is is 200 kg, compute the fuel rate of the
D. increasing 50MW. engine in kg per kw-hr.
26. The capacity of a centrifugal pump,
Q, is proportional to the A. 37.5% * A. 0.80
20. Specific speed of a centrifugal pump B. 62.5% B. 0.40
speed N in the same pump.
is to its operating speed.
A. directly * c. 60% c. 0.19 *
A. indirectly proportional D. 40% · D. 0.95
B. directly proportional * B. indirectly mf 200
C. inversely - = / = 0. 1888 kg I kw - hr
C. inversely proportional QA -QR BP
900
/ 0 .85
D. inversely and directly
D. unity 11 = QA
27. Compare the refrigerating effect per 37. What is the effect of superheating
21. Once sonic velocity has been
unit mass of refrigerant for a saturated 80-50 = 0.375 = 37.5 the suction vapor with regards to the
achieved in a duct or nozzle throat, the
cycle and for a superheated cycle that
'1=80 work of compressor per unit mass of
mass flow rate will be at its maximum
produces useful cooling, for the same refrigerant circulated?
and will remain constant for all 32. What process is employed when
vaporizing and condensing temperature. A. decreases
subsequent decreases in ambient the turbine in steam power plants
A. greater for a superheated cycle * B. remains the same
pressure. This condition is called experience excessive moisture?
B.· lower for a superheated cycle C. proportional
A. Choked flow * A. subcooling
C. greater for a saturated cycle D. increases *
B. Overflow D. the same for saturated and B. freezing
C. Divergent flow C . supercooling
superheated cycle 38. What is the pressure present inside
D. Constant flow D. reheating* the casing of an impulse turbine?
28. Compare the efficiency (COP) of a A. vacuum
22. Achieving supersonic flow, it
refrigeration cycle if the bondensing 33. If H is the work output of a heat B. atmospheric pressure*
requires a temperature is increased. engine and Q is the total heat input of C. above atmospheric
A. Convergent-divergent nozzle * A. decrease in efficiency * an engine, then determine its thermal D. zero
B. Convergent flow B. increase In efficiency efficiency.
C . Divergent flow C. efficiency remains the same A. H/Q * 39. What is the effect on saturation
D. Critical flow D. varies directly B. Q/H pressure if the temperature of the fluid is
C. 1- H/Q increased? ·
23. What is the method used to 29. It is important to take some D. Q/H - 1 A. there is no effect
ev aluate all welds performed on moisture from the air to dehumidify it B. saturation pressure increases *
pressure parts of boiler tube materials? and the relative humidity reaches high 34. What type of fuel is used with C. saturation pressure decreases
A. hydrostatic test levels. To do this, it requires cooling the stationary power plants where gas is D. Saturation pressure remains the
B. orsat analysis air available in the pipeline? same
C. vacuum test A. at wet-bulb temperature A. coal
D. radiographic test* B. above its deW point temperature B. solid fuels I 40. What is the effect on the saturation
C. g_as~ous fuels * temperature if the pressure of the fluid is
D. liqUid fuels increased?
REFRESHER MANUAL 2nd Edition by JAS TORDILLO REFRESHER MANUAL 2nd Edition by JAS TORDILLO
32- 10 1 Day 32 ~Solution Day 32 - Solution I 32 - 11
A. saturation temp increases * 44. An engine rejects 209 kJ of heat to Power and Industrial Plant Engineering
B. saturation temp decreases the atmosphere at 50 degree C. What
C. saturation temp remains the same is the amount of heat gain if the P,V,' = mRT c. 33.3
D. there is no effect temperature of the heat source is 500 95V,' =0.01(0.287)(28 +273) D. 13.3 *
degree C? v,·=0.0090933 m/sec
P(abs) = P(gage) + P(atm)
41 . For steam power plants. decreasing A. 400 kJ V0 = 11 D 2LNC = -660 + 760
the operating pressure of the boiler will: B. 600 kJ 4
= 100 mm Hg
A. increase thermal efficiency c. 500 kJ * v,0 =
11 4500
(0.040) 2 (0.055)( )4
B. decrease thermal efficiency * D. 650 kJ 4 60x2 2
C. bring thermal efficiency Vo = 0.01036725 m3 /sec P(abs) = 100 mm Hg x 101.325 kN/m
D. make thermal efficiency Eff = TH- TL = QA- QR 760 mmHg
TH QA
VI - 0.0090933 = 0.877 = 87.7% P(abs) = 13.3 kN/m 2
42. One foot of water is equal to:
500- so QA- 209 nv = v0 - 0.010367255
A. 0.41381b/square inch
B. 68.3 lb/square foot 500 + 273 QA
48. A 20 em x 36 em, twin cylinder. two
C. 0.8678 inches of Hg
QA : 5QQ kJ stroke cycle diesel engine running at
D. 62.43 lb/square foot*
650 rpm. The engine rate is 112 kw.
45. In a spark-ignition engine, what is Determine the engine displacement in
43. In an ideal Otto cycle, the initial
pressure and temperature of air are 100 being compressed? cubic meter/sec per brake horsepower.
kPa and 18 degree C. Determine the A. air· A. 0.00163 *
maximum pressure in the cycle if the B. fuel B. 0.0163
maximum temperature in the cycle is C. fuel and air* c. 0.16300
600 degree C, and the compression D. hydrogen D. 0.0631
ratio is 8.
A. 2400 kPa * 46. What is the generated frequency of 1t
V _ (0.20) 2 (0.36)(650)2
B. 1890 kPa a generator? 4
A. 120P/N B0 - 60
C. 2240 kPa p 11 2
D. 1500 kPa B. 120N/P
C. 120/NP V, 0.746
0 l
T7 =(rk,\1<-1
D. NP/120" BP = 0.001632 m /sec
HP
Tl N _ 120f
T2 = (8)t.~-l p 49. A 1000 kw-hr diesel generating set
(18 = 273) has a generator efficiency of 85%. If the
f = NP mass of the fuel is 150 kg, compute the
T2 = 668.54 °k 120 engine fuel rate .
A. 0.15 kg/kw-hr
47. Air enters the cylinder of an internal
p2 = (rk )k combustion engine at an initial pressure
B. 0.1175 kg/kw-hr
PI C. 0.127 kg/kw-hr *
and temperature of 95 kPa and 28 deg
D. 22.7 kg/kw-hr
.!2. = (8)1.4 C. A four cylinder, single-acting, 40mm
x 55 mm, four stroke engine runs at
100 mf
P2 = 1837.91 kPa
2800 rpm. The mass flow rate of air per = 1000150, =0.1275 kg I kw - hr
PJ p1
seconds is 0.01 kg. What is the BP /o.85
-=- volumetric efficiency of the engine?
T1 T2 A. 72% 50. A condenser vacuum gauge reads
P3 1837.91 B. 87% * 660 mm Hg when the barometer stands
-- =-- C. 81% at 29.92 in Hg. State the absolute
600 + 273 668.54 D. 97.7% 2
pressure in the condenser in kN/m .
PJ = 2400 kPa
A. 44.4
B. 23.3

REFRESHER MANUAL 2nd Edition by JAS TORDILLO REFRESHER MANUAL 2nd Edition by JAS TORDILLO
Day 33 - EXAM I 33 - 1
Power and Industrial Plant Engineering
and reheat temperature of the steam is
DAY 33- EXAM 540 deg C, condenser pressure is 3.4
-- kPa, engine efficiency of high pressure
1. The pressure and torque spent in;;.. and low pressure is 75%.
overcoming friction of reciprocating and Steam properties:
revolving parts of the engine and the At 13.85 MPa & 540 deg C:
automobile before it reached the drive h = 3434.1 (interpolated)
shaft.
A. friction power At 2.8 MPa & 540 deg C: h = 3548.5
B. pressure loss
C. torque I At 2.8 MPa: h = 2974.9
D. power loss
At 0.0034 MPa: h = 2204.5
2. Determine the air-standard efficiency =
hf 109.84
of an engine operating on the diesel Vf = 0.0010032
cycle when the suction pressure is
99.97 ~Pa and the fuel is injected for 6% 6. Determine the enthalpy at the
of the stroke, the clearance volume is entrance to the boiler in kJ/kg
8% of the stroke. Assume k = 1.4. A. 109.M
A. 60% B. 123.73
B. 80% c . 118.25
C. 55% D. 136.95
D. 50%
7. Find the reheat cycle turbine work in
3. In problem No. 2, what is the kJ/kg.
compression ratio? A. 1890.4
A. 13.5 B. 1250.4
B. 6 C .. 1352.4
C. 8 D. 1004.4
. '
D.4
8. Find the pump work in kJ/kg.
4. In problem No. 2, what is the cut-off A. 123.73
ratio? B. 13.89
A. 4 c. 109.84
B. 8 D. 11 .26
c. 6
D. 1.75 9. What is the heat added in the boiler
and reheating in kJ/kg?
5. In problem No. 2, what is the A. 3884
clearance ratio as a percentage of the B. 2828
piston volume displacement? c. 3454
A. 4% D. 1944
B. 10%
C. 8% 10. Determine the reheat cycle thermal
D. 6% efficiency?
A . 65.5%
Problem 6 to 10 B. 45.5%
A reheat steam cycle has 13.85 MPa · c. 54.5%
throttle pressure at the turbine inlet and D. 34.5%
a 2.8 MPa reheat pressure, the throttle

REFRESHER MANUAL 2nd Edition by JAS TORDILLO


33 - 2 I Day 33 - EXAM Day 33 - EXAM I 33 - 3
Power and Industrial Plant Engineering
Problems 11 to 14 B. 15,185
A 650 bhp diesel engine uses fuel oil of c. 11,855 Problems 22 to 24 27. What is the pressure above zero?
28°API gravity at 45 deg C, fuel D. 15, 855 A circular fuel oil tank 45 feet long and A. gage pressure
consumption is 0.56 lbs per bhp-hr. 5.5 feet diameter is used for 011 storage. · B. vacuum pressure
Cost of fuel is P7.95 per liter. 16. Find the boiler horsepower. and supplying fuel oil at 34°API to a- C. absolute pressure
A. 2578 steam boiler. The stem flow is 2000 lbs D. atmospheric pressure
11. What is the density of the fuel used B. 2278 per hour at 200 psia dry steam.
in ~g/li? C. 1978 Feedwater is 230 deg F and boiler 28. Which of the following cycle is for a
A. 0.887 D. 1178 efficiency is 75%. compression-ignition engine?
B. 0.884 Stream Properties: A. diesel cycle
c 0.869 17. What is the boiler efficiency? At 200 psia (1.38 Mpa): B. dual cycle
D. 0.821 A. 85% hg = 2789.6 kJ/kg C. otto cycle
B. 65% At 230 deg F (11 0 deg C): D. all of these
12. What is the fuel consumption in c. 74% hf = 461.3 kJ/kg
liters per hr? D. 50% 29. The heating value obtained when
A. 220.5 22. Determine the mass flow rate of fue[ water in the products of combustion is in
B. 225.0 18. An evaporator constructed of pipe in kg per hr. the liquid state:
C. 202.5 or tubing. A. 61.5 A. higher heating va lue
D. 255.5 A. expansion coil B. lower heating value
B. 89.5
B. condenser coil c. 51 .6 C. calorific value
13.. Determine the minimum volume of C. evaporative condenser D. 95.8 D. average heat value
the required day tank. D. cooling tower
A. 1952 liters 23. Determine the density of fuel in kg 30. Dry air can be approximated as
B. 51921iters 19. The name given to a procedure by per cu. meter. _ _% oxygen and _ _% nitrogen
C. 2952 liters which the efficiency of a combustion A. 1000 by mole numbers.
D. 5292 liters process is determined. B. 925 A. 30/70
A. heat balance C. 995 B. 21/79
14. Find the cost of fuel per day. B. theoretical balance D. 855 C. 70/30
A P42,071 C. mass balance D. 79/21
B. P41,271 D. actual balance 24. Calculate the number of days the
C P42,710 supply tank can hold for continuous 31. A 'volume of 450 cc of a1r 1s
D. P40,701 20. A large pipe or drum into which of a operation. measured at a pressure of 740 mm Hg
group of boiler is connected. Also used A. 25 days abs and a temperature of 20 ° C. What
Problems 15 to 17 for a large pipe from which a number of B. 30 days is the volume in cc at 760 mm Hg abs
A water tube boiler evaporated 5.05 kg smaller ones are connected in line and C. 17 days and 0°C?
of wate·r per sec from a feed water from the side of the large pipe. D. 20 days A. 516.12
temperature of 104.44 deg C to steam A. line pipe B. 620.76
at 1241 .1 kPa and quality · of 0.97; B. line drum 25. At 101.325 kPa, the boiling point of c. 408.25
weight of coal fuel per sec 0.57 kg; high C. header water is 1OOOC. If the pressure is D. 375.85
heat value of the coal as fired, 11 ,800 D. main pipe increased, the boiling temperature will:
Btu/lb. A. increase 32. A heat engine is operated between
Steam Properties: 21. A device having a pre-determined B. remains the same temperature limits of 1370 C and 260 C.
At 104.44 deg C: hf = 437.78 temperature valve for the relief of C. decrease Engine supplied with 14,142 kJ!kw-hr.
(interpolated) pressure. · D. lower than Find the the carnot cycle efficiency in
At 1241.1 kPa: hf = 805.29 A. exhaust valve percent.
hfg = 1980.6 B. fusible plug 26. In one hour, what is the BTU A. 70.10
(interpolated) C. expansion valve equivalent of one HP? B. 67.56
D. spark plug A. 778 C, 65.05
15. Determine the rate of heat B. 746 D. 69.32
absorption in kJ/kg. C. 3.516
A 11,558 D. 2545

REFRESHER MANUAL 2nd Edition by JAS TORDILLO REFRESHER MANUAL 2nd Edition by JAS TORDILLO
33 - 4 I Day 33 - EXAM Day 33 - EXAM I 33 - 5
33. What should be the temperature of
Power and Industrial Plant Engineering
39. Calculate the enthalpy of 3 lbs of
both the water and steam whenever fluid that occupy a volume of 20 cu. ft if C. 80 sensible, 20 sensible and latent
B. constant
they are present together? the internal energy is 1000 Btu per lb C. decreases heat
A. saturation temperature for the and the pressure is 2 atm abs. D. 20 latent, 80 sensible and latent heat
D. unity
existing pressure A. 3109 Btu
B. superheated temperature B. 3910 Btu 50. A 4 m x 4 m x 4 m room has a
45. A steam reciprocating pump has a
C. boiling point of water at 101.325 kPa C. 3509 Btu relative humidity of 80%. The pressure
nameplate of 6 in x 4 in x 8 in. What is
D. one hundred degree centigrade D. 4590 in the room is 120 kPa and temperatu re
the steam bore equal to:
A. 6 in is 35°C (Psat = 5.628 kPa). What is the
34. Determine the average Cp value in 40. A hydraulic turbine receives water mass of the vapor in the room?
B. 48 in
kJ/kg-K of a gas if 522 kJ of heat is from a reservoir at an elevation of 100 (Rv = 0.4615 kPa-m3/kg-°K)
C. 8 in
required to raise the temperature from meter's above it. What is the minimum A. 2.03
D. 10 in
300K to BOOK making constant water flow in kg per second to produce a B. 0.80
pressure. steady turbine output of 50 MW? C. 1.50
46. A certain gas, which flow into an
A. 1.440 A. 50,247 .
D. 4.80
expansion control and without doing any
B. 1.038 B. 50,672 work. This process is called :
C. 1.044 c. 50,968 A. condensing
D. 1.026 D. 50,465
B. priming
35. The temperature at which its vapor
pressure is equal to the pressure
41. A reaction turbine develops 500
BHP, flow through the turbine is 50
C. throttling
D.. knocking ra DAY 33- ANSWER KEY
,I )
exerted on the liquid: ft3 /sec. Water enters at 20 ft/sec with 47. A Pelton wheel is to be designed to 1.A 16.0 31 . C46.C
A. absolute humidity 100 ft pressure head. Elevation of the 2. A 17. C 32. B 47. B
run at 300 rpm under an effective head
B. thermal energy water at turbine inlet above the tailwater 3. A 18.A 33. A48.A
of 150 m. The ratio of the nozzle 4.0 19.A 34.C49.A
C. boiling point is 10 ft. What is the effective net head? diameter to the diameter of the pitch 5. C 20. C 35. C 50. A
D. calorimeter A. 130ft circle is 1/12. Assuming efficiency of 6. B 21 . B 36. C
B. 110ft 84%, what is the size of the wheel in 7. C 22. A 37. A
36. Peak load for a period of time C. 116.2 ft 8. B 23. 0 38. B
meter. Assume speed ratio of 0.45.
divided by installed capacity is: D. 120.4ft 9. A 24. C 39. A
A. 1.05 10.025.A 40. C
A. capacity factor
B. 1.55 1.1.C26.0 41 . C
B. demand factor 42. One whose pressure is above the 12.A27. C 42.0
C. 2.0
C. utilization factor saturation pressure corresponding to its D. 2.86 n 028.A 43. B
D. load factor temperature: 14.A29. A 44. A
A. mixture 15. A 30. B 45. A
48. Cooling air in intercoolers between
37. The thermal efficiency of a particular B. saturated liquid compression stages in a multi-stage air (?9 I)
of a particular engine operating on an C. saturated vapor compressor.
ideal cycle is 35%. Calculate the heat in D. compressed liquid A. keeps the moisture in the air from
kJ supplied to the engine if the engine
evaporating
develops 1200 W-hr. 43. The latent heat of vaporization B. allows slightly lower volume of free
A. 12,343 decreases as the pressure and air to be handled for a constant speed
B. 14,218 temperature of the liquid increases, at compression
C. 10,216 the critical the heat of vaporization C. decreases substantially the
D. 11,108
volumetric efficiency ·
A. decreases
D. requires less HP for the compressor
38. An otto engine has a clearance B. become zero
that without intercoolers
volume of 7%. It produces 300 kw C. constant
power. What is the heat rejected in kw? D. increases
A. 170 . 49. The sensible heat ratio is 0.8, which
means:
B. 152 44. If the load is decreased in a gas A. 20 latent, 90 sensible
c. 160 power plant, the exhaust temperature: B. 20 sensible, 20 latent
D. 145 A. increases

REFRESHER MANUAL 2nd Edition by JAS TORDILLO REFRESHER MANUAL 2nd Edition by JAS TORDILLO
33 • 6 1 Day 33 • Solution Day 33 • Solution I 33 • 7
Power and Industrial Plant Engineering.
4. In problem No. 2, what is the cut-off
DAY 33 - SOLUTION ratio? A. 1890.4 11. What Is the density of the fuel used
A.4 B. 1250.4 in l<glli?
1. The pressure and torque spent in B. 8 c. 1352.4* ·. A. 0.887
overcoming friction of reciprocating and C. 6 D. 1004.4 B. 0.884
revolving parts of the engine and the D. 1.75 * c. 0.869 *
automobile before it reached the drive Wr = (h, -h2)0.85 + (h3 - h4}0.75 D. 0.821
shaft. 5. In problem No. 2, what is the Wr = (3434.1 - 2974.9)0.75
A. friction power *
B. pressure loss
clearance ratio as a percentage of the + (3548.5- 2204.5)0.75 11
API = ~-131 .5
piston volume displacement? Wr = 1352.4 kJ/kg SG u.6
C. torque A. 4%
D. power loss B. 10% 8. Find the pump work in kJ/kg . 28 = ~-131 . 5
SG IS.6
c. 8% * A. 123.73
SG,s.e = 0.887
2. Determine the air-standard efficiency D. 6% B. 13.89 *
of an engine operating on the diesel
c = V2 = 0.08Vo = 0.08 = 8% c. 109.84
=
SG1 SG,s.e[1 - 0.0007(t- 15.6))
cycle when the suction pressure is D. 11 .26
99.97 kPa and the fuel is injected for 6% Vo Vo SG4s = 0.887[1 - 0.0007(45- 15.6))
of the stroke, the clearance volume is SG4s = 0.8687
WP = Vs(Pe- Ps)
8% of the stroke. Assume k = 1.4. Problem 6 to 10 WP = 0.0010032(13,850- 3.4)
A. 60% * A reheat steam cycle has 13.85 MPa Density of fuel= 0.8687 (1)
WP = 13.89 kJ/kg
B. 80% throttle pressure at the turbine inlet and = 0.8687 kg/li
C. 55% a 2.8 MPa reheat pressure, the throttle 9. What is the heat added in the boiler
D. 50% and reheat temperature of the steam is 12. What is the fuel consumption in
and reheating in kJ/kg?
540 deg C, condenser pressure is 3.4 liters per hr?
A. 3884 *
Vo = v,- V2 kPa, engine efficiency of high pressure A. 220.5 *
B. 2828
V2= 0.08Vo and low pressure is 75%. B. 225.0
C. 3454
VJ- V2 = 0.06Vo Steam properties: D. 1944
c. 202.5
At 13.85 MPa & 540 deg C: D. 255.5
V 1 V0 + V 2 V + 0.08V0 h = 3434.1 (interpolated) QA = (h, - he} + (h3- h2)
rk = - =--= 0 V . = mf = 0.651b x650 Bh
V2 V2 o.o8V0 QA: (3434.1 -123.73) 1
At 2.8 MPa & 540 deg C: h = 3548.5 + (3548.5- 2974.9}
P1 ' Bhp - hr p
rk = 13 .5
kg
r = V 3 = V 2 + 0.06V0 = 0.08 + 0.06 At 2.8 MPa: h = 2974.9
QA = 3884 kJ/kg x x -li--
2.205 1bs 0.869kg
c ·v V2 o.o8 10. Determine the reheat cycle thermal
2
At 0.0034 MPa: h = 2204.5 V1 = 220.5 li/hr
r0 = 1.75 efficiency?
hf= 109.84 A. 65.5%
Vf= 0.0010032 B. 45.5% 13. Determine the minimum volume of

_ I I [ r~ -I ] c. 54.5% the required day tank.


e- - r:- 1 k(rc -I) 6. Determine the enthalpy
entrance to the boiler in kJ/kg
at the D. 34.5% * A. 1952 liters
B. 5192 liters
4 A. 109.84 n = Wr- Wp = 1352.4-13.89 X 100
C. 29521iters
I [(1.75)1. -1]
e = I- (13.5)1.4-1 1.4(1.75 -1) B. 123.73 * D. 5292 liters *
I QA 3884
c. 118.25
e = 0.60 = 60% D. 136.95 I'll= 34.46% Volume= 220.5 (24) = 5292 liters

3. In problem No. 2, what is the Wpump = he - hs = Vs(Pe- Ps) Problems 11 to 14 14. Find the cost of fuel per day.
compression ratio? A 650 bhp diesel engine uses fuel oil of A. P42,071 *
A. 13.5 * hs -109.84 = 0.0010032{13,850- 3.4) 28°API gravity at 45 deg C. fuel B. P41,271
B.6 he= 123.73 kJ/kg consumption is 0.56 lbs per bhp-hr. · C. P42,710
C. 8 Cost of fuel is P7.95 per liter. D. P40,701
D.4 7. Find the reheat cycle turbine work in
kJ/kg.
REFRESHER MANUAL 2nd Edition by JAS TORDILLO REFRESHER MANUAL 2nd Edition by JAS TORDILLO
33 - 8 I Day 33 - Solution Day 33 - Solution 133 - 9
Power and Industrial Plant Engineering
Cost= P7.95 X 5292 li On = 11,800 Btu/lb =27,450 kJ/kg
= P42,071.4 per day At 200 psia (1.38 Mpa): Volume of fuel tank
m9 (h1 - h4 ) hg = 2789.6 kJ/kg
2

=(2:X~) (~)=30.29 m
Problems 15 to 17 11b = ITirQh At 230 deg F (110 deg C): 3
A water tube boiler evaporated 5.05 kg hf = 461.3 kJ/kg
of water per sec from a feed water 5.05(2726.47- 437.78) 4 3.28 3.28
llb = - - - · - - - - - -
temperature of 104.44 deg C to steam 0.57(27,450) 22. Determine the mass flow rate of fuel
at 1241.1 kPa and quality of 0.97; Mass of fuel in the tank = 30.29 x 855
llb = 0.7386= 73.86% in kg per hr.
weight of coal fuel per sec 0.57 kg; high = 25,904.33 kg
A. 61.5 *
heat value of the coal as fired, 11,800 mass of fuel
18. An evaporator constructed of pipe B. 89.5 No. of days =
Btu/lb.
Steam Properties: or tubing. c. 51 .6 mass flow rate
A. expansion coil * D. 95.8 25,904.33
At 104.44 deg C: hf = 437.78
(interpolated) B. condenser coil
2000 61.4(24)
At 1241. 1 kPa: hf = 805.29 C. evaporative condenser m5 =- - = 907.03 kg/hr
D. cooling tower 2.205
= 17.5days
hfg = 1980.6
(interpolated) On = 41,130 + 139.6(34)
25. At 101.325 kPa, the boiling point of
19. The name given to a procedure by = 45,876.4 kJ/kg water is 1000C. If the pressure is
15. Determine the rate of heat which the efficiency of a combustion m (hs- h)
1
process is determined. Boiler eft = s increased, the boiling temperature will:
absorption in kJ/kg. mrOh ,A. increase *
A. 11,558 • A. heat balance *
B. theoretical balance B. remai ns the same
B. 15,185 0. 75 = 907.03(2789.6 - 461.3)
C. mass balance C. decrease
C. 11 ,855 m 1( 45,876.4) D. lower than
D. 15, 855 D. actual balance
m, = 61.4 kg/hr:
20. A large pipe or drum into which of a 26. In one hour, what is the BTU
h1 = hf + xhfg 23. Determine the density of fuel in kg equivalent of one HP?
hl = 805.29 + 0.97(1980.6) group of boiler is connected. Also used
for a large pipe from which a number of per cu. meter. A. 778
= 2726.47 kJ/kg A. 1000 B. 746
smaller ones are connected in line and
from the side of the large pipe: 8.925 C: 3.516
QA = ms(hl - h~) C. 995 o. 254!l*
OA = 5.05(2726.47- 437.78) A. line pipe
B. line drum D. 855 *
QA = 11,557.88 kJ/sec 27. What is the pressure above zero?
C. header*
D. main pipe II API =14 1.5 - 131.5
A. gage pressure
16. Find the boiler horsepower. B. vacuum pressure
SG
A. 2578 C. absolute pressure *
21. A device having a pre-determined 14 5
B. 2278 34 = 1. -131.5 D. atmospheric pressure
c. 1978 temperature valve for the relief of SG
D. 1178 • pressure. SG = 0.85498
A. exhaust valve 28. Which of the following cycle is for a
B. fusible plug * Density of fuel = SG x density of water compression-ignition engine?
_ m 5 {3600) (h - h )
= 0.85498 X 1000 . -A. diesel cycle •
B 0. Hp - 1 4
--=...:..____:.._~..!...._---'!..:, C. expansion valve
35,322 D. spark plug = 854.98 kg/m3 B. dual cycle
C. otto cycle
Bo. Hp = 5.05S360!!__. C~?~~~I:- 437.78) D. all of these
Problems 22 to 24 24. Calculate the number of days the
35,322
A circular fuel oil tank 45 feet long and supply tank · can hold for continuous
So. Hp = 1177.97 29. The heating value obtained when
5.5 feet diameter is used for oil storage operation.
A. 25 days water in the products of combustion is in
17. What is the boiler efficiency? and supplying fuel oil at 34°API to a
steam boiler. The ~tem flow is 2000 lbs B. 30 days the liquid state:
A. 85% A. higher heating value*
per hour at 200 psia dry steam. C. 17 days •
B. 65% B. lower heating value
Feedwater is 230 deg F and · boiler D. 20 days
c. 74% * efficiency is 75%. C. calorific value •
D 50% D. average heat value
Steam Properties:
REFRESHER MANUAL 2nd Edition by JAS TORDILLO REFRESHER MANUAL 2nd Edition by JAS TOROILLO
33 - 10 1 Day 33 - Solution Day 33 - Solution 1 33 - 11
Power and Industrial Plant Engineering
30. Dry air can be approximated as 34. Determine the average Cp value in
_ _% oxygen and _ _% nitrogen kJ/kg-K of a gas if 522 kJ of heat is c. 160 m = 50.968 m•/sec x 1000 kg/m
by mole numbers. required to raise the temperature from D. 145 m = 50,968 kg/sec
A. 30/70 300K to BOOK making constant
B. 21/79 * pressure. 41 . A reaction turbine develops 500
rk = l+c = 1+0.07 BHP, flow through the turbine is 50
C. 70/30 A. 1.440 c 0.07 = IS .38
D. 79/21 B. 1.038 ft3/sec. Water enters at 20 ftlsec with
c. 1.044 * e = 1_ I = I- I
100 ft pressure head. Elevation of the
· 31. A volume of 450 cc of a1r 1s D. 1.026 wrk-l
k (1s.2sr-' = o.664 water at turbine inlet above the tailwater
measured at a pressure of 740 mm Hg is 10 ft. What is the effective net head?
abs and a temperature of 20° C. What Q = mCp(T2- T1) e= - A. 130ft
is the volume in cq at 760 mm Hg abs 522 = 1Cp(800- 300) QA B. 110ft
and 0°C? . Cp = 1.044 kJ/kg-K c. 116.2 ft *
A. 516.12 0.664 = 300 D. 120.4 ft
B. 620.76 35. The temperature at which its vapor QA
c. 408.25 * pressure is equal to the pressure vz
D. 375.85 exerted on the liquid: QA =W + QR h=P+Z+2Q
A. absolute humidity 451.8 = 300 + QR
P1V1 = P2 V2 B. thermal energy QR = 151.8 kw <2W
C. boiling point* h = 100 + 10 + 2(32.2)
T1 T2 39. Calculate the enthalpy of 3 lbs of
D. calorimeter h = 116.2 ft
740(450) 760V2 fluid that occupy a volume of 20 cu. ft if
---'-=-- 36. Peak load for a period of time the internal energy ·is 1000 Btu per lb 42. One whose pressure is above the
20+273 0+273
v2 = 408.25 cc divided by installed capacity is: and the pressure. is 2 atm abs. saturation pressure corresponding to its
A. capacity factor A. 3109 Btu* ' temperature:
32. A heat engine is operated between B. demand factor B. 3910 Btu A. mixture
temperature limits of 1370 C and 260 C. C. utilization factor* C. 3509 Btu B. saturated liquid
Engine supplied with 14,142 kJ/kw-hr. D. load factor D. 4590 C. saturated vapor
Find the the camot cycle efficiency in
percent.
A. 70.10
37. The thermal efficiency of a particular
of a particular engine operating on an
h = u +pV
h=IOO$) Btu 43.
.
D: compressed liquid *

The latent heat of vaporization


B. 67.56 * ideal cycle is 35%. Calculate the heat in decreases as the pressure and
C. 65.05 kJ supplied to the engine if the engine Btu temperature of the liquid increases, at
D. 69.32 develops 1200 W-hr. +2047) Q44 ~Qft-lb x778tt -lb the critical the heat of vaporization
A. 12,343 *
h =3000 + 108.8
B. 14,218
h = 3108.8 Btu
A. decreases
Carnot Eff = TH- TL C. 10,216 B. become zero •
TH D. 11,108 C. constant
40. A hydraulic turbine receives water
D. increases
Carnot Eff = 1370- 260 w from a reservoir at an elevation of 100
1370 + 273 = 67.56% meters above it. What is the minimum
l'lt = ·- 44. If the load is decreased in a gas
QA water flow in kg per second to produce a
power plant, the exhaust temperature:
33. What should be the temperature of 1.2 ~ _ hrx 3600 sec steady turbine output of 50 MW? .
A . increases *
both the water and steam whenever A. 50,247
they are present together? 0 _35 = sec hr
B. 50,672 B. constant
QA C. decreases
A. saturation temperature for the
QA = 12,342.87 kJ
c. 50,968 * D. unity
existing pressure • D. 50,465
B. superheated temperature
38. An otto engine has a clearance 45. A steam reciprocating pump has a
C. boiling point of water at 101.325 kPa WP= QyH
volume of 7%. 1t produces 300 kw nameplate of 6 in x 4 in x 8 in. What is
D. one hundred degree centigrade 50,000 = 0(9.81)(100)
power. What is the heat rejected in kw? 3 the steam bore equal to:
A. 170 Q = 50.968 m /sec
A. 6 in •
. B. 152*
REFRESHER MANUAL 2nd Edition by JAS TORDILLO REFRESHER MANUAL 2nd Edition by JAS TORDILLO
33- 12 I Day 33- Solution Day 34 - EXAM I 34 - 1
Power and Industrial Plant Engineering
B. 48 in C. 80 sensible, 20 sensible and latent
C. 8 in heat head of Pumps 2 where Hz = H, what is
D. 10 in D. 20 latent, 80 sensible and latent heat DAY 34 - EXAM the total head?
A. H1/H2
46. A certain gas, which flow into an 50. A 4 m x 4 m x 4 m room has a 1. Two pumps are connected in series;'" B. H, + H2
expansion control and without doing any relative humidity of 80%. The pressure if Q, is the discharge of Pump 1 and· Qz C. H2
work. This process is called: in the room is 120 kPa and temperature is the discharge of Pumps 2 where Qz < D. H,- H2
A. condensing is 35°C (Psat = 5.628 kPa). What is the a,, what is the discharge?
B. priming mass of the vapor in the room? A. a, 7. If Q' is the actual discharge flow of
C. throttling ~ (Rv = 0.4615 kPa-m 3/kg-°K) B. Q, + a2 the nozzle and Q is the theoretical
D. knocking A. 2.03 * C. az discharge flow, what will be the
B. 0.80 D. a,-~ coefficient of discharge be equal to?
47. A Pelton wheel is to be designed to c. 1.50 A. a/Q'
run at 300 rpm under an effective head D. 4.80 2. Two pumps are connected in series, B. Q'/Q
of 150 m. The ratio of the nozzle if a, is the discharge of Pump 1 and a2 C. Q'xa
diameter to the diameter of the pitch Pv = RH x Past= 0.80 (5.628) is the discharge of Pumps 2 where <a, D. 1- atQ'
circle is 1/12. Assuming efficiency of = 4.5024 kPa a 2 , what is the discharge?
84%, what is the size of the wheel in A. a, 8. What cycle is used for vapor cycle in
meter. Assume speed ratio of 0.45. PvVv = mvRvTv B. a,+ 02 power plant?
A. 1.05 4.5(4x4x4) = mv(0.461 )(35 + 273) C. a2 A. Brayton cycle
B. 1.55 * mv = 2.028 kg D. a,-Q2 B. Ericson cycle
C. 2.0 C. Diesel cycle
D. 2.86 3. Two pumps are connected in series, D. Rankine cycle
if H, is the head of Pump 1 and Hz is the
Speed ratio, += Peripheral velocity head of Pumps ? where H2 < H, what. is 9. A ton of refrigeration in Btu/24 hrs:
Jet speed the total head? A. 288,000
nON A. H, B. 290,000
• = J29h B. H, + H2 c. 28,800
C. H2 D .. 29,000
r/300) D. H,- H2
0.45 = "1. 60 1o". The relative humidity becomes
)2(9.81)150 4. Two pumps are connected in parallel, 100% and where the water vapor starts
D = 1.55m if a, is the discharge of Pump 1 and a2 to condense.
is the discharge of Pumps 2 where <az A. dew point
48. Cooling air in intercoolers between a,, what is the discharge? B. dry bulb
compression stages in a multi-stage air A. a, C. wet bulb
compressor. B. a,+ a2 D. saturated
A . keeps the moisture in the air from C. a2
evaporatin_g • D. a,-az 11. To obtain equilibrium in the
B. allows slightly lower volume of free condenser, what should be the range of
air to be handled for a constant speed 5. Two pumps are connected in the tower, in relation to the temperature
compression parallel, if a,
is the discharge of Pump 1 difference in the condenser?
C. decreases substantially the a
and 2 is the discharge of Pumps 2 A. should be higher
volumetric efficiency where a2 >a,, what is the discharge? B. should be lesser
D. requires less HP for the compressor A. Q, C . Indirectly proportional
that without intercoolers B. Q, + a2 D. it should be equal
C. a2
49. The sensible heat ratio is 0.8, which D. a,- a2 12. Room air-conditioning needs
.means: abundant supply of air, because it uses
A. 20 latent, 90 sensible * 6. Two pumps is connected in parallel, what type of condenser?
B. 20 sensible, 20 latent if H, is the head of Pump 1 and Hz is the A . chilled water system

REFRESHER MANUAL 2nd Edition by JAS TORDILLO REFRESHER MANUAL 2nd Edition by JAS TORDILLO
34 - 2 I Day 34 - EXAM Day 34 - EXAM I 34 - 3
Power and Industrial Plant Engineering
B. shell and tube C. irreversible adiabatic process
C. water cooled D. constant entropy process B. is decreased if pre-cooler is used Problems 28 to 30
D. air cooled C. does not depend on the refrigerant An air conditioning plant with a capacity
17. The effectiveness of a body as a used of 400 kw of refrigeration has an
;;..
13. Viscosity is a measure of : thermal radiator at given temperature: D. equals the increase in volume evaporating temperature and
A volume A. reflectivity condensing temperature of 3 deg C and
B. thickness B. emmisivity 23. An R-12 refrigeration system has a 37 deg C, respectively. The plant is
C. temperature C. conductivity refrigerating load of 50 tons of using Refrigerant 12.
D. pressure D. absorptivity refrigeration. The following enthalpies in
kJ!kg have seen found at condenser R-12 Properties:
14. Heat transfer processes which 18. A centrifugal pump has a discharge entrance = 372, exit = 239; at At 3 deg C: hf = 202.78 kJ/kg
include a change of phase of a fluid are of 0.1 m 3/s at a head of 80 meters. If the evaporator entrance = 239, exit = 353. hg =352.755 kJ/kg
considered. speed of the pump is increased to attain Find the compressor power in kw. vg = 50.47 Ukg
A convection a discharge of 0.12m3/s, what new kw A. 13.9
B. conduction power will bw required? B. 23.9 At 37 def C: hf = 235.503 kJ/kg
C. radiation A. 136 kw C. 19.3
D. heat energy B. 163 kw D. 29.3 28. What will be the mass of flash gas
C. 113 kw per kg of refrigerant circulated?
15. What do you call the measurement D. 79 kw 24. Instrument used to measure the A. 0.119
of the amount of water vapor present in quality of steam: B. 0.728
air? 19. A centrifugal pump delivers 80 liters A. bomb calorimeter c. 0.218
A. present saturat1on per second of water on test. Suction D. 0.782
B. hygrometer
B. relative humidity gauge reads 10 mm Hg vacuum and 1.2 C. throttling calorimeter
C. degree saturation meters below pump centerline. Power 29. What is the mass of Refrigerant 12
D. gas calorimeter
D. specific humidity input is 70 kw. Find the total dynamic must be circulated per second?
head in meters. Assume pump 25. Water enters a cooling tower at 35 A. 3.411 kg
16. Air at 29 C and 23.5 C wb enters a efficiency equal to 7 4%. °C at the rate of 180,000 kg/hr and is B. 4.411 kg
cooling tower at a rate of 102 kg/min. It A. 66 cooled to 25 °C. Atmospheric air used c. 2.411 kg
leaves the cooling tower at 38 C db and B. 62 for cooling enters tower with enthalpy of Q. 5.411 kg
humidity ratio of 0.0436 kg moisture per C. 60 24kJ!kg and leaves with enthalpy of 88
kg dry air. Hot water enters the tower at D. 64 kJ/kg. Find the mass flow rate of air in 30. What is the volume flow at the
46.5 C and a flow rate of 142.2 kg/min. kg/hr. suction in cu. m per second?
Determine the cooling tower efficiency in 20. Where is lithium bromide used in A. 172
A. 11,776
percent. refrigeration system? B. 0.712
B. 117,759
Air Properties: A. condensate return lines C. 218
C. 17,759
At 29 C db and 23.5 C wb, B. centrifugal compressors D. 171,759 D. 0.172
h = 70.02 kJ/kg C. absorbers
w = O.OH kg moisture/kg dry air D. ion exchangers 26. Air receivers in a compressed air 31. What is the term used to express
At 38 C dQ and w = 0.0436 kg/kg plant must be: the ratio of specific humidities: ·
h = 149.24 kJ/kg 21. Refers to the temperature at which A. relative humidity
A. without pressure gauges
A. 56% all molecular motion ceases according B. degree saturation
B. rectangular in shape
B. 60% to the kinetic theory of heat: C. absolute humidity
C. vented to the atmosphere
c. 43% A. 0°C D. installed in a safety valve and drain D. percent saturation
D. 48% B. critical point
valve
c. 273 °C 32. A 48 ff wall has an average thermal
0
16. Throttling of the refrigerant through D. absolute zero 27. One Newton-meter per second is conductivity of 1.2 Btu-ft/W- f-hr. If the
the expansion valve in a vapor an Sl derivation unit known as: temperature difference between two
compression refrigeration cycle is: 22. In refrigeration system, the heat sides is 18°F and thickness is 0.7 ft,
A . pascal
A reversible adiabatic process absorbed in the evaporator per kg mass what is the total heat loss in 5 hours?
B. watt
B. isometric process of refrig~rant passmg through is: A. 1480 Btu
C. joule
A equals the increase in enthalpy B. 7400 Btu
D. kilojoule

REFRESHER MANUAL 2nd Edition by JAS TORDILLO REFRESHER MANUAL 2nd Edition by JAS TORDILLO
- - - . - - - - - - -- - -- - - -- --

34-41 Day34-EXAM Day 34 · EXAM I 34 - 5


C. 5180 Btu energy leaving is 400 kJ/kg, determine Power and Industrial Plant Engineering
D. 38,500 Btu the velocity at the exit? 44. A heat engine operates on the A. decreases
A. 40 m/s Carnot cycle. It produces 50 kw of
33. The intake pipe to a hydraulic B. becomes zero
B. 777.7 m/s power while operating between ·:
turbine from a dam is: C. increases
C. 77m/s temperature limits of 800 deg C and 10cf
A tailrace D. constant
D. 888.8 m/s deg C. Determine the amount of heat
B. penstock
added to the engine. 49. The ratio of the actual velocity of a
C. surge tank 39. One horsepower is equivalent to: A . 50 kw
D. scroll case fluid to the local speed of sound in the
A. 746 kw B. 66.6 kw fluid is:
B. 550 ft-lb/min C. 60 kw A. Reynolds number
34. That reserve generating capacity is C. 33,000 ft-lb/sec D. 76.6 kw B. Mach number
connected to the bus and ready to take D. 2545 Btu/hr
C. Telephone number
alload.
45. A heat engine operating on the D. Acoustic number
A. hot reserve 40. Find the air fuel ratio, in kg air per Carnot cycle uses solar energy as the
B. system reserve kg fuel, for a combustion process to source of high-temperature heat input. 50. A device used to convert thermal
C. spinning reserve which the fuel is CsH2o with 20% excess The solar irradiation, averaged over the energy into kinetic energy and also used
D. cold reserve air. day, has a value of 0.51kw/m2. This to direct the mass flow to its specified
A. 15.7 provides energy to the cycle at a direction by means of a variable duct
35. How much heat is needed to B. 19.7 uniform temperature of 450 deg K, and area.
completely vaporize 100 kg of ice at - c. 18.5 the cycle rejects heat to the environment A throttling valve
10°C if the pressure held constant at D. 20.5
at a temperature of 300. deg K. The B. nozzle
200 kPa. (latent heat of fusion at 200
engine produces 2000 kw of power. C. diffuser
kPa = 320 kJ/kg. Latent heat of 41. Percentage air is the Determine the minimum area in square D. header
vaporization at 200 kPa = 2202 kJ/kg). ratio of the difference between air meter needed to provide this power.
A 296.1 MJ actually supplied and the theoretically A 11765
B. 340.5 MJ supplied air divided by theoretical air B. 9975
C. 320.5 MJ supplied. (,f )
D. 345.0 MJ
c. 10325
A excess air D. 7985 DAY 34- ANSWER KEY
B. actual air
1.C 16. C 31 . 846 . 0
C. theoretical air 46. At 101.325 kPa, the boiling point of 2.A 17.8 32.847.C
36 During sensible heating, the D. atmospheric air ,3. B 18.A 33. 848.A
water is 100 deg C. If the pressure is
absolute humidity remains constant but 4. B 19. A 34. C49. B
increased, the corresponding boiling 5. B 20. C 35. A 50. B
the relative humidity: 42. In the Rankine cycle, an engineer temperature will: 6. C 21. 0 36. A
A decreases wants to increase the efficiency of the A. remains the same 7. B 22. A 37. 0
B. remains the same boiler by reducing the moisture in the 8. 0 23. 0 38. B
B. decrease
C. increases final stage of the turbine. What will the C. lesser 9. A 24. C 39. 0
D. zero engineer do? 10.A25.B 40.C
D. increase 11. 0 26. 0 41. A
A. reheat the steam
12. 027. B 42. A
37. In thermodynamic system, a certain B. increase the back pressure 47. An ideal gas is flowing into an 13.B28.C 43.0
process represented by a product of C. replace the condenser 14. A 29. A 44. 0
expansion pipe or coil and without doing
pressure and volume raised to the D. decrease the back pressure any work. The process is called:
15. 030.0 45.A
exponent n is constant. If n is zero,
what is the process?
A. condensing ~
43. The available energy of a turbine is B. vaporizing
A. isentropic 1450 kJ/kg, efficiency of the engine is C. throttling
B. adiabatic · 70% and the power output at full load is D. isentropic
C. polytropic 5.5 MW. What is the engine flow rate at
D. isobaric full load in kg/kw-hr? 48. At the critical point the heat of
A 2.5 vaporization is almost zero. The latent
38. In a steady flow system, energy B. 3.0 heat of vaporization increases as the
entering is 700 kJ/kg and 70 m/s. If the C. 2.75 pressure and saturation temperature of
D. 3.5 the liquid ____.

REFRESHER MANUAL 2nd Edition by JAS TORDILLO REFRESHER MANUAL 2nd Edition by JAS TORDILLO
34 - 6 1 Day 34 - Solution Day 34 - Solution I 34 - 7
A. H,/Hz
Power and Industrial Plant Engineering
DAY 34 - SOLUTION B. H1 + H2 12. Room air-conditioning needs m,(h2- h,) =mwCPw(ta- tb)
C. H2 * abundant supply of air, because it uses 102( 149.24- 70.02)
1. Two pumps are connected in series, D. H,- H2
if Q, is the discharge of Pump 1 and 0 2 what type of condenser? = 42.2(4.187)(46.5- tb)
is the discharge of Pumps 2 where Q 2 < A. chilled water system tb =32.93 °C
7. If Q' is the actual discharge flow of . B. shell and tube
Q,, what is the discharge? ta-tb
A. Q,
the nozzle and Q is the theoretical c. water cooled Cooling tower eft, 11 = ta- twb
discharge flow, what will be the D. air cooled •
B. Q, + 02 coefficient of discharge be equal to?
c. a2* 46.5 - 32.9 =0.591=59.1%
A. 0/0' 13. Viscosity is a measure of: Tl = 46.5- 23.5
D. o,-o2 B. Q'/Q * A. volume
C. Q'xQ B. thickness* 16. Throttling of the refrigerant through
2. Two pumps are connected in series, D. 1-Q/Q'
if Q 1 is the discharge of Pump 1 and 02 C. temperature the expansion valve in a vapor
D. pressure
is the discharge of Pumps 2 where o, < 8. What cycle is used for vapor cycle in
compression refrigeration cycle is:
A. reversible adiabatic process
Qz, what is the discharge? power plant?
A. a,· A. Brayton cycle
14. Heat transfer processes which
include a change of phase of a fluid are
B. isometric process
C. irreversible adiabatic process "
B. Q, + 02 B. Ericson cycle
C. 02 considered. D. constant entropy process
C. Diesel cycle A. convection *
D. Q,-Q2 D. Rankine cycle • B. conduction 17. The effectiveness of a body as a
3. Two pumps are connected in series, C. radiation thermal radiator at given temperature:
9. A ton of refrigeration in Btu/24 hrs: D. heat energy A. reflectivity
if H, is the head of Pump 1 and H2 is the A. 288,000 *
head of Pumps 2 where Hz < H, what is B. emmisivity *
B. 290,000 15. What do you call the measurement C. conductivity
the total head?
A. H,
C. 28,800
D. 29,000
of the amount of water vapor present in D. absorptivity
B. H, + H2 * air?
C. H2 1 TOR= 12,000 Btu x 24 A. present saturation 18. A centrifugal. pump has a discharge
D. H,- H2 hr B. relative humidity of 0.1 m3/s at a head of 80 meters. If the
C. degree saturation speed of the pump is increased to attain
4. Two pumps are connected in parallel,
I TOR = 288,000 ~ D. specific humidity * a discharge of 0.12m 3/s, what new kw
24 hrs
o,
if is the discharge of Pump 1 and Oz
16. Air at 29 C and 23.5 C wb enters a
power w ill bw required?
A. 136 kw *
is the discharge of Pumps 2 where 02 < 10. The relative humidity becomes cooling tower at a rate of 102 kg/min. It B. 163 kw
Q,' what is the discharge? 100% and where the water vapor starts
A. Q, leaves the coolil)g tower at 38 C db and C. 113 kw
to condense. humidity ratio of 0.0436 kg moisture per D. 79 kw
B. a,+ a2 * A. dew point *
C. 02 kg dry air. Hot water enters the tower at
B. dry bulb
D. Q,- 02 C. wet bulb
46.5 C and a flow rate of 142.2 kg/min. P, =OwH, = 0.1(9.81)(80) =78.48 kw
D. saturated
Determine the cooling tower efficiency in 9_,_ = ~~
5. Two pumps are connected in percent. 02 Nz
o,
parallel, if is the discharge of Pump 1 11. To obtain equilibrium in the
Air Properties:
At 29 C db and 23.5 C wb, 0.1 = N!
and 02 is the discharge of Pumps 2 condenser, what should be the range of 0.12 N2
where 0 2 > O,, what is the discharge? h = 70.02 kJ/kg
the tower, in relation to the temperature w =0.017 kg moisture/kg dry air
A. o,
B. a,+ a2•
C. 02
difference in the condenser?
A . should be higher
At 38 C db and w = 0.0436 kg/kg
h = 149.24 kJ/kg
~~ (z~r
=
B. should be lesser A. 56% 3
D. Q,-Oz C. indirectly proportional B. 60% *
2_8.48 = (_Q:!._)
D. it should be equal * pl 0.12
6. Two pumps is connected in parall.el, c. 43%
Pz = 135.6 kw
if H, is the head of Pump 1 and H2 is the D. 48%
head of Pumps 2 where H2 = H 1 what is 19. A centrifugal.pump delivers 80 liters
the total head? per second of water on test. Suction
REFRESHER MANUAL 2nd Edition by JAS TORDILLO REFRESHER MANUAL 2nd Edition by JAS TORDILLO
34 - 8 1 Day 34- Solution Day 34 - Solution I 34 - 9
Power and lndustriat Plant Engineering
gauge reads 10 mm Hg vacuum and 1.2 C. 19.3
meters below pump centerline. Power D. 29.3 * Problems 28 to 30 31. What "is the term used to express
input is 70 kw. Find the total dynamic An air conditioning plant with a capacity the ratio of speci~c humidities:
head in meters. Assume pump QA = m(h, -h.) of 400 kw of refrigeration has ao A. relative humidity
efficiency equal to 74%. 50(3.516) = m(353 - 239) evaporating temperature and B. degree saturation *
A. 66* m = 1.542 kg/sec condensing temperature of 3 deg C and C. absolute humidity
B. 62 37 deg C, respectively. The plant is D. percent saturation
C. 60 We= m(h2- h,) using Refrigerant 12.
D. 64 =
We 1.542(372 - 353) 32. A 48 ft2 wall has an average thermal
0
conductivity of 1.2 Btu-ftlft2- f-hr. If the
We= 29.3 kw R-12 Properties:
At 3 deg C: hf = 202.78 kJ/kg temperature difference between two
BP = QyH hg =352.755 kJ/kg sides is 18°F and thickness is 0.7 ft,
24. Instrument used to measure the
llp quality of steam: vg =50.47 Llkg what is the total heat loss in 5 hours?
70 = 0.08(9.8l)H A. bomb calorim.e ter A. 1480 Btu
B. 7400 Btu*
B. hygrometer At 37 def C: hf = 235.503 kJ/kg
0.74
C. throttling calorimeter * C. 5180 Btu
H = 66 m
D. gas calorimeter 28. What will be the mass of flash gas D. 38,500 Btu
20. Where is lithium bromide used in
per kg of refrigerant circulated? Q = kA.6.T = 1.2(48) ( 18)
refrigeration system? 25. Water enters a cooling tower at 35 A. 0.119 X 0.7
A. condensate return lines °C at the rate of 180,000 kg/hr and is B. 0.728
cooled to 25 °C. Atmospheric air used c. 0.218 * 0=1481.\4 Btu/hr
B. centrifugal compressors
C. absorbers • for cooling enters tower with enthalpy of D. 0.782
24kJ/kg and leaves with enthalpy of 88 In 5 hours:
D. ion exchangers Q = 1481 (5) = 7405.7 Btu
kJ/kg. Find the mass flow rate of air in h3 = h. = hf + xhfg
kg/hr. . h3 = h. = hf + x(hg -hf)
21 . Refers to the temperature at which 33. The intake pipe to a hydraulic
A. 11,776 235.503 = 202.78 + x(352.755 - 202.78)
all molecular motion ceases according turbine from a dam is:
to the kinetic theory of heat: B. 117,759 * x=3.411kg
C. 17,759 A. tailrace
A. 0°C ~. penstock *
D. 171,759 29. What is the mass of Refrigerant 12
B. critical point C. surge tank
must be circulated per second?
c. 273 °c
A. 3.411 kg* D. scroll case
D. absolute zero * mwCPw.6.tw = ma(h2- h,)
180,000(4.187)(35 -25) = m.(88- 24) B. 4.411 kg
ma = 117,759.3 kg/hr c. 2.411 kg 34. That reserve generating capacity is
22. • In refrigeration system, the heat connected to the bus and ready to take
D. 5.411kg
absorbed in the evaporator per kg mass alload.
of refrigerant passing through is: 26. Air receivers in a compressed air
plant must be: QA = m(h, - h4) A. hot reserve
A. equals the increase in enthalpy* B. system reserve
A. without pressure gauges 400 = m(352.755- 235.503).
B. is decreased if pre-cooler is used C. spinning reserve *
B. rectangular in shape m = 3.411 kg
C. does not depend on the refrigerant D. cold reserve
used C. vented to the atmosphere
D. installed in a safety valve and 30. What is the volume flow at the
D. equals the increase in volume 35. How much heat is needed to
drain valve * suction in cu. m per second?
A. 172 completely vaporize 100 kg of ice at -
23. An R-12 refrigeration system has a 10°C if the pressure held constant at
27. One Newton-meter per second is B. 0.712
refrigerating load of 50 tons of 200 kPa. (Latent heat of fusion at 200
an Sl derivation unit known as: C. 218
refrigeration. The following enthalpies in kPa = 320 kJ/kg. Latent heat of
A. pascal D. 0.172 *
kJ/kg have seen found at condenser vaporization at 200 kPa = 2202 kJ/kg).
B. watt*
entrance = 372, exit = 239; at
C. joule v,· = mv, A. 296.1 MJ *
evaporator entrance = 239, exit = 353.
Find the compressor power in kw. D. kilojoule =3.411(0.05047)
3
B. 340.5 MJ
= 0.172 m /sec C. 320.5 MJ
A. 13.9 D. 345.0 MJ
B. 23.9

REFRESHER MANUAL 2nd Edition by JAS TORDILLO REFRESHER MANUAL 2nd Edition by JAS TORDILLO
34- 10 1 Day 34- Solution Day 34 - Solution I 34 - 11
Heat to vaporize Ice: c. 18.5 . Power and ln'dustrial Plant Engineering
D. 20.5
44. A heat engine operates on the 46. At 101.325 kPa, the boiling point of
Q = m[Cp1(tr- t1) + hL + Cp2(t2- !t) +hv)
Carnot cycle. It produces 50 kw of water is 100 deg C. If the pressure is
= 100(2.093(0- - 10) + 320 + Fuel +Air = Products of combustion
4.187(100- 0) +2202] power wh ile operating between,.' increased, the corresponding boiling
CsH2o + x~ + x(3. 76)N2 temperature limits of 800 deg C and 100 temperature wi ll:
= =
296,163 kJ 296.1 MJ -+yC02 + zH20 +x(3.73)N2 deg C. Determine the amount of heat A. remains the same
CaH2o + 1302 + 13(3.76)N2 added to the engine. B. decrease
36. During sensible heating, the -+8C02 +10H20 + 13(3.76)N2 A. 50 kw C. lesser
absolute humidity remains constant but
B. 66.6 kw D. increase •
the relative humidity: ~Jr._ = !.~_:+:1_3(3.76) = 6 1. 88 moles of air C. 60 kw
A. decreases *
fuel I mole fuel D. 76.6 kw * 47. An ideal gas is flowing into an
B. remains the same
C. increases air_ = ~~~ 8 (1. 20l 28 · 97 = 18.54 kg air/kg fuel expansion pipe or coil and without doing
fuel 8(12) + 20(2) • TH - TL = WN any work. The process is called:
D. zero
A. condensing
TH QA
B. vaporizing
37. In thermodynamic system, a certain 4 1. Percentage air is the
ratio of the difference between air 800-100 50 C. throttling *
process represented by a product of
actually supplied and the theoretically 800+ 273 QA D. isentropic
pressure and volume raised to the
exponent n is constant. If n is zero, supplied air divided by theoretical air QA = 76.6 kw
supplied. 48. · At the critical point the heat of
what is the process?
A. excess air • 45. A heat engine operating on the vaporization is almost zero. The latent
A. isentropic
B: actual air Carnot cycle uses solar energy as the heat of vaporization increases as the
B. adiabatic
C. theoretical air pressure and saturation temperature of
c: polytropic D. atmospheric air
source of high-temperature heat input.
the liquid ____.
D. isobaric • The solar irradiation, averaged over ·the
day, has a value of 0.51kw/m 2. This A. decreases *
42. In the Rankine cycle, an engineer provides energy· to the cycle at a B. becomes zero
38. In a steady flow system, energy
wants to increase the efficiency of the uniform temperature of 450 deg K, and C. increases
entering is 700 kJ/kg and 70 m/s. If the
boiler by reducing the moisture in the the cycle rejects heat to the environment D. constant
energy leaving is 400 kJ/kg, determine
final stage of the turbine. What will the at a temperature of 300 deg K. The
the velocity at the exit?
engineer do? engine produces 2000 kw of power. 49. The ratio of the actual velocity of a
A. 40 m/s
B. 777.7 m/s * A. reheat the steam * Determine the minimum area in square .fluid to the local speed of sound in the
B. increase the back pressure meter needed to provide this power. fluid is: '
C. 77mls
C. replace the condenser A. 11765 * A. Reynolds number
D. 888.8 m/s
D. decrease the back pressure B. Mach number*
B. 9975
C. 10325 C. Telephone number
h1 + KE = h2 + KE2 43. The available energy of a turbine ·is D. Acoustic number
(70)2 V2 D. 7985
700+ - :: 400+ ... t 1450 kJ/kg, efficiency of the engine is
2(1 000) 2(1 000) 70% and the power output at full load is 50. A device used to convert thermal
TH - TL - WN
v2 = 777.75 m/s 5.5 MW. What is the engine flow rate at
TH QA
energy into kinetic energy and also used
full load in kg/kw-hr? to direct the mass flow to its specified
39. One horsepower is equivalent to: A. 2.5 450 - 300 2000 direction by means of a variable duct
A. 746 kw B. 3.0 450 QA area.
B. 550 ft-lb/min c. 2.75 QA = 6000 kw A. throttling valve
C. 33,000 ft-lb/sec D. 3.5 * B. nozzle*
D. 2545 Btu/hr * C. diffuser
Wr = mr (h1 - h2) x nr QA = 0.5 1 D. header
Area
40. Find the air fuel ratio, in kg air per 5,500 = mt{1450) x 0.70
kg fuel, for a combustion process to mr = 5.418 kg/sec= 19,507.4 kg/kr 6000 = 0.5 l
which the fuel is CaH2o with 20% excess mr 19,507.4 Area
air. ~ = --·-· =3.45 kg/kw-hr Area = 11,764.7 m2
BP 5,500
A. 15.7
B. 19.7
REFRESHER MANUAL 2nd Edition by JAS TORDILLO REFRESHER MANUAL 2 nd Edition by JAS TORDilLO
Day 35 - EXAM I 35 - 1
Power and Industrial Plant Engineering
4. A diesel electric plant supplies energy
DAY 35- EXAM for MECO. During a 24-hr period, the
plant consumed 300 gallons of fuel at
1. A fuel gas has the following 28°C and produced 4900 kw-hr.
volumetric analysis: Industrial fuel used is 32°API and was
CH4 = 68% C2Hs = 32% purchased at P32.0 per liter at 15.6°C.
Assume complete combustion with 30% What should the cost of fuel to be
excess air at 101.325 kPa, 21°C wet produce one kw-hr?
bulb and 27°C dry bulb. What is the A. P7.05
partial pressure of the water vapor in B. P7.69
kPa? C. P6.53
A. 8.62 D. P6.00
B. 19.28
c. 14.24 5·. A certain gas at 101.325 kPa and
3
D. 16.94 21°C whose volume is 5.0 m are
compressed into a storage vessel of
2. Air enters the compressor of a gas 0.85 m3 capacity. Before admission, the
turbine at 110 kPa and 27°C with a storage vessel contained the gas at a
3
volume flow rate of 8.5 m /sec. The pressure and temperature ·of 120 kPa
compressor pressure ratio is 10 and its and 28°C; after admission the pressure
isentropic efficiency is 82%. At the inlet has increased to 1350 kPa. What
to the turbine, the pressure is 950 kPa should be the final temperature of the
and the temperature is 1400°K. The gas in the vessel in Kelvin?
turbine has an isentropic efficiency of A. 498.0
85% and the exit pressure is 100 kPa. B. 556.6
On the basis of an air standard analysis, c. 419.8
what is the thermal efficiency of the D. 620.0
cycle in percent?
A. 22.0 6. A Francis turbine is installed with a
B. 21.8 vertical draft tube. The total head to the
C. 20.2 center of the spiral casing at the inlet is
D. 27.3 60 m and velocity of water at the inlet is
3
7.5 mts. The discharge is 1.5 m /s. The
3. Steam enters a throttling calorimeter hydraulic efficiency is 0.87 and overall
at a pressure of 1.03 Mpa. The efficiency is 0.84. The vertical distance
calorimeter downstream pressure and from the centerline to the top of the draft
temperature are respectively, 0.100 Mpa tube is 1.5 m while the tailrace water
and 125°C. What is the percentage level is 2.5 m from the top of the draft
moisture of the supply steam? tube. What is the total head on the
Properties of Steam: turbine in meters?
P, Mpa hf hfg . A. 34.72
1.03 2010.7 2779.25 B. 55.20
c. 66.86
Note: At 0.100 Mpa and 125°C, D.' 48.12
h = 2726.6 kJ/kg
7. Given a horizontal conveyor, sa· m
A. 2.62 centers, 200 lbs per hour capacity and
.B. 3.15 handling coal at 0.75 m/s with 750 kg
c. 5.21 per m3 . Other data as follows:
D. 1.98.

REFRESHER MANUAL 2nd Edition by JAS TORDILLO


35 - 2 I , Day 35 - EXAM Day 35 ·- EXAM I 35 - 3
Power and Industrial Plant Engineering
Flight widtlol and depth .................. .. . 11. Steam at 2.6 MPa and 600°C (h =
............ .600 mm x 220 mm 3478.5 kJ/kg) enters a nozzle A. 150.25°C C. 20.2 em
Quantity of material. .. .. ...... .. .. .. ... ... .. undergoes reversible adiabatic process B. 523.25°C D. 32.4 em
........... 0.145 m3/m with velocity of 400 m/s. Find the C. 250.25°C
Coefficient of friction elements ... ... ....... . stagnation enthalpy. D. 750.25°C 22. Heat normally flowing from a high
......... 0.12 A. 2558.5 kJ/kg temperature body to a low temperature
Material coefficient of friction ..... .. ..... .. . B. 4558.5 kJ/kg 17. A gas initially at P1 = 600 kPa and body wherein it is impossible to convert
""" ...... 0.60 C. 3558.5 kJ/kg v, = 150 li, undergoes a process to that heat without other effects is called the:
Assume an engineering type chain with D. 5558.5 kJ/kg changes the pressure to 150 kPa and A . second law of thermodynamics
sleeve bearing rollers weighing with volume to 240 li during wh'i ch the B. first law of thermodynamics
flights, 95.3 kg/m. Calculate the chain 12. There are 100 kg/min of water enthalpy decreases 80 kJ. The specific C. third law of thermodynamics
pull in kg. enters a heating element at 5°C and heats are constant, assuming Cv to be D. zeroth law of thermodynamics
A. 3180.33 leaves at 40°C. Find the heat added 3.16 kJ/kg-°K. Determine work done
B 4567.50 during the process. during the process. 23. A two-stage compressor with an
3
C. 3550.10 A. 244.25 kw A. 54 kJ ideal intercooler receives 0.4 m /s of air
D. 4166.40 B. 344.25 kw B. -54 kJ at 100 kPa and 31 0°K. If the intercooler
C. 295.25 kw c. 80 kJ pressure is 707.1 kPa, determine the
8. In problem No. 7, What is the power D. 775.9 kw D. - 80 kJ discharge pressure in kPa.
in kw required to drive the conveyor? A. 5000 kPa
Let F11 = 76.042 kg-m/s-HP. 13. Water has an absolute temperature 18. From Problem No. 17, determine B. 3000 kPa
A. 20.20 kw of 10.5 m of water and atmospheric the change in internal energy. C. 2500 kPa
B. 33.6 kw pressure of 90 kPa. Find the gage A. -26 kJ D. 266 kPa
C. 30:50 kw pressure is meters of water. B. 26 kJ
D. 45.0 kw A. 9.1 m of water c. 50 kJ 24. Calculate the logarithmic mean
B. 3.32 m of water D. -50 kJ temperature difference for a condenser
9. A tank having a capacity of 1 cu. m is C. 5.5 m of water if the temperature difference between
filled with air to a pressure of 500 kPag D. 1.32 m of water 19. From Problem No. 17, determine condensing steam and water inleJ is
when the temperature is 60 deg C. It the specific heat constant of the gas. 650°C and that steam inlet and water is
was found out later that because of the 14. A Camot engine receives 100 Btu A. 3.076 120°C.
leak the pressure drops to 350 kPag of heat from a hot reservoir at 600°F and B. 3.125 A. 325.7°C
and the temperature has decreased to rejects ~0 Btu of heat. Calculate the c. 2.756 B. 313.7°C
45 deg C. Determine the amount of air temperature of the cold reservoir in deg D. 2.108 C. 225.7°C
that has leaked out in kg. F. D. 155.7°C
A. 1.34 kg A. 636°F 20. ' From Problem No. 17, determine
B. 3.34 kg B. 176°F the gas constant. R. 25. A pump discharges 11 0 liters per
C. 2.34 kg C. 455°F A. 3.16 kJ/kg-deg K second of water to a height of 45
D. 4.34 kg D. 100°F B. 6.56 kJ/kg-deg K meters. If the pump 'efficiency is 70%
C. 4.25 kJ/kg-deg K and the speed of the pump is 1200 rpm ,
10. A vulcanizing shop is provided with 15. An ideal gas at 50 psig and 90°F is D. 7.16 kJ/kg-deg K what is the power input in the drive
a 10 m3 of compressed air tank. During heated in a closed container to 140°F. shaft?
operation, air pressure in . the tank What is the final pressure in psia? 21. One side of a furnace wall (k = 1. 5 A. 34 kw
reduces from 700 kPa to 300 kPa while A. 70.6 psia W/m-K) is exposed to ambient air whose B. 79.4 kw
the temperature remains constant at 32 B. 60.7 psia temperature is 29°C (h = 18 W/m -K)
2
C. 44 kw
deg C. Determine the mass of air that C . .67.6 psia and the other side is exposed to D. 69.4 kw
has been used. D. 14.7 psia combustion gases whose temperature is
A. 15.7 kg 1500°C (h = 12 W/m 2-K). Given a heat 26. A fuel pump is delivering 18 gallons
B. 35.7 kg 16. A Carnot engine requires 35 kJ/sec transfer rate of 6000 W per square per minute of oil with a specific gravity of
c. 25.7 kg from a hot source. The engine meter area, determine thickness of the 0.80. The total head is 75 meters. If the
D. 45.7 kg produces 15 kw of power and the
wall. power input to the pump is 1.1 hp, what
temperature of the sink is 26°C. What is is the efficiency of the pump?
A. 15.9 em
the temperature of the hot source in °C? A. 69.3%
B. 24.2 em

REFRESHER MANUAL 2nd Edition by JAS TORDILLO REFRESHER MANUAL 2nd Edition by JAS TORDILLO
35-41 Day 35·- EXAM Day 35 - EXAM I 35 - 5
Power and Industrial Plant Engineering
B. 75%> A. expansion valve
C. 72.1% B. condenser then the maximum allowable working 45. Which property of fluids is of
D. 81.4% C. compressor pressure in MPa is: fundamental importance in the study of
D. cooling coil A. t TS.E hydraulics?
27. Heat exchanger used to provide R.FS A. unit weight
heat transfer between the exhaust 34. The state of any vapor whose B. viscosity
gases and the air prior to its entrance to pressure exceeds that at which B.~ C. mass density
R.FS.E
the combustor: condensation occurs at the prevailing D. all of the above
A. evaporator temperature. C. TS.t.E
B. regenerator A. Saturation R.FS 46. What is the electric current flowing
C. combustion chamber B. Supersaturation in one direction and which reverses and
D. R.TS.t.E
D. heater C. Critical FS
flows in opposite direction and usually in
D. Mixture a steady state?
28. A device for measuring the velocity 40. In PMC, what is the color of the A. alternating current
of wind: 35. What amount of air is required in a pipe for Carbon Dioxide? · B. direct current
A. aneroid barometer low bypass factor? A. Orange C. load current
B. anemoscope A. greater B. Green D. both alternating and direct current
C. anemometer B. indeterminate C. Red
D. anemograph C. lesser D. Yellow 47. Where is the location of servomotor?
D. does not change A. Near the shaft
29. Air standard efficiency of a diesel 41 . Servomotor controls B. Outside the shaft
engine depends on: 36. What is the clockwork-operated A. Moving vanes C. At the fulcrum
A. speed device which records continuously the B. Moving and guide vanes D. Inside the shaft
B. fuel . humidity of the atmosphere? C. Guide vanes
C. compressi6n ratio A. hetrograph D. None of these 48. Excess water from servomotor is
D. torque B. hydrodeik expelled by:
C. hygrometer 42. Difference in pressure measure A. Siphon
30. Heavy water is: D. hygrograph above or below atmospheric pressure: B. Nozzle
A. B20 {2 is written as subscript) A. draft C. Governor
B. W20 {2 is written as subscript) 37. Type of flow in which all conditions B. stack D. Draft tube
C. H20 (2 is written as subscript) at any' point in a stream remain constant C. chimney
D. D20 {2 is written as subscript) • with respect to time. D. breeching 49. Peak load for a period of time
A. Steady flow divided by installed capacity is:
31. Mechanism designed to lower the B. Uniform flow 431 The kinetic energy of a moving fluid A. capacity factor
temperature of air passing through it C. Unsteady flow is used to isentropically compressed the B. utilization factor
A. air cooler D. Nonuniform flow fluid to state of zero velocity. The C. demand factor
B. air spillover temperature of a moving fluid at the D. load factor
C. air defense 38. Water is flowing along a constant state of zero velocity is called:
D. air cycle cross section along the length of the A. stagnation temperature 50. The ratio of the sum of individual
channel, we say the flow is: B. partial temperature - maximum demands of the system to the
32. The viscosity of most commercially A. Steady flow C. critical temperature overall maximum demand of the whole
available petroleum lubricating oil B. Unsteady flow D. absolute temperature system is:
changes rapidly above: c. Steady Uniform A. diversity factor
A. 120°F D. Uniform flow 44. _Instrument used to measure fluid B. power factor
B. 150°F velocity: C. utilization factor
C. 180°F 39. From PMC, TS = ultimate tensile A. Pitot tube D. demand factor
D. 130°F strength, t = minimum thickness of shell B. Manometer
= =
plate, E efficiency of joint, R one-half C. Orsat apparatus
33. The dividing point between the high- the inside diameter of the weakest D. Speedometer
pressure and low-pressure sides of the course, FS = ·allowable factor of safety,
refrigeration cycle occurs at the:

REFRESHER MANUAL 2nd Edition by JAS TORDILLO REFRESHER MANUAL 2nd Edition by JAS TORDILLO
35 - 6 I Day 35 - Solution Day 35 • Solution I 35 • 7
Power and Industrial Plant Engineering
DAY 35 - SOLUTION c. 20.2
(d )
1. A fuel gas has the following
D. 27.3 • WN' •_ 2434.34 = 27.30%
T1 = QA - 8916.06
DAY 35- ANSWER KEY volumetric analysis:
Solving for the mass flow rate:
1.C 16. C 31 .A 46.A CH.. =68% C2Hs = 32%
2. D 17. B 32. C47. D 3. Steam enters a throttling calorimeter
Assume complete combustion with 30%
3. A 1 B. A 33. A 48. A PV = mRT at a pressure of 1.03 Mpa. The
excess air at 101.325 kPa, 21°C wet
4. C 19. A 34. B 49. B 11 0(8.5) = m(0.287)(300) calorimeter downstream pressure and
5. B 20. B 35. C 50. A bulb and 27°C dry bulb. What is the
m = 10.86 kg/s temperature are respectively, 0.100 Mpa
6. C 21 . A 36. 0 partial pressure of the water vapor in
7. B 22. A 37.A kPa? and 125°C. What is the percentage
8. B 23. A 38. 0
A. 8.62 Solving .for T2: moisture of the supply steam?
9. A 24. 0 39. C k-1 Properties of Steam:
10. 0 25.0 40. c B. 19.28
11. C26. 0
12.A27. B
41. A
42. A
c. 14.24.
D. 16.94
T2 = (P2) k P, Mpa hf
1.03
hfg
2010.7 2779.25
hg

13.D2B. C 43.A
Tt P1
14. B 29. C 44. A
Combustion reaction with theoretical air: T 1.4-1 Note: At 0.100 Mpa and 125°C,
15. A 30. D 45. 0 3g0 =(10)-1'.'4'' h =
2726.6 kJ/kg
'"1; 0.68 CH4 + 0.32 C2Hs + 2.48 02 + T2 = 579°K A. 2.62.
2.48 (3.76) N2 + 1.32 C02 + 2.32 ·H20
+ 2.48 (3.76) N2 B. 3.15
Solving for T4: c. 5.21
k-1 D. 1.98
Combustion reaction with 30% excess
air:
0.68 CH4 + 0.32 C2Hs + 1.30 (2.48) 02
~: = (:! )k" hft = 2779.25 - 2010.7 = 766.55
+ 1.30 (2.48) (3.76) N2 + 1.32 C02 + 1.4-1
2.32 H20 + 1.30 (2.48) (3. 76) N2 +
0.3(2.48) 02
. 1400 = ( 950
T4 1000
)1.4 For throttling process:
ht = h2
(hf + X hfg), = h2
Total Mols in products: T.. = 736°K 766.55 + X (2010.7) = 2726.6
X = 0.9736 = 97.38%
-= 1.~2 + 2.32 + 12.12 + 0.744
= 16.504 We = mCp (T2- T1)
Partial Pressure of Water Vapor = 10.86(1 .0)(579-300) Percentage moisture= 100 - 97.38
= 3029.94 kw = 2.62%
= (~)101.325
16.504 4. A diesel electric plant supplies energy
3029 94
= 14.24 kPa We = · = 3695.04 kw for MECO. During a 24-hr period, the
0.82 plant consumed 300 gallons of fuel at
2. Air enters the compressor of a gas Wr = m Cp (T3- T..) 28°C and produced 4900 kw-hr.
turbine at 110 kPa and 27°C with a =
10.86 (1.0) (1400- 736) Industrial fuel used Is 32°API and was
volume flow rate of 8.5 m3 /sec. The = 7211.04 kw purchased at P32.0 per liter at 15.6°C.
compressor pressure ratio is 10 and its What shoqld the cost of fuel to be
isentropic efficiency is 82%. At the inlet Wr' = 7211.04(0.85) = 6129.38 kw produce one kw-hr?
to the turbine, the pressure is 950 kPa A. P7.05
and the temperature is 1400°K. The WN' = Wr' -We' = 6129.38 - 3695.04 B. P7.69
turbine has an isentropic efficiency of =
2434.34 kw C. P6.53 *
85% and the exit pressure is 100 kPa. D. PS.OO
On the basis of an air standard analysis, QA =mCp(T3- T2)
what is the thermal efficiency of the = 10.86(1 .0)(1400- 579) Solving for density at 15.6°C:
cycle in percent? = 8916.06 kw
A. 22.0
0
API = ~ - 131.5
B. 21.8 SG1s.6

REFRESHER MANUAL 2nd Edition by JAS TORDILLO REFRESHER MANUAL 2nd Edition by JAS TORDILLO
if

35 - 8 1 Day 35 - Solution Day 35 - Solution I 35 - 9


Power and Industrial Plant Engineering
141.5 0.3388
28 = - 131.5 m2 = kg handling coal at 0.75 m/s with 750 kg Power = Force x velocity
SGts.6 R
per m 3 . Other data as follows: = 4567.50 (0.75)
SG,s.s = 0.887
Solving for the final temperature:
Flight width and depth .................... . =3425.62 kg-m/s
.............600 mm x 220 mm
Density at 15.6°C = 0.887(1) PV = mRT Quantity of material .................... .... .
= 3425.62 = 45.05 h
= 0.887 kg/li 76.042 p
1350 (0.85) = c-~23 + 0.3~88)RT ............ 0.145 m3 /m
= 45.05(0.746) = 33.6 kw
Coefficient of friction elements ........ . .... .
Solving for density at 28°C: ............ 0.12
T = 556.55 °K 9. A tank having a capacity of 1 cu. m is
SGt = SG,s.e(1 - 0.0007(t- 15.6)] Material coefficient of friction .............. .
SG2eoc = 0.887[1 - 0.0007(28- 15.6)] filled with air to a pressure of 500 kPag
6. A Francis turbine is installed with a ............ 0.60 when the temperature is 60 deg C. It
= 0.879 vertical draft tube. The total head to the Assume an engineering type chain with
was found out later that because of the
center of the spiral casing at the inlet is sleeve bearing rollers weighing with
Density at 28°C = 0.879(1) leak the pressure drops to 350 kPag
60 m and velocity of water at the inlet is flights, 95.3 kg/m. Calculate the chain
and the . temperature has decreased to
= 0.879 kglli 7.5 m/s. The discharge is 1.5 m3/s. The pull in kg.
45 deg C. Determine the amount of air
hydraulic efficiency is 0.87 and overall A. 3180.33
that has leaked out in kg.
. per kg P32 B. 4667.50 *
Pnee =- - =P36.07 per kg
0.887
efficiency is 0.84. The vertical distance
from the centerline to the top of the draft c. 3550.10 A. 1.34 kg*
B. 3.34 kg
tube is 1.5 m while the tailrace water D. 4166.40
C. 2.34 kg
Cost Per kw-hr :;:: level is 2.5 m from the top of the draft I D. 4.34 kg
tube. What is the total head or:~ the W = weight of material per meter
300gal 3.78541i 0.87~g P32.07
_ _. . : : _ _ X - - - X - - - X - -
turbine in meters?
4900<w-hr gal li kg
A. 34.72 75a<g 0.145 m3 PV = mRT
= P6.53 per kw-hr
= --x-- (500 + 101.325)(1)
B. 55.20 m3 m = m,(0.287)(60 + 273)
c. 66.86* W = 108.75 kg/m m, = 6.29 kg
5. A certain gas at 101.325 kPa and D. 48.12
21°C whose volume is 5.0 m3 are
p, = pull to move the weight of material (350 + 101.325)(1)
compressed into a storage vessel of
0.85 m3 capacity. Before admission, the
h = total head on loaded run = m2(0.287)(45 + 273)
storage vessel contained the gas at a :;::
p
w + z v 2
+ A -Vs2
= 108.75(50)(0.60) =
3262.50 kg m2 = 4.945 kg
pressure and temperature of 120 kPa 2g
P2 = pull to move conveyor parts on Solving for the kg of.air leaked out
and 28°C; after admission the pressure 2 loaded run =
mL m, - m2 = 6.29 - 4.945
has increased to 1350 kPa. What
should be the final temperature of the
= 60 + (1.5 + 2.5) + (7.5) -0
2(9.81)
.= 108.75(50)(0.12)=652.50kg = 1.34 kg
gas in the vessel in Kelvin? = 66.86 m P3 = pull to move conveyor parts on 10. A vulcanizing shop is provided with
A. 498.0
empty run a 10 m3 of compressed air tank. During
B. 556.6 * 60m = 108.75 (50) (0.12) = 652.50 kg operation, air pressure in the tank

v,.,.~ )( r~l- - --,;:~ 1


C. 419.8
reduces from 700 kPa to 300 kPa while
D. 620.0
P = total chain pull the temperature remains constant at 32
= 3262.50 + 652.50(2) deg C. Determine the mass of air that
Solving for the mass of gas which is to
= 4567.50 kg has been used.
be compressed: ·
z A. 15.7 kg
8. In problem No. 7, What is the power B. 35.7 kg
PV = mRT
in kw required to drive the conveyor? C. 25.7 kg
101.325(5) = m,R(21+273)
Let Fp = 76.042 kg-m/s-HP. D. 45.7 kg*
\.723 k A. 20.20 kw
m, = - - g
R B. 33.6 kw * PV = mRT
Solving for the mass of gas initially C. 30.50kw 700(10) = m,(0.287)(32 + 273)
contained in the vessel: D. 45.0kw m, = 79.96 kg
PV = mRT
7. Given a horizontal conveyor, 50 m
centers, 200 lbs per hour capacity and
300(10) =
m2(0.287)(32 + 273)
120 (0.85) = m2 R (28 + 273) m2 = 34.27 kg
REFRESHER MANUAL 2 nd Edition by JAS TORDILLO REFRESHER MANUAL 2nd Edition by JAS TORDILLO
35 - 10 1 Day 35 - Solution Day 35 - Solution I 35 - 11
Power and Industrial Plant Enginebring
m =
m1 - m2 = 79.96- 34.27 B . 176°F *
m = 45.7 kg C. 455°F volume to 240 li during which the and the other side is exposed to
D. 100°F enthalpy decreases 80 kJ. The specific .combustion gases whose temperature is
11. Steam at 2.6 MPa and 600°C (h = heats are constant, assuming Cv to be 1500°C (h = 12 W/m 2 -K). Given a heat
3478.5 kJ/kg) enters a nozzle QA - QR : , TH - TL 3.16 kJ/kg-0 K. Determine work done transfer rate of 6000 W per square
undergoes reversible adiabatic process Qll TH during the process. meter area, determine thickness of the
with velocity of 400 m/s. Find the A. 54 kJ wall.
stagnation enthalpy. 100 - 60 _ (600 + 460)- TL B. -54 kJ * A. 15.9 em *
A. 2558.5 kJ/kg 100 - (600 + 460) c. 80 kJ B. 24.2 em
B. 4558.5 kJ/kg TL = 636°R = 176°F D. -80 kJ C. 20.2 em
c. 3558.5 kJ/kg .. D. 32.4 em
D. 5558.5 kJ/kg 15. An Ideal gas at 50 psig and 90°F is W = PN2- PN1
heated in a closed container to 140°F. = 150(0.24) - 600(0.15) = -54 kJ q t l - t2
Vl Vl What is the final pressure in psia? A=l+x+1
hl
2 =
+- hz +-
2 A. 70.6 psla • 18. From Problem No. 17, determine Tit f n;;
(400)2 B. 60.7 psia the change in internal energy.
3478.5 + 2 ( 1000) = h2 + 0 C. 67.6,psia A. -26 kJ * 1500-29
h2 = stagnation enthalpy = 3558.5 kJ/kg D. 14.7 psia B. 26 kJ 600 0 =1 X 1
c. 50 kJ 12 + 1.5 + 18
12. · There are 100 kg/min of water P1 · Pz D. -50 kJ
x = 0.159 m = 15.9 em
enters a heating element at 5°C and rl = r2
leaves at 40°C. Find the heat added t.H = t.U + t.W
22. Heat normally flowing from a high
duril)g the process. 50+ 14.7 _ P2 -80 = t.U -54
temperature body to a low temperature
A. 244.25 kw * 90 + 460 - 140 + 460 t.U = - 26 kJ
B. 344.25 kw ; body wherein it is impossible to convert
C. 295.25 kw 19. From Problem No. 17, determine heat without other effects is called the:
P2 = 70.6 psia • A. second law of thennodynamics *
D. 775.9 kw the specific heat constant of the gas.
B. first law of thermodynamics
16. A Carnot engine requires 35 kJ/sec A. 3.076 *
100 C . third law of thermodynamics
Q = mCpt.t = (4.187)(40 - 5) from a hot source. The engine B. 3.125
60 D. zeroth law of thermodynamics
Q = 244.25 kw produces 15 kw of power and the C . 2.756
fempetature of the sink is 26°C. What is D. 2.108
23. A two-stage compressor with an
13. Water has an absolute temperature the temperature of the hot source in °C? 3
ideal intercooler receives 0.4 m /s of air
of 10.5 m of water and atmospheric A. 150.25°C 6H = :..!!£ = 3.076 = mCp-" =2 = k at 100 kPa and 31 0°K. If the intercooler
I!.U -26 mCvll~ Cv
pressure of 90 kPa. Find the gage B. 523.25°C
pressure is 707.1 I<Pa, determine the
pressure is meters of water. C. 250.25°C *
20. From 'Problem No. 17, determine discharge pressure in I<Pa.
D. 750.25°C
A. 9.1 m of water the gas constant, R. A. 5000 kPa *
B. 3.32 m of water A. 3.16 kJ/kg-deg K B. 3000 kPa
C. 5.5 m of water W = QA-QR
15 = 35- QR B. 6.56 kJ/kg-deg K * C. 2500 kPa
D. 1.32 m of water* C. 4.25 kJ/kg-deg K D. 266 kPa
QR = 20 kw
D. 7.16 kJ/kg-deg K
P(abs) = P(gage) + P(atm) Px = ~P1P2
w TL
10.5 = P(gage) + ~ - =1-- R = Cp -Cv 101.1 = ~r:-:C1""'
oo=-)c=p2....,.)
9.81 QA TH
P(gage) = 1.32 m of H20 P2 = 5000 I<Pa
15 (26 + 273) Where:
Cp = Cv(k) = 3.16(3.076)
14. A Carnot engine receives 100 Btu 35 = 1 - TH 24. Calculate the logarithmic mean
= 9.72 kJ/kg-deg K
of heat from a hot reservoir at 600°F and
rejects 60 Btu of heat. Calculate the
TH = 523.25°K =
250.25°C R =
9.72- 3.16 =
6.561<Jikg-deg K
temperature difference for a condenser
if the temperature difference between
temperature of the cold reservoir in deg 17. A gas initiplly at P1 = 600 kPa and condensing steam and water inlet is
F. 21 . One side of a furnace wall (k = 1.5
A. 636°F
v1 = 150 li, undergoes a process to that W/m-K) is exposed to ambient air whose
650°C and that steam inlet and water is
changes the pressure to 150 kPa and 2 120°C.
temperature is 29°C (h = 18 W/m -K)
REFRESHER MANUAL 2nd Edition by JAS TORDILLO REFRESHER MANUAL 2nd Edition by JAS TORDILLO
35 - 12 1 Day 35 - Solution Day 35- Solution 135- 13
Power and Industrial Plant Engineering
A.
B.
325.7°C
313.7°C"
Efficiency = !!!..
Ptn
=~
(1.1)0.7<4.6
= 81.4%
C. compressor A. t TS.E
C. 225.7°C D. cooling coil R.FS
D. 155.7°C 27. Heat exchanger used to provide
heat transfer between the exhaust B. TS.r
AT1 0Tz gases and the air prior to its entrance to 34. The state of any vapor whose R.FS.E
LMTD = lnor,/Ar2 the combustor: pressure exceeds that at which
A. evaporator condensation occurs at the prevailing C. TS.t.E *
R.FS
B. regenerator • temperature.
LMTD = 650 120
-
ln650ft20
= 313. 7°C C. combustion chamber A. Saturation D. R.TS.t.E
D. heater B. Supersaturation * FS
25. A pump discharges 11 0 liters per C. Critical
28. A device for measuring the velocity D. ·Mixture 40. In PMC, what is the color of the
second of water to a height of 45
of wind: pipe for Carbon Dioxide?
meters. If. the pump efficiency is 70%
and the speed of the pump is 1200 q>m, A. aneroid barometer 35. What amount of air is required in a A. Orange
B. anemoscope low bypass factor? B. Green
what is the power input in the drive
C. anemometer* A. greater C. Red*
shaft?
D. anemograph B. indeterminate D. Yellow
A. 34 kw
B. 79.4 kw C. lesser*
C. 44 kw 29. A ir standard efficiency of a diesel D. does not change 41 . Servomotor controls
A. Moving vanes *
D. 69.4 kw * engine depends on:
A. speed 36. What is the clockwork-operated B. Moving and guide vanes
B. fuel device which records continuously the C. Guide vanes
Pump Work = Qroh
C. compression ratio • humidity of the atmosphere? D. None of these
= (0. 11 0)(9.81 )( 45)
= 48.Q6 kw D. torque A. hetrograph
B. hydrodeik 42. Difference in pressure measure
Brake Power (Pump)
30. Heavy water is: C. hygrometer above or below atmospheric pressure:
_ PumpWork 48.56 D. hygrograph * A. draft *
A. B20 (2 is written as subscript)
- £ff = 0.70 B. W 20 (2 is written as subscript) B. stack
= 69.4 kw C. H20 (2 is written as subscript) 37. Type of flow in which all conditions C. chimney
D. D20 (2 is written as subscript) * at any point in a stream remain constant D. breeching
26. A fuel pump is delivering 18 gallons with respect to time .
per minute of oil with a specific gravity of 31. Mechanism designed to lower the A. Steady flow * 43. The kinetic energy of a moving fluid
0.80. The total head is 75 meters. If the temperature of air passing through it: B. Uniform flow is used to isentropically compressed the
power input to the pump is 1.1 hp, what A. air cooler* C. J.Jnsteady flow fluid to state of zero velocity. The
is the efficiency of the pump? B. air spillover D. Nonuniform flow temperature of a moving fluid. at the
A. 69.3% C. air defense state of zero velocity is called:
B. 75% D. air cycle 38. Water is flowing along a constant A. stagnation temperature "
c. 72.1% cross section along the length of the B. partial temperature
D. 81.4% * 32. The viscosity of most commercially channel, we say the flow is: C. critical temperature

l
available petroleum lubricating oil A Steady flow D. absolute temperature
Pump Work = Qwh changes rapidly above: B. Unsteady flow
A. 120°F C. Steady Uniform 44. Instrument used to measure fluid

=
[
18
Gal
min X~ 60sec x
B. 150°F
C. 180°F*
D. Uniform flow*

39. From PMC, TS = ultimate tensile


velocity:
A. Pitot tube *
B. Manometer
~
D. 130°F
X 3.7854...!!!.._ X strength, t = minimum thickness of shell C. Orsat apparatus
Gal . IOOOiit 33. The dividing point between the high- plate, E = efficiency of joint, R = one-half D. Speedometer
pressure and low-pressure sides of the the inside diameter of the weakest
x 9 . 8 1 ~) (75 m) refrigeratio~ cy9le occurs at the: course, FS = allowable factor of safety, 45. Which property of fluids is of
0 . 80 then the maximum allowable ~orking fundamental importance in the study of
( m3 A. expans1on valve *
pressure in MPa is: hydraulics?
= 0.668 kw B. condenser
RE.FRESHER MANUAL 2nd Edition by JAS TORDILLO REFRESHER MANUAL 2nd Edition by JAS TORDILLO
35 ·141 Day 35 ·Solution Day 36 • EXAM I 36. - 1
Power and Industrial Plant Engineering
A. unit weight
B. viscosity c. 10"
C. mass density DAY 36- EXAM D. 8" or 10"
D. all of the above *
1. Calculate the MW power capacity of a 7. What will happen to the fluid-energy
46. What is the electric current flowing Geothermal plant with a load factor of from the suction flange to the energy
in one direction and which reverses and 0.82 and 120 MW peak load. The input of the impeller?
flows in opposite direction and usually in operation is limited to 8500 hours a year A . Increase
a steady state? with a use factor of 0.70. B. Decrease
A. alternating current * A. 175 C. Does not change
B. direct current B. 145 D. Remains constant
C. load current C. 199
D. both alternating and direct current D. 233 8. A single-acting reciprocating pump,
with a diameter of 0.10 meter and stroke
47. W here is the location of servomotor? 2. A Diesel power plant has a maximum of 0.15 meter, delivers water at a rate of
A. Near the shaft demand of 120 MW with a load factor of 0.015 cubic meter per minute. What is
B. O utside the shaft 0.80 and capacity factor of 0.6. the percentage slip it delivers at 15
C. At the fulcrum Estimate the plant capacity. revolutions per minute?
D. Inside the shaft* A. 100 MW A. 15.1%
B. 180 MW B. 20.5%
48. Excess water from servomotor is C. 160 MW C. 18.4%
expelled by: D. 200 MW D. 41 .6%
A. Siphon*
B. Nozzle 3. A given equipment consumes 5000 9. A 4-stage centrifugal pump
C. Governor kw-hr/month at 24% rated plant discharges 1000 gpm of water at 1750
D. Draft tube capacity. It operates at 24 hours, 30 rpm with a total heat head of 1000 ft.
days/month. What is the rated capacity? Find the specific speed.
49. Peak load for a period of time A. 17.50 kw A. 550 rpm
divided by installed capacity is: B. 28.90 kw B . 880 rpm
A . capacity factor c. 23.90 kw C . 1200 rpm
B. utilization factor * D. 82.94 kw D. 1000 rpm
C. demand factor
D. load factor 4. The ratio of the average load during 10. Water flow through a pipe .at 100
a certain time to the peak load occurring li/sec. The inlet diameter of pipe is 12
50. The ratio of the sum of individual during the same period of time is called: em and its ex it diameter ls 15 em. If H20
maximum demands of the system to the A. demand factor is to be pumped from H20 source 10 m
overall maximum demand of the whole B. capacity factor below pump centerline and heat losses
system is: C. load factor · Is 2.3 kw, determine the power input to
A. diversity factor * D. output factor the pump.
B. power factor A. 7.5 Kw
C. utilization factor 5. If the highest water surface is below B. 5.2 Kw
D. demand factor the pump centerline, what is the static C . 9.8 Kw
head? D. 7.6 Kw
A . Negative
B. Positive 11 . Water is to be raised to a height of
C. Zero 20 m at 50 kg/s. Inlet diameter is 16
D. None of these em, exit diameter is 12 em and heat loss
is 3 kw. Determine the power input to
6. Given 10" x 8" pump, what is the the pump.
diameter? A. 13.14 kw
A. 8" B. 7.14 kw
B. 18"
REFRESHER MANUAL 2 nd Ed ition by JAS TORDILLO REFRESHER MANUAL 2nd Edition by JAS TORDILLO
36-21 Day36-EXAM
Day 36 - EXAM I 36 - 3
C. 15.90 kw 16. Determine the rate of heat Power and Industrial Plant Engineering
D. 59.7 kw absorption in kJ/kg.
A. 22,886.9
B. 30 days R-12 temperature at 30°C(hr =28,?.54
Problems 12 to 15 B. 3,433 C. 15.days kJ/kg). What is the refrigerating
A 850 bhp diesel engine uses fuel oil of D. 20 days capacity in tons of refrigeration if R-12
C. 11,855
32°API gravity at 40 deg C, fuel D. 15, 855 mass flow rate is 0.5 kg/sec?
consumption is 0.75 lbs per bhp-hr. 22. In a certain process, energy entering A. 11.53
Cost of fuel is P45 per liter. 17. Find the boiler horsepower. is 1700 kJ/kg and 170 m/s. If the B. 16.20
12. What is the density of the fuel used A. 350 energy leaving is 1000kJ/kg, determine C. 8.88
in kg/li? B. 1970 the velocity at the exit? D. 32.67
A. 0.887 C. 1978 A. 1195 m/s
B. 0.884 D. 2332 B. 978 m/s 27. A refrigeration cycle having 0. 18
C. 0.850 C. 1145 m/s MPa (h9 = 180.8 kJ/kg) and 0.8 MPa (h
D. 0.821 18. What is the boiler efficiency? D. 1020 m/s = 208.65 kJ/kg). The mass flow rate in
A. 85.5% the cycle is 0. 18 kg/sec with 18.5 kw
13. What is the fuel consumption in B. 65.6% 23. What is the atmospheric pressure rate of heat removal. Determine the
liters per hr? C. 74.6% on a planet if the absolute pressure is heat rejected per power.
A. 340 D. 55.6% 120 kPa and the gage pressure is 20 A. 4.69
B. 225 kPa? B. 5.99
c 280 Problems 19 to 21 A. 140 kPa c. 2.59
D. 450 A circular fuel oil tank 50. feet long and B. 20 kPa D. 8.69
5.0 feet diameter is used for oil storage C. 100 kPa
14. Determine the minimum volume of and supplying fuel oil at 30°API to a D. 120 kPa 28. 500 kilograms of dressed chicken
the required day tank. steam boiler. The steam flow is 2000 enter a chiller at 10°C are frozen and
A. 6720 liters : lbs per hour at 200 psia dry steam. 24. A bicycle tire has a volume of 1000 chilled to a final temperature of -15°C for
B. 10800 liters Feedwater is 230 deg F and boiler cu. em. It is inflated with carbon dioxide storage in 20 hours. Compute the
C. 5400 liters efficiency is 70%. to pressure of 200 kPa at 30°C. How product load.
D. 8162 liters Steam Properties: many grams of carbon dioxide are Specific heat above freezing
At 200 psia (1.38 Mpa): contained in the tire? .... .. ... ... 3.2 kJ/kg-°K
15. Find the cost of fuel per day. hg = 2789.6 kJ/kg A. 5.98 g Specific heat below free~i ng
A. P367,290 At 230' deg F (110 deg C): B. 4.63 g ........ . .... 1.6 kJ/kg-°K
B. P541,271 hf = 461.3 kJ/kg C. 6.43g Latent heat
C. P267,290 D. 3.49 g ............ 250 kJ/kg
D. P540,701 19. Determine the mass flow rate of fuel Freezing Temperature
in kg per hr. 25.' A simple vapor compression cycle .......·... .. - 5°C
Problems 16 to 18 A. 66.6 develops 30 tons of refrigeration which A. 2.18 kw
A water tube boiler evaporated 10 kg of B. 86.5 uses ammonia and operates at 24°C B. 6.67 kw
water per sec from a feed water C. 56.6 (hr = 312.87 kJ/kg) condensing C. 3.67 kw
temperature of 104.44 deg C to steam D. 96.8 temperature and -18°C (h9 = 1439.94 D. 7.67 kw
at 1241.1 kPa and quality of 0.97; kJ/kg) evaporating temperature. The
weight of coal fuel per sec 1.5 kg; high 20. Determine the density of fuel in kg enthalpy of the refrigerant after the 29. Water at 55°C is cooled in a cooling
heat value of the coal as fired, 11 ,800 per cu. meter. compression is 1657 kJ/kg. Determine tower which has an efficiency of 60%.
Btu/lb. A. 1000 the compressor power in kw. The ambient air is at 32°C dry bulb and
Steam Properties: B. 925 A. 10.2 kw 27°C wet bulb. The heat rejected in the
At 104.44 deg C: C. 995 B. 20.33 kw condenser is 900 kJ/sec. Find the
hf = 437.78 (interpolated) D. 876 C. 13.2 kw capacity in liters per second of the pump
At 1241 .1 kPa: D. 33.2 kw used in the cooling tower.
hf = 805.29 21. Calculate the number of days the A. 16.5
hfg = 1980.6 supply tank can hold for continuous 26. A refrigeration system operates on B. ' 12.8
(interpolated) operation. an ideal vapor compression using R-12 C. 17.5
A. 25 days with a saturated temperature of - D. 29.5
1.1°C(h9 = 351 .003 kJ/kg) and a liquid
REFRESHER MANUAL 2nd Edition by JAS TORDILLO REFRESHER MANUAL 2nd Edition by JAS TORDILLO
36 - 4 I Day 36 - EXAM
Day 36 • EXAM I 36 - 5
30. A r.efrigeration plant is rated at 30 A. solids only Power and Industrial Plant Engineering
ton capacity. How many pounds of air B. liquids only
per hour will it cool from 110 to 60°F at 43. The type of compressor that B. Use factor
C. gases only
constant pressure? incorporates a volute is a - - - - - C. Diversity factor
D. all of the above compressor.
A. 30,000 lb/hr D. Load factor
B. 52,000 lb/hr A Roller
37. Name the process that is no heat B. Screw
C. 47,000 lb/hr 50. The quantity of energy absorbed by
loss or gain and no friction loss.
D. 45,000 lb/hr A Isentropic process
C. Centrifugal a unit mass of refrigerant in the
D. Reciprocating evaporator is called.
B. Isothermal process
31. On existing installation boiler, the C. Adiabatic A Mass flow rate
lowest factor of safety permissible shall 44. In order to increase the back B. Refrigeration effect
D. Non-reversible
be: pressure of a boiler: C. Capacity .
A 7 A Increase the boiler pressure D. Volume flow rate
38. The amount of heat needed to raise
B. 5 B. Close the atmospheric line
the temperature of one lb of that
C. 6 C. Open the back pressure valve
substance one degree Fahrenheit is:
D. 4.5 D. Close the back pressure valve
A. Specific heat

32. The age limit of a horizontal return


B. Btu
C. Latent heat
45. During compression process, the
internal energy of the refrigerator vapor:
ic? I )
tubular flue or cylinder boiler having a D. Relative heat DAY 36- ANSWER KEY
longitudinal lap joint and operating at a A . Increases 1. 8 16.A 31.046.0
B. Decreases 2. C 17. D 32. B 47. A
pressure in excess of 0.345 MPa or 3.45 39. What is the most efficient 3. B 18. 0 33. A 48. A
Bar gage shall be: C. Remains constant
thermodynamic cycle? 4. C 19. A 34. A 49. 0
A. 18 years D. Internal energy have no relation to 5. A 20. 0 35. A 50. B
A Carnot
B. 30 years the vapor compression process S.C 21.C 36.0
B. Diesel
C. 35 years 7. A ' 22.A 37.A
C. Rankine 8. A 23. C 38. B
D. 20 years 46. What do you call the weight of the
D. Brayton 9. B 24. 0 39. A
column of air above the earth's surface?
10. C25. B 40.A
33. Dew point is which of the following? A. Air pressure 11.A26. C 41. B
40. The component in the refrigeration
A adiabatic saturation temperature B. Aerostatic pressure 12. C27.A 42. D
cycle that is used to store refrigerant is 13. A 28. A 43. C
B. temperature of grains of moisture per C. Wind pressure
called the 14. 029. B 44. 0
lb of bone dry air D. Atmospheric pressure
A Reeeiver 15. A30.A 45.A
C. temperature of 50% RH B. Evaporator
D. wet bulb temperature C. Metering device
47. A reactor which uses slow moving ~
neutrons.
D. Compressor
34. Air used for comfort cooling shall A. thermal reactor
maintain a movement of from: B. intermediate reactor
41. Which defrost method is commonly
A 4570 to 7620 mm/min C. slow reactor
used in small commercial applications
B. 8500 to 10,000 r:nm/min D. slow speed reactor
where ice forms on the evaporator
C. 7800 to 9200 mm/min surfaces?
D. 8.00 to 10.0 mm/min 48. A geothermal power plant which .
A Electric
utilizes a heat exchanger to transfer
B. Hot gas
35. A crack in the earth's surface heat from a primary working fluid to a
C. Off-cycle
through which some chemical secondary fluid which is used to power
D. All ofthese
substances pass: or move a turbine:
A. Fumarole A. binary. cycle
42. The type of compressor that uses
B. Geyser B. double cycle
ring valves is a
C. Earthquake C. dual cycle
compressor.
D. Storm D. none of these
A Screw
B. Centrifugal
36. Heat can be transferred by 49. Ratio of the average load to the
C. Roller
conduction through: peak load over a period of time
D. Reciprocating
A. Demand factor

REFRESHER MANUAL 2nd Edition by JAS TORDILLO REFRESHER MANUAL 2nd Edition by JAS TORDILLO
36 - 6 I Day 36 - Solution
Day 36 - Solution 136 - 7
3. A given equipment consumes 5000 Power and Industrial Plant Engineering
DA'Y 36 -SOLUTION kw-hr/month at 24% rated plant
capacity. It operates at 24 hours, 30 8. A single-acting reciprocating pump, Q
1. Calculate the MW power capacity of a days/month. What is the rated capacity? with a diameter of 0.10 meter and stroke -=
Vz = A2 ic1s x 10-1)1 = 5.65 mfs .
Geothermal plant with a load factor of A. 17.50 kw of 0.15 meter. delivers water at a rate of
0.82 and 120 MW peak load. The H = 10 + 1
C5.6s) - (8. 84>1
B. 28.90 kw* 0.015 cubic meter per minute. What is 2(9.81) = 7.64 m
operation is limited to 8500 hours a year c. 23.90 kw the percentage slip it delivers at 15
with a use factor of 0. 70. D. 82. 94 kw revolutions per minute? Power Input = QyH + losses
A. 175 A. 15.1% * = (0.100)(9.81)(7.64) + 2.3
B. 145 * Average load B. 20.5% = 9.8 kw
c. 199 =
5000 kw-hr month
x ---x ---
day C. 18.4%
D. 233 month 30 days 24 hours D. 41.6% 11. Water is to be raised to a height of
=6.94kw 20 m at 50 kg/s. Inlet diameter is 16
Load factor = Average toad V0 = ~D 2
LN = ~(0.10)2(0.15)(15) em. exit diameter is 12 em and heat loss
Peak Loa Average Load
Plant factor = Rating of equipment = 0.01767 m~/min is 3 kw. Determine the power input to
0.82 = Average toad Q = 0.015 m3/min the pump.
120 0.24 = 6.94 A. 13.14 kw *
Rattng of Equ ipment Sl' =· Vo- Q = 0.01767-0.015 = 15 _1% B. 7.14kw
Average Load = 98.4 kw lp v0 0.01767 C. 15.90 kw
Rating of Equipment (Rated Capacity of D. 59.7 kw
Use Factor = Av erage toad (8160) Pla_nt) = 28.93 kw 9. A 4-stage centrifugal pump
Plant Capacity (8500)
discharges 1000 gpm of water at 1750 Q= ~ = 2£.. = 0.05 m3/sec
= 98.4(8760)
4. The ratio of the average load during rpm with a total heat head of 1000 ft. p 1000
0.70 a certain time to the peak load occurring Find the specific speed.
Plllnl Capaclty(8500)
during the same period of time is called: A. 550 rpm v1 = !1._ = o.o5 _
A. demand factor B. 880 rpm • At ico.16)1- 2.48 mfs
Plant Capacity = 144.9 MW
B. capacity factor
C. load factor*
C. 1200 rpm
D. 1000 rpm
v2 = !1._
A2
= ~O.l2)z
o.os
= 4.42 mjs
2. A Diesel power plant has a maximum
demand of 120 MW with a load factor of D. output factor ·
H = 20 + (4.42)1 - (2.48)1
0.80 and capacity factor of 0.6. h = total head = 1000 ft 2(9.81) = 25.1 m
Estimate the plant capacity. 5. If tbe highest water surface is below
the pump centerline, what is the static
Head per stage = ~
4
= 250 ft
A. 100 MW Water Power = QyH
B. 180 MW head? N• = specific speed = N..{Q
h•l• =
0.05(9.81 )(20.6'8) 10.14 kw =
C. 160 MIJV * A. Negative * 1750 Input Power (Brake Power)
B. Positive •= {i'OOO = 880.20 rp m
D. 200 MW (250)3/• = 10.14+3 = 13.14kw
C. Zero
Load Factor = Average Load
D. None of these 10. Water flow through a pipe at 100 Problems 12 to 15
Maximum LOad lilsec. The inlet diameter of pipe is 12 A 850 bhp diesel engine uses fuel oil of
6. Given 10" x 8" pump, what is the em and its exit diameter is 15 em. If H20 32°API gravity at 40 deg C. fuel
0.80 = Average Load diameter? is to be pumped from H2 0 source 10 m consumption is 0. 75 lbs per bhp-hr.
120 A. 8" below pump centerline .and heat losses Cost of fuel is P45 per liter.
B. 18" is 2.3 kw, determine the power input to 12. What is the density of the fuel used
Average Load = 96 MW c. 10" * the pump. in kg/li?
D. a· or 10" A. 7.5 Kw A. 0.887
Average Load
Capacity Factor = Plane capacity B. 5.2 Kw B. 0.884
7. What will happen to the fluid-energy C. 9.8 Kw * c. 0.850.
96 from the suction flange to the energy D. 7.6 Kw D. 0.821
0. 60 = Plant Capacity input of the impeller?
A. Increase " Q = 100 li/sec :;: 0.1 m 3/sec 0 API =~-131.5
Plant Capacity = 160 MW B. Decrease SGu .6
C. Does not change v1 = !l. = ic12 0.1 32 =~- 131.5
D. Remains constant At xto-1)z = 8.84 m/s SG 1$.6

REFRESHER MANUAL 2nd Edition by JAS TORDILLO


SG1s.e =0.865
REFRESHER MANUAL 2nd Edition by JAS TORDILLO
36 - 8 I Day 36 - Solution Day 36 - Solution I 36 - 9
Power and Industrial Plant Engineering
SGt = SG,se[1 - 0.0007(t- 15.6)] 16. Determine the rate of heat
SG4s = 0.865[1- 0.0007(40- 15.6)] absorption in kJ/kg. 19. Determine the mass flow rate of fuel 22. In a certain process, energy entering
SG4s = 0.850 A. 22,886.9 * in kg per hr. is 1700 kJ/kg and 170 m/s. If the
B. 3,433 A. 66.6 * energy leaving is 1000kJ/kg, determine
Density of fuel = 0.850 (1) = 0.850 kg/li C. 11,855 B. 86.5 the velocity at the exit?
D. 15,855 C. 56.6 A. 1195 m/s *
13. What is the fuel consumption in D. 96.8 B. 978 m/s
liters per hr? h1 = hf + xhfg C. 1145 m/s
A. 340 * h, = 805.29 + 0.97{1980.6) 2000 D. 1020 m/s
m5 = = 907.03 kglhr
B. 225 = 2726.47 kJ/kg 2.205
c. 280 QA = ms(h, - h•) Qh = 41,130 + 139.6(30) = 45,318 kJ/kg ,].
H1: ~:h 2 +__!1.._
yl
D. 450 QA::; 10(2726.47- 437.78) 2(1000) 2(1000)
QA "' 22,886.90 kJ/sec
Boiler eft • ms(hs - htl 070 2
V = mf = ~x850Bhpx _k_g_ .. __ li_ 1700 + ) = 1000 + Vf
m,ah 2(1000) 2(1000)
r Pr Bhp - hr 2.205lbs o.ssokg 17. Find the boiler horsepower.
Vr = 340. ! 311/hr A. 350
0.70 = 907.03(2789.6-461.3)
mr(45,3 18)
v2 = 1195.36m/s
B. 1970
mr = 66.57 kg/hr 23. What Is the atmospheric pressure
14. Determine the minimum volume of c. 1978
the required day tank. D. 2332 * on a planet if the absolute pressure is
20. Determine the density of fuel in kg 120 kPa and the gage pressure is 20
A. 6720 liters
B. 10800 liters per cu. meter. kPa?
Bo. H = m5 (3600) (h 1 -h4)
C. 5400 liters p 35,322 A. 1000 A. 140 kPa
D. 8162 liters • B. 925 B. 20 kPa
10(360q - (2726.47-43J.78)
35,322
c. 995 C. 100 kPa *
Volume= 340 (24) = 8162 liters D. 876 * D. 120 kPa
= 2332.62
0 API~ 14 U - !l!.S Pabs = P.tm + Pgage
15. Find the cost of fuel per day. 18. What is the boiler efficiency?
A. P367 ,290 * A. 85.5% 14
)0 = SG
SG
!.5 - IllS
120 = Patm + 20
B. P541,271 B. 65.6% .. Patm = 100 kPa
C. P267,290 c. 74.6% SG = 0.87616
24. A bicycle tire has a volume of 1000
D. P540,701 D. 55.6% *
Density of fuel = SG x density of water cu. em. It is inflated with carbon dioxide
Cost= P45 x 8162 li Qh = 11 ,800 Btu/lb = 27,450 kJ/kg = 0.876 X 1000 to pressure of 200 kPa at 30°C. How
= P367,290 per day =
876 kg/m
3 many grams of carbon dioxide are
contained in the tire?
ms<ht - h4)
Problems 16 to 18 %= mrOh 21 . Calculate the number of days the A. 5.98 g
A water tube boiler evaporated 10 kg of !0(2726.47 - 437.78) = 0.5558 =55.58% supply tank can hold for continuous B. 4.63 g
water per sec from a feed water 'lb = 1.5(27,450) operation. c. 6.43g
temperature of 104.44 deg C to steam A. 25 days D. 3.49 g *
at 1241.1 kPa and quality of 0.97; Problems 19 to 21 B. 30 days
weight of coal fuel per sec 1.5 kg; high A circular fuel oil tank 50 feet long and C. 15 days* M = mol. wt . of C02 = 44
heat value of the coal as fired, 11,800 5.0 feet diameter is used for oil storage D. 20 days
=~ = 0.1889 kJ/kg--deg C
tark:(~} 5·0 Y(~·"=2182 d
Btu/lb. and supplying fuel oil at 30°API to a R
44
Steam Properties: steam boiler. The steam flow is 2000 vorumet fuel
4 3.28) 128)
At 104.44 deg C: lbs per hour at 200 psia dry steam. 3 3
Mass of fuel in the tank= 27.82 x 876 V = 1000 cm = 0.001 m
hf = 437.78 (interpolated) Feedwater is 230 deg F and boiler P = 200kPa
At 1241.1 kPa: efficiency is 70%.
= 24,371.51 kg
T = 30 + 273 = 303 K
hf = 805.29 Steam Properties: No. of days = mass of fuel
PV =mRT
hfg = 1980.6
(interpolated)
At 200 psia (1.38 Mpa):
hg = 2789:6 kJ/kg
mass flow rate
24.371.5 I
(200)(0.001) =
m(0.189)(303)
m = 0.00349 kg = 3.49 g
At 230 deg F (110 deg C): c 66.6(24)
hf = 461.3 kJ/kg = 15.2 days
REFRESHER MANUAL 2nd .Edition by JAS TORDILLO REFRESHER MANUAL 2nd Edition by JAS TORDILLO
36 - 10 1 Day 36 - Solution
Day 36 - Solution 136 - 11
25. A siJrlple vapor compression cycle the cycle is 0. 18 kg/sec with 18.5 kw Power and Industrial Plant Engineering
develops 30 tons of refrigeration which rate of heat removal. Determine the
uses ammonia and operates at 24°C heat rejected per power. capacity in liters per second of the pump B. 30 years *
(hr = 312.87 kJ/kg) condensing A. 4.69 * used in the cooling tower. C. 35 years
temperature and -18°C (hg = 1439.94 B. 5.99 A. 16:5 D. 20 years
kJ/kg) evaporating temperature. The C. 2.59 B. 12.8 *
enthalpy of the refrigerant after the D. 8.69 c. 17.5 33. Dew point is which of the following?
compression is 1657 kJ/kg. Determine D. 29.5 A. adiabatic saturation temperature *
the compressor power in kw. We = m(h2- h,) B. temperature of grains of moisture per
A. 10.2 kw
B. 20.33 kw *
=
0.18(208.65- 180.8) = 5.013 kw Cooling tower Eff "' ta- tb
td- twb
lb of bone dry air
C. temperature of 50% RH
C 13.2 kw QA = m(h, - h4) 0.60 = 55- tb D. wet bulb temperature
D 33.2 kw 18.5 =
0.18(180.8-h4)
55-27
tb = 38.2°C 34. A ir used for comfort cooling shall
h4 = 78.02 kJ/kg
h, = hg = 1439.94 maintain a movement of from:
QR = mCpt.t A. 4570 to 7620 mmlmln •
h2 = 1657 QR = m(h2 - h3) 900 = m(4.187)(55- 38.2)
h3 = h4 = 312.87 = 0.18(208.65- 78.02) = 23.51 kw m = 12.8 kg/sec
B. 8500 to 10,000 mm/min
C. 7800 to 9200 mm/min
QA = 30 tons ref = 105.48 kw
QR 23.51 D. 8.00 to 10.0 mm/min
QA = m(h, - h4) w., = 5.013 = 4' 69 Capacity of Pump "' 12.8 kB •
s ec
35. A crack in the earth's surface
105.48 = m(1439.94-312.87) = 12.81iters/sec
m = 0.09367 kg/s through which some chemical
28. 500 kilograms of dressed chicken
30. A refrigeration plant is rated at 30 substances pass:
enter a chiller at 10°C are frozen and
Comp Work, We = m(h2- h,) ton capacity. How many pounds of air A . Fumarole *
chilled to a final temperature of -15°C for
= 0.09367(1657 -1439.94) per hour will it cool from 110 to 60°F at B. Geyser
storage in 20 hours. Compute the
= 20.33 kw constant pressure? C. Earthquake
product load.
A. 30,000 lb/hr * D. Storm
Specific heat above freezing
26. A refrigeration system operates on ........... . 3.2 kJ/kg-°K B. 52,000 lb/hr
an ideal vapor compression using R-12 C. 47,000 lb/hr 36. Heat can be transferred by
$pecific heat below freezing
with a saturated temperature of - D. 45,000 lb/hr conduction through:
...... ....... 1.6 kJ/kg-°K
1 1°C(hg = 351 .003 kJ/kg) and a liquid A. solids only
Latent heat
R-12 temperature at 30"C(hr = 288.54 Tons of Ref = mCp(T.-T1 )
B. liquids only
............ 250 kJ/kg
kJ/kg). W hat is the refrigerating 12,000 C. gases only
Freezing Temperature
capacity in tons of refrigeration if R-12 D. all of the above *
............ - 5°C
mass flow rate is 0.5 kg/sec? A. 2.18 kw *
A. 11.53
30 = 37. Name the process that is no heat
B. 6.67 kw
B. 16.20 loss or gain and no friction loss.
C. 3.67 kw
c. 8.88 * A. Isentropic process *
D. 7.67 kw
D. 32.67 31. On existing installation boiler, the B. Isothermal process
lowest factor of safety permissible shall C . Adiabatic
Prpduct load
Refrigerating Capacity = m(h, - h4) be: D. Non-reversible
= m[Cp1(t1 - tr) + hL + Cp2(tr -12))
= 0.5(351.003- 288.54) A. 7
- ( 500 ) [3.2(10- -5) + 2501
= 31.23 kw B. 5 38. The amount of heat needed to raise
- 20(3'600) +1.6(-5- -15)
C. 6 the temperature of one lb of that
= 2.18 kw
Refrigerating Capacity D. 4.5 * substance one degree Fahrenheit is:
= 31.23 kw X ton o{re{ A . Specific heat
29. Water at 55°C is cooled in a cooling
3.516 kw 32. The age limit of a horizontal return B. Btu *
= 8.88 tons of ref tower which has an efficiency of 60%.
tubular flue or cylinder boiler having a C. Latent heat
The ambient air is at 32°C dry bulb and
longitudinal lap joint and operating at a D. Relative heat
27. A refrigeration cycle having 0.18 27°C wet bulb. The heat rejected in the
pressure in excess of 0.345 MPa or 3.45
MPa (hg = 180.8 kJ/kg) and 0.8 MPa (h condenser is 900 kJ/sec. Find the
Bar gage shall be: 39. What is the most efficient
= 208.65 kJ/kg). The mass flow rate in A . 18 years thermodynamic cycle?
REFRESHER MANUAL 2nd Edition by JAS TORDILLO
REFRESHER MANUAL 2nd Edition by JAS TORDILLO
36 - 12 1 Day 36 - Solution
Day 37 - EXAM I 37 - 1
A. Carnot* C. Remains constant Power and Industrial Plant Engineering
B. Diesel D. Internal energy have no relation to
C. Rankine the vapor compression process 6. A 75 MW stationary gas turbine
D. Brayton DAY 37 - EXAM power plant has air enters the
46. What do you call the weight of the. compressor at 100 kPa, 300 °K
40. The component in the refrigeration 1. An ammonia refrigeration compressor temperature and pressure ratio of 10.
column of air above the earth's surface?
cycle that is used to store refrigerant is A. Air pressure takes its suction from the evaporator, at The turbine inlet temperature is 1650°K.
called the B. Aerostatic pressure a temperature of -20 deg F (-28.9 degC) Using the standard Brayton cycle,
A. Receiver * C. Wind pressure and a quality of 95%. The compressor determine the back-work ratio of the gas
B. Evaporator discharges at a pressure of 100 psi turbine power plant.
D. Atmospheric pressure*
C. Metering device (689.5 kPa), liquid ammonia leaves the A. 0.255
D. Compressor condenser at 50 deg F (10 deg C). B. 0.879
47. A reactor which uses slow moving
neutrons. Properties of Ammonia: c. 0.351
41. Which defrost method is commonly A. thermal reactor* =
At 689.5 kPa (tsat 13.3 deg C) D. 1.061
used in small commercial . applications B. intermediate reactor =
h 1568 =
s 5.7676
where ice forms on the evaporator C. slow reactor At 10 deg C, hf = 246.531 7. Air enters a gas turbine power plant
surfaces? D. slow speed reactor At -28.9 deg C by interpolation at 1 bar and 30°C with a volume flow rat
A. Electric hf = 68.5 sf= 0.49366 of 5 m3/s. The compressor pressure
B. Hotgas * 48. . A geothermal power plant which hg = 1424.5 sg = 6.0452 ratio is 10 and its isentropic efficiency is
C. Off-cycle utilizes a heat exchanger to transfer Find the heat absorbed by the 80%. The turbine inlet pressure and
D. All of these heat from a primary working fluid to a evaporator in kJikg. temperature are 1000 kPa and 1300°C,
secondary fluid which is used to power A. 1356.7 respectively. The turbine isentropic
42. The type of compressor that uses or move a turbine: B. 1110.2 efficiency is 85% and the exit pressure
ring valves is a A. binary cycle * c. 1259.8 is 100 kPa. Determine the thermal
compressor. B. double cycle D. 1010.2 efficiency of the cycle in percent.
A. Screw C. dual cycle A. 29.5%
B. Centrifugal D. none of these 2. In problem No. 1, Find the heat B. 52.9%
C. Roller rejected to the condenser kJikg. c. 39.5%
D. Reciprocating * 49. Ratio of the average load to the A. 1568.5 D. 92.5%
peak load over a period of time B. 1356.5
43. The type of compressor that A Demand factor · C. 1321.5 8. A fuel oil having a 26 deg API rating,
incorporates a volute is a _ _ _ __ B. Use factor D. 1125.5 determine the lower heating value of the
compressor. C. Diversity factor fuel.
A. Roller D. Load factor* 3. In problem No. 1, Find the COP. A. 44,759.60 kJ/kg
B. Screw A. 5.25 B. 40,999.90 kJ/kg
C. Centrifugal * 50. The quantity of energy absorbed by B. 5 c. 42,005.75 kJikg
D. Reciprocating a unit mass of refrigE)lrant in the c. 4.25 D. 35,768.25 kJ/kg
evaporator is called. D. 4
44. In order to increase the back A. Mass flow rate 9. Determine the air-fuel in kg/kg of fuel
pressure of a boiler: B. Refrigeration effect * 4. In problem No. 1, Find the oil C12Hao. Assume complete
A. Increase the boiler pressure C. Capacity horsepower per ton of refrigeration. combustion with 20% excess air.
B. Close the atmospheric line D. Volume flow rate A. 0.987 A. 18.54 kg/kg
C. Open the back pressure valve B. 1.044 B. 15.4 kg/kg
D. Close the back pressure valve * c. 0.897 C. 14.54 kg/kg
D. 1.404 D. 25.65 kg/kg
45. During compression process, the
internal energy of the refrigerator vapor: 5. Determine the quality of refrigerant 10. An otto engine has a clearance
A. Increases* upon entering to the evaporator. volume of 8%. The heat rejected is 400
B. Decreases A. 10.5% kJ/s, determine t~e power produce by
B. 12.5% the engine in kw.
C. 11.5% A 334.10 kw
D. 13.1% B. 633.1 kw
REFRESHER MANUAL 2nd Edition by JAS TORDILLO
REFRESHER MANUAL 2nd Edition by JAS TORDILLO
37 - 2 I Day 37 - EXAM Day 37 - EXAM I 37 - 3
Power and Industrial Plant Engineering
C. 534.10 kw engine if the system rejects 28.8 kJ/sec
D. 733. 1 kw of heat. · A. 0.00923 kg/kg A Ammonia
A 21 .2 kJ/sec B. 0.00239 kg/kg B. Freon
11. A steam plant operates with an initial B. 28.8 kJ/sec c. 0.00329 kg/kg C. Air
pressure of 1.7 MPa and 370°C C. 50 kJ/sec D. 0.00392 kg/kg D. Water
temperature and to a condenser at 170 D. 500 kJ/sec
kPa. If the turbine efficiency and boiler 19. Calculate the humidity ratio of a 25. In vapor compression cycle,
efficiency are 75% and 85%, Problems 15 to 16 moist air at 65% RH and 34°C db when condensation occurs at
respectively, determine the cycle An air conditioning plant with a capacity the barometric pressure is 101.3 kPa. A Constant volume
thermal efficiency. of 400 kw of refrigeration has an From steam table at 34 deg C, p = 5.318 B. Constant pressure
Steam Properties: evaporating temperature and kPa. C. Constant entropy
At 1.70 MPa & 370°C: condensing temperature of 3 deg C and A. 0.'09721 k/kg D. Constant enthalpy
h = 3187.1 kJ/kg 37 deg C, respectively. The plant is B. 0.07192 k/kg
S = 7. 1081 kJ/kg-°K using Refrigerant 12. · c. 0.01297 klkg 26. In vapor compression cycle,
At 170 kPa: R-12 Properties: D. 0.02197 k/kg expansion occurs at
hf = 483.20 sf = 1.4752 At 3 deg C: A. Constant volume
hfg = 2216.0 sfg = 5.7062 hf = 202.78 kJ/kg 20. In problem No. 19, deter mine the B. Constant pressure
A. 25.6% hg =352. 755 kJ/kg enthalpy. C. Constant entropy
B. 12.1% vg = 50.47 Ukg A. 90.53 I<J/kg D. Constant enthalpy
c. 52.6% At 37 def C: hf = 235.503 kJ/kg B. 82.90 kJ/kg
D. 35.6% c. 85.35 kJ/kg 27. An adiabatic saturation process can
15. What will be the mass of flash gas D. 78.92 kJ/kg be produced with a
12. Given the absolute pressure of 150 per kg of refrigerant circulated? A. chart
kPa, determine the gage pressure in A. 0.119 21 . The ratio of the current partial B. Sling psychrometer
mm Hg if barometric pressure is 755 B. 0.728 pressure of the water vapor in the C. ordinary thermometer
mm Hg. c. 0.218 atmosphere to the saturation pressure D. wet bulb temp
A. 755 mm Hg D. 0.782 of the water vapor at the current DB
B. 101 mm Hg depression is called: 28. The law states that the total volume
C. 360 mm Hg 16. What is the mass of Refrigerant 12 A. DP temperature of a mixture of non-reacting gases is
D 370 mm Hg must be circulated per second? B. Relative humidity equal to the sum of the partial volumes.
A. 3.411 kg C. DB depression A Amagat's law
13. A certain gas undergoes a process, B. 4.411 kg D. WB temperature B. Fermat's law
initially at 500 kPa and 150 li, to a final c. 2.411 kg C. Boyle's law
state at 200 kPa and 300 li. During the D. 5.411 kg 22. The COP of a refrigeration process D. Dalton's law
process. the enthalpy increases 50 kJ . - ----with subcooling.
Assuming constant specific heats, 17. How much heat is needed to A. Decreases 29. Above the critical temperature, the
determine the specific heats at constant completely vaporize 100 kg of ice at - B. Remains the same substance will be
pressure Cp in kJ per kg per deg K. 10°C if the pressure held constant at C. Increases A water vapor
Assume Cv = 2.506 kJ/kg-deg K. 200 kPa. (Latent heat of fusion at 200 D. None of these B. liquid
A. 1. 927 kJ/kg-deg K kPa = 320 kJ/kg. Latent heat of C. a gas
B. 2.506 kJ/kg-deg K vaporization at 200 kPa = 2202 kJ/kg). 23. A thermodynamic cycle in which D. mixture
C. 2.2 15 kJ/kg-deg K A. 296.1 MJ refrigerant is compressed and then
D. 4.187 kJ/kg-deg K B. 340.5 MJ vaporized and condensed to cool the 30. A measure of the energy that is no
C. 320.5 MJ space or cabinet. longer available to perform useful work
14. A carnot engine requires 50 kJ/sec D. 345.0 MJ A Vapor-compression cycle within the current environment.
of heat from the hot source. The B. Refrigeration cycle A enthalpy
. temperature of the sink is 15 deg C and 18. Calculate the moisture content of a C. Rankine cycle B. heat loss
temperature of the hot source is 500 moist air at 20 deg C db, 15 deg C wb, D. Carnot cycle C. internal energy
deg K. Using the given temp~ratures, and 95 kPa barometric pressure. D. entropy
determine the power produced by the From Steam table at 15 deg C wb: 24. What refrigerant is used in jet
P = 1.051 kPa. cooling system?

REFRESHER MANUAL 2nd Edition by JAS TORDILLO REFRESHER MANUAL 2nd Edition by JAS TORDILLO
(
37 - 4 I Day 37 - EXAM Day 37 - EXAM I 37 - 5
31 . Convert mechanical energy into fluid
Power and Industrial Plant Engineering
C. The total pressure in the fluid drops
energy, increasing the energy above the vapor pressure 44. An upward force that acts on all C. one is greater
possessed by the fluid. D. The net pressure in the fluid drops objects that are partially or completely D. one is lesser
A. Turbine below the atmospheric pressure submerged in a fluid.
B. Pumps
A. weight in air
C. Impeller 38. Efficiencies of impulse turbines are B. weigh in water
D. Draft tube in the range of C. object's mass DAY 37 -ANSWER KEY
A. 40-50% D. Buoyant force
32. Reciprocating and rotary pumps are 1. 8 16. A 31.B46. C
B. 80 to 90% ;?.. C 17. A 32. A 47. B
A. · Positive displacement pumps C. 60-75% 45. Water lost in small droplets and 3. A 1B. A 33. 0 48. B
B. Net Positive pumps D. more than 100% carried away by the air flow. 4. C 19. 0 34. C49. 0
C. Dynamic displacement pumps A. water vapor
5. 0 20. A 35. A 50. A
D. Dynamic motion pumps 39. Also known as duct fans, generally 6. C 21 . B 36. 0
B. vapor loss 7. A 22. C 37. B
move air against less than 3 in of water C. windage loss B. C 23. A 38. B
33. Plungers pumps are only single (750 Pa). D. friction loss 9. A 24. 0 39. A
acting and are suitable up to A. Tube axial fans 10. 025. 8 40.C
approximately B. Vane axial fans 11. B 26. 0 41. 0
46. A device that passes air through a 12.027. B 42. B ·
A. 500 MPa C. blowers dense spray of circulating water. 13. A 28. A 43. 0
B. 110 MPa D. vacuum fans 14. A 29. C
A. condenser 44. 0
C. 350 MPa B. evaporative 15. c 30. 0 45. c
D. 70 MPa 40. The actual power delivered to a fan C. air washer
from its motor is D. air cooler
34. The net energy actually transferred A. Air power
to the fluid per unit time. B. Total power 47. A dimensionless ratio of a fluids
A. Brake power C. Brake power density to some standard reference
B. Motor Power D. Energy power density.
C. Hydraulic power
A. specific weight
D. Friction Power 41 . If the velocity pressure decreases B. Specific gravity
due to an increase in duct area or a C. constant
35 Pumps suitable for adding low branch takeoff, the static pressure will D. factor
pressures at high fluid flow rates. increase. This increase is known as
A. Axial-flow pumps A. Dynamic regain 48. Attractive force between molecules
B. Centrifugal B. pressure change of different substances.
C. Parallel-flow pumps C. Nozzle like A. cohesion
D. Rotary pumps D. Static regain B. adhesion
C. saturation
36. The speed at which a homologous 42. A grille used to guide the direction D. evaporation
pump would have to turn in order to of projected air.
deliver one gallon per minute at one foot A. Adjuster 49. The ratio of the actual humidity
total added head. B. Diffuser ratio to the to the saturated humidity
A. Rotational speed C. Nozzle line ratio at the same temperature and
B. Vertical speed D. Blade guide pressure.
C. Normal speed
A. Degree of saturation
D. Specific speed 43. The draft tube is used to keep the B. Saturation ratio
turbine up to _ _ above the tailwater C. Percentage humidity
37. Cavitation will occur when surface. D. all of these
A. The net pressure in the fluid drops A. 10m
above the vapor pressure B. 1m 50. When the air is saturated, all three
B. The net pressure in the fluid drops C. 15m temperatures are
below the vapor pressure D. 5m A. Equal
B. unequal

REFRESHER MANUAL 2nd Edition by JAS TORDILLO REFRESHER MANUAL 2nd Edition by JAS TORDILLO
37 - 6 I Day 37 - Solution Day 37 - Solution I 37 - 7
Power and Industrial Plant Engineering
DA'I 37 - SOLUTION COP= Q A _ 1110. 17
We - 211:3 = 5. 25 T3 = (p3)k-1Jk T2
1. An ammonia refrigeration compressor , T4 P4 303 = (10)1.4-1/1.4
takes its suction from the evaporator, at 4. In problem No. 1, Find the
a temperature of -20 deg F (-28.9 degC} horsepower per ton of refrigeration. 1650 = (10)1.4- 1/1.4 T2 = 585°K
and a quality of 95%. The compressor A. 0.987 r..
discharges at a pressure of 100 psi B. 1.044
(689.5 kPa}, liquid ammonia leaves the c. 0.897 " T4 = 854.6°K T3 = (p3)k· t;k
condenser at 50 deg F (10 deg C). D. 1.404 T4 P4
Properties of Ammonia: Back-work ratio
At 689.5 kPa (tsat = 13.3 deg C) Q A = m(h1 - h4} _ We ""j.(Tz·T, ) 579.2-300 1573 = (10)1.4 - 1/1.4
h = 1568 s = 5.7676 3.516 = m( 1110.1 7} - w, = ""j.(T3-r.J = 16SO-BS4.6 T4
At 10 deg C, hf = 246.531 m = 0.0031 67 kg/sec
At -28.9 deg C by interpolation = 0.351 T4 = 814.7°K
hf = 68.5 sf= 0.49366 g.oo3 t67(2I1.3l hp
hg = 1424.5 sg = 6.0452 We = 0.746 = 0.897 _HP
We = mCp(Tz·T,) =5.75(1.0)(585- 303)
Find the heat ·absorbed by the ec 0.80
QA ton Ton
evaporator in kJ/kg. = 2026.87 kw
A . 1356.7 5. Determine the quality of refrigerant
B. 1110.2 * Wt = mCp(TJ- T4)et
upon entering to the evaporator. = 5.75(1 .0)(1573- 814.73)(0.85)
C. 1259.8
A. 10.5% w, = 3706 kw
D. 1010.2 B. 12.5%
C. 11.5% Heat added in the combustor
h1 = hf + x(hfg)
h1 = hf + x(hg - hf)
D. 13.1% *
300°K
=
= mCp(T3- T4) 5.75(1 .0)(1573- 585)
h1 = 68.5 + 0.95(1424.5 - 68. 5)
h3 = h4 = hr + x hrg 100 kPa = 5681 kw
= 1365.7 kJ/kg P2/P, = 10
245.53 = 68.5 + x(1424.5- 68.5}
X = 13.1%
Cycle Efficiency = w,-wc = 3706-5681
Q,A
2026.87
QA: h1- h4
QA = 1356.7-246.531 7. Air enters a gas turbine power plant = 29.5%
6. A 7'5 MW stationary gas turbine
QA = 11 10.17 kJ/kg at 1 bar and 30°C with a volume flow rat
power plant has air enters the
of 5 m3 /s. The compressor pressure
compressor at 100 kPa, 300 °K 8. A fuel oil having a 26 deg API rating,
2. In problem No. 1, Find the heat ratio. is 10 and its isentropic efficiency is determine the lower heating value of the
temperature and pressure ratio of 10.
rejected to the condenser kJ/kg. 80%.· The turbine inlet pressure and fuel.
The turbine inlet temperature is 1650°K.
A. 1568.5 temperature are 1000 kPa and 1300°C, A. 44,759.60 kJ/kg
Us ing the standard Brayton cycle,
B. 1356.5 respectively. The turbine isentropic B. 40,999.90 kJ/kg
determine the back-work ratio of the gas
c. 1321.5. turbine power plant.
efficiency is 85% and the exit pressure c. 42,005.75 kJ/kg *
D. 1125.5 is 100 kPa. Determine the thermal D. 35,768.25 kJ/kg
A. 0.255
efficiency of the cycle in percent.
B. 0.879
OR= hz- h3 A. 29.5%"
c. 0.351 . Qh = 41,130 + 139.6(0 API)
QR = 1568-246.531 = 132 1.47 kJ/kg
D. 1.061
B. 52.9%
c. 39.5% =
41 ,130 + 139.6(26)
= 44,759.60 kJ/kg
3. In problem No. 1, Find the COP. D. 92.5%
A. 5.25" !! = (~)k- 1Jk
T1 Pt
PN1 = mRT1
0
API = ~- 131.5
B. 5 SG@1S.6
c. 4.25 Tl 1 1 (100)(5) = m(0.287)(303)
D. 4 3oo = ( 10)1.' - /u m = 5.75 kg/s 26 = ~-131.5
SG ,/o~SSS
We = h2- h1 = 1568 -1 356.7
= 211.3 kJ/kg
Tz = 579.2°K ?i = (Pz)k-1Jk H2 = 26 -15(SG) = 26 -15(0.898)
rl Pl = 12.53%
REFRESH ER MANUAL 2 nd Edition by JAS TORDILLO REFRESHER MANUAL 2nd .Edition by JAS TORDILLO
37 ~ 8 1 Day 37 - Solution . Day 37 - Solution I 37 - 9
·Power and Industrial Plant Engineering
Qh- Q~ = 9(H2)(2442) 11 . A steam plant operates with an initial
44,759.60- QL : 9(0.1253)(2442) pressure of 1.7 MPa and 370°C Q A :; h.1 -h 4 = 3187.1- 484.73 temperature of the sink is 15 deg C and
Lower Heating Value, temperature and to a condenser at 170 nb 0.85 temperature of the hot source is 500
QL = 42,005.75 kJ/kg kPa. If the turbine efficiency and boiler = 3179.25 kJ/kg deg K. Using the given temperatures,
efficiency are 75% and 85%, determine the power produced by the
9. Determine the air-fuel in kg/kg of fuel respectively, determine the cycle Cycle Efficiency = w,-wP = 387.36-1.53
engine if the system rejects 28.8 kJ/sec
oil C,2H30. Assume complete thermal efficiency. QA 3179.25
of heat.
combustion with 20% excess air. Steam Properties: A. 21.2 kJ/sec *
A. 18.54 kg/kg • At 1.70 MPa & 370°C: = 12.1%
B. 28.8 kJ/sec
B. 15.4 kg/kg h = 3187.1 kJ/kg C. 50 kJ/sec
c . 14.54 ~g/kg S = 7.1081 kJ/kg-°K 12. Given the absolute pressure of 150
kPa, determine the gage pressure in D. 500 kJ/sec
D. 25.65 kg/kg At 170 kPa:
hf = 483.20 sf = 1.4752 mm Hg if barometric pressure is 755
mm Hg. W: QA-QR :; 50-28.8
Fuel + Air -+ Products of Combustion hfg = 2216.0 sfg :; 5. 7062 = 21 .2 kJ/sec
A. 25.6% A 755 mm Hg
B. 12.1% * B. 101 mm Hg
C12H3o + 19.502 + 19.5(3.76)N2 Problems 15 to 16
-+ 12C02 + 15H20 + 19.5(3.76)N2 c. 52.6% C. 360 mm Hg ·
D. 370 mm Hg * An air conditioning plant with a capacity
D. 35.6% 1.70 MPa of 400 kw of refrigeration has an
Air-fuel ratio with 20% excess air: 37o•c evaporating temperature and
.----------, P(abs) = P(gage) + P(atm)
condensing temperature of 3 deg C and
- [19.5 +19.5(3.76)](1.20)(28.97) 150(~)
101.325
= P(gage) + 755 37 deg C, respectively. The plant is
- (1)[12(12)+30(1))
P(gage) = 370.1 mm Hg using Refrigerant 12.
kg air R-12 Properties:
= 18.54 kg fuel 13. A certain gas undergoes a process, At 3 deg·c:
initially at 500 kPa and 150 li, to a final hf = 202.78 kJ/kg
state at 200 kPa and 300 li. During the hg =352.755 kJ/kg
10. An otto engine has a clearance process, the enthalpy increases 50 kJ. vg = 50.47 Ukg
volume of 8%. The heat rejected is 400 Assuming constant specific heats, At 37 def C: hf = 235.503 kJ/kg
kJ/s, determine the power produce by determine the specific heats at constant
the engine in kw. pressure Cp in kJ per kg per deg K. 15. What will be the mass of -flash gas
A 334.10 kw Assume Cv = 2.506 kJ/kg-deg K. per kg of refrigerant circulated?
B. 633.1 kw S, = Sz A. 1.927 kJ/kg-deg K * A. 0.119
c . 534. 10 kw S, = (Sr + XzSr9)2 B. 2.506 kJ/kg-deg K B. 0.728
D. 733.1 kw• 7.1081 = 1.4752+xz(5.7062) C.• 2.215 kJ/kg-deg K c. 0.218.
X2 = 0.987 D. 4.187 kJ/kg-deg K D. 0.782
fk = 71+< --~
0.08
= 13.5
h2 = hr + Xz(hrg) 6H = 6U + 6W h3 = h4 = hf + xhfg
1 = 483.20 + 0.987(2216)
e = 1- F1 = 1 - (-13.5)'··-1
-- - = 64.7 oVo
= 2670.6 kJ/kg
Hz - H, = 6U + [PzV2- P,V,] h3 = h4 = hf + x(hg -hf)
50 = 6U + [200(0.3)- 500(0.15)] 235.503 = 202.78 + x(352.755- 202.78)
e : 6U = 65 kJ X= 0.218
QA-QR WT = (h, - h2)nt
QA = (3187.1 - 2670.6)(0. 75)
= 387.36 kJ/kg l!H = mCpllt =~ = ~ : k 16. What is the mass of Refrigerant 12
0.647 : QA-400 l!U mC11l!t Cu 6S must be circulated per second?
QA A. 3.411 kg*
Wp = V3(P~- P3) Cp B. 4.411 kg
-=k
QA = 1133.1 kw = ~1700
1000
-170) = 1.53 kJ/kg cp C. 2.411 kg
cp = c.(kl = 2.5o6(m D. 5.411 kg
W: QA-QR : 11 33.1-400 Wp = h4- h3 = 1.927 kJ/kg-deg K
= 733.1 kw 1.53 =
h4- 483.20 a. . = m(h, - h~)
h4 = 484.73 kJ/kg 400 = m(352.755- 235.503)
14. A carnot engine requires 50 kJ/sec
m = 3.411 kg
of heat from the hot source. The
REFRESHER MANUAL 2nd Edition by JAS TORDILLO REFRESHER MANUAL 2nd Edition by JAS TORDILLO
:;-,

37 - 10 I Day 37 - Solution Day 37 -Solution I 37 - 11


Power and Industrial Plant Engineering
17. How much heat is needed to
completely vaporize 100 kg of ice at - w = 0.622 (_____&___) 25. In vapor compression cycle, C. Impeller
10°C if the pressure held constant at P-Pv
condensation occurs at D. Draft tube·
200 kPa. (Latent heat of fusion at 2q_0 A. Constant volume
kPa = 320 kJ/kg. Latent heat of = 0.622 ( 3.4567 )
101.3-3.4567 B. Constant pressure* 32. Reciprocating and rotary pumps are
vaporization at 200 kPa = 2202 kJ/kg). C. Constant entropy A. Positive displacement pumps *
A. 296.1 MJ * w= 0.02197
B. Net Positive pumps
D. Constant enthalpy
B. 340.5 MJ C. Dyn.amic displacement_pumps
C. 320.5 MJ 20. In problem No. 19, determine the
enthalpy. 26. In vapor compression cycle, D. Dynamic motion ·pumps
D 345.0 MJ expansion occurs at
A. 90.53 kJ/kg *
A. Constant volume 33. Plungers pumps are only single
Heat to vaporize Ice: B. 82.90 kJ/kg
C. 85.35 kJ/kg B. Constant pressure acting and are suitable up to
Q =m[Cp1(lt- !1) + hL + Cp2(b - It) +hv] C. Constant entropy approximately
= 100[2.093(0+10) + 320 D. 78.92 kJ!kg
D. Constant enthalpy* A. 500 MPa
+ 4.187(100 + 0) + 2202] B. 110 MPa
= 296,163 kJ =296.1 MJ h = 1.007t- 0.026 + W(2501 + 1 .84t)
27 . An adiabatic saturation process can C. 350 MPa
h = 1.007(34)- 0.026
. + 0.02197[2501 + 1.84(34)] be produced with a D. 70 MPa *
18. Calculate the moisture content of a
h = 90.53 kJ/kg A. chart
moist air at 20 deg C db, 15 deg C wb, B. Sling psychrometer * 34. The net energy actually transferred
and 95 kPa barometric pressure. to the fluid per unit time.
21. The ratio of the current partial C . ordinary thermometer
From Steam table at 15 deg C wb: D. wet bulb temp A. Brake power
P = 1.051 kPa. pressure of the water vapor in the
atmosphere to the saturation pressure B. Motor Power
A. 0.00923 kg/kg * 28. The law states that the total volume C. Hydraulic power*
B. 0 00239 kg/kg of the water vapor at tl1e current DB
depression is called: of a mixture of non-reacting gases is D. Friction Power
C. 0.00329 kg/kg
A. DP temperature equal to the sum of the partial volumes.
D. 0.00392 kg/kg 35. Pumps su itable for adding low
B. Relative humidity * A. Amagat's law •
4 C. DB depression B. Fermat's law pressures at high fluid flow rates.
Pv = Pwb-6.66x10- Ptjdb - wb) A. Axial-flow pumps *
D. WB temperature C. Boyle's law
Pv = 1.7051-6.66x10 (95)(20-15) B. Centrifugal
D. Dalton's law
Pv = 1.38875 kPa C. Parallel-flow pumps
22. The COP of a refrigeration process
w = 0.622 (~)
p- Pv
-:-___ _ _ with subcooling.
A. Decreases
29. Above the critical temperature, the
substance will be
D. Rotary pumps

B. Remains the same A. water vapor 36. The speed at which a homologous
C. Increases • B.' liquid pump would have to turn in order to
= 0.622 ( 1.38875 )
D. None of these C. a gas* deliver one gallon per minute at one foot
95-1.38875
D. mixture total added head.
w = 0.00923 A . Rotational speed
23. A thenmodynamic cycle in which
refrigerant is compressed and then 30. A measure of the energy that is no B. Vertical speed
19. Calculqte the humidity ratio of a C. Normal speed
vaporized and condensed to cool the longer available to perform useful work
moist air at 65% RH and 34°C db when D. Specific speed *
space or cabinet. within the current environment.
the barometric pressure is 101.3 kPa.
A. Vapor-compression cycle * A. enthalpy
From steam table at 34 deg C, p = 5.318 37. Cavitation will occur when
B. Refrigeration cycle B. heat loss
kPa. A. The net pressure in the fluid drops
C. Rankine cycle C. internal energy
A. 0.09721 k/kg above the vapor pressu·re
D. Carnot cycle D. entropy *
B. 0.07192 k/kg B. The net pressure in the fluid
C. 0.01297 k/kg drops below the vapor pressure *
24. What refrigerant is used in jet 31. Convert mechanical energy into fluid
D. 0.02197 k!kg * C. The total pressure in the fluid drops
cooling system? energy, increasing the energy
possessed by the fluid. above the vapor pressure
Pv = RH (Psat) =0.65 (5.318) A. Ammonia
B. Freon A. Turbine D. The net pressure in the fluid drops
= 3.4567 kPa B. Pumps • below the atmospheric pressure
C. Air
D. Water*
REFRESHER MANUAL 2nd Edition by JAS TORDILLO 'REFRESHER MANUAL 2nd Edition by JAS TORDILLO
37 -12 I Day 37- Solution Day 38 • EXAM I 38 - 1
38. Effi~iencies of impulse turbines are C. object's mass
Power and Industrial Plant Engineering
in the range of D. Buoyant force •
6. What is the deg R equivalent of 100
A. 40-50% DAY 38- EXAM deg C?
B. 80 to 90% • 45. Water lost in small droplets and
A. 212°R
C. 60-75% carried away by the air flow. 1. Given a barometric pressure of 755 B. 460°R
D. more than 100% A. water vapor mmHg and its equivalent standard C. 360°R
B. vapor loss pressure is 80 kPa, find the gage D. 672°R
39. Also known as duct fans, generally C. windage loss • pressure in mm Hg.
move air against Jess than 3 in of water D. friction loss A. -155 mm Hg 7. Steam having an internal energy of
(750 Pa). B. 600 mm Hg 5000 ft-lb/lb, specific volume of 8 ftlllb
A. Tube axial fans • 46. A device that passes air through a C. 155 mm Hg and a pressure of 10 psi, what is the
B. Vane axial fans dense spray of circulating water. D. -600 mm Hg specific enthalpy of steam in ft-lb/lb?
C. blowers A. condenser
A. 16,520 ..
D. vacuum fans B. evaporative 2. An iron block whose volume is known B. 14,520
C. air washer • to be 300 cm 3 has a density of 4500 C. 15,520
D. air cooler 3
40. The actual power delivered to a fan kg/m , find the specific volume of the D. 12,520
from its motor is block in m3/kg.
A. Air power 47. A. dimensionless ratio of a fluids A. 5.55 X 10-~ 8. A room has to be maintained at 25
B. Total power density to some standard refer.ence B. 2.22 X 10'4 deg C at all times. Assum ing the
C. Brake power • density. C. 4.44 X 10'4 outside temperature of the room is -5
D. Energy power A. specific weight D. 1.11 X 10'4 deg C, the room gains heat at a rate of
B. Specific gravity •
42 kw. Using a heat pump, compute the
41 . -If the velocity pressure decreases C. constant 3. Hot water enters a heat exchanger at power required to maintain the room
due to an increase in duct area or a D. factor a temperature of 80°C and rejects 500 temperature.
branch takeoff, the static pressure will kJ/s of heat. The heat exchanger is A. 25.5 kw
increase. This increase is known as 48. Attractive force between molecules used to heat cold water entering at 26°C B. 4.22 kw
A. Dynamic regain of different substances. and leaving at 60°C, determine the c. 34.5 kw
B. pressure change A. cohesion mass flow rate of cold water in kg/s. D. 1.22 kw
C. Nozzle like a. adhesion* A. 10.5 kg/sec
D. Static regain • C. saturation B. 5.5 kg/sec 9. A tank having a capacity of 1 cu. m is
D. evclporation C. 8.5 kg/sec filled with air to a pressure of 500 kPag
42. A grille used to guide the direction D. 3.5 kg/sec when the temperature is 60 deg C. It
of projected air. 49. The ratio of the actual humidity was found out later that because of the
A. Adjuster ratio to the to the saturated humidity 4. An ideal gas at 180 kPa, 35°C and leak the pressure drops to 350 kPag
B. Diffuser • ratio at the same temperature and occupies a volu,me of 0.3 m3 rigi<;l and the temperature has decreased to
C. Nozzle line pressure. container is transferred to another 45 deg C. Determine the amount of air
D. Blade guide A. Degree of saturation container at 150 kPa and 0.42 m3 that has leaked out in kg.
B. Saturation ratio volume. Determi'ne the possible. A. 1.34 kg
43. The draft tube is used to keep the C. Percentage humidity temperature reached by the ideal gas B. 3.34 kg
turbine up to _ _ above the tailwater D. all of these • when transferred. C. 2.34 kg
surface. A. 360°C D. 4.34 kg
A. 10m 50. When the air is saturated, all three B. 250.3°C
B. 1m temperatures are C. 200.3°C 10. A vulcanizing shop is provided with
C. 15m A. Equal* D. 86.3°C a 10 m3 of compressed air tank. During
D. 5 m • B. unequal operation, air pressure in the tank
C. one is greater 5. Convert 700°R temperature increase reduces from 700 kPa to 300 kPa while
44. An upward force that acts on all D. oneislesser to deg K. the temperature remains constant at 32
objects that are partially or completely A. 188.9°K deg C. Determine the mass of air that
submerged in a fluid. · B. 388.9°K has been used.
A. weight in air C. 288.9°K A. 15.7 kg
B. weigh in water D. 488.9°K B. 35.7 kg

REFRESHER MANUAL 2nd Edition by JAS TORDILLO REFRESHER MANUAL 2nd Edition by JAS TORDI~LO
38 - 2 I Day 38 - EXAM Day 38- EXAM 138 - '3
Power and Industrial Plant Engineering
C. 25.7 k9 16. A Carnot engine requires 35 kJ/sec
D. 45.7 kg from a hot source. The engine temperature is 25°C (h = 18 W/mL-K) B. 79.4 kw
produces 15 kw of power and the and the other side is exposed to C. 44 kw
11. Steam at 2.6 MPa and 600°C (h = temperature of the sink is 26°C. What is combustion gases whose temperature is D. 5g.4 kw
3478.5 kJ/kg) enters a nozzle the temperature of the hot source in °C? 1200°C (h = 12 W/m2-K). Given a heat
undergoes reversible adiabatic process A 150.25°C transfer rate of 3200 W per square 26. A fuel pump is delivering 18 gallons
with velocity of 400 m/s. Find the B. 523.25°C meter area, determine thickness of the per minute of oil with a specific gravity of
stagnation enthalpy. C. 250.25°C wall. 0.80. The total head is 75 meters. If the
A 2558.5 kJ/kg D. 750.25°C A 40.2 em power input to the pump is 1.1 hp, what
B. 4558.5 kJ/kg B. 34.2 em is the efficiency of the pump?
c. 3558.5 kJ/kg 17. A common modification used with C. 24.2 em A. 6g.3%
D. 5558. 5 kJ/kg spark-ignition engines for aircraft D. 12.4 em B. 75% .
operation, which a steady-flow C. 72. 1%
12. There are 100 kg/min of water compressor used to compress the air 22. A multi-stage centrifugal pump D. 81.4%
enters a heating element at 5°C and before it enters the reciprocating discharges 1200 gpm of water at 1400
leaves at 40°C. Find the heat added engines. rpm with pressure head of 1200 ft. If the 27. A centrifugal pump delivers 70 liters
during the process. A Scavenger specific speeq during operation is 673 per second of water to a pressurized
A 244.25 kw B. Supercharger rpm, determine the no. of stages. tank whose pressure is 300 kPa. The
B. 344.25 kw C. Radiator A. 2-stage source of water is 10 meters below the
C. 295.25 kw D. Spark plug B. 3-stage pump. The diameter of the suction pipe
D. 775.g kw C. 4-stage is 300 mm and the discharge pipe is 250
18. A closed system that operates in D. 6-stage mm. Calculate the KW rating of the
13. Water has an absolute temperature cycles and exchanges only heat and driving motor assuming the pump
of 10.5 m of water and atmospheric work with its surroundings is known as: 23. A two-stage compressor with an efficiency to be 70%.
pressure of go· kPa. Find the gage A Carnot engine ideal intercooler receives 0.4 m 3/s of air A 28kw
pressure is meters of water. B. Heat engine or thermal engine at 100 kPa and 310°K. If the intercooler B. 45 kw
A g_1 m of water C. Stirling engine pressure is 447.2 kPa, determine the C. 40 kw
B. 3.32 m of water D. Heatpump discharge pressure in kPa . D. 50 kw
C. 5.5 m of water A 2000 kPa
D. 1.32 m of water 19. It is the model cycle or the B. 3000 kPa 28. A 2 -stage air compressor operates
theoretical prototype cycle fro C. 2500 kPa between constant pressure limits of 100
14. A Carnot engine receives 100 Btu reciprocating compression-ignition D. 266 kPa kPa and 1.125 MPa. The swept volume
of heat from a hot reservoir at 600°F and engines. of the low pressure piston is 0.1 cu.
rejects 60 Btu of heat. Calculate the A Carnot cycle 24. Calculate the logarithmic mean meter per second. Using PV1·3 = C,
temperature of the cold reservoir in deg B. Otto cycle temperature difference for a condenser determine the compressor power in kw.
F. C. Diesel cycle if the temperature difference between A 27.g2 kw
A 636°F D. Dual cycle condensing steam and water inlet is B. 22.92 kw
B. 176°F 650°C and that steam inlet and water is C. 29.72 kw
C. 455°F 20. In a compression-ignition engine .. 120°C. D. 17.92 kw
D. 100°F one of the methods to avoid detonation A. 325.7°C
during compression, with its limitation on B. 313.7°C 29. A pump delivers 500 gpm of water
15. An ideal gas at 50 psig and go°F is the compression ratio, is: C . 225.7°C against a total head of 500 ft and
heated in a closed container to 140°F. A To inject the fuel after the D. 155.7°C operating at 2000 rpm. Changes have
What is the final pressure in psia? compression process or stroke increased the total head to 800 ft and
A. 70.6 psia B. To inject the fuel before the end of 25. A pump discharges 110 liters per pump capacity to 632 gpm. At what rpm
B. 60.7 psia compression process or stroke second of water to a height of 45 should the pump be operated to achieve
C. 67.6 psia C. To lower the compression ratio meters. If the pump efficiency is 70% the new head at the same efficiency?
D. 14.7 psia D. To increase the cut off ratio and the speed of the pump is 1200 rpm, A. 2530 rpm
what is the power input in the drive B. 3434 rpm
21. One side of a furnace wall (k = 1.5 shaft? C. 3600 rpm
W/m-K) is exposed to ambient air whose A 34kw D. 2224 rpm

REFRESHER MANUAL 2nd Edition by JAS TOROILLO REFRESHER MANUAL 2nd Edition by JAS TOROILLO
38 - 4 I Day 38 - EXAM
Day 38 - EXAM I 38 - 5
30. At an average temperature of Power and Industrial Plant Engineering
an air compressor with a capacity of 10
11 0°C, hot air flows through a 10 m long 3
m /min measured at 100 kPa and 250C.
tube with an inside diameter of 60 mm. 0.50 kg/sec and at a temperature of B. 306 kw
A. 2307 kg
The temperature of the tube is 21°C 14°C. Calculate the exit temperature of c. 286 kw
B. 3307 kg water in deg C. D. 206 kw
along its entire length. Convective film C. 2703 kg
coefficient is 17.1 W/m2-°K. Determine D. 3508 kg
A. 57.9°C
the convective heat transfer from air to B. 77.9°C 43. A quantity of 75 cubic meters of
the tube. C. 87.9°C water passes through a heat exchanger
35. Air is compressed in a cylinder at a D. 97.9°C and absorbs 3,000 MJ. The exit
A 2868.78 W
pressure of 90 kPa, volume of 0.1 m3 temper'ature is 90°C. Determine the
B. 2624 W
and temperature of 200C to a final 40. A kilogram of water at 60 deg C is entrance water temperature in °C.
C. 1828.78 W
volume of 0.01 m3 and 600C Find the heated at constant pressure. The water A. 50.5°C
D. 2632 W
final pressure of air in the cylinder. is completely vaporized. What is the B. 60.5°C
A. 1023 kPa C. 70.5°C
31. Air enters a reversible compressor heat added?
B. 778 kPa D. 80.5°C
at an inlet temperature and pressure of A. 1414.5 kJ
C. 985 kPa
200C and 100 kPa, respectively. Air B. 1818.5 kJ
D. 655 kPa 44. The mass flow rate of a Freon
leaves the compressor at 950 kPa. C. 2424.5 kJ
D. 2828.5l<J refrigerant through a heat exchanger is
Compute the compressor work per unit 36. If 12 kg of air at 30°C are heated at
mass. 8.0 kg/min. The enthalpy of entry Freon
constant pressure until the absolute 41. A manufacturing plant of 5500 is 250 kJ/kg, and the enthalpy of exit
A. -175.85 kJ/kg
temperature doubles, calculate the heat square meters has 3.8 m high ceilings Freon is 60 kJ/kg. Water coolant is
B. -295.95 kJ/kg
required in kJ. Gas constant of air is and changes its inside air three times allowed to rise 10°C. Determine the
C. -265.65 kJ/kg 53.3 ft-lb/lb-R.
D. -305.25 kJ/kg per hour by maintaining a siightly water flow rate in kg/min.
A. 1616 kJ A. 36.3 kg/min
positive air pressure (100 kPa). The
B. 2626 kJ
32. An air conipressor of a pneumatic room temperature is maintained at 23°C. B. 46.3 kg/min
C. 3636 kJ C. 76.3 kg/min
system displaces air at 20 m3/min when To assist in the sizing of a heating
D. 4646 kJ D. 146.3 kg/min
the intake pressure is 10 1 kPa. Using a system it is assumed in the design
volumetric efficiency of 80%, determine conditions that the outside temperature
37. A closed rigid tank has a capacity of is -20°C. Basing the heating 45. Calculate the mass of the
the required time to compress air to a 10 3
3.. 0 m . It holds air at 500 kPa and requirements on the indoor conditions, combustion products of a diesel fuel oil
cubic meter receiver from 350 kPa to 25°C. Heat is added until the
910 kPa. determine the heating required to (C1eH3o) with 30% excess air if the mass
temperature is 440°C. Determine the compensate for the infiltration. of fuel is 30 kg.
A. 2.8 min heat added in kJ.
B. 3.9 min A . 881.5 kw A. 18.97 kg
A. 5211.40 kJ B. 169.1 kg
C 3.5 min B. 188.5 kw
B. 4211.40 kJ
D. 4.2 min C. 588.1 kw c. 569.1 kg
C. 2411.40 kJ D. 599.1 kg
D. 511.8 kw
D. 1244.40 kJ
33. An air compressor of a compressed
42. A laboratory building is to be heated 46. A mixture of 101 kPa and 23°C that
air system has an intake air of 16 cubic 38. What is the resulting pressure when and ventilated. Outside air is to be is 30% by weight C02 and 70% by
meter per minute at an intake pressure 5 kg kilogram of air at 110 kPaa and supplied to a room 10 m by 20 m at the weight N2, determine the partial
of 101 kPa and a discharge pressure of 80°C is heated at constant volume to
910 kPa. Compute the required rate of 1.00 m3/min of replacement air pressure of C02 .
380°C? A. 3.182 kPa
minimum receiver volume in cubic meter per square meter of floor space. The
A. 103.5 kPa B. 100 kPa
per minute. design conditions are 22°C inside air
B. 203.5 kPa C. 21.65 kPa
A . 10 temperature with -25°C outside air
C. 283.5 kPa D. 25 kPa
B. 2.9 temperature. The air is to be
D. 303.5 kPa
C. 8.5 pressurized in the room to a positive air
D. 1.8 pressure of 10.0 kPag to ensure that the 47. Find the air fuel ratio, in kg air per kg
39. A heat exchanger was installed room does not get contaminated from fuel, for a combustion process to which
purposely to cool 0. 75 kg of gas per the outside. How much heat would be the fuel is CsH2o with 30% excess air.
34. Determine the total mass of air second. Molecular weight is 30 and k =
required during the 5 hours operation of required to condition the air for this A. 3.76
1.33. The gas 'is cooled from 180°C to B. 10
room?
70°C. Water is available at the rate of
A. 406 kw
REFRESHER MANUAL 2nd Edition by JAS TORDILLO REFRESHER MANUAL 2nd Edition by JAS TORDILLO
/

38 - 6 I Day 38 - Solution Day 38. - Solution 138-7


C. 13
Power and Industrial Plant En9ineering
D. 17.1 DAY 38 - SOLUTION
Q = mCpllt and a pressure of 10 psi, what is the
1. Given a barometric pressure of 755 500 = m(4.187)(80-26) specific enthalpy of steam in ft-lbllb?
48. At 28°API specific gravity, what is m = 3.51 kg/sec
mmHg and its equivalent standard A. 16,520 *
the heating value of oil in kJ/kg? B. 14,520
A. 45,039 pressure. is 80 kPa, find the gage
pressure In mm Hg. 4. An ideal gas at 180 kPa. 35°C and C. 15,520
B. 46,790
A. -155 mm Hg * occupies a volume of .0.3 m3 rigid D. 12,520
C. 47,130 container is transferred to another
D. 48,390 B. 600 mm Hg
C. 155 mm Hg container at 150 kPa and 0.42 m 3 h = u + pV = 5000 + 10(144)(8)
D. -600 mm Hg volume. Determine the possible = 16,520 ft-lbllb
49. A mixture contains 5 moles of
temperature reached by the ideal gas
oxygen, 6 moles of Nitrogen, and 7 8. A room has to be maintained at 25
P(abs) = 80 kPa X 760mmHg
when transferred.
moles of carbon dioxide at 120°C and 101.325 kPa A. 360°C deg C at all times. Assuming the
250 kPa. What is the partial pressure of = 600 mm Hg outside temperature of the room is -5
B. 250.3°C
carbon di~xide? C. 200.3°C deg C, the room gains heat at a rate of
A. 250 kPa P(abs) = P (gage) + P(atm) D. 86,3°C * 42 kw. Using a heat pump, compute the
B. 140 kPa 600 = F!(gage) + 755 power required to maintain the room
C. 101 kPa P(gage) =-
155 mm Hg P1 V1 PzVz temperature.
D. 97.2 kPa
2. An iron block whose volume is known
r;-=r; A. 25.5 kw
B. 4.22 kw *
50. In an air standard diesel cycle, to be 300 'cm 3 has a density of 4500 C. 34.5 kw
compression starts at 100 kPa· and -303 kg/m3 , find the spedfic volume of the D. 1.22kw
35 + 273 'l'z
block in m /k~.
K. The compression ratio is 20 to 1. 3

The maximum cycle temperature is A 5.55 X 10·


Tz = 359.33°K = 86.33°C Thermal Eff = ~ = TH-TL
2200 K. Determine the thermal B. 2.22 X 10-4. QA TH
efficiency. C. 4.44 x 10-c
A. 46% D. 1.11 X 10-c
5. Convert 700°R temperature increase w 25-(-5)
B. 64% to deg K. 42 = 25 + 273
C. 69% A. 188.9°K
V· = .!,=_1_
D. 70% p 4500 B. 388.9°K * W = 4.22 kw
, C. 288.9°K
= 2.22 x 10·~ 3
m /kg D. 488.9°K 9. A tank having a capacity of 1 cu. m is
(c-1 ) filled with air to a pressure of 500 kPag

DAY 38- ANSWER KEY


3. Hot water enters a heat exchanger at
a temperature of 80°C and rejects 500
.
A0 R = A°F = 700°R when the temperature is 60 deg C. It
was found out later that because of the
1. A 16. C 31.C46 ..C kJ/s of heat. The heat exchanger is ll°C = ~9 A°F = ~9 (700) = 388 .9°C leak the pressure drops to 350 kPag
2. B 17 ..9 32. C47. 0 used to heat cold water entering at 26°C
3. 0 18. B 33. 048. A and the temperature has decreased to
and leaving at 60°C, determine the 45 deg C. Determine the amount of air
4. 0 19. C 34.049.0 6. What is the deg R equivalent of 100
5. B 20. A 35. A 50. B mass flow rate of cold water in kg/s. that has leaked out in kg.
deg C? ·
6. 0 21 . B 36. C A 10.5 kg/sec A. 212°R A. 1.34 kg*
7. A 22. C 37. A B. 5.5 kg/sec
8. B 23. A 38. B 8'. 460°R B. 3.34 kg
C. 8.5 kg/sec C. 2.34 kg
9. A 24. B 39. A c. 360°R
10. 025.0 40. C D: 3.5 kg/sec * D. 672°R * D. 4.34 kg
11 . C26. 0 41 . A
12. A27.C 42. 0
13 028. A 43.0 °F = 5~·c + 32= ~(lOO)+
5
32=212°F
.
14 B29. A 44. A
15. A 30. A 45. 0 25•c
so•c ('1 m (kg/s)
0
R = 212 + 460 = 672°R
(1;
7. Steam having an internal energy of
5000 ft-lb/lb, specific volume of 8 ft3 /lb

REFRESHER MANUAL 2nd Edition by JAS TORDILLO REFRESHER MANUAL 2nd Edition by JAS TORDILLO
38 - 8 1 Day 38 -Solution Day 38 - Solution I 38 - 9
Power and Industrial Plant Engineering
PV = Q1RT Vl Vl
(500 + 101.325)(1) hl + 2 = h2 + 2 15. An ideal gas at 50 psig and 90°F is 18. A closed system that operates in
= m1(0.287)(60 + 273) heated in a closed container to 140°F. cycles and exchanges only heat and
m1 = 6.29 kg 2
(400) = h2 + 0 What is the final pressure in psia? · work with its surroundings is known as:
(350 + 101.325)(1) 3478.5 + 2(1000) A. 70.6 psia * A. Carnot engine
= m2(0.287)(45 + 273) B. 60.7 psia B. Heat engine or thermal engine *
m2 = 4.945 kg h2 = stagnation enthalpy C. 67.6 psia C. Stirling engine
= 3558.5 kJ/kg D. 14.7psia D. Heat pump
Solving for the kg of air leaked out:
mL = m1 - m2 = 6.29- 4.945 12. There are 100 kg/min of water pl p2 19. It is the model cycle or the
= 1.34 kg enters a heating element at 5°C and = theoretical prototype cycle fro
T1 T2
leaves at 40°C. Find the ·heat added recipr,ocating compression-ignition
10. A vulcanizing shop is provided with
a 10 m3 of compressed air tank. During
during the process. so+ 14.7 p2 engines.
A. 244.25 kw * 90 + 460 140 + 460 A. Carnot cycle
operation, air pressure in the tank B. 344.25 kw B. Otto cycle
reduces from 700 kPa to 300 kPa while C. 295.25 kw C. Diesel cycle *
P2 = 70.6 psia
the temperature remains constant at 32 D. 775,9 kw D. Dual cycle
deg C. Determine the mass of air that
16. A Carnot engine requires 35 kJ/sec
has been used. 100 20. In a compression-ignition engine,
A. 15.7kg Q = mCpt.t = 60 (4.187)(40- 5) from a hot source. The engine
produces 15 kw of power and the one of the methods to avoid detonation
B. 35.7 kg Q = 244.25 kw
temperature of the sink is 26°C. What is during compression, with its limitation on
C. 25.7 kg the compression ratio, is:
13. Water has an absolute temperature the temperature of the hot source in °C?
D. 45.7 kg* A. To inject the fuel after the
of 10.5 m of water and atmospheric A. 150.25°C

(--
compression process or stroke*
-r B. 523.25°C

r.._~~~
pressure of 90 kPa. Find the gage
C. 250.25°C * B. To inject the fuel before the end of
700 kPa 300 kPa pressure is meters of water.
32•c 32•c D. 750.25°C compression process or stroke
A. 9.1 m of water
\ 10~(air) 10m3 (air) C. To lower the compression ratio
B. 3.32 m of water
Ill: \.q/cP C. 5.5 m of water W = QA-QR D. To increase the cut off ratio
0 . 1.32 m ofwater* 15 : 35- QR
QR = 20 kw 21. One side of a furnace wall (k = 1.5
PV = mRT W/m-K) is exposed to ambient air whose
P(abs) = P(gage) + P(atm)
700(10) = m1(0.287)(32 + 273)
m, = 79.96 kg 10.5 = P(gage) + ~
~ = 1_ Tt temperature is 25°C (h = 2
18 W/m -K)
9.81 0A TH and the other side is exposed to
P(gage) = 1.32 m of H20 combustion gases whose temperature is
300(10) = m2(0.287)(32 + 273) 15 (26 + 273) 1200°C (h = 12 W/m 2-K). Given a heat
m2 = 34.27 kg - = 1 - -'------
14. A Carnot engine receives 100 Btu 35 TH transfer rate of 3200 W per square
of heat from a hot reservoir at 600°F and TH = 523.25°K = 250.25°C meter area, determine thickness of the
m = m, - m2 = 79.96-34.27 rejects 60 Btu of heat. Calculate the wall.
m = 45 7 kg temperature of the cold reservoir in 17. A common modification used with A. 40.2 em ·
degF. spark-ignition engines for aircraft B. 34.2 em*
11. Steam at 2.6 MPa and 600°C (h = A. 636°F operation, which a steady-flow C. 24.2 em
3478.5 kJ/kg) enters a nozzle B. 176°F * compressor used to compress the air D. 12.4 em
undergoes reversible adiabatio process C. 455°F before it enters the reciprocating
with velocity of 400 m/s. Find the D. 100°F engines. q t l - t2
stagnation enthalpy. A. Scavenger A=1+x+1
A. 2558.5 kJ/kg 0A-QR B. Supercharger * 7li 1( n;;
- - - =TH-TL
---
B. 4558.5 kJ/kg ~ QA TH C. Radiator
C. 3558.5 kJ/kg * D. Spark plug 1200-25
D.. 5558.5 kJ/kg 100 - 60 _ (600 + 460)- TL 3200 = 1 X
1
100 - (600 + 460) 12 + 1.5 + 18
TL = 636°R = 176°F
x = 0.342 m ;:: 34.2 em
REFRESHER MANUAL 2nd Edition by JAS TOROILLO REFRESHER MANUAL 2nd Edition by JAS TOROILLO
'
38 - 10 I Day 38 - Solution Day 38 -Solution 1 38 - 11
Power and Industrial Plant Engineering
Px = .jP1P2

l
x=?
-<--> 447.2 = .J"'c1=oo=)=cp"2) Pump Work = Qwh
2 2
P2 = 2000 kPa P2 V2 - Vt
Hr = - + + z2
~
d 2g
24. Calculate the logarithmic mean J8Gal x 2
temperature difference for a condenser min 60sec x 300 (1.42) - (0. 99) 2 IO

~~
= - + +
~
if the temperature difference between

~·.
[ 9.81 2(9.81)
condensing steam and water inlet is X 3.7854J!!_ X
Gal IOOOiit Hr = 40.63 m
650°C and that steam inlet and water is
h,=12W/m-K ho = 18 W/m-K
120°C.
A. 325.7°C (
0.80 x 9.81 k~ ) (75 m)
m,
Water Power
B. 313.7°C *
=(0.070 m 3/sec)(9.81kN/m 3)(40.63 m)
= 27.90 kw
k= 1.5W/m-K C. 225.7°C
D. 155.7°C = 0.668 kw
2790
22. A multi-stage centrifugal pump Po - 0.668 = 81.4% Input Power =- - = 39.85 kw
AT1 ATz
Efficiency = Pin - (1.1)0.74 6
0.70
discharges 1200 gpm of water at 1400 LMTD = !nATt/ATz .
rpm with pressure head of 1200 ft. If the
28. A 2 -stage air compressor operates
specific speed during operation is 673 27. A centrifugal pump delivers 70 liters
650 120 between constant pressure limits of 100
rpm, determine the no. of stages. LMTD = ln6so;120 = 313.7°C
- per second of water to a pressurized
kPa and 1.125 MPa. The swept volume
A . 2-stage tank whose pressure is 300 kPa. The
of the low pressure piston is 0.1 cu.
B. 3-stage source of water is 10 meters below the
25. A pump discharges 110 liters per meter per second. Using PV 1·3 = C,
C. 4-stage * pump. The diameter of the suction pipe
second of water to a height of 45 determine the compressor power in kw.
D. 6-stage is 300 mm and the discharge pipe is 250
meters. If the pump efficiency is 70% A. 27.92 kw *
mm. Calculate the KW rating of the
1200 and the speed of the pump is 1200 rpm, B. 22.92 kw
Head per stage = -- =h driving motor assuming the pump
what is the power input in the drive C. 29.72 kw
n efficiency to be 70%.
shaft? D. 17.92 kw
A. 28 kw
A. 34 kw
N, = specific speed = NjQ
ft3/4 B. 79.4 kw
B. 45 kw
c. 40 kw * Px = J11-P2 j iOO(ll25)
C. 44 kw D. 50 kw = 335.4 kPa
673 = 14 00~1200 D. 69.4 kw*
m3
(ft)3/4
Q = 0.07- Po = Compressor Power by Double
Pump Work = Qwh sec Staging ·

2nPn -11V1'[(PxP )"~!


h = 300ft

300 = 1 200
n
= (0.110)(9.81)(45) = 48.56kw

Brake Power (Pump)


V, = (:
Q
)D2
0.070 m3 I sec

(: )co.3) 2 m 2 =
1
_1]
= 0.99 m/sec
n = 4 -sl age _ PumpWork = 48.56 = 69.4 kw
- Eft 0.70 _ 0.070 m3 / sec
V2 - = 1.42 m/sec
23. A two-stage compressor with an 2 2
ideal intercooler receives 0.4 m3/s of air
at 100 kPa and 310°K. If the intercooler
26. A fuel pump is delivering 18 gallons
per minute of oil with a specific gravity of
( ; } o.25) m
.1.3) (100) (0. 1) [
1.3-1
("'.4 )?;' - ']
100
pressure is 447. 2 kPa, determine the 0.80. The total head is 75 meters. If the From Bernoulli's Equation:
discharge pressure in kPa. power input to the pump is 1.1 hp, what
A. 2000 kPa * is the efficiency of the pump? P1 V? = 27. 92 kw
B. 3000 kPa A. 69.3% - + zl + - + Hr
C. 2500 kPa d 2g
B. 75% 29. A pump delivers 500 gpm of water .
D. 266 kPa C. 72.1% P2 v22 against a total head of 500 ft and
D. 81.4% * - + z2 + - operating at 2000 rpm. Changes have
d 2g
increased the total head to 800 ft and
RE FRESHER MANUAL 2nd Edition by JAS TORDILLO REFRESHER MANUAL 2nd Edition by JAS TORDILLO
l
38 - 12 1 Day 38 - Solution
Day 38 - Solution I 38 - 13
pump capacity to 632 gpm. At what rpm Power and Industrial Plant Engineering
should the pump be operated to achieve
the new head at the same efficiency?
We= nmRT1 ( P2 )n-_
n- 1
1 Solving for the required time to 35. Air is compressed in a cylinder at a

l
n-1 [ P1 compress air:
A. 2530 rpm* pressure of 90 kPa, volume of 0.1 m3
B. 3434 rpm . volume of air compressed and temperature of 200C to a final
T 1me =
C. 3600 rpm 1.4-1 air flow rate volume of 0.01 m3 and 600C Find the
D. 2224 rpm We= 1.4(0.287X20+273) (950) 1~4 _1 final pressure of air in the cylinder.
1.4 - 1 [ 100 3 ' A. 1023 kPa *
Time= ~=3.456mm
3
B. 778 kPa
_ll =(~)2
m3 /'
We= 265.65 kJ!kg 16 . C. 985 kPa
H2 N2 mm D. 655 kPa
2 Compressor work = -265-65 kJ/kg
33. An air compressor of a compressed
500 = ( 2000) (negative work for it is an input work) . Pj_Vt P2V2
air system has an intake air of 16 cubic -=-
800 N2 meter per minute at an intake pressure TI T2
32. An air compressor of a pneumatic
Nz = 2529.8 rpm system displaces air at 20 m3/min when of 101 kPa and a discharge pressure of 90(0.1) P2(0.01)
- - = --"-'---"-
the intake pressure is 10 1 kPa. Using a 910 kPa. Compute the required 20+273 60+273
30. At an average temperature of volumetric efficiency of 80%, determine minimum receiver volume in cubic meter P2 = 1022.9 kPa
11 0°C, hot air flows through a 10 m long the required time to compress air to a 10 per minute.
tube with an inside diameter of 60 mm. cubic meter receiver from 350 kPa to A. 10
36. If 12 kg of air at 30°C are heated at
The temperature of the tube is 21°C 910 kPa. B. 2.9 constant pressure until the absolute
along its entire length. Convective film
2 A. 2.8 min c. 8.5 temperature doubles, calculate the heat
coefficient is 17.1 W/m -°K. Determine B. 3.9 min 0.1.8.
required in kJ. Gas constant of air is
the convective heat transfer from air to C. 3.5 min • 53.3 ft-lb/lb-R.
the tube. D. 4.2 min P,V, = mRT A. 1616 kJ
A. 2868.78 W * 101 (16) = m(0.287)(21 + 273) B. 2626 kJ
B. 2624W P,V = m,RT, m = 19.15 kg/min c. 3636 kJ.
C. 1828.78W (350)(10) = m 1(0.287)(21 +273) D. 4646 kJ
D. 2632W m, = 41.48 kg PN=mRJ
(91 O)V = 19.15(0.287)(21 + 273) T, = 30 + 273 = 303°K
A = heat trasnfer area = 1tDL PN = mzRT2 V = 1.775 m 3/min T2 = 2(303)' = 606°K
= 7t(0.060)(10) = 1.885 m2 (910)( 10) = m 2 (0.287)(21 +273)
mz = 107.85 kg 34. Determine the total mass of air Q = mCp(T2- T,)
Ocon = hcA(tz - t,) required during the 5 hours operation of Q = 12(1.00)(606- 303)
= 17.1(1.885)(110-21) Llm = mz -m, = 107. 85 - 41 .48 an 'air compressor with a capacity of 10 Q = 3636 kJ
3
= 2868.78W = 66.37 kg m /min measured at 100 kPa and 250C.
A. 2307 kg 37. A closed rigid tank has a capacity of
31 . Air enters a reversible compressor Volume of air compressed B. 3307 kg 3
3.0 m . It holds air at 500 kPa and
at an inlet temperature and pressure of C. 2703 kg 25°C. Heat is added until the
200C and 100 kPa, respectively. Air
= l!.m = 66.37 =
55 _3 m
3
D. 3508 kg *
p 1.2 temperature is 440°C. Determine the
leaves the compressor at 950 kPa. heat added in kJ.
Compute the compressor work per unit Solving for air flow rate: PV= mRT A. 5211.40 kJ ..
mass. 1oo( 1o) = m(o.287)(25 + 273) B. 4211.40 kJ
A. -175.85 kJ/kg v m = 11 .69 kg/min C. 2411.40 kJ
nv=-
B. -295.95 kJ/kg
c. -265.65 kJ/kg * vD Solving for the total mass required for 5
D. 1244.40 kJ

D. -305.25 kJ/kg 0.80= -


v hours operation: P,V, = mRT,
20 = 11.69 kg x 5hr x 60 min 500(3) = m(0.287)(25 + 273)
3
V = 16m /min min hr m = 17.53 kg
= 3507.7 kg Q = mCv(T2 - T,)
Q = 17.53(0. 716)(440- 25)
Q = 5211.40 kJ
REFRESHER MANUAL 2nd Edition by JAS TORDILLO
REFRESHER MANUAL 2nd Edition by JAS TORDILLO
38 -141 Day 38- Solution Day 38 - Solution 138 - 15
Power and Industrial Plant Engineering
38. What is the resulting pressure when is completely vaporized. What is the
5 kg kilogram of air at 110 .kPaa and heat added? design conditions are 22°C inside air A. 36.3 kg/min *
80°C is heated at constant volume to A. 1414.5 kJ temperature with -25°C outside air B. 46.3 kg/min
380°C? B. 1818.5 kJ temperature. The air is to be C. 76.3 kg/min
A. 103.5 kPa c. 2424.5 kJ * pressurized in the room to a positive air D. 146.3 kg/min
B. 203.5 kPa * D. 2828.5 kJ pressure of 10.0 kPag to ensure that the
C. 283.5 kPa room does not get contaminated from mwCpt.T = mF(h, - h2)
D. 303.5 kPa ~ = mCp6t + mhrg the outside. How much heat would be mw( 4.187)(10) = 8(250 - 60)
QA : 1.0(4.187)(100-60) + 1.0(2257) required to condition the air for this mw = 36.3 kg/min
p p2
2.= - (Constant Volume Process) QA : 2424.48 kJ room?
T1 T2 45. Calculate the mass of the
A. 406 kw
41. A manufacturing plant of 5500 B. 306 kw combustion products of a diesel fuel oil
110 p2
square meters has 3.8 m high ceilings C. 286 kw (C,sH3o) with 30% excess air if the mass
80 + 273 = 380 + 273 and changes its inside air three times D. 206 kw * of fuel is 30 kg.
per hour by maintaining a slightly A. 18.97 kg
P2 = 203.48 kPaa 2
positive air pressure (1 00 kPa). The Floor area = 10 x 20 = 200 m B. 169.1 kg
room temperature is maintained at 23°C. 3
Air flow ,required = 200 m /min C. 569.1 kg
39. A heat exchanger was installed
To assist in· the sizing of a heating D. 599.1 kg*
purposely to cool 0.75 kg of gas per
second. Molecular weight is 30 and k =
system it is assumed in the design Air Pressure = 10 + 101.3
conditions that the outside temperature =111.3kPa C,sH3o + (23.5)02 + (23.5)3.76Nz
1.33. The gas is cooled from 180°C to is -20°C. Basing the heating -+ (16)C0 2 + (15)H20 + (23.5)3.76N2
70°C. Water is available at the rate of
requirements on the indoor conditions, PV 111.3(200)
0.50 kg/sec and at a temperature of determine the heating required to m = C,sH30 + (23.5)(1.30)02
14°C. Calculate the exit temperature of compensate for the infiltration. RT 0.287(295) + (23.5)(1 .30)3.76N 2 -+ (16)C02
water in deg C. A. 881.5 kw * = 262.92 kg/min +(15)H20 + (23.5)(1.30)3.76N2
A. 57.9°C * B. 188.5 kw = 4.38 kg/s + (23.5)(0.30)02
B. 77.9°C C. 588.1 kw
C. 87.9°C a = mCp6T = 4.38(t.oX22-(-25))
D. 97.9°C
D. 511.8kw A 23.5(1.30) + 23.5(1.30)(3.76)
= 205.86 kw heating fi = 1
Gas Constant,
Density •. = 145.418 mols air
p IOOkPa 43. A quantity of 75 cubic meters of mol fuel
R = 8.3143 =
0.277 __!!!_ p = -= kJ water passes through a heat exchanger
30 kg- "K RT 0.287 - - (296K) mair _ 145.41(28.97)
and absorbs 3,000 · MJ . The exit
kg-K mfuel - 16(12)+30(1) = 18.97 kg
temperature is 90°C. Determine the
Cp
Cv
= k = 1.33 = 1.177 kg/m3 entraoce water temperature in °C. air/kg fuel
A. 50.5°C
Cp = 1.33 Cv
Heat required,
B. 60.5°C
malr = 18.97 kg air/kg fuel
Q = density x Cp xVolume C. 70.5°C
30
Cp- Cv = R
X air changes/hr (Ti - To)
mair = 569.1 kg air
1.33Cv - Cv = 0.277 D. 80.5°C *
Cv = 0.8394
Cp = 1.33(0.8394) = 1.116~
a=(un ~)(1.o ~)·
m3 kg - K. Q = mCp 6T = mCp(T:i - T 1)
Mass of combustion products
= m. + mr = 569.1 + 30
ko.-"K 3,000,000 = 75(1000)(4.187)(90- T1 ) = 599.1 kg
x(ssoox3.8 m3 ~3/hr{~)(23-(-20))< T, = 80.44°C
Heat gain by water = Heat loss by gas '\3600s 46. A mixture of 101 kPa and 23°C that
mwCpw6tw = m 9Cpg61g 44. The mass flow rate of a Freon is 30% by weight C02 and 70% by
Q = 881.47kw
0.5(4.187)(t - 14) = 0.75(1.116)(180- refrigerant through a heat exchanger is weight N2, determine the partial
70) 8.0 kg/min . The enthalpy of entry Freon pressure of C02 .
42. A laboratory building is to be heated
t = 57.9°C is 250 kJ/kg, and the enthalpy of exit A. 3.1 82 kPa
and ventilated. Outside air is to be
Freon is 60 kJ/kg. Water coolant is B. 100 kPa
supplied to a room 10 m by 20 m at the ·
40. A kilogram of water at 60 deg C is allowed to rise 10°C. Determine the C. 21 .65 kPa *
rate of 1.00 m3/min of replacement air
heated at constant pressure. The water water flow rate in kg/min . D. 25 kPa
per square meter of floor space. The
REFRESHER MANUAL 2nd Edition by JAS TORDILLO REFRESHER MANUAL 2nd Edition by JAS TORDILLO
38 -16 I Day 38- Solution
Day 39 - EXAM I 39 - 1
0
Pco2 =( ~~ 2 )Pr Po2 =(" f'b=(-
02
"T
7
- }250)
5+6+7
Power and Industrial Plant Engineering
100 kg of gas mixture: C. 800 kPa
= 97.2 kPa DAY 39- EXAM D. 950 kPa
Mass: Mol
C02 = 30 kg 30/44 = 0.682 mol 50. In an air standard diesel cycle, 1. If the pressures at the beginning and 5. From the previous problem,
N2 =
70 kg 70/28 = 2.5 mol compression starts at 100 kPa and 303
K. The compression ratio is 20 to 1.
end of compression in an Otto cycle determine the Indicated power.
Nr = 3. 18.2 mol engine are 100 kPa and 950 kPa, A. 11 kw
The maximum cycle temperature is respectively, the maximum cycle
Pco, =( - - 101 = 21.65
0.682J kPa B. 20 kw
2200 K. Determine the thermal temperature is 1500 K. Determine the
- 3. 182 C.31 kw
efficiency. air standard efficiency. D. 49 kw
A 46% A. 35.7%
47. Find the air fuel ratio , in kg air per kg B. 64% * B. 39.5% 6. A Carnot engine rejects 230 kw of
fuel, for a combustion process to which c. 69% C. 47.5% heat to the atmosphere at 90 degree C.
the fuel is CsH2o with 30% excess air. D. 70% D. 57.7% What is the amount of heat gain if the
A 3.76
B. 10 temperature of the heat source is 600
Process 1-2, isentropic process: 2. An Otto en'gine has a clearance
C. 13 degree C?
D. 17.1 * T =
_2_
T1
(v_1_)k- 1
v2
volume of 8%. It- produces 500 kw
power. Compression starts at 100 kPa
A. 355 kw
B. 435 kw
and 25 C. What is the heat rejected in C. 449 kw
Fuel +Air = Products of combustion
kw? D. 553 kw
CaH2o + x02 + x(3.76)N2-+ yC02 + zH20 2L = (20)1.4-1 A. 273 kw
+x(3.73)N2 303 B. 372 kw 7. Naga private power plant is using
CsH2o + 1302 + 13(3.76)N2 T2 = 1004.27 K C. 327 kw water as the working fluid that operates
-+8C02 +10H20 + 13(3.76)N2
D. 732 kw on an ideal Rankine cycle. Superheated
Process 2-3, isobaric process:
vapor enters the turbine at 10,000 kPa
air = 13:!: l~Q-762 =6 1. 88 m~le~- o~~r 3. The following data are the results on
fuel I mole fuel v2 = v3 and 520°C and is exhausted into the
a test of a two cylinder, four stroke cycle condenser at 8 kPa. The net output of
air _ 61.88(1.30)28.97 _ k . /k f T2 T3
- - 17 . 13 g a1r g ue1 Otto engine: torque = 2000 N-m; the Rankine cycle is 100,000 kW.
fuel 8(12) + 20(2) VJ T3 = 2200 indicated mean effective pressure =
2.19 Determine the mass flow rate of steam
rc = v2 T2 700 kPa; fuel consumption = 0.006
1004.27 in kg per second.
48. At 28°API specific gravity, what is kg/sec; fuel heating value = 43,000 Steam Properties:
the heating value of oil in kJ/kg? p, 2 kJ/kg; bore. x stroke = 30 em x 40 em; At 10, 000 kPa( 100 bar) and 520 C:
A. 45,039 * speed = 600 rpm. Calculate the brake h = 3425.1
B. 46,790 mean effective pressure. s = 6.622
C. 47,130 A 445 kPa
D. 48,390 B. 375 kPa At 8 kPa : hr = 173.88 sr = 0.5926
C. 325 kPa hg = 2577 S0 = 8.2287
Qh = 41 ,130 + 139.6 (0API) 4 D. 285 kPa Vr = 0.0010084
= 41,130 +139.6(28) A. 967 kg/sec
= 45,038.80.kJ/kg ~------------------ v
4 . A four-cylinder, four-cycle engine with B. 267 kg/sec
10-cm diameter pistons and an 16-cm C. 74.4 kg/sec
Cycle Thermal Efficiency stroke operates at a speed of 600 rpm
49. A mixture contains 5 moles of D. 20.1 kg/sec
oxygen, 6 moles of Nitrogen, and 7 and yields an indicator diagram. The
moles of carbon dioxide at 120°C and 1 [rcK - 1] area under the curve (PV diagram) is
250 kPa. What is the partial pressure of = 1 - rkK-1 k(rc -1) equal to 12.0 cm 2. The length of the
8. In problem No. 7, Determine the heat
flow into the boiler in kJ per second.
carbon dioxi~e? diagram is 9.0 em, and the spring A. 241.14 x 103 kJ/sec
A 250 kPa 4 constant of the indicator spring is 600 B. 341.14 x 103 kJ/sec
B. 140 kPa 1 1 [(2.19)1. - 1] kPa/cm . Determine the mean ef(ective C. 441 .1 4 x 103 kJ/sec
C. 101 kPa = - (20)1.4-1 1.4(2.19 -1) pressure. D. 541 .14 x 103 kJ/sec
D. 97.2 kPa * A. 400 kPa
= 63.84% B. 600 kPa
REFRESHER MANUAL 2nd Edition by JAS TORDILLO
REFRESHER MANUAL 2nd Edition by JAS TORDILLO
~
~

39 - 2 I Day 39 - EXAM Day 39 - EXAM I 39 - 3


Power and Industrial Plant Engineering
9. A 15,000 kW steam turbine-generator B. 32,679.50 kw
power plant has full load steam rate of C. 37,125.50 kw B. 3,307 kw 21. Dodecane (C12H2s) is burned at
7.5 kglkw-hr. No load steam D. 45,500.50 kw C. 2,810 kw constant pressure with 140% excess air.
consumption is around 10% of full load D. 3,503 kw Determine the number of moles of
steam consumption. Calculate the 13. An ideal gas turbine operates with a oxygen in the products of combustion?
hourly steam consumption at three- pressure ratio of 7.5 and temperature 17. An impulse turbine which has a A. 22.9
fourths load in kg/hr. limits of 25°C and 1100°C. The energy diameter, D =5 ft, speed, N = 7 rev/sec, B. 25.9
A. 87,187.50 kg/hr input in the high temperature heat bucket angle, 13 = 150°, coefficient of C. 44.5
B. 112,500 kg/hr exchanger is 500 kw. Determine the air velocity, c. = 0.9760, relative speed, D. 55.5
C. 15,500 kg/hr flow rate in kg/hr. .0 = 0.470, a generator efficiency, em=
D. 6, 750 kg/hr A. 59.3 0.90, and a jet diameter from nozzle of 22. A thermodynamic system contains
B.235 0.625 feet. Compute the power input in 20 cubic meters of air at 390 kPa and
10. A 20,000 MW gas turbine operating C. 2135 HP. 120°C. Determine the weight of this
in the simple open cycle has an exhaust D. 3450 A. 5479 HP system inN.
gas flow of 100 kg per second. The gas B. 6754 HP A. 297.3
enters a waste heat recovery boiler at 14. Air at 400 kPa and 590 K is C. 7625 HP B. 492.5
450°C and leaves at 180°C, specific extracted from a jet engine compressor D. 8950 HP c. 578.9
heat of the gas is 0. 85 kJ/kg°K, heat to be used for the generation of auxiliary D. 678.4
required to produce 1 kg of steam at 4.5 power for the cabin. The extracted air is 18. Pipe color for the substance like
MPa and 320°C from feedwater at cooled in a constant pressure heat water: 23. Air In a closed piston-cylinder device
110°C is 5.5 kJ. Calculate the quantity exchanger down to 460 K. It then A. green arranged to maintain a pressure of 300
of steam that can be produce in kg per enters an isentropic turbine and B. red kPa 'is heated from 30°C to 250°C.
second. expands to 100 kPa before being C. silver-gray Initially the volume of the air is one liter.
A. 1723 kg/s rejected into the cabin. If the mass flow D. wh ite What is the final air volume in m3 ?
B. 21?.3 kg/s is 15 kg/min, determine the power A 1.26 m3
C. 3173 kg/s developed by 1he turbine in kw. 19. A typical steam generator with an B. 0.716 m3
D. 4173 kg/s A 37.6 kw efficiency of 85% is producing 18 kg/sec C. 0.001656 m3
B. 63.7 kw of steam. The enthalpy of the entering p. 0.001726 m3
11. A simple Brayton cycle which uses C. 73.6 kw water is 140 kJ/kg and is superheated to
helium as the working fluid, has a D. 76.3 kw an enthalpy of 3300.50 kJ/kg. The fuel 24. A 12,000 kW steam turbine-
maximum temperature of 1150°K, and a used has a heating value of 44,500 generator power plant has full load
pressure ratio of 2.90. At the start of the 15. A Hydroelectric power plant has the kJ/kg. The AIF ratio by weight is 20. steam rate of 5.5 kg/kw-hr. No load
compression, the helium pressure and following data: Determine the amount of air needed in steam consumption is around 10% of
temperature are 90 kPa and 300°K. Type of turbine .. ..... . .. . . .Francis kg/hr. full load steam consumption. Calculate
Based upon cold air-standard analysis Gross head . .. . . .. .... . ... 75 m A. 168,317 the hourly steam consumption at two-
assumptions, determine the thermal Rate of flow . ....... ... .... 5 m3/sec B. 108,288 third of load in kg/hr.
efficiency of the cycle in percent? Penstock length ........... .65·m C. 178,328 A. 40,250
A. 17.5% Penstock diameter . . . . . . . . . 850 mm D. 96,123 B. 46,200
B. 23.7% Assume that the turbine efficiency is C. 36,830
C. 28.7% 85% and generator efficiency is 90%. 20. A spherical tank 8.5 m in diameter D. 35,820
D. 34.7% Assume f = 0.025. Find the brakepower contains methane (CH4) at 450 kPa and
developed in MW. 35°C after 10 kg had been used. 25. A 60-m wide and 3.0 m deep river
12. The mass flow rate of the gas io a A1 .1MW Calculate the original mass of methane flows at the rate of 2.5 m/sec. A
gas turbine is 30 kg/sec. The specific B. 1.8 MW in the tank in kg. hydroelectric installed nearby develops
enthalpy and velocity in the inlet are C. 2.8 MW A. 904 a gage pressure of 450 kPa at the
1300 kJ/kg and 200 m/sec respectively D. 3.5 MW B. 914 turbine entrance. Calculate the
while in the outlet are 300 kJ/kg and 50 c. 1014 maximum power available in MW
m/sec, respectively. Calculate the 16. F.rom the previous problem, What is D. 1114 considering a head loss of 5.0 m
power output in kw of the turbine if there the power output generated .by the developed by the casing and turbine
1s a heat loss of 3 MW. hydraulic Plant in kW? blades.
A. 27,562.50 kw A. 2,530 kw A. 198 MW

REFRESHER MANUAL 2nd Edition by JAS TOROILLO REFRESHER MANUAL"2nd Edition by .JAS TORDILLO
._
39 - 4 I Day 39 - EXAM Day 39 - EXAM I 39 - 5
Power and Industrial Plant Engineering
B. 180 MW temperature is 400°C. Determine the
C. 194 MW heat added in kJ. 34. In Problem No. 32, What is the 38. Calculate the gross steam rate in
D. 200 MW A. 3545 thermal efficiency of the cycle? kg/kw-hr.
B. 4190 A. 54.2% A. 3.75
26. A Carnot engine operates using a C. 5854 B. 34.2% B. 5.45
560°C energy reservoir and a 32°C D. 5560 C. 44.2% c. 4.45
energy reservoir. Find the D. 24.2% D. 6.45
thermodynamic efficiency of this engine. 31 . A hydro-electric power plant
A. 50% consumes 60,000 MW-hrs per, annum. 35. A condenser gauge reads 25.5 in Hg 39. From Problem No. 37, What is the
B. 76.3% Expected flow is 60 m 3/sec and overall vacuum when the barometer stands at annual load factor of the plant?
C. 63.4% efficiency is 72%. What is the net head 10.32 m of H20. Determine the A. 0.392
D. 100% in meters? absolute pressure in the condenser in B. 0.492
A. 4 1.2 mmHg. C. 0.392
27. The mass flow rate of the gas in a B. 85.5 A. 78.9 mm Hg D. 0.552
gas turbine is 40 kg/sec. The specific C. 29.9 B. 235.6 mm Hg
enthalpy and velocity in the inlet are D. 16.2 C. 157.6 mm Hg 40. From Problem No. 37, Determine
1600 kJ/kg and 200 m/sec respectively D. 111.6mmHg the annual plant capacity factor.
while in the outlet are 500 kJ/kg and 50 32. In a Rankine cycle, saturated liquid A. 0.482
m/sec, respectively. Calculate the water at 1 bar is compressed 36. The temperature of a solution is -40 B. 0.514
power output in kw of the turbine if there isentropically to 150 bar. First by degree C. Convert the equivalent C. 0.646
is a heat loss of 1.5 MW. heating in a boiler, and then by temperature reading to degree Rankine D. 0.864
A. 37,462 superheating at constant pressure of temperature.
B. 40,872 150 bar, the water substance is brought A. 560 degree R 41 . From Problem No. 37, Determine
C. 34,605 to 750°K. After adiabatic reversible B. 420 degree R the annual gross power generated.
D. 43,250 expansion in a turbine to 1 bar, it is then C. 500 degree R A. 102 MW
cooled in a condenser to a saturated D. 360 degree R B. 145 MW
28. The compression ratio of an air- liquid. How much energy required in the C. 180 MW
standard gasoline engine is 7.5. The pump, considering a steam flow rate of 37. A 200 MW power plant has the D. 200 MW
heat added is 1890 kJ/kg. If the initial Q.495 kg/s? following data:
pressure and temperature are 1 bar and Properties of Steam: Annual gross kw-hrs generation 42. From PfOblem No. 37, Determine
27°C respectively, determine the At 150'bar (15 MPa) and 750°K (477°C): = 800 X 106 the annual plant use factor.
temperature in °C at the end of the h1 = 3240.5 kJ!kg Annual net kw-hrs generation A. 0.865
isentropic expansion. S1 = 6.2549 kJ/kg-°K = 760x106 B. 0.765
A. 2968 At 1 bar (0.1 MPa): Annual operating hours generation c. 0.465
B. 1765 hr = 417.46 Sr = 1.3026 = 7,840 D. 0.538
C . 1347 hr9 = 2258.0 Srg = 6.0568 Annual average load in kw
D. 1206 h3 = hr = 417.46 = 99,500 43. From Problem No. 37, Find the fuel
V3 = Vr = 0.0010432 Annual maximum peak load economy in kw-hrs per liter of fuel.
29. Calculate the thermal efficiency of A. 3.5 kw = 180,000 A. 6.583
an air-standard Diesel cycle operating B. 7.7 kw Throttle steam flow B. 4.064
with a compression ratio of 21 and cut- C. 9.9 kw = 300 X 107 kg c. 5.502
off ratio of 3. D. 10.5 kw Fuel oil consumed D. 3.504
A. 55. 9% = 52 x 106 gal
B. 61.4% 33. In Problem No. 32, How much Calculate the annual Reserve Over 44. A mini-hydropower plant has an
C. 66.9% power is generated in the turbine? Peak. available head of 8 m. The water flow is
D. 71.9% A. 976.6 kw A. 10,000 kw 1.3 m3/sec, efficiency of the t:..;rbine is 87
B. 876.9 kw B. 20,000 kw %, efficiency of the generator is 90%.
30. A closed rigid tank has a capacity of C. 776.9 kw C. 30,000 kw Determine the maximum output of the
3
3.5 m • It holds air at 500 kPa and D. 483.6 kw D. 180,000 kw generator in kW.
23°C. Heat is added until the A. 102
B. 69

REFRESHER MANUAl 2nd Edition by JAS TORDillO REFRESHER MANUAL 2nd Edition by JAS TORDillO
39 - 6 I Day 39 - EXAM Day 39 - Solution I 39 - 7
C. 75 Power and Industrial Plant Engineering
D. 80
(d DAY 39 - SOLUTION rk = l+ c = 1+0.08 = !3. 5
45. A <:;arnot engine requires 40 kJ/sec c 0.08
DAY 39- ANSWER KEY . I 1
from the hot source. The engine 1. If the pressures at the beginning and
produces 18 kw of power and the
1. C 16. A 31.0 46. A e = I -l('::f = 1- ( _)1.4_ 1 = 0.6469
2. A 17. B 32.8 47. B end of compression in an Otto cycle rk 13 .,
temperature of the sink is 24.5°C. What 3.A 18.A 33.0 48. B engine are 100 kPa and 950 kPa,
4. C 19.9 34.9 49. B w
is the temperature of the hot source in respectively, the maximum cycle e =-
oc? s. e 20. e. · 35.0 50. B QA
6. 0 21 . 9 36.8 temperature is 1500 K. Determine the
A. 545.5 7.C 22.0 37.9 air standard efficiency. 500
0.6469 =
B. 240.8 8.,A 23. 9 38.A A. 35.7% QA
c. 211 .9 9. A 24. B 39.0
B. 39.5% QA : 772.92
10.0 25. B 40.A
D. 267.9 11 .0 26. c 41.A c. 47.5% *
12.A 27. 0 42.0 D. 57.7% QA=W+QR
46. Thermodynamic properties that are 13.C 28. 0 43.8 772.92 = 500 + QR
14.A 29. B 44.0' rK = compression ratio
V1
dependent of the mass. OR= 272.92 kw
A. Extensive properties 15.C 30. 0 45.0 v2 p, 3
B. Intensive properties
C. Specific properties
G p2 = (~Jk
D. Pressure P1 V2

47. Open system is also known as:


A. C.ontrolled mass system
rk = ~
V
= (p2 J"k
P 4
2 1
B. Controlled volume system
C. Transient system
D. Isolated system fK = ( -950 ) 1/1.4 = 4.993 L-- - - - - - -----V
100
3. The following data are the resu lts on
48. The equivalent of 1 Btu: I I a test of a two cylinder, four stroke cycle
Eff = 1 - - - = 1 - - - --
A. 0.746 Hp
rk k- l (4.993)1.4-1 Otto engin!;l: torque = 2000 N-m; .
B. 778.169 ft lb indicated mean effective pressure =
C. 33, 000 ft lb = 0.4744 = 47.44% 700 kPa; fuel consumption = 0.006
D. 1.055 Hp kg/sec; fuel heating value = 43,000
P1 3
kJ!kg; bore x stroke = 30 em x 40 em;
49. The vapor that has temperature speed =· 600 rpm. Calculate the brake
equal to the saturation temperature mean effective pressure.
corresponding to an existing pressure. A. 445 kPa *
A. Wet vapor B. 375 kPa
B. Saturated vapor C. 325 kPa
4
C. Superheated vapor D. 285 kPa
D. All of the above
L-------------------V BP = 2rrTN
50. The ones defining the physical
condition of the fluid are · called: 2. An Otto engine has a clearance
BP = 2rr(2.o)e6 0 °°) = 12s.66 kw
A. Thermodynamic properties volume of 8%. It produces 500 kw
B. State properties power. Compression starts at 100 ·kPa VD = '!..D 2 LNC
4
C. Cycle-properties and 25 C. What is the heat rejected in vD = '!.. (0.30)2 (0.40) (~) 2
D. Substance properties kw? 4 2:<60

A. 273 kw *
= 0.2827 m3/sec
B. 372 kw
C. 327 kw BP = Pmb Vo
125.66 = Pmb (0.2827)
D. 732 kw
Pmb = 444.49 kPa
REFRESHER MANUAL 2nd Edition by JAS TORDILLO
REFRESHER MANUAL 2nd Edition by JAS TORDILLO
39 - 8 1 Day 39 - Solution Day 39 -Solution I 39 - 9
Power and Industrial Plant Engineering
4. A four-cylinder, four-cycle engine with 7. Naga private power plant is using
10-cm diameter pistons and an 16-cm water as the working fluid that operates 8. In problem No. 7, Determine the heat C. 3173 kg/s
stroke operates at a speed of 600 rpm on an ideal Rankine cycle. Superheated flow into the boiler in kJ per second. D. 4173 kg/s *
and yields an indicator diagram. The vapor enters the turbine at 10,000 kPa A. 241.14 x 103 kJ/sec *
area under the curve (PV diagram) is and 520°C and is exhausted into the B. 341.14 x 103 kJ/sec Heat rejected by exhaust gases
equal to 12.0 cm 2 . The length of the condenser at 8 kPa. The net output of C. 441 . 14 x 103 kJ/sec = heat absorbed by feedwater
diagram is 9.0 em, and the spring the Rankine cycle is 100,000 kW. D. 541 .14 x 103 kJ/sec mgCp(T4- Ts) = m, (3.5 kJ)
constant of the indicator spring is 600 Determine the mass flow rate of steam 100(0.85) (450- 180) = m. (5.5)
kPa/cm. Determine the mean effective in kg per second. QA = m(h1 - h4) m, = 4172.72 kg/sec
pressure. = 74.4 (3425.1 - 183.96)
A. 400 kPa Steam Properties: = 241.14 x 103 kJ/sec 11. A simple Brayton cycle which uses
B. 600 kPa At 10,000 kPa(100 bar) and 520 C: helium as the working fluid, has a
C. 800 kPa * h..= 3425.1 9. A 15,000 kW steam turbine-generator maximum temperature of 1150°K, and a
D. 950 kPa s = 6.622 power plant has full load steam rate of pressure ratio of 2.90. At the start of the
7.5 kg/kw-hr. No load steam compression, the helium pressure and
Area under curve At 8 kPa: hr = 173.88 sr = 0.5926 consumption is around 10% of full load temperature are 90 kPa and 300°K.
MEP = stroke h0 = 2577 · S0 = 8.2287 steam consumption. Calculate the Based upon cold air-standard analysis
·v, = 0.0010084 hourly steam consumption at three- assumptions, determine the thermal
x spring scale
A. 967 kg/sec fourths load in kg/hr. efficiency of the cycle in percent?
12 B. 267 kg/sec A. 87,187.50 kg/hr * A. 17.5%
MEP = - x 600 = 800 kPa C. 74.4 kg/sec* B. 112,500 kg/hr B. 23.7%
9 D. 20.1 kg/sec C. 15,500 kg/hr C. 28.7%
D. 6,750 kg/hr D. 34.7% *
5. From the. previous problem, 51 = 52
determine the Indicated power. 6.622 = 0.5926 + X2(8.2287- 0.5926) ms = kx + b le-t
A. 11 kw X2 :: 0.7896 m, (at full load) = 7.5 (15,000) !! =
T1
(!2)....
P1
B. 20 kw * = 112,500 kg/hr
C. 31 kw h2 = 173.88 + 0.7896(2577 -173-.88) b = 0.10(112,500) _.!!.. - ( 1.667- 1
D. 49 kw = 2071.38 11 ,250 kg/hr (at zero load) 300 - 2.90)"TT67

Ind. Power = MEP x Vd Wp = 0.0010084(10,000-8) k = 112,500-11,250 = 6.75 T2 = 459.28°K

= 800 x 2:(o.J0)2(o.t6{
600
)4 =
10.08 kJ/kg
X
15.000
= 15,000 (0.75)
h4 = h3 + Wp :: 173.88 + 10.08 T3 = 1150°K
4 \2x60 = ·11,250 kw (3/4 load)
= 183.96 kJ/kg
= 20.1 kw k-1

Wnet = m(Wt- Wp) m, = 6.75(11,250)+ 11,250 2:! =


r4
(!2)....
P4
6. A Carnot engine rejects 230 kw of = 87,187.50 kg/hr at Y. load
100,000 (3600) . 1150 = (2.90)'~.:~;'
heat to the atmosphere at 90 degree C.
What is the amount of heat gain if the
=
m((3425.1-2071.38)-10.08] r.
T4 = 751.1°K
m = 267.928 x 103 kg/hr 10. A 20,000 MW gas turbine operating
temperature of the heat source is 600 = 74.4 kg/sec in the simple open cycle has an exhaust
degree C? Turbine gas flow of 100 kg per second. The gas We :: mCp(T2- T1)
A. 355 kw 10 MPa Output enters a waste heat recovery boiler at = 1.0(459.28- 300) = 159.28 kJ/kg
B. 435 kw r-----5::2:;0;...•c;:_, 100MW 450°C and leaves at 180°C, specific Q A = mCp(T3- T2)
C. 449 kw heat of the gas is 0.85 kJ/kg°K, heat = 1.0(1150-459.28)
0.553kw* req uired to produce 1 kg of steam at 4.5 = 690.72 kJ/kg
MPa and 320°C from feedwater at WT = mCp(T3- T4)
Eff = TH-TL - QA- QR 11 0°C is 5.5 kJ . Calculate the quantity = 1.0(1150-751 .1) = 398.9kJ/kg
TH - Q:;- of steam that can be produce in kg per
600-90 QA- 230
second. Eff = Wr-Wc = 398.9-159.28 = 0. 347
--- = --- A. 1723 kg/s QA 690.72
600+273 QA
QA = 553.14 kw B. 2173 kg/s
= 34.7%
REFRESHER MANUAL 2nd Edition by JAS TORDILLO REFRESHER MANUAL 2nd Edition by JAS TORDILLO
39 - 10 1 Day 39 - Solution Day 39- Solution 139 -11
Power and Industrial Plant Engineering
12. The rnass flow rate of the gas in a is 15 kg/min, determine the power
gas turbine is 30 kg/sec. The specific developed by the turbine in kw. H. = 75 - 7.56 = 67.43 m V = actual velocity of jet
enthalpy and velocity in the inlet are A. 37.6 kw* BP ,
1300 kJ/kg and 200 m/sec respectively e, = -X 0
100to = 0.976J2(32.2) (849.87)
B. 63.7 kw
WP V = 228.33 ft/sec
while in the outlet are 300 kJ/kg and 50 C. 73.6 kw
m/sec, respectively. Calculate the BP = e0 (WP)
D. 76.3 kw
power output in kw of the turbine if there Q = area x velocity
Is a heat loss of 3 MW. WP = QyH. = (5)(9.81)(67 .43) = 0.30679 (228.33)
T2 = 590 K T3 = 460 K =3307.44 kW
A. 27,562.50 kw * Q = 70.05 fe/sec
B. 32,679.50 kw
p )(k-1)/k { 1 )(1.4-1)/1.4 BP = 0.85(3307.44) Power input = Qyh
C. 37,125.50 kw T4 = T3 ( 3. = 46 -
=2811 .32 kW = 2.81 MW
D. 45,500.50 kw PJ 4 70.05(62.4) (849.87)
= 309.56 K
= 550
m(V11 -vl) Q 16. From the previous problem, What is
Po = m(h1- h2) + 2(1000) - L the power output generated by the = 6,754.33 HP
Power developed by the turbine hydraulic Plant in kW?
Po = 30(1300- 300)
+ 30((200) 1 - (50) 1) _
= mCp(T3- T4) A. 2,530 kw * 18. Pipe color for the substance like
3000
= 27,562.50 kw
2(1000)
= (~~}LoX460- 309.56) = 37.61 kw
B. 3,307 kw
C. 2,810 kw
water:
A. green*
D. 3,503 kw B. red
13. An ideal gas turbine operates with a G.O. C. silver-gray
15.' A Hydroelectric power plant has the e9 = - - x 100%
pressure ratio of 7.5 and temperature D. white
following data: BP
limits _of 25°C and 11 00°C. The energy Type of turbine .............Francis
input in the high temperature heat G.O. = e9 (BP) = 0.90(2811 .32) 19. A typical steam generator with an
Gross head .... .. .... ..... 75 m = 2530.2 kW = 2.53 MW
exchanger is 500 kw. Determine the air efficiency of 85% is producing 18 kg/sec
Rate of flow ..... ... ....... 5 m3/sec
flow rate in kg/hr. of steam. The enthalpy of the entering
Penstock length ............ 65 m 17. An impulse turbine which has a
A. 59.3 water is 140 kJ/kg and is superheated to
Penstock diameter . . . . . . . . . 850 mm diameter, D = 5 ft. speed, N = 7 rev/sec,
B. 235 an enthalpy of 3300.50 kJ!kg . The fuel
Assume that the turbine efficiency is bucket angle, [3 = 150°, coefficient of
c. 2135 * 85% and generator efficiency is 90%.
used has a heating value of 44,500
D. 3450 velocity, Cv = 0.9760, relative spee9. kJ/kg. The ·A!F ratio by weight is 20.
A~sume f = 0.025. Find the brakepower 0 = 0.470, a generator efficiency, em = Determine the amount of air needed in
k-1
developed in MW. 0.90, and a jet diameter from nozzle of kg/hr.
!! = f!:!)K"" A. 1.1 MW 0.625 feet. Compute the power input in
\.Pt A. 168,317
T1 B. 1.8 MW HP.,
~ = ·(7.5)'·:.:' C. 2.8MW* B. 108,288 *
25+273 A. 5479 HP C . 178,328
Tz = 529.9°K D. 3.5 MW B. 6754 HP * D. 96, 123
C. 7625 HP
T3 = 1100°C = 1373°K Net head of the turbine, Ha D. 8950 HP
He= H9 - hL Boiler Eff = msCh2- tt1 l
m{Qh
QA = mCp(T3- Tz) h = .net head
500 = m(1.0)(1373- 529.9) hL = hr + he +hn _ 11DN 0.85 = 1 8(3600)(3300.S0-140)
m = 0.593 kg/s = 2135.1 kg/hr Relative speed. 0 mr (44.500)
Pipe friction loss, hr - J 2gh
14. Air at 400 kPa and 590 K is Q =ApVp mr = 5414.42 kg/hr
extracted from a Jet engine compressor 5 7!( 60)(420)
to be used for the generation of auxiliary
Vp= --:::-~ = 8.81 m/sec 0.47 = 12 60. m11ir = 20
(7!/4) ( 0.85) J2(32.2)h m.fuel
power for the cabin. The extracted air is
cooled in a constant pressure heat LV 2 ( 8.81) 2 h = 849.87 ft
exchanger down to 460 K. It then
hr = f - - = 0.025(65) mal r
S414.42 = 20
enters an isentropic turbine and
expands to 100 kPa before being
2gD
= 7.56 m
2(9.81) ( 0.85)
A = area of jet = !!..
4
(I2)
12
2

mair = 108,288.38 kg/hr


rejected into the cabin. If the mass flow = 0.30679 ft2
hL= hr = 7.56 m
REFRESHER MANUAL 2nd Edition by JAS TORDILLO REFRESHER MANUAL 2nd Edition by JAS TORDILLO
39 -121 Day 39- Solution Day 39 - Solution 1 39 - 13
Power and Industrial Plant Engineering
20. A sptlerical tank 8,5 m in diameter PV = mRT
contains methane (CH4) at 450 kPa and 390(20) = m(0.287)(120 + 273) a gage pressure of 450 kPa at the 2
p 0 = 40( 1600 - 500) + 40((200) - (50)2)
35°C after 10 kg had been used. m = 69.15 kg turbine entrance. Calculate th~
- 1500 2(1000) .
Calculate the original mass of methane W = mg = 69.15(9.81) = 678.4 N maximum power available in MW
in the tank in kg. considering a head loss of 5.0 m = 43,250 kw
A. 904 23. Air in a closed piston-cylinder device developed by the casing and turbine
B. 914 * arranged to maintain a pressure of 300 blades. 28. The compression ratio of an air-
C. 1014 kPa is heated from 30°C to 250°C. A. 198 MW standard gasoline engine is 7.5. The
D. 1114 Initially the volume of the air is one liter. B. 180 MW * heat added is 1890 kJ/kg. If the initial
3
What is the final air volume in m ? C. 194 MW pressure and temperature are 1 bar and
V = ~ nr 3 = i rr(4.25) 3
= 321.55 m 3 A. 1.26 m
B. 0.71 6 m 3
3
D. 200 MW 27°C respectively, determine
temperature in °C at the end of the
the

C. 0.001656 m3 Q = 60(3.0)(2.5) = 450m 3/sec isentropic expansion.


PV = mRT D. 0.001726 m3 * A. 2968
450(321.55) = m(
8
·~~ )(35 + 273)
43
450 B. 1765
Hr = = 45.87 m
m ·= 904.07 kg .!i = !!. 9.81
Net head, h = 45.87 - 5.0 = 40.87 m
C. 1347
T1 T~
1 Vz
D. 1206 *
Original mass = 904.07 + 10 3o+273 = 2sii+273 p = Qyh = 450(9.81)(40.87)
= 914.07 kg !! = rl;-1
= 180,420.62 kw = 180.42 MW
Vz ::; 1.72611 = 0.001726 m 3 r,
21. Dodecane (C12H2s) is burned at T2
26. A Carnot engine operates using a (27+273) = (7.5)1.4-1
constant pressure with 140% excess air. 24. A 12,000 kW steam turbine-
560°C energy reservoir and a 32°C
Determine the number of moles of generator power plant has full load
energy reservoir. Find the
oxygen in the products of combustion? steam rate of 5.5 kglkw-hr. No load Tz = 671.65°K = 398.65°C
thermodynamic efficiency of this engine.
A. 22.9 steam consumption is around 10% of
A. 50%
B. 25.9 * full load steam consumption. Calculate
B. 76.3% Q,., = Cv(T3 - Tz)
c. 44.5 the hourly steam consumption at two- m

D. 55.5 third of load in kg/hr.


c. 63.4% * 1890 = 0.716(T3- 671.65)
D. 100% "13 = 3,31 1.31°K
A. 40,250
Combustion Reaction with Theoretical a 46,200*
Eff = = ~ k-1
Air: C. 36,!bO TH-TL
(560+273)
!!= Tk
TH r.
C12H2e + (18.5)02 + (18.5)(3.76)N2 D. 35,820 = 0.6338 = 63.38%
-+ 12C02 + 13H20 + (18.5)(3.76)N2 ~ = (7.5)1.4-1
ms = kx + b 27 .• The mass flow rate of the gas in a r.
Combustion Reaction with 140% Excess gas turbine is 40 kg/sec. The specific T4 = 1479.03°K = 1206.03°C
Air: m. (at full load) = 5.5(12,000) enthalpy and velocity in the inlet are
c12H2e + (1 +1 .4)(18.5)02 + = 66,000 kg/hr 1600 kJ/kg and 200 m/sec respectively 29. Calculate the thermal efficiency of
(1+1.4)(18.5)(3.76)N2 -+
12C02 + b = 0.10(66,000) while in the outlet are 500 kJ/kg and 50 an air-standard Diesel cycle operating
13H20 + (1+1.4)(18.5)(3.76)Nz + = 6,600 kg/hr (at zero load) m/sec, respectively. Calculate the with a compression ratio of 21 and cut-
(1.4)(18.5)02 power output in kw of the turbine if there off ratio of 3.
- 66.000-6,600 = 4.95 is a heat loss of 1.5 MW. A. 55.9%
k - 12,000 B. 61.4% *
Mols of 02 = 1.4(18.5) = 25.9 mols A. 37,462
B. 40,872 C. 66.9%
22. A thermodynamic system contains
X = 12,000 (2/3) C. 34,605 D. 71.9%
20 cubic meters of air at 390 kPa and
= 8,000 kw (half load) D. 43,250 *
120°C. Determine the weight of this
m. = 4.95(8,000) + 6,600 Eff = 1 -
1 [rt-1]
rf-t k(rc- 1)
system inN. m(v12 - vi)
= 46,200 kg/hr at 213 load Po =m,
(h -2
h ) + -QL (3)'·· -1]
A. 297.3 2(1000) Eff = 1 - (z 1 )~-•-• [1.4(3-11
8. 492.5
25. A 60-m wide and 3.0 m deep river
C. 578.9 flows at the rate of 2.5 m/sec. A Eff = 0.6137 = 61.37%
D. 678.4 * hydroelectric installed nearby develops
REfRESHER MANUAL 2nd Edition by JAS TORDILLO REFRESHER MANUAL 2nd Edition by JAS TORDILLO
39 - 14 I Day 39 - Solut io n Day 39- Solution 1 39 -15
Power and Industrial Plant Engineering
30. A closed rigid tank has a capacity of hrg = 2258.0 Stg = 6.0568
3
3.5 m . It holds air at 500 kPa and h3 = hr = 417.46 A. 54.2% 37. A 200 MW power plant has the ·
23°C. Heat is added until the V3 = Vt = 0.0010432 B. 34.2% • following data:
temperature is 400°C. Determine the A. 3.5 kw C. 44.2% Annual gross kw-hrs generation
heat added in kJ. B. 7.7 kw * D. 24.2% = 800 X 106
A. 3545 C. 9.9 kw Annual net kw-hrs generation
B. 4190 D. 10.5 kw QA = m(h, - h4) = 760 x 106
C. 5854 = 0.495(3240.5- 433) =1389.71 kw Annual operating hours generation
D. 5560* Solving for h2: = 7,840
s, = 52 Thermal Efficiency of the Cycle Annual average load in kw
PN, = mRT, S1 = St + XStg = 99,500
= Wr - Wp
500(3.5) = m(0.287)(23 + 273) 6.2549 =
1.3026 + x(6.0568) Annual maximum peak load in kw
m = 20.6 kg X = 0.8176 = 81:76% QA = 180,000
Throttle steam flow
= 483.56- 7.7 = 34.24%
Q = mCv(T2- T,) h2 = hr + xhrg = 300 X 107 kg
Q = 20.6(0.716)(400- 23) 1389.71
h2 = 417.46 + 0.8176(2258) Fuel oil consumed
Q = 5560.60 kJ =
2263.6 kJ/kg
35. A condenser gauge reads 25.5 in Hg
= 52 x 106 gal
Calculate the annual Reserve Over
31. A hydro-electric power plant vacuum when the barometer stands at Peak.
Solving for h4:
consumes 60,000 MW-hrs per annum.
Expected flow is 60 m3/sec and overall
Wp = h4- h3 =
V3(P,4- P3)
10.32 m of H20 . Determine the
absolute pressure in the condenser in
A. 10,000 kw
h4 -417.46 = 0.0010432 (15,000-100) B. 20,000 kw *
efficiency is 72%. What is the net head h. = 433.0 kJ/kg ' mm Hg. C. 30,000 kw
in meters? A. 78.9 mm Hg D. 180,000 kw
A. 41.2 B. 235.6 rom Hg
Wp = m(h• - h3) = 0.495(433- 417.46)
B. 85.5 C. 157.6 mm Hg Reserve Over Peak
= 7.69 kw
C. 29.9 D. 111.6 mm Hg * = Plant Capacity - Peak Load
150 bar
D. 16.2 * 750°K _ . H 760 mm Hg = 200,000- 180,000
Pg- - 25 .5 m gx = 20,000 kw
BP = Qyh (eft) 29.92 in Hg ·.
Pg =- 647.73 mm Hg 38. Calculate the gross steam rate in
~ 8760
= (60)(9.81)h (0.72) Patm = 10.32 m H20 X 9.81 -
kN kg/kw-hr.
h = 16.16 m . m3 A. 3.75 *
B. 5.45
;: 101 .24 kPa = 759.3 mm Hg
C. 4.45
32. In a Rankine cycle, saturated liquid
D. 6.45
water at 1 bar is compressed P(abs) = P(gage) + P(atm)
7
isentropically to 150 bar. First by = -647.73 + 759.3
heating in a boiler, and then by w. = 111.57 mm Hg
Gross steam rate = 300xl0
6
800xio
superheating at constant pressure of
33. In Problem No. 32, How much = 3.75 kg/kw-hr
150 bar, the water substance is brought 36. The temperature of a solution is -40
to 750°K. After adiabatic reversible power is generated in the turbine? degree C. Convert the equivalent
A. 976.6 kw 39. From Problem No. 37, What is the
expansion in a turbine to 1 bar, it is then temperature reading to degree Rankine
B. 876.9 kw annual load factor of the plant?
cooled in a condenser to a saturated temperature.
C. 776.9 kw A. 0.392
liquid. How much energy required in the A. 560 degree R
D. 483.6 kw * B. 0.492
pump, considering a steam flow rate of B. 420 degree R *
0.495 kg/s? C. 500 degree R
c. 0.392
Properties of Steam: WT = m(h, - h2) D. 360 degree R
D. 0.552 *
At 150 bar (15 MPa) and 750°K (477°C): = 0.495(3240.5 - 2263.6)
= 483.56 kw Annual load factor
h, = 3240.5 kJ/kg F =~(5 0C) + 32 =5~(-40) + 32
S1 = 6.2549 kJ/kg-°K = annual average load
34. In Problem No. 32, What is the = -40°F
At 1 bar (0.1 MPa):
thermal efficiency of the cycle? R = -40 + 460 = 420°R max imum peak load
hr = 417.46 sr = 1. 3026

REFRESHER MANUA L 2nd Edition by JAS TORDILLO REFRESHER MANUAL 2nd Edition by JAS TORDILLO
39- 16 I Day 39- $elution Day 39- Solution 139- 17
Power and Industrial Plant Engineering
99,500 = 0.5527 = Gross energy produced
= 180,000
liters of fuel consumed T

THI---~EJ
1
40. From Problem No. 37, Determine v, = 52 x 106 gal
the annual plant capacity factor. = 196,840,800 liters WorkNet ,,
A. 0.482 * 'r , = 297.s'K 1.. .
800 106 3: :2
B. 0.514 Fuel economy = x
' '
C. 0.646 196,840,800
D. 0.864 = 4.064 kw-hr/liter · .._~~---~-- s
s, = s. s, = s,
Annual plant capacity factor 44. A mini-hydropower plant has an
available head of 8 m. The water flow is 46. Thermodynamic properties that are
= annual energy produced 3 dependent of the mass.
1.3 m /sec, efficiency of the turbine is 87
Equipment Ratingx8760 %, efficiency of the generator is 90%. A. Extensive properties *
DeteriJline the maximum output of the B. Intensive properties
760x!0
6
generator in kW. C. Specific properties
= 0.482
180,000x8760 A. 102 D. Pressure
B. 69
C. 75 47. Open system is also known as:
41. From Problem No. 37, Determine
the annual gross power generated. D. 80* A. Controlled mass system
A . 102MW* B. Controlled volume system *
B. 145 MW G.O. = WPxetXeg C. T ransient system
D. Isolated system
C. 180 MW
D. 200 MW WP = QyH = (1.3)(9.81)(8)
= 102.024 kW 48. The equivalent of 1 Btu:
Power Generated = 800xl o6 A. 0.746 Hp
7840 G.O. = 102.024(0.87)(0.90) B. 778.169 ft lb *
= 79.88 kW C. 33, 000 ft lb
42. From Problem No. 37, Determine D. 1.055 Hp
the annual plant use factor. 45. A Oarnot engine requires 40 kJ/sec
A. 0.865 from the hot source. The engine 49. The vapor that has temperature
B. 0.765 produces 18 kw of power and the equal to the saturation temperature
C. 0.465 temperature of the .sink is 24.5°C. What corresponding to an existing pressure.
D. 0.538 * is the temperature of the hot source in A. Wet vapor
oc? B. Saturated vapor *
Annual Plant use factor A. 545.5 C. Superheated vapor
B. 240.8 D. All of the above
annual energy produced '
c. 211.9
Equipment Rating x No.of hrs operation 50. The ones defining the physical
D. 267.9 *
6 condition of the fluid are called:
= 760xl0 = 0 _5385 Tc = 24.5 + 273 = 297.5°K A. Thermodynamic properties
I 80,000( 7840) B. State properties *
.
Effi1c1ency = -
w =-
~-~
- C. Cycle properties
43. From Problem No. 37, Find the fuel aA TH D . Substance properties
economy in kw-hrs per liter of fuel.
_!! = TH - 197.5
A. 6.583
40 TH
B. 4.064 *
c. 5.502 TH = 540.9°K
D . 3.504 T H = 540.9 -273 = 267.9°C

Fuel economy

REFRESHER MANUAL 2nd Edition by JAS TORDILLO REFRESHER MANUAL 2nd Edition by JAS TORDILLO
Day 40 - EXAM I 40 - 1
Power and Industrial Plant Engineering
5. Helium is contained in a 2.5-m0 rigid
DAY 40- EXAM volume at 50°C and 220 kPa. Calculate
the heat transfer needed to increase the
1. Air enters a compressor at pressure to 1100 kPa.
atmospheric conditions of 30°C and 80 A. 2200 kJ
kPa and exits at 900 kPa and 220°C. B. 3300 kJ
Calculate the rate of heat transfer if the C. 4400 kJ
power input is. 300 kW. The air exits at D. 4300 kJ
25 m/s through an exit diameter of 10
em.
6. A 8-kg block copper at 290°C is
A. -62.7 kw
submerged in 30 liters of water at 0°C
B. -72.6 kw
contained in an insulated tank. Estimate
C. -76.2 kw
the final equilibrium temperature. Use
D. -89.2 kw
Cp of copper equal to 0.39 kJ/kg-C.
A 3.04°C
2. A hydraulic turbine operates on a B. 5.04°C
stream in which 120 kg/s of water flows. C. 7.04°C
Estimate the maximum power output if D. 10.04°C
the tu rbine is in a dam with a distance of
50 m from the surface of the reservoir to 7. A piston-cylinder arrangement
the surface of the backwater. contains 0.03 m3 of air at 60°C and 500
A. 28,86 kw kPa. Heat is added in the amount of 90
B. 38.86 kw kJ and work is done by a paddie wheel
C. 48.86 kw until the temperature reaches 600°C. If
D. 58.86 kw the pressure is held constant how much
paddle-wheel work must be added to
3. Water enters a radiator through a 4- the air? Assume constant specific
cm-diameter hose at 0.05 kg/s. It heats.
travels down through all the rectangular A. 2.25 kJ
passageway on its way to the water B·. 3.45'kJ
pump. The passageway are each 10 x C. 5.25 kJ
1 mm and there are 800 of them in a D. 7.85 kJ
cross section . How long does it take
water to traverse from the top to the 8. A 7.5-hp fan is used in a large room
bottom of the 80-cm-hig h radiator? to provide for air circulation. A~s um i ng
A 1.13min a well-insulated, sealed room, determine
B. 2.1 3 min the internal energy increase after 1 h of
C. 3.13 min operation.
D. 4.14 min A. 20.142 X 106 J
6
B. 30.142 X 10 J
4. The air in the cylinder of an air . C. 40.142 X 106 J
compressor is compressed from 100 D. 50.142 X 106 J
kPa to 12 MPa. Determine the work
. requ ired if the air is initially at 100 °C. 9. The heat transfer from a 3-m-
A. 573 kJ/kg diameter sphere to a 23 °C air steam
B. 684 kJ/kg over a time interval of one hour is 4000
C. 783 kJ/kg kJ. Estimate the surface tem;:Jerature of
D. 829 kJikg the sphere if the heat transfer coefficient
is 10W/m2 K. ·
A. 16.93 °C

REFRESHER MANUAL 2nd Edition by JAS TORDILLO


40 - 2 I Day 40 - EXAM Day 40- EXAM 140-3
Power and Industrial Plant Engineering
B. 19.93 uc C. -297.5 Btu
C. 26.93 °C D. -197.5 Btu maintain a constant pressure. How Determine the mass flow rate of steam
D. 66.93 °C much heat in kJ/kg is required to raise in kg/sec.
15. A typical steam generator with an the temperatu re of this air to 550°C? A. 1.0
10. A 15-m-long by 4-m-high wall is efficiency of 83% is producing 12 kg/sec A. 480.50 B. 1.3
com~osed of an insulation layer with R = of steam. The enthalpy of the entering B. 528.25 c. 1.7
2 m -KfiN and a wood layer with R = water is 130.5 kJ/kg and is superheated C. 370.75 D. 2.0
0.1 5 m2-KfiN. Estimate the heat transfer to an enthalpy of 3304.2 kJ/kg. The fuel D. 104.20
rate through the wall if the temperature used has a heating value of 43,500 24. A tank containing liquid nitrogen at -
difference is 30 °C. kJ/kg. The AIF ratio by weight is 18. 20. Air is compressed in an insulated 180°C is suspended in a vacuum shell
A. 330W Determine the amount of air needed in piston-cylinder device. Using constant by three stainless steel rods 1.2 em in
B. 440W kg/hr. specific heats and treating the process diameter and 3 meters long with a
C. 550W A 36,371.5 kg/hr as internally reve rsibl~, Determine the thermctl conductivity of 18.5 W/m-°C. If
D. 720W B. 46,873.5 kg/hr amount of work required to compress the ambient air outside the vacuum shell
C. 58,378.5 kg/hr form 100 kPa, 25°C to 1.5 MPa, 800°C. is 23°C, ca lculate the magnitude of the
11. The drive shaft in an automobile D. 68,352.5 kg/hr A. 178.40 kJ/kg conductive heat flow in watts along the
delivers 180 N-m of torque as it rotates B. 298.50 kJ/kg support rods.
at 3500 rpm. Calculate the horsepower 16. A spherical tank 10 m in diameter C. 512.40 kJ/kg A. 0.942
delivered. contains methane at 400 kPa and 33°C D. 1430 kJ/kg B. 0.149
A. 66.4 hp after 15 kg had been used. Calculate C. 0.249
B. 77.4 hp the original mass of methane in the tank 21 . A refrigerator has a coefficient of D. 0.429
C. 88.4 hp in kg. performance of 2.2. How much work in
D. 101 hp· A. 289.36 kg kJ must be supplied to this refrigerator 25. Heat is transferred from hot water to
B. 489.36 kg for it to reject 1500 kJ of heat? an oil in a double pipe counterflow heat
12. A pressurized can contains air at a c. 589.36 kg A. 468.75 . . exchanger. Water enters the outlet pipe
gage pressure of 44 psi when the D. 689.36 kg B. 578.75 at 100°C and exits at 50°C while the oil
temperature is 74°F. The can will burst C. 680.25 enters the inner pipe at 30°C and exits
when the gage pressure reaches 300 17. A mixture composed of 70% carbon D. 725.50 at 60°C. Calculate the log mean
psi. At what temperature will the can dioxide and 30% helium by volume temperature difference.
burst? analysis is contained at 2.5 MPa. What 22. A Carnot heat pump uses thermal A 23.54
A. 1402.8 °F is the partial pressure in kPa of the reservoirs at 25°C and 59°C. How much B: 28.85
B. 2402.8 °F helium in this mixture? power does this pump consume to C. 32.75
C. 3402.8 °F A. 750 produce a 120 kw heating effect? D. 35.65
D. 4402.8 °F B. 600 A. 12.2 kw
C. 550 B. 18.3 kw 26. A pump delivers 4 m3/min of water
13. An automobile tire with a volume of D. 900 C. 32.7 kw from 103 kPa to 350 kPa. The inlet and
0.75 m3 is inflated to a gage pressure of D. 40.7 kw outlet pipe diameters are 15 em and 12
200 kPa. Calculate the mass of air in 18. If air contained in a room loses heat em respectively. Both pipes are on the
the tire if the temperature is 25 °C. to the surroundings at a rate of 60 23. The following data were obtained in same level. The inlet and outlet
A. 0.5 kg kJ/min while work is supplied to the a small power generating plant: temperatures are both 25°C and the
3
B.1 .5kg room by computer, 1V, and lights at a Power developed by the steam turbine mass density of water is 997 kg/m
c. 2.64 kg rate of 1.5 kW. What is the net amount = 3200 kw Calculate the horsepower capacity of
D. 3.75 kg of energy change in kJ of the air in the Heat supplied to the steam in the steam the pump.
room during a 45-minute period? generator = 3900 kJ/kg A. 20
14. Two pounds of air is compressed A. 1100 kJ Heat rejected by the steam to the B. 15
from 25 psia to 300 psia while B. 1350 kJ cooling water in the condenser C. 23
maintaining the temperature constant at c. 950 kJ = 2300 kJ/kg D. 10
120 °F. Calculate the heat transfer D. 780 kJ Power required by the feedwater pump
needed to accomplish this process. to return the condensate to the steam 27. If 10 kg of air at 27°C are heated at
A. -497.5 Btu 19. Air at 1 MPa, 25°C is contained in a generator = 15 kw constant press·ure until the absolute
B. -397.5 Btu piston-cylinder device that is arranged to

REFRESHER MANUAL 2nd Edition by JAS TORDILLO REFRESHER MANUAL 2nd Edition by JAS TORDILLO
40 - 4 I Day 40 - EXAM Day 40 - EXAM I 40 - 5
temperature doubles, calculate the heat 32. A boiler uses 2300 kg of coal per
Power and Industrial Plant Engineering
required in kJ. hour. The amount of air needed for the
36. A gas moving with a velocity in 42. The velocity always changes from
A. 1785 combustion of one kg coal is 18 kg. If
excess of approximately 300 ft/sec ( 100 supersonic to subsonic across a shock
B. 2552 ash loss is 8%, calculate the mass of
m/s). wave and there is no loss of heat
c. 3018 gas entering the chimney in kg/hr.
A. low-velocity gas energy, therefore a shock wave is
D. 6245 A. 43,516 B. high-velocity gas A. An adiabatic process
B. 40,600 B. Polytropic process
C. medium-velocity gas
28. A reversed Carnot cycle is used for C. 47,300 C. Isentropic
D. average-velocity gas
cooling . The input work is 15 kw, while D. 58,950 D. Hydraulic process
the COP is 4.2. Calculate the
37. A type of turbine that consists of a
refrigerating effect in tons of 33. An ideal gas turbine operates with a 43. A drop in internal energy, is seen as
rotating drum with small nozzles
refrigeration. pressure ratio of 9 and temperature a· drop in
(reaction jets) located around the drum's
A. 17.9 limits of 25°C and 11 00°C. The energy
periphery. A. Pressure
B. 15.4 input in the high temperature heat
A. Reaction turbine B. Enthalpy
c. 12.6 exchanger is 300 kw. Determine the air
B. Impulse turbine C. Volume
D. 10.1 flow rate in kg/hr. D. Energy
C. Water turbine
A. 1650
D. Pelton turbine
29. A pure Diesel cycle operates with a B. 1325 44. This is an ad iabatic flow with friction,
compression ratio of 19, and cut-off, ratio c. 1859 38. If the gas flow is adiabatic and which would be approximated as flow
of 3.0. What is the rate of heat input in D. 1732 through a long, insulated duct.
frictionless (that is, reversible), the
kJ/sec required for this cycle to produce A . Rayleigh flow
entropy change is zero and the flow is
400 hp? 34. The surface of a furnace wall is at a
known as B. Fanno flow
A. 229 temperature of 1500°C. The outside
A. isentropic flow C. Radial flow
B. 349 wall temperature is 45°C. The furnace
B. polytropic flow D. Ideal flow
C. 499 wall is made of 25 em of refractory
C. isovolumic flOw
D. 599 material having a thermal conductivity of
D. isenthalpic 45. A high gas velocity is often achieved
1.4 W/m-°K. The outside wall is steel, 1 at the expense of what type of energy.
30. A simple Rankine cycle produces em thick with thermal conductivity of 47 A. Kinetic energy
39. The point where sonic velocity has
50 MW of power, 60 MW of process W/m-°K. Calculate the thickness in B. Internal energy
been achieved is known as a
heat and rejects 70 MW of heat to the meters of oi:ick to be installed in c·. Potential energy
A. median
surroundings. What is the utilization between the refractory material and D: GraVitational energy
B. critical point
factor of this cogeneration cycle steel if its thermal conductivity is 0.28
C. maximum
neglecting the pump work? W/m-°K and the heat loss is not to 46. This is frictionless flow with heating.
2 D. minimum
A. 55.55% exceed 825 W/m . Flow can be assumed frictionless if the
B. 77.77% A. 0.443 cluct is short. The heat can come from
40. A type of turbine which is
c. 61.11% B. 0.475
characterized by stationary jets chemica l reactions, phase changes,
D. 81.22% c. 0.502 discharg ing against vanes mounted on electrical current, or external sources.
D. 0.613
the peri phery of a wheel. A. Rayleigh flow
31. A centrifugal pump delivers 2.0 B. Fanno flow
A. Reaction turbine
m 3 /sec of water against a head of 30 m 35. Calculate the compressor work in kJ
C. Turbine flow
B. Impulse turbine
when rotating at 1200 rpm. The required to compress 1 kg of an ideal D. Terrestrial flow
C. Francis turbine
.diameter of its impeller is 60 em. If a gas from an initial volume and pressure
D. Propeller turbine
pump with the same specific speed is to of 0.85 m 3 and 101.3 kPa to a final 47. The ratio of net work out (or net
discharge 7.5 m3/sec at 1000 rpm, what · . pressure of 700 kPa. Compression is
41. The Mach number of a sonic velocity heat in) divided by the heat input.
is its impeller diameter in centimeters? polytropic with n = 1.32. A. Power factor
is
A. 69 A. 212.3 B. Thermal efficiency
A . M<1 .0
B. 99 I B. 248.3 C. Brake power efficiency
B. M = 1.0
c. 61 c. 305.8 C. M > 1.0 D. Power ratio
D. 45 D. 442.7
D. M = 10

REFRESHER MANUAL 2nd Edition by JAS TORDILLO REFRESHER MANUAL 2nd Edition by JA~ TORDILLO
40 - 6 1 Day 40 - Solution Day 40 - Solution 1 40 - 7
48. The ratio of the actual to ideal Power and Industrial Plant Engineering
(isentropic) energies extracted from the DAY 40 - SOLUTION
flowing gas. m = pN,A, = PN2.A2 6. A 8-kg block copper at 290uC is
A Nozzle efficiency 1. Air enters a compressor at v2 = ~= 0.05 . submerged in 30 liters of water at 0°C
B. Pipe efficiency atmospheric conditions of 30°C and 80 P2A2 (1oooX(sooxo.o1Xo.oo1)] contained in an insulated tank. Estimate
C. Work ratio kPa and exits at 900 kPa and 220°C. = 0.00625 m/s the final equi librium temperature. Use
D. Pressure ratio Calculate the rate of heat transfer if the Cp of copper equal to 0.39 kJ/kg-C_
power input is 300 kW. The air exits at A. 3.04°C
49. The purpose of this component is to 25 m/s through an exit diameter of 10 t = .!::.
V -
- 0.80
0.00625 = 128 s or 2.13 min B. 5.04°C
increase the total energy content of the em. C. 7.04°C *
A. -62.7 kw* D. 10.04°C
fluid flowing through it. 4. The a1r 1n the cylinder of an air
A . Pump B. -72.6 kw
C. -76.2 kw compressor is compressed from 100 m·c(Cp)c( ~T)c = mw(Cp)w(~T)w
B. Turbine kPa to 12 MPa. Determine the work
C. Francis D. -89.2 kw
required if the air is initially at 100 °C. (8)(0.39)(290- T2)
D. Motor A. 573 kJ/kg = (0.03)(1000) (4.18)(T2-0)
m =pAY .!:__AV B. 684 kJ/kg
50. These are very thin (several RT T2 =7.04°C
c. 783 kJ/kg *
molecules thick) and separate areas of 900
= (n)(O 05)2 (25) D. 829 kJ/kg 7. A piston-cylinder arrangement
radically different thermodynamic (o.2s1 X22o + 273) ·
properties. .. contains 0.03 m3 of air at 60°C and 500
~~ r-l)lk
= 1.2489 kg/s kPa. Heat is added in the amount of 90
A. Shock waves T2 = T, (
B. Heat waves kJ and work is done by a paddle wheel
Q = (1.2489){1.00)(220- 30) + (- 300) · until the temperature reaches 600°0. If
C. Radiation = -62.71 kW
D. Energy waves ; (373)(12000)(J.4- 1)1!.4 the pressure is held constant how much
1464.7 paddle-wheel work must be added to
2. A hydraulic turbine operates on a !-00
the air? Assume constant specific
stream in which 120 kg/s of water flows. K
heats.
W = ~u = -Cv(T2- T,)
rrr )
Estimate the maximum power output if
the turbine is in a dam with a distance of
50 m from the surface of the reservoir to
= -(0.717)(1464.7- 373)
= -782.77 kJ/kg
A. 2.25 kJ
B. 3.45 kJ
OAY 40 - ANSWER KEY
the surface o'f the backwater.
c. 5.25 kJ *
1. A 16.0 31. B46. A D. 7.8~ kJ
2. 0 17. A 32. A 47. B A 28.86 kw 5. Helium is contained in a 2.5-m 3 rigid
3. B 1B. B 33. B 48. A B. 38.86 kw volume at 50°C and 220 kPa. Calculate
4. C 19. B 34. A49. A Q- Wpaddle = m(h2- h1) = mCp (T2- T,)
5. B 20. A 35. A 50. A
c. 48.86 kw the heat transfer needed to increase the
6. C 21. A 36. B D. 58.86 kw* pressure to 1100 kPa.
A. 2200 kJ _ PV (500)(0.03) = 0.15695 kg
7. C 22. A 37. A
m - RT = (0.287X273 + 60)
8. A 23. 0 38. A Wr =(120)(9.81)(50) B. 3300 kJ *
9. C 24. D
10.025. B
39. B
40. B
=58,860 W or 58.86 kW C. 4400 kJ
D. 4300 kJ W paddle = Q- mCp (T2- T1)
11 . C26. C 41. B = 90- (0.15695)(1.00)(600--60) '
12.B27.C 42.A 3. Water enters a radiator through a 4-
cm-diameter hose at 0.05 kg/s. It PV =- 5.247 kJ
13. C 28. A
14. 029. C
43. B
44. B travels down through all the rectangular
Q =m~u = mCv ~T = - Cv(T2- T,)
RT
15.030. C 45. B passageway on its way to the water P1 P2 8. A 7.5-hp fan is used in a large room
to provide for air circulation. Assuming
~ pump. The passageway are each 10 x T1 = T2
a well-insulated, sealed room, determine
1 mm and there are 800 of them in a
220 1100 the internal energy increase after 1 h of
cross section. How long does it take
water to traverse from the top to the
323 =T2 operation.
bottom of the 80-cm-high radiator? T21615K A. 20.142 X 106 J *
A 1.13 min B. 30. 1..;2 X 106 J
B. 2.13 min* o= (22ox2.5) (3. 11 6)(1615-323) C. 40.142 x 106 J
C. 3.1 3 min (2.077Xm) D. 50.142 X 106 J
D. 4.14 min = 3300.53 kJ
REFRESHER MANUAL 2nd Edition by JAS TORDILLO REFRESHER MANUAL 2nd Edition by JAS TORDILLO
F
~

40 - 8 I Day 40 - Solution
Day 40 - Solution 140 - 9
-W=t.U Power and Industrial Plant Engineering
65,973.60 = 88.4 hp
W = (-7.5 hp)(1 hrJ(746W/hp)(3600 s/hr) Hp = 746
= -20.142 X 10 J PV = mRT
Q = mRT ln.!}_
400(268.08) = me·~~·
3
t.U = -(-20.142 X 106) = 20.142 X 106 J 12. A pressurized can contains air at a Pz )C33 + 273)
gage pressure of 44 psi when the =(2.0)(53'.3)x (580°R) m = 674.37 kg
9. The heat transfer from a 3-m- temperature is 74°F. The can will burst
diameter sphere to a 23 °C air steam when the gage pressure reaches 300 1
Btu )I 25 Original mass = 674.36 + 15
over a time interval of one hour is 4000 psi. At what temperature will the call' x ( 77 8 ft -
lbf n 300 = 689.36 kg
kJ. Estimate the surface temperature of burst? = -197.47Btu
the sphere if the heat transfer coefficient A. 1402.8 °F 17. A mixture composed of 70% carbon
is 10 W/m 2 K. B. 2402.8 °F " 15. A typical steam generator with an dioxide and 30% helium by volume
A . 16.93 °C c. 3402.8 °F efficiency of 83% is producing 12 kg/sec analysis is contained at 2.5 MPa. What
B. 19.93 °C D. 4402.8 °F of steam. The enthalpy of the entering is t~e partial pressure in kPa of the
c. 26.93 °C. water is 130.5 kJ/kg and is superheated helium in this mixture?
D. 66.93 °C _ mRT1 _ mRTz to an enthalpy of 3304.2 kJikg. The fuel A. 750 *
V ------ used has a heating value of 43,500 B. aoo
PI P2
Q = hcA(T,- T.,)t.t kJ/kg. The A/F ratio by weight is 18. C. 550
4x1 06 = 10 X 41t X (1.5)2 (Ts- 23) X 3600 Tl = T2 Determine the amount of air needed in D. 900
T. = 26.93 °C Pi p2 kg/hr.
.. A. 36,371 .5 kg/hr PHe = (~•) PT = 0.30(2500)
10. A 15-m-long by 4-m-high wall is T2 = T, P2 = (74 + 460) B. 46,873.5 kg/hr
PI = 750 kPa
com~osed of an insulation layer with R = C. 58,378.5 kg/hr
2 m -KNI/ and a wood layer with R = (300 + 14.7;(i44) .D. 68,352.5 kgthr •
2 (44 + 14.7XI44) 18. If air contained in a room loses heat
0.15 m -KJW. Estimate the heat transfer
rate through the wall if the temperature = =
2862.8 °R 2402.8 °F Boiler Eff ,;; m,(h,- h,)
to the surroundings at a rate of 60
kJ/min while work is supplied to the
difference is 30 °C. mrQh
room by computer, TV, and lights at a
A . 330W 13. An automobile tire with a volume of
0. 75 m 3 is inflated to a gage pressure of 0.83 = 12(3600)(3304.2-130.5) rate of 1.5 kW. What is the net amount
B. 440W
200 kPa. Calculate the mass of air in mr(43,SOO) of energy change in kJ of the air in the
C. 550W
room during a 45-minute period?
D. 720 W* the tire if the temperature is 25 °C.
A 0.5 kg m, = 3797.36 kg/hr A 1100 kJ
B. 1.5 kg B. 1350 kJ"
Rtotal = Rtnsulation + Rwooo
= 2 + 0.5 = 2.5 m2.KM/ c. 2.64 kg* malr = 18 C. 950 kJ
mru• l D. 780 kJ
D. 3.75 kg
Q = _A_IlT = ISx4 X 30 = 720 W
P = 200 + 101.325 = 301.325 KPa
matr
3797.36 = 18 01 = 60 kJ/min = 1.0 kJ/sec
Rtotal 2.5 Oz = 1.5 kJ/sec
T = 25 + 273 = 298 K
11. The drive shaft in an automobile
mair = 68,352.55 kg/hr
Onet = (1 .5-1 .0)(45)(60) = 1350 kJ
delivers 180 N-m of torque as it rotates m= PV = (301.325X0.75) ·= 2 .64 kg
16. A spherical tank 10 m in diameter
at 3500 rpm. Calculate the horsepower RT (0.287(298 K) 19. Air at 1 MPa, 25°C is contained in a
contains methane at 400 kPa and 33°C
delivered. piston-cylinder device that is arranged to
after 15 kg had been used. Calculate
A. 66.4 hp maintain a constant pressure. How
14. Two pounds of air Is compressed the original mass of methane in the tank
B. 77.4 hp much heat in kJ/kg is required to raise
in kg.
c. 88.4 hp" from 25 psia to 300 psia while
A . 289.36 kg the temperature of this air to 550°C?
D. 101 hp maintaining the temperature constant at A. 480.50
B. 489.36 kg
1 120 °F. Calculate the heat transfer B. 528.25 *
w = (3500) (2n) ( ) = 366.52 rad/s needed to accomplish this process.
C. 589.36 kg
C. 370.75
60 ·D. 689.36 kg •
A -497.5 Btu D. 104.20
Power= Tw = (180)(366.52)
B. -397.5 Btu
j, = 65,973.60 w C. -297.5 Btu V = ~rrr 3 = ~rr(S) 3 = 3
268.08 m 1 a = mCp(T2 _ T 1 )
D. -197.5 Btu* Q = 1.0062(550- 25) = 528.25 kJikg
REFRESHER MANUAL 2nd Edition by JAS TORDILLO REFRESHER MANUAL 2nd Edition by JAS TORDILLO
L
40 - 10 1 Day 40 -Solution Day 40 - Solut ion 140 - 11
Pow er and Industrial Plant Engineering
20. Air is compressed in an insulated We = QR • QA = 120-107.76
piston-cylinder device. Using constant = 12.24 kw at 60°C. Calculate the log mean A. 1785
specific heats and treating the pro~s temperature difference. B. 2552
as internally reversible, Determine the' 23. The following data were obtained in A. 23.54 c. 3018 *
amount of work required to compress a small power generating plant: B. 28.85 * D. 6245
form 100 kPa, 25°C to 1.5 MPa, 800°C. Power developed by the steam turbine c. 32.75
A. 178.40 kJ/kg * = 3200 kw D. 35.65 T1 = 27 + 273 = 300°K
B. 298.50 kJ/kg Heat supplied to the steam in the steam Tz = 2(300) = 600°K
c. 512.40 kJ/kg generator = 3900 kJ/kg LMTD =
4TA-4T s
In ~
D. 1430 kJ/kg Heat rej~cted by the steam to the Q = mCp(Tz - T1)
cooling water in the condenser Q = 10(1.006)(600 - 300)
= 2300 kJ/kg 6 TA = 100 - 60 = 40°C Q = 3018 kJ
W = nmRT1
n-1
[(!2)";'
P
_ 1] Power required by the feedwater pump 6 T e = 50 - 30 '= 20°C
1
to return the condensate to the steam 28. A reversed Carnot cycle is used for

w = 1.4--1
1.4(0.287)(25+273) (1500)--r.:i'"' - 1]
generator = 15 kw LMTD = 4
°-.!o
1n 20
= 28.85°C cooling . The input work is 15 kw, w hile
the ·COP is 4.2. Calculate the
[ 100
Determine the mass flow rate of steam
1.4-1
in kg/sec. refrigerating effect in tons of
= 178.35 kJ/kg A. 1.0 26. A pump delivers 4 m3/min of water refrigeration .
B. 1.3 from 103 kPa to 350 kPa. The inlet and A. 17.9 *
21. A refrigerator has a coefficient of c. 1.7 outlet pipe diameters are 15 em and 12 . B. 15.4
performance of 2.2. How much work in D. 2.0 * em respectively. Both pipes ar1= on the C. 12.6
kJ must be supplied to this refrigerator same level. The inlet and outlet . D. 10.1
for it to reject 1500 kJ of heat? Heat Balance: temperatures are both 25°C and the
A. 468.75 * ~ + Wp = Wr + QR mass density of water is 997 kg/m3 . COP = QA
B. 578.75 m.(3900) + 15 = 3200 + m.(2300) Calculate the: horsepower capacity of We
c. 680.25 (1600) ms = 3185 the pump.
D. 725.50 A. 20 4.2 : QA
m. = 1.99 kg/sec 15
B. 15
COP =
QA-
We -
~
QrQA 24. A tank containing liquid nitrogen at - c. 23 * Q A = 6 3 kw
180°C is suspended in a vacuum shell D. 10 ~ = 63 kwx ~ = 17.92TOR
' 3.516 kw
QA by three stainless steel rods 1.2 em in 3 3
2.2 : 1500- QA diameter and 3 meters long with a ' Q = 4m /min = 0.0667 m /sec
29. A pure Diesel cycle operates with a
thermal conductivity of 18.5 W/m-°C. If
compression ratiq of 19, and cut-off ratio
QA: 1Q31 .25kJ the ambient air outside the vacuum shell Q -~=3.77 m/s
vl = A,- i<o.IS)2 of 3.0. Wh at is the rate of heat input in
is 23°C, calculate the magnitude of the
Q _ o.o667 = 5.89 m/s kJ/sec required for this cycle to produce
We = QR- QA = 1500-1031.25 conductive heat flow in watts along the Vz = ~ - 1<o.n) 2 400 hp?
= 468.75 kJ support rods.
A. 229
A. 0.942 H= P2 - P1 +vJ -v} B. 349
22. A Carnot heat pump uses thermal B. 0.149 Zg
reservoirs at 25°C and 59°C. How much C. 0.249 H = ~+
y
2
(5.89) - (3.77)
2 c. 499 *
D. 599
power does this pump consume to D. 0.429 * 9 .81 2 (9.81)
H = 26.22 m
produce a 120 kw heating effect?
A. 12.2 kw * Oc = hA(T2- T1) Eff
1 [cr/
= 1 - ;p - 1)]
k(rc- 1)
B. 18.3 kw Wp = QyH
Oc = 18.5 [~ (0.012) 2 ) [25- ( -180)] Wp = (0.0667)(9.81)(26.22)
C. 32.7 kw 31
·• - > ]
1
D. 40.7 kw Oc = 0.429W = 17.15 kw= 23 hp Eff = 1 - -
1
-
(19)1·•-1
[<1.4(3- 1)
= 0.5979

25. Heat is transferred from hot water to 27. If 10 kg of air at 27°C are heated at
COP = ~ = .....!..!,_ 0.5979 = 400
Qa-QA TH-TL an oil in a double pipe counterflow heat constant pressure until the absolute QA
~=~ exchanger. Water enters the outlet pipe temperature doubles, calculate the heat
59-25 120- QA
at 100°C and exits at 50°C while the oil required in kJ . QA = 669 hp = 499 kw (kJ/sec)
~ = 107.76 kw enters the inner pipe at 30°C and exits
REFRESHeR MANUAL 2nd Edition by JAS TORDILLO REFRESHER MANUAL 2nd .Edition by JAS TORDILLO

40 - 12 I Day 40 • Solution Day 40- Solution 140- 13
Power and Industrial Plant Engineering
30. A simple Rankine cycle produces ma + mr = m9 + 0.10(mr)
50 MW of power, 60 MW of process 41,400 + 2,'300 = m9 + 0.08(2300) 35. Calculate the compressor work in kJ 40. A type of turbine which is
heat and rejects 70 MW of heat to th_e m9 = 43,516 kg/hr required to compress 1 kg of an ideal characterized by stationary · jets
surroundings. What is the utilization gas from an initial volume and presSU£e discharging against vanes mounted on
factor of this cogeneration cycle 33. An ideal gas turbine operates with a of 0.85 m3 and 101.3 kPa to a final the periphery of a wheel.
neglecting the pump work? pressure ratio of 9 and temperature pressure of 700 kPa. Compressi.on is A . Reaction turbine
A. 55.55% limits of 25°C and 11 00°C. The energy polytropic with n = 1.32. B. Impulse turbine *
B. 77.77% input in the high temperature heat A. 212.3 * C. Francis turbine
c. 61.11% * exchanger is 300 kw. Determine the air B. 248.3 D. Propeller turbine
D. 81 .22% flow rate in kg/hr. c. 305.8
A. 1650 D. 442.7 41 . The Mach number of a sonic velocity
Utilization Factor =
Maximum Demand B. 1325 * is
Rated Capacity
50+60
c. 1859 W = nP1 v; [(!2)'";' _ 1] A. M < 1.0
50+60+70 D. 1732 n-1 P1 B. M = 1.0 *
C.M>1.0
k-1
= 0.611 = 61.11%
:!! =
T1
(!2)"
P1
w = 1.32(101.3)(0.05)
1.32-1
[(~)~:.~;'
101.3
- 1] D. M = 10

31. A centrifugal pump delivers 2.0 ___2L = (9) '·;_;' 42. The velocity always changes from
m3/sec of water against a head of 30 m 25+273 w = 212.3 kJ supersonic to subsonic across a shock
T2 = 558.28°K wave and there is no loss of heat
when rotating at 1200 rpm. The
diameter of its impeller is 60 em. If a 36. A gas moving with a velocity in energy, therefore a shock wave is
pump with the same specific speed is to
T3 = 1100°C = 1373°K excess of approximately 300 ftfsec (100 A. An adiabatic process *
discharge 7.5 m3/sec at 1000 rpm, what m/s). B. Polytropic process
is its impeller diameter in centimeters?
QA = mCp(T3- T2) A. low-velocity gas C. Isentropic
300 = m(1.0)(1373- 558.28) B. hlgh-velot::lty gas •
A . 69 D. Hydraulic process
m = 0.36822 kg/s = 1325.60 kg/hr C. medium-velocity gas
B. 99 *
C. 61 D. average-velocity gas 43. A drop in internal energy, is seen as
34. The surface of a furnace wall is at a
D. 45 a drop in
'"temperature of 1500°C. The outside 37. A type of turbine that consists of a .A. Pressure
Q, Q, wall tempef~ture is 45°C. The furnace rotating drum with small nozzles _B. Enthalpy*
N 1 D~ = N2 D: wall is made of 25 em of refractory
material having a thermal conductivity of
(reaction jets) located around the drum's C. Volume
2 7.5 periphery. D. Energy
1200(60)3 = 10000~ 1.4 W/m-0 K. The outside wall is steel, 1 A. Reaction turbine *
D2, = 99 em em thick with thermal conductivity of 47 B. Impulse turbine 44. This is an adiabatic flow with friction,
W/m-°K. Calculate the thickness in C. Water turbine which would be approximated as flow
32. A boiler uses 2300 kg of coal per. meters of brick to be installed in D. Pelton turbine through a long, insulated duct.
hour. The amount of air needed for the between the refractory 'material and
A. Rayleigh flow
combustion of one kg coal is 18 kg. If steel if its thermal conductivity is 0.28 38. If the gas flow is adiabatic and B. Fan no flow*
ash loss is 8%, calculate the mass of W/m-°K and the heat loss is not to frictionless (that is, reversible). the C. Radial flow
gas entering the chimney in kg/hr. exceed 825 W/m2 . entropy change is zero and the flow is D. Ideal flow
A. 43,516 * A. 0.443 * known as
B. 40,600 B. 0.475 A. Isentropic flow * 45. A high gas velocity is often achieved
C. 47,300 C. 0.502 B. polytropic flow at the expense of what type of energy.
D. 58,950 D. 0.613 C. lsovolumic flow A. Kinetic energy
75in D. lsenthalpic B. Internal energy *
~ = mn = 18 kg air Q T2 -T1
p mr kg fuel A' = xl•"•+:!:l C. Potential energy
kl k, k, 3.75 lbs 39. The point where sonic velocity has D. Gravitational energy
75} (3.75) been achieved is known as a
m. = 18(2300) = 41,400 kg/hr 825 = •.•!5~~:45.l' 1 A. median 46. This is frictionless flow with heating.
u+47+o.z. B. critical point* .
mr = 2300 kg/hr x = 0.443 m Flow can be assumed frictionless if the
C. maximum duct is short. The heat can come from
D. minimum
REFRESHER MANUAL 2nd Edition by JAS TORDILLO REFRESHER MANUAL 2nd Edition by JAS TORDILLO
40 - 14 1 Day 40 - Solution
chemical reactions, phase changes,
electrical current, or external sources.
A. Rayleigh flow"
B. Fanno flow
C. Turbine flow
D. Terrestrial flow

4 7. The ratio of net work out (or net


heat in) divided by the heat input
A. Power factor
B. Thermal efficiency"
C. Brake power efficiency
PART 3
D. Power ratio

48. The ratio of the actual to ideal


(isentropic) energies extracted from the
flowing gas.
A. Nozzle efficiency "
B. Pipe efficiency
C. Work ratio
D. Pressure ratio

49. The purpose of this component is to MACHINE DESIGN


increase the total energy content of the
fluid flowing through it
A. Pump"
ENGINEERING MATERIALS
B. Turbine
C. Francis MACHINE SHOP PRACTICE
D. Motor

50. These are very thin (several


molecules thick) and separate areas of
radically different thermodynamic
properties.
A. Shock waves "
B. Heat waves
C. Radiation
D. Energy waves

REFRESHER MANUAL 2nd Edition by .JAS TORDILLO


Day 41- EXAM 141 -1
Machine Design
C. cutting
DAY 41 -EXAM D. reaming

1. The major element/component of a 8. Commonly used for tubes of water


babbitt metal: tube boilers made of -large pipes which
A. antimony a, number of smaller ones lead to
B. tin consuming points. This is called _ _.
,C. lead A. header
D. zinc B. manifold
C. crotch
2. A given body in static equilibrium D. line pipe
means. that the same body cannot
9. This is a bearing, material with a low
A. have any displacement coefficient of friction, resistance to
B. move by. any force chemical, self-lubricated and can be
C. have acceleration utilized In a wide temperature, range.
D. is irremovable A. stainless steel
B. Teflon
3. Pipe color for acid. C. cast iron
A. violet D. chromium
B. red
C. blue 10. A simple term for is
D. white force:
A. shear stress
4. This acts as deoxidizer and promotes B. linear momentum
fine grain in steel melting. C. concurrent forces
A. silicon 0 . push and pull
B. tin
C. pig iron 11 . A metal characteristic that absorbs
D. scrap iron .extreme deformation without rupture is
called,_ _ _ __
5. Cold working or similarly in peening A . ductility
condition steel, material tend to be B. toughness
A. harder C. malleability
B. more ductile D. plasticity \
C. in plasticity condition
D. more malleable 12. The ideal limit of teeth that sprockets
can have in operation is
6. Measuring instrument to check ft I (minimum/maximum).
hardness of rubber, Plastic and other A. 17/127
soft elastic materials. B. 20/130
A. pyrometer c. 12/80
B. durometer D. 10/15
C. plammeter
D. tacho 13. A chromium SAE 4340 steel 6
Inches diameter x 24 inches long
7. is an operation of shafting weighing 87.3 kgs. How about
pro.du.Qi]:lg a circular hole by the removal the weight (in lbs) of an AISI 1030 steel
oi.a_solidrnetat-- · shaft of same dimension?
A. drilling A. 192.3
B. shaping
REFRESHER MANUAL 2nd Edition by JAS TORDILLO
41-21 Day 41- EXAM Day 41 -EXAM 141 - 3
Machine Design
B. 240.4 C. fight
c. 80.4 D. heavy 26. A pair of gear and pinion having a C. 1 Y,
D. 87.4 ratio of 5 with a circular pitch of 0.5 inch,: D. 4
20. Internal and/or residual stress in a compufe the center distance, in inches,
14. Cutting lubricants used in drilling steal work like heavy welding and/or between the gear of 70 tooth and pinion. 32. A cargo truck is traveling in the
reaming and tapping for aluminum and casting can be remedied by___.___. highway at a velocity of 30 rn/s. If it is
A. 6.18
topper. A . forging B. 7.05 decelerating at a rate of 4 m per sec,
A. turpentine B. quenching
B dry c. 6.70 how far, in meters, does it travel before
C. cold working D. 8.61 stopping?
C. soda water
D. tempering A. 90.5
D. kerosene
27. Design a safe shaft diameter for a B. 101.5
1~ is a material ability to resist
21. SAE 1030 Carbon steel contains power. transmitting s haft to transmit 40 c. 12.5
deformation tinder stress. -=-~-::-:::--::-:- %C. hp at a shaft speed of 205 rpm . D. 112.5
A. steel plate A. 0.28/0.34 A. 2 inches
B. stiffness B. 0.26/0.36 B. 2.4 inches 33. Compute for the circular pitch, in
C. magnesium c. 0.32/0.38 C. 1 Y. inch inches, of a pair of gear and pinion with
D. rigid frame D. 0.22/0.28 D. 2 Y. inches ratio of 5. The center distance. indicated
is at 18. The gear has 90 teeth and the
16. In the-transmission shafting design the 22. AISI 8620 has an average carbon 28. What would be the diameter in pinion has 18 teeth .
allowable maximum twist should not content is in the range of %C.
exceed ___ deg/foot length of the millimeters of a main, power transmitting A. 1.047
A. 0.21 steel shaft SAE 1040 to transmit 150 kw B. 0.074
shaft.
B. 0.19 at 900 rpm? C. 1.740
A. 0.10 C. 0.16/0.25
B. 0.90 A. 2.3 D. 1.470
D. 0.18/0.23% B. 66.4
c. 0.08
D. 0. 12 C. 2.6 34. What is the developed acceleration
23. SAE 46xx is a steel designation for D. 76.5 a roller coaster in meter per sec per sec
A. chromium when the velocity is at 20 m/s at a
17. Dimeoslo.oal variation.
B. carbon
allowed/prescribed In order to secure 29. Compute for the height from which a £adius of curvature of 6500 em?
C. molybderium-chromium pile driver hammer must be drop to A. 4 .61
difference of fits between mating parts.
D. molybdenum-nickel attain c; velocity of 12.8 m/s. Assume no B. 6.15
A. allowance
B. plus or minus air and guide resistance. c. 5.8
24. A file is a piece of high-carbon A. 8.4m D. 8.15
C. fits crucible steel having teeth cut upon its
D. tolerance B. 4.8 m
body by parallel rows. The parts of the C. ~.5m 35. A 36-inch wide conveyor belt feeder
file are: tang, heel and _ _ . D. 5.9 m having a speed of 800 ftlmin moves a
18. High temperature surface heating of
A. single cut 260 kgs of magnetite concentrates.
iron based material or alloyed steel
B. length 30. A material weighing 800 lbs is being Determine the developed kinetiy energy
Above the transforming temperature
C. rough moved on a conveyor belt feeder at a in ftllbs of the materials.
range I then followed by quenching is
D. face speed of 15 ftlsec. Find the kinetic A. 1957
called -:---::--
A. core beefing energy in ft-lbs of the materials. B. 1795
· B. cyaniding
25. This is a tool with hardened- steel A. 2795 c. 1759
points used for scribing circles or laying B. 9000 D. 1579
C flame hardening
off distances. It is adjustable and is c. 4589
D. heat treatment
classified according to size by the D. 7600 36. Find the center distance, in inches,
maximum opening between the points of a 90 teeth gear meshing with 18 teeth
19. Very long center distances can
and this must be slender and sharp. 31 . Determine the number of Y2 inch pinion having a circular pitch of 0.545.
cause catenary tension in the _ __
A vernier caliper diameter set-screws needed to transmit Gear/Pinion ratio is 5.
operation. The ideal design center B. divider
distance is about eighty pitches. 10 hp at a shaft speed of 1200 rpm and A. 8
C. file a shaft diameter of 1 Y, inch. B. 9.36
A. sprocket
D. gage A. 1.3 C. 5
B. chain
B. 2 D. 7 .25

REFRESHER MANUAL 2nd Edition by JAS TORDILLO REFRESHER MANUAL 2nd Edition by ..J.AS TORDILLO
41-41 Day 41- EXAM Day 41 -Solution 141 - ·~
Machine Design
37. A motorcycle slides at a distance of using SAE 1060 material mounting a 12
90 meters in a horizontal asphalt road B. 30,000 ft-lbs
inches pulley at a speed of 170 rpm.
C. 15,000 ft-lbs DAY 41 - SOLUTION
(with rubber to asphalt coefficient of A. 21 .3
friction of 0.6). Compute the motorcycle B. 28.5 D. 10,000 ft-lbs
1. The major ·elemenUcomponent of a
speed in m/s. c. 25.8 .babbitt metal:
A. 18.2 D. 34.5 48. A 60-inch diameter steel Pipe, 3/8.
inch thick, carries Water under a A . antimony
B. 20.5
pressure head of 550 ft. Determine the B. tin*
C. 45.8 43. A pair of gear/pinion of 45 teeth and
hoop stress in the steel. C. lead
D. 32.5 15 teeth, respectively, with center
A. 34,320 psi D. zinc
distance of 40 em, addendum at 0.725/p
38. Find the center distance, in inches, and dedendum at 1/p. The gear B. 19,066 psi
C. 25,780 psi 2. A given body iQ static equilibrium
of a 11 0 tooth gear mesh with a pinion pressure angle is 20 degree. Compute
D. 15,920 psi means, that the same body cannot
having a circular pitch of 0.905. for the diametral pitch in teeth per em.
Gear/pinion ratio is 5. A. 0.60
49. Determine the length of belt needed A. have any displacement
A. 15 B. 0.90 B. move by. any force
B. 30. c. 0.70 for a two flat belt pulleys ·having a
diameters of 70 em and 30 M. The C. have acceleration *
c. 19 D. 0.75
D. is irremovable
D. 37 center distance is 400 em and pulleys
44. A punching machine flywheel rotate in same direction. Assume power
transmitted is 10 hp at 200 rpm . 3. Pipe color for acid.
39. Determine the polar -section accelerates from rest with7 radians/sec2 .
A. 958 em A. violet*
modulus. Zp (in 3 ) of a shaft delivering 5 ln. 10 seconds, how many radians are
B. 589 em B. red
hp at 150 rpm. The diameter of the shaft achieved?
C. 859 em C. blue
is 1 inch and allowable sum of 6000 psi. A. 45 '
D. 895 em D. white
A. 0.175 .B. 250
B. 0.489 c. 35 4. This acts as deoxidizer and promotes
C. 0.196 D. 350 50. A gear set having a gear ratio of 3 is
to be used at a center distance of mm. If fine grain in steel melting.
D 0.895
45. Compute for the polar section the gear has 60 teeth, what must be the A· silicon *
circular pitch? ~. tin
40. Determine the polar moment of modulus of a hollow shaft with an
A. 18.95 mm C. pig iron
mertia, in mm 4 , of a shaft 32 mm internal diameter of 2 inches and
B. 25.95 mm D. scrap iron
diameter delivering 10, kw at 200 rpm. outside diameter of 2 % inches. The
A . 102,944 shaft is 60 inches long and shear stress C. 19.95 mm
D. Q9.95 mm 5. Cold working or similarly in peening
B 124, 944 of 4.5 ksi. · condition steel, material tend to be
c. 109,944 A. 1.79
(c=j ) A. harder*
D. 194,944 B. 3.81 B. more ductile
c. 2.51 DAY 41- ANSWER KEY C. in plasticity condition
41 A car racer Is accelerating at 3 D. 9.81 1. 8 16.C 31 . 846.C
2.C 17.A 32.047.0 D. more malleable
m/sec/sec on road on hill w ith a radius
of curvature of 180 m. Determine the 46. A baseball was thrown at 45-degree 3. A 18. C 33. A 48. 8
4. A 19. 8 34. 8 49. A 6. Measuring instrument to check ft
magnitude of the total acceleration angle. Determine the highest altitude the 5. A 20. C 35. 0 50. C hardness of rubber, Plastic and other
(m/se/sec) when the car's speed ball can attain in meters if the assumed 6. 8 21 A 36. 8 soft elastic materials.
reaches 24 m/s. velocity is 5000 cm/s with no drag. 7. A 22 0 37. 0
8. A 23. 0 38. C A. pyrometer
A. 3.2 A. 55
9. 0 24. 0 39. C B. duro meter*
B. 4.4 B. 75
10.0 25. 8 40. A C . plammeter
C. 318 C.64 11.0 26. C 41 . 8 D. tacho
D 5.9 D. 85 12.A 27. 0 42. A
13.A 28. 8 43. 0
14.0 29. A 44. 0 7. is an operation of
42. Determine the power transmitted, in 47. The maximum moment induced in a producing a circular hole by the removal
15.8 30. A 45. C
kilowatt, by a main power transmitting simply supported, beam of 20 foot span,
'L of a solid metal.
steel shaft with 2 3/8 inches diameter by a 2,000 pound load at midspan is: A. drilling *
A. 125,000 ft-lbs '
B. shaping

REFRESHER MANUAL 2rid Edition by JAS TORDILLO REFRESHER MANUAL 2nd Edition by..JAS TORDILLO
41 -61 Day 41 -Solution Day 41 -Solution 141 - 7
C. cutting C. 80.4 Machine Design
D. reaming D. 87.4
20. Internal and/or residual stress in a compute the center distance, in inches,
steal work like heavy welding and/or between the gear of 70 tooth and pinion.
8. Commonly used for tubes of water 14. Cutting lubricants used in drilling
casting can be remedied by _ __ . A. 6.18
tube boilers made of -large pipes which reaming and tapping for aluminum and
A. forging B. 7.05
a number of smaller ones lead to topper.
consuming points. This is called _ _. A. turpentine B. quenching c. 6.70.
B. dry C. cold working • D. 8.61
A. header • D. tempering
C. soda water
B. manifold
D. kerosene • Pc = nDI
C. crotch 21. SAE 1030 Carbon steel contains
D. line pipe %C. Tl
15. is a material ability to resist
deformation tinder stress. A-.o=-.~28::-:/o-=-.-=347 *
7
nD 1
9. This is a bearing, material with a IW A. steel plate B. 0.26/0.36 0.5 =-
coefficient of friction, resistance to 14
B. stiffness • C. 0.32/0.38
chemical, self-lubricated and can be C. magnesium D1 = 2.23 in
D. 0.22/0.28
utilized In a wide temperature, range. D. rigid frame Pc = nD2
A. stainless steel T2
B. Teflon 16. In the~l0n$hatt~esign the 22. AISI 8620 has an average carbon
allowable maximum twist should not content is in the range of %C. nD2
C. cast iron A. 0.21 0.5 = --
_exceed _ __ deg/foot length of the
D. chromium •
shaft. B. 0.19 70
A. 0.10 c. 0.16/0.25 D2=11 .14in
10. A simple term for is D. 0.18/0.23% *
B. 0.90
force: c. 0.08.
A. shear stress
C = Dl +D2 = 2.23+11.14 =6.68 in
D. 0.12 23. SAE 46xx is a steel designation for 2 2
B. linear momentum A. chromium
C. concurrent forces B. carbon
17. Dimensional variation. 27. Design a safe shaft diameter for a
D. push and pull • allowed/prescribed In order to secure C. molybderium-chromium power transmitting shaft to transmit 40
difference of fits between mating parts. D. molybdenum-nickel • "fip at a shaft speed of 205 rpm.
11 . A metal characteristic that absorbs
A. allowance* · A. 2 inches ·'
extreme deformation without rupture is 24. A file is a piece of high-carbon
called._ _ _ __ B. plus or minus B. 2.4 inches
C. fits . crucible steel having teeth cut upon its C. 1 Yz inch
A. ductility body by parallel rows. The parts of the
D. tolerance D. 2 'h inches •
B. toughness file are: tang, heel and .
C. malleability 18. High temperature surface h_e_ating of A. single cut --
D. plasticity • 3
irQn base-d material or alloyed steel B. length p = D N
Abgve the transforming temperature C. rough 80
12. The ideal limit of teeth that sprockets
range I then followed by quenching is D. face • 3
can have in operation is 40 = D (205)
I (minimum/maximum). called---=- -
A7ore beefing 25. This is a tool with hardened- steel 80
A. 17/127* points used for scribing circles or laying
B. cyaniding D = 2.5 in
B. 20/130 C. flame hardening • off distances. It is adjustable and is
C. 12/80 classified according to size by the
D. heat treatment 28. What would be the diameter in
D. 10/15 maximum opening between the points millimeters of a main, power transmitting
19. Very long center distances can and this must be slender and sharp. steel shaft SAE 1040 to transmit 150 kw
13. A chromium SAE 4340 steel 6
cause catenary tension in the _ _ A vernier caliper at 900 rpm?
Inches diameter x 24 inches long
operation. The ideal design center B. divider • A. 2.3
shafting weighing 87.3 kgs. How about
distance is about eighty pitches. C. file B. 66.4 *
the weight (in lbs) of an AISI 1030 steel
shaft of same dimension?
A. sprocket D. gage c. 2.6
B. chain* D. 76.5
A. 192.3 * 26. A pair of gear and pinion having a
C. fight
B. 240.4
D. heavy ratio of 5 with a circular pitch of 0.5 inch, P = 150 kw = 201.1 kw
REFRESHER MANUAL 2nd Edition by JAS TORDILLO REFRESHER MANUAL 2nd Edition by· JAS TORDILLO
41 -8 1 Day 41 -Solution Day 41 -Solution 141 - 9
No. of setscrews power per screw Machine Design
p =--
D3N
= total power transmitted 34. What is the developed acceleration 37. A motorcycle slides at a distance of
80
power per screw a roller coaster in meter per sec per seo 90 meters in a horizontal asphalt· road
201 .1 = 03(900) when the velocity is at 20 m/s at a (with rubber to asphalt coefficient of
10
80 = - - = 1.37 use 2 setscrews radius of curvature of 6500 em? friction of 0.6). Compute the motorcycle
D = 2.61 in= 66.4 mm
7.31 A. 4.61 speed in m/s.
B. 6.15 * A. 18.2
29. Compute for the height from which a 32. A cargo truck is traveling in the C. 5.8 B. 20.5
pile driver hammer must be drop to highway at a velocity of 30 m/s. If it is D. 8.15 C. 45.8
attain a velocity of 12.8 m/s. Assume no decelerating at a rate of 4 m per sec, 0. 32.5 *
air and guide resistance. how far, in meters, does it travel before Weight = Centrifugal force
stopping? Frictional force = REF
A. 8.4 m*
A. 90.5
mV 2
B. 4.8 m ma mgf ;:: ma
C. 9.5m B. 101.5 (9.81 )(0.6) ;:: a
D. 5.9m C. 12.5 ? 2 a ;:: 5.886 m/s2 (decelerating)
D. 112.5 * (20) - = 6. 15 m/s
a = 65
2 vl = V, 2
+ 2as
H = yl = (12.8) v, = 30 m/s o;:: v, 2 + 2(-5.886)(90)
2g 2(9. 8 l) =8.35 m v2 = o . 35. A 36-inch wide conveyor belt feeder v, = 32.5 m/s
having a speed of 800 ft/min moves a
30. A material weighing 800 lbs is being vl =Vl2 + 2as 260 kgs of magnetite concentrates. 38. Find the center distance, in inches,
0 = (30) + 2(-4)8 Determine the developed kinetic energy of a 11 0 tooth gear mesh with a pinion
moved on a conveyor belt feeder at a
S = 112.5m in ftllbs of the materials. having a circular pitch of 0.905.
speed of 15 fUsee. Find the kinetic A. 1957
energy in ft-lbs of the materials. Gear/pinion ratio is 5.
33. Compute for the circular pitch, in B. 1795 A . 15
A . 2795 *
inches, of a pair of gear and pinion with C. 1759 B. 30.
B. 9000
ratio of 5. The center distance indicated 0. 1579 * C.19 •
C. 4589
D. 7600 is at 18. The gear has 90 teeth and the Q, 37
pinion has 18 teeth. W = 260 kgs = 572 lbs
A.1.047" . V = 800 ftlmin = 13.33 fUsee Pc = n02
2 T2
wv 800(15? = 2795 ft-lbs B. 0.074
KE = 2g = 2(32.2) C. 1.740 2 2 1lD?
D. 1.470 wv 572(13.33) = 1579 ft-lbs 0. 905;:: - -
KE = 2g = 2(32.2) 110
31. Determine the number of Y2 inch D2 = 31.68 in
diameter set-screws needed to transmit D2 =5
10 hp at a shaft speed of 1200 rpm and Dl
36. Find the center distance, in inches, .!2. = 5
of a 90 teeth gear meshing with 18 teeth 01
a shaft diameter of 1 Y. inch. 02 =so, pinion having a circular pitch of 0.545.
A. 1.3
Gear/ Pinion ratio is 5. o, = 02 = 31.68=6.33
B. 2 * A. 8 5 5
C. 1 Y. B. 9.36 *
D. 4 C = 0 1 + 0 2 = 6.33 + 3 1.68 = 19 in
C. 5
18 = D1 +5D 1 2 2
D. 7.25
Power transmitted per screw, 2
39. Determine the polar -section
P= - -
DNd 2·3 o, =6 Pd X Pc ;:: 1t
modulus. Zp (in3 ) of a shaft delivering 5
50 1l hp at 150 rpm . The diameter of the shaft
Pc = - .-=5.764
23 nD1 _ 7t(6) = 1.047 in is 1 inch and allowable sum of 6000 psi.
= (LsXt2ooXo.s) · Pc = --=r;- - 18
0.545
A 0.175
50 C ;:: T1 +T2 = 18+90 = .
9 36 B. 0.489
=7.31 2Pd 2(.764) c. 0.196 *
D. 0.895
REFRESHER MANUAL 2nd Edition by JAS TORDILLO REFRESHER MANUAL 2nd Edition by JAS TORDILLO
41 - 10 I Day 41 -Solution Day41-Solution 141-11
Machine Design
3 3 43. A pair of gear/pinion of 45 teeth and
nD _ 7t(l) =0.196 in 3
Zp = J6- 16 15 teeth, respectively, with center
distance of 40 em, addendum at 0.725/p
46. A baseball was thrown at 45-degree 49. Determine the length of belt needed
angle. Determine the highest altitude the for a two flat belt pulleys having a
and dedendum at 1/p. The gear ball can attain in meters if the assumed diameters of 70 em and 30 M. The
40. Determine the polar moment of pressure angle is 20 degree. Compute velocity is 5000 cm/s with no drag. center distance is 400 em and pulleys
inertia, in mm\ of a shaft 32 mm for the diametral pitch in teeth per em. A.55 ' rotate in same direction. Assume power
diameter delivering 10, kw at 200 rpm. A. 0.60 B. 75 transmitted is 10 hp at 200 rpm.
A. 102,944 • B. 0.90 c. 64 * A. 958 em"
B. 124, 944 C. 0.70 D. 85 B. 589 em
c. 109,944 D. 0.75 * C. 859 em
D. 194,944 Yo = 5000 cm/s = 50 m/s D. 895 em
Tt +T2
4 4 C= - - L = 2C+2:(D?+D)+(D2-DI)2
2
1tD _ n(32) = 102,943.7 mm 4 2Pd v o2-·sin 8 2 -
1
4C

r
j = 32 - 32 H = -!:.. _..:..
40 = 15+45 . 2g

41. A car racer is accelerating at 3


2Pd
_ (so 2 sin 2 45 °
L = 2(400) + ~2 (10 + 30)+ (704(-40030
Pd = 0.75 teeth per em
m/sec/sec on road on hill with a radius - 29.81) L = 958 em
of curvature of 180 m. Determine the ' H = 63.71 m
44. A punching machine flywheel
magnitude of the total acceleration accelerates from rest with7 radians/sec2. 50. A gear set having a gear ratio of 3 is
(m/se/sec) when the car's speed 47. The maximum moment induced in a to be used at a center-distance of mm. If
ln. 10 seconds, how. many radians are
reaches 24 m/s. simply supported, beam of 20 foot span, the gear has 60 teeth, what must be the
achieved?
A. 3.2 by a 2,000 pound load at midspan is: circular pitch?
A. 45
B. 4.4 * A. 125,000 ft-lbs A. 18.95 mm
B.250
C. 3 18 B. 25.95 mm
C. 35 B. 30,.000 ft-lbs
D. 5.9 C.19.95 mm *
D. 350 * C. 15,000 ft-lbs
D. 10,000 ft-lbs * 0'. 99.95 mm
an =
y2 - (24)2
-r-- 180
=3.2 m/s2
w1 =0 llo

9. = W1t +% af From the shear diagram: T2 =3


ar=~at+an = 0 +% (7)(10)2 Tl
= 350 radians Maximum moment = 1000(10)
= J(3)2 + {3.2)2 = 4.38 m/s2 = 10,000 ft-lbs 60 = 3
45. Compute for the polar section Tl
42. Determine the power transmitted, in modulus of a hollow shaft with an 48. A 60-inch diameter steel Pipe, 3/8. T1 = 20
kilowatt, by a main power transmitting internal diameter of 2 inches and inch thick, ca rries Water under a
steel s haft with 2 3/8 inches diameter outside diameter of 2 % inches. The pressure head of 550 ft. Determine the .!21.. = 3
shaft is 60 inches long and shear stress hoop stress in the steel. Dl
using SAE 1060 material mounting a 12
inches pulley at a speed of 170 rpm. of 4.5 ksi. A. 34,320 psi D2 = 3D1
, A. 21 .3"' A. 1.79 B. 19,066 psi • D l +D2
C. 25,780 psi C= - -
B. 28.5 B. 3.81 2
c. 25.8 c. 2.51 * D. 15,920 psi ·
Dl +3D!
.4 (550) = ~38.33 psi
D. 34.5 D. 9.81 62 254 = 2
P= yH=
144
1t [ 3
Po - Di3]
D = 2 318 in= 2.375 in D = 127 mm
Zp = 16 PD
S,=- nD 1 _ n(l27) = 19.95 mm
3 3 21 Pc = - -
Tl 2
o
= (2.375)
Power = D N (170)
= ~[(2.75) 3
-(2)3] - 238.33(60)
80
= 28.46 hp = 21.23 kw
80
Zp
16
= 2.51 ----;m
S, = 19,066psi

REFRESHER MANUAL 2nd Edition by JAS TORDILLO REFRESHER MANUAL 2nd Edition by JAS TORDILLO
Day 42 - EXAM I 42 - 1
Machine Design
· 6. Device that prevents entrance of
DAY 42- EXAM moisture and other fluids and permits
pressure differential between the
1. In the SAE identification code of ste~ exterior and the interior:
shafting the 3rd and 4th digits A. gasket
represents the content of: B. seal
A . % manganese on the steel C. washer
B. % carbon contents D. spacer
C. % alloy elements
D. % chromium in the steel 7. Device used to guard surfaces
against marring:
2. A moving truck skids to a stop 20 A. loGker
meters .after the brakes are applied B. washer
while traveling 9£J kph. What is the C. bearing
acceleration in m/s2 ? D. oi~seal
A. 15.6
B. -15.6 8. When the hole is smaller than the
c. 16.5 shaft, it will take force to put the parts
D. -16.5 together, the allowance is said to be
negative and is called:
3. The process of increasing the carbon ·A. interference
c_9ntent to the surface of steel by B. allowance
exposing it to hot carbonaceous material C. clearance
above the transformation temperature of D. toughness
1650 to 1750 deg F.
A. carbonitriding 9. The stress in an elastic material is:
B. carburizing A. inversely proportional to the yield
C. case hardening strength of the material
D. hardening B. inversely proportional to the force
acting '
4. It is ~e_cal practice to use the C. proportional to the displacement of
following allowable stresses, 49_00_ psi the material acted upon by the force
for main-power transmitting shaft and D. proportional to the length of the
8500 psi for small, short, countershafts. material subject to the force
For lineshaft carrying pulley it is,
_ __ _ psi. 10. A micrometer measured by:
A. 8500 A. 108
B. 6000 B. millions of an inch
C. 7000 C. root means square
D. 6500 D. Thousands of an inch

5. Steel that has been_Qe.oxid.iz.ed with a 11 . An impact test is used to test a


str.ong deoxidizing ~nt such as silicon metal for:
or aLuminum in order to eliminate A. toughness
oxygen and carbon re3ction during B. ductility
solidification. C. strength
A. rimmed steel D. hardness
B. killed steel
C. stainless steel 12. A partial bearing is one in which:
D. monel

REFRESHER MANUAL 2nd Edition by JAS TORDILLO


42 - 2 I Day 42 - EXAM Day 42 - EXAM I 42 - 3
Machine Design
A . the bearing is supplied with less oil C. annealing
than full bearing D. tempering C. arc of action 31 . indicates how many
B. the bearing encloses less than 36,0 D. arc of approach times a volume of material is heavier
dog of the journal 19. The term used in referring to the than an equal volume ofwater.
C. the bearing is slightly loaded cogtj~ous increase in the strain, or 25. Which of the following steel contains· A. specific gravity
D. the bearing is supplied with oil deformation, of-any material ~ubjected chromium? B. specific weight
intermittently to stress. A. SAE 2340 C. specific volume
A. elasticity B. SAE 1230 D. spec. density
13. For a .completely~co((osiQDJe~ant B. creep C. SAE 4230
stainless - steel, what ...mmlffiJ!m C. tolerance D. SAE 5240 32 . The flux that should be provided in
gercentage of chromium in the alloy is D. all of the above ~ldering ele.c.llli:.al -.eonnection or
required? 26. The QQnical surface at the startin9. commutator wires as it tends to corrode
A. 3% 20. Poison's ratio is the ratio of the: .§lQ of the thread is called the_connections .
B. 11% A. unit lateral deformation to the unit A. pitch cone , A. sal ammoniac
C. 7% longitudinal deformatien B. crest B. stearin
D. 15% B. unit stress to unit strain C. chamfer C. zinc chloride
C. elastic limit to proportional limit D. flute D. acid fl uxes
14. The a.rno unt by wh icb th_e~idth of a D. shear strain to compressive strain
tooth space exceeds the thiclsness Di.an 27 . It is a ~des.ign practice for .@I 33. Bevel gears subjected to corrosion
~gag ing_ tooth on the pitch circles of a 21. Jlt.Lelatively high temperatures and linespaftin to consider a limit to the and lightly loaded are usually made of
gear: low rates of straifls, structures will linear eflection of inch/foot
A. clearance perform better if the material is: length of maximum. A bronze
B. backlash A. fine-grained A. 0.050 B. all of these
C. tip relief B. .their behaviour is independent of B. 0.020 C. brass
D. chordal thickness grain c. 0.010 D. duralumin
C. course grained D. 0.100
15. Steels containing large amounts of D. all of the above 34. A major-<X>~UP~t of cast steel.
mild nickeland chromium: 28 Determine the number of 1/2 in A. silicon -
A. carbon steel - ·22. The most important factor in diameter setscrews needed to transmit B. manganese
B. alloy steel determining ·high-temperature behavior 8 hp at a shaft speed of 1000 rpm . The C. Iron
C. stainless steel . of an alloy is: shaft diameter is 3.81 em of SAE 1040 D. chromium
D. cast steel A. dispersion material.
B. crystallization A. 1 35. The factor of safety generally
16.' The product of the mass and the C. ionization B. 2 applied in w ire rope design starts-at..3=4
Jlnear velocity of a body: D. composition C. 4 for standing rope application, 5-12 for
A . ang ular impulse D.3 operati ng rope and for hgar_ct.__to .life
B. linear impulse 23. Modulus of · rigidity of _a steel and property application like foundry
C. angular momentum member is: ---- 29. The soldering material _ccmmo.nly operation.
D. linear momentum A. a function of the length and width appl ied to automobile radi.g1.or cores and A. higher values
B. equal to the modulus of elasticity roofing seatm> B. 8-10
17. When a steel is described as SAE divided by one plus Poisson's ratio A. 15/85% tin and lead c. 7-10
1820: C. defined as the unit show stress divided B. 45/55% tin and lead D. 10-18
A. itTs plain carbon steel by the unit shear deformation C. 50150% tin and lead
B. it is nickel-chromium steel
C. it contains 18 to 20% carbon
D. it contains 0.18 to 0.23% carbon

18. Which is not a heat treatment


process?
D. defined as the length divided by the
moment of inertia

24. AJC of the pitch circle through which


a tQ.91b travels from the point of contact
with the mating tooth, to the pitch point of
D. 20/80% tin and lead

30. Axially loaded rectangular groove in


a hub and shaft:
A. keyseat
B. flute
36.~11eys have their maximum
sc.e
~s:
limited

A . 65 to 75%
B. 55 to 60%
C. 45 to 50%
to OT""-solid
--
A. sintering a gear: C cotter pin set D. 80 to 90%
B. hardening A. arc of recess D. setscrew point
B. arc of relief
REFRESHER MANUAL 2nd Edition by·JAS TORDILLO REFRESHER MANUAL 2nd Edition by JAS TORDILLO
42 - 4 I Day 42 - EXAM
Day 42 - Solution 142 - 5
37. When an external gear is meshed C.6to10 Machine Design
with an internal gear, the gears will D. 3 to 5
rotate in: of safety of at least 10. Assume the
A. same direction ultimate strength of the material -to be DAY 42 - SOLUTION
44. Hardened steel parts have:
B. will not rotate 110,000 psi and modulus of elasticity to -
A. fine grains
C. opposite direction be, 15,500,000 psi. Determine the" 1. In the SAE identification code of steel
B. medium grains
D. none of these
diameter of the rod. shafting the ~d and _4th digits
C. coarse grains
A. 2.14 in represents the content of:
D. none of these
B. 4.14 in A. % manganese on the steel
38. Taps are resharpened by grinding:
A. flutes - C.3.14in B. % carbon contents *
45. A 90 inches long, shaft rotates at
D. 5.14 in C. % alloy elements
B. diameter about one end at 12 rad/sec. It starts to D. % chromium in the steel
C. threads accelerates at 12 rad/sec2. How long
D. relief 50. Compute the lineshaft diameter that
in-sec will the velocity of free end
transmit 40 kw at 1200 rpm. The 2. A [Tloving truck skids to a stop 20
reach 150 m/sec.
ultimate shear stress is 81.95 MPa and meters after the brakes are applied
39. Steel that has been ,.Qe oxjdiz~d with A. 2.47
safety factor is 10. while traveling 90 kph . What is the
manganese: B. 4.47
A. 5.3 in accel~ration in m/s2 ?
A. capped steel C. 3.77
B. 3.3 in A. 15.6
B. killed steel D. 5.67 B. -15.6 *
C. 4.3 in
C. rimmed steel C. 16.5
D. chrome steel
D. 2.3 in
46. If the ultimate shear strength of a D. -16.5
steel is 42,000 lblin2 • The force to punch
40. If the weight of 6 inches diameter two 3-inches diameter hole in a 5/8 inch
by 48 inches long SAE 1030 shafting is steel plate is:
v1 = 90 kph = 25 m/s
195 kg, then what will be the weight of A. 494,800 kg . DAY 42- ANSWER KEY v2 = o 2
1. 8 16.0 31. A 46.C Vl = V1 + 2aS
chromium SAE 1416 of, same size? B. 210,450 kg 2. 8 17. [} 32. 0 47. 8 0 = (25) 2 + 2a(20)
A. 3051bs C. 224,91 0 kg 3. 8 18. A 33. 8 48. A a=- 15.625 m/s2
B. 430 lbs D. 490,900 kg 4. 8 19, 8 34. c 49. c
C. 3261bs 5. B 20. A 35. A 50. 0
6. 8 21' c 36. 8 3. The process of increasing the carbon
D. 4651bs 47. An oil cylinder, closed at both ends, 7. 8 22 D 37. A co'ntent to the surface of steel b_y
contains oil ·at a pressure of 200 psi. 8. A 23. C 38. A ei<"posing it to nof carbonaceous material
41 . Main alloying element in ~eed The inside diameter of the cylinder is 30 9. C 24. D 39. C
above the transformation temperature of
·Steelis: inches. If the yield ~:Joint of the material 10.0 25. 0 .40, 8
11.A 26. C 41 . 8 1Ss0 to 1750 deg F.
A. chromium is 38,000 .psi and a safety factor of 4 is 12.8 27. c 42. c A. carbonitriding
B. 'tungsten used, the wall thickness should be: 13.8 28. 8 43. 8 B. carburizing *
·C. vanadium A.6mm 14.8 29. B 44. A C. case hardening
D. nickel B. 8mm 15.C 30. A 45 . 8
D. hardening
C. 7mm
42. A type of welding_proce ~s.-Wlecei n D. 10 mm 4. It is a general practice to use the
a_...rnixtum- of atumin urn_ powder-and following allowable stresses, 4000 psi
[@tal oxide powder is ~ by 48. Compute the circular pitch of a gear for main-power transmitting shaft and
a-special powder i!lli crucible. having a 5 if)ches pitch diameter and 40 8500 psi for small, short, countershafts.
A. resistance welding teeth. For lineshaft carrying pulley it is,
B. stud welding A. 10 mm _ _ __ psi.
C. !hermit welding B. 12 mm A.8500
D. plasma arc welding C. 11 mm B. 6000 *
D. 15mm C. 7000
43. In flywheel design, the normal factor D.6500
of safety is: 49. A round steel bar 12 inches long is
A. 5 to 8 to withstand a dead load of 40,000 lb in 5. Steel that has been deoxidized with a
B. 10 to 13 compression. The rod must not shorten strong deoxidizing agent such as silicon
more than 0.004 inch and have a factor

REFRESHER MANUAL 2nd Edition by.JAS TORDILLO


\"'-- or aluminum in order to el)rli'inate ·

REFRESHER MANUAL 2nd Edition by JAS TORDILLO


~

42 - 6 I Day 42 - Solution Day 42 - Solution 1 42 - 7


oxygen and carbon reaction during A. toughness * Machine Design
solidification. B. ductility " 18 to 20% carbon
C. it contains C. defined as the unit show stress
A. rimmed steel C. strength
D. it contains 0.18 to 0.23% carbQn * divided by the unit shear deformation •
B. killed steel • D. hardn~ss
C. stainless steel
D. defined as the length divided by the
18. Which is not a heat treatment moment of inertia
D. monel 12. A partial bearing is one in which:
process?
A. the bearing is supplied with less oil 24. Arc of the pitch circle through which
A. sintering *
6. Device that prevents entrance of than full bearing
B. hardening a tooth travels from the point of contact
moisture and other fluids and permits B. the bearing encloses less than 360
C. annealing with the mating tooth t~_129int of
pressure differential between the dfig of the journal *
D. tempering a gear:
exterior and the interior: C. the bearing is slightly loaded A. arc of recess
A. gasket D. the bearing is supplied with oil
B. seal* 19. The term used in referring to the B~ arc of relief
intermittently C. arc. of action
C. washer continuous incrsase in the strain, or
deformation, of any material subjected D. arc of approach *
D. spacer 13. For a c.Q.!DPI.e.~ corrosion resistant
to stress.
stainless steel, what - -mtn-imum 25. Which of the following steel contains
7. Qevice used to guard surfaces A. elasticity
percentage of chromium in the alloy is chromium?
against marring: B. creep •
required? A. SAE 2340
A. locker C. tolerance
A. 3% B. SAE 1230
B. washer* B. 11% * D. all of the above
C. SAE 4230
C. bearing C. 7%
20. Poison's ratio is the ratio of the: D. SAE 5240 *
D. oil seal D. 15%
A. unit lateral deformation to the unit
longitudinal deformation * 26. The conical surface at the starting
8. When the hole is smaller than the 14. The amount by which the widtb.of a
B. unit stress to unit strain end of the thread is called
shaft, it will take force to put the parts iQQtb_space exceeds the thickness of an
C. elastic limit to proportional limit A pitch cone
together, the allowance is said to be engaging tooth on the pitch circles of a
D. shear strain to compressive strain B. crest
negative and is called: gear: C. chamfer*
A. interference * A. clearartce
B. allowance 8. backlash • 21. N relatively high temperatures and D. flute
C. clearance loY'{ rztes oT s1ralns, structures will
C. tip relief 27. It is a good design practice for steel
D. toughness D. chordal thickness per!flrm better if the material is:
A. fine-grained lineshafting to consider a limit to the
B. their behaviour is independent of linear deflection of inch/foot
9. The stress in an elastic material is: 15. Steels containing large amounts of
grain 1 length of maximum.
A. ir;wersely proportional to the yield mild nickel and chro.mium:
C. course grained • A 0.050
strength of the material A. carbon steel
D. all of the above B. 0.020
B. inversely proportional to the force B. alloy steel c. 0.010 *
acting C. stainless steel •
22. The · most important factor in D. 0.100
C. proportional to the displacement D. cast steel
of the material acted upon by the determining high-temperature behavior
of an alloy is: 28. Determine the number of 1/2 in
force • 16. The product of the mass and the
A. dispersion diameter setscrews needed to transmit
D. proportional to the length of the · linear velocity of a body: 8 hp at a shaft speed of 1000 rpm. The
material subject to the force A. angular impulse B. crystallization
C. ionization shaft diameter is 3.81 em of SAE 1040
B. linear impulse material.
10. A micrometer measured by: C. angular momentum D. composition •
8 A1
A. 10 D. linear momentum •
23. Modulus of rigidity of a steel B. 2 *
B. millions of an inch
C. root means square member is: C.4
17. When a steel is described as SAE D. 3
D. Thousands of an inch * A. a function of the length and width
1820:
A. it is plain carbon steel
B. equal . to the modulus of elasticity
divided by one plus Poisson's ratio D = 3.81 em= 1.5 in
11. An impact test is used to test a B. it is nickel-chromium steel
metal for:

REFRESHER MANUAL 2nd Edition by JAS TORDILLO REFRESHER MANUAL 2nd Ed!tion by JAS TORDILLO
L,._
~

-·· ·
42 - 8 1Day 42 - Solution Day 42 - Solution 1 42 - 9
Machine Design
Power transmitted per screw C. brass
23 D. duralumin 195 kg, then what wil l be the weiglit of 150 = 2.286 (w2)
= 0Nd · = 1.5(IOooXo.5)2-3 chromium SAE 1416 of, same size?: W2 = 65-61 rad/sec
50 50 34. A major component of cast steel. A. 305 1bs a= w2 -wl
= 6.09 HP per screw A. silicon B. 430 lbs * t
B. manganese C. 326 1bs
No. of setscrews C. Iron* D. 465lbs 12 = 65.61-12
= total power transmitted D. chromium t
power per screw 41 . Main alloying element in High Speed t = 4.47 sec
35. The factor of safety generally Steel is:
8 applied in wire rope design starts-at 3-4 A. chromium 46. If the ultimate shear strength of a
= - - = 1.31 say 2 setscrews
6.09 for standing rope application, 5-12 for B. tungsten* steel is 42,000 lb/in2. The force to punch
operating rope and for hazard to life C. vanadium two 3-mches diameter hole in a 5/8 inch
29. The soldering material commonly and property application like foundry D. nickel steel plate is:
applied to automobile radiator cores and operation. A. 494,800 kg
roofing seams. A. higher values * 42. A type of welding process wherein B. 210~450 kg
A. 15/85% tin and lead B. 8-10 a mixture of aluminum powder and c. 224,910 kg*
B. 45/55% tin and lead • c. 7-10 metal oxide powder is ignited by D. 490,900 kg
C.· 50150% tin and lead D. 10-18 a-special powder in a crucible .
D. 20/80% tin and lead A. resistance welding F = Su (TTdt) X 2
36. Split pulleys ha~re their maximum B. stud welding ~ = 42,000(TT)(3)(0.625) X 2
30. Axially loaded rectangular groove in speed limited to of solid C. thermit welding * F =494,800.84 lbs =224,909.5 kg
a hub and shaft: pulleys: D. plasma arc welding
A. keys eat * A. 65 to 75% 47. An oil cylinder, closed at both ends,
B. flute B. 55 to 60% * 43. In flywheel design, the normal factor contains oil at a pressure of 200 psi.
C cotter pin set C. 45 to 50% of safety is: The inside diameter of the cylinder is 30
D. setscrew point D. 80 to 90% A. 5 to 8 inches. If the yield point of the material
B. 10 to 13 * is 38,000 psi and a safety factor of 4 is
31 . indicates how many 37. When an external gear is meshed C. 6to10 used, the wall ~hickness should be:
times a volume of material is heavier with an internal gear, the gears will D. 3 to 5 A .. 6 miT\
than an equal volume of water. rotate in: B. 8 mm *
A . specific gravity* A. same direction * 44. Hardened steel parts have: c. 7 Jllm
B. specific weight B. will not rotate A. fine grains * D. 10 mm
C. s~ecific volume C. opposite direction . B. medium grains
D. spec. density D. none of these C. coarse grains PO
D. none of these St = -
38. Taps are resharpened by grinding: 2t
32. The flux that shou ld be provided in
soldering electrical connection or A. flutes* 45. A 90 inches long, shaft rotates at 3&,ooo = 20o(3o)
commutator wires as it tends to corrode B. diameter abuut one end at 12 rad/sec. It starts to 4 2t
the connections. C. threads
D. relief
accelerates at 12 rad/sec2. How long
in-sec will the velocity of free end
t = 0.3157in = 8mm
A. sal ammoniac
B. stearin reach 150 m/sec. 48. Compute the circular pitch of a gear
C. zinc chloride 39. Steel that has been deoxidized with A. 2.47 having a 5 inches pitch diameter and 40
D. acid fluxes * manganese: B. 4.47 * teeth.
A. capped steel c. 3.77 A.10 mm *
33. Bevel gears subjected to corrosion B. killed steel D. 5.67 B. 12 mm
and lightly loaded are usually made of C. rimmed steel * C. 11 mm
D. chrome steel r = 90 in = 2.286 m D. 15 mm
A. bronze w1 = 12 rad/sec
B . all of these * 40. If the weight of 6 inches diameter
by 48 inches long SAE 1030 shafting is v = rW
REFRESHER MANUAL 2nd Edition by .J.AS TORDILLO REFRESHER MANUAL 2nd Eclifion by JAS TORDILLO
~
42 -10 I Day 42- Solution Day 43 - EXAM I 43 - 1
Machine Design
rrD n:(5) .
Pc = - = - = 0.3927 rn = 9.97 mm
T 40 DAY 43- EXAM c. 58.2
D . 38.2
49. A round steel bar 12 inches long is
1. The center-to-center distance· 7. With an electric arc welding rate of 12
to w ithstand a dead load of 40,000 lb in
between sprockets should not- be less inches per minute, how long will it take
compression . The rod must not shorten
than_ _ times the pitch. to weld a 1/4 in thick plate by 4 ft long
more than 0.004 inch and have a factor
A. 10 to 30 seam?
of safety of at least 10. Assume the
B. 30 to 50 A2 min
ultimate strength of the material -to be
C. 20 to 40 B. 4 mi n
110,000 psi and modulus of elasticity to
D. 50 to 80 C. 3min
be. 15,500,000 psi . Determine the
diameter of the rod. D . 5 min
2 A center distance between sprockets
equivalent to ti.mes pitch is 8. How long will it take to saw a
A. 2.14 in
considered maximum. Very long center rectangular piece of steel plate size 1/2
B.4.14in
distances result in catenary tension in x 4 x 8, if the length of cut is 4 ft? The
C. 3.14 in*
the chain . · power h acksaw makes 120 •pm and the
D. 5.14 in
A. 30 feed per stroke is 0.008 in?
B.80 A. 48 min
FL C. 50
y = - B. 50 hr
AE D. 120 C. 0.83 hr \
40.000(12) D. 52 min
0.004 = 3. The smaller sprockets of a drive are
2 usually made of._ _ _ _ _.
1t D (15.500.000) 9. A machinery operation whereby done
4 A . steel · with the work accurately fastened has a
D = 3.14 in B. cast iron reciprocating (forward and backward)
C . stainless steel motion and the tool head is stationary.
50. Compute the lineshaft diameter· that D . bronze A. shaping
transmit 40 kw at 1200 rpm. The
B. turning
ultimate shear stress is 81.95 MPa and 4. Commonly used material for large C , planning
safety factor is 10. .sp.rook-ejs. especially in drives with large D. reaming
A. 5.3 in speed ratios.
B. 3.3 in A. cast iron 10. A mechanism which usually do the
C. 4.3 in B. steel indexing ln a machine tool:
D. 2.3 in* C. carbon A . universal chuck
D. pig iron B. dividing head
P = 2rrTN C . slotter
1200) 5. Preferred large sprocket material for D. indexing
40,000 = 21TT 6Q severe service is:
(
A. cast steel 11. A machine shop- equipment that can
T = 3 18.3 N-m B. boron flatten horizontally, vertically or angular
C . chromium A . shaper machine
S, = 16T D. brass B. drill mad
n:DJ C . welding machine
6 . Calculate the rpm for machining a D. planer
81.95xl0 6 _ 16(3 18.3) cast iron work piece 10 inches in
10 - 1t03 diameter and the cutting .speed is 100 12. What is the required base area of
fpm. the foundation to support an engine with
D = 0.058266 m = 2.3 in A. 68.2 speed of 1800 rpm and weight of 5000
B. 48.2 kg . Assume bearing capacity of soil as
45.5 kPa. Usee = 0.11.

REFRESHER MANUAL 2nd Edition by JAS TORDILLO .


REFRESHER MANUAL 2nd Edition by JAS
. TORDILLO
43 - 2 I Day 43 - EXAM Day 43 - EXAM I 43 - 3
Machine Design
A. 6.1 m ·C. 10 to 20
B. 8.1 m 2 D. 20 to 30 C. 12 A. 3.2 to 4.2
2
C. 7.1 m D. 20 B. 5.2 to 7.2
D. 6.9 m2 18. For .stability the total combined C. 4.2 to 5.2
engine driven equipment, and 24. Foundation bolts of specified size D. 1.2 to, 2.2
13.Foundation should be isolated from foundation center of gravity must be should be used and surrounded by a
floor slabs or building footings by at kept_ the foundation's top. pipe sleeve with an inside diameter of at 30. For steam turbines, ft should be
least - - - mm around its perimeter to A. Below least 3 times the diameter of the anchor designed to support the machine load
eliminate transmission of vibration. B. bottom bolt and a length of at least plus for impact, condenser load floor
A. 15 C. top times the diameter of the bolt. - - - - slabs and dead loads.
B. 25 D. beside A. 10 A. 10%
c. 20 B. 1!;1 B.40%
D., 35 19. The weight of the machine plus the C. 15 C, 25%
weight of the foundation should be D. 20 D. 50%
14. Foundations are preferably built of distributed over a sufficient soil area
concrete in the proportion of~ The which is large .enough to cause a 25.To prevent pulling out of the bolts 31 . The act of cuffing out a piece of
machine should ri6t be placed on the bearing stress within the safe bearing when the nuts are tightened, the length metal ( at a desired shape and size.
foundation until I_days have elapsed or capacity of the soil with a factor of safety embedded in concrete should be equal A. broaching
operated until another days of: · to at least_ times the bolt diameter. B. slitting
have passed. A.2 A. 10 C. blanking
A. 14 B.5 B. 20 D. dinking
B. 5 C. 3 C. 15
c. 9 D. 7 D. 30 32. A foot rule has how many 118 in?
D. 7 A. 96
20. Foundations should be isolated from 26. For die·sel engine, the engine B. 120
15. AlL heavy machinery shalL be floor slabs or building footings by at should not be placed on the foundation C.48
supported on solid foundations of least__ around its perimeter to until days have elapsed, nor operated D. 108
sufficient mass and base area to eliminate t ransmission of vibration. until another days.
prevent or minimize the transmission of: A. 10 mm A. 7, 7 33. Calculate the rpm for machining a
A. objectionable vibration to the building B. 25 mm B. 7, 10 cast iron work piece 6 inches in
B. objectionable vibration to the C. 20 mm C. 10,10 diameter. The lowest cutting speed for
occupied space D. 35mm D. 10,7 cast iron is 50 fpm.
C. to maintain the supported machines A. 35.3
elevation and alignment 21 . Foundations are preferably built of 27. For good firm soil, reinforced . B. 43.3
D. all of these concrete in the proportion of: concrete foundations for large engines c. 31.8
A. 1:3:5 may use a leaner mixture down to D. 53.3
16. Foundation mass should be B. 1:2:4 A. 1:2:3
from times the. weight of the C. 2:3:4 B. 1:3:6 34. To facilitate the milling (roughing)
macbinery it is supposed to support. D. 1:2:5 C. 1:2:4 work of a cast steel material using a 1 %
A. 1 to 3 D. 1:3:5 in diameter cutter choose between the
B. 5 to 8 22. Weight of reinforcing steel· should be two available stock. Find the speed of
C. 3 to 5 from ___ of the weight of foundation. 28. The weight of concrete is: the cutter in rpm.
D. 7to10 A. 1/2 to 1 % A. 2604 kg/m 3 High speed steel cutter with a cutting
B. 1/2 to 3% B. 2204 kg/m3 speed of 50 fpm
17. If the unbalanced ineRial forces C. 1 to 2% C. 2806 kg/m3 Carbide -tipped cutter with a cutting
produced by the machine can be D. 1 to 5% D. 2406 kg/m 3 speed of 200 fpm
calculated, a mass of weight ~qual A. 389
to times the forces should be 23. No foundation bolts shall be less 29. For diesel engines, the foundation B. 509
used to dampen vibration. than mm in diameter. depth may be taken as a good practical
A. 5 to 10 A. 10 rule, to be times the engine c.572
B. 15 to 20 B. 14 stroke. D.412

REFRESHER MANUAL 2nd Edition by JAS TORDILLO REFRESHER MANUAL 2nd Edition by JAS TORDILLO
43 - 4 I Day 43 - EXAM Day 43 - EXAM I 43 - 5.
Machine Design
40. With an electric are welding rate of
35. A concrete foundation should have 18 in/min, how long will it take to weld a 45. Standard railings shall be at least the use of runways not less than
steel bar reinforcements placed both 1/2 in thick plate by 3ft long seam? -:-.,..-.,..---,. mm from the upper surface ...,--.,...,..,..--in width.
vertically and horizontally to avoid A. 3 min of the top rail to floor leveL A. 340 mm
thermal cracking. Weight of reinforcing B. 1.5 min A. 1000 B. 670mm
steel bars should be at least: C. 2 min B. 2000 C. 560mm
A. 4% to 4% of the weight of the D. 4min C. 1500 D. 750mm
foundation D. 2400
B. 6% of the weight of the foundation 41 . As a general requirement, work
C. 3% to 3% of the weight of the
foundation
rooms shall be at least_'_
is height from floor to ceiling.
mm 46. Standard railings shall ,have posts
not more than apart and an rc? DAY 43 -ANSWER KEY
I)
D. 1/2% to 1% of the weight of the A. 1000 intennediate rail halfway between top
1. 8 16.C 31 . C46.B
foundation B. 3000 rail and the floor. 2.8 17. C 32.A47.8
c. 1500 A. 1000 mm 3. A 18. A 33. C 48. 8
36. Avoid using in a lathe D. 3500 B. 2000 mm 4. A 19. 8 34. 8 49. C
machine operation. C. 1500 mm 5. A 20. 8 35. 050. C
42. As a requirement, the maximum 6. 0 21. 8 36. A
A. wooden shoes D. 2400 mm 7. 8 22. A 37. 8
B. measure ruler number of persons working or will be 8. c 23. c 38. c
C. outside caliper working shall not exceed ___ person 47. All railings shall be of sound material 9. C 24. B 39. 8
D. safety gloves per 12 cu meter. free from defects and all sharp corners 10.8 25. 0 40. c
A. one shall be rounded and smoothed. 11.A 26. C 41 . B
12.A 27. 8 42. A
37. Recommended best cutting angle of B. three Toeboards shall be at least ___ mm 13.8 28. 0 43. 8
drill for work on steel or cast iron is C. two in height. 14.0 29. A 44. A
____degrees. D. four A. 100 15.0 30. C 45. A
A 48 B. 150
B. 59 43. Floor openings into which person C. 120 CJ:
c. 63 cannot accidentally walk on account of D. 180
D. 50 fixed machinery, equipment or walls
shall be guarded by covers. securely 48. All stairs, platforms, and landings
38. How long will it take to saw a held in place and leaving no openings shall be of sufficient strength to sustain
rectangular piece of aluminum plate 8 in more than in width or by safely a live load of not less than 500 kg
wide and 1 1/2 in thick if the length of toeboards on all exposed sides. with a factor of safety of _ _ __
the cut is 8 in, the power hacksaw A. 15 mm A. two
makes 120 rev/min and the average B. 25 mm B. four
feed per stroke is 0.0060 in ? C. 20 mm C. three
A. 13.11 min D. 50 mm D. eight
B. 14.01 min
C. 11.11 min 44. All wall openings shall be solidly 49. Fixed ladders shall be so installed
D. 12.03 min enclosed or guarded by fixed or rolling that the distance from the front of the
barrier rails, picket fences, half doors, or rungs to the nearest fixed object on the
39. How long will it take to mill a 3/4" by · equivalent barriers, capable of climbing side of the ladder is at least
2" long keyway in a 3" diameter shafting withstanding a load or at least _ _ kg
with a 24 tooth cutter tu rning at 100 rpm applied in any direction except vertically A. 550 mm
and 0.005 feed per tooth ? upward at any point on the top rail. B. 890 mm
A. 0.136 min A. 100 C. 760mm
B 0.166 min B. 150 D. 920 mm
C. 0.196 min c. 130
D. 0.106 min D.210 50. Catwalks used for fllling of tanks,
cars or for oiling may have the railing on
one side omitted, if necessary subject to
the hazard of failing being reduced by

REFRESHER MANUAL 2nd Edition by J.AS TORDILLO REFRESHER MANUAL 2nd Edition by JAS TORDILLO
43 - 6 I Day 43 - Solution Day 43- Solution 143-7
V = rrDN Machine Design
DAY 43 - SOLUTION
1. The center-to-center distance
100 = lT C~)N C. slotter
D. indexing
C. 9
D. 7 *
between sprockets should not- be less N = 38.19 rpm
than_ _ times the pitch. 11. A machine shop- equipment that can 15. All heavy machinery shall be
A. 10 to 30 flatten horizontally, vertically or angular supported on solid foundations of
7. With an electric arc welding rate of 1
.2
B. 30 to 50* A. shaper machine * sufficient mass and base area to
inches per minute, how long will it take
C. 20 to 40 B. drill mad preve nt or minimize the transmission of:
to weld a 1/4 in thick plate by 4 ft long
D 50 to 80 C. welding machine A. objectionable vibration to the building
seam? 1
D. planer B. objectionable vibration to the
A2min
2 A center distance between sprockets B. 4 min* occupied space
equivalent to times pitch is 12. What is the r~ u ired base area of C. to maintain the supported machines
C. 3min
considered maximum. Very long center the foundation to support an engine w1tt elevation and alignment
D. 5min
distances result in catenary tension in speed of 1800 rpm and weight of 5000 D. all of these*
the chain. kg. Assume bearing capacity of soil as
Length of seam = welding rate x time
A 30 45.5 kPa. Use e = 0.11. 16. Foundation mass should be
4(12) = 12 x time 2
B. 80 * A. 6.1 m * from times the weight of the
time = 4 min 2
C. 50 B. 8.1 m machinery it is supposed to support.
2
D. 120 C.7. 1m A. 1 to 3
8. How long will it take to saw a
D. 6.9 m2 B. 5 to 8
rectangular piece of §!eel plate size ·1/2
3. The smaller sprockets of a drive are x 4 x 8, if the length of cut is 4 ft? The C. 3 to 5 *
0.7to10
usually made of _ __ __ power hacksaw makes 120 rpm and the w~ ~ ejNx WM
A. steel* feed per stroke is 0.008 in?
B. cast iron A. 48 min = 0.11 J~
ts~p_o_x_s-oo-o = 23,334.45 kg 17. If the unbalanced inertial forces
C. stainless steel B. 50 hr produced by the machine can be
D. bronze c ..0.83 hr * Sb = WM + Wr calculated, a mass of weight equal
D. 52 min Area to times the forces should be
4. Commonly used material for large ,_ 9 81 used to dampen vibration.
sprockets. especially in drives with large kN
\-'000+ 23,334.45) --: .. kN A. 5to 10
Time = length of cut 45.5 - = 1.000 B. 15 to 20
speed ratios.
A. cast iron * cutting rate m2 Area C.10to20*
B. steel _ 4(12)in Area = 6.1 m 2 D. 20 to 30
C. c;,arbon - 0.008 in 120 rev stroke
D. pig iron ·· --- · ··· x -~-- ·x ··-·· 13. ~oundation should be isolated from 18. For stability,.the total combined
stroke min rev floor slabs or building footings by at engine - driven roipn{ent, and
5. Preferred large sprocket material for = 50 min =0.83 hr least ___ mm around its perimeter to foundation center o grav1 y must be
severe service is: eliminate transmissior) of vibration. kept_ the foundation's top.
A . cast steel * 9. A machinery operation whereby done A. 15 A. Below*
B. boron with the work accurately fastened has a B. 25 * B. bottom
C. chromium reciprocating {forward and backward) C. 20 C. top
D. brass motion and the tool head is stationary. D. 35 D. beside
A shaping
6. Calculate the rpm for machining a B. turning 14. Foundations are preferably built of 19. The ~ght ofthe me9hine plus the
cast iron work piece 10 inches in C. planning * concrete in the proportion of 1:2:4. The weight of the foundation should.be -
diameter and the cutting speed is 100 D. reaming mach.ine should ~oJ_ Qf! placed on the distributed over a sufficient soil area ,
fpm . fo u ndati~ da¥s_bave elapsed or which is large enough to cause a
A. 682 10. A mechanism which usually do the ope rate_Q_u~he r days bearing stress witi1in._!!le safe D§lring
B. 48.2 indexing In a machine tool: have passed. ~~pacit[t>fffie soil with a factor o1 safety
c. 58.2 A. universal chuck A. 14
D. 38.2 * B. dividing head * B.5 A. 2
B. 5*
REFRESHER MANUAL 2nd Edition by JAS TORD ILLO REFRESHER MANUAL 2nd Edition by JA¥TORDILLO
43 - 8 I Day 43 - Solution Day 43- Solution 143-9
C. 3 c. 15 Machine Design
D. 7 D. 30"
32. A foot rule has how many 118 in? C. outside caliper
A. 96 • D. safety gloves
20. Foundations should be isolated from 26. FQr diesel engine, the engine
B. 120
floor slabs or building footings by at should not be placed on the foundation
until days have elapsed, nor operated
C. 48 37. Recommended best cutting angle of
least__ around its perimeter to
D. 108 drill for work on steel or cast iron is
eliminate transmission of vibration. until another days. _ _ _ _degrees.
A. 10 mm A. 7, 7
33. Calculate the rpm for machining a A.48
B. 25 mm * B. 7, 10
cast iron work piece 6 inches in B. 59 "
C. 20 mm C.10,10 *
D. 35 mm D. 10, 7
diameter. The lowest cutting speed for c . 63
cast iron is 50 fpm. · D. 50
27. For good firm soil, reinforced A. 35.3
21. Foundations are preferably built of
B. 43.3 38. How long will it take to saw a
concrete in the proportion of: concrete foundations for large engines
A. 1:3:5 may use a leaner mixture down to c. 31 .8 * rectangular piece of aluminum plate 8 in
D. 53.3 wide and 1 1/2 in thick if the length of
B. 1:2:4 * A. 1:2:3
the ct:1t is 8 in, the power hacksaw
C. 2:3:4 B. 1:3:6 •
34. To facilitate the milling (roughing) makes 120 rev/min· and the average
D. 1:2:5 C. 1:2:4
work of a cast steel material using a 1 Y:. feed per stroke is 0.0060 in ?
D. 1:3:5 A. 13.11 min
in diameter cutter choose between the
22. Weight of reinforcing steel should be
two available stock. Find the speed of B. 14.01 min
from _ __ of the weight of foundation. 28. The weight of con,erete is:
the cutter in rpm. C. 11.11 min •
A. 1/2 to 1% * A. 2604 kg/m5
B. 2204 kg/m 3
high speed steel cutter with a cutting D. 12.03 min
B. 1/2 to 3%
C. 2806 kg/m 3 speed-of 50 fpm . length of cut
C. 1 to 2% T1me= -~-__:.._
carbide -tipped cutter with a cutting
D. 1 to 5% D. 2406 kg/m 3 * cutting rate
speed of 200 fpm
23. No foundation bolts shall be less 29. For diesel engines, the foundation A. 389 r Sin
than depth may be taken as a good practical B. 509 * lme = 0.006in 120rev stroke
min in diameter.. ... - X X
rule, to be times the engine C. 572 stroke min rev
A. 10
stroke. D. 412
B. 14
C.12* A. 3.2 to 4.2 " = 11 .11 min
B. 5.2 to 7.2 V = 1rDN
D. 20
200(12) = 1T(1 .5)N
C. 4.2 to 5.2 39. How long will It take to mill a 3/4" by
N = 509 rpm
24. •Foundation bolts of specified size D. 1.2 to, 2.2 2" long keyway in a 3" diameter shafting
ould be used and surrounded by_ a with a 24 tooth cutter turning at 100 rpm
35. A concrete foundation should have
.Pipe sleeve with an inside g~ oi at 30. For steam turbines, ft should be
and 0.005 feed per tooth ?
steel bar reinforcements placed both
least ;3 tiffies the diameter of the anchor designed to s upport the machipe load
A. 0.136 min
bolt and a length of=at least _ __ plus for impact, condenser load floor vertically and horizontally to avo.id
thermal cracking. Weight of reinforcing B. 0.166 min *
times the diameter of the bolt. slabs and dead loads.
steel bars should be at least: C. 0.196 min
A. 10 A.10%
A. 4% to 4% of the weight of the D. 0.106 min
B. 18 * B.40%
c. 15 c. 25% * foundation = length of cut
B. 6% of the weight of the foundation Time
D. 20 D. 50% cutting rate
C. 3% to 3% of the weight of the
25.To prevent pulling out of the bolts
foundation T 2in
D. 1/2% to 1% of the weight of the lme = -;0~.0::-:0::-:5:-ci:-n--.:2:-:4:-t:-o-ot'"'h--.:1:-:0:-::0:-r-ev-
when the nuts are tightened, the length 31 . The act of cu~ng out a piece of - - · -- · X - .. · X - - · -.. -
metal ( at a desired shape and size. foundation • tooth rev min
embedded in concrete should be equal
to at least _ times the bolt diameter. A. broaching = 0.166 min
36. Avoid using in a lathe
A. 10 B. slitting
C. blanking • machine operation. 40. With an electric are welding rate of
B. 20
D. dinking A. wooden shoes " 18 in/min, how long will it take to weld a
B. measure ruler 1/2 in thick plate by 3ft long seam?
REFRESHER MANUAL 2nd Edition by JAS TORDILLO REFRESHER MANUAL.2nd Edition by JAS TORDILLO
43 - 10 1 Day 43 - Solution Day 44 - EXAM I 44 - 1
Machine Design
A. 3 min B. 2000
B. 1.5 min C. 1500 the crest and root. Sometimes called as
C. 2 min* D.2400 DAY 44 - EXAM partial thread, washout thread or.thread
D. 4 min \ run out.
46. Standard railings shall have posts 1. Top surface joining two sides of
Time = length of seam = 3(12) =2 min not more t han apart and an thread in a screw.
A. half thread
welding rate! 18 B. vanish thread
intermediate rail halfway between top A. pitch
C. virtual thread.
41. As a general requirement, work rail and the floor. B. top-land D. th read series
rooms shall be at least _ _ mm A . 1000 mm C. bottom land
is height from floor to ceiling. B. 2000 mm * D. crest
8. is a continuous and
A . 1000 C . 1500 mm pr.qjecting helical ridge usually of
B. 3000 * D. 2400 mm 2. Screw thread projecting from a
unifOrm section on a cylindrical or
C. 1500 cylindrical surface. conical surface.
D. 3500 47. All railings shall be of sound material A. thread series
A. screw thread
free from defects and all sharp corners B. straight thr~ad B. sharp root
42 . As a requirement, the maximum shall be roundea and smoothed. C. thread shear area C. taper thread
number of persons working or will be Toeboards shall be at least _ __ mm D. taper thread D. lead thread
working shall not exceed_ _ _ person in height.
per 12 cu meter. A. 100 3. The conical surface at the starting 9. The imaginary cylinder that would
A. one* B. 150 * end of the thread is called :
bound the crests of an external straight
B. three C. 120 A. pitch cone thread or the roots of an internal straight
C. two D. 180 B. crest thread.
D. four C. chamfer A. major cone
48. All stairs, platforms, and landings D. flute B. major clearance
43. Floor openings into which person shall be of sufficient strength to sustain C. major cylinder
cannot accidentally walk. on account of safely a live load of.not less than 500 kg 4. Surface connecting the crest and D. major clearance
fixed machinery, equipment or walls with a factor of safety of _ __ roots of the screw thread.
shall be · guarded by covers. securely A. two A. lead angle 10. The reciprocal of the axial pitch in
held in place and leaving no openings B. four* B. top land inches.
more than in width or by C. three C. length of engagement A , inch per thread
toeboards on all exposed sides. D. eight D. flank B. lead
A.15mm
C. pitch per thread
B. 25 mm * 49. Fixed ladders shall be so installed 5. is the theoretical D. Thread per inch
C. 2Qmm that the distance from the front of the profile of the thread for a length of one
D. 50 mm rungs to the nearest fixed object on the pitch in til~ axial plane on which the 11 . The -apex formed by the intersection
climbing side of the ladder is at least design forms of both the external of the .f!soks_of a thread when extended,
44. All wall openings shall be solidly thread~ are based. if necessary beyond the crest.
enclosed or guarded by fixed or rolling A. 550 mm A. basic form of thread A. crest apex
barrier rails, picket fences, half dpors, or B. 890 mm B. basic profile of thread B. sharp root
equivalent barriers, capable of C. 760 mm * C. effective head C. sharp screw
withstanding a load or at least _ _ kg D. 920 mm D. design form of external thread D. root apex
applied in any direction except vertically
upward at any point on the top rail. 50. Catwalks used for filling of tanks, cars 6. is the distance, measured 12. The apex formed by the intersection
A. 100 * or for oiling may have the railing on one radially, between the major and minor of the adjacent flanks of adjacent
B. 150 side omitted, if necessary subject to the cylinders or cones respectively. threads when extended if necessary,
C . 130 ha;zard of failing being reduced by the use A. Depth of thread beyond the root.
D. 210 of runways not less than In B. pitch A. sharp crest
width. C. length of thread
A 340 mm B. sharp root
45. Standard railings shall be at least D. lead C. crest apex
_____ mm from the upper surface B. 670 mm
D. pitch root
of the top rail to floor level. C. 560 mm * 7. That pdrtion- of the incomplete thread
A. 1000 * D. 750mm which is not fully formed at the root or at
REFRESHER MANUAL. 2nd Edition by JAS TORDILLO REFRESHER MANUAL 2nd Editton by JAS TORDILLO
44 - 2 I Day 44 - EXAM
Day 44 - EXAM I 44 - 3
13. The reciprocal of the Wed in inches. C. resolution of Woes Machine Design
A. turns. per inch D. concurrent forces
27 _The time rate of change of VeloCity. 34. The distance between the center of
B. threads per inch
C. lead per inch 20. Any influenced capable of producing
A. position oscillation and the point of suspension.
B. velocity A. radius of oscillation
D. pitch per inch of change in the motion of an object is
C. acceleration B. center of oscillation
called:
D. distance C. center of percussion
14. The radial distance between the root A. force
of, the internal thread and, the crest of B. velocity D. center of gravity
the external. thread of the coaxially C. vector 28. A quantity which has magnitude
only. 35. When the motion is decreasing
assembled designed forms of mating D. acceleration · A. vector
threads. instead of increasing.
B. scalar A. accelerated motion
A. major diameter 21. Finding the resultant of two or more
forces is called:
C. direction B. retarded motion
B. major crest
D. force C. uniform motion
C. major clearance A. composition of forces
D. lead B. components of forces D. constant velocity
29. The separate forces which can be
C. concurrent forces
so combined are called: 36. Flat belt that is too tight will induce
15. The radial distance by which their D. collinear forces
thread forms overlap each other.
A. composition of forces undue strain on the bearing and belt
B. components
A. height of thread engagement 22. Time rate of doing work:
C. resolution
- -·
A. result to uneconomical operation
B. pitch of thread engagement A. power
C. lead of thread engagement B. energy D. collinear B. suffer/shatter
D. crest of thread C. heat C. life will be shortened
D. work rate 30. Finding two or more components of D. will be sheared/cut
a given force is called the:
16. Deals only with the motion of bodies
23. The first derivative of kinetic energy A . resolution of forces 37. Pulley made of _ _ ordinarily is 45
without reference to the forces that
cause them: with respect to' time is: B. concurrent forces to 55% less in weight and 2.35 to 2.70~
A. kinetics A. motion . C. resultant lesS"-slippage comparecrwith -=._
B. energy D. scalar pulley.
B. motion
C. kinematics C. power A. wood/iron
D. acceleration D. work 31. Two forces equal in magnitude, B. steel/iron
parallel, and in opposite direction is C, iron/steel
24. Defined simply as push or pull: called: D. wood!steel
17. Progression change of position of a
A. motion
A. a resultant
body is called:
B. a vector 38. If the pulley is split or formed of
A. at:celeration B. load
C. a component separate sections that are bolted
B. force C. power
D. a c6uple together at the rim, the maximum speed
C motion D. force
should be limited to about _ _ _ of the
D. momentum
25. Property of matter which causes it to 32. Forces that are non-concurrent, maximum speed for solid pulleys.
18. The frictional force depends on resist any change in its motion or state nonparallel, and non-coplanar. A. 55-60.%
of rest. A. couple B. 45-55%
coefficient of friction and:
A. torque ...i A. matter B. skewed forces C. 50-60%
B. inertia C. vector force D. 60-70%
B. normal force
C. mass D. CO!Jlposition forces
C. weight of the object
D. moment D. velocity_ 39. The maximum safe rfm speeds for
33. of a body, volume or area, solid cast iron pulleys as a general rule
or line is that point at which if the body isabout _~_ -_.
19. Two or more forces acting together 26. The measure of the inertia of a body
is called: were suspended it would be perfectly A. 6000 fpm
could be replaced by a single force with
A. mass. balanced in cyl positions. B . 6600fpm
same effect in a mass.
B. weight. A. moment of inertia C. 5000 fpm
A. couple of forces
C. force B. polar moment of inertia D. 7000 fpm
B. resultant
D. gravity C. center of gravity
D. center of body
REFRESHER' MANUAL 2nd Edition by J_AS TORDILLO REFRESHER MANUAL 2nd Edltlon by JAS TORDILLO
44 - 4 I Day 44 - EXAM Day 44 - EXAM 144-5
. Machine Design
40. If the diameter of driven pulley is a 4 45. Most of the cast-iron pulleys have
inches, its required speed, 800 -:---- arms. C. excess of 6000 fpm C. modulus of elasticity in shear
revolutions per minute. and the diameter A.4 D. excess of 5000 fpm D. Poisson's ratio
of the driver. 26 inches, then the B.6
required speed of the driver is: C.6 51 . Which of the following types of 57. Which of the following is/are the
A. 123 rpm D. 10 leather belting that is used for primary functions of cutting fluids:
B. 199 rpm high-speed small diameter pulley A. cooling of the tool, workpiece and
C. 155 rpm 46. Cast-iron pulleys 5 feet or larger in applications and is also suitable for high chip
D. 300 rpm diameter often have arms. speed motor drives, B. reducing friction at the sliding
A. 2 A. oak tanned contacts
41 . If the diameter of the driven pulley is B.6 B. combination C.._ preventing welding or adhesion
36 inches and its required speed is 150 C.4 C. mineral retanned D. all of these
r'pm and the speed of the driving pulley D. 8 D. none of these
is 600 rpm , then the diameter of the 58. lf__a_cutting fluid is used, it should be
driving pulley is: 47. Pulley face widths are nominally the 52. Which of the following types should a soluble oil mixed to a consistency of
A. 3 inches same as the widths of the belts they are be used however, when a possibility about ·~ part oil to _ _ parts water.
B. 9inches to carry. The pulley face should be exists of llrulkLacid coming in contact A. 10 to 20
C. 6 inches approximately ___ inch f')'lore than with the belt or when the ambient B. 20 to 30
D. 12inches the belt width for belts under 12 inches temperature is above 140 deg F. C. 30 to40
wide, inches more for belts A. all of these · D. 60 to 80
42. If the diameter of the driving pulley is from 12 to 24 inches wide, and _ __ B. combinat;on
24 inches and its speed is 100 rpm, and inches more for belts over 24 inches C. oak tanned 59. For ordinary turning, boring, drilling
the driven pulley is to rot~te 600 rpm, width. D. noRe of these and milling on medium and low strength
then the diameter of the driven pulley is: A. 2,3,4 steels, use a soluble oil having a
A. 4 inches B.2,4, 6 53. Type of leather belt being applied consistency of 1 part oil to _ __
B. 8inches C. 1, 2, 3 with waterproof cement: parts water.
C. 6 inches D. 1, 3, 5 A. combination of oak/mineral A. 10 to 20
D. 10inches B. oak tanned B. 20 to 30
48. The angle on open belt drives C. all of those C. 15 to 25
43. If the diameter of the driving pulley is connecting pulleys on short centers with D. mineral retanned D ~ 1 to 80
15 inches and its speed is 180 rpm and one pulley is larger than the other, may
the diameter of the driven pulley is 9 be increased by the use of -:-:-:---:-::-- 54. Tile number of cycles required to 60. For ..to.aL steels a~h alloy
inches, then the speed of the driven pulleys on one or both sides of the belt. cause failure for given stress level is steels, a beavy Q!Jty ~oluble oil having a
pulley is: A . spilt infinite. consi~tency of , . part oil to - parts
A. 200 rpm B. idler A. endurance limit water is recommended for turning and
B. 500 rpm C. wood B. fatigue life milling.
C. 300 rpm D. divided C. fatigue A. 10
D. 800 rpm D. rupture B. 10 to 20
49. Type of belt used in places exposed c. 20
44. Wood pulleys are not only much to weather or the action of steam, as 55. Modulus of elasticity in shear is also D. 20 to 30
lighter than cast-iron pulleys but they they do not absorb moisture or stretch. known as:
are superior as transmitters of power, in A. rubber belting A. modulus of elasticity 61 . Type of cutting fluid which form is
fact it is claimed that they will transmit B. angular belting B. modulus of rigidity extracted from petroleum such as
from more power for the C. leather belting C. yield strength paraffin of, mineral seal oil, and
same belt tension. D. oak tanned belting D. modulus of rigidity in tension kerosene.
A . 30 to 50% A. soluble oils
B. 50 to 60% 50. Which of the following speeds is not 56. Modulus of elasticity in tension is B. mineral oils
C. 25 to 4o% recommended for the use of flat leather also known as: C. base oils
D. 35 to 50% belting? A. modulus of rigidity D. all of these
A. below 6000 fpm B. Young's modulus
B. below 5000 fpm

REFRESHER MANUAL 2nd Edition by JAS TORDILLO REFRESHER MANUAL 2nd Edition by JAS TORDILLO
44 - 6 I Day 44 - EXAM Day 44 - EXAM I 44 - 7
62. Type of cutting fluid which form B. round nose Machine Design
emulsions when mixed with water and C. none of these
A. minimize the risk 79. Moment of inertia of a square with
are extensively used in machining both D. base B. maximize the risk axis at the base:
ferrous and non-ferrous metals.
68. Term given to the rounded tip of the
C. control the risk A. a4/3
A. soluble oils D. maintain the risk B. a 3/12
B. base oils cutting end.
A . face
C. a3 /12
C. cutting oils 74. For applications in which material D. a/2
D. mineral oils B. nose properties are .!lot reliable and where
C. shank
loading and environmental conditions 80. Section modulus of a circle:
63. For ordinary turning and milling D. side cutting edge A. d3
are not severe, or where reliable
stainless steel a heavy duty -soluble oil materials areto be used urider difficult B. rrd 3/32
mixed with I part oil to _ _ parts water
69. The main body of the tool bit.
loading and environmental conditions, c>TTd2 /4
is recommended.
A. shank
the recommend':!d factor of safety range D. d/4
A.5 from :
B.20
C . 10
D. 30
B. flank
C. end-cutting
D. base
A. 1-3
B. 2-5
C. 3-4
81. R'adius of gyration, k = fr while

70. is the r-utting edge on


D. 1.5-2 section modulus Z is:
64. As a general rule, .fUs preferably to
supply from gpm for the side of the tool. A. 1/y
75. For use with highly reliable materials B. y/1
each single-point tool on machine such A. side-cutting edge
where loading and environmental C.ly
as turret lathe or automatic. B. end cutting edge
conditions are not severe, and where b. ly/J
A. 1 to 2 C. rake
weight is an important consideration, the
B. 3 to 5 D. back rake
recommended factor of safety to use is: 82. Polar moment of inertia of a circle:
C. 2 to 5 A.1.3-1 .5
D. 5 to 8 71 . The transition from low- to A. TTd 4/32
B.' 1.5-2 B. TTd3/16
high-cycle fatique behavior occurs in the
range from approximately 10,000 to
c. 3-4 c. d/2
65. A cutting tool a!!9.!e ~een the D. 4-7
~ide £.Utting_ edge and the e,!an~ 100,000 cycles. Many define low-cycle D. d 4
~cular to direction of feed trave fatique as failure that occurs in
_ _ __ cycles or less. 76. Moment of inertia of a>square aAi,s at 83. Polpr moment of inertia of a hollow
!§ _ _ angle.
the center: shaft:
A . side relief A. 10,000 4
A. a /3 A. 0 .098{D4 d4 )
B. lead B. 50,000
B. a 4/6 B.0.1S6D
2 -
.
C. sic.!e cutting edge C. 30,000 4
C. a C. 0.098{Da - da)
D. side rake D. 70,000
D. a 4 /12 D. D4 -d 4
66 . .Be.lieJ angles of single-point cutting 72. is a failure mode where
cyclic stresses- and - a
77. Moment of ir.ertia of a rectangle axis 84. A heat treatment process which
test in culling mild steel, cast lronand at the center: ......._
_ _ __
Oilier average work as recommended corrosion-producing environment 3
describes a
A. bd /3 time-temperature-dependent change in
should be in the range of combine to initiate and propagate cracks
in fewer stress cycles and at lower
B. bd3/12 the properties of certain alloys.
:--.,...---- - degrees for high speed c. bd3
tools. stress amplitudes than would be A. tempering
D. bd2/6 B. aging
A. 8to 12 required in a more inert environment.
B. 5 to 7 A. thermal fatique C. cementation
78. Moment of inertia of a circle of D. hardening
C.10to15 B. surface fatique
diameter d:
D.12to16 C. corrosion fatique
A. TTd2 85. The temperature at which a change
D. factor of safety 4
B. 0.049d in phase occurs.
67. The tool bit made of tool blank C. 0.098d 3
consists of face, nose, shank and 73. Factor of safety can be incorporated A. temperature range
D. d/2 B. dry bulb temperature
into design calculations in many ways.
A. cutting edge The purpose of a factor ·of safety is to: C. transformation temperature
D. quenching temperature
REFRESHER MANUAL 2nd Edition by JA.S TORDILLO REFRESHER MANUAL 2nd Edition by JAS TORDILLO
44 - 8 I Day 44 - EXAM Day 44 - Solution 144 - 9
86. A process to reduce internal residual C. cold working Machine Design
stresses in a metal object by heating the D. all of these 99. Replacing the carbon lost in the
object to a suitable temperature and surface layer from previous processing by DAY 44 - SOLUTION
holding for a proper time at that 93. Rapid cooling of steel. carburizing this layer to substantially the
temperature. A. transformation original carbon layer. 1. Top surface joining two sides of
A. tempering B. quenching A. carbon Potential thread in a screw.
B. quenching C. preheating B. carbon restoration A . pitch
C. overheating D. tempered C. carbon placing B. top-land
D. stress relieving D. carbon loading C. bottom land
94. A process of heat treatment applied D. crest*
100. After exposure to an unduly high
87. Prolonged heating of a metal at a to medium or high carbon steel in wire
temperature, it develops an undesirably
selected temperature. making prior to the wire drawing. coarse grain structure but is not 2. Screw thread 12J'Ojecnng from a
A. soaking A. patenting perrr.anently damaged. cylindrical surface.
B. overheating B. bluing A. preheated A. thread series
C. gaining C . blowing B. overheated B. straight thread *
D. quenching D. spraying C. heat treatment C. thr,ead shear area
D. none of these D. taper thread
88. A treatment applied to stabilize the 95. The interior portion of an iron-base
dimensions of a workpiece or the alloy which after case hardening is (d ~ 3. The conical surface at the starting
structure of a material. substantially softer than the surface DAY 44- ANSWER KEY end of the thread Is called:
A. equalizing layer or case. 1. D 16. C 31 . D 46. D A. pitch cone
B. stabilizing A . center 2. 8 17. C 32. 8 47.C B. crest
B. core 3. C 18. 8 33. C 48. 8
C. snapping .4. D 19. 8 34. A 49.A C. chamfer*
D. preheating C. middle 5. 8 20. 8 35.8 50. c D. flute
D. none of these 6. A 21.A·. 36. c 51. c
89. Heating to a low temperature in 7. 8 22. A' 37. 8 52. D 4. Surface connecting the crest and
order to remove entrained gases. 96. A process of case hardening an 8. A 23. C 38. A 53.C
roots of the screw thread.
iron-base by the simultaneous 9. C 24. 0 39. c 54. A-
A. baking 10.0 25. 8 40. A 55. 8 A. lead angle
B. cooking absorption of carbon and nitrogen by 11.A 26. A 41 . 8 56.8 B. top land
C. bluing heating in a cyanide salt. 12.8 27. c 42.A 57. 0 C . length of engagement
D. carburizing A. cyaniding 13.A 28. 8 43.C 58.8 0'. flank*
B. hardening 14.C 29. 8 44. 0 59. A
15.A 30.A 45. C 60. A
90. Heating to and holding at a suitable C. heating 5. is the theoretical
temperature followed by cooling at a D. case hardening profile of the thre<W, for a length of one
suitable rate. pitch in the axial plane on which the
A. heating process 97. The loss of carbon from the surface design forms of both the external
B. an nealing of an iron-base alloy as the r~sult of threads are based.
DAY 44- ANSWER KEY
C. cold treatment heating. 61. B 76. 0 91. C A. basic form of thread
D. homogenizing A. carburizing 62.A 77. 8 92.0 B. basic profile of thread *
B. cementation 60. A 78. 8 93. 8 C. effective head
91. Heating and cooling in a cycle C. carbon restoration 64.8 79. A 94.A
D. design form of external thread
65.8 80. 8 95. B
designed to produce a globular form of D. decarburization
66. A 81 . A 96. A
carbide. 67. 0 82. A 97.0 6. is the distance, measured
98. A treatment at the surface of an
A. austempering iron-base alloy, usually in the f.arm-.of-sheet
68. 8 83. A 98. A radially, between the major and rrlTnor
B. drawing 69. A 84. 8 99.8 cyjinders or cones respectively.
pr strip, on which ..b¥-tbe-action of -air-or 70. A 85. C 100. 8
C. spheroidizing ~. at a su],table temperature. 71 . 8 86. 0
A. Depth of thread *
D. induction heating A. bluing · 72. C 87. A B. pitch
B. surfacing 73.A 88. 8 C. length of thread
92. Stress relieving is also _ _ __ C. casing 74. C 89. A D. lead
A. quenching D. hardening 75. A 90. 8
B. normalizing 7. That portion- of the incomplete thread
which is not fully formed at the root or at
REFRESHER MANUAL 2nd Edition by JAS TO~DILLO REFRESHER MANUAL 2nd Editk»ii by JAS TORDILLO
44 - 10 I Day 44 - Solution Day 44 - Solution 144 - 11
the crest and root. Sometimes called as 13. The reciprocal of the Wed in inches. Machine Design
Qartial thread, washout thread or thread A. turns per inch * C. resolution of Woes C. force
runout. B. threads per inch D. concurrent forces D. gravity
A. half thread C. lead per inch
B. vanish thread * D. pitch per inch
20. Any influenced capable of producing 27. The time rate of change of Velocity.
C. virtual thread. lliha~ in the...motion of §n oBjeg_j_s A. position
D. thread series 14. The radial distance between the root called: B. velocity
of, the internar fh~~ and, the crest of Aforce C. acceleration *
8. is a continuous and the extern(!l ~ of the coaxially B. velocity"
assembled designed forms of mating
D. distance
projecting helical ridge usually of C. vector
uniform section on a cylindrical or threads. D. acceleration 28. A quantity which has magnitude
conical surface. A. major diameter
B. major crest
Q.Q.I.y.
A:' screwthread* 21 . Finding the resultant of two or more A. vector
B. sharp root C. major clearance* forces is called : B. scalar*
C. taper thread D. lead A: composition-of forces* C. direction·
D. lead thread B. components of forces D. force
15. The radial distance by which their
C. concurrent forces
9. The ima_gina!)' c_ylinder that would thread forms overlap each other.
D. collinear forces 29: The separate force.§....Y"hich can be
bound the crests of an externa l straight A. height of thread engagement*
so combined are callect
thread or the roots of an internal straight B. pitch of thread engagement
22. Time rate of doing work: A. composition of forces
thread. C. lead of thread engagement
A. power* B. components *
A. major cone D. crest of thread B. energy. C. resolution
B. major clearance C. heat D. coll inear
C. major cylinder* j_6. Deals only with the motion of bodies
D. work rate
D. major clearance withq_ut reference __to the forces tj1_at
30. Finding two or more components of_
cause them: 23. The first gerivative of kinetic energy a given force is called the:
10. The reciprocal of ttl~iaJ pjtcJLi.D A. kinetics with respect to tim.eJs:- A. resolution of forces *
inches. e. motion· A. motion B. concurrent forces
A. inch per thread C. kinemati~ * B. energy C .. resultant
B. lead D. acceleration C. power* D. scalar
C. pitch per thread D. work
D. Thread per inch * 17. Progression ~a n~ oJ,opo§iti.Ol!_of a
31 . Two forces equal in. magnitude,
body--is-calle.d:
24. Defined simply as push or pu ll: parallel, and in opposite direction is
11. The -apex formed by the intersection A. acceleration
A. motion called:
of the flanks of CLthrea.d...w.ben...e.xte.ru:!e.d~ B. force B. load A. a resultant
if necessary beyond the crest. C. motion*
C. power B. a vector
A. crest apex* D. momentum D. force* C. a component
B. sharp root D. a couple*
C. sharp screw 18. The fcjctjonal force depends on
25. Property uf matter which causes .itto
D . root apex ~oefficient
of friction and:
resist any Chao.ge in its motion or state_ 32. Forces that are non-concurr ent,
A. torque JlQ!l_Qarallel, and non-copliiu1ar.- - · -
_Qf_rest.
12. The apex formed by the intersectioJ:t B. normal force *
A. matter A. couple
of the adjacent flanks of adjacent C. weight of the object
B. inertia* B. skewed forces *
threads when extended if necessary, D. moment
C. mass C. vector force
beyond the root D. velocity D. composition forces
A. sharp crest 19. '[yvo_gr 1J10re forces acting together
B. sharp root * could be replaced by a single force ·with
26. The measure of the inertia of a' body 33. of a body, volume or area,
C. crest apex s.ame effect in a mass. -
is called: or line is that point at which if the body
D. pitch root A~ couple of forces A. mass* were suspended it WOL!I9 be perfectly
B . resultant*
B. weight. balanced in all positions.
A. moment of inertia

REFRESHER MANUAL 2nd Edition by JA..S TORDILLO REFRESHER MANUAL 2nd Edition by JAS TORDILLO
44 • 12 1 Day 44 ·Solution Day 44 • Solution I 44 - 13
Machine Design
B. polar moment of inertia C. 5000 fpm *
C. center of gravity * D. 7000 fp~ C. 25 to 4o% . 50. Which of the following speeds is not
D. center of body D. 35 to 50%* recommen~ the..use of flat leatber
40. if the .diameter of driven pulley is a 4 belting?
34. The distance between the center of inches, its required speed, 800 45. Most of the cast-iron piJ.!Ieys have A. below 6000 fpm
oscillation and the point of sus_pension. revolutions per minute, and the diameter -:-~-arms. - - - B. below 5000 fpm
A. radius of oscillation * of the driver, 26 inches, then the A.4 C. excess of 6000 fpm *
B. center of oscillation required speed of the driver is: B.6 D. excess of 5000 fpm
C. center of percussion A. 123 rpm* c. 6 *
D. center of gravity B. 199 rpm D. 10 51. Which of the following types of
C. 155 rpm leather belting that is used for
35. When the motion is decreasing D. 300 rpm 46. Cast-iron pulleys 5 feet or lf![Q~ in high-speed small diameter _ pulley
instead of increasing. diameter often have arms. a~lications and is also suitable for hig!l
A. accelerated motion 41. If the diameter of the driven pulley is A. 2 speed motor drives.
B. retarded motion * 36 inches and its required speed is 150 B. 6 A. oak tanned
C. uniform motion rpm and the speed of the driving pulley C.4 B. combination
D. constant velocity is 600 rpm, then the diameter of the D. 8 * C. mineral retanned *
· driving pulley is: D. none of these
36. Flat belt that is too tight will induce A. 3 inches 47. Pulley face widths are nomihally the
undue strain on the bearing and belt B. 9 inches" same as the widths of the beTts_the~.are.. 52. Which of the following types should
C. 6 inches to carry. The pulley face should be be used however, when a possibility
A. result to uneconomical operation D. 12 inches · approximately ___ inch more than exists of liquid acid coming in contact
B. suffer/shatter the belt width for belts under 12 inches with the belt or when the ambient
C. life will be shortened • 42. If the diameter of the driving pulley is wide, . inches more for belts temperature is above 140 deg F.
D. will be sheared/cut 24 inches and its speed is 100 rpm, and from 12 to 24 inches wide, and__,.___,.- A. all of these
the driven pulley is to rotate 600 rpm, inches more for belts over 24 inches B. combination
37. Pulley made of __ ordinarily is 45 then the diameter of the driven pulley is: width . C. oak tanned
to 55% less in weight and 2.35 to 2. 70% A. 4 inches" A. 2,3, 4 D. none of these *
less slippage compared with B.8inches B. 2,4,6
pulley. C. 6 inches c. 1, 2, 3 * 53. Type of leather belt being applied
A. wood/iron D. 10 inches D. 1, 3, 5 with waterproof cement:_
B. steel/iron • A. combination of oak/mineral
C. iron/steel 43. If the diameter of the driving pulley is 48. The angle on open belt drives B. oak tanned
D. wood/steel 15 inches and its speed is 180 rpm and connecting pulleys on short centers with C. all of those*
the diameter of the driven pulley is 9 one pulley is larger than the other, may D. mineral retanned
38. If the pulley is split or formed of inches, then the speed of the driven be increased by the use of - , - - - -
separate sections- that are -bolted pulley is: pulleys on one or both sides of the belt. 54. Tile_ number of cycles required_to_
together at the rim, the maximum speed A. 200 rpm A. spilt · cause failure for given stress lavel is
should be limited to about of the B. 500 rpm B. idler* infinite.
maximum speed for solid pulleys. C. 300 rpm* C. wood A. endurance limit"
A . 55-60.% * D. 800 rpm D. divided B. fatigue life
B. 45-55% C. fatigue
c. 50-60% 44. Wood pulleys are not only much 49. Type of belt used in places exposed D. rupture
D. 60-70% lighter than cast-iron pulleys but they to weather or the action of steam, as
are superior as transmittersof power, in they do not absorb moisture or stretch. 55. Modulus of elasticity in shear is also
39. The maximum safe rfm speeds for . fact it is claimed that they will transmil A. rubber belting * known as:
solid cagj_ron pulleys as a general rule from more power for the B. angular belting A. modulus of elasticity
is about _ __ same belt tension. C. leather belting B. modulus of rigidity *
A . 6000 fpm A. 30 to 50% D. oak tanned belting C. yield strength
B. 6600 fpm B. 50 to 60% D. modulus of rigidity in tension

REFRESHER MANUAL 2nd Edition by J~S TORDILLO REFRESHER MANUAL 2nd Edition by JAS TORDILLO
44- 14 1Day 44 -Solution Day 44 - Solution 1 44 - 15
Machine Design
56. Modulus of elas!icity in teosionJ.s C. base oils
also known as: D . all of these 67. The tool bit made of tool 'blank 73. Factor of safety can be incorporated
A. modulus of rigidity _ consists -.at
face, nose, shan!¢ and into design calculations in many ways.
B. Young 's modulus • 62. Type of cutting fluid which JD.rm The purpose of a factor of safety is to:
C. modulus of elasticity in shear emulsions when mixed with water and A. cutting edge A. minimize the risk *
D. Poisson's ratio are extensively used in machining both B. round nose B. maximize the risk
ferrous and non-ferrous metals. C. none of these C. control the risk
57. Which of the following is/are the A. soluble oils * D. base '• D. maintain the risk
primary functions of cutting fluids: B. base oils
A. cooling of the tool, workpiece and C. cutting oils 68. Term given to the rounded ![Qcof the 74. For applications in which material
chip D. mineral oils QUtt[!:]g end. properties are not reliable and where
B. reducing friction at the sliding A. face loading and environmental conditions
contacts 63. For ordinary turning ·and milling B. nose" are not severe, or wbere reliable
C. preventing welding or adhesion stainless steel a hea~ uty -soluble oil C. shank materials are to be used under difficult
D. all of these * · mixed with I part oil to_ _ parts water D. side cutting edge loading and environmental conditions,
is recommended. the recommended factor of safety range
58. lLa cutting_fluid is us~. it should be A.5" 69. The main body of the tool bit. from:
a soluble oil mixed to a consistency of B. 20 A. shank* A. 1-3
about 1 part oil to _ _ parts water. c. 10 B. flank B. 2-5
A. 10 to 20 D. 30 C. end-cutting c. 3-4 *
B. 20 to 30 • D. base D. 1.5-2
C 30 to 40 64. As a general rule, ft is preferably to
D. 60 to 80 supply from gpm for 70. is the cutting edge on 75. For use with highly reliable materials
each single-point tool on machine such the side of the tool. where loading and environmental
59. For ordinary turning, boring, drilling as turret lathe or automatic.' A. side-cutting edge • conditions are not severe, and where
and milling on medium and low strength A. 1 to 2 B. end cutting edge weight is an important consideration , the
stee l~ use a soluble oil having a B. 3 t o 5 • C. rake recommended factor of safety to use is:
consistency of 1 part oil to _ __ C. 2 to 5 D. back rake A. 1.3-1.5 *
parts water. D. 5 to 8 B. 1.5-2
A. 10 to 20" 71. The transition from low- to C. 3-4
B. 20 to 30 65. A cutting tool angle between the high-cycle fatique behavior occurs in the D. 4-7
C. 15 to 25 .side ·cutting edge and tne plane range from approximately 10,000 to
D. 1 to 80 perpendicular to direction of feed travel 100,000 cycles. Many define low-cycle 76. Moment of inertia of a square axis at
is _ _ angle. fatique as failure that occurs in (he center:
4
60. For tool steels and tough alloy A . side relief _ _ _ cycles or less. A . a /3
steels, a heavy duty soluble oil haVing a B. lead* A. 10,000 B. a 416
consistency of I part oil to _ parts C. side cutting edge B. 50,000" C. a 4
water is recommended for turning and D. side rake C. 30,000 D. a 4/12 *
milling. D. 70,000
A. 10 • 66. RelieLangles of single-Q_oint cutting 77. Moment of inertia of a rectangle axis
B. 10 to 20 test in cl1111ng mild steel, cast iron and 72. is a failure mode where at the center:
c. 20 other average work as recommended cyclic stresses- and a A. bd 3/3
D. 20 to 30 should be in the range of corrosion-producing environment B. bd 3/12 *
- -- - - - degrees for high speed combine to initiate and propagate cracks C. bd3
2
61 Type of cutting fluid ...wbich form is tools. in -fewer stress cy_c.les_anLaLJuweL D. bd /6
extracted. from petroleum~usll_.as A. 8 to 12 * stress amplitudes than would be
paraffi n of, mineral seal oil> and B. 5 to 7 required in a more inert environment. 78. Moment of inertia of a circle of
ketose.ne_ C. 10 to 15 A. thermal fatique diameter d :
A. soluble oils D. 12 to 16 B. surface fatique A. TTd2
B. mineral oils • C. corrosion fatique • · B. 0.049d 4 *
D. factor of safety

REFRESHER MANUAL 2nd Edition by 'J.AS TORDILLO REFRESHER MANUAL 2nd Edition by JAS TORDILLO
44 - 16 1 Day 44 - Solution Day 44;, Solution 144 -17
Machine Design
c. 0.098d C. transformation temperature*
D. d/2 D. quenching temperature 92. Stress relieving is also _ _ __ B. surfacing
A. quenching C. casing
79. Moment of inertia of a square with 86. A pro_cess to reduce internal residual B. normalizing D. hardening
axis aUhe base: stresses in a metal object by heating the C. cold working
99. Replacing the carbon lost in the
A. a4 /3 . - - object to a suitable temperature and D. all of these *
3
surface layer from previous processing oy
B. a /12 holding for a proper time at that carburizing this layer to substantia~ the
3 93. Rapiq cooling of steel.
C. a /12 temperature. original carbon layer. ·
D. a/2 A. tempering A. transformation A. carbon Potential
B. quenching B. quenching * B. carbon restoration •
80. Section modulus of a circle: C. overheating C. preheating C. carbon placing
A. d3 D. stress relieving * D. tempered D. carbon loading
B. rrd 3/32 * 100. After exposure to an unduly high
C. TTd 2/4 87. Prolonged heating of a metal at a 94. A process of heat treatment applied
temperature, it develops an undesirably

rr
D. d/4 selected temJ:Jerature. - tq_ medium or high carbon steelin wire coarse _grain structure but is not
A. soaking ..- - making prior to the w ire drawing. permanently damaged.
81. Radius of gyration, k = while 8 . overheating A. patenting * A. preheated
C. gaining B. bluing B. overheated *
D. quenchi'ng C. blowing. C. heat treatment
section modulus Z is:
A. 1/y * D. spraying D. none of these
B. y/1 88. A treatment apRJled to stabilize the
C. ly dimensions of a workpiece or the 95. The interior portion of an iron-base
D. ly/J structure of a material. - alloy which after case hardening is
A. equalizing substantially softer than the surface
82. Polar moment of inertia of a circle: B. stabilizing * la:t.ecor cas~ .
A. rrd 4 /32 • · C. snapping A. center
B. TTd3 /16 D. preheating B. core •
c. d/2 C. middle
D. d 4 S9. Heating to a low temperature in D. none of these
order to remove entrained gases.
83. Polar moment of inertia of a hollow A. baking- 96. A process of case hardening an
shaft: B. cooking iron-base by the simultaneous
A. 0.098(D4 d 4 ) * C. bluing absorption of carbon and nitrogen by
B. 0.196D2 - D. carburizing heating in a cyanide salt.
c. 0.098( D3 - d3} A. cyaniding *
D. D4 - d4 90. Heating to and_h.oldlng at a suitable B. hardening
temperature followed by cooling at a C. heating
84. A heat treatment process which suitable rate. D. case hardening
describes a A. heating process
time-temperature-dependent chang~ in B . annealing * 97. The loss of carbon from the surface
the properties of certain alloys. C. cold treatment of an iron-base alloy as the result of
A. -tempering D. homogenizing heating.
B. aging • A:Carburizing
C. cementation B. cementation
D. hardening 91. Heating and cooling in a cycle C. carbon restoration
designed to produce a globular form of D. decarburization *
85. The temperature at which a change carbide.
98. A treatment at the surface of an
in phase occurs. -- A. austempering
iron-base alloy, usually in_the form of sheet
A . temperature range B. drawing or strip, on which by the action pf air or
B. dry bulb temperature C. spheroidizing * steam at a suitable temperature.
D. induction heating A. bluing*

RE.FRESHER MANUAL 2'!d Edition by J.AS TORDILLO REFRESHER MANUAL 2nd Edition by JAS TORDILLO
Day 45 - EXAM I 45 - 1
Machine Design

DAY 45- EXAM and the cutting tool has a reciprocating


(forward and return) motion:
A. planer
1. A place in which metal parts are cut
B. milling machine
to size required and put together to form
C. turning machine
mechanical machines, and it the basis
D. shaper
of all mechanical production:
A. Machine Shop
B. Welding Shop 8. A machine which is ordinarily used for
C. Machine Tools finishing flat or partly curved surfaces of
D. Machine Shaft metal pieces few in number and not
us~:Jally over a foot or two long.
A. planer
2. The following are considered B. lathe
standard machine tools:
C. boring machine
A. drill press
D. shaper
B. boring mill
C. all of these
9. The cuts on the shaper is
D. grinding machine
- o - : - - - - stroke
only.
A. backward
3. Which of the following is not
B. sideward
considered as a manufacturing
machine?
C . forward
D. straightward
A. the turret lathe
B. the slotter i
10. The cutting tools used il) the shaper
C. the milling machine
are similar to the turning tools used in
D. the gear-cutting machine
the lathe machine.
A. cannot be
4. In this machine the work is securely B. false
held while a revolving cutting tool is fed C. true
into it.
D. not applicable
A. Shaper
B. drill press
11 . In this machine the table on which
C. taper
D. planer the work is sec'*l:ely fastened, has a
reciprocating (forward and return)
motion, the tool head may be
5. Is a machine tool used mainly for
automatically fed horizontally, and is
producing holes in metal.
used in the production of flat surfaces
A. Lathe machine
on pieces too large or too heavy to hold
B. drilling machine
in a shaper.
C. shaper
A. shaper
D. None of these
B. planer
C. boring
6. The cutting tool most commonly used
D. Gutter
in a drilling machine is called:
A. drill bit
12. Cutting tools used in planer work are
B. puffer
the same as those used in the shaper.
C. driller
A false
D. slotter
B. true
C. cannot be
7. In this machine the work is usually
D. not applicable
held in a vise bolted to the work table,
REFRESHER MANUAL 2nd Edition by JAS TORDILLO
"""l:ij ,
45 - 2 I Day 45 - EXAM Day 45 - EXAM I 45 - 3
Machine Design
13. A machine in which metal is C. floor work
removed by means of a revolving Gutter D. bench work 25. One of the following is an _un~a.fe 30. The chief accident when grinding is:
with many teeth, each tooth having a operating practice of a milling machine. A. eye injuries from flying particles
cutting edge which removes its share of 19. in a machine shop opelator. B. eye injuries from wearing goggles
the stock. consists of laying out, assembling, and A tightening the arbor nut by using the C. injuries from wearing rolled up
A. lathe the fitting of parts on parts on heavy power of the machines sleeves
B. boring w orks. B. never adjust the work or tool while D. injuries from running coned speeds
C. holing A. machine work the machine is in motion
D. milling B. mechanic work C. never wear loose clothing 31. is the. term used in
C. floor work D. all of these industry to mean the safe storage of
14. A machine tool in which an abrasive D, bench work tools, parts, and all the rest of the items
wheel is used as a cuffing -tool to obtain 26. Which of the fol lowing is an unsafe used in manufacturing.
a very degree of accuracy and a smooth 20. Which of the following is/are a practice of a planer and shaper A. store room
finish on metal parts: branch/s of toolmaking? operator? B. housekeeping
A. milling wheel A. all of these A. chang ing stop dogs while the C. stockroom
B. boring wheel B. diemaking machine is in motion D. machine room
C. welding C. jig making B. all of these
D. grinding machine D. gage making C. riding the table during the operation 32. A general safety precaution in a
D. wearing sleeves machine shop " Do not attempt to oil,
15. A machine which is a modification of 21 . The chief hazard of machine tools clean, or any machine
the lathe machine. that perform in milling function is: 27. One the following is a safe practice while ft is running.
A. boring mill A. accidental contact with the table of a shaper operator? A. adjust
B. miiUng machine B. accidental contact with the revolving A. watching his job B. maintain
C. honing cutter B. all of these C. stop
D. fitting C. accidental contact w ith the chips C. he is wearing safety glasses D. talk
D. all of these D. There are no tools lying in the
16. A type of boring mill in which the machine 33. A general safety precaution in a
cutting tool is arranged above the table 22. Loose clothing creates a serious rnachine shop "Do not try to
and may be fed laterally (toward or h<!zard around a rotating cutter in a 28. Which of the following is a good ,.-------- the machine with your
away from the center of the table) and milling machine. Which of the following solely practice in using lathe machine? hands or body.
up or down in any position, and the work is ·a safE! dress for an operator? A . from being drawn into the lathe if A. stop
table revolves on a vertical axis. A . loose sleeve with necktie loose clothing catches on the revolving B. dean
A. vertical boring mill B. rolled up loose sleeve with goggles work C. run
B. horizontal boring mill C. loose sleeve witn.goggles B. frbm contact with chuck or lathe dog D. all of these
C. center boring mill D. apron C. from attempting to remove dips with
D. above boring mill the hands 34. When working with another student,
23. One of the following is a safe D. from wearing apron and safety only should operate
17. A type of boring mill in which the practice to avoid eye hazard of a milling glasses machine or switches.
cutting tool may be fed longitudinally machine operator. A. one
through the spindle head and the cutting A. wear rolled up sleeves 29. Which of the following is NOT a B. three
tool revolves on a horizontal axis. B. wear necktie cause of accidents of boring in a lathe C. two
A. vertical boring mill C. wear goggles machine? · D. four
B. horizontal boring mill D. wear apron A. 1f the work is not securely clamped
C. center boring mill B. the proper clearances am not 35. Do not wear rings, watches,
D. up-down boring mill 24. Which of the following is a safe and maintained bracelets, or other jewelry or
a good machine operator practice? C. when adjusting the machine without clothing that could get caught in moving
18. in a machine shop A. wear sleeves rolled up bothering to stop it machinery.
consists of laying out, assembling, and B. all of these D. clearing away chips with the hands A. neckties
the final fitting of parts. C. wear goggles while the drill is not in motion B. shirts
A. machine work D. standing on the going away side of a C. rolled up jumpers
B. mechanic work cutter D. face shields

REFRESHER MANUAL 2nd Edition by JAS TORDILLO REFRESHER MANUAL 2nd Edition by J.~S TORDILLO
45 - 4 I Day 45 - EXAM Day 45 - EXAM I 45 - 5
Machine Design
36. Al','.(ays remove gloves before C. rule
turning _ _ or operating any D. sine bar A. caliper A. to loosen screws only
machine. if the material is rough or B. micrometer B. to tighten screws only
sharp and gloves must place 42. A measuring tool used in linear C. spring ru le C. all of these
or handle material with machine turned measurements in which the dimension D. sine rule D. to loosen and tighten screws
off. is read directly from a graduated scale
A. on, be worn or standard: 48. One of the following is NOT a style 54. The three main parts of a screw
B. off, be worn A. dividers of a caliper: driver are: the handle, the shank, and
C. on. not be worn B. calipers A. transfer caliper the _ _
D. off, not be worn C. sine bar B. spring-joint caliper A. blade
D. steel rule C. firm-joint caliper B. body
37. For clothing and safety equipment" D. none of the these C. tip
Always wear designed for 43. A measuring tool used in linear D. Phillips screw
the type of work when operating any measurements which is used to transfer 49. In rapid-production work it is usually
machine. the measurement: advisable to use gages for determining 55. A part of the screw driver, the end
A. safety glasses A. dividers the correct sizes. For cylindrical work which fits into the slot of the screw is
B. goggles B. calipers the is used and where a light called
C. face shields C. surface gage variation over or under nominal size is A. blad:-e- -- -
D. all of these D. all of these allowable a limit gage is used. B. shank
A. limit gage C. body
38. For housekeeping safety "Do not 44. Many related tools such as B. snap gage D. Ph illips
leave tools or work on the table of a straightedges, steel, squares and C. flat gage
machine even if the machine is protractors are used in conjunction with D. angular gage 56. A part of the screw driver, the steel
_ _ _ _ _ .Tools or work may fall off linear measuring tools to determine: portion extending from the handle is the
and cause tpe or foot injury." A. flatness 50. Micrometer calipers are made in a
A . running B. straightness variety of sizes ranging from _ _ _ A. blade
B in operation C. angularity A . 1 into 60 in B. shank
C. not running D. all of these B. 1/2 in to 48 in ~-body
D. none of these C. 1 in to 50 in D. handle
45. For round work, measurements are D. 1/2 in to 24 in
39. In a shaper, after setting the stroke usually • made by contact, using tools 57. The most common types of screw
length and position, check to see that with contact points or surfaces such as 51 . A very simple striking tool and made drivers are standard, offset, ratchet and:
adjusting nuts are tight. Stand -:-:--- spring calipers, micrometers and of a weighted head and a handle which A . double-end
to the direction of stroke of machine A. vernier calipers dire~ts its course. B. single-end
when it is running. B. straight calipers A. box C. Phillips
A. vertical C. flexible calipers B. hammer D. blade
B. horizontal D. all of these C. pedal
C. parallel D. metal drivers 58. An instrument for exerting a twist ing
D. in front 46. A useful measuring tool in the strain, as in turning bolts and nuts.
machine shop which makes possible 52. Machinists' hammer are made of A. Phillips screw driver
40. In a shaper, never remove chips accurate measurements against shallow steel, hardened and tempered. The top B. wrench
while ram is.,----- - shoulders. of the hammer head is called the C. standard screw driver
A. not in motion A. hooked rule _ _ _ and the bottom is called the D. hammer
B. not running B. flexible rule
C. in motion C. spring rule A. face , peen 59. A type of wrench used by machinists
D. not in motion D. all of these B. peen, face to determine how much twisting force
C. head, tail you are applying.
41 . One of the following is not used in 47. A tool which is used in measuring D. upper, lower A. torque wrench
Linear Measurements. diameters. It is always used with a steel B. open wrench
A. steel tape scale. It is used as a mea'suring tool or 53. One of the main functions of screw C. socket wrench
8 . divider as a gage. drivers is to: D. alien wrench

REFRESHER MANUAL 2nd Edition by JAS TORDILLO REFRESHER MANUAL 2nd Edition by JAS TORDILLO
45 - 6 I Day 45 - EXAM Day 45 - EXAM I 45 - 7
Machine Design
60. Another type of wrench used in 67. Type of drill press useful when
machine work on w hich -~ockets of drilling several holes in the larger and 73. The-operation of eRiarging a hole by C. body
different sizes are attached to the heavier pieces. means of an adj ustable cutting tool witl:l D. point
handle for various sizes of nuts. A. sensitive only one cutting edge.
A. torque wrench B. gang A. expansion 80. The backbone of the drill bit and it is
B. socket wrench C. radial B. boring the narrow section -between the flutes.
C. alien wrench D. vertical C holing A. body
D. adj ustable wrench D . reaming B. cone
68. Type of drill press which is a C. web
61. Another type of wrench used in collection of one machine of the 74. The operation of forming internal D. margin
machine work especially designed for essential speed and feed units of from threads .
screws with allen head. two to eight single drill presses mounted A . threading 81 . The most commonly used type of
A. allen screw on one base. B. tapping drill bit in the machine shop are:
B. allen wrench A. sensitive C. facing A . straight drills
C. allen nut B. horizontal D. grinding B. short drills
D. special wrench C. gar.tg • C. twist drills
D. multiple 75. The operation of making a D. long drills
62. A n instrument used to saw metal. co ne-s haped enlargement of the end of
A. cutter 69. A part of the drill press which serve a hole, as 82. Type of drill bit used for drilling
B. hacksaw as a table of machine on which the work for a recess for a- flathead screw. brass, copper, and other soft metals.
C. flatter Is clamped or on which the Vise are A. counterboring A. cross-fluted dril l
D. bender placed and clamped . B. countersinking B. side-fluted drill
A. base C. spot-facing C. straight-fluted drill
63. The width of a hacksaw blade is: B. spindle D. expanding D. curved-fluted drill
A. %in C. handwheel
B. Yz in D. box w ork 76. The purpose of this part is to help 83. The angle. of clearance at the center
C. 3/4 in drive the drill, since the hold of the taper must be proportional to the angle at the
D. 1 in 70. A part of the drill press which is the alone is not sufficient. outside. The clearance on the drill is
part holding the drill chuck or socket. A. tang about _ _ _ degree at the cutting
64. The number of teeth per inch of a A. head claming B. shank edge.
hacksaw blades ranges from: B: box column C. taper A. 15
A. 14 to 32 C. head adjusting box D. chuck B. 25
B. 16 to 28 D. spindle c. 12
C.14to24 77. A gripping device with two or more D. 30
D. 16 to 2-4 71. The operation of produ cing a circular adjustable jaws set radially.
hole by removing solid metal. A. drill chuck 84. A correctly sharpened drill bit, the
65. The length of a hacksaw blades A. reaming · B. spindle edge of the angle across the web ilf the
ranges from: B. boring C. drift slot drill w ill be about _ _ _ degree with
A. 8 to 16 in C. spotti ng D. control shaft the line of the cutting edges.
B. 10 to 24 in D. drilling A. 24
C. 8 to 24 in 78. Part of the drill bit that fits into the B. 60
D. 10to30in 72 . A n operation of sizing and finishing holding device. C. 45
a hole by means of a cutting tool having A. shank D. 70
66. Type of drill press used for drilling several cuffing edges. B. Flute
small holes with hand feed. A. honing C. tang 85. The angle of clearance of a drill bit
A. rad ial drill press B. boring D. point used in culling soft and medium
B. spindle drill press C. holing A. 12 degree
C. sensitive drill press D. reaming 79. The groove of the drill bit and carries B. 7 to 12 degree
D. gang drill out the chips and admits the coolant. C. 12 to 15 degree
A . flute D. 15 degree
B. lips

REFRESH ER M ANUAL 2 nd Edition by JAS TORDILLO REFRESHER MANUAL 2nd Edition by JAS TORDILLO
~ -~

45 - 8 I Day 45 - EXAM Day 45 - Solution I 45 - 9


Machine Design
86. Lip angle of drills used in general 93. Which of the following operation
work is: ~ may also be done on the lathe? B. tailstock
A. 12 degree A. drilling C. headstock DAY 45 - SOLUTION
B 15 degree B. all of these D, live center
C . 59 degree C. reaming 1. A place in which ~tal parts are cut
D. 78 degree D. tapping 100. This center which does not move to size required and put togetherto form
with the work is called: mechanical machines, and it the basis
87. An angle that forms the cutting 94. The common types of lathes are A . live center of all mechanical production:
wedge: bench lathe, engine lathe and _ _ _. B. dead center A. Machine Shop •
A lip angle A. standard lathe C. center bearing B. Welding Shop
B. rake angle B. turret lathe D. center of attraction C. Machine Tools
C. clearance angle C: e ngine lathe D. Machine Shaft
D. tang angle D. bo~;_e- lathe
I' 2. The following are considered
88. The point angles of twist drills 95. A ·small lathe, usually mounted on a (r-1 ) standard mar;hine tools:
ra nges from . - - - -· bench, and is used for small work. A. drill press .
A. 40 to 60 degree A. bench lathe DAY 45- ANSWER KEY B. boring mill J
1. A 1<3. A 31. 8 46.A
B. 60 t o 150 degree B. engine lathe 2. C 17. 8 32. A 47. A
C. all of these *
C. 60 to 118 degree C. turret lathe 3. C 1B. 0 33. A 48. 0 D. grinding machine-
D. 50 to 70 degree D. all ofthese 4. 8 19. C 34. A 49. 8
5. 8 2D. A 35. A 50. 8 3. Which of the _following is nQt
89. The point angle of twist for drilling 96. The machine tool in which many 6. A 21 . 8 36.A 51. 8 ~d as a manufacturing
7. 0 22. 8 37. 0 52. 8
copper is: lathe operations are done. This lathe is, 8. 0 23. c 38. c 53. 0
machine?
A. 60 ·degree most valuable machine tool in the shop. 9. C 24. 8 39. C 54. A A. the turret lathe
B. 100 degree A. bench lathe 10 C 25: A 40. C 55. A B. the slotter
C . 90 degree B. engine lathe 11.8 26. 8 41 . 0 56.8 C. the milling machine •
D. 118 degree C. turret lathe 12.8 ~7. B 42. 0 57. C D. the gear-cUlling machine
13.0 28. 0 43. 0 58. 8
D. all of these 14.0 29. 0 44. 0 59. A
90 The point angle of twist for drilling 15.A 30. A 45. A 60. 8 4. In this machine the work is securely
average class of work is: 97. This lathe is a manufacturing ~ while a revolving cutting tool rs fed
A. 60 degree machine. Considerable mechanical skill ~ !=============~
'-./--= into it.
B. 118 degree is required to make and adj ust the A . Shaper
C. 75 degree several cutting tools . B. drill press •
D. 150 degree A . bench lathe (d ) C. taper

I
B. engine lathe OAY 45- ANSWER KEY D. planer
91 . A machine tool in which the work C. turret lathe 61. 8 76. A 91. B
revolves and the tool us ually moves D. all of these 62. 8 77. A 92. 8 5 . Is a machine tool used-main!Y.Jor
along a straight line. 63. 8 78. A 93.8 ~m>ducing holes in metal.
64. A 79.A 94.8
A. shaper 98. Part of the· lathe machine, which 65. A 80. C 95. A
A. Lathe machine
B. lathe serves as the foundation on which the 66.C 81. C 96. 8 B. drilling machine •
C. milling lathe is bui lt. 67.C 82. C 97. C C. shaper
D. drill press A. bed 68. C 83. C 98. A D. None of these
B. base 69. 0 84. C 99. 8
70. D 85. C 100. B
92. The general lathe operations are C. chuck 71 D 86. C 6. The cutting tool most commonly used
straight turning, taper turning, boring D. carriage 72. D 87. 8 in a drilling machine is called:
and....,.-,.,.-- 73. 8 88. B A. drill brt "
A. welding 99. Part of the lathe machine, for the 74. B 89. 8 B. puffer
75. B 90. 8 C. driller
B. cylindrical turning purpose primarily of giving an outer
C. pressing
D. filing
bearing and support for work being
turned on centers.
G D. slotter

A. bed 7. In this machine the work is usually


held in a vise bolted to the work table,
REFRESHER MANUAL 2 nd Edition by JAS TORDILLO·: REFREStiER MANUAL 2 nd Edition by .!_AS TORDILLO
45 - 10 1 Day 45 - Solution Day 45 -Solution 145 - 11
Machine Design
and the C\)tting tool has a reciprocating 13. A machine in which metal is
~rd and retu..[.n) motion: removed by means of a revolving ~tter C. floor work D. standing on the going away side of a
A. planer with many teeth, each tooth having a D. bench work* cutter
B. milling machine cutting edge which removes its share of
C. turning machine the stock. 19. in a machine shop 25. One of the following is an ~fe
D.shaper* A. lathe consists of laying out, assembling, and OBeratipg practice of a milling machine ·
B. boring the fitting of parts on parts on ~ operator.
8. A machine which is ordinarily used for C. holing ~S . A. tightening the arbor nut by using
fllli.shing flat or partw cprved surfaces of D. milling* A . machine work the power of the machines •
,a!Jllill pieces few in number and Wli B. mechanic work B. never adjust the work or tool while
ll$!J2lly O'iAC a foot or two long: 14. A machine tool in which an abrasive C. floor work* the machine is in motion
A. planer -. wlleel is used as a cuffing -tool to obtain D. bench work C. never wear loose clothing
B. lathe a very -degree of accuracy and a smooth D. all of these
C. boring machine finish on metal parts: 20. Which of the following is/are a
D.shaper* A. milling wheel braaQ.lj s of toolmaking? 26. Which of the following is an unsafe
B. boriog wheel A. all of these * ~ of a planer and Sha'Per
9. The cuts on the shaper is C. welding B. diemaking operator? .
_ _ ___ stroke only. D. grinding machine * C. jig making A. changing stop dogs while the
A. backward D. gage making machine is in motion ·
B. sideward 15. A machine which is a modification of B. all of these *
C. forward* the lathe machine. 21 . The phief hazard of machine tools C. riding the table duril:lg the operation
D. straightward A. boring milf' that eertorm.irt..milling function is: D. wearing sleeves
B. milling machine A. accidental contact with the table
10. The cytling taols used in the shaper G. honing B. accidental contact with the 27. One the following is a safe practice
are~ to th'e turning tools used in D. fitting revolving cutter * of a shaper operator?
the lathe machine. C. accidental contact with the chips A. watching his job
A. cannot be 16. A type of boring mill in which the D. all of these B. all of these *
B. farse cutting tool js arranged ab.Q.ve...the table <i· he is wearing safety glasses
C. true • a~d may be fed lg_tera.J.!y (toward or 22. Loose clothing creates a serious D. There ar.e no tools lying in the
D. not applicable away from the center of the table) and hazard around a rotating cutter in a machine
up· or down in any position, and the work milling machine. Which of the following
11 . In this machine the table on which table revQlyes on a yertjcal axis. - - is a safe d(,es_s for a!lJ)peraklr? 28. Which of the following is a QQ.Qd
the work is securely fastened, has a A. vertical boring mill * A. loose sleeve with necktie _solely practjce in usjng lathe machine?
reciprocating (forward and return) B. horizontal boring,JT1ill B. 'rolled up loose sleeve with A. from being drawn into the lathe if
motion, the tool head may be C. center boring mill goggl~s* loose clothing catches on the revolving
automatically fed horizontally, and is D. above boring mill C. loose sleeve with goggles work
used in the production of flat surfaces D. apron • B. from contact with chuck or latht: dog
on pieces too large or too heavy to hold 17. A type of boring mill in which the C. from attempting to remove dips with
in a shaper. cutting tool may be fed longitudinally 23. One of the following is a safe the hands
A. shaper through the spindle head and the cutting practice to avoid .eY..e hazard of a milling D. from wearing apron and safety
B. planer* tool revolves on a horizontal axis. ma~ine operator. glasses*
C . boring A. vertical boring mill A. ear rolled up sleeves
D. Gutter B. horizontal boring mill * B. ear necktie 29. Which of the following is N.QI. a
C. center boring mill C. wear goggles * ~of accidents of boring in a lathe
12. Cutting tools used in planeLWO!k are D. up-down boring mill D. wear apron machine?
the same as those used in the shaper. A. if the work is not securely clamped
A-:tais_e_ 18. in a machine shop 24. Which of the following is a safe and B. the proper clearances am not
B. true* consists of laying out, assembling, and a good machine operator practice? maintained
C. cannot be tbe final fitting of ~s. - A. wear sleeves rolled up C. when adj usting the machine without
D. not applicable A. machine work B. all of these * bothering to stop it
B. mechanic work C. wear goggles

REFRESHER MANUAL 2nd Edition by JAS TORDILLO REFRESHER MANUAL 2nd Edition by JAS TORDILLO
45 -121 Day 45- Solution
Day 45 - Solution 1 45 - 13
D. clearing away chips with the C. rolled up jumpers Machine Design
hands while the drill is not in motion" D. face shields
41 . One of the following is not used in
36. Always relllQve gloves before Linear Measurements. 47. A tool which is used in measuring
30. The chief accident when grinding is: -<
tucni!J.g _ _ or operating any A. steel tape diameters. It is always used with a steel
A. eye injuries from flying particles "
B. divider scale:If is used as a measuring tool or
B. eye injuries from wearing goggles machine. if the material is rough or
C . rule
C. injuries from wearing rolled up sharp and gloves must place ~-
or handle material with machine turned
D. sine bar • A. caliper *
sleeves
D. Injuries from running coned speeds off. B. micrometer
A. on, be worn * 42. A measuring tool used in linear C. spring rule
B. off, be worn measurements in which the dimension D. sin·e rule
31 . is the term used in
C. on, not be worn is read directly from a gradUated scale
industry to m~ the §.§fiLsior~f
or standard: - - - 48. One of the following is NOT a style
!QQ!s, ~s. ar)d all the rest of the items D. off, ·not be worn
used jn manufacturing.
A dividers of a caliper:
37. Fc>r clothing and safety equipment " B. calipers A transfer caliper
A. store room -.
Always wear · designed for C. sine bar B. spring-joint caliper
B. housekeeping "
D. steel rule * C. firm-joint caliper
C. stockroom the type of work wl:len operating any
D. machine room machine. D. none of the these *
43. A measuring tool used in linear
A. safety glasses
B. goggles measurements which is used to transfer 49. In rapid-production work it is usually
32. A ~oeral safety___Q@_«_aJ,Jtjon in a
the measurement: advisable to use gages for determining
machine shop " Do not attempt to oil, C. face shields
clean, or any machine D. all of these "
A. dividers the correct sizes. Far QYiindrical work
B. calipers
h
while is running.
C. surface gage
the is used and where a light
variation over or under nominal size is
A. adjust" 38. For housekeeping safety "Do not
D. all of these* allowable a limit gage is used.
B. maintain leave tools or work on the table eta -
C. stop machine even if the machine is A. limit gage
- . Tools or work may fall off 44. Many related tools such_ as B. snap gage *
D. talk
and cause toe orfoot injury." straightedges, gee~._ ~s and C. flat gage
protractors are used in conjunction with D. angular gage
33. A general safety precaution in a A, running
B. in operation linear measuring tools to determine:
machine shop "Do not try to
,-----,---:---:-- the machine with your c: not running *
A. flatness
B. straightness
50. M.Lc(Qm~r calipers are made in a
variety of sizes rangmg from _ __
hands or body. D. none of these
C . angul~ A 1 into 60 in
A. stop*
39. In a shaper, after setting the stroke D. all df these " B. 1/2 in to 48 in *
B. dean
C. run length and position, check to see that C. 1 in to 50 in
adjusting nuts are tight. Stand 45. for round work, measurements are D. 1/2 in to 24 in
D. all of these
to the direction of stroke of mac
'"'h:-:i-n e-- usually made by contact, using tools
when it is running. with contact points or surfaces such as 51 . A very simple s~l an..d made
34. When working with another student,
A. vertical spring calipers, micrometers and of a weighted head and a handle which
only should operate
B. horizontal A. vernier calipers " directs its course.
machine or switches.
B. straight calipers A box
A. one " C. parallel*
D. in front C. flexible calipers B. hammer*
B. three
D. all of these C. pedal
C. two
D. four 40. In a shaper, never remove chips D. metal drivers
while ram is - 46. A useful measuring tool in the
A. not in motion machine shop which makes possible 52. Machinists' hammer are made of
35. Do not wear rings, watches,
bracelets, or other jewelry or B. not running accurate measurements against shallow steel, hardened and tempered. The 121?
shoulders. · -
clothing that could get caught in moving c. In motion " of the hammer head is called the
machinery.
A. neckties •
B. shirts
D. not in motion .A:llOoi<ed rule "
B. flexible rule
C. spring rule
D. all of these
A face, peen
B. peen, face *
-
_ _ _ and the bottom is called the

REFRESHER MANUAL 2nd Edition by JAS TORDILLO


REFRESHER MANUAL 2nd Edition by JAS TORDILLO
45 - 14 1 Day 45 -Solution Day 45 - Solution 145 - 15
A. torque wrench *
Machine Design
C. head, tail
D. upper, lower B. open wrench B. _spindle drill press C. holing
C. socket wrench C. sensitive drill press * 0. reaming •
53. One of the main functions of screw D. alien wrench D. gang drill
drivers is to: 73. The-operation of enlarging a ~ole by
A. to loosen screws only 60. Another type of wrench used in 67. Type of drill press useful when means of an adjustable cutting tool with
B. to tighten screws only machine work on which .sockets of drilling several holes in the~ only one cutting edge.
C. all of these different sizes are attached to the ,beavier pieces. A. expansion
D. to loosen and tighten screws * handle.lor various sizes of nuts. - A. sensitive B. boring •
A. torque wrench - B. gang C. holing
54. The three main parts of a screw B. socket wrench * C. radial" D. reaming
driver are: the handle, the shank, and C. alien wrench D. vertic~!
the _ _ D. adjustable wrench 74 . The operation of forming internal
A. blade* 68. Type of drill press which is a threads.
B. body 61. Another type of wrench used in ~oJJ~n oj on~ ~machiD~f _ the A. threading
C . tip machtne work especially .designed for esseQJiaL speed and feed-Units of from B. tapping •
D. Phillips screw screws with allen head. two to eight single drill presses mounted C. facing
A. allen screw on one base-. -~­ D. grinding.
55. A part of the screw driver, the end B. allen wrench * A.Sensitive
which fits into the slot of the screw is C. allen nut B. horizontal 75. The operation of making a
called _ _ __ D. special wrench C. gang* cone-shaped enlargement of the end of
A. blade* D. multiple a hole, as
B. shank 62. An instrument used to saw metaL for a recess for a- flathead screw.
C. body A. cutter 69. A part of the drill press which grve A. counterboring
D. Phillips B. hacksaw* a~able of machine on which the work B. countersinking *
C. flatter Is clamped or on which the ~ C. spot-facing
56. A part of the ~ driver, the steel D. bender ~dclamped. D. expanding
portion extending from the handle is the A. base~
63. The width of a hacksaw blade is: B. spindle l 76. The purj:lose of this part is fo ~
A. blade A. Y. in C. handwheel drive the drill,' since the hold of the taper
B. shank* v.
a·. in'* D. box work • ~De is not sufficient. -
C. body C. 3/4 in A. tang*
D. handle D..1 in 70. A part of the drill press which is the B. shank
12ari holding the_slrill chu~or socket. C. taper
57. The most common types of screw 64. The number of teeth per inch of a A. head claming D. chuck
drivers are standard, offset, ratchet and: ..hacksaw b.lades ranges from: B. box column
A. double-e~ ~- . - A. 14 to 32 * C. head adjusting box 77. A gripping device with two -or more
B. single-end B.16to28 D. spindle* adjustable jaws set raQJaUy.
C. Phillips" C. 14 to 24 A. drill chuck *
D. blade D. 16 to 2-4 71. The operation of producing a circular B. spindle
hole by removing solid metal. C. drift slot
58. An instrument for exerti.Qg a twisting 65. The length of a hacksaw blades A. reaming D. control shaft
§!cain, as in turning bolts and nuts. ranges from: B. boring
A. Phillips screw driver -- A. 8 to 16 in • C. spotting 78. Part of the drill bit that fits into the
B. wrench • B. 10 to 24 in D. drilling • boldiog device.
C. standard screw driver C. 8 to 24 in A. shank *
D. hammer D.10to30in 72. An operation of sizing and finishing B. Flute
a hole by means of a cutting tool having C. tang
59. A type of wrench used by machinists 66. Type of drill press .~,!g!d for drilling several cuffing edges. D. point
to determine how mugh twisting force ~mall holes with. hand feed. A. honing
~r~lying. A. radial drill press B. bonng

REFRESHER MANUAL 2nd Edition by JAS TORDILLO REFRESHER MANUAL 2nd Edition by JAS TORDILLO
"'11
~

45 -1 6 I Day 45- Solution Day 45 - Solution I 45 - 17


79.The~~es C. 12 to 15 degree* Machine Design
out the · ·s lant. D. 15 degree C. pressi ng
A. flute* 99. Part of the lathe machine, fQL.the
D. filing pur ose rim~ing an outer
B. lips 86 . ..bie an~ le of drills used in general
C. body work is: _earing_ a Sw:wort forwon<Oeing
93. Which of the following operation turned on centers.
D. point A. 12 degree
may also be done on the lathe?
B. 15 degree A. bed
A drilling B. tailstock *
80. The .b.a.Q_kQp.ne of the drjll bit and it is c. 59 degree * B. all of these * C. headstock
the narrow section -between the flutes. D. 78 degree
C. reaming D. live center
A. body
D. tapping
B. cone 87. An angle that forms the cutting
C. web * wedge: 100. This center which ~es not move
D. margin A. lip angle
94. The common types of lathes are witb the work. is called: ...
bench lathe, engine lathe and _ __ A. live center -
B. rake angle *
A. standard lathe B. dead center *
81. The most commonly used ..!vruLQf C. clearance angle
B. turret lathe* C. center bearing
.Qrill bit in the machine shop are: D. tan~ angle
C. engine' lathe D. center of attraction
A. straight drills
D. bore-lathe
B. short drills 88. The point angles of twist drills
C. twist drills * 9 nges from----· 95. A Jim all latbe, ~Y mouoted on a
D. long drills A. 40 to 60 degree
bench, and is used for small work.
B. 60 to 150 degree* A. benchJathe *
82. Type of drill bit Wied for drilling C. 60 to 118 degree
B. engine lathe
brass .~ metaLs. D. 50 to 70 degree C. t'urret lathe
A. cross-fluted drill
D. all of these
B. side-fluted dr'i.ll 89. The point angle of twist for drilling
C. straight-fluted drill * ~ris:
96. The machine tool in which .manY
D. curved-fluted drill A. 60 degree
B. 100 degree*
-~~ne. T.his lathe is,
83. The angle of clearance at the center C. 90 degree ~o~~~:~u::machin~ooJ io tll~op.
must be proportiQnal to the angte.."ai1Fie D. 118 degree
B. engine lathe *
outside. The clearance o n the drill is
C. turret lathe
about ___ degree at the cutting 90 The point angle of twist for drilling
D. all of these
edge. average class of work is: -
A. 15 A. 60 degree -
97. This lathe is a manufacturing
B 25 B. 118 degree*
~e . Considerable mechanical skill
C. 12 * C. 75 degree
is required to make and adjust the
D. 30 D. 150 degree
several cutting tools.
A. bench lathe
84. A correctly sharpened drill bit, the 91. A machine tool in ~t~ork
B. engine lathe
edge oftiie angle acr6Ss tfie w~b of the ]evolves and the tool usually moves
C. turret lathe *
d..!ill-will-b.a about ___ degree with _along a straightline.- ---
D. all of these
the line of the cutting edges. A. shaper
A. 24 B. lathe"
98. Part of the lathe machine, which
B.60 C. milling
c. 45 .. serves as the foundation on which the
D. drill press
lathe is built. -
D. 70
A. bed*
92. The general lathe operations are
B. base
85. The an le of clearance of a drill bit straight turning, taper turning, boring
C. chuck
used in_ G.u.lting_s and mediul]l and --,-,.,.--- D. carriage
A. 12 degree - A. welding
B. 7 to 12 degree B. cylindrical turning *

REFRESHER MANUAL 2nd Edition by JAS TORDILLO REFRESHER MANUAL 2nd Edition by·JAS TORDILLO
Day 46 EXAM I 46 1
w w

Machine Design
C. module
DAY 46- EXAM D. pitch diameter

1. Arc of the pitch circle through which a 7. Ideal herringbone gear helix in the
tooth travels from the point of contact range of ____.
with the mating tooth to the pitch point of A. 30 to 45
a gear. B. 25 to 30
A. arc of recess C. 35 to 50
B. arc of approach D. 15 to 20
C. arc of action
D. arc of relief 8. 'A machine member which is used to
connect shafts al'ld to disconnect them
2. An eyebolt is lifting a block weighing at will:
350 lbs. The eyebolt is of SAE C 1040 A. coupling
material with So =67 ksi and Sv =
55 B. cluteh
ksi, what is the stress area (in inches C. collar
square) of the bolt if it is under the D. universal joint
unified coarse series thread?
A. 0.1341 9. Poisson's ratio is the ratio of:
B. 0.0991 A. lateral stress to longitudinal stress
C. 0.1134 B: shear stress to shear strain
D. 0.1043 C. compressive stress to tensile stress
D. lateral strain to longitudinal strain
3. Molybdenum-chromium-nickel steel ·
designation: 10 . In a chain drive design for large
A. SAE 48xx speed reduction it is preferable to use a
B. SAE 56xx double redt.tetion of compound type of
C. SAE 43xx transmission instead of single two
D. SAE 46xx sprockets transmission . Drives should
be so designed that the angle between
4. What pressure. is required for two tight chain strand does not exceed
punching a hole 2 inches dia thru Y. inch _ ___ degrees.
steel plate? A. 45
A. 40 tons B. 55
B. 45 tons C. 60
C. 50 tons D. 90
D. 35 tons
11. The product of the mass and the
5. Bearing surface that completely linear velocity of a body:
surrounds the journal is also called A. linear impulse
B. linear momentum
A. offset bearing C. couple
B. centrally loaded bearing D. vector
C. full bearing
D. babbit bearing 12. The total weight of 2 steel plates
size "!." 3' x 20 ft is:
6. The ratio of the pitch diameter in A. 18361bs
inches to the number of teeth. B. 3667lbs
A. pitch circle C. 1698 lbs
B. English module D. 1848 lbs

REFRESHER MANUAL 2nd Edition by JAS TORDILLO


46- 2 1 Day 46- EXAM Day 46 - EXAM I 46 - 3
13. If the ultimate shear stress of a m long (E = 207 Gpa) to stretch the bar Machine Design
steel plate is 35.000 psi, what force is 1.3 mm?
necessary to punch a 1.5 inch diameter A. 52.840 C. 10 hp 29. Steels containing large amounts of
hole in a 1/8 inch thick plate? B. 60.000 D. 7.1 hp mild nickel and chromium:
A. 20,617 lbs C. 53.048 A. carbon steel
B. 41,234lbs D. 42.562 24. Type of bolt commonly used in the B. stainless steel *
C. 30,925 lbs construction that is threaded at both C. alloy steel
D. 61,850 lbs 19. A car is traveling at 90 kph when ends. D. cast steel
the brakes are applied. If it travels a A. stud bolt
14. A cylindrical tank has an inside B. acme thread bolts 30. Eccentrically loaded bearing is also
distance of 60 meters before coming to
diameter of 6 meters and is subject to stop, what is its acceleration in C. hex bolt called:
an internal pressure of 120 Kpa. m/sec/sec? D. square threaded bolts A., hydrostatic bearing
Determine the wall thickness of the tank A. -7.06 B. J'artial bearing·
if the allowable tangt!ntial stress is 40 B. 6.36 25. Find the torsibnal stress in MPa in a C. split bearing
MPa. c. -5.21 40 mm diameter shaft that transmits' D. offset bearing
A. 25.4 mm D. 5.76 20.53 kw at 600 rpm .
B. 12.7 mm A. 26 MPa 31. In' the selection of wire rope , Lang
C. 19 mm B. 46 MPa lay means the wires and strand s are
20. The most known lubricants being
D. 9mm utilized in whatever category of load and C. 36 MPa twisted:
speed are oil, air, grease and dry D. 56 MPa A. in same direction
15. Imaginary formed by the lubricants like: B. in any direction
intersection of the flanks of a thread A. bronze 26. is the plane perpendicular C. in opposite direction
when extended is called to the axial plane and to the pitch plane. D. in along direction
B. silicon
beyond tne root. C. lead In gears the P.arallel axes, and the plane
rotation coincide. 32. Continuous stretching under load
A. sharp crest D. graphite
A. tangent plane even if the stress is less than the yield
B. root apex
C. sharp root 21. Compute the nominal shear stress B. straight plane point:
D. all of these at the surface in MPa for a 50 mm C.' transverse A. plasticity
diameter shaft that is subjected to a D. reference plane B. creep
'
16. A hydrostatic bearing is one which: torque of 0.48 KN-m. C. elasticity
D.. ducti,lity
A. lube oil is supplied under pressure A. 16.95 27. A steel shaft 1.4375 inches in
B. there is no lube oil B. 21 .65 diameter transmits 40 hp at 1400 rpm.
Assuming a modulus of rigidity of 12 x 33. The product of the mss and the
C. lube oil is not pressurized C. 19.56
D. bearing is slightly loaded D. 25.12 106 psi, find the torsional deflection of linear 'Velocity of a body:
the shaft in degrees per foot length. A. angular impulse'·
A. 0.246 B. angular momentum
17. A lines haft runs at 450 rpm. A 40 22. An idler of 24 teeth is introduced
em pull ey on the same shaft is belt between the 12tooth pinion with a B. 0.624 C. linear impulse
connected to a 18 em pulley on the turning speed of 800 rpm driving a final C. 0.426 D. linear momentum
countershaft. From a 25 em pulley on 60 tooth gear. What would be the final D. 1.246
the countershaft, motion is transmitted 3.4. Opposite directional forces equal in
aped of the driven gear and its direction
28. The length of the hub should not be · magnitude and parallel is called
to the machine. Compute the required relative to the driving gear rotation.
diameter of the pulley on the machine to A. 400 rpm and opposite direction made lesser than the face width of the
a spindle speed of 500 rpm. B. 160 rpm and opposite direction gear. Hub lengths usually vary from A. non coplanar
'- B. couple
A. 50 em C. 400 rpm and same direction 1.25D to 2D, where D is th·e shaft
B. 40 em D. 160 rpm and same direction diameter. The reasonable diameter of C. centro
C. 65 em steel hub should be: D. coplanar
D. 35 em 23. What horsepower can a 1-inch A. 2D
diameter short shaft transmit at 380 B. 1.55D 35. The SAE 2340 shafting contains
____%nickel.
18. What load in Newtons must be rpm? C. 1.8D
applied to a 25 mm round steel bar 2.5 A. 3 hp D. 1.20D A. 3.85% Ni
B. 18 hp B. 3.25 to 3.75% Ni

REFRESHER MANUAL 2nd Edition by JAS TORDILLO


REFRESHER MANUAL 2nd Edition by JAS TORDILLO
46 - 4 I Day 46 - EXAM Day 46 - Solution 146 - 5
Machine Design
C. 4% C. partial bearing
D. 3.84% Ni D. fitted bearing 48. A flywheel weighing 457 kg has a
radius of 375 mm. How much energy in
DAY 46- SOLUTION
36. is a unit of energy. 42. A 76.2 mm diameter shafting of N-m does the flywheel loss from 3.0
A. ft-lb SAE 1040 grade, cold rolled, having a rev/sec to 2.8 rev/sec? 1. Arc of the pitch circle through which a
B. Btu/hr yield point of 50 ksi and with a 3/ . x Y. x 5 A. 368 N-m tooth travels from the point of contact
C. hp inches key. Compute the minimum yield B. 150 N-m with the _ITlating tooth to the pitch point of
D. watt point in the key in order to transmit the C. 1472 N-m a gear.
torque of the shaft. The factor of safety D. 38 N-m A. arc of recess
37. A spur pinion rotates at 600 rpm to use is 2 and Sys:::: 0.5Sy. B. arc of approach *
and transmits 25 kw to a mating gear. A. 39.120 ksi 49. The lack of backlash is a spur gear C. arc of action
The pitch diameter of the pinion is 100 B. 279.20 ksi design/operation may result to: D. arc of relief
mm, and the pressure angle is 20°. C. 42.130 ksi A. overloading
Determine the tangential load in D. 47.120 ksi B. all of these 2. l',n eyebolt is lifting a block weighing
Newton. C. jamming 350 lbs. The eyebolt is of SAE C 1040
A. 7960 43. Which is not a heat treatment D. overheating material. with Su :::: 67 ksi and Sv :::: 55
B. 6790 process? k§>i, what is the stre~s area (in inches
C. 3980 A. sintering 50. The strength of non-ferrous alloys is square) of the bolt if it is under the
D. 3098 B. hardening at maximum at room temperature while unified coarse series thread?
C. annealing that of ferrous metal has a maximum A. 0.1341
38. The moment of inertia of a rectangle D. tempering strength at °F. B. 0.0991
whose base is "b" and height "h" about A 400 c. 0.1134 *
its base is: 44. Instrument used to analyze the B. 450. D. 0.1043
A bh3/4 composition of metals. C. 1200
B. bh3/24 A. spectrometer D. 350 Fe :::: SyA~''
C. bh3 /12 B. radiometer 6
D. bh3/3 C. profilometer
D. ultimate analyzer (d ) 350
:;:: 55,000 As
.,.
39. The function of the clutch in the 6
machine tool is: 45. Creep is the term used in referring DAY 4S- ANSWER KEY 2
1. 8 16. A 31 . A46.0 As.:::: 0.1,1338in
A. lowering the drive speed to the: 2. C 17.A 32. 847. B
B. to disconnect or connect the shaft at A. elongation before yield point 3. C 18. A 33. 0 48. C 3. Molybdenum-chromium-nickel steel
will the drive B. maximum stress if brittle materials 4.A 19.C 34.849.8
designation:
C. alignment of drive shaft C. fatique limit of ductile materials 5. C 20. 0 35. 8 50. A
D. to ensure that two shafts line up at D. continuous increase in strain, or 6.8 21.C 36.A A SAE 48xx
high speed deformation, of any material subjected
7.A 22.0 37.A B. SAE 56xx
8. 8 23. C 38. 0 C. SAE 43xx *
to stress 9. 0 24. A 39. B D. SAE 46xx
40. Should two equal and opposite 10.A 25. A 40 . 8
collinear forces is added to the forces 46. Splines are used when: 11.8 26. C 41. B
12.8 27. A 42. 0 4. What pressure Is required for
already in equilibrium, which of (he A. the power to be transmitted is high 13.A 28. C 43. A punching a hole 2 inches dla thru Y. inch
following statement is true? B. the torque is high 14.0 29. 8 44. A steel plate?
A. unbalanced moment is maintained C. the speed is high 15.C 30. 0 45. 0 A. 40 tons*
B. equilibrium is maintained D. axial relative motion between shaft
C. equilibrium is distributed and hub is necessary G B. 45 tons
C. 50 tons
D. it creates an unbalanced situation
D. 35 tons
47. Which of the following contains
41. Also called eccentrically lad bearing chromium?
is --,-.,..,..,...--,-·
A. full bearings
B. offset bearing
A SAE 2340
B. SAE 5240
C. SAE4230
:;:: (2) m
Pressure or Force :::: d x t x 80

:::: 40 tons
(80)

D. SAE 1230

REFRESHER MANUAL 2nd Edition by JAS TORDILLO REFRESHER MANUAL 2"d Edili~n by JAS TORDILLO
46 - 6 1 Day 46 - Solution
Day 46 -.Solution 146-7
5. Bearing surface that completely A. linear impulse Machine Design
surrounds the journal is also called B . linear momentum *
C. couple 15. _lm?ginaiY. form~ t~.L.J~ 18. What load in Newtons must be
A. offset bearing D. vector intersection of the flan ks of a thtead applied to a 25 mm round steel bar 2.5
B. centrally loaded bearing whem extended is - called m long (E = 207 Gpa) to stret ch the bar
C. full bearing * 12. The total weight of 2 steel plates beyond the root. 1.3 mm?
D. babbit bearing size o/." 3' x 20 ft is: A. sharp crest A. 52.840 *
A. 18361bs B. root apex B. 60.000
6. The ratio oLthe__E!itch di~meter in B. 36671bs * C. sharp root* c. 53.048
inches to the number of teeth. C. 1698lbs D. all of these D. 42.562
A. pitch circle D. 18481bs
B. English module * 16. A hydrQ_s.!§ltic be<Jring_is one which : FL
C. module Density of steel plate = 0.283 lb/in 2 A. lube oil is supplied under y =: AE F(2500)
D. pitch diamet,e r pressure* 1.3 =: !!(0.025)2)( 207 X 109 ) ..._
Weight = volume x density B. there is no lube oil •
7 . Ideal herringbone gear helix in the = (3/4)(3 X 12)(20 X 12)(0.283) C. lube oil is not pressurized F = 52,837.66 N
range of__ - x 2 plates D. bearing is slightly loaded
A. 30 to 45 * = 3667.68 lbs 19. A car is traveJin·g at 90 kph when
B. 25 to 30 17. A lineshaft runs at 450 rpm. A 40 the brakes are applied. If it travels a
C. 35 to 50 13. If the ultimate shear stress of a em pu lley on the same shaft is belt distance of 60 meters before coming to
D. 15 to 20 steel plate is 35.000 psi, what force is connected to a 18 em pulley on the stop, what is its acceleration in
necessary to punch a 1.5 inch diameter countershaft. From a 25 em pulley on m/sec/sec?
8. A machine member which is used to hole in a 1/8 inch thick plate? the countershaft, motion is tra nsmitted A. -7.06
~ect shafts and to disconnect them A. 20,617 lbs * · to the .machine. Compute the required B. 6.36
.a.Lwill: B. 41,234 lbs diameter of the pulley on the machine to c. -5.21 *
A. coupling C. 30,925 lbs a spindle speed of 500 rpm. D. 5.76
B. clutch* D. 61,850 lbs A. 50 em*
C. collar B. 40 em V, = 90 kph = 25 m/s
D. universal joint F = (1Tdt) Su C. q5 em v2 = o
= (1T)(1.5)G) (35,000) D. 35 em
9. Poisson's ratio is the ratio of: vl = V12 + 2aS
2
A. lateral stress to longitudinal stress
= 20,616.70 lbs
0 = (25) + 2a(60)

o,~,s?
B. shear stress to shear strain a = - 5.21 m/s
2
C. compressive stress to tensile stress 14. A cylindrical tank has an inside
diameter of 6 meters and is subject to
D. lateral strain to longitudinal strain* 20. The most known lubricants beio.g
an internal pressure of 120 Kpa.
Determine the wall thickness of the tank utilized in whatever cat~IY...Q!.!Qad and
10. \D. i! chain drive design for large 8 speed are oil, air, grease and dry
if the allowable tangential stress is 40
speed reduction it is preferable to use a iubricants like:
MPa.
double reduction of compound type of
A. 25.4 mm A. bronze
transmission instead of single two
sprockets transmission . Drives should
be so designed that the angle between
B. 12.7 mm
C. 19 mm
D. 9mm *
8 ~c
MACHINE B. sil icon
C. lead
D. graphite *
two tight chain strand does not exceed
-:--:-::-:-- degrees. D,NA = D2NB 21. Compute the nominal shear stress
A. 45. St = PD
Zt 40(450) = 18 Ns at the surface in MPa for a 50 mm
B. 55 Ns = 1000 rpm diameter shaft that is subj'ected to a
C. 60 40,000,000 = 120,000(6) torque of 0.48 KN-m.
D. 90 Zt
D3Ns = D4Nc A. 16.95
t = 0.009 m = 9 mm 25(1000) = D4(500) B. 21.65
11. The product of the mass and the D4 = 50 em c. 19.56 *
_linear velocity of a body:· D. 25.12

REFRESHER MANUAL 2nd Edition by JAS TOROILLO


REFRESHER MANUAL 2 nd Editron by JAS TOROILLO
46 - 8 1 Day 46 - Solution Day 46 - Solution 1 46 - 9
s - 167" - 16(480) -Machine Design
s - rt0 3 - rr(0.050) 3 T = 326,745 N-m
30. !;_ccentrically loaded bearing is also 37. A spur pinion rotates at 600 rpm
Ss = 19,556,959.41 Pa = 19.56 MPa
s - 167" - 16(326.75)
called: and transmits 25 kw to a mating gear.
s - ;;[)3 - rr(D.040) 3
A - hydrostatic bearing The pitch diameter of the pinion is 100
= 26,001,548 Pa = 26 MPa
22. An idler of 24 teeth is introduced B. partial bearing mm, and the pressure angle is 20°.
between the 12tooth pinion with a C. split bearing Determine the tangential load in
26. is the_glane perpendicular
turning speed of 800 rpm driving a final D. offset bearing • Newton. ·
to the axiC!!_plane and to the Jlllch plan~
60 tooth gear. What would be the final A. 7960 *
l!l.g~h~rallel axes. aiJ.d the plane B. 6790
aped of the driven gear and its direction rotation coincjde 31. In the selection of wire rope, Lang
relative to the driving gear rotation. .!.9.Y_!!leans the wires and strand s are C. 3980
A. tangent plane
A. 400 rpm and opposite direction B. straight plane twisted : D. 3098
B. 160 rpm and opposite direction C. transverse * A. in same direction •
C. 400 rpm and same direction B. in any direction' P = 2rrTN
D. reference plane
D. 160 rpm and same direction • C . in opposite direction

D1N1 = D2N2
27. A steel shaft 1.4375 inches in D. in along direction 2s.oo9 = 2rrTe6 °:)
diameter transmits 40 hp at 1400 rpm. T = 397.88 N-m
12(800) = 60N2 Assuming a modulus of rigidity of 12 x 32. Continuous stretching under load
N2 = 160 rpm and same direction 6 even if the stress is less than the yield
10 psi, find the torsional deflection of T = F1 x~
the shaft in degrees per foot length. .12.Q.int: 2
23. What horsepower can a 1-inch A. 0.246 * A. plasticity 397 •88 = F, X O.lOD
2
diameter short shaft transmit at 380 B. 0.624 B. creep* f, ::;: 7957.75 N
rpm? C. 0.426 C. elasticity
A. 3 hp D. 1.246 D. ductility 38. The moment of inertia of a rectangle
B. 18 hp whose base is "b" and height "h" about
c. 10 hp. P = 2rrTN 33. The product of the mss and the its base is:
D. 7.1 hp 40(33,000) = 2rrT(1400) linear velocity of a body: A. bh3 /4
T = 150 ft-lb ::;: 1800 in-lb A. angular impulse • B. bh3 /24
3
p : D N B. angular momentum 3
C. bh /12
38 C. linear impulse
= 10 hp 8 = !.!: D. bh3/3 *
p = . _ 3_8_
(1)'(380) ]G D. linear momentum •
8 = (1800)(12) X 180
39. The function of the clutch in the
~(1.4375) 4 (12 X 106 ) rr
34. Opposite directional forces equal in
24. Type of bolt commonly used in the e = 0.246° machine tool is:
construction that is threaded at both magnitude and parallel is called A-:iowering the drive speed
ends. B. to disconnect or connect the shaft
28. The length of the hub should not be A. non coplanar
A. stud bolt • at will the drive *
made lesser than the face width of the B. couple •
B. acme thread bolts C. alignment of drive shaft
gear. Hub lengths usually vary from C. centro
C. hex bolt D. to ensure that two shafts line up at
1.25D to 2D, where D is the shaft D. coplanar
D. square threaded bolts high speed
diameter. The reasonable diameter of
steel hub should be: 35. The SAE 2340 shafting contains
25. Find the torsional stress in MPa in a 40. Should two equal and opposite
A. 2D _ _ _ %nickel.
40 mm diameter shaft that transmits B. 1.550 collinear forces is added to the forces
20.53 kw at 600 rpm. A. 3.85% Ni already in equilibrium, which of the
A. 26 MPa *
c. 1.8D * B. 3.25 to 3. 75% Ni * following statement is true?
D. 1.20D c. 4%
B. 46 MPa A. unbalanced moment is maintained
C. 36 MPa D. 3.84% Ni B. equilibrium is maintained *
29. Steels containing large amounts of
D. 56 MPa C. equilibrium is distributed
mild nickel and chromium: 36. is a unit of energy.
A. carbon steel D. it creates an unbalanced situation
P = 2rrTN B. stainless steel *
A. ft-lb *
B. Btu/hr 41 . Also called eccentrically lad bearing
C. alloy steel is _ _ __
20 ' 530 = 2rrT(~) C. hp
60 D. cast steel D. watt A. full bearings
REFRESHER MANUAL 2nd Edition by JAS TORDILLO REFRESHER MANUAL 2nd Edition by JAS TORDILLO
46 - 10 1 Day 46 - Solution Day 47 - EXAM I 47 - 1
Machine Design
B. offset bearing * D. continuous increase in strain, or
C. partial bearing deformation, of any material C. 10
D. fitted bearing subjected to stress * DAY 47- EXAM D. 32.2

42. A 76.2 mm diameter shafting of 46. ,SJilin~ are used when: 1. A punch punches a 1-in diameter hole 6. A short cylindrical cast-iron post
SAE 1040 grade. cold rolled, having a A. the power to be transmitted is high in a steel plate o/. inch thick every 10 supports a compressive load of 20 tons
yield point of SO ksi and with a % x o/. x 5 B. the torque is high sec. The actual punching takes 1 sec. (40 kips). If the factor of safety is taken
inches key. Compute the minimum yield C. the speed is high The ultimate shear strength of ti"'e plate equal to 10, find the diameter of the
point in the key in order to transmit the D. axial relative motion between is 60,000 psi. The ftywheel of the punch post. Ultimate stress for compression is
torque of the shaft. The factor of safety shaft and hub is necessary • press has a onass moment of inertia of 80,000 psi. Neglect slenderness ration
to use is 2 and Sys = 0.5Sy. 500 in-lb sec2 and rotates at a mean and no buckling.
A. 39.120 ksi 47. Which of the following contains speed of 150 rpm. What is the A. 2~25 in
B. 279.20 ksi chromium? horsepower req~ired for the punch B. 3.25 in
C. 42.130 ksi A. SAE 2340 operation?
A. 7.04 hp
c: 2.52 in
D. 47.120 ksi • B. SAE 5240 * D. 3.52 in
C. SAE 4230 B. 9.04 hp
Shaft diameter = 76.2 mm = 3 in D. SAE 1230 C. 8.04 hp 7. A punch and die is set to punch 1 /4-
D. 10.04hp in hole in a 3/8-in thick plate. If the
Design Stress, Ss = 0.5Sy = 0.50(50) 48. A flywheel weighing 457 kg has a ultimate shearing stress or strength of
= 25 ksi = 25,000 psi radius of 375 mm. How much energy in 2. In problem No. 1, what is the total the plate is 50,000 psi, what force will be
N-m does the flywheel loss from 3.0 speed fluctuation of the flywheel in rpm? r~quired?
Ss = l6T rev/sec to 2.8 rev/sec? A. 50 rpm A.40001b
rro•
A. 368 N-m B. 65 ·rpm B. 42,000 lb
25,000 :::; 6
l T
B. 150 N-m C. 60 rpm C. 9000 lb
7r(J)' C. 1472 N-m * D. 70 rpm D. 44,200 lb
D. 38 N-m
T = 132,535.94 in-lb 3. In problem No. 1, what is the 8. In testing a specimen of a material for
V1 = 2rrRN1 = 2rr(0.375)(3) = 7.068 m/s coefficient of fluctuation of the flywheel? elongation, a unit deformation of 0.0005
_
F -
r _ t32.s3s.9~
D/2- 3/2
= 88,357.29 lbs Vz =2rrRNz =2rr(0.375)(2.8)=6.597 m/s A. 0.25 in was obtained with a load producing a
B. 0.50 unit ten~ile stress of 15,000 psi. Find the
E = % m[Vl- Vl] C. 0.43 modulus of elasticity in tension.
43. Which is not a heat treatment E = % (457)[(7.068)2 - (6.597)2] D. 0.76 A. 10 X 106 psi
process? E = 1470 N-m B. 30 x 106 psi
A. sintering * 4. The valve push rod for an overhead 6
C. 20 x 10 psi
B. hardening 49. The lack of b<~cklash is a spur gear valve engine is Y. inch in diameter and D. 40 x 106 psi
C. annealing design/operation may result to: 14 inches long. Find the moment of
D. tempering A. overloading
4
inertia of the rod in inces . 9. How much torque, in inch-lb can be
4
B. all of these * A. 1.917 X 10' safely transmitted by a 1 7/16-inch
44. Instrument used to ana_!yze the C. jamming B. 1.917 X 10' 3 diameter shaft if safe stress is 7000 psi?
com.£2sition of metals. D. overheating C. 1.917 X 10'5 A . 4100
A. spectrometer * D. 1.917 X 10'6 B. 2100
B. radiometer
C. profilometer
50. The strength of non-ferrous alloys is c. 3100
at maximum at room temperature while 5. A test specimen is under tension. The D. 1100
D. ultimate analyzer that of ferrous metal has a maximum load is 20,000 lb, allowable stress is
strengtha t- °F. 10,000 psi, modulus of elasticity is 30 10. It is desired to check the design of a
45. Creep is the term used in referring A. 400 * million psi, and original length of 2-in medium steel shaft subjected to a
to the_: _
B. 450 specimen is 40 in. What is the required turning moment of 40,000 in-lb.
A. elongation before yield point c. 1200 cross section, in square inches, if the Determine the factor of safety used in
B. maximum stress if brittle materials D. 350 resulting elongation must npt be greater the design if ultimate stress is 50,00 psi.
C. fatique limit of ductile materials than 0.001 inch? A . 1.50
A. 2 B. 2.50
B. 26.6
REFRESHER MANUAL 2nd Edition by JAS TORDILLO REFRESHER MANUAL 2nd Edi'tion by JAS TORDILLO
47-21 Day47-EXAM Day 47 - EXAM I 47 - 3
C. 1.96
Machine Design
A. use 2-in shaft
D. 4.95 B. use 2 o/. -in shaft
B. 20,900 lb 24. In problem No. 23, determine the
C. use 2 Y:. -in shaft torsional shear stress of the spring.
C. 81 ,600 lb
11. A hydraulic turbine is a water-power D. use 3-in shaft D, 11,9951b A. 49,000 psi
plant is rated at 12,000 hp. The steel
B. 60,000 psi ·
vertical shaft connecting the turbine and 15. What horsepower is required to lift a 20. The cylinder head of a stream C. 51,000 psi
generator is 24-in diameter and rotates load of 5,000 lb by means of a cable
engine is held bY. 14 bolts. The diameter D. 66,000 psi
at 60 rpm. Calculate the maximum wrapped around the drum hoist, the
of the cylinder is 14-in and the stream
shearing stress developed in the shaft at drum being 40-in diameter and making
pressure is 125 psi. What size of bolts is 25. A coiled spring with 1 o/. -in outside
full load. 24 rpm? required if tensile stress is 3000 psi? diameter is required to work under load
A. 4,640 psi . A. 10 hp
A. use 1-in bolt nominal diameter oi 140 lb. Wire diameter used is 0.192
B. 3,240 psi · B. 29 hp in, 'spring is to have seven active coils,
C. 4,150 psi B. use o/. -in bolt nominal diameter .
C. 18 hp and the ends are to be closed and
D. 1,450 psi
C. use Y:. -in bold nominal diameter
D. 38 hp ground. Determine the unit deflection
D. use 1 Y:. -in bolt nominal diameter
when under the load. Assume modulus
12. An automobile weighing 3,000 lb is 16. A solid circular shaft is used to
·21. Calculate the thickness of the wall of of rigidity equal to 12 million psi and
slowed down uniformly from 40 to 10 . transmit 200 hp at 1000 rpm. What shaft
a tank cylinder containing oxygen at mean radius to be 0:779 in.
mph in a distance of 100 ft by a brake diameter is required if the allowable
2500· psi. Tank is 10-in inside diameter. A. 1 in
on the drive shaft. Rear wheels are 28- maximum shearing stress is 20,000 psi?
Allowable stress may be taken as B. 1.817 in
in diameter. Drive-shaft diameter is 1- A. 1.475 in
20,000 psi in walls. C. 1.225 in
in. Ratio of differential is 40.11. B. 1.750 in A. 0.250 in _D. 3.516 in
Calculate the extreme fiber stress in C. 1.250 in
drive shaft, neglecting tire, bearing, and
B. 0.625 in
D. 2.000 in 26. A solid steel machine shaft with a
gear losses. C. 0.425 in
D. 0.725 in safe shearing stress of 7000 psi
A. 15,000 psi 17. In problem No. 16, if a hollow shaft stransmits a torque of 10,500 in-lb. A
B. 25,000 psi is used having an inside diameter equal
22. What is the necessary thickness of square is used whose width is equal to
C. 21,900 psi to the diameter of the solid shaft
the cylinder wall of an internal- one-fourth the shaft diameter and whose
D. 29,400 psi determine what must be the outside
combustion-engine chamber when the length is equal to 1 Y:. times the shaft
diameter of this shaft if the angular twist diameter. Find key dimensions.
pressure in the cylinder is 400 psi? The
13. A circular bar of solid cast iron 60-in of the hollow and solid shafts is to be
cylinder diameter in the engine is 3.75 in A. Y:. in, and 3 in
long carries a solid circular head 60-in equal?
and maximum allowable stress may be B. o/. in and 2.5 in
diameter. The bar is subjected to a A. 1.475 in
taken as 2000 psi. C. Y:. in and 2/5 in
torsional moment of 60,000 in-lb which B. 1.755 in D. o/. in and 3 in
A.0.250 in
is supplied at one end. It is desired to C. 1.402 in
B. 0.500 in
keep the torsional deflection of the D. 1.250 in 27. In problem No. 26, determine the
C. 0.421 in
circular head below 1/32-inch when the
D. 0.625 in crushing stress of the key.
bar is transmitting power over its entire 18. A square bar of wrought irort 2-in on
A. 7,000 psi ·
length in order to prevent the chattering each side is raised to a temperature of
23. The load on a helical spring is 1600- .B. 14,000 psi
of the piece. What would be the 1OOdeg F above its normal. If held so
lo and the corresponding deflection is to C. 9,000 psi
diameter of the-bar, if the working stress that it cannot expand, what s~ress will be
be 4-in. Rigidity modulus is 11 million D. 17,000 psi
is taken as 3000 psi and the transverse induced in it? k = 6.8 x 10.q per deg F.
psi and the maximum intensity of safe
modulus of elasticity is 6 million psi? A. 15,400 psi 28. In problem No. 26, obtain the factor
torsional stress is 60,000 psi. Design the
A. 4.7 inch B. 18,400 psi of safety of the key in crushing, allowing
spring for the total number of turns if the
B. 8.8 inch C. 16,400 psi an ultimate shearing stress of 50,000 psi
wire is circular in cross section with a
C. 6.7 inch D. 20,400 psi and ultimate stress for compression of
diameter of 5/8 in and a center line
D. 9.5 inch
radius of 1 Y:. in. The spring is squared 60,000 psi.
19. In problem No. 18, what force must
and ground ends. A. 2.5
14. Calculate the diameter of a steel it to prevent expansion? Modulus of
A. 19 coils B. 7.4
shaft to transmit 2200 hp at 2000 rpm elasticity E may be taken as 30 million
8. 21 coils C. 3.4
with an extreme fiber stress of 15,0000 psi.
C. 20 coils D. 4.3
psi. A. 91,600 lb
D. 22 coils
REFRESHER MANUAL 2nd Edition by JAS TORDILLO REFRESHER MANUAL 2nd .Ed-ition by JAS TORDILLO.
47 - 6 I Day 47 - Solution Day 47 -Solution I 47 - 7
Machine Design
DAY 47 - SOLUTION
(d_ j Speed fluctuation = (w,- w2)
1. A punch punches a 1-in diameter hole = 0 _5626 rad x ~ x 60sec'
DAY 47- ANSWER KEY
1. C 16.A 31 . A46. C in a steel plate % inch thick every 10 sec 2nrad min •
2. 8 17. 8 32. C47. A sec. The actual punching takes 1· sec. = 64.5 rpm
3 C 18. D 33. 8 48. C The ultimate shear strength of the plate
4. A 19. C 34. C49. D 3. In problem . No. 1, what is the A = 26.67 in2
5. 8 20. A 35. C 50 C
is 60,000 psi. The flywheel of the punch
6. C 21. 8 36. 8 press has a mass moment of inertia of coefficient of fluctu_ation of the flywheel?
Therefore, the safest cross section area
7. 0 22. C 37. 0 500 in-lb sec2 and rotates at a mean A. 0.25
8. 8 23. 0 38. A B. 0.50 will be 26.67 in2 .
speed of 150 rpm. What is the
9. A 24. 0 39. A horsepower required for the punch c. 0.43 .
10.C 25. 8 40. A
operation? D. 0.76 6. A short cylindrical cast-iron post
11.A 2~. A 41. C suppdrts a compressive load of 20 tons
Cr = 001 -w 2 = ~= 0.433
12.0 27. 8 42. 0 A. 7.04 hp
13.8 28. 0 43. 0 B. 9.04 hp (40 kjps). If the factor of safety is taken
(1) 150 equal to 10, find the diameter of the
14,0 2~ 8 4~8 c. 8.04 hp .. post. 'u ltimate stress for compression is
15.0 30.0 45. 8 D. 10.04hp 4. The valve push rod for an overhead 80,000 psi. Neglect slenderness ration
~ valve engi ne is y. inch in diameter and and no buckling.
F = Su(1Tdt) 14' incnes long. Find the moment of A. 2.25 in
= 60,000(1T)((1)(%) = 141,371.671bs 4
inertia of the rod in inces • B. 3.25 in
A. 1.917 X 10-4" C. 2.52 in •
Energy = Yz (force x thickness) B. 1.917 X 10'
3
·D. 3.52 in
= Yz (141,371.67)(%) C. 1.917 X 10'5
= 53,014.37 in-lb = 4417.86 ft-lb D. 1.917 X 10'6 F
4 Sc =-
Power = energy 4417.86ft-lb hp
x-- I=~ A
time lsec 550 64 80,000 = 40,000
4
Power = 8.03 hp = _n(0.25) 10 .7t d2
1 4
64
2. In problem No. 1, what is the total 4 d = 2.52 in
I = 1.917 X 10' in
4
speed fluctuation of the flywheel in rpm?
A. 50 rpm 7. A punch and die is set to punch 1 /4-
5. A test specimen is under tension. The
B. 65 rpm* in hole in a 3/8-in thick plate. If the
load is 20,000 lb, allowable stress is
C. 60 rpm ultimate shearing' 'stress or strength of
10,000 'psi, modulus of elasticity is 30
D. 70 rpm million psi, and original length of the plate is 50,000 psi, what force will be
specimen is 40 in. What is the required required?
w = 150 rpm = 15.7 rad/sec cross section, in square inches, if the A. 4000 lb
B. 42,000 lb
Mean speed w = -00[ -+002
- resulting elongation must not be greater
C. 9000 lb
'· 2 than 0.001 inch?
A.2 D. 44,200 lb * ·
001 + 00 2
15.7 B. 26.6 *
2 F = Su(1Tdt)
w, + w2 = 31.41 rad/sec
c. 10
D. 32.2 = 50,000(1T) (%)3/8)
= 44,178.651bs
KE = Yz I (wl-w/)
KE = Yz I (w, - w2)( w, + w2) St = £. 8. In testing a specimen of a material for
53,014.37 = Yz(500)(w,- w2)(31.41) A elongation, a unit deformation of 0.0005
10,000 = 20.000 in was obtained with a load producing a
A unit tensile stress of 15,000 psi. Find the
2 modulus of elasticity in tension.
A = 2 in
A . 10 X 106 psi
REFRESHER MANUAL 2nd Edition by JAS TORDILLO REFRESHER MANUAL 2nd ~tqition by JAS TORDJLLO
47 - 8 1 Day 47 - Solution Day 47- Solution 147-9
B. 30 x 1o• psi * Machine Design
shearing stress developed in the shaft -at
C. 20 x 106 psi full load.
D. 40 x 106 psi A. 4,640 psi " 21.000 40 0.00104 = 60,000(60)

Modulus of Elasticity, E = stress


B. 3,240. psi Td II 4 o6)
31t2 d (6xl
strain c: 4,150psi Td = 5775 in-lb
D. 1,450 psi d=8.75in
= 15,000 16Td
s. (driveshaft) = nd 3
0.0005 P = 2rrTN Therefore, the safest diameter of the bar
= 30 x 106 psi 12,000(33,000) = 2rrT(60) is 8.8 in.
16(5775) = 2 9,411 .83 psi
T = 1,050,422.62 ft-lb = n(l )3
9. How much torque, in inch-lb can be = 1,050,422.62(12) in-lb 14. Calculate the diameter of a steel
safely transmitted by a 1, 7/16-inch sl'laft to transmit 2200 hp at 2000 rpm
13. A circular bar of solid cast iron 60-in
diameter shaft if safe stress is 7000 psi? s~ = 16[1,050,422.62(12)]
long carries a s~lid circular head 60-in
with a..n extreme fiber stress of 15,0000
A. 4100 * psi.
B.2100 n(24)
3
diameter. The bar is subjected to a A. use 2-in shaft
c. 3100 = 4643.90 psi torsional moment of 60,000 in-lb which B. us~ 2 '!.-in shaft
D. 1100 is supplied at one end. It is desired to C. use 2 .Y. -in shaft
12. An automobile weighing 3,000 lb is keep the torsional deflection of the D. use 3-in shaft •
slowed down uniformly· from 40 to 10 circular head below 1/32-inch when the
s. = 16T
mph in a distance of 100 ft by a brake bar is transmitting power over its entire
nD 3 P = 2rrTN
on the drive shaft. Rear wheels are 28- length in order to prevent the chattering 2200(33,000) = 2rrT(2000)
16 in diameter. Drive-shaft diameter is 1- of the piece. What would be the T = 5777.32 fl-lb = 69,327.9 in-lb
7000 = T
in. Ratio of differential is 40.11. diameter of the bar, if the working stress
n(l.4375)3 Ss = 16T
Calculate the extreme fiber stress in is taken as 3000 psi and the transverse
T = 4082.73 in-lb modulus of elas.ticity is 6 million psi? 7td3
drive shaft, neglecting tire, bearing, and
gear losses. A. 4.7 inch ' 15,000 = 16(69,327.9)
10. It is desired to check the design of a A. 15,000 psi B. 8.8 inch*
2-in medium steel shaft subjected to a C. 6.7 inch nd
B. 25,000 psi
turning moment of 40,000 in-lb. D. 9.5 inch d = 2.86 in use 3 in shaft
C. 21,900 psi
Determine the factor of safety used in
D. 29,400 psi *
the design if ultimate stress is 50,00 psi. Solving for diameter using shear stress 15. What horsepower is required to lift a
A. 1.50 load of 5,000 lb by means of a cable
B. 2.50
v1 = 40 mph = 58.67 ftisec formula:
wrapped around the drum hoist, the
c. 1.96 * v2 = 10 mph = 14.67 ft/sec 16T drum being 40-in diameter and making
Ss= - -
D. 4.95 nD3 24 rpm?
Vl = Vl
2
+2aS
A. 10 hp
(14.67) .= (58.67)2 + 2a(100) = I 6(60.000)
3000 B. 29 hp
S, = 16T a = -16.1 ft/s2 7td3
nDJ C. 18 hp
d = 4.67 in D. 38 hp •
Su _ 16T F = Wa = 3000(16.1) = 1500 lb
FS - nDJ g 32.2 Solving for diameter using torsional Power = force x velocity
deflection:
50.000- 16(40,000) Torque(Differential shaft) = Fx
0
S = rO = 5000 X 1t
40 (24)ft - (b X~
----ps- - n( 2}3 2 12 min 33,000

FS = 1.96 28 .I_= (60) 0 =38 hp


= 1500 x
2 = 21 000 in-lb
0
32 2
11. A hydraulic turbine is a water-power Power(driveshaft) = Power(differential) e = 0.00104 rad 16. A solid circular shaft is used to
transmit 200 hp at 1000 rpm . What shaft
plant is rated at 12,000 hp. The steel 2rrTdNd = 21TToNo e = TL diameter is required if the allowable
vertical shaft connecting the turbine and Nd = To= 40 JG maximum shearing stress is 20,000 psi?
generator Is 24-in diameter and rotates
at 60 rpm. Calculate the maximum
N0 Td 11 A.1.475 in*
B. 1.750 in

REFRt:SHER MANUAL 2nd Edition by JAS TORDILLO REFRESHER MANUAL 2nd Edition by JAS TORDILLO
47- 10 I Day 47- Solution Day 47 -Solution 1 47 - 11
Machine Design
C. 1.250 in 19. In problem No. 18, what force must
D. 2.000 in it to prevent expansion? Modulus of Total no. of coils (squared & ground)
20.000 = 2500(10)
elasticity E may be taken as 30 million = n+2
P = 2nTN 2l
psi. = 19.42 + 2
200(33,000);:;: 2TTT(1000) A. 91,600 lb t = 0.625 in = 21.42 say 22 coils
T = 1050.42ft-lb;:;: 12,605.07 in-lb B. 20,900 lb
c. 81,600 lb * 22. What is the necessary thickness of 24. In problem No. 23, determine the
Ss = 16T the cylinder wall of an internal-
n:dJ D. 11,9951b torsional shear stress of the spring.
combustion-engine chamber when the A. 49,000 psi
20 000 = 16(12.605.07) F = StxA pressure in the cyiinder is 400 psi? The B. 60,000 psi
.. n:DJ = 20,400 (2x2) cylinder diameter in the engine is 3.75 in C. 51,000 psi
= 81,600 lbs and maximum allowable stress may be D. ~6,000 psi *
D = 1.475in taken as 2000 p~i.
A.0.250 in
17. In problem No. 16, if a hollow shaft 20. The cylinder head of a stream
B. 0.500 in K= 4C-l + 0.615 = 4t 8~ - 1 + 0.615
is used having an inside diameter equal engine is held by 14 bolts. The diameter
C. 0.421 in* 4C-4 C 44:8-4 4.8
to the diameter of the solid shaft of the cylinder is 14-in and the stream '
pressure is 125 psi. What size of bolts is D. 0.625 in = 1.32
determine what must be the outside
required if tensile stress is 3000 psi?
diameter of this shaft if the angular twist
of the hollow and solid shafts is to be A. use 1-in bolt nominal diameter* t= Q ( {&-;P -
2 f&=P 1) ss = 8FkDm s(16ooX1.32X3)
B. use % -in bolt nominal diameter
equal?
C. use ~ -in bold nominal diameter n:d l

A. 1.475 in
B. 1.755 in* D. use 1 ~ -in bolt nominal diameter t = 3.75 ( 2ooo + 4oo _ 11 n:UJ
C. 1.402 in .2 2000 - 400 ) 66,086.87 psi
D. 1.250 in F= PA
t = 0.421 in
;:;: 125 ~(14)2 25. A coiled spring with 1 :Y. -in outside
Di = D = 1.475 in 4 23. The load on a helical spring is 1600- diameter is required to work under load
e (hollow) = e (solid) = 19,242.25 lbs lb and the corresponding deflection is to of 140 lb. Wire diameter used is 0.192
n n '!Ioree pe; bolt, Fb = 19,242.25
be 4-in. Rigidity modulus is 11 million in, spring is to have seven active coils,
psi and the maximum intensity of safe and the ends are to be closed and
7r [o4 - D4 L"" 7r D4G -··. 14
torsional stress is 60,000 psi. Design the ground. Determine the unit deflection
32 O I .P 32 = 1374.45 lbs when under the load. Assume modulus
4 spring for the total number of turns if the
Do - D14 = D4 but Di = D Fb of rigidity equal to 12 million psi and
4 4 St=- wire is circu lar in cross section with a
Do = 2 D . A mean radius to be 0.779 in.
diameter of 5/8 in and a center line
Do4 ,= 2(1.475) 4
radius of 1 Ya in. The spring is squared A. 1 in
Do = 1.7541n 3000 = 1374.45
and ground ends. 8.1 .817 in*
1t d2 A. 19 coils C. 1.225 in
18. A square bar of wrought iron 2-in on 4
B. 21 coils D. 3.516 in
each side is raised to a temperature of d = 0.764 in say 1 in bolt diameter C. 20 coils
1OOdeg F above its normal. If held so
D. 22 coils* Dm 1,75-0.192=8.11
that it cannot expand, what stress will be 21. Calculate the thickness of the wall of
induced in it? k = 6.8 x 10'6 per deg F. a tank cylinder containing oxygen at
c= - d- 0.192
A. 15,400 psi 2500 psi. Tank is 10-in inside diameter. c = ~-= 2(1.5) = 4.8 8FC 3 ~ 8(140X8.11)3 (7) = 1.815 in
B. 18,400 psi Allowable stress may be taken as d 5/8
C. 16,400 psi y = ---c3d = 12x!06 (0.192)
20,000 psi in walls. RFC n
1
D. 20,400 psi * A. 0.250 in Y= --
B. 0.625 in ·* Gd 26. A solid steel machine shaft with a
S= k~T C. 0.425 in 3 safe shearing stress of 7000 psi
= 8(1600X4.8) n
= (6.8 X 10.0)(30 X 106)(100) D. 0.725 in 4 stransmits a torque of 10,500 in-lb. A
= 20,400 psi
PO
llxi0
6
U) square is used whose width is equal to
one-fourth the shaft diameter and whose
St = - n = 19.42 coils
2t
REFRESHER MANUAL 2nd Edition by JAS TORDILLO REFRESHER MANUAL 2nd tqition by JAS TORDILLO
47- 12 I Day 47- Solution Day 47- Solution 147 -13
Machine Design
length is equal to 1 Y2 times the shaft of belt width. Belt thickness is 0.2 in. .~

diameter. Find key dimensions. Calculate the width of belt. 37. In problem No. 36, determine the
PL = FV X f
A. % in and 3 in * A. use 3-in belt required shaft center-to-center distance
B. :Y. in and 2.5 in B. use 5-in belt* =20,000 [7r(~)l800]0.002jr
12
- /b x.J.p__ •
min 33,000 -.
if diameter pitch is taken as 5.
C. Y2 in and 215 in C. use 4-in belt A. 3.1 in
D. :Y. in and 3 in D. use 6-in belt PL = 4.56 hp B. 5.1 in
C. 4.1 in
16T P = 2rrTN 32. Which of the following pressure D. 7.1in*
Ss = - 10(33,000) = 2rrT(1700)
nD 3 angles is most commonly used for
T =
30.894 ft-lb =
370.73 in-lb involute gears? C = ' 7j+T2 = 15+56 = 7.1 in
7 000 = 16(10,500) 0 A. 20 deg
T = (F1- F2) - 2Pu 2(5)
.' n0 3 2 B. 25 deg
D = 1.97 in, say 2 in diameter shaft C. 14.5 deg *
8 38. Design a pair of spur gears for a
._ D 2_ . 370.73= (F1-F2) ( ) D. 18 deg
b - -= - - 11 nln 2 motor .drive as follows: motor delivers 50
4 4 F1- F2 = 92.684 lbs 33. What is the form of pitch surface in hp at ~ ,200 rpm; gear reduction, 6 to 1.
L =1.5D =1.5(2) =3 in bevel gears?
· Assume pinion pitch diameter to be 5-in;
Width = ~ (92.684) lb A. cylindrical
number of teeth, 15;· diametral pitch, 3.
27. In problem No. 26, determine the 20 lb Indicate the pitch diameter, number of
B. conical*
crushing stress of the key. = 4.63 in (use 5-in belt) teeth and speed of the mating spur
C. tubeaxial
A. 7,000 psi gear.
D. involute
B. 14,000 psi * 30. A journal bearing with a diameter of A. 30-in, 90-teeth, 200 rpm *
C. 9,000 psi 2.25-in is subjected to a load of 1000 lb B. 90-in, 30 teeth, 200 rpm
34. is rrg,d e of a line of ac.!jpn
D. 17,000 psi while rotating at 200 rpm. If the C. 20-in, 80-teeth, 150 rpm
of the tooth with a line tangent to both
D. 90-in, 30-teeth, 159 rpm
coefficient of friction is taken as 0.02 th~res where Jhese ~orne
0 500 and UD is 3.0, find the total work of
Ft = _2_ = I · =10' 500 lbs . rogeib.er.
0 2 friction in ft-lb per min. A . line of action .!!J_=~
2 2 A. 15.19 B. pressure line N2 I
h = b = 0.5 in for square key B. 1958 C. pressure angle * D1N1 = D2N2
- -F;- -
- I-
0.000 - 14 000 psi. c. 6.750 D. angle of act:(ln (5)(6) = D2 (1)
S c- --
D. 2356 *
h(L) 0.5(J) ' D2 = 30'-in
2 2 35. Which of the following _shQJ.Jid be
W = FVxf properly related in a pair of spur gearsin · T1N1 = T2N2
5
10oo[ne;~ }oo]xo.o2
28. In problem No. 26, obtain the factor order_tog.teVent interference? - - (15)(6) =T2(1)
of safety of the key in crushing, al lowing
w = A. addendum T 2 = 90-teeth
an ultimate shearing stress of 50,000 psi B. dedendum
W = 2356.2 ft-lb/min C. all of these*
and ultimate stress for compression of
D. pressure angle
.!!J_ = ~
60,000 psi. N2
31. An 8-in nominal diameter journal
A. 2.5 1200 6
bearing is designed for 140 deg 36. A mild-steel-spur gear ( 15-tooth, 20
B. 7.4
optimum when bearing length is 9-in, deg, full depth involute) pinion, rotating N2
c. 3.4 . speed is 1800 rpm, speed is 1800 rpm ,
D. 4.3 * at 750 rpm, is to transmit 30 hp to a N2 = 200 rpm
and total load is 20,000 lb. Calculate the mild-steel gear rotating at 200 rpm .
FS = Su = 60.000 = 4 .28 frictional horsepower loss when this Determine the number of teeth in gear. 39. ln problem No. 38, what is the
Sc 14.000 journal operates under the coefficient of A . 46 velocity of the driven gear in fpm?
friction of 0.002. B. 56* A.1570 *
29. A double-ply leather belt transmits A. 4.56 hp * C. 50 B. 1200
10 hp from a motor with a pulley 8-in in B. 10.2 hp D. 76 C. 200
diameter. runnig at 1700 rpm , to a 24-in c. 5.64 hp T1N1 = T:iN2 D. 1500
diameter pulley. The difference in D. 15.6 hp (15)(750) = T2 (200)
tension may be taken as 20 lb per inch T2 = 56 tooth

REFRESHER MANUAL 2nd Edition by JAS TORDILLO REFRESHER MANUAL 2nd Ed(ti'on by JAS TORDILLO

~
47 -141 Day 47- Solution Day 47- Solution 147 -15
Machine Design
v = TTDiN2 = ~r( ~~)zoo = 1570.8 fpm
16T
Ss = JCD3
t6(145xl2) = 3170 psi
11'(2f v, = 60 mph = 88 ftlsec
V, = 30 mph = 44 ftlsec
B. 1158
C. 974
- ·,.2- ,., 44 - 88 - 22 ftl .2 D. 2356 *
40. In problem No. 38, find the tooth 43. What pressure is required for a - - - =- - - - sec
punching a hole 2 inches in diameter I 2
load.
A. 1050 lb * through %-inch steel stock? r -_ ~=0.75m
2
A. 10 tons ·solving for the distance traveled in
B. 20501b
B. 30 tons coming to stop, V2 = 0: a= lOO~x 2Jrrad x min
C. 1570 lb
D. 28901b C. 20 tons vl = vl2 + 2aS min-sec rev 60 sec
D. 40 tons* 0 = (88) + 2(-22)S = 10.47 rad/sec
2

P = 2TTTN s = 176 ft
50(33,000) = 2TTT(1200) Pressure = d x t x 80 Torque = Ia = Y:z m~(aJ
T = 218.83 ft-lb = 2626 in-lb = (2) (Y.) 80 =
40 tons ' a press rotating at 120
47. A flywheel on = Y:z (800)(0.75) (1 0.47)
rpm is slowed down to 102 rpm during a = 2355.75 N-m
2626
Ft = .!._= = 1050 lb 44. What pressure is required for punching operation that requires o/.
Di 5/ punching a 1-inch square hole in a Y. second for the punching portion of the
, 2 ' 2 50. Which of the following steels
inch thick plate steel? cycle. What angular deceleration does contains nickel?
41. Calculate the torque in the shaft of a A 10 tons the flywheel experience in radians per A. SAE 1056
blower from the following data: shaft B. 26 2/3 tons * sec per sec? B. SAE 3140
diameter 2-in, speed 200 rpm, total C. 20 tons A. -2.52 * C. SAE 2040 *
head of air equivalent to 2-in WG, D. 32 1/3 tons B. 2.52 D. SAE 4140
volume discharged 30,000 cfm, and C. -2.25
blower efficiency 60%.
A. 315 ft-lb
_ Perimeter (t
Pressure-
3 . .
xso)- 4(1)(
-- -
3 4
I }o D. 2:25

B. 515ft-lb w, = 120 rpm = 12.56 rad/sec


c. 415 ft-lb *· = 26.67 tons w2 =
102 rpm =
10.68 rad/sec
D. 615 ft-lb
2
(62.4)
45. What pressure is required for
punching a 1 by 2 inch rectangular hole co2
a=--,--
-w,
_ 10.68- 12.56 = -2.52 rad/sec2
0.75 ·
h = hwdw =_12_ _ = ~ ft of air in Y. inch thicK brass?
da da da A. 15 tons
48. A body of mass 50 kg is being
JO,OOO(da { I0.4~ B. 32 '!. tons *
C. 20 tons hoisted by ::! winch , and the tension in
Odah \ da ) the cable is 600 newtons. What is the
D. 40 tons
BP -=~ = 0.60 acceleration in meters per sec per sec?
= 520 000 ft -lb Perimeter ( X s) A. 1.19
' min Pressure = t 6 B. 3.19 I
3
c. 2.19 *
BP = 2rrTN = [2(1)+ 2(2)] ( ~ }s D. 5.19

IFv = 0
52o,ooo fi ~ lb = 21rr(2oo~) = 32.5 tons T- W- REF= 0
mm nun 600 - 50(9.81)- 50( a) =0
T = 413.8 ft-lb 46. A car is moving at 60 mph when the a = 2.19 m/sec2 '
brakes are suddenly locked and the cars
42. In problem No. 41, calculate the begins to skid. If it takes 2 seconds to 49. A flywheel has a diameter of 1.5 m,
torsional stress in the shaft. slow the car to 30 mph, how far will it and a mass of 800 kg. What torque, in
A. 1950 psi have traveled in coming to stop? N-m, is needed to produce and angular
B. 2950 psi A. 146 feet acceleration of 100 revolutions per
C. 2125 psi B. 195 feet minute, per second?
D. 3170 psi* C. 176 feet* · A. 225
D. 255 feet
REFRESHER MANUAL·2nd Edition by JAS TORDILLO REFRESHER MANUAL 2nct ·E(,Jition by JAS TORDILLO
Day 48 - EXAM I 48 - 1
Machine Design
B. 987 mm
DAY 48- EXAM C. 275 mm
D. 352 mm
1. Stress shaft transm its 40 HP at 1400
rpm . Considering allowable shearing 6. In problem No. 5, find the tangential
stress based on pure torsion to be 5000 load on the teeth in Newtons.
psi, find the shaft diameter of nearest A. 1180 N
commercial size. B. 5328 N
A. 1 in C. 2680 N
B. 1 7/16 in D. 8525 N
C. 1% in
D. 2 in
7. In problem No. 5, determine the
tangential velocity.
2. In Problem No. 1, find the torsional A. 560 m/s
deflection of the shaft in degrees per B. 9,38 m/s
foot. C. 650 m/s
A. 0.004 deg D. 3.28 m/s
B. 0.462 deg
C. 0.246 deg 8. A 1.5 MTon holst is to be installed in
D. 0.102 deg an 1-beam mounted -Qn a truck platform,
to lift loads. The 1-beam is framed
3. A shearing machinerequires 150 kg-m horizontally, canti levered, with overhang
of energy to shear a steel sheet, and of 1.5 meter beyond the cantilever
has a normal speed of31ev/sec, showing fulcrum. Determine the section modulus,
down to 2.8 rev/sec during the shearing in cubic meter, of the 1-beam if allowable
process. The flywhdel of the machine stress is 100 Mpa.
has a mean diameter of 75 em and .,.A. 0.0002207
weighs 0.0155 kg/cm3. The width of the B. 0.0007022
rim is 30 em. If the hub and arms of the C. 12.24
flywheel account for15%of its total D. 0.002007
weight, find the weight of the flywheel.
A. -457 kg 9. A line shaft with a power of 100 kw at
B. 754 kg a speed of 1200 rpm , had a rectangular
c. 547 kg key used in its pulley connection.
D. 985 kg Consider the shearing stress of the shaft
to be 40 Mpa and the key to. be 200
4. In problem No. 3, find the thickness of Mpa, determine the width of the
the rim. rectangular key if it is one-fourth of the
A. 3.5 em shaft diameter.
B. 5.3 em A. 23.65 mm
C . 4.2 em B. 11.65 mm
D. 9.7 em C. 14.65 mm
" D. 9.65mm
5. A pinion and gear set is carefully cut
in a 20o full depth design. The 16-tooth 10. W ith the present water interruptions
steel pinion and 72-tooth cast iron gear prevailing in Metro Manila, you have
transmit 50 kw power when the pinion been asked to design an upright
turns 1400 rpm. The face width is 90 cylindrical water tank 6 meters in
mm and the module is 8. Find the center diameter and 6 meters high, vented, and
distance between pinion and gear. to be filled completely with water.
A. 225 mm
REFRESHER MANU At 2nd Edition by .J.AS TORDILLO
48 - 2 I Day 48 - EXAM
Day 48 - EXAM I 48 - 3
Determine the minimum thickness of the C. 174.53 deg Machine Design
tank plate if the stress is limited to 40 D. 185.47 deg
Mpa. 21. The area of a m3chine shop where C. contraction mandrel
A. 3.3 mm 15. Which of the following is not a metal is being melted to form ·a new.:. D. gang mandrel
B. 5.5 mm classification of iron ore? shape is ~
C. 4.4 mm A. hematite A. welding 28 . In the u=n II project, steel railroad
D. 8.8 mm B. sulfurite B. foundry area , rails of 10 meters long are to be
C. magnetite C. mass production area installed. If lowest temperature
11 . In problem No, 10, if the water tank D. siderite D. tool and die area considered is 160C, and a . maximum
had to be elevated 10 meters from its temperature of 360C is designed,
bottom and has to be filled up in 2 16. Of the following metals, which will 22. A machine used in shaping metal by assuming coefficient of thermal
hours, determine the motor power to respond to heat treatment? means of abrasive wheel or the removal expansion of steel to be 11.6 x 10-6
drive the pump, it velocity and head loss A. cast iron of metals with an abrasive is ca lled: mlm-oC and modulus of elasticity of
is 5 meters. Consider pump efficiency of B. medium carbon steel A. planer steel to be 207 Gpa, determine the
70%. C. wrought iron B. shaper clearance between rails such that the
A. 9.34 kw D. low cartoon steel C. power saw adjoining rail will just touch at maximum
B.7.12kw D. grinding machine design temperature.
C. 8.15kw 17. The ability of metal to withstand A. 1.32 mm
D. 6.94 kw loads without breaking down is: 23. A machining operation whereby the B. 3.32 mm
A. stress tool rotates wh ile the feed is stationary C. 2.32 mm
12. In the design of a marble tiling B. strain is called : D. 4.32 mm
machine, which components are a drive C. strength A. shaping
motor, V-belt transmission to a D. elasticity B. turning 29. In problem No. 28, determine the
C milling induced stress in the rai ls if during high
horizontal shaft, at the end of which is
D. reaming summer temperature of 40°C.occur.
mounted a cir.cular diamond blade 1 18. Which of the following metals is non-
A. 205 Mpa
meter distance from the v-belt shaft magnetic?
pulley. If a 1 meter diameter blade is A. cast iron 24. Which of the following metals is B. 9.6 Mpa.
used which requires a circular speed of B. cast steel easy to chisel? C. 5.4 Mpa
40 meters per second, and the drive A. alloy steel ~- 6.9 Mp!'l
C. alloy steel
motor runs at 1700 rpm, find the speed D. manganese steel B. manganese steel
C. stainless steel 30. A flanged coupling has an outside
ration.
D. cast iron diameter of 200 mm and connects two
A. 2.23 19. A machining operation whereby the
40 mm shafts. There are four 16 mm
B. 3.22 tool reciprocates and the feed is
25. Ferrous metals contain a relatively bolts on a 140 mm bolt circle. The radial
C. 5.32 stationary, is called
large amount of: flanged thickness is 20 mm. If the
D. 3.52 A. shaping
A. ca rbon torsional stress in the shaft is not to
B. planning
B. phosphorous exceed 26 Mpa, determine the shear
13. In problem No. 12, determine the C. reaming
·C. manganese stress in the bolts if uniformity
center distance between sheaves, if the D. turning
D. sulfur distributed. -
pitch diameter of the small sheave is
20. A 48 inches diameter saw blade is A.1.2Mpa
152 mm and the larger sheaves is 338
mounted on a pulley dr.iven steel shaft, 26 . Which of the following is not a B. 4.3 Mpa
mm, and using standard V-belt D-120
requiring a blade peripheral linear speed strength property of metals? C. 2.9 Mpa
(actual belt length is 3131.82 mm).
of 150 ftlsec. Motor drive is 125 HP at A. tensile strength D. 5.8 Mpa
A.1 178mm
B. 1778 mm 1200 rpm, with 6 inches diameter pulley. B. fatigue strength
Determine the shaft pulley diameter. C. rocki ng strength 31. In problem No. 30, determine the
C. 1871 mm
A. 8 inches D. torsional strength bearing pressure in the bolt.
D. 181 7 mm
B. 12inches A. 3.65 Mpa
14. In problem No. 12, determine the arc C. 10 inches 27. Which of the following is not kind of B. 6.35 Mpa
of contact. D. 15 inches mandr'21? C. 5.63 Mpa
A. 170.53 deg , A. expa ns1on mandrel D. 8.95 Mpa
B. 189.47 deg B. taper mandrel

REFRESHER MANUAL 2nd Edition by JAS TORDILLO REFRESHER MANUAL 2nd Edition by J.AS TORDILLO
48 - 4 I Day 48 - EXAM
Day 48 - EXAM I 48 - 5
32. A machine wh ich is used to finish C. 0.50 Machine Design
internal and external surfaces by the D. 2.00
use of a cutter, which has a series of 42. A 1200 mm cast iron pulley is 47. Galvanized iron is a term referring to
cutting edges or teeth. 38. A 12.7 mm 'diameter high-speed- fastened to a 112.5 mm s haft by means. iron coated with:
A. broaching machine steel end mill has four teeth and is t6 cut of a 28.13 mm square key 175 mm long. A. tin
B. milling machine 02 oil hardening tool steel having a What force acting at the pulley rim will B. magnesium
C. lathe machine hardness of 200-220 HB. The cutting shear the key if shear stress of the key C. zinc
D. planer speed for this steel is 15.2 meters per is 20.67 kg/mm2 ? D. aluminum
minute. Calculate the spindle speed. A. 9538 kg
33. Which of the following is not a kind A. 810 rpm B. 5839 kg 48. Stainless steel is obtained principally
of cast iron? B. 731 rpm C. 8593 kg by the use of which alloying elements;
A. malleable iron C. 651 rpm D. 3659 kg A. chromium
B. gray iron D. 381 rpm B. carbon
C. head iron 43. In a high rise building, the elevator C. ricket
D. white iron 39. A Mercedez Benz gas engine valve rises 366 meters with an operating D. tungsten
spring is to have a mean diameter of speed of 5 meters per second and
34. The kind of center which is being 3.81 em and a wire diameter of 0.635 reaches full speed in 10.68 meters. The 49. The internal stresses existing in a
attached and meshed to the tailstock em. The maximum load it will have to loaded elevator weighs 2000 kg. welded con nection are:
spindle which is also static while the sustain is 45.45 kg with corresponding Determine the acceleration of the A. not relieved by heat treatment
work is rotating is: deflection of 1.27 em. The spring is to elevator in m/sec2 . B. not relieved when the welded is
A. dead center be made of tempered steel wire . A . 1.17 peened
B. focal center Determine the no. of coils to be used. a. 3.17 C . may relieved wben the weld is
C. live center Use modulus of rigidity equal to 11.6 x C. 2.17 peened
D. work center 106 psi. D. 4.17 D. relieved by x-ray analysis
A. 7.4
35. Which process does not belong to B. 10.4 44. In general, the design stress and 50. Herringbone gears are gears which:
the group? C. 8.1 factor of safety are related as follows: A. do not operate on parallel shafts
A. resistance welding D. 14.4 A. desigr> stess = ultimate stress times B. have a line contact between the teeth
B. hardening factor of safety ~· consist of two left handed helical
C. soldering 40. A V-belt is to transmit 50 HP to a B. design stress = ultimate stress gears _:-:
D. brazing NH3 compressor. The sheave is 203.2 divided by factor of safety D. tend to produce and thrust on the
mm in diameter and turns at 1150 rpm, C. factor of safety = design stress shafts
36. A square single thread jackscrew while the larger sheave turns at 400 divided by ultimate stress
has 2 threads per inch. It is used to lift rpm. The service factor may be taken as D. Ultimate stress = factor of safety by
15 tons. The friction radius of the collar 1.5 and the center distance is equal to desi'gn stress
is 1 inch. The coefficient of friction the diameter of the larger sheave.
4t' . In the usual spur gearing, the: DAY 48 - ANSWER KEY
between the threads of the screw and Determine .the length of belt. 1. 8 16.8 31. A46. 0
base is 0.15, and that between the A . 67 in A. pitch ci rcle and base circle are the
2. c 17. c 32. A47. C
screw and collar is 0.13. Determine the B. 87in same 3. A18.0 33. C48. A
torque required to overcome collar C. 77 in B. working depth includes the clearance 4. A 19.A 34. A 49. 8
friction in fl-lbs. D. 97 in C. tooth outline are usually involute 5. 0 20. c 35. B 50. B
curves 6. B 21. B 36. A
A. 325 7. 8 22.0 37. c
B. 2500 41. In problem No. 40, determine the D.· tooth outline are .always cycliodal 8.A 23. C 38. 0
C. 950 no. of belts if the corrected Hp per belt is curves 9. 8 24. B 39. 8
D. 3950 5.96. 10.C 25. A 40. 0
A. use 10 belts 46. One of the following is not a 11.0 26. c 41 . B
12.A 27. C 42. A
37. In problem No. 36, determine the B. use 13 belts common term relating to the
13.A 28. C 43. A
pitch of the screw. C. use 11 belts classification of fits: 14.A 29. 8 44. B
A. 0.25 D. use 12 belts A. tunki ng 15.8 30. 0 45.C
B. 0.75 . B. medim force fit
C. snug
D. bound
REFRESHER MANUAL 2nd Edition by JAS TORDILLO
REFRESHER MANUAL 2nd Edition by J,AS TORD ILLO
48 - 6 I Day 48 • Solution
Day 48 - Solution 1 48 - 7

DAY 48 - SOLUTION V1 = rrDN1 = rr(0.75)(3) , Machine Design


= 7.068 m/sec
Vz = rrDN2 =rr(O. 75)(2.8) m=-1
D M = 1.5(1000)(1 .5)
1. Stress shaft transmits 40 HP at 1400 = 6.597 m/sec = 2550 kg-m = 22072.5 N-m
Tl
rpm. Considering allowable shearing
stress based on pure torsion to be 5000 D
psi, find the shaft diameter of nearest w[
KE = - lV 1 -V2
2 2] 8 = _I
16 z= 22072.56 = 0.0002207 m3
commercial size. 2g lOOxiO
D1 _= 128 mm = 0.128 m
A. 1 in
B. 1 7/16 in* 150 = _(w) [(7.068)2 2
- (6.597) ]
P = 2rrTN
9. A line shaft with a power of 100 kw at
2 9.81 a speed of 1200 rpm, had a rectangular
C. 1 o/. in
D.2in w= 457.25 kg
50 000 = 2rtJ
1400
)
key used in its pulley connection.

4. In problem No. 3, find the thickness of


' 'l 60 Consider the shearing stress of the shaft
to be 40 Mpa and the key to be 200
P =2rrTN T = 341 N-m
40(33,000) = 2rrT(1400) the rim . Mpa, determine the width of the
T = 150 ft-lb = 1800 in-lb A. 3.5 em*
T= Fr(~l) rectangular key if it Is
one-fourth of the
B. 5.3 em shaft diameter.
C. 4.2 em A. 23.65 mm
=
Ss 16T
7tD3
D. 9.7 em T = Fr (0-~28) B. 11.65 mm *
C. 14.65 mm
5000 = 16(1800) Wr+0.15W = W
Fr = 5328 N D. 9.65 mm
nD 3 Wr =
0.85W =
0.85(457) = 388.45 kg
P = 2rrTN
•D = 1.22 in use 1 7/16-in
Wr = rrDmb(t){p)
7. In problem No. 5, determine the
tangential velocity. 1200
2. In Problem No. 1, find the torsional
=
388.45 {75)(30)(t){0.0155)
A. 560 m/s
100 000
'
= 2rrT( 60
)
t = 3.54 em
deflection of the shaft in degrees per B. 9,38 m/s * T = 795.775 N-m
foot. C. 650 m/s
5. A pinion and gear set is carefully cut Ss = 16T
A. 0.004 deg D. 3.28 m/s
in a 20o full depth design. The 16-tooth 1tD3
B. 0.462 deg
steel pinion and 72-tooth cast iron gear
C. 0.246 deg * P = FrV
transmit 50 kw power when the pinion 40 000 000:>.:: 16(795 ·775 )
D. 0.1 02 deg
turns 1400 rpm. The face width is 90
50,000 = 5328 v ' ' 1tD3
V = 9.38 m/s
S= TL = 180CX:12) mm and the module is 8. Find the center D = 0.04662 m = 46.62 mm
JG ~rt (1.4375)4 (12x I 06 )
distance between pinion and gear.
A. 225 mm
8. A 1.5 MTon hoist is to be installed in
. .)2 .
e = 0.0074293 rad =0.246 deg B. 987 mm
an 1-beam mounted on a truck platform,
to lift loads. The 1-beam is framed
W idth w
'
= -D4 =46.62
-- =11.65 mm
4
C. 275 mm
horizontally, cantilevered, with overhang
D. 352 mm *
3. A shearing machinerequires150 kg-m of 1.5 meter beyond the cantilever 10. With the present water interruptions
of energy to shear a steel sheet, and fulcrum. Determine the section modulus, prevailing in Metro Manila, you have
Center distance
has a normal speed of3rev/sec, showing in cubic meter, of the 1-beam if allowable been asked to design an upright
down to 2.8 rev/sec during the shearing
T2 + T1 m(T2 + Tt)
= - - =__,_...::..___!..!. stress is 100 Mpa. cylindrical water tank 6 meters in
process. The flywheel of the machine 2Pd 2 A. 0.0002207 * diameter and 6 meters high, vented, and
has a mean diameter of 75- em and B. 0.0007022 to be filled completely with water.
weighs 0.0155 kg/cm3. The width ·of the = s(n+ 16) = 352 mm C. 12.24 Determine the minimum thickness of the
rim is 30 em. If the hub and arms of the 2 D. 0.002007 tank plate if the stress is limited to 40
flywheel account for15%of its total Mpa.
weight, find the weight of the flywheel. 6. In problem No. 5, find the tangential
Sf= Me=~- M A. 3.3 mm
A. 457 kg*
B. 754 kg
load on the teeth in Newtons.
A. 1180 N lc --z B. 5.5mm
c. 4.4 mm*
C. 547 kg B. 5328 N * D. 8.8 mm
D. 985 kg C. 2680 N Section modulus, Z = M
D. 8525 N Sf
P = yH = 9.81(6) = 58.86 kPa
REFRESHER MANUAL 2nd Edition by JAS TORDILLO
REFRESHE~ MANUAL 2nd Edition b¥. JAS TORDILLO
48 - 8 1 Day 48 - Solutio'n Day 48 - Solution 1 48 - 9
Machine Design
PD
St = - Nm = Ds = 1700 = 2 .
23
2t Ns Dm 764 B. medium carbon steel • 21.The area of a machine shop where
C. wrought iron _metal is _Q_eing melted to form a new
40 ooo = (s8.86X6) D. low cartoon steel ,.§ha_p~ is:
• 2t 13. In problem No. 12. determine the A. welding
t = 4.41 x 10'3 m = 4.41 mm center distance between sheaves, if the
17. The ability of metal to withstar:!d B. foundry area *
pitch diameter of the small sheave is loads without breaking down is: C. mass production area
11. In problem No. 10, if the water tank 152 mm and the larger sheaves is 338 D. tool and die area
A. stress
had to be elevated 10 meters from its mm. and using standard V-belt D-120 B. strain
bottom and has to be filled up in 2 (actual belt length is 3131.82 mm). C. strength • 22. A machine used in shaping metal by
hours, determine the motor power to A.1178 mm * means of abrasive wheel or the removal
D. elasticity
drive the pump, if velocity and head loss B. 1778 mm of metals with an abrasive is called:
is 5 meters. Consider pump efficiency of C. 1871 mm A. planer
18. Which of the following metals is non-
70%. D. 1817 mm
magnetic? B. shaper
A. 9.34 kw A. cast iron C. power saw
B. 7.12kw From Philippine Mechanical Code,p. 22: D. grinding machine *
B. cast stee l
C. 8.15 kw C . alloy steel
D. 6.94 kw* b = 4L - 6.28 {0 + d) 23. A machining operation whereby the
D. manganese steel *
2 = 4(3131.82)-6.28(388+152) tool ~ while the feed is stationary
Q = n(6) (6) =84.82 m3/hr = 9450.08 is called:
4(6) ,.----- - 19. A machining operation whereby the
c 2
= b+Jb -32(D-d)2 tool ~tes and the feed is A. shaping
= 0.02356 m3/sec stationary, is calred: B. turning
H = 10 + 6 + 5 = 21 m 16 A. shaping* C. milling *
BP = QyH 8 . planning D. reaming
c = 9450+ J(9450)2 - 32(338-1 52)2 C. reaming
TJP
16 D. turning 24. Which of the following metals is
BP = (0.02356X9.81X21) = 6 .934 kw C = 1177.6 mm easy to chisel?
0.70 20. A 48 inches diameter saw blade is ~- alloy steel
1-4. In problem No. 12, determine the arc mounted on a pulley driven steel shaft, B. manganese steel *
12. In the design of a marble tiling of contact. requiring a blade peripheral linear speed C. stainless steel
machine, which components are a drive A. 170.53 deg * of 150 ft/sec. Motor drive is 125 HP at D. cast iron
motor, V-belt transmission to a B. 189.47 deg 1200 rpm, with 6 inches diameter pulley.
horizontal shaft, at the end of which is C. 174.53 deg Det~rmi ne the shaft pulley diameter. 25. Ferrous metals contain a relatively
mounted a circular diamond blade 1 D. 185.47 deg A. 8 inches largeamount or.
meter distance from the v-belt shaft B. 12 inches A. carbon*
pulley. If a 1 meter diameter blade is e = 18oo - (o- d)6oo C. 10 i nches* B. phosphorous
used which requires a circular speed of c D. 15 inches C. manganese
40 meters per second, and the drive D. sulfur
motor runs at 1700 rpm, find the speed = 1800 - (338 -1 52)600 =170.2560 V = DaNa
ration. 1178 26. Which of the following is not a
A. 2.23 *
B. 3.22 15. Which of the following is n.~
150 = n( ~~)Ns streQ9.tb pr:oped.y .Qf_metals?
A. tensile strength
c. 5.32 classjficatjon of iron cu:e? Na = 12 rev/sec = 720 rpm B. fatigue strength
D. 3.52 A. hematite C. rocking strength *
B . sulfurite * Speed of saw blade, Na = Speed of D. torsional strength
V = lTDwNw C. magnetite shaft,Ns
40 = lT {1)Nw D. siderite 27. Which of the following is _Q_Ot k.in.d of
Nw = Ns = 12.73 rev/sec DmNm = DsNs mandrel?
= 764 rev/min 16. Of the following metals, which will 6(1200) = 0.(720) A."ei<Pansion mandrel
DmNm ·= DsNs ~ond to heat treatment? D.= 10 in (shaft pulley diameter) B. taper mandrel
A. cast iron

REFRESHER MANUAL 2nd Edition by JAS TORDILLO REFRESHER MANUAL 2nd Edition by·JAS TORDILLO
48- 10 1 Day 48- Solution Day 48 - Solution 1 48 - 11
Machine Design
C. contraction mandrel*
Ss = 16T
D. gang mandrel C. head iron * 38. A 12.7 mm diameter high-speed-
nD 3
D. white iron steel end mill has four teeth and is to cut
28. In the LRT II project, steel railroad
rails of 10 meters long are to be
26x106 = ~ 02 oil hardening tool steel having a
n(0.040)l 34. The kind of center which is being hardness of 200-220 HB. The cutting
installed. If lowest temperature attached and meshed to the tailstock speed for this steel is 15.2 meters per
considered is 160C, and a maximum
T = 326.73 N-m
spindle which is also ..s_tatic while the minute. Calculate the spindle speed.
temperature of 360C is designed, work is rotating is: A. 810 rpm
assuming coefficient of thermal Db A. dead center *
T = Frx - B. 731 rpm
expansion of steel to be 11.6 x 10-6 2 B. focal center C. 651 rpm
m/m-oC and modulus of elasticity of
( 0 ·~40 )
C. live center D. 381 rpm*
steel to be 207 Gpa, determine the 326.73 = Frx D. work center
clearance between rails such' that the V = ON
adjoining rail will just touch at maximum Fr = 4667.5 N 35. Which process does not belong to 15.2 = (0.0127)N
design temperature. 4667.5 the group? N = 381 rpm
A. 1.32 mm Force per bolt, Fb - - = 1166.9 N A. resistance welding
B. 3.32 mm
- 4
B. hardening * 39. A Mercedez Benz gas engine valve
C. 2.32 mm * . Fb C. soldering spring is to have a mean diameter of
Shear stress 1n the bolts, Ssb = --
D. 4.32 mm
~d2 D. brazing 3.81 em and a wire diameter ot' 0.635
4 em. The maximum load it will have to
Clearance, c = kl(t.2- t,) 36. A square single thread jackscrew sustain is 45.45 kg with corresponding
:: 11.6 X 10'6 (10)(36 -16) = 1166.9 = 5.8 MPa has 2 threads per inch. It is used to lift deflection of 1.27 em. The spring is to
= 2.32 x 10'3 m = 2.32 mm ~ (16)2 15 tons. The friction radius of the collar be made of tempered steel wire.
4 is 1 inch. The coefficient of friction Determine the no. of coils to be used.
29. In problem No. 28, determine the between the threads of the screw and Use modulus of rigidity equal to 11.6 x
induced stress in the rails if during high 31. In problem No. 30, determine the base is 0.15, and that between the 106 psi.
summer temperature of 40°C occur. bearing pressure in the bolt. screw and collar is 0.13. Determine the A. 7.4
A. 205 Mpa A. 3.65 Mpa * torque required to overcome collar $\. 10.4 * '
B. 9.6 Mpa * B.. 6.35 Mpa friction in fl-lbs. , C. 8.1
C. 5.4 Mpa c.. 5.63 Mpa A. 325 * D. 14.4
D. 6.9 Mpa D. 8.95'Mpa B.2500
C. 950 C = Dm =~=6
Induced stress, S = kE(tr -II) Bearing pressure, Sc = £!!.. D. ~950
d 0.635
= 11.6x 10-e(207x 109 )(40-36) td 3
= 9,604,800 Pa = 9.6 Mpa 1166 7 y = 8FC n
= · = 3.646 M a Tc = Wx rmxf Gd
20(16) p = 15(2000)(1)(0.13)
30. A flanged coupling has an outside 3
= 3900 in-lbs = 325 ft-lbs _ = 8(45.4Sx9.8tX3) n
diameter· of 200 mm and connects two 0 0127
32. A machine which is .used to finish 80x10 9 (0.0035)
40 mm shafts. There are four 16 mm internal and external surfaces by the 37. In problem No. 36, determine the
bolts on a 140 mm bolt circle. The radial use of a cutter, which has a series of · pitch of the screw. n = 8.37
flanged thickness is 20 mm. If the cutting edges or teeth. A. 0.25 actual no. of coils = n + 2 = 8.37 + 2
torsional stress in the shaft is not to A. broaching machine * B. 0.75 = 10.37 coils say 10.4
exceed 26 Mpa, determine the shear B. milling machine c. 0.50 *
stress in the bolts if uniformity D. 2.00 40. A V-belt is to transmit 50 HP to a
C. lathe machine
distributed. D. planer NH3 compressor. The sheave is 203.2
A. 1.2 Mpa Pitch of the screw mm in diameter and turns at 1150 rpm,
B. 4.3 Mpa 33. Which of the following is not a kind while the larger sheave turns at 400
l I .
C. 2.9 Mpa of cast iron? = =-= 0.50 InCh rpm . The service factor may be taken as
D. 5.8 Mpa * threadsperinch 2 1.5 and the center distance is equal to
A. malleable iran
B. gray iron the diameter of the larger sheave.
Determine the length of belt.

REFRESHER MANUAL 2nd Edition by JAS TORDILLQ REFRESHER MANUAL 2nd Edition by·JAS TORDILLO
r-

48 - 12 I Day 48 - Solution
A. 67 in
B. 87in
C. 77 in
{12;0)=(101,753t 1~.5) 47. Galvanized iron is a term referring to
Day 48 - Solution 148 - 13
Machine Design l
F = 9538 kg iron coated with:
D. 97 in*
A. tin
D1N1 = D2N2 43. In a high rise building, the elevator B. magnesium
C. zinc*
203.2(1150) = D2(400) rises 366 meters with an operating
D2 = 584.2 mm = 23 in speed of 5 meters per second and D. aluminum
D1 = 203.2 mm = 8 in reaches full speed in 10.68 meters. The
Center distance, C D2 = =23 in loaded elevator weighs 2000 kg. 48. Stainless steel is obtained principally
Determine the acceleration of the by the use of which alloying elements;
L = 2C+~(D 2 +0 1)+ (D 2 - 0 1)2 elevator in m/sec2. A. chromium *
B. carbon
2 4C A.1.17*
C. ricket
( ( -&r B. 3.17
L = 2(23)+~(23+8)+
2
23
4 23 .
c. 2.17 D. tungsten
D. 4.17
L = 97 in 49. The internal stresses existing in a
v1 = o welded connection are:
41. In problem No. 40, determine the v2 = s A. not relieved by heat treatment
no. of belts if the corrected Hp per belt is V/ = V12 + 2aS B. not relieved when the welded is
5.96. (5)2 = 0 + 2a(10.68) peened*
A. use 10 belts a = 1.17 m/s2 C. may relieved when the weld is
B. us~ 13 belts* peened
C. use 11 belts 44. In general, the design stress and D. relieved by x-ray analysis
D. use 12 belts factor of safety are related as follows:
A. design stess = ultimate stress times 50. Herrin bone ears are gears which:
DeslgnHP factor of safety A. do not operate on arallel shafts
No. of belts = Corrected Rated HP B. design stress : ultimate stress B. have a line contact between the
divided by factor of safety * teeth*
_ 50(1.5)
- 5.96
= 12 _58 say 13 belts C. factor of safety · = design stress
divided by ultimate stress
C. consist of two left handed helical
gears
D. Ultimate stress = factor of safety by D. tend to produce and thrust on the
42. A 1200 mm cast iron pulley is design stress shafts
fastened to a 112.5 mm shaft by means
of a 28.13 mm square key 175 mm long. 45. In the.usual sour gearing, \he:
What force acting at the pulley rim will A. pitch circle and base circle are the
shear the key if shear stress of the key same
is 20.67 kg/mm 2 ? B. working depth includes the clearance
A. 9538 kg* C. tooth outline are usually involute
B. 5839 kg curves*
C. 8593 kg D. tooth outline are always cycliodal
D. 3659 kg curves
F
Ss = - 1 46. One of the following is not a
wL common tenm relating to ---u;e-
20.67 = ___5 classification of fits:
28.13(175) A. tunking
F1 = 101 ,753 kg B. medim force fit
C. snug
{12200)=Ff 1~.5) I D. bound*

REFRESHER MANUAL 2nd Edition by JAS TORDILLO


REFRESHER MANUAL 2nd Edition by·JAS TORDILLO

.... II
Day 49 - EXAM I 49 - 1
Machine Design
D. 0.95 Pa
DAY 49- EXAM
6. A thin hollow sphere of radius 10
1. A shearing machine is used t~ in and thickness 0.10 in is subjected
crop off lengths of round bar 20 mm to an internal pressure of 100 psi.
diameter. If the ultimate shear The maximum normal stress on an
strength of the· material is 160 MPa, element of the sphere is:
calculate the force needed to crop A. 5000 psi
the bar. B. 1410 psi
A. 48 kN G; 7070 psi
B. 50.3 kN D. ""4500 psi
C. 53 kN
D. 55 kN 7. Determine the load in kN on a 30
mm diameter by 1000 mm long steel
2. The force of a point of a shaper shaft'if its maximum elongation will
when cutting is 1500 N. If the length not exceed 1.2 mm:
of the stroke is 120 mm, how much A. 167
work is done in one cutting stroke? B. 199
A. 180 J C.176
B. 200 J D.245
C. 195 J
D. 10_0 J 8. For most metals Poisson's ratio
lies in the range:
3. The shear s-trength of a plate is 300 A. 0.25 to 0.35
MPa. Cakculate the force required to B. 0.35 to 0.45
punch a hole 40 mm diameter in a C. 0.30 to 0.40
plate 7 mm thick. D. 0.40 to 0.50
A . 200 kN
B. 250 kN 9; How long will it take for a 51 mm
C. 230 kN length' keyway to be milled if the
D. 264 kN milling machine has a 24 teeth cutter
turning at 130 rpm and feed rate of
4. The shear strength of a shaft is not 0.127 mm per tooth?
to exceed 30 MPa. The shaft is 100 A. 0.281 min
mm. What is the greatest torque that B. 0.128 min
can be placed on it? C. 0.218 min
A. 5900 J D. 0.812 min
B. 6200 J
C. 6100 J 10. Compute the polar section
D. 5200 J modulus in cu. inches of an SAE
1060 shafting having a diameter of 3
5. Two shafts are connected by a inches. Use a factor of safety of 2 and
flange coupling. The coup ling is design stress at 8000 psi.
secured by 6 bolts, 20 mm in A. 4.7
diameter on a pitch circle diameter of B. 4.2
150 mm. If torque of 1.20 N-m is C. 6.1
applied, find the shear stress in the D. 5.3
bolts.
A. 0.85 N/mm2
B. 0.85 Pa
C. 0.85 kPa ,,
REFRESHER MANUAL 2nd Edition by JAS TORDILLO.
49 - 2 I Day 49 - EXAM Day 49 - EXAM I 49 - 3
Machine Design
11. What spindle speed is required to B. 3in
produce a cutting speed of 150 fpm C. 2.5 in D. chromium
on a 2 in diameter bar? D. 3.2 in 29. It is a general practice to use the
A 120 rpm 23. Increases hardenability of alloy following allowable stresses, 4000
B. 287 rpm 17. Determine the power transmitted steels: psi for main-power transmitting shaft
C. 200 rpm • by 1 in short shaft rotating at 380 A. manganese and 8500 psi for small, short,
D. 1000 rpm rpm. B. molybdenum countershafts. For lineshaft carrying
A. 10 kw C. phosphorous' pulley it is ___ psi.
12. A round steel shaft transmits 50 B. 9.7 kw D. all of these A 8500
kw an<;l rotates at 150 rad/s. Find the C. 7.46 kw B.6000
angle of twist for a 3 meters length D. 6.10 kw 24. Improves the machinability of C. 7000
shaft is allowable shearing stress of alloy steels but affects different D~£500
40 MPa and G = 68 GPa. 18. A flywheel has diameter of 1.5 m alloys differently:
A. 4.3 deg and a mass of 1000 kg. What torque A. all of .these 30. Steel that ha·s been deoxidized
B. 5.8 deg is needed to produce an angular B. vanadium with a strong deoxidizing agent such
C. 2.5 deg acceleration of 120 revolutions per C.- selenium as sH.icon or aluminum in order to
D. 6.7 deg minute, per second? D. lead eliminate oxygen and carbon reaction
A. 3534 J during solidification.
13. The cutting speed of a too l B. 3345 J 25. Which of the following is not a A. rimmed steel
passing th ru 2.5 in diameter material C. 3354 J classification of alloy steels? B. killed steel
at 200 rpm . D. 3453 J A. medium-carbon alloy C. stainless steel
A. 110 fpm B. high-alloy 0. monel
B. 140 fpm 19. The most effective alloy in C. low-alloy
C. 115 fpm increasingly toughness of steel: D. medium-all9y 31. Device that prevents entrance of
D. 131 fpm A. copper moisture and other fluids and permits
B. nickel 26. In the SAE identification code of pressure differential between the
14. A shaft is subjected to a twisting C. lead steel shafting the 3'd and 4 1h digits exterior and the interior:
moment of 3.6 KN-m . The shaft is D. all of these represents the content of: A. gasket
steel having 83 GPa modulus of A. % manganese on the steel e. seal
rigidity. T he torsional shear stress (in 20. When steel castings are heat B. %carbon contents C. washer
MPa) in a 76 mm circular shaft is : treated, the carbon content generally C. % alloy elements D. spaber
A. 40 falls within the range of: D.% chromium in the steel
B. 36 A. 0.20% 32. Device used to guard surfaces
c. 39 B. 0.15 to 0.20% 27. A moving truck skids to a stop 20 against marring
D. 41 C. 0.25 to 0.50% meters after the brakes are applied A. locker
D. 0.60% while traveling 90 kph. What is the · B. washer
15. A spur pinion rotates at 1800 rpm acceleration in m/s2 ? C. bearing
and transmit to a mating gear 50 HP. 21. Improves the ratio of endurance A. 15.6 D. oil seal
The pitch diameter is 6 in and the strength to ultimate strength .o f B. -15.6
pressure angel is 14 Y:z. Determine the medium-carbon steels: C. 16.5 33. When the hole is smaller than the
tangential load. A. copper D. -16.5 shaft, it will take force to put the
A. 2831bs B. vanadium parts together, the all owance is said
B. 5831bs C. tantalum 28. The process of increasing the to be negative and is ca lled:
C. 2651bs D. all of these carbon content to the surface of steel A. interference
D. 4851bs by exposing it to hot carbonaceous B. allowance
22. Improves hardenability material above the transformation C. clearance
16. Compute the line shaft diameter economically, resistance to corrosion temperature of 1650 to 1750 deg F. D. toughness
to transmit 25 HP at 300 rpm with strength at high temperatures: A. carbonitriding
torsional deflection of 0.08 deg per A. manganese B. carburizing 34. The stress in an elasti c material
foot length. G = 12,000,000 psi. B. aluminum C. case hardening is:
A. 2in C. copper D. hardening

REFRESHER MANUAL 2nd Edition by JAS TORDILLO REFRESHER MANUAL 2nd·f~iticm by JAS TORDILLO
49 - 4 I Day 49 - EXAM Day 49 -Solution 1 49 - 5
Machine Design
A. inversely proportional to the yield 40. Steels containing large amounts
strength of the material of mild nickel and chromium: C. ionization
B. inversely proportional to the force A. carbon steel D. composition
DAY 49 - SOLUTION
acting B. alloy steel
C. proportional to the displacement C. stainless steel 48. Modulus of rigidity of a steel 1. A shearing machine is used to
of the material acted upon by the D. cast steel member is: crop off lengths of round bar 20 mm
force A. a function of the length and width diameter. If the ultimate shear
D. proportional to the length of the 41. The product of the mass and the B. equal to the modulus of elasticity strength of the matehal is 160 MPa,
material subject to the force linear velocity of a body: divided by one phis Poisson's ratio calculate the force needed to crop
A. angular impulse C. defined as the unit shear stress the bar.
35. A micrometer measured by: B. linear impulse divided by the unit shear deformation A. 48 kN
A. 108 C. angular momentum D. defined as the length divided by B': 50.3 kN *
B. millions of an inch D. linear momentum the moment of i'nertia · C. 53,kN
C. root means square D. 55 kN
D. thousands of an inch 42. When a steel is described as SAE 49. Arc of the pitch circle through F
Ss = -
1320: which a tooth travels form the point ' A
36. An impact test is used to test a A. it is plain carbon steel of contact with the mating tooth to F
metal for: B. it is nickel-chromium steel the pitch point of a gear: 160 = - -
A. toughness C. it contains 18 to 20% carbon A. arc of recess 2: (zo?
B. ductility D. it contains 0.18 to 0.23% carbon B. arc of relief 4
C. strength C. arc of action F = 50,265 N = 50.26 kN
D. hardness 43. Which is not a heat treatment D. arc of approach
process? 2. The force of a point of a shaper
37. A partial bearing Is one in which: A. sintering 50. Which of the following steel when cutting is 1500 N. If the length
A. the bearing is supplied with less B. hardening contains chromium? of the stroke is 120 mm, how much
oil than full bearing C. annealing A. SAE 2340 work is done in one cutting stroke?
B. the bearing encloses less than 360 D. tempering B. SAE 1230 A. 180 J *
deg of the journal C. SAE 4230 B. 200 J
C. the bearing is slightly loaded 44. The term used in referring to the D. SAE 5240 c. 195 J
D. the bearing is supplied with oil
intermittently
continuous increase in the strain, or
deformation, of any material
D. 100 J
. .
subjected to stress. W = Fxd
38. For a completely corrosion A. elasticity =1500(0.120)
resista nt stai nless steel, what B. creep OAY 49 - ANSWER KEY w = 180 J
minimum percentage of chromium in C. tolerance 1.8 16.C 31.846.C
the alloy is required? D. all of the above 2.A 17.C 32. 847. 0 3. The shear strength of a plate is 300
3.0 18. A 33.A48. C MPa. Cakculate the force required to
A. 3% 4. A 19. 6 34. C 49. 0
B. 11% 45. Poison's ratio is the ratio of the: 5. A 20. C 35. 0 SO. 0 punch a hole 40 mm diameter in a
C. 7% A. unit lateral deformation to the unit 6. A 21 . 6 36.A plate 7 mm thick.
D. 15% longitudinal deformation 7. C 22. 0 37. 6 A. 200 kN
8. 0 23. 0 38. 8 B. 250 kN
B. unit stress to unit strain 9. 6 24. 0 39. 6
39. The amount by which the width of C. elastic limit to proportional limit 10.0 25. 0 40. C
C. 230 kN
a tooth space exceeds the thickness D. shear strain to compressive strain 11.8 26. B 41 . 0 D. 264 kN *
of an engaging tooth on the pitch 12.6 27. B 42. 0
circles of a gear: 46. At relatively high temperatures 13.0 28. 6 43. A F = Ss(A)
14.0 29. 8 44. B = Ss(TTdt)
A. clearance and low rates of strains, structures 15.8 30. B 45. A
B. backlash will perform better if the material is: = 300(TT)(40)(7)
C. tip relief A. fine-grained F = 263,893 N = 263. 9 kN
D. chordal thickness B. their behaviour is independent ·of
grain 4. The shear strength of a shaft is not
C. course grained to exceed 30 MPa. The shaft is 100
I
REFRESHER MANUAL 2nd Edition by JAS TORDILLO REFRESHER MANUAL 2nd,ditlon by JAS TORDILLO
49 - o 1 Day 49 - Solution
Day 49 - Solution 149 - 7
What is the greatest torque that can PD Jlllachine Design
be placed on it? s, = -
41
A. 5900 J * .......___ 1060 shafting having a diameter of 3 TL 180
B. 6200 J """'-._, S _ I00(2xiO) inches. Use a factor of safety of 2 and e= -x-
C. 6100 J I - 4(0.10) design stress at 8000 psi.
JG n
D. 5200 J - 333.38(3) 180
Sr = 5000 psi - x-
A. 4.7
Ss = 16T n(0.0349)4 { &xl09)
6 1t
7. Determine the load in kN on a 30 B. 4.2
nD 3 C. 6.1 32
mm diameter by 1000 mm long steel 32
I6T D. 5.3 *
30 106 = - - ,
X shaft if its maximum elongation will e = 5.78 deg
n(o. 1oo)J not exceed 1.2 mm. nDJ
T = 5890.48 J A. 167 zp = -
16 13. The cuftin g speed of a tool
B. 199
passing thru 2.5 in diameter material
n{3)- = )- . .)~ 111
C.176 * 3
5. Two shafts are connected by a -
. 3 at 200 rpm.
flange coupling. The coupl ing is 0.245
16 A. 110 fpm
secured by 6 bolts, 20 mm in B. 140 fpm
diameter on a pitch circle diameter of E = 30 x 106 psi = 206,786 MPa for
11. What spindle speed is required to C. 115 fpm
150 mm. If torque of 120 N-m is steel
produce a cutting speed of 150 fpm D. 131 fpm *
app·lied, find the shear stress in the y = FL on a 2 in diameter bar?
bolts. AE A. 120 rpm V = TTDN
A. 0.85 Nlmm 2 *
= TT(~;}oo
B. 287 rpm *
B. 0.85 Pa 1 _2 = F(IOOO)
C. 200 rpm
C. 0.85 kPa -n._ (0.030)
2
~60,786xl 06 ) D. 1000 rpm
D. 0.95 Pa 4 = 130.9 fpm
F = 175,402.1 N = 175.4 kN V = ON
F = total force on bolts 14. A shaft is subjected to a twisting

F =
T 120
D/ = O.IS9; =1600 N
8. For most metals Poisson's ratio
lies in the range:
150 = ltC~)N moment of 3.6 KN-m. The shaft is
steel having 83 GPa modulus of
/2 .2 A. 0.25 to 0 35 N = 286.5 rpm 'ligidity. Th~torsional shear stress (in
1600 B. 0.35 to 0.45 MPa) in a 76·mm circular shaft is:
Fe = force per bolt = - - = 266.67 C. 0.30 to 0.40 12. A round steel shaft transm its 50 A. 40
6 D. 0.40 to 0.50 *
N kw and rotates at 150 rad/s. Find the B. 36
angle of twist for a 3 meters length C. 39
Sse = shear stress in the bolts
9. How long will it take for a 51 mm shaft is allowable shearing stress of D. 41 *
Fs length keyway to be milled if the 40 MPa and G = 68 GPa.
Sse=- Ss = 16T
As milling machine has a 24 teeth cutter A. 4.3 deg
turning at 130 rpm and feed rate of B. 5.8 deg * nD 3
266 67
= · = 0.849 N/mm 2 0.127 mm per tooth? C. 2.5 deg - 16(3,600)
1t (20 )2 A. 0.281 min D. 6.7 deg - n(0.076)3
4 B. 0.128 min*
C. 0.218 min N = 150 rad/s = 23.87 rev/s = 41 ,766,860 Pa = 41.7 MPa
6. A thin hollow sphere of radius 10 D. 0.812 min
in and thickness 0.10 in is subjected
to an internal pressure of 100 psi.
P = 2TTTN 15. A spur pinion rotates at 1800 rpm
and transmit to a mating gear 50 HP.
Time = length of cut 50,000 = 2TTT(23.87)
The pitch diameter is 6 in and t he
The maximum normal stress on an cutting rate T = 333.38 N-m
element of the sphere is: pressure angel is 14 ~ - Determine t he
A. 5000 psi* 51
s. = 16T tangential load.
= , =0.1287 mm
.
icD 3 A. 283 1bs
B. 1410 psi
C. 7070 psi X 6 = 16(333.38)
B. 583lbs *
D. 4500 psi 3 10 C. 2651bs
10. Compute the polar section nD D. 4851bs
modulus in cu. inches of an SAE D = 0.0349 m
I
REFRESHER MANUAL 2nd Edition by JAS TORDILLO
REFRESHER MANUAL 2nd Edition by JAS TORDILLO
49 - 8 I Day 49 - Solution Day 49 - Solution 1 49 - 9
Machine Design
.. A. 3534J *
B. 3345 J
P = 2rrTN 24. Improves the machinahilitv of 29. It is a general p ractice to use the
50(33,000) = 2rrT(1800) c. 3354 j aj!Qy__ steels but following allowable stresses, 4000
T = 145.892 ft-lbs = 1750.7 in-lbs D. 3453 J alloys~ntly : psi for main-power transmitting shaft
A . all of these and 8500 ps i fo r small, short,
a = 120 rev/min-sec = 12.56 B. vanadium countershafts. For lineshaft carrying
F = - T . = 1750.7
6 = 583.6 lbs red/sec2 C. selen ium pulley it is ___ psi.
D' 2 / '2· 1.5 D. lead • A.8500
k = - = 0.75m
2 B. 6000 *
16. Compute the line shaft diameter T = Y, mk2 (a) 25. Which of the following is not a C. 7000
to transmit 25 HP at 300 rpm with = y, (1000)(0.75)2 (12.56) classifi cation of alloy steels? Q.6500
torsional deflection of 0.08 deg per = 3532.5 j . A. medium-carbon alloy
foot length. G ='= 12,000,000 psi. B. high-allo y '
A. 2 in 19. The most effective alloy in C. low-alloy
B. 3in increasingiYtaughness of steel: D. medium-alloy *
C. 2.5 in • A. copper
D. 3.2 in B. nickel* 26. In the SAE identifi cation code of
C. lead steel shafting the 3'd and 4tn digits
P = 2rrTN D. all of these represents the content of:
25(33,QOO) = 2rrT(300) A. % manganese on the steel
T = 437.67 ft-lbs = 5252.11 in-lbs 20. When steel castings are ~t B. % carbon contents *
C . % alloy elements
e = ___!_!: treated, the carbon content generally
1aJis:wilbin the ra~of: D. %chromium in the steel 31. Device that prevents entrance of
. nD4 (G) A. 0.20%
moisture and other fluids and permits
32 ' B. 0.15 to 0.20% 27. A moving; truck skids to a stop 20 pressure differential between the
5252. 11(12) C. 0.25 to 0.50% * meters after the brakes a re applied exterior and the interior:
o.o8(...2:...)
180 D. 0.60% while traveling 90 kph. What is the A. gasket
nD4 V2xl06) acceleration in m/s2 ? B. seal*
32 21. Improves the ratio of endurance A . 15.6 C. washer
D = 2.48 in strength to ultimate strength of B. -15.6 * p. spacer
medium-carbon steels: - c. 16.5
17. Determine the power transmitted A. copper D. -16.5 32. Device used to guard surfaces
by 1 in short shaft rotating at 380 B. vanadium * against marring:
rpm. C. tantalum v1 = 90 kp h = 25 m/s A. locker
A. 10 kw D. all of these v2 = o B. washer*
B. 9.7 kw C. bearing
C. 7.46 kw * 22. Improves hardenability vl = V1 2 + 2aS D. oil seal
D.6.10kw ~onomica lzy_JeSLstance to corros10n 0 = (25) 2 + 2a(20)
strength at high teme!!ratu res: a = -15.625 m/s2 33. Whe n the hole is smaller than the
p =
D3 N shaft, it will take force to put the
A . manganese
38 28. The process of increasing the parts together, the allowance is said
B. aluminum
3 C. copper carbon content to t he surface of steel to be negative and is called:
= (1) (380) by exposing it to hot carbonaceous A. interference *
D. chromium *
38 material above the transformation B. allowance
10 hp = 7.46 kw 23. Increases hardenability of alloy tempe ratu re of 1650 to 1750 deg F. C. clearance
A. carbonitriding · D. toughness
~:
18. A flywheel has diameter of 1.5 m A. manganese B. carburizing *
and a mass of 1000 kg . What torque C. case hardening 34. The stress in an elastic material
B. molybdenum
is needed to produce an angular C. phosphorous D. hardening is:
acceleration of 120 revolutions per D. all of these * A. inversely proportional to the yield
minute, per second? strength of the material

REFRESHER MANUAL 2nd Edition by JAS TORDILLO REFRESHER MANUAL 2"d'dHion by JAS TORDILLO
·:
49 -10 1 Day 49- Solution Day 49 - Solution 1 49 - 11
B. inversely proportional to the force
Machine Design
B. alloy steel
acting C. stainless steel * 47. The most important factor in
C. proportional to the displacement D. cast steel determ ining high-temperpture
of the material acted upon by the
behavior of an alloy is:
force" 41 . The product of the mass and the A. dispersion
D. proportional to the length of the linear velocity of a body: B. crystallization
material subject to the force A. angular impulse C. ionization
B. linear impulse D. composition ~
35. A micrometer measured by : C. angular momentum
A. 108 D. linear momentum * 48. Modulus of rigidity of a steel
B. millions of an inch
C. root means square ~r is : -
42. When a steel is described as SAE A. a function of the length and width
D. thousands of an inch * 1320: B. equal to the•modulus of elasticity
A. it is plain carbon steel divided by one plus Poisson 's ratio
36. An impact test is used to test a B. it is nickel-chromi um steel C. defined as the unit shear stress
metal for: C. it contains 18 to 20% carbon divided by the unit shear
A. toughness * D. it contains 0.18 to 0.23% carbon* deformation"
B. ductili ty
D. defined as the length d ivided by
C. strength 43. Wh ich is not a heat treatment the moment of inertia
D. hardness process?
A. sinterlng * 49. Arc of the pitch circle through
37. A partial bearing is one in which : B. hardening which a tooth tra vels form the point
A. the bearing is supplied with less C. annealing of co_ntact with the mating tooth to
oil than full bearing D. tempering the pitch point of a gear:
B. the bearing encloses less than 360
A. arc of reces.,s
deg of the journal * 44. The term used in referring to the B. arc of relief
C. the bearing is slightly loaded continuous increase in the strain, or C. arc of action
D. the bearing is supplied with oil deformation, of any material D. arc of approach "
intermittently subjected to stress.
A. elasticity 50. Which of the following steel
38. For a completely corrosion B. creep • contains chromium?
resistant sta~at C. tolerance A. SAE 2340
mmrm-ur;:lpercilllm of chromium in D. all of the above
B. SAE 1230
~y lS required?
C. SAE 4230
A. 3% 45. Poison's ratio is the ratio of the: D. SAE 5240 *
B.11% * A. unit lateral deformation to the unit
C. 7% longitudinal defonnation *
D. 15% B. unit stress to unit strain
C. elastic limit to proportional limit
39. The amount by which the width of D. shear strain to compressive strain
a tooth space exceeds the thickness
of an engaging tooth on the pitch 46. At relatively high temperatures
circles of a gear: and low rateS4ff' strams, s~s
A. clearance wrll peiio.rm...b1ill_er if the material is:
B. backlash * A. fine-grained - -
C. tip relief B. their behaviour is independent of
D. chordal th ickness grain
C. course grained *
40. Steels containing large amounts D. all of the above
of mild nickel and chromium:
A. carbon steel

REFRESHER MANUAL 2nd Edition by JAS TORDILLO REFRESHER MANUAL 2nd Ecli_ti-oh by JAS TORDILLO.
Day 50 - EXAM I 50 - 1
.Machine Design
7. The narrow surface followed along
DAY 50- EXAM the groove that gives the size of the drill
and keeps it straight/aligned.
1. Reamers are made of the following A. web
part: body, shank and _ _ __ B. tang
A. tang C. margin
B. angle of chamber D. shank
C. point
D. flute 8. Different forces that can be combined
are called----:--:-----:.
2. Rack gears have identical A. concurrent forces
term/definition as to the of the B. couple
spur gear. C. component forces
A. pitch circle D. parallel
B ..addendum
C. pitch diameter 9. equal in m&gnitude but
D. clearance opposite in direction and parallel called
couple.
3. It Is Nickel steel type is AISI or SAE A. momentum
designation: B. acceleration
A. 15xx C. forces
B. 40xx D. energy
C. 33xx
D. 23xx 10. The three main parts of a screw
driver are: handle, blade and _ _ __,_
4. Compute the tooth circular thickness A. snap
of the pitch circle of a 14 Y. degree full i. head
depth tooth of 15 diametral pitch. C. shank
Material used is aluminum bronze. D. grip
A. 0.1047
B. 0.0147 11 . For easy drilling, a
C. 0. 1074 punch is applied to have the indentation.
D. 0'.0174 The tip of this tool should always be
ground to 90 deg angle and made out of
5. of the par:ts is the boundary hardened steel material.
that separates the object from another A. drill
object or space. B. prick
A. waviness C. center
B. surface D. drift
C. texture
D. roughness 12. SAE 1215 belongs to the
steel family.
6. A lathe machine used in A. carbon
manufacturing and with several cutting B. nickel
tools. This is a mass production lathe. C. manganese
A. turret D. chrome
B. all of these
C. spindle 13. Pipe color code for water.
D. automatic A. blue
B. natural

REFRESHER MANUAL 2nd Edition by JAS TORDILLO


.--
50 - 2 I Day 50 - EXAM Day 50- EXAM !50- 3
-
Machine Design
C. green C. floor work
D. white D. table work unknown speed of the gear if the power 31. The section modulus of 8 em x 15
tra nsmitted is 15 kw and tooth thickne~s em rectangular beam, in cm2, is closest
14. Steel is the generic term of variety 20. Poisson's ratio is the ratio of of 20 mm. to:
of: A. lateral deformation of the longitudinal A. 1000 rpm A. 150
A. AISI metal deformation B. 90 rpm B.400
B. iron-carbon alloy B. shear to yield strength C. 100 rpm C. 300
C. big iron C. elasticity limit to shear strength D. 45 rpm D.600
D. SAE metal D. unit strain to unit strain
27. A steel rod 3 em in diameter and 80 32. What will be the elongation of the 1-
15. Working stress in which a machine 21. Cutting lubricants used in drilling, em long has an allowable elongation not inch round steel bar 8 ft long of
member is subjected exceeds the reaming and tapping of cast iron parts. to exce~d 1.5 mm, find the allowable subjected to a load of 12,250 lbs?
ultimate strength of the said member. To A turpentine load in kN. A. 0.01 inch
avoid this situation, a designer, based B. soda water A. 0.274 B. 0.05 inch
on his experience and/or known C. soluble oil B. 0.316 C. 0.005 inch
engineering standards, should employ D. dry C.)74 D. 0.25 inch
D. 316 .
A. ultimate strength 22. Gear tooth cur inside a cylinder or 33. A steel . specimen is 3/8 inch
B. factor of safety ring is called gear. 28. ·A horizontal beam 20 ft long is diameter at the root of the thread. It is to
C. yield strength A. ninter subjected to a load of 1000 lbs located be stressed to 40,000 psi tension . What
D. fatique strength B. ring at its center. The dimension of the beam load, in lbs, must be applied?
C. rack is 3 x 6 inches, respect[vely and its unit A.4418
16. Forces acting on a single pint are D. internal weight is 150 lb/ft. Find the reactions at B.4481
called forces. both ends. C. 4814
A. kinematic 23. The angle of action to the pitch A. 200 lbs/200 lbs D. 8441
B. kinetic angel ratio in a gear operation is defined B. 2000 lbs/2000 lbs
C. vector as _ __ C 1000 lbs/1 000 lbs 34. A cantilever beam is 5 ft long and of
D. concurrent A. gear/pinion D. 3000 lbs/3000 lbs square cross-section, 2 inches on each
B. contract ratio \ ide, is loqded at its end by a vertical
17. For a simply supported steel beam C. module 29. A horizontal beam 20 ft long is force of magnitude 500 lbs. Determine
with load at the midpoint, the moment D. arc of action subjected to a load of 1000 lbs located the magnitude of the reaction at the
diagram shows in the form of a _ _. at its cente r. The dimension of the beam fixed end.
A. parabola 24. Anvil, hammer, burners are useful in is 3 x 6 inches, respectively and its unit A . 100 lbs
B. triangle shop operation. weight is 150 lb/ft. Determine the radius B. 500 lbs
C. semi-circle A. pattern of gyration of the beam . C. 250 lbs
D. none of these B. machine A. 1 inch D. 1000 lbs
C. foundry B. 2 inches
18. This refers to the phenomena of D. forge C. 1.732 inch 35. A cantilever beam is 5 ft long and of
continuous stretch load even if the D. 3.512 inches square cross-section, 2 inches on each
stress is less than the material yield. 25. Fire of ordinary materials such as side, is loaded at its end by a vertical
A. elasticity cloth, papers, rubber, plastic etc. is a 30. A horizontal beam 20 ft long is force of magnitude 500 lbs. Find the
B. ductility class _ _ fire. subjected to a load of 1000 lbs located a radius of gyration of the beam.
C. creep A Class C t its center. The dimension of the beam A. 1 inch
D. plasticity B. Class A is 3 x 6 inches, respectively and its unit B. 1.33 inch
C. Class B weight is 150 lb/ft. Find the flexural C. 0 .576 inch
19. Where laying out, assembly and D. Class D stress of the beam. D. 2inches
final fitting work on parts is being A. 7333 'psi
worked out specially on handy parts, 26. A 120 tooth gear 14 1-1 degree B. 9333 psi 36. A cantilever beam is 5 ft long and of
etc. involute pressure angle of unknown C. 8333 psi square cross-section, 2 inches on each
A bench work speed meshed with a 36 tooth pinion D. 3333 psi side, is loaded at its end by a vertical
B. laboratory turning at 300 rpm. Determine the force of magnitude 500 lbs.

REFRESHER MANUAL 2nd Edition by JAS TORDILLO REFRESHER MANUAL 2nd Edition by J_AS TORDILLO

....,_ II
50 - 4 I Day 50 - EXAM Day 50 - Solution I 50 - 5
Machine Design
A. 22,556 psi steel plate. The design stress of steel
B. 26,556 psi plate is 8500 psi.
C. 25,556 psi A. 80 tons
C. 0.37 in DAY 50 - SOLUTION
D . 0.65 in
D. 35,556 psi B. 20 tons
C. 65 tons 1. Reamers are made of the following
48. In designing a cylindrical pressure part: body, shank and _ _ __
37. Determine the angular deflection in D. 10 tons tank 3 feet in diameter, a factor of safety
degrees of a SAE 1040 steel shaft in a A. tang
of 2.5 is used. The cylinder is -made of
length of Y. meter. The shear stress is 43. A 100 inches long aluminum bar is B . angle of chamber •
steel having a yield point of 30 ksi, and
69 Mpa, shaft diameter is 62 mm and subjected to a tensile stress of 25,000 C. point
will contain pressures up to 1000 psi.
steel modulus of elasticity is 79.3 Gpa. psi. Find the elongation if E = 10 x 106 D. flute
What is the required wall thickness t?
A. 0.08 psi.
B. 0.1 A. 0.75 in 2. Rack gears have identical
A. 0.025 in
B. 3in term/definition as to the of the
C. 0.01 B. 0.45 in
D. 0.8
C. 1.5 in spur gear.
C. 0.25 in
D. 3.75 in A . pitch circle
D. 0.§5 in
38. Determine the power developed by B. addendum •
49. A car starts from rest and moves
the shaft rotating at 1200 rpm and the 44. A'6 inches diameter rivet undergoes 2 C. pitch diameter
with a constant acceleration of 6 m/s .
twisting moment of 1050 in-lb. Shear a shear force of 1750 lbs. Find the D. clearance
What is the speed of the car after 4
stress of the shaft is 6500 psi. average shear stress in the rivet.
A. 10 hp seconds? 3. It is Nickel steel type is AISI or SAE
A. 15.5 psi
B. 18 hp A. 18 m/s
B. 42.7 psi designation:
C. 15 hp B. 35 m/s A .. 15xx
C. 37.0 psi
D. 20 hp b. 61.9 psi C. 24 m/s B. 40xx
D. 55 m/s C. 33xx
39. A short 6.1 em diameter shaft 45. ('.. copper column of annular cross D. 23xx *
1 50. A ball is thrown vertically upward
transmits 120 hp. Compute the linear section has an outer diameter of 15 ft
with an initial speed of 80 ft/sec. How 4. Compute the tooth circular thickness
speed of a pulley 55 em diameter and is subjected to a force of 45,000 lbs.
mounted on the shaft. long will it take for the ball to return to of the pit.ch circle of a 14 Y. degree full
The allowable compressive stress is 300
A. 1870 fpm lb/ff. What would be the wall thickness? the thrower? 'tlepth toottt. of 15 diametral pitch.
B. 1780 fpm A. 3ft A. 2.25 sec Material used is aluminum bronze.
C. 1807 fpm B. 4.59, ft B. 4.06 sec A. 0.1047 *
D. 1807 fpm C. 3.52 ft C. 2.62 sec B. 0.0147
D. 5.03 ft
D. 4.97 sec C. 0.1074
40. Compute the working strength of a D. 0.0174
bolt having a diameter of 1 inch under a 46. A specimen ~- subjected to a load.
tensile stress of 4000 psi. When the load is removed, the strain
Tooth thickness, t = 1.5708
OAY 50 - ANSWER KEY pd
A. 1000 lbs disappearS. Which of the following can 1. 8 16. 0 31.C46. A
B. 1800 lbs be deduced about this material? 2. 8 17. 8 32. 8 47. C
- 1.5708 -
- _1_5-=0.1047
C. 1200 lbs A . it is elastic 3. 0 18.C 33.A48. C
D. 2500 lbs B. it is ductile 4. A 19. A 34. 8 49. C
5. 8 20. A 35. C 50. 0
C. it is plastic 5. of the parts is the boundary
6. A 21. 0 36. A
41 . A double threaded bolt having a 5 D. it has a high modulus of elasticity 7. C 22. 0 37. 0 that separates the object from another
threads per inch. Compute the lead, in 8. C 23. 8 38. 0 object or space.
inches, of a screw in one turn. 47. Find the elongation of an 8 inches 9. C 24. C 39. A A. waviness
A . 0.2 diameter, 100 inches long aluminum bar 10.C 25. 8 40. C
11.C 26 . 8 41.C B. surface"
B. 0.5 when loaded to its yield point 37 ksi. 12.A 27. A 42. 8 C. texture
C. 0.4 Modulus of elasticity is 10 x 106 psi. 13.C 28. 8 43. C D. roughness
D. 1.0 Neglect the weight of the bar. 14.8 29. C 44. 0
A. 0.25 in 15.8 30. C 45. 8 6. A lathe machine used in
42. What pressure is required to punch B. 0.43 in manufacturing and with several cutting
a hole 1 inch diameter through a ~ inch tools. This is a mass production lathe.

REFRESHER MANUAL 2nd Edition by JAS TORDILLO REFRESHER MANUAL ~nd Edition by JAS TORDILLO
50 - 6 1 Day 50 - Solution Day 50 - Solution 150 - 7
A. turret~ 13. Pipe color code for water.
Machine Design
B. all of these A. blue
C. spindle w orked out specially on handy parts, 26. A 120 tooth gear 14 Y:. degree
B. natural involute pressure angle of unknown
D. automatic etc.
C. green*
A. bench work * speed meshed w ith a 36 tooth pinion
D. white
B. laboratory turning at 300 rpm. Determine the
7. The narrow surface followed along
C .. floor work unknown speed of the gear if the power
the groove that gives the size of the drill 14. Steel is the generic term of variety
D. table work transmitted is 15 kw and tooth thickness
and keeps it straight/aligned. of:
A web of20 mm.
A. AJSI metal
20. Poisson's ratio is the ratio of A. 1000 rpm
B. tang B. iron-carbon alloy *
A. lateral deformation of the B. 90 rpm"
C. margin* C . big iron
D. shank longitudinal deformation * C. 100 rpm
D. SAE metal
B. shear to yield s.trength D. 45 rpm
8. Different forces that can be combined
C. elasticity limit to shear strength
15. Working stress in which a machine
D. unit strain to unit strain T,N, = T2N2
are called - - - - - - ' member is subjected exceeds the
36(300) = 120 N2
A. concurrent forces ultima'te strength of the said member. To
21 . Cutting lubricants used in drilling, N2 = 90 rpm
B. couple avoid this situation; a designer, based
reaming and tapping of cast iron parts.
C. component forces " on his experience and/or known
A. turpentine 27. A steel rod 3 em in diameter and 80
D. parallel engineering standards, should employ
B. soda water em long has an allowable elongation not
C. soluble oil to exceed 1.5 mm, find the allowable
9. equal in magnitude but A. ultimate strength
D. dry* load in kN.
opposite in direction and parallel called B. factor of safety * A. 0.274 *
couple. C. yield strength
22. Gear tooth cut inside a cylinder or B. 0.316
A. momentum D. fatique strength
B. acceleration
ring is called gear. c. 274
A. ninter D. 316
C. forces* 16. Forces acting on a single pint are
D. energy B. ring
called forces.
C. rack E = 30 X 106 psi = 206,786 kPa for steel
A. kinematic
D. internal * FL'
10. The three main parts of a screw
driver are: handle, blade and----'
A. snap
B. kinetic
C. vector
D. concurrent ,. 23. The angle of action to the pitch
'=- AE
angel ratio in a gear operation is defined = F(800)
B. head 15
C. shank • 17. For a simply supported steel beam
as _ _.
. ~ (o.03)2~o6,786xl0 3 )
. A. gear/pinion 4
D. grip with load at the midpoint, the moment
diagram shows in the form of a _ _.. B. c'ontract ratio • F = 274 N = 0.274 kN
C. module
11. For easy drilling, a A. parabola
B. triangle * D. arc of action 28. A horizontal beam 20 ft long is
punch is applied to have the indentation.
The tip of this tool should always be C. semi-circle subjected to a load of 1000 lbs ~ocated
D. none of these 24. Anvil, hammer, burners are useful in at its center. The dimension of the beam
ground to 90 deg angle and made out of
shop operation. is 3 x 6 inches, respectively and its unit
hardened steel material.
18. This refers to the phenomena of A. pattern weight is 150 lb/ft. Find the reactions at
A. drill
continuous stretch load even if the B. machine both ends.
B. prick
stress is less than the material yield. C. foundry* A. 200 lbs/200 lbs
C. center*
A. elasticity D. forge B. 2000 lbs/2000 lbs *
D. drift
B. ductility c. 1000 lbs/1000 lbs
C. creep* 25. Fire of ordinary materials such as D. 3000 lbs/3000 lbs
12. SAE 1215 belongs to the
D. plasticity cloth, papers, rubber, plastic etc. is a
steel family.
class __ fire.
A. carbon * 1000 + 150(20) = 2000 lbs
B. nickel 19. Where laying out, assembly and A. Class C R1 =R2 = 2
final fitting work on . parts is being B. Class A*
C. manganese
C . Class B
D. chrome
D. Class D
REFRESHER MANUAL 2nd Edition by JAS TORDILLO REFRESHER MANUAL 2nd Edition by JAS TORDILLO
50 - 10 I Day 50 - Solution Day 50 - Solution 1 50 - 11
Machine Design
41. A dquble threaded bolt having a 5 F
threads per inch. Compute the lead, in Ss = -
(~)
inches, of a screw in one turn . A 32.2 = 0-80
y = 37,000
A. 0.2 s. = 1750 10x l06 t
B. 0.5 t = 2.48 sec
~(6)2 Y = 0.37 in = = 4.97 sec
c. 0.4 * 4
total time 2(2.48)
D. 1.0 s. = 61.9 psi 48. In designing a cylindrical pressure
tank 3 feet in diameter, a factor of safety
I 45. A copper column of annular cross of 2.5 is used. The cylinder is -made of
p .:: -= 0.2
no. of threads per inch 5 section has an outer diameter of 15 ft steel having a yield point of 30 ksi, and
and is subjected to a force of 45,000 lbs. wi ll contain pressures up to 1000 psi.
The allowable compressive stress is 300 What is the required wall thickness t?
L = 2P for double threaded
lb/ft2. What would b'e the wall thickness? A. 0.75 in
L =
2(0.2) = 0.4
A. 3ft B. 3 in
42. What pressure is required to punch
B. 4.59 ft * C. 1.5 in*
C. 3.!}2 ft D. 3.75 in
a hole 1 inch diameter through a Y. inch
D. 5.03 ft
steel plate. The design stress of steel
plate is 8500 psi. PO
F F St = -
A. 80 tons Sc = 2t
B. 20 tons*
C. 65 tons A= ~~5-o~] 30.000 = !000(3xl2)
D. 10 tons 2.5 2t
45,000 t = 1.5 in
300
p.:: d X t X 80
p = 1(Y.)(80)
~ [(15)2 - D f ] 49. A car starts from rest and moves
2
P = 20 tons D = 5.83 ft with a constant acceleration of 6 m/s .
15 5 83 What is the speed of the car after 4
43. A 100 inches long aluminum bar is Wall thickness, t =
. - 2 · = 4.585 ft seconds?
subjected to a tensile stress of 25,000 A 18 m/s
psi. Find the elongation if E = 10 x 106 B. 35 m/s
46. A specimen is subjected to · a load. C. 24 m/s •
psi.
When the load is removed, the strain D. 55 m/s
A. 0.025 in
disappears. Which of the following can
B. 0.45 in
be deduced aboutihis material?
c. 0.25 in* A. it is elastic * a = Vz- V 1
D. 0.65 in t
B. it is ductile
L C. it is plastic VJ- 0
Y= s - 6 = ---
E D. it has a high modulus of elasticity 4
v2
y = 25,000 (~) 47. Find the elongation of an 8 inches
= 24 m/s
IOxiO diameter, 100 inches long aluminum bar 50. A ball is thrown vertically upward
Y = 0.25 in w hen loaded to its yield point 37 ksi. with an initial speed of 80 ft/sec. How
Modulus of elasticity is 10 x 106 psi. long will it take for the ball to return to
44. A 6 inches diameter rivet undergoes Neglect the weight of the bar. the thrower?
a shear force of 1750 lbs. Find the A. 0.25 in A. 2.25 sec
average shear stress in the rivet. B. 0.43 in B. 4.06 sec
A. 15.5 psi C. 0.37 in* C. 2.62 sec
B. 42.7 psi D. 0.65 in D. 4.97 sec*
C. 37.0 psi
D. 61.9 psi * L
Y=Sy- g = V2- VI
E l
REFRESHER MANUAL 2nd Edition by JAS TORDILLO. REFRESHER MANUAL 2nd Edition by J.AS TORDILLO
.-- --
Day 51 • EXAM I 51 • 1
Machine Design
7. Recommended hardness of pinion for
DAY 51 -EXAM helical/herringbone gear tooth should be
BHN point to sustain life.
1. It is ideal for maximum quietness In A. 50-59
sprocket drive operation to choose B. 40-50
_ _ _ or more teeth. c. 48-65
A. 25 D. 30-60
B.27
c. 26 8. Fast process of analyzing all
D. 28 elements and chemical components of
casting:
2. Deals only with the motion of bodies A. pyrometer
without reference to the forces that B. wet analyzer
cause them: C. carbon analyzer
A. kinetics D. spectrometer
B. motion
C. kinematics 9. In small quantity this alloy is effective
D. acceleration for improving strength at temperature.
A. chromium
3. The shaft that is used to connect or B. selenium
disconnect the drive at will is called: ·c. manganese
A. clutch D. molybdenum
B. brake shaft
C. bearing 10. The ratio of the arc of action to
D. lock shaft circular pitch or length of action to the
base pitch.
4. The distance from a point on a screw A. approach ratio
thread to a corresponding point on the B. arc of action
next measure along the axis: C. contact ratio
A. radial D. arc of approach
B. thickness
C. module 11. Which of the following Is not a kind
D. pitch of mandrel?
A. expanded
5. Martensite (stainless steel) contains 4 B. contractio·n
to 26% Cr and a maximum of _ _ _. C. extended
%nickel. D. taper
A. 1.5
B. 2.5 12. A car is travelling 120 kph, it takes
C. 3.5 0.5 sec to apply the brake after which it
D. 4.5 decelerates at 8 m/s2 Compute the
distance it traveled in meters before it
6. Progression change of position of a stops.
body is called: A.15.6
A. acceleration B. 64.8
B. force c. 26.5
C. motion D.69.4
D. momentum
13. All four compression coil spring
support one load of 700 kg. All four

REFRESHER MANUAL 2nd Edition by JAS TORDILLO

~ . _,.;!: II
51 - 2 I Day 51 - EXAM Day 51 - EXAM I 51 - 3
Machine Design
springs are arranged in parallel and and its foundation by at least _ __
rated same at 0.609 kg/mm . Compute mm. 26. What pressure is required to punch A 2in
the deflection in mm. A. 30 five holes 3 inches in diameter through a B. 2% in
A. 64 B.25 Y. inch steel plate? C.2% in
B. 178 C. 20 A. 150 tons D. 3 Y. in
C. 147 D. 36 B. 250 tons
D. 287 C. 300 lbs 32. Find the tooth thickness on the
21. Grout material use under the D. 300tons tooth circle of 6 20 degree full depth
14. A section in a machine shop machine bed and foundation surface involute tooth having a circular pitch of
operation. should be: 27. A flywheel rotates at . 220 rpm 1.25 in and whole depth of 0.75 inch.
A coremaking A 1 part cement and % parts screened slowed down to 65% of its revolution A. 10mm
B. fitting sand and Y. part gypsum during the three-fourth second punching B. t4 mm
C. machining B. 1 part cement and 1 part screened portion of the cycle. Compute the C. 12mm
D. pattern sand angular acceleration of the flywheel in· D. 16mm
C. 1 part cement and I part screened rad per second squared.
15. Shear modulus is also: sand and Y. part gypsum A. 10.7 33. D~termine the speed of the milling
A shear elasticity D. 1 part cement and I % part screened B. -10.7 cutter with diameter-.of 5 inches and a
B. modulus of elasticity sand and % part stone C. 7.10 cutting speed of 400 fpm.
C. poisson's ratio D. -7. 10 A. 215 rpm
D. modulus of rigidity 22. A gear set having a gear ratio of 3 B. 285 rpm
is to be used at a center distance of 16 28. Compute the diameter of the shaft C. 260 rpm
16. To prevent leakage in dynamic seal: inches. If the gear has 80 teeth , what receiving power from a driving motor D. 306 rpm
A. gaskets must be the circular pitch? that transmit 250 kw at 900 rpm.
B. packings A 21 .6 mm A. 76.2mm 34. Given a shear stress of 10,000 psi,
7
C. seals B. 29.3 mm B. 3%in and a shear modulus G = 1. 15 x 10
D. felts C. 23.6 mm C. 2 3/4 in psi, find the shear strain.
D. 32.9 mm D. 74.2mm A. 8.69 X 104
B. 9.68 x 1o·
4
17. The frictional force depends on
coefficient of friction and: 23. A body weighing 1000 lb falls 6 29. A flywheel has a diameter of 1 m, C. 8.96 X 10'~
A torque inches and strikes a 2000 lb per in and a mass of 500 kg . What torque Q. 9.96,X 10·~
B. normal force spring. The deformation of the spring is: needed to produce an angular
C. weight of the object A. 1 in acceleration of 120 rev per second per 35. Determine the linear speed of a
D. moment B. 3 in minute? pulley 100 em mounted on a 50.8 mm
C. 2 in A. 785 J short shaft transmits 100 HP.
18. Not part of the headstock: D. 6 in B. 735 J A 4295 fpm
A. back gear C. 758 J B. 4895 fpm
B. anvil 24. For a single universal joint, shaft D. 620 J C. 4375 fpm
C. spindle angle should be degrees D. 5460 fpm
D. motor maximum and much less if on high 30. Compute for the ·load in kN on a
19. Two or more forces acting together rotational speed. 7.62 em diameter, 150 em long steel rod 36. What is the working strength of a 3
could be replaced by a single force with A. 12 if its maximum elongation will not inches bolt which is screwed up tightly
same effect in a mass. B. 15 exceed 1.5 mm. with an allowable working stress of
A. couple of forces c. 16 A 943 12,000 psi?
B. resultant D. 14 B. 1050 A . 45,406 lbs
C. resolution of forces C.995 B. 25.2 tons
D. concurrent of forces 25. The recommended lubricant for the D. 1450 C. 49,800 lbs
chain drive operation. D. 27.5 1bs
20. To prevent engi ne vibration to A. SAE 4140 31. Compute the shaft diameter to
affect the surrounding area, it is B. moly slip transmit 30 HP at 300 rpm with to 37. A helical spring having squared and
standard practice to isolate the engine C. Petroleum oil torsional deflection of 0.1 1 deg per ft ground ends has a total of 24 coils
D. heavy grease length. material modulus of elasticity in shear of

REFRESHER MANUAL 2nd Edition by JAS TORDILLO REFRESHER MANUAL 2nd Edition by JAS TORDILLO

.... Ill
51 - 6 1Day 51 - Solution Day 51 -Solution 151 - 7
Machine Design
A. 50-59
DAY 51 - SOLUTION B. 40-50 * 13. All four compression coil spring 19. Two or more forces acting together
C. 48-65 support one load of 700 kg. All four could be replaced by a single force with
1. It is ideal for maximum quietness in D. 30-60
sprocket drive operation to choose springs are arranged in parallel and.., same effect in a mass.
_ _ _ or more teeth. rated same at 0.609 kg/mm. Compute A. couple of forces
8. Fast process of analyzing all the deflection in mm. B. resultant "
A. 25 elements and chemical components of
B. 27 * A. 64 C. resolution of forces
casting: B. 178 D. concurrent of forces
C. 26 A pyrometer
D. 28 c. 147
B. wet analyzer D. 287 * 20. To prevent engine vibration to
C. carbon analyzer affect the surrounding area, it is
2. Deals only with the motion of bodies D. spectrometer * Y = Y1 = Y2 = Y~ = Y4 standard practice to isolate the engine
without reference to the forces that
700 and its foundation by at least _ _ __
cause them: 9. In small quantity this alloy is effective F1 = - = 175 kg
A. kinetics mm.
for improving strength at temperature. 4
B. motion A. 30
C. kinematics •
A. chromium "
B. selenium
Y = .!i =~ = 287 .3 mm "' B. 25~
K .0.609 C. 20
D. acceleration C. manganese D. 36
D. molybdenum 14. A section in a machine shop
3. The shaft that is used to connect or
operation. 21 . Grout material use under the
disconnect the dFive at will is called: 10. The ratio of the .arc of action to A . coremaking machine bed and foundation surface
A. clutch • circular pitch or length of action to the B. fitting should be:
B. brake shaft base pitch. C. machining • A. 1 part cement and Y2 parts· screened
C. bearing A. approach ratio D. pattern sand and Y. part gypsum
D. lock shaft B. arc of action B. 1 part cement and 1 part screened
C. contact ratio • 15. Shear modulus is also: sand*
4. The distance from a point on a screw D. arc of approach A. shear elasticity C. 1 part cement and I part screened
thread to a corresponding point on the
B. modulus of elasticity sand and Y. part gypsum
next measure along the axis: 11 . Which of the following is not a kind C. poisson's ratio D: 1 part cement and I Y2 part screened
A . radial of mandrel? D. modulus of rigidity • sand an,d Y2 part stone
B. thickness A. expanded
C. module B. contraction * 16. To prevent leakage in dynamic seal: 22. A gear set having a gear ratio of 3
D. pitch* C. extended A. gaskets is to be used at a center distance of 16
D. taper B. packings * inches. If the gear has 80 teeth , what
5. Martensite (stainless steel) contains 4
C. seals must be the circular pitch?
to 26% Cr and a maximum of _ _ _. 12. A car is travelling 120 kph, it takes D. felts A. 21.6 mm
%nickel. 0.5 sec to apply the brake after '.Yhlch it B. 29.3 mm
A . 1.5 decelerates at 8 m/s2 . Compute the
B. 2.5* 17. • The frictional force depends on C. 23.6mm *
distance it traveled in meters before it coefficient of friction and: D. 32.9 mm
C. 3.5 stops.
D. 4.5 A torque
A 15.6 B. normal force *
B. 64.8 T2 =3
C. weight of the object
6. Progression change of position of a c. 26.5 T,
body is called: D. moment
D. 69.4 *
A. acceleration 80 =3
18. Not part of the headstock:
B. force v1 =120 kph = 33.33 m/s A. back gear
Tt
C. motion* V2=0 T1 = 26.67 say 27
D. momentum B. anvil*
C. spindle Dz =3
7. Recommended hardness of pinion for
V2
2
=V/ + 2as D. motor D,
0 = (33.33) 2 + 2(-8)S
D2 = 3D1
helical/herringbone gear tooth should be
BHN point to sustain life..
=
S 69.4 m

REFRESHER MANUAL 2nd Edition by JAS TORDILLO REFRESHER MANUAL 2nd Edition by JAS TORDILLO

L
- ~

51 - 8 I Day 51 - Solution Day 51 - Solution 1·51 - 9


Machine Design
c = o, + 02 26. What pressure is required to punch
2 five holes 3 inches in diameter through a
acceleration of 120 rev per second per 32. find the tooth thickness on the
Y. inch steel plate? tooth circle of 6 20 degree full depth
16 = Dt+30 1 minute?
A. 150 tons
2 A. 785 J • involute tooth having a circular pitch of
B. 250 tons
B. 735 J 1.25 in and whole depth of 0.75 inch.
D, =8 in C. 300 lbs
C. 758 J A.10mm
D. 300 tons*
D. 620 J B. 14 mm
_ nD 1 = n(8) = 0. 9308 in= 23.6 mm C. 12 mm
Pc - T 27 P=dxtx60x5
1 p = 3(Y. )(80)(5)
a = 120 r~v/sec-niln = 12.56 rad/sec2 0.16 mm •
P = 300 tons
23. A body weighing 1000 lb falls 6 T =I x a = Y2 m~ x a Po.X Pc = TI

inches and strikes a 2000 lb per in T = Y2 (500)(0.5)~(12.56) PdX(1.25)= TI


27. A flywheel rotates at 220 rpm
spring. The deformation of the spring is: slowed down to 65% of its revolution
T =785 J pd = 2~1
A. 1 in during the three-fourth second punching t = 1.~708 = 1.5708
B. 3 in" 30. Compute for the load in kN on a .p__d .2.51
po~lon of the cycle. Compute the
C. 2 in 7.62 em diameter, 150 em long steel rod = 0.6258 in =15.9 mm
angular acceleration of the flywheel in
D. 6 in if its maximum elongation will not
rad per second squared.
A. 10.7 exceed 1.5 mm. 33. Determine the speed of the milling
A. 943 * cutter with diameter of 5 inches and a
k= .£.y B. -10.7 *
C. 7.10 B. 1050 cutting speed of 400 fpm.
D. -7. 10 c. 995 A. 215 rpm
F D. 1450
2000 = - B. 285 rpm
y
V, = 220 rpm 23 rad/sec C. 260 rpm
F = 2000y E = 30 x 106 psi.= 206,786 MPa for steel
v2 = 23(0.65) = 14.95 rad/sec FL 1

D. 306 rpm*
y =-
(~) AE
a= v2 -v1 = 14.95-23 V =nON
W(h+y)=F
l
= -10.73 rad/secf
0.75 t
5 = F(t soo) 400 = n
.
(2)N 12
1000(6 + y) = 2000 ( ~) . !: (0.0762)2 (206,786x0 6 )
4 ~ = 305.6 rpm
28. Compute the diameter of the shaft F = 0.943 N = 943 kN
/-y -6=0 receiving power from a driving motor 34. Given a shear stress of 10,000 psi
(y- 3)(y + 2) = 0 that transmit 250 kw at 900 rpm. 7
31. Compute the shaft diameter to and a shear modulus G = 1. 15 x 10
y = 3in A. 76.2 mm" transmit 30 HP at 300 rpm with to psi, find the shear $train.
B. 3 Y2 in torsional deflection of 0.11 deg per ft A. 8.69 10-4.
24. For a single universal joint. shaft c. 2 3/4 in length.
X
B. 9.68 X 10"4
angle should be degrees D. 74.2mm A. 2 in C. 8.96 X 10-4
maximum and much less if on high B. 2% in D. 9.96 X 10-4
rotational speed. P = 250 kw = 335.12 hp C. 2% in*
A. 12 o3N D. 3 Y. in
8.15 * p = -- . stress_ 10,000 = 8.69 x 10-4
80 strain = - E-- 1.1Sxt07
C. 16 P = 2nTN
D. 14
335.12 = o3(9oo) 30(33,000) = 2nT(300)
35. Determine the linear speed of a
80 T = 525.21 ft-lbs
25. The recommended lubricant for the pulley 100 em mounted on a 50.8 mm
chain drive operation. D = 3 in = 76.2 mm e= TL short shaft transmits 100 HP.
A. SAE 4140 JG A. 4295 fpm
29. A flywheel has a diameter of 1 m,
B. moly slip
and a mass of 500 kg. What torque 0. 110 = (~) = 6302.53(12) B. 4895 fpm *
C. Petroleum oil * C. 4375fpm
D. heavy ·grease needed to produce an angular 180
~- o 4 (t2x·to6 ) D. 5460 fpm
32
D = 2.4 in use 2 Y2 in shaft
REFRESHER MANUAL 2nd Edition by JAS TORDILLO REFRESHER MANUAL 2nd Edition by JAS TORDILLO

kL Ill
51 -10 1 Day 51 -Solution Day 51 - Solution I 51 - 11
Machine Design
D = 50.8 mm = 2 in 38. With an electric arc welding rate of
D3 N 20 inches per minute, how long will it
p = --
38
take to weld a Y. inch thick plate by 4 ft
long seam ..?
F ;:: __!__=
D/
/2
1273 24
· = 42.441.31 N
0.060/
/2
D = 4.6 4 J!{!-
100 = (2)3N A. 98 sec
38
B. 135 sec
C. 110 sec
W = 3/8 in = 9.525 mm
7.54=4.64~HP
360
N = Np =475 rpm F
D. 144sec * =- HP = 2602
DP =100 em= 3.28 ft Ss
wL
V =nDpNp .
t 1me=
length of cut 70 = 42,441.31 43. The 2/5 of 3/16 is equal to:
V = n (3.28)(475) cutting rate 9.525L A_ 3/80
V = 4894.6 fpm L ;:: 63.66 mm 8 0:074
4 12 c. 3140*
time= ( ) = 2.4 min= 144 sec
36. What is the working strength of a 3 20 41. A spur pinion rotates at 1800 rpm D. 0.076
inches bolt which is screwed up tightly transmits 900 kg of tangential load.
with an allowable working stress of 39. A hollow shaft has an outside Determine the power transmitted to the 2 ·-~ 6 3
diameter of 100 mm and wall thickness -x-=-=-
12,000 psi?
of 15 mm. Determine the polar moment
gear if the pitch diameter of pinion is 4 5 t6 ·so 40
A. 45,406 lbs inches.
B. 25.2 tons * of inertia of the hollow shaft.' Assume A. 95 HP 44. Two shafts are connected by a
C. 49,800 lbs shear stress of the material to be 5 B.113HP* flanged coupling. The coupling is
D. 27.51bs N/mm2 • C. 100 HP. secured by 6 bolts, 20 mm in diameter
A. 7.46 x 10'6 m4 * D. 220 HP on a pitch circle diameter of 150 mm. If
W =St[0.55d2 - 0.25d] B. 7.46 x 10' 5 m4
torque of 120 N-m is applied, find the
w = 12,000[0.55(3)2 - 0.25(3)] C. 7.64 x 10'5 m4 D = 4 in= 0.1016 m shear stress in the bolts.
= 50,400 lbs =25.2 tons D. 8.25 x 10'6 m4 Ft = 900 kg 1'980 lbs=
W A. 1.245 Nfmm2
B. 0.995 Nlmm2
37. A helical spring having squared and Do= 100mm=0.100m
0; = 100- 2(15)-= 70 mm = 0.070 m
~ =Ft ( ~) =198o(~) = 3960 in-lb C. 1. 115N/mm2
ground ends has a total of 24 coils D. 0.848 N/mm2 "
material modulus of elasticity in shear of 1t [
J = 32 Do- Di
4.:-> 4]
89 Gpa The spring has an outside P = 2-nTN T 120
diameter of 12 em and a wire diameter
of 0.65 em. Compute the maximum
4 4
J = ..2:..[(0.100) -(0.070) ]
P = 2n(330)(1800)
= 3,732,212 ft-lbs/min
F = D~ = 0 _15 %= 1600 N
deflection that can be produced in the 32 p = 113 hp
spring due to a load of 60 kg. J = 7.46 x 10-s m4 1600
Force per bolt, Fb = - = 266.67 N
A. 870 mm 42. A 20 ft steellineshaft has a bending . 6
B. 999 mm 40. Determine the length of a square action of pulleys What power in HP can
key for gear driven shaft transmits 40 kw _ Fb _ 266.67 _ O N/ 2
C. 954 mm * the shaft deliver at a speed of 360 rpm. S•b- - - - - - .8488 mm
D. 1030 mm at 300 rpm. The shaft diameter is 60 Consider that the torsional deflection will Ab ~(20)2
mm with design stress of 70 Mpa in not exceed 0.08 degree per ft length. 4
shear. the thickness and width of the A. 2600 *
c = ~= 12-0.65 = 17.46 key is 3/8 in. B. 1250 45. Two shafts are connected by spur
d 0.65 A. 43 mm C. 1100 gears. The pitch radii of gears A and B
3 B. 64mm* D.900 are 100 mm and 500 mm respectively. If
y = 8FC n
C. 56mm shaft A makes 800 rpm and is subjected
Gd
D. 72 mm Using an empirical formula from to a resisting torque of 113 N-m, what is
y = 8(60x9.81XI7.46)l(24-2) Machinery's Handbook: the torque in shaft 8?
9 P = 2nTN A. 375 N-m
89x10 (0.0065)
B. 565 N-m *
Y = 0.953 m = 953 mm 40 .000 = 2nT( 60 )
300
L W
= 5.2 3
C. 495 N-m
20= s.2 W
3 D. 690 N-m
T = 1273.24 N-m
D = 7.54 in
REFRESHER MANUAL 2nd Edition by JAS TORDILLO REFRESHER MANUAL 2nd Editton by JAS TORDILLQ
51 -12 I Day 51 -Solution Day 51 - Solution I 51 - 13
RANA = ReNa
Machine Design
overall speed reduction ratio Is 8 to 1,
~= NA = 500 = 5 power transmission efficiency is 80%.
~-~.!!..[ D~ - D, ]
4
RA N8 100 A. 5.0
B. 4.0
T 16 D0
PA = Pa
c. 3.3 * ...!:!.... =--:"----"-
D. 2.66 Ts Sstt DJ
TANA = TaNs
Ta =TA (~: )= 113(5) =565 N-m
Power a Q
p = kQ
TH
[
4
16 . 0
4
Dri- D, ]' . [(2Ro) - Rri
Do
.
2R0
l
46. Find the torsional moment (in N-mm) .!l=.£2
Ql Q2
Ts = DJ = (2RoP
developed when the shaft delivers 40.2 4 4.
HP at 260 rpm. -~ TH 16R0 - R0
A 901 x 10 to the 3 rd power 16,000 - ! (19)2(25)6 Ts = 16Rri
B. 1102 x 10 to the 2nd power 4
C. 901 X 10 P2 = 2.65 kw _TH_ = ~ = 93.75%
D. 1102 x 10 to ~he 3 rd power* Ts 16
Motor power required ·to drive with a
p = 40.2 hp = 29.9892 kw transmission efficiency of 80%: 50. A ste.el collar is to be shrunk into a
P = 2rrTN 2 65
= 0.80
· = 3.32 kw shaft. The hole in the collar is 199.8 mm
at 15°C and is required to be 200.8 mm
29 989.2
'
=2rrT ( 260
60
)
49. A hollow circular bar bas an inner
when heated so that it fits easfly over
the shaft. Find the temperature to which
radius r, and an outer radius r2 . If r, = Yz the collar must be heated for this
T =1101.44 N-m =1101.4 x 103 N-mm r2, what percentage of torque can the diameter to be obtained. (Coefficient of
shaft carry in comparison with a Solid linear expansion is €l.OOO 012 m/m-°C.
47. Compute the speed of the satellite to shaft?
orbit the earth at an elevation of 200 km A. 324°C #
A. 25% B. 234°C
Earth's radius is at 6400 km. Assume no
8.94%* C. 342°C
change of gravity with the elevation.
A. 8046.5 m/s *
c. 50% D. 432°C"
D. 75%
B. 9075.2 m/s
C. 8756.2 m/s =
y kl..6T
D = Do= outside· dia of hollow (dia of 200.8 -1 99.8 = 0.000012(199.8)(T2 -15)
D. 1056.5 m/s solid shaft) · T2 = 432°C
R; = inside diameter of hollow
Fe= W
Ro = outside diameter of hollow
mY 2
R=mg Ro = 2R;
Do= 2Ro
y2
7
( 6-40:-::0-+-2--=- oo~o = 9.81
ooT)J-=- D;=2R; =2 (~0 )=Ro
V = 8046.5 m/s Ss = 16T shear stress solid shaft
48. A multi-spindle drill press is used to 1tD3
drill six 19 mm hole simultaneously at
Ss = 16TDo (shear stress hollow
the, rate of 25 mm/min. If this kind of
steel requires 1 kw to remove 16,000
1t[ng- Dt]
3
mm of metal per minute. For a speed of shaft)
135 rpm; determine the motor power
required in kw to drive the system. The

REFRESHER MANUAL 2nd Edition by JAS TORDILLO REFRESHER MANUAL 2nd Edition by JAS TORDILLO
Day 52 - EXAM I 52 - 1
Machine Design
B. 149 kg
DAY 52 - EXAM c. 857 kg
D. 163 kg
1. Find the tooth thickness on the tooth
circle of a 20 degree full depth involute 7. If the ultimate shear strength of a
tooth having a circular pitch of 26.6 mm steel is 42,000 lb/in2 . The force to punch
and whole depth of 16 mm. a 4 in diameter hole in a % in steel plate
A. 0.52360 in is equal to:
B. 0.71681 in A 119,952 kg
C. 0.41361 in B. 263,894 kg
D. 0.89713 in C. 129,687 kg
D. 180,900 kg
2. What pressure is required for
punching 6 holes of 2 in diameter thru a 8. Two shafts are connected by spur
%.. inch steel plate? gears. The pitch radii of gears A and B
A. 300 tons are 100 mm and 700 mm, respectively.
B. 340tons If shaft A make ' s 1200 rpm and Is
C. 320 tons subjected to a resisting moment of 130
D. 480 tons . N-m. What is the torque in shaft B?
A. 910 N-m
3. Compute the maximum deflection of a B. 810 N-m
22 coils helical spring having a load of C. 710 N-m
120 kgs. The spring is squared and D. 890 N-m
ground ends with modulus of elasticity in
shear of 80 GPa, outside diameter of 8 9. A prismatic bar at 50°F is imbedded in
em and wire diameter of 8 mm. a rigid concrete wall. The· bar is 40
.. A. 22!:1 mm inches long and has a cross-'sectional
B. 16.5 min area of 4-m2. What is the axial force in
C. 245 mm the bar if its temperature is raised to
D. 214 mm 100°F? The coefficient of thermal
expansion is 5x10'6 m/m-°F.
4. What is the working strength of a 4 in A. 5000 lbf
bolt which is screwed up tightly in a B. 24,000 lbf
packed joint when the allowable working c. 8000 lbf
stress is 12,000 psi. D. 30,000 lbf
A. 82,500 lbs
B. 93,600 lbs 10. A pinion and a gear is carefully cut
C. 95,600 lbs in a 20 degree full depth design. The 16
D. 9§,000 lbs tooth steel pinion and 72 tooth cast-iron
gear transmit 50 kw power when the
5. Determine the strength of a Y. in bolt pinion turns 1400 rpm. The face width is
having a yield stress of 60 ksi and stress 90 mm and the module is 8. Find the
area of 0. 1134 in2 • center distance, in mm, between pinion
A. 3201bs and gear.
B.4001bs A. 352
C. 382 1bs 8.280
D.4201bs C. 380
D. 368
6. The total weight of steel plate size Y.
X 4 X 20.
A. 372 kg
REFRESHER MANUAL 2nd Edition by JAS TORDILLO
52 - 2 I Day 52 - EXAM
Day 52 - EXAM I 52 - 3
11 . A flywheel weighing 6 kN has a C. 0.25 in Machine Design
mean diameter of 90 em. The maximum D. 0.65 in
speed of the flywheel is 7 rev/sec 21. The following pertain to joining of C. tantalum
slowed down to 4 rev/sec during the 16. A drop hammer of 1 ton dead metals except: D. all of these
shearing process. Determine the energy weight capacity is propelled downward A. welding
released by the flywheel. B. casting 28. When steel casting are heat treated,
by a 12 in diameter cylinder. At 100 psi
A. 7856 kg-m air pressure. what is the impact velocity C. soldering the carbon content generally falls within
B. 9872 kg-m if the stroke is 28 inches? D. brazing the range of :
C. 8145 kg-m A. 63.2 fps A. 0.20%
D. 8241 kg-m B. 31.6 fps 22. It is considered semi-solid lubricant: B. 0. 15 to 0.20%
C. 15.8 fps A. lube oil C. 0.25 to 0.50%
12. In designing a cylindrical pressure D. 47.4 fps B. grease D. 0.60%
tank 3 feet in diameter, a factor of safety C. graphite
of 2.5 is used. The cylinder is made of· 17. A flywheel has a diameter of 2 m D. all of these 29. The most effective alloy in
steel (yield stress = 30 ksi), and will and a mass of 1500 kg. What torque is increasing toughness of steel:
contain pressures up to 1000 psi. What needed. to produce an angular 23. Which of the following is not a A. copper
is the required wall thickness, t, based acceleration of 150 revolution per classification of alloy steels? B. nickel
on circumferential stress minute, per second? A. low-alloy steel C. lead
considerations? A. 9875 J B. medium-carbon alloy steel. D. all of these
A. 0.75 in B. 10456 J C. high-alloy steel
8 . 3in C. 9845 J D. medium-alloy steel 30. A very inert plastic, solves many
C. 1.50 in D.11775J obdurate problems of gasket and seals
D 3.75-in 24. Improves the machinability of alloy because of its ability to tolerate a
18. Determine the linear speed of a steels but affects different alloys continuous temperature.
13. A shaft has··a diameter of 200 mm pulley 50 em mounted on a 1 inch shad differently. A. thermo plastics
and the load on the bearing is 20 kN shaft that transmits 10 hp. A. lead B. Teflon
The coefficient of friction between the A. 1998 fpm B. vanadium C. nylon
C. selenium D. oil seal
shaft and its bearing is 0.05 and the
shaft revolves at 600 rev/min. Calculate
the power loss in friction .
B. 2000 fpm
C. 1999 fpm
D. 1958 fpm
(, D. all of these
31. Molybdenum-chromium-nickel steel
A . 6.284 kw 25. Increases hardenability of alloy designation:
B. 8.624 kw 19. How long will it take to mill a %"by steels: A. SAE 48xx
C. 4.356 kw 2" long keyway in a 3 ' inch diameter A. manganese B. SAE 56xx
D. 6.844 kw shafting using 24 tooth cutter turning at B. phpsphorous C. SAE 43xx
1000 rpm at 0.005 inch feed per tooth? C. molybdenum D. SAE 46xx
14. What is the maximum allowable A. 0.167 min D. all of these
load, F if the factor of safety is 1.5 and B. 0.198 min 32. Alloy steel used in manufacturing
the yield stress 3000 psi? Assume area C. 0.225 min 26. Improves hardenability bolts, studs, tubings subjected to
equal to 25 in2 . D. 2.167 min economically, resistance to corrosion, torsional stresses.
A. 50 kips strength at high temperatures and high A. AISI3141
B. 60 kips 20. A 20 feet steel lineshaft has no carbon. B. AISI4830
C. 55 kips bending action except its own weight. A. chromium C. AISI2330
D. 64 kips What power in HP can the shaft deliver B. aluminum D. AISI4310
at a speed of 360 rpm. Consider that the C. copper
15. A 100 inches long aluminum bar is torsional deflection will not exceed 0.08 D. manganese 33. Molybdenum steel standard
s ubjected to a tensile stress of 25,000 degree per ft length. designation, SAE _ _ _ _.
psi. Find the elongation (E.= 10 x 106 A. 100 27. Improves the ratio of endurance A. 88xx
psi). B. 55 strength to ultimate stren!]th of medium B.40xx
A. 0.025 in C.120 ·carbon steels. C. 48xx
B. 0.45 in D. 135 A. copper D. 46xx
B. vanadium

REFRESHER MANUAL 2nd Edition by JAS TORDILLO


REFRESHER MANUAL 2nd Edition by JAS TORDILLO
52-41 Day 52- EXAM
Day 52 - EXAM I 52 - 5
34. Big chain sprocket wheels are 39. A solid steel shaft having a diameter Machine Design
designed with spokes/arms are of 3 in twists through an angle of 5 deg
generally elliptical in cross sectional with in 20 ft of length because of the action 45. Surface connecting the crest and
major axis
axis.
times the minor of a torque. Determine the maximum
shearing stress in the shaft.
roots of the screw thread.
A. lead angle
!c?--- I
)
A. 2 A 3550 psi B. top land DAY 52- ANSWER KEY
B. 1 1/2 B. 6550 psi C. length of engagement 1. A 16. 8 31.C46. 0
C. 2 1/2 D. flank 2. D 17. D 32. C47. B
C. 4556 psi 3. D 18. D 33. B 48. B
D.3 D. 8550 psi 4. B 19. A 34. A49. B
46.A good general Purpose deoxidizer 5. C 20. A 35. A 50. 0
35. In designing the spoke or wheel arm 40. Determine the torque that can be and Promotes fine grain in steel. 6. A 21. B 36. C
of a flywheel, it is ideal to consider that A. copper 7.A 22. B 37.C
appiied to a. 1 -in diameter circular shaft 8.A 23. D 38. D
the cross sectional area of the spoke at if the -shearing stress is not to exceed B. molybdenum 9. D 24. A 39. B
the rim side should not be less than 8000 psi. C. magnesium- 10.A 25. D 40. A
of the cross area of the hub side. A 1570 in-lb D. silicon 11 .0 26.A 41 .C
A 2/3 B. 2750 in-lb 12.C 27. B 42. B
B. 80% 47. Nickel-chromium-molybdenum steel 13.A 28. C 43. C
C. 1750 in-lb 14.A 29. B 44. B
c. 90% D. 3560 in-lb designation, SAE _ . 15.C 30. B 45. D
D. 3/8 A. 51 XX
41 . The top and bottom for gears is B. 94xx ~
36. A thin hollow sphere of radius 10 in similar formed as to crest and._ _ _ c. 74xx
and thickness 0.1 in, is subjected to an screw thread. D. 64xx
internal pressure of lob psig. The A. flank
maximum normal stress on an element B. face 48. The radial distance between top of a
of the sphere is~· C. root tooth and bottom of the mating tooth
A 2500 psi D. topland space.
B. 7,070 psi A . addendum
C. 5000 psi 42. Silicon-manganese steel B. clearance
D. 10,000 psi designation, SAE_ _ . C. dedendum

37. A steel rod 30 ft long used in a


A. 72xx
B: 92xx'
( \
D. tip relief

control mechanism must transmit a C. 40xx 49. Plus or minus tolerance can also be
tensile force of 980 lb without stretching D. 9xxx called as _ _
more than 1/8 in, nor exceeding an A . tQtal tolerance
allowable stress of 20,000 psi. What 43. The maximum stress induced in a B. bilateral tolerance
must the diameter of the rod be? (E = 30 material w hen subjected to intermittent C. unflateral
x 106 psi). or repeated load without causing failure D. allowance
A 1/4 fn is called _ _ __
B. 1 1/2 in A ultimate stress 50. Which of the following is not a
C. 3/8 in B. ultimate torque viscoelastic material?
D. 3/4 in C. endurance limit A. teflon
D. elastic limit B. all of these
38. A steel bar initially free of stress, is C. plastic
held between rigid supports. Determine 44. Compute how many, 1/4 inch D. metal
the stress in the bar if the temperature diameter set screws required to transmit
drops 130 deg F. (k = 6.5 x 10~ per deg 3 HP at a shaft speed of 900 rpm. The
F). shaft diameter is 1 1/4 in.
A. 11,400 psi A.2
B. 20,600 psi B.4
C. 15,600 psi c. 3
D. 25,400 psi D. 2.5

REFRESHER MANUAL 2nd Ec:tiffon by JAS TORDILLO


REFRESHER MANUAL 2nd Edition by .JAS TORDILLO
52 - 6 1 Day 52 -Solution
Day 52- Solution 152 -7
Machine Design
DAY 52 - Solution 8(120x:9.81X9)3 (22- 2) = 0.2145 m
y = 80xl09 (0.008) B. 263,894 kg 10. A pinion and a gear is carefully cut
1. Find the tooth thickness on the tooth C. 129,687 kg in a 20 degree full depth design. The 16
= 214.5 mm
circle of a 20 degree full depth involute D. 180,900 kg tooth steel-pinion and 72 tooth cast-iron
tooth having a circular pitch of 26.6 mm gear transmit 50 kw power when the
4. What is the working strength of a 4 in
and whole depth of 16 mm. F = Ssu (ndt) pinion turns 1400 rpm. The face width is
bolt which is screwed up tightly in a
A. 0.52360 in " F = 42,000(n)(4)(1/2) 90 mm and the module is 8. Find the
packed joint wpen the allowable working
B. 0.71681 in
C. 0.41361 in
stress is 12,000 psi. F =263,893.81bs = 119,951.72 kg center distance, in mm, between pinion
A. 82,500 lbs and gear.
D. 0.89713 in 8. Two shafts are connected by spur A. 352 *
B•.S3,600 lbs *
C. 95,600 lbs gears·. The pitch radii of gears A and B 8.280
Pc =26.6 mm =1.047 in D. 95,0'00 lbs are 100 mm and 700 mm, respectively. C. 380
If shaft A make ' s 1200 rpm and is 0.368
Pd X Pc = n · W,;, St [0.55d2 - 0.25d] subjected to a resisting moment of 130
pd X (1.047) =
pd = 3
1T
w = 12,000[0.55(4)2 - 0.25(4)] N-m. What is the torque in shaft B? DI
m =_
W = 93,600 lbs A. 910 N-m *
Tooth thickness, t B. 810 N-m
Tl
= 1.5708 = 1.5708 = 0 _5236 in 5. Determine the strength of a Y. in bolt C. 710 N-m 8 =~
Pd 3 having a yield stress of 60 ksi and stress D. 890 N-m 16
~rea of 0. 1134 in2 • 0 1 = 128 r:nm
2. What pressure is required for A. 320 lbs R 8 = NA = 700 = 7
punching 6 holes of 2 in diameter thru a B.400ibs
m =D2
RA N 8 100
Y2 inch steel plate? C. 382lbs * -
D.4201bs PA = Pe T2
A. 300 tons
TANA =TeNs
B. 340 tons
SA~ · 8 =D2
C. 320 tons
D. 480 tons*
Fa= _Y_ _
6 T 8 = TA (~: }= 130(7) = 910 N-m o2
72
= 576 mm
p =d X t X
80 F~ = 60,000(0.1134)~ C=
D + D2 128+ 576 = 352 mm
12 = 2
9. A prismatic bar at 50°F is imbedded in
P =2(7'2 )80 x·6 6
a rigid concrete wall. The bar is 40
P = 480 tons Fa= 381 .91bs ( inches long and has a cross-'sectional 11. A flywheel weighing 6 kN has a
area of 4 in2. What Is the axial force in mean diameter of 90 em. The maximum
3. Compute the maximum deflection of a 6. The total weight of steel plate size l4. the bar if its temperature is raised to
22 coils helical spring having a load of X 4 X 20.
speed of the flywheel is 7 rev/sec
100°F? The coefficient of thermal slowed down to 4 rev/sec during the
120 kgs. The spring is squared and A. 372 kg*
expansion is 5x10'6 m/m-°F. shearing process. Determine the energy
ground ends with modulus of elasticity in B. 149 kg
A. 5000 lbf released by the flywheel.
shear of 80 GPa, outside diameter of 8 c. 857 kg B. 24,000 lbf
em and wire diameter of 8 mm. D. 163 kg A. 7856 kg-m
C. 8000 lbf B. 9872 kg-m
A. 225 mm
D. 30,000 lbf * C. 8145 kg-m
B. 16.5 min Density of steel = 0.284 lb/in3
C. 245mm D. 8241 kg-m *
D. 214mm * E = 30 x 106 psi
Weight = density x volume
= 0.284[ (-11.)(4 X 12)(20 X 12)] v, =nON,= n(0.90)(7) = 19.79 m/s
YT = YL Vz = nDN2 = n(0.90)(4) = 11.3 m/s
C = Dm _ 8-0.8 = 817.921bs = 371.8 kg
d - - --- =9 kt.aT = FL
0.8
3 7. If the ultimate shear strength of a AE w [ 2 2]
KE= -lV1 -V2
y = 8FC n steel is 42,000 lb/in2 • The force to punch F 2g
Gd a 4 in diameter- hole in a Yz in steel plate 5x10-s(100-50)= ( )
4 30x10 6
is equal to:
A. 119,952 kg,. F = 30,000 lbf
KE = 6000
2(9.81)
[(19.79)2 -(ll.3)2 ]
KE = 80,840.98 N-m = 8240.7 kg-m
REFRESHER MANUAL 2nd Editiqn by JAS TORDILLO
REFRESHER MANUAL 2nd Edition by JAS TORDILLO
52 - 8 1 Day 52 - Solution Day 52 - Solution 1 52 - 9
12, In designing a cylindrical pressure
Machine Design
15. A 100 inches long aluminum bar is
tank 3 feet in diameter, a factor of safety subjected to a tensile stress of 25,000 T= l xa C.120
6
of 2.5 is used. The cylinder is made of psi. Find the elongation (E.= 10 x 10 T = Y. mk2 x a D. 135
steel (yield stress = 30 ksi), and will psi). T = Y. (1500)(1) 2(15.7)
contain pressures up to 1000 psi. What A 0.025 in T = 11,775 J L = 8.95 Jfrj
is the required wall thickness, t, based B. 0.45 in
on circumferential stress 20 = 8.95 3fr)i
C. 0.25 in* 18. Determine the linear speed of a
considerations? D. 0.65 in pulley 50 em mounted on a 1 inch shad D = 3.34 in
A. 0.75 in
= S.!: = 25,00J~)
shaft that transmits 10 hp.
4~
B. 3in Y = 0.25 in A. 1998 fpm
E 6
vl10x10 0=4.6
C. 1.50 in* B. 2000 fpm
D 3.75 in C . 1999 fpm
16. A drop hammer of 1 ton dead 0 . 1958 fpm * 3.34 = 4.6 4 /P
St = -
PD weight capacity is propelled downward v360
2l by a 12 in diameter cylinder. At 100 psi Dp = diameter of pulley P = 100 HP
30.000 1000(3x 12) air pnessure, what is the impact velocity = 100 em= 1.64 ft
if the stroke is 28 inches? 21. The following pertain to joining of
2.5 2t A. 63.2 fps D3 N
P = -- metals except:
t = 1.5 in B. 31.6 fps * 38 . A. welding
C. 15.8 fps B. casting*
13. A shaft has a diameter. of 200 mm D. 47.4 fps
(1)3 N
10 = - - C. soldering
and the load on the bearing is 20 kN 38 D. brazing
The coefficient of friction between the W = 1 ton = 2000 lbs N = 380 rpm
shaft and its bearing is 0.05 and the
22. It is considered semi-solid lubricant:
shaft revolves at 600 rev/min. Calculate F = PA Vp = nDpNp A lube oil
the power loss in friction. Vp = n(1 .64)(380) = 1958 ft/min
A. 6.284 kw * F = 100 2:(12)2 = 11,309.731bs B. grease*
4 C. graphite
B. 8.624 kw 19. How long will it take to mill a %" by
F = ma D. all of these
C. 4.356 kw 2" long keyway in a 3 ' inch diameter
D. 6.844 kw
11 309.73 = (
'
2000
32.2
)a shafting using 24 tooth cutter turning at
1000 rpm at 0.005 inch feed per tooth?
23. Which of the following is not a
classification of alloy steels?
Tr = Fr X r X f = 20(0.100)(0.05) A. 0.167 min *
= 0. 1 kN-m a = 182.1 ft/s 2 A. low- alloy steel

Power loss =2nTN =2n(O. 1) ( 6o


600) Vz2 = V1 2 + 2(a +'ms ( B.,0.198 min
C. 0.225 min
D. 2.1 67 min
B. medium-carbon alloy steel.
C. high-alloy steel
D. medium-alloy steel •
Power loss = 6.283 kw '
2
V2 =0+. 2(182.1 +32.2>(~~)
Time = lengtlr of cut 24. Improves the machinability of alloy
14. What is the maximum allowable v2 = 31.6 ft/sec culling rate steels but affects different alloys ·
load, F if the factor of safety is 1.5 and 2 in
differently.
17. A flywheel has a diameter of 2 m Time = A. lead*
the yield stress 3000 psi? Assume area
2
equal to 25 in . and a mass of 1500 kg. What torque is 0.005 in x 24 !OO.tk xl 00 ~e.~- B. vanadium
needed to produce an angular tooth rev min C. selenium
A. 50 kips* = 0.167 min
B. 60 kips acceleration of 150 revolution per D. all of these
C. 55 kips minute, per second?
A. 9875 J 20. A 20 feet steel lin:=shaft has no 25. Increases hardenability of alloy
D. 64 kips bending action except its own weight.
B. 10456 J steels :
Design Stress = £. c. 9845 J What power in HP can the shaft deliver
at a speed of 360 rpm. Consider that the
A. manganese
A 0.11775 J * B. phosphorous
3000 F torsional deflection will not exceed 0.08 C. molybdenum
- =·- a= 150 revfmin-sec = 15.7 radfsec2 degree per ft length. D. all of these *
1.525 A. 100 *
F = 50,000 lbs = 50 kips B. 55
REFRESHER MANUAL 2nd Edition by JAS TORDIL~O REFRESHER MANUAL 2nd Edition by JAS TORDILLO
52 - 10 I Day 52- Solution Day 52 - Solution 152 - 11
26. Improves hardenability A. AISI3141
Machine Design
economically, resistance to corrosion, B. AISI4830 more than 1/8 in, . nor exceeding an
strength at high temperatures and high C. AISI 2330 * B= TL
allowable stress of 20,000 psi. What
carbon. D. AIS14310 must the diameter of the rod be? (E = 30
JG
A. chromium *
B. aluminum 33. Molybdenum steel standard
x 106 psi). s(~) _ T(20x12)
C . copper designation, SAE _ _ _ _.
A. 1/4 in
B. 1 1/2 in 180 - -~(3)4~2x106)
D. manganese A. 88xx 32
C. 3/8 in*
B. 40xx * D. 3/4 in T = 34,697.83 in-lb
27. Improves the ratio of endurance C. 48xx'
strength to ultimate strength of medium D. 46xx y = FL I6T
carbon steels. AE s. = n:d3
A. copper 34. Big chain sprocket wheels are 6
B. vanadium *
_
0 125
= 98oGoxJ 0 ) 16(34,697.83) =6545 psi
designed with spokes/arms are
C. tantalum generally elliptical in cross sectional with 7tct 2 {30xl0 6 ) s. = n:(3)3
D. all of these major axis times the minor 4
axis. d = 0.346 in say 3/8 in
40. Determine the torque that can be
28. When steel casting are heat treated , A.2*
applied to a. 1 -in diameter circular shaft
the carbon content generally falls within B. 1 1/2 F
S=- if the shearing stress is not to exceed
the range of : C. 2 1/2 A 8000 psi.
A. 0.20% D. 3
980 A. 1570 in-lb •
B. 0. 15 to 0.20% 20 000 =
' 7t/ d2 B. 2750 in-lb
C. 0.25 to 0.50% * 35. In designing the spoke or wheel arm 14 C. 1750 in-lb
D. 0.60% of a flywheel, it is ideal to consider that
d = 0.25 in say Y. in D. 3560 in-lb
the cross sectional area of the spoke at
29. The most effective alloy in the rim side should not be less than
increasing toughness of steel: of the cross area of the hub side. -
Therefore, use 3/8 inch diameter rod. s. = l6T
A . copper 1tDJ
A.2/3*
B. nickel* 38. A steel bar initially free of stress, is 26
B. 80% 8000 = T
C. lead held between rigid supports. Determine
C, 90%
D. all of these D. 3/8 . the stress in the bar if the temperature n:(I)3
drops 130 deg F. (k = 6.5 x 10"6 per deg T = 1570 in-lb
F).
30. A very inert plastic, solves many 36. A thin hollow sphere of radius 10~·
A. 11,400 psi 41. The top and bottom for gears is
obdurate problems of gasket and seals and thickness 0.1 in, is subjected to n
B. 20,600 psi similar formed as to crest and_ __
because of its ability to tolerate a internal pressure of lob psig. T
continuous temperature. C. 15,600 psi screw thread.
maximum normal stress on an element
A. thermo plastics D. 25,400 psi * A . flank
of the sphere is:
B. Teflon* A . 2500 psi B. face
C. nylon . B. 7,070 psi S =kE(h-t~ C. root*
D. oil seal S = 6.5 X 10 (30 X 106)(130) D. topland
C. 5000 psi*
S = 25,350 psi
D. 10,000 psi
31 . Molybdenum-chromium-nickel steel 42. Silicon-manganese steel
designation: 39. A solid steel shaft having a diameter designation, SAE_ _.
PD
A. SAE 48xx St= 4t of 3 in twists through an angle of 5 deg A. 72xx
B. SAE 56xx in 20 ft of length because of the action B. 92xx *
C. SAE 43xx * of a torque. Determine the maximum C. 40XX
100(20) = 5000 psi
D. SAE46xx St = 4(0.1) shearing stress in the shaft. D. 9xxx
A. 3550 psi
32. Alloy steel used in manufacturing B. 6550 psi • 43. The maximum stress induced in a
37. A steel rod 30 ft long used in a C. 4556 psi
bolts, studs, tubings subjected to material when subjected to intermittent
control mechanism must transmit a D. 8550 psi or repeated load without causing failure
torsional stresses. tensile force of 980 lb without stretching is called _ _ __
REFRESHER MANUAL 2nd Edition by JAS TORDILlO REFRESHER MANUAL 2nd Edition by JAS TORDILLO
52 - 12 1 Day 52 - Solution Day 53 - EXAM I 53 - 1
Machine Design
A. ultimate stress 48. The radial distance between top of a
B. ultimate torque tooth and bottom of the mating tooth The design stress for the plate and the
C. endurance limit* space. DAY 53- EXAM rivets are 170 MPa and 220 MPa,
D. elastic limit A. addendum respectively. Determine the thickness of
B. clearance * 1. A jackhammer consumes air at the the plate. Assume 70% efficiency of the
44. Compute how many 1/4 inch C. dedendum rate of 0.01 m3/sec at 30°C and 800 joints.
diameter set screws required to transmit D. tip relief kPa. Determine the minimum capacity of A. 17.2 mm
3 HP at a shaft speed of 900 rpm. The the air compressor in m 3/sec at 21°C B. 21.2 mm
shaft diameter is f 1/4 in. 49. Plus or minus tolerance can also be and 100 kPa if 10 jackhammers operate C. 25.2mm
A. 2 called as_ simultaneously. D. 32.2 mm
B. 4 .. A. total tolerance A. 0.276
C. 3 B. bilateral tolerance * B. 0.776 6. Determine the theoretical force
D. 2.5 C. un'flateral c. 0.376 designed in an air cylinder operating at
D. allowance D. 0.986 10 bar and having an inside diameter of
Power per screw 100 mm.
23 50. Which of the following is not a 2. What maximum output load can a
= DNd = 1.25(9(lOX0.25)2- 3 pneumatic cylinder sustain If the cylinder
A. 4587 N
viscoelastic material? B. 5784 N
50 50 A. teflon is operated with an air pressure of 500 C. 5874 N
= 0.927 HP per screw B. all of these kPa. The diameter of the cylinder is 40 D. 7854 N
C. plastic mm. Assume a friction loss of 15%.
No. of setscrews D. metal* A. 534 N 7. What is the size of an air cylinder
·= total power transmitted B. 725 N operating at 10 bar with a required force
power per screw C. 630 N of 7854 N? Use a load ratio of 90%.
D. 894 N Load ratio means the ratio of required
3 force and theoretical force.
0.927 3. Compute the bursting pressure of a A. 100 mm
=3.23 screws say 4 setscrews spherical air tank with diameter of 2 a B. 150mm
and wall thickness of 'A in if the tensile C. 105 mm
45. Surface connecting the crest and stress is 460 Mpa and joint efficiency of D. 300mm""
roots of the screw thread. 80%.
A. lead angle A. 8,200 kPa 8. What is the pressure working on an
B. 8,000 kPa
B. top land
C. length of engagement
D. flank*
( C. 8,128 kPa
£?. 7,500 kPa
air cylinder if it is pr.oducing 5 kN force
on the piston? Piston diameter is 5 mm
and cylinder diameter is 80 mm.
A. 995 kPa
46. A good general Purpose deoxidizer 4. A steel cylindrical air receiver with 6.0 B. 775 kPa
and Promotes fine grain in steel. feet diameter and pressure load of 220 C. 885 kPa
A. copper psi, design stress of 10,000 psi D. 665 kPa
B. molybdenum maximum. Assume a 90% weld joint
C. magnesium- efficiency. The lap welding tensile 9. A 60 mm diameter by 200 mm stroke
D. silicon* strength is 70,000 psi. Determine the double acting air compressor cylinder is
bursting steam pressure of this air working with 8 strokes per minute at a
47. Nickel-chromium-molybdenum steel receiver. pressure of 10 bar. Determine Me
designation. SAE _ . A. 1540 psi average amount of compressed air in
A. 51 XX B. 1450 psi liters per minute if this kind of cylinder
B. 94xx * C . 1045 psi can compress 2 liters per 100 mm
C. 74xx D. 1054 psi stroke.
D. 64xx A. 64
5. A cylindrical tank is subjected to a B. 55
maximum pressure of 3 MPa.. The C.46
inside diameter is 2,000 mm while the D. 24
riveted joint has a longitudinal pitch of
'
REFRESHER MANUAL 2nd Edition by JAS TORDILLO 70 mm and there are two straps placed.
53 - 2 I Day 53 - EXAM Day 53 - EXAM I 53 - 3
Machine Design
10. The inside diameter of the cylindrical 10 in. The power hacksaw does 120
air receiver is 600 mm and ft allowable strokes/min and the feed per stroke is C. 19 B. 16.1
tensile stress of the tank material is 70 0.127 mm. D.29 C. 15.1
Mpa. Determine the required wall A. 20 D. 17.1
thickness of the tank is the cylinder is B. 500 20. Determine the most economical
capable of handling pressures up to 4 C. 1000 acetylene gas consumption in cu . ft to 25. A gear of 80 teeth and a pinion of 20
Mpa. D. 250 manually cut crosswise a 4ft x 20ft x 1 in teeth has a gear and pinion ratio of 4.
A. 11 mm thick steel plate into five sheets of 4ft x Find the diameter pitch of these gears if
B. 17 mm 16. How long would it take to cut by 4ft x 1 in steel plate. The manual cutting the center distance is 20.
C. 15 mm oxy-acetylene gas cutter crosswise a 4 · rate of oxy-acetylene gas cutter is 12 A. 2.0
D. 30 mm ft x 20 ft x 1 in thick steel plate if min in 11 ft to 16ft length of steel plate. B. 3.0
oxy-acetylene machine can finish cutting Oxygen consumption is 60 cu. ft and C. 2.5
11 . The inside diameter of an air
cylinder is 50 mm and can handle a
a 10 ft long steel in 12 min?
A. 288 sec
.. acetylene gas rate is 10 cu. ft per hour.
A. 2.4
D. 3.5
pressure of 10 Mpa. Assuming an B. 400 sec B. 7.2 26. An air cylinder having an internal
allowable tensi le stress of the cylinder C. 300 sec C . 3.5 diameter of 16 inches and an external
material of 50 Mpa, determine the D. 550 sec D. 9.5 diameter of 32 inches is subjected to an
required thickness of this cylinder. internal pressure of 8,000 psi and an
A. 5.61 mm 17. On the average, how long would it 21 . A pair of gear and pinion having a external pressure of 3,000 psi.
B. 4.61 mm take to cut manually using an ratio of 4 with the gear having 100 teeth Determine the hoop stress at the inner
C. 5.01 mm oxyacetylene gas cutter to cut crosswise and the pinion with 25 teeth . Find the surface of the cylinder.
D. 4.01 mm a 4 ft x 20 ft x I in steel plate into 4 circu lar pit~h. if the center distance is 15 A. 4,333.33 psi
sheets of 4 ft x 5 ft x 1 in steel plates. and the pitch diameter is at 6. B. 5,333.33 psi
12. Determine the time, in seconds, The manual cutting rate of A. 0.94 C. 6,333.33 psi
required to drill a 8-inch deep hole on a oxy-acetylene gas cutter is 12 min in 8 B. 0.610 D. 10,000 psi
steel plate if the feed is 0.0025 in/rev to 12ft length of steel plate. C. 0.754
and the drill turns at 200 rpm. A. 12.1 min 0 . 0.552 27. A 750 mm steel pulley transmits 120
A. 20 B. 15 min .,hp at 700 RPM. The arc of contact
B. 750 C. 14.4 min 22. A gear of 80 t~ethnd a pinion of 20 between t~ belt and pulley is 152
C. 120 D. 18 min teeth has a gear an pinion ratio of 4. degrees, the coefficient of friction
D. 960 Compute the circu ar pitch, in inches, if between belt and pulley is 0.30 and the
18. On the average, compute the time to the center distance is 20. safe working stress of the belt is 2.0
13. How long will it take to drill a hole cut manually using an oxy-acetylene A. 1.05 MPa. Find the width of the belt used in
through a 10 em thick steel plate if the gas cutter to cui--lengthwise two 8ft x B.'1.35 mm if its thickness is 20 mm.
drill feed is 0.1 mm per rev and a %-in 20ft x 3/4 in steel plates into four 4ft x C. 1.25 A. 148 mm
diameter drill is turning at 750 rpm? 20ft x 3/4 in steel plates. The manual D. 2.24 B. 184 mm
A. 80 sec cutting rate of oxy-acetylene gas cutter C. 481 mm
B. 500 sec is 12 min in 11 ft to 16 ft length of steel 23. A gear of 80 teeth and a pinion of 20 D. 841 mm
C. 100 sec plate. teeth has a gear and pinion ratio of 4.
D 1000 sec A. 36 min ·Find the tooth thickness on the pitch 28. A flat belt is to transmit 100 Hp to an
B. 55 min circle of a 14 Y2 degree full depth tooth air compressor. The small sheave' is
14. How long will it take to cut a 4-in C. 40 min if the center distance is 20. 180 mm in diameter and turns at 1200
long thread at 100 rpm if the threading D. 90 min A. 0.268 rpm, while the larger sheave turns at
machine has a configuration of 20 B. 0.862 450 rpm. The service factor may be
threads per inch? 19. Determine the most economical taken as 2.0, the center distance is
C. 0.628
A. 1 sec oxygen gas consumption in cu. ft to D. 0.925 equal to two times the diameter of the
B. 48 sec manually cut lengthwise a 4 ft x 20 ft x I larger sheave. Determine the length of
C. 25 sec in thick steel plate. Oxy-acetylene gas 24. A gear of 80 and a pinion of 20 teeth belt in mm and arc of contact of small
D. 90 sec cutting rate is 18 in per min. Acetylene has a gear and pinion ratio of 4. Find the sheave in degrees.
gas used is 6 cu. ft and oxygen gas rate center distance, in inches, if the circular A. 2980 mm, 161 deg
15. Compute the time, in seconds, to is 130 ~ per hr. pitch is 0.885. B. 2890 mm, 261 deg
saw a rectangular aluminum bar 6 in A. 15 A. 14.1
wide and I in thick if the length of cut is B. 25
53 - 4 I Day 53 - EXAM Day 53 - EXAM I 53 - 5
· Machine Design
C. 2080 mm. 116 deg 36. A fluid power system Is one that
D. 2280 mm, 111 deg transmits energy through the use of C. gate valve C. pneumatic air
pressurized ____. D. stop valve D. compressed air
29. Determine the section modulus of a A. liquid only
10 em x 25 em beam. B. mixture of liquid and gas 43. Drains all the · water condensing in 50. A machine which is used to increase
A 1 041.6 em3 C. gas only the tank without supervision. the pressure of air by decreasing its
B. 1416.1 em3 D. liquid or gas A. fusible plug volume.
C. 1604.1 cm 3 B. discharge valve A. air compressor
D. 4611.0 em3 37. It helps to keep the line free from C. intake valve B. pump
dust, water and oil in the pneumatic D. drain valve C. actuator
30. Determine the maximum stress of a system. D. cylinder
beam, 10 em x 25 em. if the maximum A. line filter 44. Releases compressed air off If the
moment is 1000 N-m. B. adjuster pressure in the tank rises above the
A. 690 kPa C. valve allowed pressure.
B. 960 kPa D. actuator A. safety valve (c-3 )
C. 780 kPa B. air pocket DAY 53- ANSWER KEY
D. 1500 kPa 38. In the-air pro..duction system. a C. release valve 1. 6 16.A 31 . A46.C
device that indicates the pressure in the D. line valve 2. A 17. C 32. A47. D
31. What is the radius of gyration of a 10 tank. 3. A 18. A 33. C48. A
em x 25 em beam? A. fusible plug 4. A 19. D 34. 6 49. 0
45. The device used to avoid having 5. C 20. A 35. D 50. A
A. 7.21 em B. pressure gauge water in the downstream system. 6. D 21 . C 36. D
B. 140 em C . pressure tank A . Refricerated air dryer 7. C 22. C 37. A
C. 12.5 em D. drain valve B. Refrigerated air cooler 8. A 23. C 38. 6
D. 12 ern 9. A 24. A 39. C
39. A component of the air production
C. air trap
D. air filter
10.8 25. C
11.A 26. B
40. A
41 . 6
.
32. Determine the moment of Inertia of system that transforms mechanical 12.0 27. A 42. B
beam having an area, of 150 cm2 and energy into pneumatic energy. 46. A system that is powered by air is 13.A 28. A 43. D
radius of gyration 5 em. A. pulley 14.6 29. A 44. A
called:
A. 3750 cm 4 B. piston 15.C ~(). 6 45. A
A. hydraulic
B. 7530 cm 4 C. compressor
C. 4580 cm 4 D. crankshaft
B. hydrostatic G
C. pneymatic
D·. 5370 cm 4
D. airmatic
40. It transforms electrical energy into
33. The moment of inertia of a circle of mechanical en~y and supplies 47. A device used as the storage of
radius 5 em is nearest to: mechanical power to the compressor. compressed air.
A. 391 cm 4 A. electric motor · A. filter
B. 591 cm 4 B. generator B. pan
C. 491 cm4 C. motor pulley C. reservoir
D. 691 cm 4 D. electric power D. tank

34. The section modulus of a circle of 41. A device that controls the electric 48. A basic pneumatic system consists
radius 5 em is equal to: motor by sensing the pressure In the of two main sections:
3
A. 58.2 cm tank. A. air production arid air consuming
B. 98.2 cm3 A. pressure tank system
C. 78.2 cm3 B. pressure switch B. air production and air distribution
D. 108.2 cm3 C. pressure line C. air distribution and air circulation
D. safety valve D. air distribution and air production
35 The polar moment of inertia of a
shaft of radius 5 em is equal to: 42. A device that prevents the 49. is mainly used to do
A. 482 cm4 compressed air back to the compressor work by acting on a position or vain.
B. 782 cm 4 when the compressor is stopped. A. atmospheric air
C . 582 cm 4 A. safety valve B. moist air
D. 982 cm4 B. check valve
53 - 6 I Day 53 - Solution Day 53 - Solution 153 - 7
stress is 460 Mpa and joint efficiency of
Machine Design
DAY 53 - SOLUTION 80%. respectively. Determine the thickness of B. 775 kPa
A. 8,200 kPa * the plate. Assume 70% efficiency of tht;l C. 885 kPa
1. A jackhammer consumes air at the B. 8,000 kPa joints. D. 665 kPa
rate of 0.01 m3/sec at 30°C and 800 C. 8,128 kPa A. 17.2 mm
kPa. Determine the minimum capacity of . D. 7,500 kPa B. 21.2 mm
the air compressor in m3/sec at 21°C
C. 25.2 mm *
p= £.
and 100 kPa if 10 jackhammers operate t = Y. in= 12.7 mm A
D. 32.2 mm
simultaneously. PD 5
A. 0.276 St=- P= = 994.72 kPa
B. 0.776 * 4tE PD 3(2000) • = 25.21 mm ~ (0.08)2
C. 0.376 t = 2s1e = 2(170X0.70) 4
400,000 = P(2000)
D. 0.986 p - 4(12.7p.80
6. Determine the theoretical force 9. A 60 mm diameter by 200 mm stroke
- 8,128 kPa double acting air compressor cylinder is
PV=mRT1 designed in an air cylinder operating at
(800)(0.01) = m(0.287)(30 + 273) · 10 bar and having an inside diameter of working with 8 strokes per minute at a
Therefore, the bursting pressure should pressure of 10 bar. Determine Me
m = 0.09199 kg/s be greater than 8128 kPa. 100 mm.
A. 4587 N average amount of compressed air in
Total mass of air needed for 10 units of B. 5784 N · liters per minute if this. kind of cylinder
4: A steel cylindrical air receiver with 6.0 can compress 2 liters per 100 mm
air compressors: feet diameter and pressure load of 220 C. 5874 N
= 0.09199 X 10 D. 7854 N * stroke.
psi, design stress of 10,000 psi
= 0.9199 kg/s A.64*
maximum. Assume a 90% weld joint 8 . 55
efficiency. The lap welding tensile F = PA
Solving for the capacity of air C. 46
strength is 70,000 psi. Determine the
compressors: F = 1D(10o)2:(o.t)2 D. 24
bursting steam pressure of this air 4
P2 V2 = mrRT2 receiver.
100Vz = 0.9199(0.287)(21 + 273) F = 7.85398 kN = 7853.98 N Amount of air compressed
A.1540 psi*
3
v2 = 0. 776 m /sec 2li 200 mm 8 strokes
B. 1450 psi 7. What is the size of an air cylinder =---X X X 2
100 mm stroke min
2. What maximum output load can a
C. 1045 psi
D. 1051 psi
operating at 10 bar with a required force
=

64 li/rnin
pneumatic cylinder sustain if the cylinder of 7854 N? Use a load ratio of 90%.
is operated with an air pressure of 500 Load ratio means the ratio of required
Solving for the wall thickness based on force and,AtreG.t:etical force. 10. The inside diameter of the cylindrical
kPa. The diameter of the cylinder is 40 operating pressure: A. 100 m'm air receiver is 600 mm and ft allowable
mm. Assume a friction loss of 15%.
St == PD B. 150 mm tensile stress of the· tank material is 70
A. 534 N * ZtE
Mpa. Determine the required wall
B. 725 N C.105 mm *
10,000 = 2Z0{72) D. 300 mm thickness of the tank Is the cylinder is
C. 630 N capable of handling pressures.up to 4
D. 894 N t _ O . Zt(0.90)
- .88 In . 7854 Mpa.
Load mput, F c - - = 8726.67 N A 11 mm
F = PA Solving the bursting pressure: 0.90 8.17 mm *
F= soo.ooo ( ~ }o.o4o)2 S = PD
u 2t p = £. C. 15 mm
D. 30mm
P(1Z) A
70•000 = 2(0.88)(0.90)
F = 628.3 N
P = 1540 psi 10(100)1000 = 8726.67 PD
1t D2 St=-
Output load = Input load -friction 2t
= 628.3 0.15(628.3) 5. A cylindrical tank is subjected to a 4
= 534 N maximum pressure of 3 MPa. The D = 0. 1064 m = 105.4 mm 70 = 4(600)
inside diameter is 2,000 mm while the 2t
3. Compvte the bursting pressure of a riveted joint has a longitudinal pitch of 8. What is the pressure working on an t = 17.14 mm
spherical air tank with diameter of 2 a 70 mm and there are two straps placed. air cylinder if it is producing 5 kN force
and wall thickness of 'A in if the tensile The design stress for the plate and the on the piston? Piston diameter is 5 mm 11. The inside diameter of an air
rivets are 170 MPa and 220 MPa, and cylinder diameter is 80 mm. cylinder is 50 mm and can handle a
A. 995 kPa *
53 - 8 I Day 53 - Solution
Day 53 - Solution 153 ~ 9
pressure,. of 10 Mpa. Assuming an 14. How long will it take to cut a 4-in Machine Design
allowable tensile stress of the cylinder long thread at 100 rpm if the threading
material of 50 Mpa, determine the 4ft manually cut lengthwise a 4 ft x 20 ft X. I
machine has a configuration of 20 Time=
required thickness of this cylinder. 10ft in thick steel plate. Oxy-acetylene gas
threads per inch?
A. 5.61 mm * cutting rate is 18 in per min. Acetylene
A. 1 sec 12min
B. 4.61 mm gas used is· 6 cu . ft and oxygen gas rate
B. 48 sec *
C. 5.01 mm
Time = 4.8 min = 288 sec is 130 ft 3 per hr.
C. 25 sec
D. 4.01 mm D. 90 sec
A. 15
17. On the average, how long would it B. 25
take to t:Ut manually using an c. 19
Time = length of cut
t= 0[ St + P _ I]
cutting rate
oxyacetylene gas cutter to cut crosswise
a 4 ft x 20 ft x I in steel plate into 4
D. 29 *
2 St- P
sheets of 4 ft x 5 ft x 1 in steel plates.
' 4in Time = --'--:...····
Time= The manual cutting rate of
t = 50 [ 50 + I 0 _ I ] . in thread 100 rev cutting rate
-- X -···-· X---·- oxy-acetylene gas cutter is 12 min in 8
2 50 - 10 20 threads rev min to 12 ft length of steel plate.
t = 5.61 mm Time = 0.80 min = 4j3 sec A. 12.1 min . _ 20(12) in
T lme- 18in
B. 15 min
12. Determine the time, in seconds, 15. Compute the time, in seconds, to C. 14A min*
min
required to drill a 8-inch deep hole on a saw a rectangular aluminum bar 6 in D. 18 min
Time= 13.33 min= o:222 hr
steel plate if the feed is 0.0025 in/rev wide and I in thick if the length of cut is
and the drill turns at 200 rpm. 10 in. The power hacksaw does 120 . length of cut
T1me= _ ..::::...._ __ Oxygen gas consumption
A. 20. strokes/min and the feed per stroke is cutting rate = 0.222 (130)
B. 750
c. 120
0.127 mm.
A. 20 Time = (3)4 ft
=28.86 ft3
D. 960 * B. SOO 10ft
20. Determine the most economical
C.1000 * 12min acetylene gas consumption in cu. ft to
Time = length of cut D. 250 manually cut crosswise a 4ft x 20ft x 1 in
Time= 14.4 min
cutting rate lhick steel plate into five sheets of 4ft x
Time = length of cut 18. On the average, compute the time to 4ft x 1 in steel plate. The manual cutting
8in rate of oxy-acetylene gas cutter is 12
cutting rate cut manually using an oxy-acetylene
Time = in 200 rev x !TJin min in 11 ft to 16ft length of steel plate.
. 10(25.4)mm gas cutter to cut lengthwise two ~ x
0.0025 rev x mm·· 60 sec Oxygen consumption is 60 cu. ft and
T1me = - - --'--- ' - - - - - 20ft x 3/4 in steel plates into ·tour 4ft x
mm 120 strokes 20ft' x 3/4 in steel plates. The ma al acetylene gas rate is 10 cu. ft per hour.
Time = 960 sec 0.127 -·--- X • · - --- ·
stroke min cutting rate of oxy"acetylene gas cutter A. 2.4 *
13. How long will it take to drill a hole Time= 16.67 min= 1000 sec is 12 min in 11 ft to 16ft length of steel B. 7.2
plate. C. 3.5
through a 10 em thick steel plate if the
16. How long would it take to cut by A. 36 min* D. 9.5
drill feed is 0.1 mm per rev and a %-in
diameter drill is turning at 750' rpm? oxy-acetylene gas cutter crosswise a 4 B. 55 min
A. 80 sec ~ . ft x 20 ft x 1 in thick steel plate if C. 40 min . length of cut
T 1me= - = - - -
B. 500 sec oxy-acetylene machine can finish cutting D. 90 min cutting rate
C. 100 sec a 10ft long steel in 12 min?
D. 1000 sec A. 288 sec* . length of cut . - 4(4)ft
B. 400 sec T1me = _..:::__ _ T1me- 13.S in
cutting rate
length of cut C. 300 sec 12 min
Time= D. 550 sec Time = 2(20) ft
cutting rate Time= 14.22 min = 0 .237 hr
. 13.5 ft
10(10)mm Time = length of cut 12min
Time= Acetylene gas consumption
0• I
. mm
X
75 0 rev
X
min cutting rate Time = 35.56 min p
= 0.237 0)
re v min 60 sec. = 2.37 ft
Time= 80 sec 19. Determine the most economical
oxygen gas consumption in cu. ft to

--- - - - - - -
53 - 10 I Day 53 - Solution Day 53 - Solution I 53 - 11
Machine Design
21. A pair of gear and pinion having a
ratio of 4 with the gear having 100 teeth 20 =
diameter of 32 inches is subjected to an
and the pinion with 25 teeth. Find the
D, =8 internal pressure of 8,000 psi and a*l F 2 =Fi- -
circular pitch, if the center distance is 15 external pressure of 3,000 psi. 2.2J6
and the pitch diameter is at 6.
F, - -Fi- = 3256.6
Determine the hoop stress at the inner
A. 0.94 p - 21.=
Dl
20 =2.5
8
surface of the cylinder.
2.216
B. 0.610 d -
A. 4,333.33 psi
c. 0.754 * B. 5,333.33 psi * F1 ;:;5934.72 N
D. 0.552 1.5708 - 1.5708 = 0.628
T = ~- 2.5 C. 6,333.33 psi F
Stress= - 1
D. 10,000 psi bt
Pc = nO I = n(6 ) = 0.754 in ~4. A gear of 80 and a pinion of 20 teeth 5934.72
T1 25
h?S a gear and pinion ratio of 4. Find the
center distance, in inches, if the circular
Pi (rJ + tl )- 2poro 2 20
' = b(20)
s~~;:; 2 2
22. A gear of 80 teeth and a pinion of 20 pitch is 0.885. ro - ri b = 148.36 mm
teeth has a gear and pinion ratio of 4. A. 14.1 *
Compute the circular pitch, in inches, if B. 16.1 Where: n = 8" Pi= 8,000 psi 28. A flat belt is to transmit 100 Hp to an
the center distance is 20. C. 15.1 ro = 16" p0 = 3,000 psi air compressor. The small sheave is
A. 1.05 D. 17.1 180 mm in diameter and turns at 1200
B. 1.35 rpm, while the larger sheave turns at
C.1.25 * Pd X Pc = TT sti = 8.ooo(16 2 +8 2 )- 2(3_oooXJ 6)2 450 rpm. The seryice factor may be
D. 2.24 pd (.0885} = TT taken as 2.0, the center distance is
162 -8 2
pd = 3.55 = 5,333.33 psi
equal to two times the diameter of the
D2 C_ 1J + T2 _ 20 + 80 _ larger sheave. Determine the length of
-=4 14.08 in belt in mm and arc of contact of small
Ot - 2Pd - 2(3.55) - 27. A 750 mm steel pulley transmits 120
sheave in degrees.
D2 = 4D1 hp at 700 RPM. The arc of contact
A. 2980 mm, 161 deg •
between the belt and pulley is 152
C = 01 + 0 2 25. A gear of 80 teeth and a pinion of 20 Bj)90 mm, 261 deg
degrees, the coefficient of friction
2 teeth has a gear and pinion ratio of 4. C. 2 80 m~]r. 116 deg
between belt and pulley is 0.30 and the
Find the diameter pitch of these gears if D. 80 mm, 111 deg
20 = 01 +402 lhe center distance is 20.
safe working stress of the belt is 2.0
2 MPa. Find the width of the belt used in
A. 2.0 D1 = 180 mm
mm if its thickness is 20 mm.
D, = 8 ( ) B. 3.0 D1N1 = D2N2
c. 2.5 * A.J48 mm *
Pc =
n01
--:r;--~=
20
1.25
D. 3.5 B. 184 mm
C. 481 mm
180(1200) = D2(450)
D2 = 480 mm
D. 841 mm
C = 2D2 2(480) = = 960 mm
23. A gear of 80 teeth and a pinion of 20 D2
-=4 p = 120 hp ::: 89.52 (D 2 _,.D )2
teeth has a gear and pinion ratio of 4. Dt P = ~2nTN L=2C+1.57(02+D1)+ I
Find the tooth thickness on the pitch 4C
2nTC6°~)
D2 = 4D1
Circle of a 14 Y2 degree full depth tooth 89.52 = = 2(960) + 1.57(480 + 180)
C = D1 +D 2
if the center distance is 20. + (480-180)2
A. 0.268 2 T = 1.2212 kN-m
4~60)
B. 0.862 20 = Dl+4D 2 . torque
c. 0.628 * F1 - F2 = tangential force = - - - = 2,979.64 mm
2 radius
D. 0.925 o, = 8 212 (D- i'l£·0°
= 1.2 = 3.2566 kN Arc of Contact = 180° - JU
0.75 / 2
Dl =4 = 2!_- 20
Ot
pd
o --g=2.5 = 3256.6 N =1800- (480-180)600 = 161.250
1 960
D2 = 4D1
C = Dt + D2
2
26. An air cylinder having an internal
diameter of 16 inches and an external
£!.. =e'e;:; e 0.30(152x 180
" )
=2.216 29. Determine the section modulus of a
F2 10 em x 25 em beam.
53 - 12 I Day 53 -Solution Day 53 - Solution I 53 • 13
Machine Design
A.1 041.6 cma 33. The moment of inertia of a circle of
8.1416.1 cm3 radius 5 em is nearest to: B. pressure gauge * 45. The device used to avoid having
C. 1604.1 cm 3 A. 391 cm 4 · C. pressure tank water in the downstream system.
3 4
D. 4611.0 cm B. 591 cm D. drain valve A. Refrigerated air dryer *
2 C. 491 cm 4 * B. Refrigerated air cooler
bd D. 691 cm 4
z =- 39. A component of the air production C. air trap
6 system that transforms mechanical D. air filter
2 4 4
10 25 energy into pneumatic energy.
z= ( ) =1041.37 cm3 I=
nd
M= n(l0)
64
=490.9cm•
A. pulley 46. A system that is powered by air is
6
B. piston called:
C. compressor* A. hydraulic
30. Determine the maximum stress of a 34. The section modulus of a circle of
D. crankshaft B. hydrostatic
beam, 10 em x 25 em, if the maximum radius 5 em is equal to: C. pneumatic *
moment is 1000 N-m. A. 58.2 cm 3 40. It transforms electrical energy into D. airmatic
A. 690 kPa B. 98.2 cm 3 * mechanical energy and supplies
B. 960 kPa * C. 71>.2 cm 3 47. A device used as the storage of
mechanical power to the compressor.
C. 780 kPa D. 108.2. cm3 compressed air.
A. electric motor *
D. 1500 kPa
B. generator A. filter
3 3 B. pan
C. motor pulley
3
I= bd = 0.10(0.25) =
3
z = n=
nd n(10)
32
= 98.2 cm3
D. electric power C. reservoir
12 12 D. tank*
= 1.302083x10"' m4 35. The polar moment of inertia of a 41. A device that controls the electric
motor by sensing the pressure in the 48. A basic pneumatic system consists
shaft of radius 5 em is equal to:
100J0.25) A. 482 cm4 tank. of two main sections:
2 4 A. pressure tank A. air production arid air consuming
Sr = Me = \. 960 000 B. 782 cm
[ 1.302083 X I 0-4 ' C. 582 cm
4 B. pressure switch * system*
C. pressure line B. air production and air distribution
Pa = 960 kPa D. 982 cm 4 *
D. safety valve ·C. air distribution and air circulation
D. air distribUtion and air production
31. What is the radius of gyration of a 10 4 4
em x 25 em beam?
A. 7.21 em*
J= -n=n- =
1td 7t(10 ) 981.8 em• 42. A device that prevents the
compressed air back to the compressor 49. is mainly used to do
work by acting on a position or vain.
when the compressor is stopped.
B. 140 CM A. atmospheric air
36. A fluid power system Is one that A. safety valve
C. 12.5 em B. check valve " B. moist air
transmits energy through the use of
D. 12 em C. pneumatic air

ff pressurized _ __ C. gate valve


A. liquid only D. stop valve D. compressed air *
k= = 0.289h = 0.289(25) = 7.22 em B. mixture of liquid and gas
C. gas only 43. Drains all the water condensing in 50. A machine which is used to increase
D. liquid or gas * the tank without supervision. the pressure of air by decreasing its
. 32. Determine the moment of Inertia of volume.
A. fusible plug
beam having an area, of 150 cm2 and A. air compressor *
37. It helps to keep the line free from B. discharge valve
radius of gyration 5 em. B. pump
dust. water and oil in the pneumatic C. intake valve
A. 3750 cm4 • C. actuator
system. D. drain valve *
B. 7530 cm 4 D. cylinder
C. 4580 em• A. line filter •
4 B. adjuster 44. Releases compressed air off If the
D. 5370 cm pressure in the tank rises above the
C. valve
k= ff D. actuator

38. In the-air production system. a


·· allowed pressure.
A. safety valve *
B. air pocket
5= (I device that indicates the pressure in the C. release valve
{!5o tank. D. line valve
I = 3750 cm 4 A. fusible plug
Day 54 ~ EXAM I 54 ~ 1
Machine Design
C. 4.75 hp
DAY 54- EXAM D. 5.50 hp

1. A solid shaft is to be used to transmit 6. A hollow shaft has an inner diameter


90 'rON at 750 RPM. If the shaft design of 0.04 m and an outer diameter of 0.07
stress will not exceed 29 Newton/mm2 , m. Compute the torque if the shear
determine the diameter of the solid stress is not to exceed 150 MPa in N-m.
shaft. A. 6025 N-m
A. 48.6 mm B. 7025 N-m
B. 58.6 mm C. 8025 N-m
C. 68.6 mm D. 9025 N-m
D. 78.6 mm
7. In SAE 1030 steel 1'h inch diameter
2. The shaft is subjected to a steady solid shaft with a deflection not to
load of 40,000 in-lb at a shear stress of e-xceed 0.08 deg/ft length. Modulus of
8,000 psi. Compute the diameter of the rigidity is equal to 12,000,000 psi.
said shaft in inches Determ'ine the power transmitted if the
A. 2.3· in shaft rotates at 330 rpm.
B. 3.3 in A. 1.8 hp
C. 4.3 in B. 2.5 hp
D. 5.3 in C. 3.6 hp
D. 4.5 hp
3. The torsional deflection of a SAE
1040 steel shaft is to 1.0° per meter 8. Compute for the torsional deflection in
length. The shear stress is 70 MPa. degrees of a 90 mm diameter, 2.0 m
Compute the diameter of the shaft in long shaft subjected to twist moment of
millimeter. Steel modulus of elasticity is 3.5 X 106 N-mm. The torsional modulus
79,300 MPa. 2
of elasticity is 80,000 N/mm .
A. 70 mni A. 0.52deg
B. 80 mm B. '0.65 deg
C. 101 mm C. 0.78 deg
D. 110 mm D. 0.95 deg

4 .. A 20 ft steel lineshaft has no bending 9. Find the power in watts transmitted


action except its own weight. What by a main power transmitting shaft with
horsepower can the shaft deliver at a diameter of 90 mm and speed of 500
speed of 300 rpm . Consider that the rpm
torsional deflection will not exceed. 0.08 A. 106,837.2 w
deg/ft length. B. 206,837.2 W
A. 38 hp C. 306,837.2 W
B. 43 hp D. 406,837.2 W
c. 65 hp
D. 83 hp 10. A power plant turbine developing
12,000 hp and it turns the shaft at 600
5. A small shaft is 1 inch in diameter rpm. . The propeller which ,is attached
and has an allowable stress of 8QOO psi. the shaft develops a thrust of 120,000
Find the horsepower delivered by the lbs. A hollow steel shaft with an outside
shaft at a speed of 20 rad/sec. diameter of 12 in will be used, compute
A. 1.25 hp for the torque.
B. 2.75 hp A. 105,042.3 ft-lb

REFRESHER MANUAL 21l<~· fdition by JAS TORDILLO


54 - 2 I Day 54 - EXAM Day 54 - EXAM I 54 - 3
Machine Design
B. 125,042.3 ft-lb C. 7,453.8 in-lbs
C. 155,042.3 ft-lb D. 5,453.8 in-lbs 19. It Is found that a shearing machine C. 0.1038 in
D. 195,042.3 ft-lb requires 300 J of energy to shear a D. 0.1338 in2
15. From Problem No. 14, Determine specific gauge of sheet metal. The
11. From Problem No. 10, determine the aiameter of bolts required. mean diameter of the flywheel is to be 23. From Problem No. 22. Determine
the inside diameter of ·the shaft if the - A. 0.404 in 762 mm. The normal operating speed is the torque applied on bolt in N-mm.
maximum shearing stress based on the B. 0.304 in 200 rpm, and slows down to 180 rpm A. 10,881.05 N-mm
torsion alone is not to exceed 8,000 psi. C. 0.2,04 in during shearing process. The rim width B. 25,881 .05 N-mm
A. 10.3 in D. 0.104 in is 304.8 mm.· and the weig ht of the cast C. 35,881.05 N-mm
B. 12.5 in iron is 7,200 kg/m 3 . Find the thickness D. 40,881 .05 N-mm
C. 15.5 in 16. From Problem No. 14, How thick of the rim, assuming that the hub and
D. 18.5 in should the flange be? arms account for 10% of the rim weight 24. From Problem No. 22, determine the
A. 0.823 in concentrated on the mean diameter. iJJitial tension on bolt in Newton.
12. A rectangular key was used in a B.0.1 23in A. 44,126.4 N
A . 5.6 mm
pulley connected to a lineshaft with a C. 0.423 in B. 34,126.4 N
B. 6.6 mm
power of 200 kw at a speed of 800 rpm. D. 0.723 in ~- 24,126.4 N
C. 7.6 mm
If the shearing stress of the shaft is 43 D. 14,126.4 N
D. 8.6 mm
N/mm2 and the key to be 25 N/mm 2 . 17. A flange coupling has an outside
Determine the length of the rectangular diameter of 200 mm and connects two 20. A punching machine flywheel 25. A sing le threaded trapezoidal metric
key if the width is one fourth that of the 50 mm shafts. There are five 20 mm thread has a pitch of 4 mm and a mean
accelerates from rest with a = 10
shaft diameter. bolts on a 150 mm bolt circle. The radians/sec-sec. In 20 seconds, how diameter of 18 mm. It is used as a
A. 157.5 mm radial flange thickness is 30 mm. If the translation screw in conjunction with a
many revolutions are achieved?
B. 177.5 mm torsional stress In the shaft is not to collar having an outside diameter of 37
P.-... 318.3rev
C. 197.5 mm exceed 28 MPa, determine the shearing mm and an inside diameter of 27 mm.
B. 318.3 rev
D 227.5 mm stress in the bolts if uniformly Find the required torque in N-m to raise
C. 318.3 rev
distributed. a load of 500 kg if the coefficient of
D. 318.3 rev
13. A key is to be designed for a 16 em A. 2.83 N/mm2 friction is 0.3 for both threads and collar.
shaft which will trarsmit power of 180 B. 4.83 N/mm2 21. A flywheel weighing 1000 kg has a A. 30.7 N-m
kw at 380 rpm. The width of the key is 4 C. 5.83 N/mm 2 radius of gyration of 1650 mm. The B. 40.7 N-m
2
em and the height is 3 em. If the D. 6.83. N/mm shaft journals are 304 .8 mm in diameter C. 50.7 N-m
allowable shear stress for the key is 950 D. '60.7 N-m
2 and have a coefficient of friction of 0.06.
kg/cm and the allowable compressive 18. A 50 inches diameter spoked steel After the wheel reaches 120 rpm, the
stress is 1300 kg/cm 2 , determine the flywheel having a 10 inches wide x 8 26. Determine the number of Yz in
driving force is withdrawn and the wheel
length of the key to be used. inches deep . rim rotates at 150 rpm. diameter set $Crews needed to transmit
slows to 60 rpm. How much energy
A. 2 em How long a ·cut (in inches) can be 12 hp at a shaft speed of 1350 rpm .
·B. 3 em thus the wheel loss?
stamped in 1 inch thick aluminum plate The shaft diameter is 1Y. in of SAE 4140
A. 151 .6 kN-m
C.4cm if . ultimate shearing strength of material
B. 251.6 kN-m
D. 10 em aluminum is 50,000 lbiin2 . During A. 1
C. 351 .6 kN-m
stamping, the force exerted by the B. 2
D. 451.6 kN-m
14. A flange coupling connects two-2 stamp varies from a maximum F lb at c. 3
inches diameter shafts. The flanges are the point of contact to zero lb ~JXhen the 22. A 12 em x 16 em air compressor is D.4
fitted with 6 bolts of SAE 1040 steel on a stamp emerges from the metal. Neglect operating with a maximum pressure of
8 inches bolt circle . The shaft runs at the weight of the flywheel and spokes 27. A 1% inch single threaded, square
10 kg/cm2 • There are five 12.7 mm bolts
400 rpm and transmits 60 Hp. Assume and use 0.28 lb/in 3 density for flywheel 2 screw has a TPI of 4 and coefficient qf
with a yield stress of 4300 kg/cm which
a factor of safety of 5, ultimate tension material. friction of 0.33. What is the efficiency?
held the cylinder head to the
of 78,000 psi, and ultimate shear of A. 46.6 inches A. 10.5%
compressor. The core gasket area is 60
60,000 psi. What is the torque B. 36.6 inches B. 12.7%
cm 2 with . a flange pressure of 120
transmitted? C. 26.6 inches
kg/cm2 . Determine the stress area of c. 15.7%
A. 9,453.8 in-lbs D. 16.6 inches D. 20.1%
the bolt used.
B. 8,453.8 in-lbs 2
A. 0.3338 in
2
B. 0.2338 in
REFRESHER MANUAL 2nd Edition by JAS TORDILLO REFRESHER MANUAL 2nd Edition by JAS TORDILLO
54 - 4 I Day 54- EXAM
Day 54- EXAM I 54-5
28. The large diameter and face of the A. 22.25 em Machine Design
disk of a multiple disk clutch are 255 B. 27.25 em
mm and 25 mm respectively. The C. 37.25 em C. 30 inches determine the whole depth of a 25
helical compression spring used to D. 47.25 em D. 25 inches degree ful l depth involute tooth.
engage the clutch has 12 effective coils A. 12.3 mm
of 10 mm steel wtre. The outer coil 32. From Problem No. 30, determine the 36. From Problem No. 34, determine the B. 13.2 mm
diameter is 80 mm. The free length of number of active coils . recommended center distance.
the spring is 185 mm. When in place A. 6.5 A. 10 inches C. 15.3 mm
with clutch engaged, its length is 130 B. 7.3 B. 20inches D. 20.3 mm
mm. Assuming that there are 12 pairs C. 9.5 C. 30 inches
of friction surface in contact, that the D. 10 D. 45 inches 42. A pinion rotating at 30 rev/sec and
motor runs at 11 00 rpm, with coefficient transmits 24.13 hp to a mating spur
of friction on contact surface f = 0.17, 33. A pulley 61 em in diameter transmits 37. From Problem No. 34, determine the , gear. If the pitch diameter is 10.2 em
determine the axial force available for 40 kw at 500 rpm. The arc of contact length of chiin in pitches. a{1d the load normal to the tooth is 2.03
the clutch. (G =83 GPa spring modulus) between the belt and pulley is 144 A. 198 pitches kN, determine the pressure angle of a
A. 1,116.3 N degrees, the coefficient of friction B. 258 pitches full depth involute tooth .
B. 1,386.3 N between the belt and pulley is 0.35 and C. 298 pitches A, 15.75 deg
C. 2,386.3 N the safe working stress of the belt is 2.5 D. 398 pitches B. 22.75 deg
D. 3,386.3 N MPa. Find the width of'the belt used if C. 25.75 deg
its thickness is 12 mm. 38. A disk clutch having an outside D. -30.75 deg
29. From Problem No. 28, determine the A. 143 mm diameter of 32 em and an inside
power that the clutch can transmit. B. 243 mm diameter of 12.7 em is connected to an 43. USC machine shop manufactures a
A. 27 .6 'rWV C. 343 mm engine that turns at 900 rpm. The· pair of spur gear of 3.5 module and to
B. 37.6 'rWV D. 434 mm coefficient of friction is 0.55 while the be installed in a center distance of 120
C. 47.6 'rWV pressure between the friction surfaces is mm. Assuming a speed ratio of 5:1 ,
D. 57.6 KW 34. Due to the high cost of electric 2.5 kg/em~, Compute the force on the determine the outside diameter of the
power, high maintenance cost and high clutch pedal necessary to disengage the pinion.
30. A dumb waiter designed to travel a investment of air-conditioning units, air clutch disc from the engine. A. 36.66 mm
height of 5 meters when loaded wi ll circulatiqg fans are becoming popular A. 1,693.93 kg B. 46.66 mm
have a maximum gross weight of 150 particuTarly in rural areas. A fast food B. 2,693.93 kg C. 56.66 mm
kg. A coil spring is provided below to owner has decided to install a used fan c. 3,693.93 kg D. 66.66mm
absorb shock in the event that the dumb that could deliver 18,000 CFM of air D. 4,693.93 kg
waiter should fall freely because of when running at 300 rpm. The fan is 44. From Problem No. 43, determine the
sudden breakage . of the wire rope estimated to require at least 4.5 HP to 39. From Prob!em No. 38, determine the outside diameter of the gear.
carrying it. If the coil spring will be deliver the volume of air needed. The power transmitted by the engine. A. 106.66 mm
deflected to 40 em, if the dumb waiter owner has also a second~hand motor A. 302.2 kw B. 306.66 mm
should accidentally fall and it hit the rated at 5 Hp and 1800 rpm. These B. 252.2 kw C. 206.66 mm
spring from its maximum height of data are all that the owner could provide C. 202.2 kw D. 406.66 mm
travel, determine the wire diameter. you. He wou ld like to use a standard D. 102.2 kw
NOTE: Use spring index of 9 and chain and sprocket for the transmission. 45. A band brake is installed on a drum
assume maximum induced stress and Determine diameter of small sprocket if 40. Determine the circular pitch, in rotating at 300 rpm, and a diameter of
shear modulus of elasticity to be 3900 the number of teeth is 21 using RC 40. inches, of a pair of gear and pinion with 800 millimeters. The angle of contact is
kg/cm 2 and · 820,000 kg/cm 2 A.1.35in ratio of 4:5 and the center distance of 12 1.5TT radians and one end of the brake
respectively. B. 2.35 in inches. The gear has 72 teeth. band is fastened to a fixed pin while the
·A. 4.25 em C. 3.35 in A. 0.986 in/tooth other end to the brake arm 140
B. 5.25 em D. 4.35 in B. 0.956 in/tooth millimeters from the fixed pin. The
· C. 6.25 em C. 0.856 in/tooth coefficient of friction is 0.25 and the
D. 7.25 em 35. From Problem No. 34, determine the D. 0.556 in/tooth straight brake arm is 900 millimeters
diameter of big sprocket. long and is placed perpendicular to the
31 From Problem No. 30. determine the A. 15 inches 41 . A set of gears with a gear ratio of diameter bisecting the angle of contact.
mean coil diameter. B. 20inches 4.5 is to be used at a ce.nter distance of Determine the minimum force in Newton
300 mm. If the gear has 90 teeth , applied at the end of the brake arm
REFRESHER MANUAL 2nd Edition by JAS TORDillO
REfRESHER MANUAl 2nd Edition by JAS TORDillO

-- ~- - - - - -- ------------..:'""'
:::L.._------------~---
54 - 6 I Day 54 - EXAM
Day 54 - Solution 1 54 - 7
necessary to stop the drum if 70 pulley are respectively 14 inches and 54
Machine Design
kilowatts is being absorbed. inches. The center distance is 54 inches
A. 172.5 N and both pulleys are made of cast iron. DAY 54 - SOLUTION s _ 16T
s-~

B. 272.5 N Coefficient of friction of leather on cast ;rD3


iron is 0.35. The allowable stress of the 1. A solid shaft is to be used to transmit 70,000 = 16T
C. 372.5 N belt is 350 psi and the belt weighs 0.035 90 'rWV at 750 RPM. If the shaft design
D. 572 5 N 3 1tD3
lb/in • What is the horsepower capacity stress will not exceed 29 Newton/mm2 ,
of the belt? determine the diameter of the solid
46. From Problem No. 45, If a steel T = 13,744.46D3 kN-m
A. 43.9 hp shaft.
band with maximum tensile stress of 60 B. 53.9 hp A. 48.6 mm
MPa and 5 mm thick will be used, what C. 63.9 hp B. 58.6 mm • e= TL
should be its width in millimeters? • D. 73.9 hp C. 68.6 mm JG
A. 16.8 mm
B. 26.8 mm
D. 78.6 mm
t.o(2._J = 3
13,744.6D (t.O)
C. 36.8 mm ) Power= 27t x Torque x Speed
180 1tD4
- --- (?9 300 000)
D. 46.8 mm 32 ' ' '
DAY 54- ANSWER KEY 90 = 27t(T)(750/60)
1. B .16. 8 31. D46. 8 T = 1.1459 kN-m = 1,145,915.6 N-mm D=0.1011 m=101.1 mm
47. A fiat-belt system has a small pulley 2. A 17. C 32.847.8
3. C 18. D 33. A48. 8 toT
with a 5-inch diameter and a large pulley s,=- 4. A 20 ft steel .lineshaft has no bending
with a 12-inch diameter. The small 4. D 19. 0 34. c ·4a. 8 Jr01
5 . c 20.8 35. 8 50. 8 action except its own weight. What
pulley transmits 2 hp at 2,700 rpm, and 6. D 21. A 36. c 29 = 16(1.145,9 15.6) horsepower can the shaft deliver at a
a back force of 10 pounds in the belt is 7. C 22. D 37.A speed of 300 rpm . Consider that the
7t03
required for frictional purposes. Find the 8. c 23. c 38. A
D = 58.6 mm torsional deflection will not exceed. 0.08
belt surface velocity. 9. 8 24. D 39. D
10. A25. 8 40. C deg/ft length.
A . 2,534.3 fVmin A. 38 hp
11. A 26. 8 41 . A 2. The shaft is subjected to a steady
B. 3,534.3 ft/min 12.827. 8 42. 8 B. 43 hp
C. 4,534.3 ft/min 13. 8 28. 8 load of 40,000 in-lb at .a shear stress of
43. 8 C. 65 hp
D. 5,534.3 ft/min 14. A29. 8 44. C 8,000 psi. Compute the diameter of the
15. C 30. B . 45.8 said shaft in inches D. 83 hp *
48. From Problem No. 47, Find the
A. 2.3 in •
output speed. B. 3.3 in 0 21) = _ L_
C. 4.3 in 8.95
A. 900 rpm
B. 1125 rpm D. 5.3 in
0 =(20)3/2
C. 1725 rpm
D. 2125 rpm
T =40,000 (f) = 20,000D in-lbs
8.95 =3.34 in

4
49. A 5.31 horsepower rating No. 40 s, = 16~ Hp = ( - 0 )4 N = ( -3·34)
4.6 4.6
- 300 = 83.38 hp
roller chain has an input through a 12- ~rD ·'
tocth sprocket turning at 1,800 rpm and 8.000 = 1 6(20,0~00) 5. A small shaft is 1 inch in diameter
the output turns at 900 rpm. Determine
Jt0 and has an allowable stress of 8000 psi.
the length of chain in inches.
D = 2.33 in Find the horsepower delivered by the
A. 15.7 inches
shaft at a speed of 20 rad/sec.
B. 20.7 inches
3. The torsional deflection of a SAE A. 1.25 hp
C. 30.7 inches
1040 steel shaft is to 1. 0° per meter B. 2.75 hp
D. 40.7 inches
length. The shear stress is 70 MPa. c. 4.75 hp.
Compute the diameter of the shaft in D. 5.50 hp
50. A fan is driven by a belt from ·a
mnlimeter. Steel modulus of elasticity is
motor which runs at 880 rpm . A
79,300 MPa. S 16T
medium double ply leather belt 5/16 inch s= -
A. 70 mm
thick and 10 inches ·wide is used. The Jt03
B. 80 mm
diameters of the motor pulley and driven
C. 101 mm •
D. 110 mm
REFRESHER MANUAL 2nd Edition by JAS TOROILLO
REFRESHER MANUAL 2nct Edition by JAS TOROillO

- ---- ----==
-....._-------------~-~~-~~-~
54- 8 I Day 54- Solution Day 54 - Solution I 54 - 9
Machine Design
8. Compute for the torsional deflection in
8000= ~
. 11:(1.0)3 degrees of a 90 mm diameter, 2.0 m C. 155.042.3 ft-lb D = shaft diameter = 65.6 mm
long shaft subj ected to twist moment of D. 195,042.3 ft-lb
T = 1,570.79 in-lbs = 130.89 ft-lbs 3.5 x 106 N-mm. The torsional modulus
20 6 T 2,387,324 = 72,784.2 N
N= ( 0) = 190.98 rev/ min of elasticity is 80,000 N/mm2 P = 2nTN F = D,i = 65.%
2Jr A. 0.52deg 12,000(33,000) = 2nT(600) /2
P = 2nTN B. 0.65 deg T = 1 05,04~ . 26 ft-lb
w
o 65.6 = 16.4 mm
= 4=-4-
= 2rr(1 30.89X190.98) = . h c. 0.78 deg *
4 76 D. 0.95 deg 11 . From P'roblem No. 10, determine
33,000 p
the inside diameter of. the shaft if the Shear stress of key:
TL
9 =-
maximum shearing stress based on the ,
Ss = F
6. A hollow shaft has an inner diameter torsion alone is not to exceed 8,000 psi.
of 0.0'4 m and an outer diameter of 0.07 JG li'L
A. 10.3 in •'
m. Compute the torque if the shear 6 25 = 72.784.2
_ (3.5xi 0 N - mm)(2000 mm) x 180 deg B. 12.5 in
stress is not to exceed 150 MPa in N-m. 16.4L
A. 6025 N-m
B. 7025 N-m
-
-- (90)4mm
n 4(80,000-N-2 ) n rad
C. 15.5 in
D. 18.5 in b. = 177.5 mm
32 mm
C. 8025 N-m 13. A key is to be designed for a 16 em
= 0.778 deg Ss = __
16TDo
n[o; -Dn
D. 9025 N-m * shaft which w ill transmit power of 180
kw at 380 rpm. The width of the key is 4
9. Find the power in watts transmitted
em and the height is 3 em. If the
s,. = !6TD" by a main power~transmitting shaft with
IT~,~ -
8000 = 16(105,042.26xl2)12 allowable shear stress for the key is 950
Di4 ] diameter of 90 mm and ·Speed of 500
.

150 X 106 = ~ 16T(0.07)


rpm n[ (t2)
4
-ot) kg/cm2 and the allowable compressive
stress is 1300 kg/cm 2 , determine the
A . 106,837.2 W
n (O.D7)4 - (0.04)4] D1 = 10.26 in length of the key to be used.
B. 206,837.2 W *
A. 2cm
T = 9025 N-m C. 306,837.2 W
12. A rectangular key was used in a B. 3 em*
D. 406,837.2 W
7. In SAE 1030 steel 1Y2 inch diameter pulley connected to a lineshaft with a C.4cm
solid shaft with a deflection not to power of 200 kw at a speed of 800 rpm .. . D. 10cm
For main-power transmitting shaft:
exceed 0.08 deg/ft length. Modulus of If the shearing stress of the shaft is 43
D 3N N/mm2 and the key to be 25 N/mm 2 . ·
rigidity is equal to 12,000,000 psi. P= -
Determine the power transmitted if the 80 Determine the length of the rectangular
shaft rotates at 330 rpm . where: key if the width is one fourth that of the
shaft diameter.
A. 1.8 hp D = ~=3.54 in A. 157.5 mm w = 4 em
B. 2.5 hp 25.4
B . 177.5 mm * h = 3 em
C. 3.6 hp *
D. 4.5 hp
P = (3.s4}3(soo) C. 197.5 mm
80 D. 227. 5 mm Power= 2n x Torque x Speed
p = 277.2 hp 3
0=-
TL
p = 277.2 (746) P = 2rrTN 180 = 2n T ( :~ )
JG p = 206,837.2 w
l
O0 .(.-2:.._ = T(l 2)
10. A power plant turbine developing
200 = 2rrT ( -800)
60
T = 4.52335 kN-m = 452.335 kN-cm

. \. t8o, Ln2)1.sl4(12 xt o6 ) T = 2.3873 kN-m = 2,387,324.14 N-mm F = force = torque _ 452.335


12,000 hp and it turns the shaft at 600
rpm. The propeller which is attached radius - 1'6/2
T = 693.95 in-lbs = 57.83 ft-lbs
the shaft develops a thrust of 120,000 Shear stress of the shaft: = 56.54 kN
lbs. A hollow steel shaft with an outside 16T
Power = 2rr TN Ss = -
diameter of 12 in will be used, compute Let L = Length of key
= 2 .1( (57 .83) (330) (-hp_)
33,000
for the torque.
A. 105,042.3 ft-lb *
43
;r/):.

= 16(2,3 87.324) Solving for L based on allowable


Power= 3.63 hp B. 125,042.3 ft-lb n0 3 shearing stress:

REFRESHER MANUAL 2nd Edition by JAS TORDILLO REFRESHER MANUAL 2nd Edition by JAS TORDILLO
54 - 10 1 Day 54 - Solution Day 54 - So lution 1 54 - 11
Machine Des ign
S, = force C. 0.204 in*
D. 0.104 in F = force on bolts = Torco<i! _ 607.223.39 V = velocity of flywheel rim at centerline
shear area Df2 -~~
F=9162.97 N 2 = 11D,.N = n( l2
42 )( 60
150) = 27.49 ft/sec
950(0.00981) = 56.54 T = F (r)
'
4 L 9453 8 916 7
E = energy supplied by flywheel
L = 1.516 em
F= 8 · = 2 ' 363. 45 lbs Fb =
5
B = 1,832.59 N
w 2
= -(V1 - V2)
2
2 -,
/o
Solving for L based on allowable 2363.45
Shearing Stress on bolt = !..!!.
A,,
= (~)(20}'
J.ll32.s~
compressive stress: F per bolt= - - = 393.91bs
6 = 5.83 N/mm2 ' Assuming all the energy of the flywheel
Sc = force
S=-
F is spent in the stamping operation,V2 0=
(~} A
60,000 393.9
18. A 50 inches diameter spoked steel
flywheel having a 10 inches wide x 8
E = 2,955.6 1 [(2749)2
2(32.2)
-o]
5654 - 5 - = (7t/4)d2
inches deep rim rotates at 150 rpm.
1300(0.00981) = = 34,682.52 ft-lbs
How long a cut (in inches) can be
(~} d = 0.204 in stamped in 1 inch thick aluminum plate
if ultimate shearing strength of
Let F = maximum force
L = 2.95 em 16. From Problem No. 14, How thick
should the flange be?
aluminum is 50.000 lb/in 2 During ( F;o X~) = 34,682.52
stamping, the force exerted by the
Therefore, L = 2.95 em say 3 em A. 0.823 in stamp varies from a maximum F lb at F = 832,380.48 lbs
B. 0.123 in* the point of contact to zero lb w hen the
14. A flange coupling connects two-2 C. 0.423 in stamp emerges from the metal. Neglect Let L = length of cut
inches diameter shafts. The flanges are D. 0.723 in the weight of the flywheel and spokes
fitted with 6 bolts of SAE 1040 steel on a and use 0.28 lb/in 3 density for flywheel
Stress = Force =..!:..
Area Lt
8 inches bolt circle. The shaft runs at F material.
400 rpm and transmits 60 Hp. Assume S=-
A A. 46.6 inches 50 000 = 832,380.48
a factor of safety of 5, ultimate tension B. 36.6 inches ' L(l)
78,000 393.9
of 78,000 psi, and ultimate shear of --=-- C. 26.6 inches L = 16.64 inches
60,000 psi. What is the torque 5 1(0.204)
D. 16.6 inches * 10"
t = 0.123 in
transmitted?
A. 9,453.8 in-lbs * t 19. It is found that a shearing machine
requires 300 J of energy to shear a
B. 8,453.8 in-lbs 17. A flange coupling has an outside 8" specific gauge of sheet metal. The
C. 7,453.8 in-lbs
D. 5,453.8 in-lbs
diameter of 200 mrn and connects two
50 mm shafts. There are five 20 mm t mean diameter of the flywheel is to be
762 mm. Th~normal operating speed is
bolts on a 150 mm bolt circle. The 200 rpm, and slows down to 180 rpm

If
radial flange thickness is 30 mm. If the

~
during shearing process. The rim width
torsional stress in the shaft is not to 50" is 304.8 mm. and the weight of the cast
exceed 28 MPa, determine-the shearing iron is 7,200 kg/m 3 . Find the thickness
stress in the bolts if uniformly of the rim , assuming that the hub and
'--r------1 distributed. arms account for 10% of the rim weight
A. 2.83 N/mm2 concentrated on the mean diameter.
B. 4.83 N/mm2 A. 5.6 mm
-~> <-- C. 5.83 N/mm2 • B. 6.6 mm
t D. 6.83 N/mm2 D,. = mean diameter of flywheel C. 7.6 mm
Power = 2nTN
60(33,000) = 211T (400)
T = 787.817 ft-lbs = 9,453.8 in-l bs
S.- •_ 16T
~
rrD'
= 50-2(f) =42in
D. 8.6 mm *

E = '!!.._ (V1 2 = V22 )


28 = ~ W = weight of flywheel rim 2g
15. From Problem No. 14, Determine rr(So)3
= volume x density where
the diameter of bolts required.
T = 687,223.39 N-mm = rr(42)( 10)(8)(0.28) E = energy required to shear
A. 0.404 in
B. 0.304 in = 2,955.61 lbs I = 300 J = 300 N-m

REFRESHER MANUAL 2nd Edition by JAS TORDILLO REFRESHER MANUAL 2nd Edition by JAS TORDILLO
Day 54- Solution 1 54 -13
54 - 12 1 Day 54 - Solution
Machine Design
g = 9.81 m/sec· shaft journals are 304.8 mm in diameter
and have a coefficient of friction of 0.06. Sy = 4300 X 2.205 X (2.54) Find the required torque in N-m to raise
V, =nON,= n(0.762) ( 6Q
200)
After the wheel reaches 120 rpm, the = 61 ,170.84 psi a load of 500 kg if the coefficient of
friction is 0.3 for both threads and collar.
driviog force is withdrawn and the wheel = 1>1.170.8 A5 1·5
= 7.98 m/sec slows to 60 rpm. How much energy 499 A. 30.7 N-m
6 B. 40.7 N-m *
V2 = nDN2 = n(0.762) ( 180) 6o thus the wheel loss? As= 0.1338 in2 C. 50.7 N-m
A. 151 .6 kN-m *
B. 251.6 kN-m D. 60.7 N-m
= 7: 182 m/sec
W = weight of the flywheel C. 351 .6 kN-m
D. 451 .6 kN-m 500 kg
30.48 em
t (Jl
1
= lZO~x360° Xrrradx
min rev !80°
min
60 sec
I

t = 41T rad/sec

76.2 em
W2 = 60~x360" x~x
min rev 180°
min
60 sec
23. From Problem No. 22, Determine
the torque applied on bolt in N-mm.
A. 10,881.05 N-mm
B. 25,881 .05 N-mm
@7mm
1;7 m~
= 21T rad/sec C. 35,881.05 N-mm*
D. 40,881 .05 N-mm
k' -
KE = ~(wl- wl) Collar
2
F-, = initial t~nsion on bolt= 120(60)
2 5
KE = 1oooc1.6J [(41r) 2 _ . (Z1r)2] = 1440 kg Tc = torque to overcome collar friction
~[(7.98)2 -(7.182)2]
2
300 = 37 27)
2(9.81) KE = 151,597.12 N-m = 151 .59 kN-m T =torque applied on bolt= 0.20 D F,
= fNl!'o
~ ) 0.3(500{
+~ = 2
+
2
W = 486.47 N = 49.59 kg = 0.20( 1.27)(1440) = 365.76 kg-em
2 2
22. A 12 em x 16 em air compressor is _ k 9.8 1 N . 10nun = 2400 kg-mm
W, + 0.10W, =W = 49.59 kg operating with a maximum pressure of - 36576
. g-cmx - - x - -
kg Cll1
w, = 45.08 kg 10 kg/cm 2 . There are five 12.7 mm bolts
Ts =torque to turn screw
Weight = Volume x Density
45.08 = n(O. 762)(0.3048)(t)(7200)
with a yield stress of 4300 kg/cm 2 which
held the cylinder head to
compressor. The core gasket area is 60
the
= 35,881 .05 N-mm

24. From Problem No. 22, determine the


= W D 111 [ cos $ tan x +
' 2 eos $ - f tan x
f]
t = 0.008581 m = 8.58 mm cm 2 with a flange pressure of 120 initial tension on bolt in Newton.
kg/cm 2 Determine the stress area of A. 44,126.4 N Trapezoidal metric thread is similar to
20. A punching machine flywheel the bolt used. B. 34 ,126.4 N Acme thread in which
accelerates from rest with a = 10 A. 0.3338 in
2 C. 24,126.4 N f/J = 14.5°, therefore
radians/sec-sec. In 20 seconds. how B. 0.2338 in 2 D. 14,126.4 N * Cos 0 =cos 14.5° = 0.968
many revolutions are achieved? C. 0.1038 in2 Lead 4
2
can x = - - = - - = 0.0707
A. 318.3 rev D.0.1338in * Initial Tens ion on Bolt nDm n(l8)
B. 318.3 rev *
= 1440 kg X Q.BI N Ts = 500(18)[0.968{0.0707)+ 0.30]
C. 318.3 rev F = force on cylinder head ku
"' 2 0.968- 0.30(0.0707 )
D. 318.3 rev = pressure x area
= 14,126.4 N = 1751.14 kg-mm
e = w,t + y. ae .= 10(~) 2
(12) = 1131 kg
25. A single threaded trapezoidal metric
= 0 + Y.(10)(20)2 T = total torque= 2400 + 1751.1
Load per bolt= 1131/5 = 226.2 kg thread has a pitch of 4 mm and a mean = 4151 .1 kg-mm
0 = 2000 rad = 318.3 rev
S y A s3/ 2 diameter of 18 mm. It is used as a
4~51.1(9.81) = 40.72 N-m
F. = - - translation screw in cot<junction with a
21 . A flywheel weighing 1000 kg has a 6 1000
collar having an outside diam~ter of 3'7
radius of gyration of 1650 mm. The F.= 226.2 x 2.205 = 499 lbs mm and an inside diameter of 27 mm .
... ~. · -.

REFRESHER MANUAL 2nd Edition by JAS TORDILLO REFRESHER MANUAL 2"fEdition by JAS TORDlLLO
54· 14 1 Day 54· Solution Day 54 -Solution I 54 - 15
Machine Design
26. Determine the number of Y2 in Dm = mean diameter = 1. 75- 0.125
diameter set screws needed to transmit = 1.625 in Fa ="1 .38ElkN = 1,386.3 N
12 hp at a shaft speed of 1350 rpm. l10o kgl
The shaft diameter is 1Y2 in of SAE ~ 140 Tan It = __!:.___ _ 0.25 29. From Problem No. 28, determine-~he
material xDm - n(l.62S) = 0.04897

NFc'
power that the clutch can transmit. ...
A. 1 A. 27.6 KW
}m
~ I~
B. 2* Eff = tan.-1.(1-ftan..l.)
B. 37.6 KW *
C. 3 tan..\+ f
C. 47.6 KW .
D. 4 Eff = 0.04897[1- 0.33(0.04897) D. 57.6 KW
0.04897 + 0.33
DNd 2·3 Eff = 12.71% Do= 255 mm r?.::..~ ~~
P=--
50 D, =255 - 2(25) = 205 mm ~
'· • ~- ~- 2::~
11 I it I

P = horsepower transmitted per set 28. The large diameter and face of the _ 2 r 0 3 - r[3 2 (127.5)3 - (102.5)3
disk of a multiple disk clutch are 255 rr- - -- -=-
screw 3 f6'J 2 3 (127 . )•)2 (1014,..)•)2 .. D
mm and 25 mm respectively. The - f. -

helical compression spring used to


I C= _!]!_ =9 or Dm = 9d
D = shaft diameter, inches
engage the clutch has 12 effective coils
=115.453 mm '- d
K = 4C-I + 0.6l5
of 10 . mm steel wire. The outer coil
D = diameter of the set screw, inches T = n f Fo rr= 12(0.17)(1 .386)(0.115453) 4C-4 C
diameter is 80 mm. · The free length of
the sprin9 . is 185 mm. When in place
=0.3264 KN-m = 4(9)-1 + 0.615 = 1.162
1 0
~~ )
2
p = 1.5(1350) (0.5) ·3 with clutch engaged, its length is 130 Power= 2nTN = 2n(0.3264) ( 4(9)- 4 9
50 mm. Assuming that there are 12 pairs Let F = maximum force on the spring
= 8.224 Hp per set screw of fricti"on surface in contact, that the = 37.6 IWJ
motor runs at 1100 rpm, with coefficient The energy of the falling weight is
of friction on contact surface f = 0.17, 30. A dumb waiter designed to travel a absorbed by the spring.
No. of set screw = ~ determine the axial force available for height of 5 meters when loaded will
. 8.22 150(500+40)= O+F (40)
the clutch. (G =8.3 GPa spring modulus) have a maximum gross weight of 150 2
= 1.45 say 2 set screws kg. A coil spring is provided below to
A. 1,116.3N F =4050 kg
8.1,38~ N* absorb shock in the event that the dumb
27 . A 1o/. inch single threaded, square waiter should fall freely because of
C. 2,386.3 N
screw has a TPI of 4 and coefficient of
D. 3,386.3 N sudden breakage of the wire rope S, = K 8FDm3
friction of 0.33. What is the efficiency? carrying it. If the coil spring will be nd
A. 10.5% deflected to 40 em. if the dumb waiter 3900 = 1.162(8) ( 4050) (9d)
B. 12.7% * should accidentally fall and it hit the
c 15.7% ""'- nd3
spring from its maximum height of
D. 20.1% d = 5.25 em
d travel, determine the wire diameter.
NOTE: Use spring index of 9 and
n = 12 coils 31 . From Problem No. 30, determine the
d=lOmm
assume maximum induced stress and
mean coil diameter.
D.= 80 mm shear modulus of elasticity to be 3900
2 A. 22.25 em
Dm = 80 - 10 = 70 mm Free length kg/cm 2 and 820,000 kg/cm ,
B. 27.25 em
= 130mm respectively.
C. 37.25 em
y = 185 - 130 = 55 mm A. 4.25 em D. 47.25 em •
B. 5.25 em •
C = -Dm C. 6.25 em Dm = 9d =9(5.25) =47.25 em
Pitch =
___ . :. . _____ =_I_ = _: d
70- 7
=-
10- D. 7.25 em
Teeth per lnch(TPI) TPI 4
32. From Problem No. 30, determine the
y= 8rc 3 n number of active coils.
= 0.25 inch per tooth Gd
A. 6. 5
3
0.055 = 8 Fa(7) (12) B. 7.3 *
Depth =
0 25
· = 0.125 in 83,000,000(0.0 J) c. 9.5
2 ' . o. 10
REFRESHER MANUAL 2nd Edition .by JAS TORDILLO REFRESHER MANUAL ir•ii·,E.:dition by JAS TORDIL.LO
54- 16 1 Day 54- Solution Day 54- Solution 1 54- 17
Machine Design
3 when running at 300 rpm . The fan is
y = 8FC n
Gd estimated to require at least 4.5 HP to For uniform pressure:
- C Tl +T2 (T2 - TJ)2
deliver the volume of air needed. The L - 2 + - - + -'---"---'-'-- F = P 1t (R2-~) = 2.5 (1t)[(16)2 - (6 35)2)
= 8(4050) (9)3n owner has also a second-hand motor
2 40C
= 1,693.93 kg
40 3
820.000(5.25) rated at 5 Hp and 1800 rpm. These where: C = ~ = 60 pitches
n = 7.29 active coils data are all that the owner could provide 0.) 39. From Problem No. 38, determine the
you. He would like to use a standard 2 power transmitted by the engine.
chain and sprocket for the transmission.
L = 2( 6 0) 2 1+126 + (126-21)
· A. 302.2 kw
33. A pulley 61 em in diameter transmits 2 40(60)
40 kw at 500 rpm. The arc of contact Determine diameter of small sprocket if B. 252.2 kw
between the belt and pulley is 144 t he number of teeth is 21 using RC 40. = 198.09 say 198 pitch es C. 202.2 kw
degrees, the coefficient of friction A. 1.35 in D. 102.2 kw *
between the belt and pulley is 0.35 and B. 2.35 in Big Sprocket
the safe worki ng stress of the belt is 2.5 C. 3.35 in* 02 r1 -;::. mean friction radius
D. 4.35 in 300 rpm 3
MPa. Find the width of the belt used if 2 R 3 - r3 2 ( 16) 3 - (6.35)
its thickness is 12 mm. ,- 3 R - r = 3 (16) - (6.35) 2
2 2 2
T 1 = 21 teeth and P = Y. inch
A. 143_mm *
B. 243 mm
C. 343 mm
D. 434mm
o',
s1n
tJT
s1n .
5
= - 180 = . ('180 , = 3.35 in
\. 21
= 11 .869 em .

T = torque = nfFrr
= 1(0.55)(1693.93)(11.869)
= 11,057.89 kg-em
Power = 27tTN 35. From Problem. No. 34, determine the = 11,057.89 kg-em x
40 = 2Jt T (500/60) diameter of big sprocket.
T = 0.763944 kN-m A. 15 inches 0.0098 1 kN m
- ---x--
B. 20 inches * 1800 kg 100 em
T = (F, - Fz) r C. 30 inches =1.084779 kN-m
0.763944 = (F1- F2) (0.610/2) D.25inches 38. A d1sk clutch having an outside
Power = 27tTN = 21t(1 .084779)( ~~)
9
F,- F2 = 2.505 kN = 2505 N (Eq. 1) diameter of 32 em and an inside
1800 diameter of 12.7 em is connected to an
2 : e' 9 : e(0.35) (144 X w/180) : 2_41 T 2 =21r: ) =126teeth =102.2kw
Fz -~: .300 engine that turns at 900 rpm. The
coefficient of friction is 0.55 while the
o.s 20-InChes 40. ' Determine the circular pitch, in
F2 = .f.L (substitute the value of F2 to D2= -- = pressure between the friction surfaces is
. 180 inches, of a pair of gear and pinion with
2.41 Stn 2.5 kg/cm 2 Compute the force on the
Eq. 1) 126 ratio of 4.5 and the center distance of 12
clutch pedal necessary to disengage the
inches. The gear has 72 teeth.
clutch disc from the engine.
F1 - ..2_ = 2 t 505 36. From Problem No. 34, determine the A. 0.986 in/tooth
2.4 1 A. 1,693.93 kg *
recommended center distance. B. 0 .956 in/tooth
F, = 4282 N B. 2,693.93 kg C. 0.856 in/tooth*
A. 10 inches C. 3,693.93 kg
B.20 inches D. 0.556 in/tooth
Sw = working stress :2 D. 4,693.93 kg
bt C. 30 inches*
D. 45 inches c = 12

~
4262
2.5 : b(lZ) .!'-!!_ = T2 = D2 =4 _5
b = 142.7 mm
C =1.5D2 =1.5(20) = 30 inches N2 T1 D1

1~9P
D2 =4.5D1
34. Due to the high cost of electric 37. From Problem No. 34, determine the
length of chain in pitches. C = _D_
I +_D_2
power, high maintenance cost and high
A. 198 pitches* 2
investment of air-conditioning units, air
circulating fans are becoming popular B. 258 pitches = _D_,_J_+_4 ._SD
-----'1
C. 298 pitches 12
particularly in rural areas. A fast food 2
owner has decided to install a used fan D. 398 pitches D 1 =4.36 in
that could deliver 18,000 CFM of air T2 = 4.5T1

REFRESHER MANUAL 2nd Edition by JAS TORDILLO REFRESHER MANUAL 2n'J-~difion by JAS TORDIL~~
54 - 18 1 Day 54 - Solution Day 54 -Solution I 54 - 19
Machine Design
7
~ -- 16
T,=---:
V = pitch line velocity = n:DN
ODp = 40 +' 2(3.33) = 46.66 mm = 2.22817
= n(0102)C!~0 )
4 .) F, _ F
=9.613m/sec 2
0.40
no,
Pc = Tl . Power 18
44. From Problem No. 43, determine the = 5.57042 kN Eq. 1
F, = tangent1alload = - - -
Velocity
=-9.613
.- outside diameter of the gear.
_ n{4.36) = 0.856 in/tooth A. 106.66 mm
- 16 = 1.872 kN B. 306.66 mm !}_ = etB = e0.2S( I.Sn) =).248
Fn = 2 .03 kN C. 206.66 mm ·* F2
D. 406.66 mm F, = 3.248F2 Eq.2
41 . A set of gears with a gear ratio of
1 872
4. 5 is to be used at a center distance of Cos e = £L = · = 0.922 3.248F2- F2 = 5.57042
300 mm. If the gear has 90 teeth, Fn 2.03 009 = 200 + 2(3.33) = 206:66 mm
determine the whole depth of a 25 ' F-2 = 2.4779 kN
8 = 22.75 deg
degree full depth involute tooth. 45. A band bnake is installed on a drum
rotating at 300 rpm, and a diameter- of Summation -of moments about the fixed
A . 12.3 mm * 43. JST machine shop manufactures a
800 millimeters. The angle of contact is end= 0
8 .. 13.2 mm pair of spur gear of 3.5 module and to
1.5rr radians and one end of the brake F2 ain 45°(140)- Fa(900) = 0
C. 15.3 -mm be installed in a center distance of 120
band is fastened to a fixed pin while the Fa= 0.1 09994F2 =. 0.1 09994(2.4779)
D. 20.3 mm mm. Assuming a speed ratio of 5:1,
other end to the brake arm 140 = 0.2725 kN = 272.5 N
determine the outside diameter of the
millimeters from the fixed pin. The
pinion.
Gear ratio = T2 =..':::1. = 4.5 A 36.66 mm coefficient of friction is 0.25 and the 46. From Problem No. 45, If a steel
T1 N2 straight brake arm is 900 millimeters band with maximum tensile stress of 60
B. 46.66 mm * MPa and 5 mm thick will be used, what
Speed ratio = ~= 0
2 = 4.5 C. 56.66 mm long and is placed perpendicular to the
should be its width in millimeters?
N2 Dt D. 66.66 mm diarneter bisecting the ang le of contact.
Determine the minimum force in Newton · A. 16.8 mm
0 2 = 4.5D,
applied at the end of the brake arm B. 26.8 mm *
- - =120mmEq.(1)
C= Dp+Dg necessary to stop the drum if 70 C. 36.8 mm
C= D2+Dt 2 kilowatts is being absorbed. D. 46.8 mm
2 Np _ Dg =5 A. 172.5 N
Eq. (2)
300 = 4.5Dt + Dt Ng- Dp B. 272.5 N * . s = 11
., C. 372.5 N A
o, = 1'09.1 mm and _0 2 = 4.5(109.1) =
Dp +SOp= 240
D. 572.5 N '
60 000 = 3.248(2.4779)
490.9 mm Dp = pitch dia. of pinion = 40 mm ' 0.005 b
0 9 =pitch dia.of gear= 5(40) = 200 mm 0 = 800 mm
Diametral Pitch , b = 0.0268 m =-.26.8 mm
Module= pitch diameter = E_ =3 _5 F.
9 47. A flat-belt system has a small pu lley
Pd= T2 = ( 0)
02 490.9
=

W hole depth = 2 .25/Pd = 2.25/0.1 833


0. 1833tooth/mm no. of teeth N

Np = no. of teeth of pinion =~


3.5
l with a 5-inch diameter and a large pulley
with a 12-inch diameter. , The small
pulley transmits 2 hp at 2,700 rpm , and
= 12.27 mm =12 teeth a back force of 10 pounds in the belt is

42. A pinion rotat1ng at 30 rev/sec and


transmits 24.1 3 hp to a mating spu r
200
N9 =no. of teeth of gear =-
3.5 ~ goo mm
required for frictional purposes. Fi nd the
belt surface velocity.
A. 2,534.3 ft/min
= 57 teeth
gear. If the pitch diameter is 10.2 em Power= 2rrTN B. 3,534.3 ft/min *
and the load normal to the tooth is 2.03 _ . I 't h _ circumferenct: ltD
P c - c1rcu ar p1 c - =-
kN, determine the pressure angle of a
full depth involute tooth. 11(40)
no. of teeU1 N
10 = 2nre;0°) C. 4,534.3 ft/min
D. 5,534:3 ft/min

A. 15.75 deg Pep = Pc9 = - - = 10.47 mm T =2.22817 kN-m


B. 22.75 deg *
12 Vm = nON
C. 25.75 deg a= addendum= 1/Pd =PJn T = (F1- F2)r
12
D. 30. 75 deg = 10.47/n = 3.33 mm
Outside Dia. = Pitch Dia. + 2(a)
I

REFRESHER MANUAL 2nd Edition by JAS TORDILLO REFRESHER MANUAL2n~ifion by JAS TORDILLO
54 - 20 I Day 54 -Solution Day 55 - EXAM I 55 - 1
Machine Design
Vm = rr(5)2,700 L = 2(5.745) + 1.57(1.93 + 3.83)
6. The torsional deflection of steel shaft
12 + (3.83-1.93)2 DAY 55- EXAM is 0.6 deg per meter length. The shear
Vm = 3,534.3 ft/min 4(5.745) stress is 60 Mpa. Compute the diameter
L = 2'0.7 inches 1. A hollow shaft with outside diametet of the shaft in mm. Steel modulus of
48. From Problem No. 47, Find the of 14 em and wall thickness of 0.80 em elasticity is 79 Gpa.
output speed. 50. A fan is driven by a belt from a transmits 200 kw at 400 rpm. What must A. 90 mm
A. 900 rpm motor which runs at 880 rpm. A be the angular deflection of the shaft if B. 130 mm
B. 1125 rpm* medium double ply leather belt 5/1Q inch the length is 5 meters? Take G = C . 105 mm
C. 1725 rpm thick and 10 inches wide is used. The 12,000,000 psi. D. 145 mm
D. 2125 rpm diameters of the motor pulley and driv~n A. 0.019 deg
pulley are respectively 14 inches and 54 B. 1.94 deg 7~ A small shaft is 2 inches diameter and
C. 1.14deg
mw =02
- inches. The center distance is 54 inches has an allowable stress of 8500 psi.
01 and both pulleys are made of cast iron. D. 2.44 deg Find the horsepower delivered by the
12 2.4 Coefficient of friction of leather on cast shaft"at a speed of 18.85 rad/sec.
mw = =- - iron is 0.35. The allowable stress of the 2. The shaft is subjected to. a steady A. 25'-hp
5 1 load of 40,000 lb at a shear stress of
belt is 350 psi and the belt weighs 0.035 B. 38 hp
The output speed is
lb/in3 . What is the horsepower capacity 12,000 psi. Compute the diameter of the c. 32 hp
2,700 said shaft in, inches.
N = 2:4 =1,125 rpm of the belt? D. 45 hp
A. 43.9 hp A. 3
B. 53.9 hp * B. 2 8. A kilowatt is not equal to
C. 63.9 hp C. 2.5
49. A 5.31 horsepower rating No. 40 D. 73.9 hp D. 1.5 A. 8857.2 in-lb/sec
roller chain has an input through a 12- B. 57 Btu/min
tooth sprocket turning at 1,800 rpm and 61 = 180°- 2 sin' 1 (R- r)/C 3. Compute the safe wall thickness of a C. 1140 Joule/sec
the output turns at 900 rpm. Determine = 108°- 2 sin'1 (27 - 7)f54 = 136.6° 90 em diameter steel tank. The tank is D. 44,200 ft-lb/min
the length of chain in inches. 62 = 180° + 2 sin' 1 (R - r)/C = 223.4° subjected to 7 MPa pressure and the
A 15.7 inches steel material has a yield stress of 220 9. The total weight of steel plate size, 1
B. 20.7 inches* MPa. Use a factor of safety of 3. x 4 20 is:
F1 -wv 2 tg te A. 4.3 em
C. 30.7 inches -'---....,.-=- =e A. 33721b
D. 40.7 inches ~ - wV2 tg B. 3.2 em 8. 2372 lb
1800 w = 12(10)(5/16)(0.035) = 1.31 lb/ft. C. 3.8 em C. 32721b
12 tooth • = 24 teeth 2 D. 2.5 em
900 g =32.2 ft/sec , D. 21761b
D= _ _P
= =
v nDN n(14/12 ft) (880/60 rps)
4. A foot rule has how many 1 /16 inch?
= 53.7 ft/sec, 10. What is the working strength of a 2
. (180)
Sin
T
F1 = (350 lb'/in2 )(5/16 x 10 in2 ) = 10941b A . 210
B. 160
in bolt which is screwed up tightly in a
packed joint when the allowable working
C. 192 stress is 11 ,000 psi?
0.5 93 . . 1094 - !17 = e0.35(136.6rr/180) = 2.30 D. 96
D1 = ---:-180 = 1. ln. F2 - 117
A. 12,400 lb
Sin B. 18,700 lb
12 F2 = 542 .lb 5. Compute the diameter of a solid shaft C. 15,500 lb
0.5. transmitting 90 Hp at 1750 rpm. The D. 22,500 lb
D2= ~ = 3.83tn. Horsepower capacity allowable shear stress on pure torsion is
Sin
24 = (Ft-F2)v = (t094-542X53.7) 6000 psi. 11. Compute the deflection of a 20
550 550 A. 1 Y. in coils helical spring having a load of 120
C = 1.6D2 = 1.5(3.83) = 5.745 in
2 = 53.9 hp B. 1 3/4 in kgs. The modulus of elasticity in shear
L = 2C + 1.5l(D1 + D2) + (D 2 - Dtl C. 1 'h in of spring is 98 Gpa, outside diameter of
4C
D. 1 in 10 em and wire diameter of 10 mm. The
spring is squared and ground ends.
A 162.1 mm ·
B. 134.1 mm

REFRESHER MANUAL 2nd Edition by JAS TORDILLO REFRESHER MANUAL 2nd f<titlon by JAS TORDILLO
55-21 Day55-EXAM Day 55 - EXAM I 55 - 3
Machine Design
C. 154.4 mm B. 7.99 mm
D. 126.1 mm C. 3.93 mm 23. What force is required to punch a 29. The distance from a point on. a
D. 3.97 mm 1-in hole on a 3/4 in thick plate. if the 'screw thread to a corresponding point
12. A 72 tooth pm1on with a turning shear strength on the plate is 42 ,00Q: on the next Measure along the axis:
speed of 450 rpm drives 180 tooth gear 18. Determine the bursting steam psi? A radial
of 14 Y2 degree involute angle. What pressure of a steel shell with diameter of A 31,500TT lbs B. thickness
would be the speed of the driven gear ? 12 in and made of 1/4 in steel plate. The B. 39,800TT lbs C. module
A. 120 rpm joint efficiency is at 75% and the tensile C. 35, 100TT lbs D. pitch
B. 160 rpm strength is 55 ksi. D. 41 ,500rr lbs
C. 140 rpm A . 3,440 psi 30. Martensite (Stainless steel) contains
D. 180 rpm B. 3,430 psi 24. A 2 in diameter by 4 Inches long ~ to 26% Cr and a maximum of %
C. 3,437 psi journal bearing is to carry a load of nic1<el.
13. A cylindrical tank w ith 12 inches D. 3,427 psi 10,000 lb at 1150 rpm using, $AE 40 A. 2.5
inside diameter contains air at 2000 psi. lube oil. Compute the bearing pressure. B. 2.5
Calculate the required thickness under 19. If the, pitch of a screw is 2/7, find the A 1950 psi C. 3.5
stress of 25,000 psi. thread per Inch. B. 1250 psi D. 4.5
A. 0.48 mm · A. 2.5 C. 1750 psi
B. 12.2 mm B. 3.5 D. 1000 psi 31. Progression change of position of a
C. 4.8 mm C. 4.5 body.is called:
D. 21.9 mm D. 5.5 25. What modulus of plasticity A. acceleration
intension is · required obtain a unit B. force
14. Compute the tooth thickness of a 20. Two parallel shaft connected by deformation of 0.00110 m/m 'from a load ·c. motion
gear tooth having a diametral pitch of pure rolling him in the same direction prodt:~cing a unit tensile stress of 42,500 D. momentum
12. The gear tooth pressure angle is 14 and having a speed ratio of 4. What is psi? .
V2 degree full depth tooth. the distance of the two shaft if the small A 3.8.6 x 105 psi 32. Recommended hardness of pinion
A. 0.0309 in cylinder is 8 em? B. 38.6 X 103 psi for helical/herringbone gear tooth should
B. 0.0507 in A. 10 em C. 38.6 X 106 psi be _ _BHN point to sustain life.
C. 0.1309 in B. 18 em D. 38.6 X 107 psi A. 50-59
D. 0.1906 in C. 12 em B. 40-50
D. 20 em 26. It is ideal for maximum quietness in c. 48-65
15. What pressure Is, required for sprocket drive operation to choose _ _ D. 30-60
punching a hole 4 inches in diameter 21. What modulus of elasticity in or more teeth.
thru a Y2 inch steel plate? tension is, required to obtain a unit A 25 33. Fast process- of analyzjng all
A 120 tons deformation of 0.00107 m/m from a load B. 27 elements and chemical components of
B. 180 tons producing a unit tensiie stress of .50,000 C. 26 steel casting .
C. 160 tons psi? D. 28 A. pyrometer
D. 250tons A 34.5 X 106 psi B. Wet analyzer
B. 46.7 X 106 psi 27. Deals only with the motion of bodies C. carbon allalyzer
16. A bolt has a tensile lo ad of 300 lbs. C. 46.7 x 105 psi without reference to the forces that D. spectrometer
6
The bolt is made of SAE 1040 material D. 4.67 X 10 psi cause them:
With Sy = 55 ksi, what is the stress area A. kinetics 34. In small quantity this alloy is
of the bolt? 22. If the. ultimate she& strength of a B. motion effective for improving strength at high
A. 0.1023 in2 steel plate is 42,000 lblin2. what force is C. kinematics temperature.
B. 0.327 in 2 necessary to punch a 1.5 In diameter D. acceleration A. chromium
C. 0.1230 in 2 We in a 1/2 in thick plate? B. selenium
D. 0.5190 in2 A. 98,960 lbs 28. The shaft that is used to connect or C. manganese
B. 100,960 lbs disconnect the drive at will is called : D. molybdenum
17. Find the tooth thickness of a 20 C. 99,960 lbs A . clutch
degree full depth involute tooth having a D. 105,960 lbs B. brake shaft 35. The ratio .of the arc of action to
diametrat pitch of 4. C. bearing circular pitch or length of action to the
A. 9.97 mm D. lock shaft base pitch.

REFRESHER MANUAL 2nd Edition by JAS TORDILLO REFRESHER MANUAL 2nd Eqiflon by JAS TORDILLO
55-41 Day55-EXAM Day 55 - Solution 1 55 -5
Machine Design
A. approach ratio 42. The frictional force depends on
B. arc of action coefficient of friction and: 48. A body weighing 1000 lb falls 6
C. contact ratio A. torque inches and strikes a 2000 lb per in
DAY 55- SOLUTION
D. arc of approach B. normal force spring. The deformation of the spring is;.' ·
C. weight of the object A. 1 in 1. A hollow shaft with outside diameter
36. Which of the following Is recta kind D. moment B. 3 in of 14 em and wall thickness of 0.80 em
of mandrel? C. 2 in · transmits 200 kw at 400 rpm. What must
A. expanded 43. Not part of the headstock: D.6in be the angular deflection of the shaft if
B. contraction A. back gear the length is 5 meters? Take G =
C. extended B. anvil 49. For a single universal joint, shaft 12,000,000 psi.
D. taper C. spindle angle should be ___ - degrees A. 0.019 deg
D. motor maximum and much less if on high 8".'1.94 deg
37. A car is travelling 120 kph, it takes rotational speed: C. 1.14 deg •
0.5 sec to apply the brake after which it 44. Two or more forces acting together A. 12 D. 2.44 deg
decelerates at 8 m/s2. Compute the could be replaced by a single force with B. 15
distance it traveled in meters before it same effect in a mass. C. '16 G = 1-.2,000,000 psi
stops. A. couple of forces D. 14 = 82,714,285.71 kPa = 82.7 GPa
A. 15.6 B. resultant
B. 64.81 C. resolution of forces 50. The recommended lubricant for the P = 2rrTN
C. 26.5 D. concurrent of forces chain drive operation.
D. 69.4 A. SAE 4140 .200,000 = 2rrT ( 400) 6o
45. To prevent engine vibration to affect B. moly slip
38. All four compression coil spring the surrounding area, it is standard C. Petroleum oil
T = 4774.65 N-m
support one load of 700 kg. All four practice to isolate the engine and its D. heavy greas_e
springs are arranged in parallel and foundation by at least mm. Do = 14 em
rated same at 0.609 kg/mm. Compute A. 30 D; = 14 - 2(0.80) = 12.4 em
the deflection in mm. B.25 ((-j ) 1!{,4 4)
A. 64 c. 20 J = 32 \Do -Di
DAY 55 - ANSWER KEY
B. 178 D. 36
~
1.C 16. A 31.C46. B 71
C. 147 2. A 17. A 32. B 47. C = [(o.14)4 -(0.124)4 ]
D. 287 46. Grout material use under the 3. A 18. A 33. 048.8 32
machine bed and foundation surface 4. C 19.8 34.A49. B J = 1.45 x 10'5 m•
5. c 20. c 35. c 50. c
39. A section in a machine shop stress be: 6. D 21 . 8 36. 8
operation. A. 1 part cement and ~ parts screened 7.8 22. A 37.0
A. coremaking sand and 1/4 part gypsum 8. C 23. A 38. D
B. fitting B. 1 part cement and 1 part screened 9. C 24. 8 39. C
.. --,I
,..,x -180
C. machining sand 10.8 25. C 40. 0
11 .0 26. 8 41.8 .. J -
D. pattern C. 1 part cement and 1 part screened 12.0 27. C 42. 8
7t
sand and Y. part gypsum 13.8 28. A 43. B
40. Shear modulus is also: D. 1 part cement and 1 ~ part screened 14.C 29. D 44. B
e = 1.14°
A. shear elasticity sand and ~ part stone 15.C 30.8 45. 8
B. modulus of elasticity 2. The shaft is subjected to a steady
C. poisson's ratio
~ load of 40,000 lb at a shear stress of
47. A gear set having a gear ratio of 3 is
D. modulus of rigidity to be used at a center distance of 12,000 psi. Compute the diameter of the
inches. If the gear has 80 teeth, what said shaft in, inches.
41. To prevent leakage in dynamic seal: must be the circular pitch? A. 3 *
A. gaskets A. 23.6 mm 8.2
B. packings B. 29.3 mm C. 2.5
C. seals C. 23.9mm D. 1.5

T=F(~)
D. felts D. 32.9 mm

'
REFRESHER MANUAL 2nd·Edition by JAS TORDILLO REFRESHER MANUAL.2nd Eqlflon by JAS TORDILLO
55 - 6 1 Day 55 - Solution Day 55 - Solution 1 55 - 7
Machine Design

~) = 20,000D = 16T ;.
Ss 8. A ki lowatt is not equal to
T = 40,000 (
n03 y = 8FC3n = 8(120x9.8tX9)3(20-2)

6 000 = 16(3241.3) A 8857.2 in-lb/sec Gd 98xJo9 (0.0l)


Ss = 16T
7t03 ' 7t03 B. 57 Btu/m in Y = 0. 1261 m = 126.1 mm
D = 1.4 in use 1 Y2 in shaft
C. 1140 J9ule/sec *
12 000 = i6(20,0000) D. 44,200 ft-lb/min 12. A 72 tooth pinion with a turning
, n03 speed of 450 rpm drives 180 tooth gear
6. The torsional deflection of steel shaft
D = 2.91 in say 3 in is 0.6 deg per meter length. The shear
1 kw = 1000 IA(atts = 1000 J/s of 14 Y2 degree involute ang le. What
stress is 60 Mpa. Compute the diameter would be the speed of the driven gear?
3. Compute the safe wall thickness of a. 9. The total weight of steel plate size. 1 A,_ 120 rpm
of the shaft in mm. Steel modulus of
90 em diameter steel tank. The tank is elasticity is 79 Gpa. x 4 20 is: B. '160 rpm
subjected to 7 MPa pressure and the A. 3372 1b C. 140 rpm
A 90 mm
steel material has a yield stress of 220 B. 130 mm B. 2372 lb D. 180 rpm •
MPa. Use a factor of safety of 3. C. 105 mm
c. 32721b.
A. 4.3 em* • 0.145 mm * D.. 217q lb T,N, '= T2N2
B. 3.2 em 72(450) = 180N2
C. 3.8 em Density of steel, p = 0.284 lb/in3 N2 = 180 rpm
D. 2.5 em Ss = 16T
PO
1t03 Weight = density x volume 13. A cylindrical tank with 12 inches
St = - 16 = 0.284((1}(4 X 12)(20 X 12)]10. inside diameter contains air at 2000 psi.
2t 60x106 = T =327 1.68 lbs 'Cal<;ulate the required thickness under
7t03
220. = 7(90) stress of 25,000 ps i.
T = 11780972.45 D3 N-m 10. What is the working strength of a 2
3 2t A 0.48 mm
e=TL in bolt which is screwed up tightly in a B. 12.2 mm •
t = 4.29 em
JG packed joint when the allowable working C. 4 .8 mm
3 stress is 11,000 psi? D. 21 .9 mm
4. A foot rule has how many 1 /16 inch?
A. 210
0.6 (_.2:.._) 11,780,972.450 (t) A 12,400 lb
B. 160 18o 04 ( ) B. 18,700 lb * · PO
~ 79x109 C. 15,500 lb St = - ·
c. 192. 32 ' 41
D. 22,500 lb
D. 96 D = 0.145 m = 145 mm 25 000 = lOOo(l)
. 2t
1 foot rule = 12 inches 7. A small shaft is 2 inches diameter and
Wt =St [0 .55d 2 -0.25d]
= 11,000[0.55(2) 2 - 0.25(2)] t = 0.48 in = 12.2,mm
12in = has an allowable stress of 8500 psi.
192 ::: 18,700 lbs
II· Find the horsepower delivered by the 14. Compute the tooth thickness of a
/16m shaft at a speed of 18.85 rad/sec.
11. Compute the deflection of a 20 gear tooth having a diametral pitch of
A 25 hp 12. The gear tooth pressure ang le is 14
5. Compute the diameter of a solid shaft coils helical spring having a load of 120
B. 38 hp* V2 degree ful l depth tooth.
transmitting 90 Hp at 1750 rpm. The kgs . The modulus of elasticity in shear
C. 32 hp A. 0.0309 in
allowable shear stress on pure torsion is of spring is 98 Gpa, outside diameter of
D. 45 hp B. 0.0507 in
6000 psi. 10 em and wire diameter of 10 mm. Th'e
spring is squared and ground ends. C. 0.1309 in •
A. 1 Y. in N = 18.85 rad/sec = 180 rpm
A. 162.1 mm D. 0.1906 in
B. 1 3/4 in
C.1% in • D3 N
Power= - · -
B. 134. 1 mm
D. 1 in 38 C. 1544 mm Tooth thickness, t = 1.5708
D. 126.1 mm * pd
3
P = 2TTTN = (2) (180}
1.5708 = o.1309 in
90(33,000) = 2TTT(1 750) 38 Do = 10 em t = -,2
T = 270.1 _ft-lbs = 3241.3 in-lbs Power = 37.9 hp d = ·i Q mm = 1 em
C= ~=10- 1 = 9
d 1
REFRESHER MANUAL 2nd Edition by JAS TORDILLO REFRESHER MANUAL 2nd Eqlti'on by JAS TORDILLO
55 - 8 I Day 55 - Solution Day 55 - Solution 1 55 - 9
Machine Design
15. What pressure Is, required for strength is 55 ksi.
punching a hole 4 inches in diameter A. 3,440 psi " producing a unit tensile stress of 50,000 F F
thru a Y2 inch steel plate? B. 3,430 psi psi? sb = A LD
6
A. 120 tons C. 3,437 psi A. 34.5 x 10 psi
B. 180 tons D. 3,427 psi B. 46.7 x 106 psi sb 10,000 = 1250 psi
C. 160 tons • C. 46.7 x 105 psi 4(2)
D. 250 tons PO D. 4.67 X 106 psi.
St =- 25. What modulus of plasticity
4t
Empirical formula, Machinery's stress intension is required obtain a unit
Handbook: 55 000 = P(I 2) E = strain
deformation of 0.00110 m/m 'from a load
• 4(0.25) (0.75) fMOducing a unit tensile stress of 42,500
P = dxtx80 P = 3437.5 psi = 50,000 psi?
= 4xi'.x80 0.00107 A. 3.8.6 x 105 psi
= 160 tons Bursting pressure> 3437.5 psi. = 46.7 x 106 psi B. 38.. 6 x 103 psi
C. 38.6 x 106 psi*
16. A bolt has a tensile load of 300 lbs. 19. If the, pitch of a screw is 2/7, find the 22. If the. ultimate she& strength of a D. 38~6 x 107 psi
The bolt is made of SAE 1040 material thread per Inch. steel plate is 42,000 lb/in2, what force is
With Sy = 55 ksi, what is the stress area A. 2.5 necessary to punch a 1.5 In diameter stress
of the bolt? B. 3.5 * We in a 1/2 in th ick plate? E =- -
C. 4.5 strain
A. 0.1023 in2 * A. 98,960 lbs •
B. 0.327 in2 D. 5.5 B. 100,960 lbs 42,500
c. 0.1230 in2 Threads per inch = - -
1 C. 99,960 lbs 0.00110
D. 0.5190 in2 pitch D. 105,960 lbs 38.6 x 106 psi

Empirical formula from MD by Faires:


'/
= 741 = 3.5 F =
Ssu(lTdt)
F = 42,000( n: n)(1.5)(1/2)
26. It is ideal for maximum quietness in
sp rocket drive operation to choose _ _
SyAsh F = 98,960 lbs or more teeth.
Fe =
6 A. 25
20. Two paralle l shaft connected by B. 27 *
23. What force is required to punch a
= 55,000As.% pure rolling him in the same direction C. 26,
300 1-in hole on a. 3/4 in thick plate, if the
and having a speed ratio of 4. What is
6 shear strength on the plate is 42,000 0.28
2 the distance of the two shaft if the small
As = 0.1023 in psi?
cylinder is 8 em? 27. Deals only with the motion of bod ies
A. 31,50 0rr lbs *
A. 10 em without reference to the forces that
17. Find the tooth thickness of a 20 B. 39,8001T lbs
B. 18 em cause them:
degree full depth involute tooth having a C. 35 ,1001T lbs
C.12 em* A. kinetics
diametral pitch of 4. D. 41 ,5001T lbs
D. 20 em B. motion
A. 9.97 mm"
B. 7.99 mm F = Ssu(ndt) C. kinemat ics •
C. 3.93 mm .!:'!!_ = 02 =4 F = 42,00 O(lT)( 1)(3/4) D. acceleration
N2 Dl F = 31 ,5001T lbs
D. 3.97 mm
28. The shaft that is used to connect or
02 =4
8 24. A 2 in diameter by 4 Inches long disconnect the drive at will is called:
t = 1.5708 journal bearing is to carry a load of A. clutch*
pd D2 = 32 em.
10,000 lb at 1750 rpm using, $AE 40 B. brake shaft
lube oil. Compute the bearing pressure. C . bearing
_
-
1.5708 = 0 _392 7 in = 9.97 mm
4
-o, -_ 322- 8 = 12 em
c = .D2-2- A. 1950 psi D. lock shaft
B. 1250 psi •
C 1750psi 29. The distance from a point on a
18. Determine the bursting steam 21. What modulus of elasticity in screw thread to a :.;orresponding point
D. 1000 psi
pressure of a steel shell with diameter of tension is, required to obtain a unit ori the next measured along the axis:
12 in and made of 1/4 in steel plate. The deformation of 0.00107 m/m from a load A . radial
joint efficiency is at 75% and the tensile
REFRESHER MANUAL 2nd Edition by JAS TORDILLO REFRESHER MANUAL 2nd edition by JAS TORDILLO
55- 10 1 Day 55- Solution Day 55 - Solution 1 55 - 11
B. thickness
Machine Design
36. Which of the following Is recta kind
C. module of mandrel? ·
C. poisson's ratio 47. A gear set having a gear ratio of 3 is
D. pitch* A expanded D. modulus of rigidity* to be used at a center distance of
B. contraction * inches. If the gear has 80 teeth, what
30. Martensite (Stainless steel) contains C. extended
41 . To prevent leakage in dynamic seal: must be the circular pitch?
4 to 26% Cr and a maximum of % D. taper A. gaskets A. 23.6 mm
nickel.
B. packings * B. 29.3 mm
A. 2.5 37. A car is travelling 120 kph, it takes
C. seals C. 23.9 mm *
B. 2.5 * 0.5 sec to apply the brake after which it
D. felts D. 32.9 mm
c. 3.5 decelerates at 8 m/s2. Compute the
D. 4.5 distance it traveled in meters before it
42. The frictional force depends on
stops. .:!1_ = D2 = 3
coefficient of friction and:
31 . Progression change of position of a A . 15.6 T2 .. D1
body is called: A. lorque '
B. 64.81
B. normal force * D2 = 3D, .....
A acceleration c. 26.5 C. weight of the object o, + 02
B. force D. 69.4 * C = , --
C. motion* D. moment 2
D. momentum v, =120 kph = 33.33 m/s 0 43. Not part of the headstock: 16
D1 +301
= --
v2 = o A back gear 2
32. Recommended hardn~ss of pinion Vl=V,2 +2aS
2 B. anvil* D, = 8 in
for helical/herringbone gear tooth should 0 = (33.33) + 2(-8)S.
C. spindle D2 = 3(8) = 24 in
be _ _BHN point to sustain life. S = 69.4 m
D. motor T2 = 80
A. 50-59
B. 40-50 * 38. All four compression coil spring
44. Two or more forces acting together
c. 48-65
D. 30-60
support one load of 700 kg. All four
springs are arranged in parallel and
could be replaced by a single force with Circu lar pitch, Pc = T
7t02

same effect in a mass.


rated same at 0.609 kg/mm. Compute
33. Fast process- of analyzing all the deflection in mm.
A. couple of forces = 7t(24) = 0.94247 in =23.9 rnm
B. resultant* 80
elements and chemical components of A64
steel casting. C. resolution of for<;es
B. 178
A pyrometer D. concurrent of forces 48. A ,body weighing 1000 lb falls 6
C. 147
B. Wet analyzer D. 287 * inches and strikes a 2000 lb per in
45. To prevent engine vibration to affect spring. The deformation of the spring is:
C. carbon analyzer
D. ~pectrometer * the surrounding area, it is standard A. 1 in
For parallel springs:
practice to isolate the engine and its B. 3 in *
y, =Y2=Y3=Y4
foundation by at least mm. C. 2 in
34. In small quantity this alloy is 700
effective for improving strength at high F, = - = 175kg A. 30 D. 6 in
temperature.
4 B. 25 * F
A. chromium * F1 175 c. 20 k =-
y
Y, = -=--=287mm D.36
B. selenium k 0.609
C. manganese F
D. molybdenum 46. Grout material use under the 2000 = y
39. A section in a machine shop machine bed and foundation surface F=
operation. 2000
Y
35. The ratio of the arc of action to
circular pitch or length of action to the
A. coremaking
B. fitting
stress be:
A. 1 part cement and 'h parts screened
sand and 1/4 part gypsum
W(h+ y) = F (r)2
base pitch. C. machining* B. 1 part cement and 1 part screened
A. approach ratio
B. arc of action
C. contact ratio *
D. pattern
sand *
C. 1 part cement and 1 part screened
1000(6 +y) = 2000
y
(r)
2
40. Shear modulus is also: sand and Y. part gypsum . f _y _ 6 = o
D. arc of approach A shear elasticity D. 1 part ~ement and 1 'h pifrt screened (y _ 3 )(y + 2) = 0
B. modulus of elasticity sand and Y. part stone y = 3 in
'.::::.... 4~ •

REFRESHER MANUAL 2nd Edition by JAS TORDILLO REFRESHER MANUAL 2nd Edifion by JAS TORDILLO
55- 12 I Day 55- Solut ion
Day 56 - EXAM I 56 - 1
49. For a single universal joint, shaft Machine Design
angle should be _ _ _ degrees
maximum and much less if on high 6. In problem No. 3, determine the
rotational speed.
DAY 56 - EXAM stress at exerted by the spring at solid •
A. 12 height.
1. A solid transmission shaft is 1OOmm A. 60,000 psi
8.15 *
C. 16 diameter and it is desired to replace it B. 134,436 psi
D. 14 with a hollow shaft of the same material C. 74,500 psi
and same torsional strength but its D . 107,200 psi
weight should only be 40% of the solid
50. The recommended lubricant for the
shaft. Find the outside diameter of the 7. How long a wire is needed to make a
chain drive operation.
hollow shaft. · helical spring having a mean coil
A. SAE 4140
B. moly slip A. 172 mm diameter of 0.820 inch if there are five
C. Petroleum oil * B. 107 mm coils in 'the spring?
D. heavy grease
C. 185 mm A . 4.5 in
D. 139.3 mm B. 7.6 .in
C. 6.E.)n
2. In problem No. 1, find the inside D. 12.8 in
diameter of the hollow shaft.
A . 124.1 mm 8. In problem No. 7, what will this spring
B. 170 mm weigh if it is made of spring steel 0.055
C. 135.1 mm inch in diameter? Use density of steel
D. 140 mm 3
as 0.282 lb/in .
A. 0.00863 lb
3. A helical compression spring has a B. 0.0886 lb
scale of 650 lbs/inch, an inside diameter c. 0.0863 1b
of 3.0 inches, a free length of 18 inches D. 0.8861b
and with squared and ground ends.
Material is to be chrome vanadium steel 9. In problem No. 7, what will be the
with a working stress of 85,992 psi and weight of the spring if it is made of
modulus of rigidity of 11 ,000,000 psi. stainles~ steel? Use density of steel as
For a load P, of 2900 pounds, determine 0.286 lb/in3.
the wire diameter. A . 0.00875 lb
A. Y. inch B. 0.0887 Jb
B. 5/8 inch C. 0.08751b
C. 'h inch D. 0 .8871b
D. "!.inch
10. Two shafts are connected by spur
4. In problem No. 3, determine the gears. The pitch radii of the gears A and
number of active coils. B are 203.2 mm and 812.8 mm
A. 12.7 coils respectively. .I f shaft A makes 450
B. 10.5 coils revolutions per minute and is subjected
C. 8.9 coils to twisting ,,,oment of 300 N.m, what is
D. 19.2 coils the RPM of gear B.
A. 166.1 rpm
5. In problem No. 3, determine the solid B. 112.5 rpm
height. C. 186.4 rpm
A. 17 in D. 100 rpm
B. 8 in
C. 11 in 11. In problem No. 10, determine the
D. 10 in torqt:e in shaft B.
A. 1005 N-m
REFRESHER MANUAL 2nd Edition by -J.AS TORDILLO
REFRESHER MANUAL 2nd Edition by JAS TORDILLO

~
56 - 2 I Day 56 - EXAM Day 56 - EXAM I 56 - 3
Machine Design
B. 990 N-m 18. In problem No. 13, determine the
C. 1200 N-m separating force. 23. In problem No. 22, Determine the 30. One of the methods of determining
D. 860 N-m A. 646.41bs the buckling load for an intermediate
circular pitch.
B. 550.41bs A. 0.523 in column is
12. In problem No. 10, determine the C. 602.6 1bs B. 0.235 in A. secant formula
tooth pressure of the two gears if the D. 350.2 1bs C. 0.325 in B. sine rule
pressure angle is 20 deg. D 0.75 in C. right hand rule
A. 1305 N 19. An automobile engine creates a D. caliper rule
B. 1571 N torque of 500 foot-pounds at a rotational 24. In problem No. 22, determifole the
C. 1450 N speed of 3,000 rpm. In fourth gear, the lead angle. 31 . Assumed to be perfectly elastic
D. 1780 N transmission has a one-to-one ratio. A. 8.23 deg within its working range.
The differential has a ratio of 4 to 1, B. 6.45 deg A 'springs
13. A double threaded right handed what is the force being exerted on the C. 5.11 deg B. beam
worm gear transmits 20 Hp at 1200 rpm. road by the automobile tire if the C. catenary
D. 4.77 deg
The pitch -of the worm is 0.625 inches diameter of the tire is 32 inches? D. bearing
and pitch diameter of 3.25 inches. The A. 500 lbs 25. In problem No. 22, Determine the
pressure angle is 14 Y. 0 and the B. 15001bs centerline distance. 32. The difference · between the inner
coefficient of friction is 0.15. Find the C. 1000 lbs and outer dimensions.
A. 2 in
helix angle. D 2000 lbs B. 10 in A. circular clearance
A. 4.98° B. diametral interference
C. 8 in
B. 6.98° 20. If the automobile io Problem No. 19 C. interference
D. 6 in
C. 5.98° was going up a long, steep hill at this ·D. metric allowance
D. 9.68° speed, how much power was done in 26. In problem No. 22, Determine the
the period of a second? output speed. 33. The coefficient of friction for press
14. In problem No. 13, determine the A. 200 hp A. 29 rpm fits is highly variable, having been
normal pressure angle. B. 285.6 hp B. 60 rpm reported in the range of
A. 14.39° c 230 hp C. 1250 rpm A. 0.03 to 0.33
B. 16.98° D. 340.5 hp D. 1750 rpm B. 1-3.5
C. 15.98° G. 15 to 60
D. 19.68° 21. Determine a standard belt length of 27. For ductile materials, the material D. 0.3 ,to 1
the input and output shafts need to be strength used is
15. In problem No. 13, determine the about 40 inches apart. The input 34. The resistance the object offers to
A. yield strength
efficiency of worm and gear. sheave diameter is 10 in and output B. ultimate strength attempts to accelerate it in a linear
A. 23.5% sheave diameter of 24 in. direction.
C. flexural strength
B. 35.5% A. 134 mm D. fracture strength A. friction force
c. 30.1% B. 3419 mm B. reaction force
D. 43.3% C. 250 mm 28. For steel, the factor of safety ranges C. Inertia force
D. 2479 mm from D. normal force
16. In problem No. 13, determine the A. 3 - 5
tangential force on gear. 22. A worm-gear set has a single-start 35. Mass is a scalar quantity, the
B. 1.5 to 2.5
A. 2287.4 lbs worm. The worm has a pitch diameter' C. 5-10 acceleration is a
B. 2455.4 lbs of 2 inches. The gear has 60 teeth with
D. 20-30 A. vector
C. 2366.4 lbs a diametral pitch of 6. The input shaft is B. scalar
D. 2988.4 lbs driven at 1,750 rpm from a 5-hp motor. 29. Typical critical slenderness ratios C. can be scalar or vector
The pressure angle is 20°. Find the range from D. direction
17. In problem No. 13, determine the pitch diameter of the gear. A. 8 to 10
tangential force on worm. A. 6 in 36. Linear acceleration means that
B. 80 to 120
A. 17981bs B. 10 in C. 20 to 80 A. the acceleration increases uniformly
B. 646.41bs C. 8in with time
D. 200
C. 1045 lbs D. 12 in B. the acceleration decreases uniformly
D. 255.41bs with time

REFRESHER MANUAL 2nd Edition by JAS TORDILLO REFRESHER MANUAL 2nd Edlfion by JAS TORDILLQ
56 - 4 I Day 56 - EXAM Day 56 - Solution 1 56 - 5
Machine Design
C. the acceleration varies uniformly with C. carbon steels
time C. Boyle's load
D. the acceleration varies inversely with
D. manganese steels
D. Euler load DAY 56 - SOLUTION
time 44. An increase in wear resistance
50. The quantity Uk is known as 1. A solid transmission shaft is 1OOmm
A. increases toughness
A. stiffness ratio diameter and it is desired to replace it
37. In projectile, the impact velOcity is B. reduces toughness
B. roughness ratio with a hollow shaft of the same material
A. more than initial velocity C. increases strength
c. slenderness ratio and same torsional strength but its
B. equal to initial velocity D. reduces strength
D. Poisson's ratio· weight should only be 40% of the soiid
C. less than initial velocity
D. higher or equal initial velocity '
45. These are heat-treatable stainless
shaft. Find the outside diameter of the
hollow shaft.
steels and also part of the AISI 400
38. A real force on the body toward the A.-172 mm
center of rotation.
series, that contains no nickel.
A. martensitic
(d ) B. 1Q7 mm
C. 185 mm
A. centrifugal force B. molybdenum DAY 56 - ANSWER KEY
B. concentric force 1. D 16. A 31. A46. B D. 139.3 mm *
C. high-grade stainless
2.A 17.8 32.847.A
C. acceleration force
D. centripetal force

39. The rise in the outer edge of the


D. none of the above

46. A .Joss of zinc in the presence of


3. D
4. A
5. C
18. C
19. 8
20. B
6. B 21 . B
33. A 48. A
34. C49. D
35. A 50. C
36. A
3.5IC) )
roadway
certain corrosive media or at high
temperatures, is a special problem that 7. D 22. 8
8. A 23. A
37. 8
38. D
I< L )l
A. rise in elevation occurs in brasses containing more than Solid
9. A 24. D 39. B Shaft
B. superelevation 15 percent zinc 10.8 25. D 40. B
C. banking angle A. lnzincification 11.C 26.A 41.C

~~·
D. banked B. Dezincification 12.8 27. /!. 42. D
C. Bizincification 13.8 28. B 43. A
14.A 29. 8 44. 8
40. When the body has more supports D. debrassing 15.D 30. A 45. A
than are necessary for equilibrium, the
force system is said to b~ 47. A primary element in achieving high ~ I< >I L
A. statically determinate mechanical strength in aluminum alloys Hollow Shaft
B. statically indeterminate at elevated temperatures.
C . statically balanced A. Copper
16 T
D. statically unbalanced B. Tin For solid shaft: s,= - -
C. Aluminum 1!d3
41 .' The subject that encompasses the D. Phosphorous
procurement and production of metals 16 T do
For hollow shaft:
A. Engineering materials 48. A machine tool in which metal is s.= (4 - d~)
11 do 1
B. Manufacturing removed by means of a revolviog cutter
C. Metallurgy with many teeth, each tooth having a , ,Equating the stresses:
D. Material testing cutting edge which removes its share of 16 T 16 T do
42. Commonly used for drills, cutting
the stock.
A. Milling Machine
n(l 00) 3 = n{d~ - df)
4 4 3
tools and knives . B. Lathe Machine d0 - di = 100 do Eq.1
A. Low-carbon steels C. Grinding Machine
B. Medium-carbon steels D. Bench Machine Let L = length of shaft
C. low-alloy steels P =density
D. High-carbon steels 49. The theoretical maximum load that
an initially straight column can support
For solid shaft: V = ~ d2 L
43. Alloys containing less than 8.0 without buckling. 4
percent total alloying ingredients A. Mcloid load
A. low-alloy steels B. Evan's load For hollow shaft: V = 2: (do2 - d?) L
B. chromium steels 4

REFRESHER MANUAL 2nct Edition by JAS TORDILLO REFRESHER MANUAL 2nd. EatUon by JAS TORDILLO
56-6 1 Day 56- S~lution Day 56 - Solution I 56 - 7
Machine Design
C .= Drnld = 3.7510.75 = 5
~
o.4o( }10o)2Lp = ~(d~- d~ ~P 7. How long a wire is needed to make a to twisting moment of 300 N.m, what is
helicaj spring having a mean coil the RPM of gear B.
d/ - d? = 4000 Eq. 2 K= 4C -I + 0.615 + 4(5)-l + 0.615
diameter of 0.820 inch if there are five , A. 166.1 rpm
4Q'-4 c
4(5)-4 5
coils in the spring? 8. 112.5 rpm*
d?4 = d 2 2- 4000
0 K = 1.31 (satisfied) A. 4.5 in C. 186.4 rpm
d; = (d0 - 4000) 2 B. 7.6 in D. 100 rpm
d0 4 -(d02 -4000) 2 = 1003 do Therefore: d = o/. in. C. 6.6 in
do2 - 125 do - 2000 = 0 D. 12.8 in* D1N1 = D2N2
do= 139.3 mm 4. In problem No. 3, determine the 2(812.8) Ns = 2(203.2)(450)
number of active coils. Length of wire = 1t Dm n Ns = 112.5 rpm
2. In problem No. 1, find the inside A. 12.7 coils* = n(0.820)(~) = 12.88053 in
diameter of the hollow shaft. B. 10.5 coils 11. In problem No. 10, determine the
A. 124.1 mm • C. 8.9 coils 8. In problem No. 7, what will this spring torque in shaft B.
B. 170 mm D. 19.2 coils weigh if it is made of spring steel 0.055 A. 1005 N-m
C. 135.1 mm inch in diameter? Use density of steel B. 99b N-m
D. 140 mm 2900 3 C. 1200 N-m *
y = deflection = = 4.46 in. as 0.282 lb/in .
650 A . 0.00863 lb * D. 860 N-m
d; = J(\39.3/~-4000 = 124.1 mm 8FC 3 n
B. 0.08861b
F, = transmitted load or tangential force
y=-- c. 0.08631b
Gd
3. A helical compression spring has a
D. 0.8861b = torque=~ = 1476.4 N
scale of 650 lbs/inch, an inside diameter .4 = 8(2900) (5)3 n radius 0.2032
4 6
Cross-sectional area = l:d2
of 3. 0 inches, a free length of 18 inches 11,000,000(0.75)
4 ~ = 1476.4
and with squared and ground ends. n = 12.7 coils 2
0.8128
Material is to be chrome vanadium steel = l:(o.oss)2 = 0.002375829 in T s = 1200 N-m torque in shaft B
4
with a working stress of 85,992 psi and 5. In problem No. 3, determine the solid
modulus of rigidity of 11,000,000 psi. height. Volume =12.88053 (0.002375829) 12. In problem No. 10, determine the
3
For a load P, of 2900 pounds, determine A. 17 in = 0.030602 in tooth pressure of the two gears if the
the wire diameter. B. 8.in Weight of spring steel spring p~essure angle is 20 deg.
A. Y. inch C. 11 in* =0.030602(0.282) A. 1305 N
B. 5/8 inch D. 10 in = 0.00863 lb 8 . 1571 N *
C. Y. inch C. 1450 N
D.• % inch • ·Solid height= (n + 2)d 9. In problem No. 7, what wi ll be the D. 1780 N
= (12.7 + 2)(0.75) = 11.025 in weight of the spring if it is made of
Sw =working stress= 85,992 psi stainless steel? Use density of steel as F = tooth pressure = force normal to the
6. In problem No. 3, determine. the 0.286 lb/in3 . tooth surface
G = 11 ,ooo,oqo psi stress at exerted by the spring at solid A. 0.00875 lb * For 14 1/2° pressure angle:
height. B. 0.0887 lb 1476 4
C. 0.08751b F= · = 1571 N
8FDm A. 60,000 psi cos20°
Ss = K - -- where Dm = 3.0 + d B. 134,436 psi* D. 0.8871b
7td 3
C. 74,500 psi 13. A double threaded right handed
First assume a value of K; Let K = 1.25 Weight of stainless steel spring
D. 107,200 psi worm gear transmits 20 Hp at 1200 rpm.
85 992 = 1.31(8) ( 2900) ( 3 + d) = 0.030602(0.286)
= 0.008752 lb The pitch. of the worm is 0.625 inches
' nd 3 y = 18-11 .025 = 6.975 in and pitch diameter of 3.25 inches. The
0
10. Two shafts are connected by spur pressure angle is 14 Y. and the
By trial and error, d =o/. in. , F = 650(6.975) = 4533.751bs coefficient of friction is 0.15. Find the
gears. The pitch radii of the gears A and
s.= 1.31(8) (4533.75) (3.75) helix angle.
B are 203.2 mm and 812.8 mm
Then solve for K: n(0.75)3 respectively . If shaft A makes 450 A. 4.98°
Dm =3 + 0.75 = 3.75 in. revolutions per minute and is subjected 8. 6.98°*
= 134,436.3 psi
...
REFRESHER MANUjJ.L 2nd Edition by JAS TORDILLO REFRESHER MANUAL 2nd Edition by JAS TORDILLO
56 - 8 1 Day 56 - Solution Day 56 - Solution I 56 - 9
Machine Design
C. 5.98° v9 = pitch line velocity of gear
D. 9.68°
= 1.25
12
('zoo) = 2.083 ftlsec
60
C. 1000 lbs
D 2000 lbs
C. 250 mm
D. 2479 mm

Power transmitted to gear (output) T= Fd L = 2C + 1.57(D1 + D2) + (o2 - o,)2


= 20(0.433147) F =.!. 4C
= 8.663 hp d
24
8 66 550 L =2(40) + 1.57(10 + 24) + ( - !0)2
Ft = tangential force on gear= · 3( ) F = 500(4) ft - lb . 4(40)
2.083 32in. (tt)
= 2287.41bs L = 134.6 in= 3418.9 mm
2 (12in.)
17. In problem No. 13, determine the F = 1500 lb 22. A worm-gear set has a single-start
L = lead = 2p = 2(0.625) = 1.25" worm. ' The worm has a pitch diameter
tangential force on worm.
20. If the automobile in Problem No. 19
tan x = Lead = ~ = 0.1224268 A 17981bs
was going up a long, steep hill at this
of 2 in.ches. The gear has 60 'teeth with
nD w n(J .25) B. 646.4 lbs * a diametral pitch of 6. The input shaft is
C. 10451bs speed, how much power was done in driven at 1'750 rpm from a 5-hp motor.
X= 6.9798°
D. 255.41bs the period of a second? The pressure angle · is 20°. Find the
A. 200 hp pitch diameter of the gear.
14. In problem No. 13, determine the B. 285.6 hp *
Vw = pitch line velocity of worm A. 6in
normal pressure angle.
c
A. 14.39° *
B. 16.98°
= x( 3 25
·
12
x 1200
60
) = 1.7.017 ftlsec
230 hp
D. 340.5 hp
B. 10 in*
C. 8 in
D. 12 in
C. 15.98° 20 550 First. as work is force times distance, we
W = tangential force on worm = < )
D. 19.68° t 17.017
need to calculate the distance traveled Pitch diameter:
= 646.41bs in one second: ·
tan 0n = tan 0 cos x Ng
~ ( ) 32in. ft min Op =--
=tan 14.5° cos 6.9798° 18. In problem No. 13, determine the .J,000rpm x Pd
0n = 14.39° separating force. d= · rev 12in 60sec 60
4 Dp = = 10 in.
A. 646.41bs 6
15. In problem No. 13, determine the B. 550.4 lbs
efficiency of worm and gear. C. 602.6 lbs * d = 104.72 ft 23. In 'problem No. 22, Determine the
A. 25.5% D. 350.21bs circular pitch.
B. 35.5% W=Fxd A. 0.523 in •
c. eo.1% S = separating force W = 1500 lb X 104.72 ft B. 0.235 in
D. 43.3% *

E =tan x
cos ~n -
[ cos ~n tan
f tan x
x + f
l =
Wt sin c!ln
cos c!ln sinx + fcos x
646.4 . sin 14.39
W = 157,079.6 ft-lb of work

The power to do this would then be


p= Fd =W
C. 0.325 in
D. 0.75 in

Circular Pitch:
- --~--~----~--------- t t
6.9798 + 0.1 5 cos 6.9798
= 0.122426 [ cosl4.39-0.15(0. 122426) J cos 14.39 ~in
p = 157,079.63 fl - lb Pc =
" Dp
NQ
(cos 14.39) (0.122426)+0. 15 = 602.57 lbs
I sec
= 0.433147 = 43.3% _ "10 in.
19. An automobile engine creates a = 157,079.63 ft - lb = 285.6 hp P c - --
60
torque of 500 foot-pounds at a rotational sec
16. In problem No. 13, determine the Pc = 0.524 in.
tangential force on gear. speed of 3,000 rpm. In fourth gear, the
A. 2287.4 lbs * transmission has a one-to-one ratio. 21 . Determine a standard belt length of
the input and output shafts need to be 24. In problem No. 22, determine the
B. 2455.4 lbs The differential has a ratio of 4 to 1,
about 40 inches apart. The input lead angle.
C. 2366.4 lbs what is the force being exerted Of1 the
sheave diameter is 10 in and output A. 8.23 deg
D. 2988.4 lbs road by the automobile tire if the
sheave diameter of 24 in. B. 6.45 deg
diameter of the tire is 32 inches?
A. 134 mm C. 5.11 deg
A 500 lbs
B. 3419 mm • D. 4.77 deg *
B.1500 lbs *
REFRESHER MANUAL 2nd Edition by JAS TORDILLO REFRESHER MANUAL 2nd Edition by JAS TORDILLO
56- 10 1 Day 56- Solution Day 56 - Solution I 56 - 11
Machine Design
B. ultimate strength
As this is a single-start worm, C. flexural strength A. friction force 41. The subject that encompasses the
L = Pc D. fracture strength B. reaction force procurement and production of metals
C. Inertia force * A. Engineering materials
Lead angle: 28. For steel, the factor of safety ranges D. normal force B. Manufacturing
from C. Metallurgy *
"- =tan·, ("LoJ A. 3-5 35. Mass is a scalar quantity, the D. Material testing
B. 1.5 to 2.5 * acceleration is a 42. Commonly used for drills, cutting
"- = tan·, ( .52~ ) c. 5-10 A. vector • tools and knives.
"2 1n D. 20 - 30 B. scalar A. Low-carbon steels
A = 4.77° C. can be scalar or vector a. Medium-carbon steels
29. Typical critical slenderness ratios D. direction C. low-alloy steels
25. In problem No. 22, Determine the range from D. High-carbon steels *
centerline distance. A. 8 to 10 36. Linear acceleration means that
A. 2 in B. 80 to 120 * A. the acceleration increas.es 43. Alloys containing less than 8.0
B. 10 in C. 20 to 80 uniformly with time * perceht total alloying ingredients
C. Bin D. 200 B. the acceleration decreases uniformly A low-alloy steels"'
D. Sin* with time B. chromium steels
30. One of the methods of determining C. the acceleration varies uniformly with C. carbon steels
Centerline distance: the buckling load for an intermediate time D. manganese steels
C -C = Ow +Dg column is D. the acceleration varies inversely with
2 A. secant fonnula * time 44. An increase in wear resistance
B. sine rule A. increases toughness
C -C = ( 2 ln+ to ln.) C. right hand rule 37. In projectile, the impact velocity is B. reduces toughness *
2
D. caliper r.ule A. more than initial velocity C. increases strength
C -C = 6 in. B. equal to initial velocity " D. reduces strength
31. Assumed to be perfE!ctly elastic C. less than initial velocity
26. In problem No. 22, Determine the within its working range. D. ' higher or equal initial velocity 45. These are heat-treatable stainless
output speed. A springs * steels and also part of the AISI 400
A. 29 rpm* B. beam 38. A real force on the body toward the series, that contains no nickel.
B. 60 rpm C. catenary center of rotation. A. martensitic *
C. 1250 rpm D. bearing A. centrifugal force B. molybdenum
D. 1750 rpm B. concentric force C. high-grade stainless
32. The difference between the inner C. acceleration force D. none of the above
Velocity ratio: and outer dimensions. D. centripetal.force *
A. circular clearance 46. A loss of zinc in the presence of
V, = N, gear
B. diametral interference * 39. The rise in the outer edge of the certain corrosive media or at high
Vstarts worm C. interference roadway temperatures, is a special problem that
60 D. metric allowance A. rise in elevation occurs in brasses containing more than
V, = = 60 to 1
I B. superelevation * 15 percent zinc
33. The coefficient of friction for press C. banking angle A. lnzincification
Output speed: fits is highly variable, having been D. banked B. Dezinclfication *
- nln reported in the range of C. Bizincification
nout - -
Vr A. 0.03 to 0.33 * 40. When the body has more supports D. debrassing
B. 1-3.5 than are necessary for equilibrium , the
nout = ...!.... (1 ,750 rpm) C . 15 to 60 force system is said to be 47. A primary element in achieving high
60
noul = 29.2 rpm D. 0.3 to 1 A. statically determinate mechanical strength in aluminum alloys
.•
B. statically indeterminate'' at elevated temperatures .
27. For ductile materials, the material 34. The resistance the object offers to C. statically balanced A. Copper*
strength used is attempts to accelerate it in a linear D. statically unbalanced B. Tin
A . yield strength * direction.

REFRESHER MANUAL 2nd Edition by JAS TORDillO REFRESHER MANUAL 2nd Editron by JAS TORDILLQ
56 -121 Day 56- Solution Day 57 - EXAM I 57 - 1
Machine Design
C. Aluminum
D. Phosphorous 2820 kg/sq.cm. will be used. Determine
DAY 57- EXAM the size of wire.
48. A machine tool in which metal is A. 1/8 in
1. A square single thread jackscrew has B. Yz in
removed by means of a revolving cutter
two threads per inch. It is to lift 15 tons. C. Y. in
with many teeth, each tooth having a
The friction radius of the collar is 1 inch. D. %in
cutting edge which removes its share of
The coefficient of friction between the
the stock.
threads of the screw and base is 0.15, 6. From Problem No. 5, determine the
A. Milling Machine*
that between the screw and collar is number of coils to be used.
B. Lathe Machine
0. 13. Determine the diameter of the A..._ 6.37
C. Grinding Machine
root of the screw if the allowable stress B. '10.37
D. Bench Machine
in compression is 6,000 psi. C. 8.37
49. The theoretical maximum load that A. 1.5in D. 14.37
B. 2.75 in
an initially straight column can support
G. 2.0 in 7. A 'V-belt is to transmit 50 Hp to a
without buckling.
D. 3.25 in compressor. The sheave is 203.2 mm
A. Mcloid load
B. Evan's load in diameter and turns at 1150 rpm, while
2. From problem No. 1, determine the the larger sheave turns at 400 rpm. The
C. Boyle's load
pull required at the end of a 5 feet bar to service factor may be taken as 1.5 and
D. Euler load *
raise the load. the center distance is equal to the
50. The quantity Uk is known as A. 218.50 lbs diameter of the larger sheave.
B. 518.0 lbs Determine the center distance of the
A. stiffness ratio
C. 581 .21bs installation.
B. roughness ratio
D. 815.2 lbs A. 10 in
C. slenderness ratio*
D. Poisson's ratio B. 16in
3. From Problem No. 1, determine tha C. 13 in
twisting moment exerted at the root of D. 23 in
the threads.
A. 2100.6 in-lb 8.. From problem No. 7, determine the
B. 6210.9 in'-lb
C. 2690.1 in-lb
of
length belt to be used.
A. 23 in
-D. 9210.6 in-lb B. 97.2 in
C. 79.2 in
4. From Problem No.1, determine the D. 120 in
efficiency of the screw and collar.
A. 12.8% 9. A 1200 mm cast iron pulley is
B. 18.21% fastened to a 112.5 mm shaft by means
C. 15.8% of a 28.13 mm square key 175 mm long.
D. 28.11% Th_e key and shaft are SAE 1030 steel
annealed with a shear stress of 29,400
5. A Mitsubishi TDC gas engine valve psi. What force acting at the pulley rim
spring is to have a mean diameter of will shear this key?
3.81 em. The maximum load it will have A. 9538 kg
to sustain is 45.45 kg with B. 3895 kg
corresponding deflection of 1.27 em. c. 8539 kg
The spring is to be made of tempered D. 5938 kg
steel wire with G = 11,600,000 psi.
Since the material is to be subjected to 10. From Problem No. 9, What force
repeated loading, and fatig\Je must be acting at the pulley rim will crush the
considered. a low working stress of cast iron keyway if the strength of the

REFRESHER MANUAL 2nd Edition by JAS TORDILLO REFRESHER MANUAL 2nd Editfon by JAS TORDILLO

~
57 - 2 I Day 57 - EXAM Day 57 • EXAM I 57 • 3
Machine Design
cast iron is 1691.3 kg/em~ in tension and 15. From Problem No. 14, determine the
6765.3 kg/cm2 in compression? shaft diameter. B. 0.249 cross section, in square inches, if the
A. 11,156 kg A. 1.78 in C. 0.0249 resulting elongation must not be greater
B. 16,511 kg B. 2.1 in D. 0.429 than 0.0025 inch?
C. 11,516 kg C. 1.45 in A. 1.8
D. 15,611 kg D. 2.75 in 21 . A 20-tooth, 8 pitch, 1-inch-wide, 20° B. 26.6
pinion transmits 5 hp at 1,725 rpm to a C. 11.46
11. A flanged coupling has an outside 16. From Problem No. 14, determine 60-tooth gear. · Determine the driving D. 31.2
diameter of 200 mm and connects two the shaft pulley diameter. force in lb.
40 mm shafts. There are four 16 mm A. 5.5 in A. 1461b 27. What horsepower is required to lift a
bolts on a 140 mm bolt circle. The B. 15.5 in B. 155 1b IQad of 7,000 lb by means of a cable
radial flange thickness is 20mm. If the C. 10.05 in C. 50 lb wrapped around the drum hoist, the
torsional stress in the shaft is not to D. 18.7 in D. 2551b drum'being 30-in diameter and making
exceed 26 MPa, determine the power 30 rpm?
that can be transmitted at 600 rpm. 17. A shearing machine requires 150 22. From Problem No. 21, find the A. 15 hp
A. 20.53 kw kg-m of energy to shear a steel sheet, separating force. B. 50'hp
B. 35.20 kw and has a normal speed of 3.0 rev/sec, A. 20 lb C. 28 hp
C. 23.50 kw slowing down to 2.8 rev/sec during the B. 123 lb D. 68 hp
D. 53.20 kw shearing process. The flywheel of the C. 531b
machine has a mean diameter of 75 em D. 185 lb 28. A square bar of wrought iron 2-in on
12. From Problem No. 11, determine the and weighs 0.155 kg/cm3. The width of each side is raised to a temperature of
shearing stress in the bolts if uniformly the rim is 30 em. If the hub and arms of 23. From Problem No. 21, determine 130 deg F above its normal. If held so
distributed. Express your answer in the flywheel account for 15% of its total the maximum force. that it cannot expand, what stress will be
MPa. weight, determine the weight of the rim. A. 541b induced in it? k = 6.8 X 10'6 per deg F.
A. 2.3 A. 288.4 kg B. 155 lb A. 12,400 psi
B. 5.8 B. 488.4 kg C. 150 lb B. 15,400 psi
C. 4.5 C. 388.4 kg D. 225lb C. 14,400 psi
D. 8.5 D. 457 kg D. 26,520 psi
24. A diver deflects a diving board 4
13. From Problem No. 11, determine the 18. From Problem No. 17, determine inches. This diving board is made from 2fl. In a pair of gears, is
bearing pressure in the bolt. the thickness of the rim in mm. an aluminum plate 1 inch thick by 12 the plane perpendicular to the axial
A. 3.647 MPa A. 3.55 mm inches wide and 72 inches long. What plane and tangent to the pitch surface.
B. 6.347 MPa B. 15.5 mm is the force at this point? Use E = 10 x A. pitch
C. 4.897 MPa C. 5.55mm 106 psi. B. pitch circle
D. 7.647 MPa D. 23.4 mm A. 250 lb C. pitch plane
B. 3221b D. pitch point
14. A 48 in. diameter diamond saw 19. A steel shaft transmits 40 HP at C. 290 1b
blade is mounted on a p ulley driven 1400 rpm. Considering allowable D. 4251b 30. The phenomenon of continuous
steel shaft, requiring a blade peripheral shearing stress based on pure torsion to stretching under load even if the stress
linear speed of 150 ft/s. Motor drive is be 5000 psi, find the shaft diameter of 25. From Problem No. 24, Determine is less than the yield point:
125 Hp at 1200 rpm, with 6 inches nearest commercial size. the stiffness of the diving board. A. elasticity
diameter pulley. Determine the shaft A. 1 in A. 40 lb/in B. plasticity
rpm to attain blade peripheral speed B. %in B. 80 lb/in C. ductility
required. C. 1 7/16 in C. 60 lb/in D. creep
A. 716.2 rpm D. 1 5/8 in_ D. 110 lb/in
B. 216.7 rpm 31. The machinery operation whereby
C. 617.2 rpm 20. From Problem No. 19, find the 26. A test specimen is under tension. done with the work accurately fastened
D. 167.2 rpm torsional deflection of the shaft in The load is 21,500 lb, allowable stress is has a -reciprocating (forward and
degrees per foot. Use G =
12,000,000 12,000 psi, modulus of elasticity is 30
million psi, and original .' length of
backward) moti9n and the tool head is
stationary.
psi.
A. 0.0429 specimen is 40 ·in. What is the required A. shaping

REFRESHER MANUAL 2nd Edition by JAS TORDILLO REFRESHER MANUAL 2nd eti\Hon by JAS TORDILLO
57 - 4 I Day 57 - EXAM Day 57 - EXAM I 57 - 5
Machine Design
B. turning 38. What happens to a drill if the
C. planing operating speed is too fast? A. shake-out machine C. tip undercut
D. reaming A. drill will crack B. coremaking D. pressure angle cut
B. drill will become overheated and will C. molding machine
32. Finding the resultant of two or more bend D. forging machine 50. An object thrown upward will return
forces is called: C. drill will become overheated and will to earth with the magnitude of the
A. coplanar forces be drawn from the steel 45. What is the property of a material terminal velocity equal to:
B. couple D. drill will become very dull and wear which resists forces acting to pull the A. zero
C. non - coplanar dust material apart? B. one-half of the initial velocity
D. composition A. shear strength C. twice the initial velocity
39. Which of the following is not a B. torsional strength 0.: the initial velocity
33. The capacity of metal to withstand structural steel class? C. tensile strength
load without breaking is: A. low carbon steel D. compressive 'strength
A. strength
B. elasticity
B. medium carbon steel
C. stainless steel 46. The reciprocal of diametral pitch
(q
C. stress D. tool and die steel equivalent to the ratio of pitch diameter .DAY 57- ANSWER KEY
D. strain 1. B 16. C 31.C45. B
to the number of teeth: 2.A 17. C 32. 047.A
40. The section modulus of a rectang le A. lead 3. 0 18. A 33. A48. B
34. Outstanding safety record of base "b" and height "h" about its base B. module 4. 8 19.C 34. 049. B
contributes to: is: C. clearance 5. C 20. B 35. B 50. 0
A. High productivity and quality of work A. bh2/6 D. involute 5.B21.A 36. 0
B. bh'/12 7. 0 22. C 37. c
B. Outstanding performance that 8. 8 23. B 36. C
expects reward from management C. bh3 /3 47. -What is the difference between 9. A 24. B 39. 0
C. Confidence in foremen's ability to D. bh3 /12 brass and bronze? 10.0 25. B 40.A
perform well A. Brass is compostld of copper and 11.A 26. C 41 . A
D. Pride and enthusiasm to one-self 41 . A material that should be avoided in 12.8 27. B 42. A
zinc while bronze is composed of 13.A 28. 0 43. A
constructing wood pattern: copper and tin 14A 29. C 44. 0
35. A type of coupling that allows slight A sapwood B. Brass is composed of copper and 15.8 30. 0 45. c
amount of torsional/angular flexibility B. heart wood zinc whi(e bronze is basiG,ally copper
due to introduction with some elastic C. kiln dried wood and tin plus non-ferrous alloy such as G
material cylindrically wrapped around D. core portion of wood manganese, aluminum and chromium.
the bolts in the flange. C. Bronze is reddish in color while
A. simple elastic bonded coupling 42. The gradual chemical reaction by brass is a mixture of copper and
B. elastic-material bushed coupling other substance such that metal is antimony ,
C. elastic material bonded coupling converted to an oxide or other D. Bronze is mostly an alloy of copper
D. all of these compounds: and tin while brass is a mixture of
A. corrosion copper and antimony.
36. In a uniformly loaded beam, the B. Rusting
maximum vertical shearing force occurs C. Cheaping 48. The first derivative with respect to
A. at the center D. Weathering velocity of kinetic energy is:
B. at the section of maximum moment A. power
C. at beam bottom fiber 43. The center of gravity of a solid B. momentum
D. at either end support pyramid or cone with a total height of H C. energy
is: . D. none of these
37. Copper and most of its alloys can be A. y. H
hardened by: B. 1/3 H 49. The arbitrary modification by
A. patenting C. 3/5 H removing a small portion of a tooth
B. case hardening D. Y. H material at the tip of the gear tooth is
C. cold working called:
D. soaking 44. Which do not belong to foundry or A. tip removal
metal casting shop? B. tip relief

REFRESHER MANUAL 2nd Edition by JAS TORDILLO REFRESHER MANUAL 2nd E'Ci,'tion by JAS TORDILLO
57 - 6 1 Day 57 - Solution Day 57 - Solution I 57 - 7
Machine Design
Tc= torque required to overcome friction
DAY 57 - SOLUTION in collar S, = K 8 F Dm = 8(45.45X6)3n .
= fc W rm = 0.13 (30,000)(1) d3
1 27
1. A square single thread jackscrew has = 3-,900 lb-in
lt . 815,421(0.635)
two threads per inch. It is to lift 15 tons. K= 4C-1+0.615 C = Dm n = 8.37 active coils
The friction radius of the collar is 1 inch. T= total torque to operate jackscrew 4C-4 C d
The coefficient of friction between the = 9,210.6 + 3,900 = 13,110.61b-in Actual number of coils
threads of the screw and base is 0. 15, where: F = 45.45 kg = 8.37 + 2 = 10.37
that between the screw and collar is Torque= Force x Distance Dm = 3.8-1 em
0.13. Determine the diameter of the 13,110.6 = F (5 X 12) s. = 2820 kg/cm2 · 7. A V-belt is to transmit 50 Hp to a
root of the screw if the allowable stress F = 218.511bs compressor. The sheave is 203.2 mm
in compression is 6,000 psi. First, try K = 1.2; this w ill be checked in diameter and turns at 1150 rpm, while
A. 1.5 in 3. From Problem No. 1, determine the after solving for a the larger sheave turns at 400 rpm. The
B. 2.75 in* twisting moment exerted at the root of .service factor may be taken as 1.5 and
C. 2.0 in the center distance is . equal to the
the threads. 2820 = u(sX4sAsX3.sl)
D. 3.25 in A. 2100.6 in-lb 3
lt d diameter of the larger sheave.
B. 6210.9 in-lb d = 0.5725 em Determine the center distance of the
Stress = Force C. 2690.1 in-lb installation.
Area D. 9210.6.in-lb * C= ~ =6.655 A. 10in
0.5725
6000 = 15(2000) B. 16in
K = 4(6.655)- 1 + 0.615 = 1_225
"ol From Problem No. 2, twisting moment or 4(6.655)- 4 6.655
C. 13 in
4 r D. 23 in"
torque at the root of the threads
0 = 2.523 in. say 2.750 in. T. = 9,210.6 lb-in.
For second trial, let K = 1.227 N 1 = 1150rpm
2. From problem No. 1, determine the = Lm(sX~5.45X3.81) D1 = 203.2 mm
4. From Problem No.1, determine the 2820
pull required at the end of a 5 feet bar to efficiency of the screw and collar. " dJ 50 Hp
raise the load. A. 12.8% d = 0.5768 em
A. 218.50 lbs * e. 18.21% * c= ~ =6.605 o, =203.2 mm = 3 in.
B. 518.0 lbs C. 15.8% 0.568 =
Q1N1 D2N2
C. 581.21bs D. 28.11% K= 4(6 ·605 )-l + 0 ·615 = 1.227 (satisfied) 203.2(1150) = D2(400)
D. 815.2 lbs 4(6.605)- 4 6.605 D2 = 584.2 mm = 23 in.
e = Work output =_ __::.30:.::,o:.:o.::.:
o<c:,o·:.:.:5l
Work input 218.51 x 2n(5 x 12)
p = pitch = 'h = 0.50 in. d = 0.5768 em = 0.227 in. C = D2 = 23 in.
= 18.21% Therefore, d = Y. in. = 0.635 em ....
Do= major diameter= 2.75 + 0.50 8. From problem No. 7, determine the
= 3.25 in. 5. A Mitsubishi TDC gas engine valve 6. From Problem No. 5, determine the length of belt to be used.
Dm =mean diameter= (3.25 + 2.75)/2 spring is to have a mean diameter of number of coils to be used. A. 23 in
=3 in. 3.81 em. The maximum load it will have B. 97.2 in *
A. 6.37
to sustain is 45.45 kg with B. 10.37 * C. 79.2 in
x = lead angle: corresponding deflection of 1.27 em. C. 8.37 D. 120in
The spring is to be made of temperet1 2
tan x = Lead = 0·50 = 0.0530516 steel wire with G = 11,600,000 psi.
D. 14.37
L = 2C + 1.57(D2 + D,) + (D2 - oJ)
TC Dm 7!(3)
Since the material is to be subjected to 4C
T,= torque required to overcome friction G =11,600,000 psi.
in threads
W Dm(tanX+f)
repeated loading, and fatigue must be
considered, a !ow working stress of y=
8 F C3n 23 8
= 2(23) + 1.57(23 + 8) + ( (- ?
4 23
2820 kg/sq.cm. will be used. Determine G d
= 2(1-f tanx) the size of wire.
= 97.115 in
= 30.000(3 0.0530516 + 0. 15 A. 1/8 in C=~ =6
0.635 9. A 1200 mm cast iron pulley is
21-0. 15 X 0.05305 16 B. 'h in
11,600,000 = 815,421· 1sg/cm2 fastened to a 1:12.5 mm shaft by means
C. Y. in* G-
= 9,210.61b-in - 2.205(2.54)2 of a 28.13 mm square key 175 mm long.
D. '!.in
The key and shaft are SAE 1030 steel

REFRESHER MANUAL 2nd Edition by JAS TORDILLO REFRESHER MANUAL 2nd e~lfion by JAS TORDILLO
57 - 8 1 Day 57 - Solution Day 57 -Solution 1 57 - 9
Machine Design
annealed with a shear stress of 29,400 torsional stress in the shaft is not to
psi. What force acting at the pulley rim exceed 26 MPa, determine the power 14. A 48 in. diameter diamond saw the rim is 30 em. If the hub and arms of
will shear this key? that can be transmitted at 600 rpm. blade is mounted on a pulley driven the flywheel account for 15% of its total
A. 9538 kg* A. 20.53 kw • steel shaft. requiring a blade peripheral weight, determine the weight of the rim .
B. 3895 kg B. 35.20 kw linear speed of 150 ft/s. Motor drive is A 288.4 kg
c. 8539 kg C. 23.50kw 125 Hp at 1200 rpm , with 6 inches B. 488.4 kg
D. 5938 kg D. 53.20 kw diameter pulley. Determine the shaft c. 388.4 kg ..
rpm to attain blade peripheral speed D. 457 kg
. s. = 29,400 2 = 20.6667 kg/mm 2 Ss = 16T required.
2.205(25.4) JtD3 A. 716.2 rpm * E = -w (V,2 - v22)

s =F'-
B. 216.7 rpm 2g
26000 = ~ C. 617.2 rpm where: E = 150 kg-m
• wL ' 1t(0.040)3 D. 167.2 rpm g = 9.81 m/sec2
F' T = 0.3267256 KN-m V 1 = 1tDN, = n:(0.75)(3)
20.6667 = 28 . 13(175) V = n:DN = 7.0686 m/sec
P = 2n:TN
F' = 101,737(kg)
600 150(60) = n:( ~~) N. Vz = 1tDN2 = 1t(0.75)(2.8)
= 6.5973 m/sec
= 2n(0.3267256)( )
Fc2;0) =F'c~~.s) ' = 20.53 KW
' 60 Ns = 716.2 rpm W = weight of flywheel

15. From -Problem No. 14, determine the Substituting:


12 0
F( ; ) = 101,737( 11 ~· 5 ) 12. From Problem No. 11, determine the shaft diameter. 150 = ___!!__... f(7_o6&6)2 - (6.5973)2 ]
F = 9,538 kg (to shear key) shearing stress in the bolts if uniformly A. 1.78 in 2(9.81) ~
distributed. Express your answer in B. 2:1 in* w = 456. 935' kg
MPa. C . 1.45in
10. From Problem No. 9, What force
acting at the pulley rim will crush the A. 2.3 D. 2.75 in w, + 0. 15(456.935) = 456.935
cast iron keyway if the strength of the B. 5.8 *
C. 4.5
w, = 388.395 kg
cast iron is 1691.3 kg/cm 2 in tension and Using PSME Code, for lineshafts
6765.3 kg/cm 2 in compre~sion? D. 8.5 carrying pulleys: 18. From Problem No. 17, determine
~--
A 11,156 kg D = ~ 53.5P = 53.5(125) the thickness of the rim in mm.
B. 16,511 kg Force on bolts = Torque = 0.3267256 N 716.2 A. 3.55 mm"
c. 11,516 kg Radius (0.140/2) B. 15.5 mm
2.1 .inches
D. 15,611 kg* = 4.6675 kN = 4,667.5 N C. 5.55 mm
4667.5 D. 23.4 mm
Force per bolt= -·-- = 1,167 N 16. From Problem No. 14, determine
F' 4 the shaft pulley diameter.
Sc= ...,.,.....,...:,..--
28. 13 (175) Weight = Volume x Density = JtDmbtw
Shearing stress in bolts = ~ A. 5.5 in
388.395 =rr(75)(30)(t)(0.155)
2 . 1t 2 B. 15.5 in
6765.3 F' .. (16)
4 C. 10.05 in* t = 0.3545 em = 3.55 mm
(10)3 = 2~:13 (175) = 5.804 N/mm2 D. 18.7 in
19. A steel shaft transmits 40 HP at
2
1400 rpm. Considering allowable
F' = 166,519.4 kg 13. From Problem No. 11 , determine the DmNm = DsNs
6(1200) =D.(716.2) shearing stress based on pure torsion to
bearing pressure in the bolt.
F(' ~ }2 0
= 166,519.4(' l~.5) A. 3.647 MPa * D.= 10.053 inches be 5000 psi, find the shaft diameter of
nearest commercial size.
B. 6.347 MPa
F = 15,611 kg (to crush the cast iron 17. A shearing machine requires 150 A 1 in
C. 4.897 MPa
keyway) B. Y, in
D. 7.647 MP<! kg-m of energy to shear a steel sheet,
and has a normal speed of 3.0 rev/sec, C. 1 7/1!) in*
11. A flanged coupling has an outside D. 1 5/8 in
Bearing pressure in bolts slowing down to 2.8 rev/sec during the
diameter of 200 mm and connects two shearing process. The flywheel of the
40 mm shafts. There are four 16 mm = ~ = 3.647 N/mm2 P = 2n:TN
16(20) machine has a mean diameter of 75 em
bolts on a 140 mm bolt circle. The and weighs 0.1 55 kg/cm 3• The width of 40(33,000) = 2nT (1400)
radial flange thickness is 20mm. If the
REFRESHER MANUAL 2nd Edition by JAS TORDILLO REFRESHER MANUAL 2nd Edition by JAS TORDILLO
Day 57 - Solution I 57 - 11
57 - 10 1 Day 57 -Solution Machine Design
T = 150 ft-lbs = 1800 in-lbs D = 20teeth· =2 5 1
S, = 16T P 8teethlin.diameter · n. 8 = -Fl:
3 y =~
3 AE
~tD 3EI
0.0025 = 21,500(40)
5000 = 16(1 800) Ft= .3.!_ F = 3/iEI 6
!t03 Dp L3 A(3ox1o )
D =1.224 in.
0~
2
4 A = 11.46 in
Ft = (2)183 in.-lb = 1461b F = 3 • 4 in{ lOx I lb/ in(l in. )
D= ,2. in. (1 .4375 in) Therefore, the safest cross section area
16 2.5 in. will be 11.46 in2 •

20. From Problem No. 19, find the 22. From Problem No. 21, find the
(n inf 'L-7 ~What horsepower is required to lift a
torsional deflection of the shaft in separating force. = 322lb load ,?f 7,000 lb by means of a cable
degrees per foot. Use G = 12,000,000 A. 20 lb wrapped around the drum hoist, the
psi. B. 123 lb 25. From Problem No. 24, Determine drum being 30-ln diameter and making
A. 0.0429 c. 531b * the stiffness of the diving board. 30 rpQ1?
B. 0.249 * D. 185 lb A. 40 lb/in A . 15 hp
C. 0.0249 B. 80 lb/in * B. 50 hp *
D. 0.429 Fn = Ft tan 9 C. 60 lb/in C. 28 hp
=
Fn 1461b tan 20° D. 110 lb/in D. 68 hp
e= TL Fn = 531b
JG F = kx or F =ko .Power =force x velocity
23. From Problem No. 21, determine
k = ~ = 3EI = 7QQQ X 1t
30 30 ft -lb X~
where: T = 1800 in-lbs the maximum force. o LJ 12 min 33,000
= =
l 1 ft 12 in. A. 541b = 50 hp
B. 155lb * = 10x106 2 4
~t(l.4375 )
4 k lb/in. 1 in.
J= = 0.4192 in4 C. 1501b
(n inf
32
28. A square bar of wrought 1ron 2-in on
D. 2251b each side is raised to a temperature of
G = 12 x 106 psi for steel
130 deg F above its normal. If held so
e= 180 12
0: ) = 0.00429 rad/ft k =80 lb/in. that it cannot expand, what stress will be
0.4192(12,000,000)
F,= __!i_
cos e 26. A test specimen is under tension.
induced in it? k = 6.8 x 10-s per deg F.
= 0.246 deg per foot A. 12,400 psi
The load is 21,500 lb, allowable stress is
F,= 146 lb B. 15,400 psi
21 . A 20-tooth, 8 pitch, 1-inch-wide, 20° 12,000 psi, modulus of elasticity is 30
cos 20° C. 14,400 psi
pini6n transmits 5 hp at 1, 725 rpm to a million psi, and original length of
D. 26,520 psi *
60-tooth gear. Determine the driving F,=1551b specimen is 40 in. What Is the required
force in lb. cross section, in square inches, if the
S = kED.T
A. 1461b * resulting elongation must not be greater
24. A diver deflects · a diving board 4 = (6.8 X 10-6)(30 X 106)(130)
B. 1551b than 0.0025 inch?
inches. This diving board is made from
A. 1.8
=26,520 psi
C. 50 lb an aluminum plate 1 inch thick by 12
D. 2551b inches wide and 72 inches long. What B. 26.6 29. In a pair of gears, is
is the force at this point? Use E = 10 'X c. 11.46 * the plane perpendicular to the axial
TN 106 psi. D. 31.2
plane and tangent to the pitch surface.
p = 63,000 A. 2501b A. pitch
B. 3221b * F
63,00<F s, ='A B. pitch circle
T= - N- c. 290 lb C. pitch plane *
D. 4251b 12,000 = 21,500 D. pitch point
(63,000)5 = 183 in.-lb
T=~
N .
D = _E_
I = bh3 = 12 in{ 1 in.) . 4
A = 1.79 in
2
A
30. The phenomenon of conti nuous
stretching under load even if the stress
pd 12 12 = 1 m.
p is less than the yield point:
A. elasticity
REFRESHER MANUAL 2 nd E"Clifibn by JAS TORDILLQ
REFRESHER MANUAL 2nd Edition by JAS TORDILLO
57- 12 1 Day 57- Solution Day 57 - Solution I 57 - 13
Machine Design
B. plasticity C. at beam bottom fiber ~

C. ductility D. at either end support * 43. The center of gravity of a solid B. momentum *
D. creep* pyramid or cone with a total height of H C. energy
37. Copper and most of its alloys can be is: D. none of these
31 . The machinery operation whereby hardened by: A. Y•H*
done with the work accurately fastened A. patenting B. 1/3 H 49. The arbitrary modification by
has a reciprocating (forward and B. case hardening C. 3/5 H removing a small portion of a tooth
backward) motion and the tool head is C. cold working * D. Y:. H material at the tip of the gear tooth is
stationary. D. soaking called:
A. shaping 44. Which do not belong to foundry or A. tip removal
B. turning 38. What happens to a drill if the metal casting shop? ~· tip relief*
C. planing* operating speed is too fast? A. shake-out machine C. -tip undercut
D. reaming A. drill will crack B. coremaking • D. pressure angle cut
B. drill will become overheated and will C. molding machine
32. Finding the resultant of two or more bend D. forging machine * 50. An object thrown upward wi ll return
forces is called : C. drill will become overheated and to earth with the magnitude of the
A. coplanar forces will be drawn from the steel * 45. What is the property of a material terminal velocity equal to:
B. GOUple D. drill will become very dull and wear which resists forces acting to pull the A. zero
C. non - coplanar dust material apart? B. one-half of the initial velocity
D. composition * A. shear strength C. twice the initial velocity
39. Which of the following is not a B. torsional strength D. the initial velocity*
33. The capacity of metal to withstand structural steel class? C. tensile strength *
load without breaking is: A. low carbon steel D. campressive strength
A. strength * B. medium carbon steel
B. elasticity C. stainless steel 46. The reciprocal of diametral pitch
C. stress D. tool and die steel * equivalent to the ratio of pitch diameter
D. strain to the number of teeth:
40. The section modulus of a rectangle A. lead
34. Outstanding safety record of base "b" and height "h" about its base B. module*
contributes to: is: C. clearance
A. High productivity and quality of work A. bh 2/6 * D. involute
B. Outstanding performance that B. bh2/12
expects reward from management C. bh 3/3 47. What is the difference between
C. • Confidence in foremen's ability to D. bh3/12 brass and bronze?
perform well A. Brass is composed of copper and
D. Pride and enthusiasm to one-self* 41. A material that should be avoided in zinc while bronze is composed of
constructing wood pattern: copper and tin *
35. A type of coupling that allows slight A. sapwood* B. Brass is composed of copper and
amount of torsional/angular flexibility 8. heart wood zinc wh ile bronze is basically copper
due to introduction with some elastic C. kiln dried wood and tin · plus non-ferrous alloy such as
material cylindrically wrapped around D. core portion of wood manganese, aluminum and chromium.
the bolts in the flange. C. Bronze is reddish in color while
A. simple elastic bonded coupling 42. The gradual chemical reaction by brass is a mixture of copper and
B. elastic-material bushed coupling * other substance such that metal is • antimony
C. elastic material bonded coupling converted to an oxide or other D. Bronze is mostly an alloy of copper
D. all of these compounds: and tin while brass is a mixture of
A. corrosion * copper and antimony.
36. In a uniformly loaded beam, the B. Rusting
maximum vertical shearing force occurs C. Cheaping 48. The first derivative with respect to
A. at the center D. Weathering velocity of kinetic energy is: ' I
B. at the section of maximum moment A. power I

REFRESHER MANUAL 2nd Edition by·JAS TORDILLO REFRESHER MANUAL 2nd E~llon by JAS TORDILLO
Day 58 - EXAM I 58 - 1
Machine Design
external pressure of 3,000 psi.
DAY 58- EXAM Determine the hoop stress at the inner
surface ofthe cylinder.
1. A flat belt is to transmit 100 Hp to an A. 4,333.33 psi
air compressor. The small sheave is B. 3,333.33 psi
180 mm in diameter and turns at 1200 C. 5,333.33 psi
rpm, while the larger sheave turns at D. 6,333.33 psi
450 rpm. The service factor may be
taken as 2.0, the center distance is 6. A cylindrical tank is subjected to a
equal to two times ,the diameter of the maximum pressure of 3 MPa. The
larger sheave. Determine the length of inside diameter is 2,000 mm while the
belt. riveted joint has a longitudinal pitch of
A. 1,579.64mm 70 mm and there are two straps placed.
B. 1,979.64 mm The design stress for the plate and the
C. 2,579.64 mm rivets are 170 MPa and 220 MPa,
D. 2,979.64 mm respectively. Determine the thickness of
the plate. Assume 70% ~fficiency of the
2. In Problem No. 1, Find the arc of joints.
contact of small sheave in degrees. A. 15.21 mm
A. 121.25° B. 25.21 mm
B. 141.25° C. 35.21 mm
c. 161_25° D. 39.21 mm
D. 181 .25°
7. A steel cylindrical air receiver with 6.0
3. A 750 mm steel pulley transmits 120 feet diameter and pressure load of 220
hp at 700 RPM. The arc of contact psi, design stress of 10,000 psi
between the belt and pulley is 152 l(laximum. Assume a 90% weld joint
degrees, the coefficient of friction efficiency. . :-.The lap welding tensile
between belt and pulley is 0.30 and the strength is 70,000 psi. Determine the
safe working stress of. the belt is 2.0 bursting steam pressure of this air
MPa. Find the width of the belt used in receiver.
mm if its thickness is 20 mm. A. 1530 psi
A. 148.36 mm B. 1600 psi
B. 158.36 .mm C. 1350 psi
C. 168.36 mm · D. 1000 psi
D. 178.36 mm
8. The Coriolis acceleration depends on
4. An 8 inches diameter pulley turning the latitude,
at 600 rpm is belt connected to a 14 A. Because the earth's tangential
inches diameter pulley. lfthere is a 4% velocity is less near the poles than at
slip, find the speed of the 14 inches the equator.
pulley. B. Because the earth's tangential
A. 329 rpm velocity is greater near the poles than at
B. 239 rpm the equator.
C. 932 rpm C. Because the earth's tangential
D. 500 rpm velocity is not enough near the poles
than at the equator.
5. An air cylinder having an internal D. Because the earth's tangential
diameter of 16 inches and an external velocity is sometimes greater near the
diameter of 32 inches is subjected to an poles than at the equator.
internal pressure of 8,000 psi and an
REFRESHER MANUAL 2nd Edition by ·J.AS TORDILLO
58-21 Day58-EXAM ·Day 58 - EXAM I 58 - 3
9. This is used when motion of a particle
Machine Design
the position of one or more other
is described with respect to something particles, the motions are said to be
20. Part of the subject .known as C. central point
else in motion A. kinetics
engineering mechanics and it is the ... D. point of tangency
A. Relative motion B. independent study of rigid bodies that are stationary.
B. Inertia C. dependent
A . Statics 27. The line in the direction of the force
C. Polar D. plotted
B. Kinetics extended forward and backward.
D. Static
C. Displacement A. moment of a force
15. A block-and-pulley system with one B. line of action of a force
10. The particle's position, velocity, and D. Force
fixed roped end is
C. rotation of a force
acceleration may be specified with A. a dependent system
21. A force on a rigid body caused by D. all of these
respect to another moving particle or B. an independent system
other bodies.
with respect to , a moving frame of C. a popular system
A. tangential force 28. The name given to the tendency of a
reference , known as D. a plane system
B. central force force to rotate, turn, or twist a rigid body
A. an inertial frame of reference
C. internal force about an actual or assumed pivot point.
B. a functional frame of reference 16. The sum of a translational D. external force A. radial
C. a non-inertial frame of reference component and a rotation about a fixed B. couple
D. a pivot point axis
22. A force which holds parts of the rigid C. moment
A. circular motion
body together. D. positive
11 . It is a study of the geometry of B. dependent motion
motion without consideration of the A. tangential force
C. radial motion 29. Another name for moment is
causes of motion B. central force
D. plane motion
C. internal force A. torque
A. Kinematics
D. external force B. latent
B. Kinetics 17. A point at which the body could be C. resultant
C. Physics fixed (pinned) without changing the
23. These are the tensile and D. opposite
D. Translation instantaneous angular velocities are
compressive forces within parts of the
concerned, the body seems to rotate 30. When acted upon by a moment,
body. as found from the product of
12. Kinematics deals only with about a fixed instantaneous center.
stress and area. unrestrained bodies will
relationships among A. The instantaneous center
A. Internal forces fl., remain steady
A. weight, velocity, acceleration, and B. The pivot center
B. External forces B. rotate • :"
time C. The wheel center
C. Inherent forces C. remain static
B. vectors, velocity, acceleration, and D. The support center
D. Axial forces D. maintain its position
time
C. forces, velocity, acceleration, and 18. For a rolling wheel, the 31. When a restrained body is acted
time
24. A, vector of unit length directed along
instantaneous center is upon by a moment ·
a coordinate axis.
D. position, velocity, acceleration, and A. The point of contact with the load A. there is no rotation
time A. unit length
surface B. rotate
B. unit vector
B. The point of contact with the moving C. move
13. Particles do not possess rotational surface
C. unit displacement
D. unit direction D. divert
kinetic energy. All parts of a particle C. The point of contact with the
have supporting surface
25. A push or pull that one body exerts 32. Typical units of moments are
A. The different instantaneous D. The point of contact with the static
on another. A. foot-pounds
displacement, velocity and acceleration. surface
A. Force B. inch-pounds
B. The same instantaneous
C. newton-meters
displacement, velocity and acceleration 19. Used to compute the absolute B. Energy
C. Moment D. All of these
C. Unknown instantaneous acceleration of a point as if a body is in
displacement, velocity and acceleration. pure rotation about that point. D. Work
33. The line of action of the moment
D. All of these A. instantaneous center of acceleration
26. A vector having magnitude, direction vector is
B. instantaneous center of velocity A. Normal to the plane containing the
and location in three-dimensional space.
14. When the position of one particle in C. instantaneol!S center of displacement
A. concentrated force/point force force vector and the position vector
a multiple-particle system depends on D. instantaneous center of rotation B. Tangent to the plane containing the
B. pivot point
force vector and the position vector

REFRESHER MANUAL 2nd Edition by JAS TORDILLO REFRESHER MANUAL 2nd Edition by JAS TORDILLO

jJ
58 - 4 1. Day 58 - EXA!'JI Day 58- EXAM 158 - 5
Machine Design
C. Dependent to the plane containing C. coplanar
the force vector D. station?ry B. steady force system B. unequal and tangent
D. Parallel to the plane containing the C. concurrent force system C. equal but opposite
position vector 39. The combination of the moved force D. stationary force system D. unequal but opposite
and the couple is known as
34. Place the position and force vectors A. a moment system 46. All of the forces share the same line 53. A three-force member can be in
tail to tail. Close your hand and position B. a turning system of action. equilibrium only if the three forces are
it over · the pivot point. Rotate the C. a force-couple-system A. collinear force system A. linear
position vector into the force vector, and D. linear force system B. steady force system B. concurrent
position your hand such that your C. concurrent force system C. opposite
fingers curl in the same direction as the 40. A force system in which all forces D. statiQnary force system D. coplanar
position vector rotates. Your extended are parallel and applied along a straight
thumb will coincide with the direction of line. 47. All of the forces are parallel 54. For beams, the two most common
the moment. A. linear force system A. opposite force system types of supports are the roller support
A. Rule of Thumb B. circular force system B. parallel force system and the
B. Newton's rule C. normal force system C. coplanar force system A. pinned support
C. Right hand rule D. stationary force system D. equal force system B. backed support
D. Magic rule C. block support
41. A straight beam loaded by several 48. All of the forces are in a plane. D. friction
35. A statement of how the total moment concentrated forces is an example of a A. steady force system
is derived from a number of forces A. linear force system B. concurrent force system 55. Generally, the terms simple support
acting simultaneously at a point B'. circular force system C. general force system and simply~supported refers to
A Ptolemy's Theorem C. normal force system D. coplanar force system A. a roller support
B. Varignon's Theorem D. stationary force system B. cantilever
C. Planck's Theorem 49. All other combinations of non- C. single support
D. Moment Theorem 42. Force which is equal to the sum of concurrent, non-parallel, and non- D. elastic support
the individual forces coplanar forces.
36. The sum of individual moments A. equivalent resultant force A. general three-dimensional system tiS. Supports both vertical and horizontal
about a point caused by multiply B. distributed force B. static system forces.
concurrent forces is equal to the C, redundant force C. support system A. roller support
moment of the resultant force about the D. tangential force D. roller system B. friction support
same point. C. block support
A. Plato's Theorem 43. If an object is continuously loaded 50 . .t\n object is static when it is D. pinned support
B. Varignon's Theorem over a portion of its length or load result A. moving
C. Planck's Theorem from dead load and hydrostatic B. rolling 57. When a body is in contact with a
D. Torque Rules pressure. C. pushed frictionless surface, there is
A. Moving load D. stationary A. no frictional force component-parallel
37. Any pair of equal, opposite, and B. Distributed load to the surface
parallel forces constitute a C. Fixed load 51 . To be stationary, all of the forces on B. a frictional force component parallel
A. concurrent D. Parallel load the object to be in equilibrium, the to the surface
B. diagonal resultant force and moment vectors C. more frictional force component
C. couple 44. The equivalent load in a distributed must be parallel to the surface
D. equilibrium load is A. equal D. less frictional force component
A. the area under the loading curve B. equal to one parallel to the surface
38. Two forces can be replaced by a B. the graph under the loading curve C. parallel
moment vector, which can be moved to C. the volume under the loading curve D. zero 58. Frictionless contact is particularly
any location on a body. Such a moment D. the product under the loading curve useful when dealing with systems of
is known as 52. A two-force member can be in spheres and cylinders in contact with
A . a free moment 45. All of the forces act at the same equilibrium only if the two forces have rigid supports. Frictionless contact is
B. collinear point. the same line of action and are also assumed for
A. coplanar force system A. equal and parallel A. roller and rocker supports

REFRESHER MANUAL 2nd EEiition by JAS TORDILLO REFRESHER MANUAL 2nd Edition by JAS TORDILLO

·'
58-61 Day58-EXAM Day 58 - EXAM I 58 - 7
Machine Design
B. block support A. free body diagram
C. welded support B. body graph B. mechanical advantage 77. The loss factor is the
D. wooden support C. line graph C. force diagram A. reciprocal of the pulley efficiency
D. body sketch D. load implication B. equal to the pulley efficiency
59. When the equations of equilibrium C. two times the pulley efficiency
are independent. a rigid body force 65. A graph of the magnitude of a 71. This is used to change the direction D. None of these
system is said to be reaction as a function of the load of an applied tensile force
A. statically detefminate placement. A. bearing 78. An axial member is capable of
B. statically indeterminate A. influence diagram B. belt supporting
C. unbalanced system B. magnitude diagram C. pulley or sheave A. axial and radial forces
D. port system C. load diagram D. clutch B. axial forces only
D. lever diagram C. radial forces
60. When the b'ody has more supports 72. A series of pulleys working together D, triaxial forces
than are necessary for equilibrium, ttie 66. In influence diagram, the x-axis of known as
force system is said to be the graph corresponds to the location on A. train motion 79. Graphs of shear and moment as
A. statically determinate the b-sdy (along the length of a beam) B. rope system functions of position along. the beam are
B. statically indeterminate and the y-axis corresponds to the C. a block and tackle known as
C. unbalanced system A. deflection D. flywheel system A. shear and moment diagrams
D. port system B. position of the reaction B. schematic diagram
C. direction of the reaction 73. If the pulley is attached by a bracket C. load diagram
61. In a statically indeterminate system, D. magnitude of the reaction or cable to a fixed location, it is known D. graphical presentation
one or more of the supports or members as
can be removed or reduced in restraint 67. These are added to structures to A. a free pulley 80. The shear at any point is equal to
without affecting the equilibrium prevent translation while permitting B. solid pulley A. the product of the loads and reactions
position. These· supports and members rotation. C. vertical pulley from the point to the left end
are known as A. rivets D. a fixed pulley B. the sum of the loads and reactions
A. redundant members B. bolt from the point to the left end
B. lever C. lever 74. If the pulley is attached to a load, or ~· the sum of the loads and reactions
C. pulley Q. hinges if the pulley is free to move, it is known from the poin1 to the right end
D. continuous members as D. the product of the loads and
68. A frictionless hinge A. a free pulley reactions from the point to the midpoint
62. The number of redundant members A. can support a force, but cannot B. a solid pulley
is known as the transmit a moment C. a vertical pulley 81. The magnitude of the shear at any
A. number of supports B. cannot support a force, but can D. a' fixed pulley point is equal to
B. position of members transmit a moment A. the slope of the moment line at that
C. degree of indeterminacy c. cannot support a load, but can 75. This is used· to account for rope point .
D. number of frames transmit a torque rigidity . B. the angle of the moment !ins at that
D. can support a load, but cannot A. breaking factor point.
63. A body that is statically transmit a rotation B. loss factor C. the value of shear at that point
indeterminate requires additional C. power factor D. the slope of shear at that point
equations to supplement the equilibrium 69. A simple mechanical machine with D. friction factor
equations. The additional equations the ability to increase an applied force. 82. The moment at any point is equal to
typically involve A . hinge 76. For most wire ropes and chains with A. the product of the moments and
A . moment B. lever 180" contact, the loss factor at low couples from the poi'nt to the left end
B. summation C. pin speeds varies between B. the sum of the moments and couples
C. deflection D. roller A. 1.03 and 1.06 from the point to the left end
D. elongation B. 1.5 and 2.0 C. the length of the moment diagram at
70. In a lever, the ratio of the load- C. 3 and 4 that point
64. A representation of a body in bearing force to applied force is known D. 10and15 D. the total load at that point
equilibrium, showing all applied forces, as the
moments and reactions. A. power solution

REFRESHER MANUAL 2nd Ec;41ion by JAS TORDILLO REFRESHER MANUAL 2nd Edition by JAS TORDILLO
58 - 8 I Day 58 - EXAM Day 58 - EXAM I 58 - 9
Machine Design
83. The magnitude of the moment at A. Shear stress
any point is equal to B. Bending or flexural stress 96. The method has the advantage of
A. the area under the shear line up to C. Thermal stress being able to handle beams of varying (c-1
that point D. Modulus of rigidity cross sections and materials and
B. the length of the· shear line up to that changes a deflection problem into one DAY 58 -ANSWER KEY
point 90. The lower surface of the beam of drawing moment diagrams. 1. 0 16.D 31.A46. A
2. C 17.A 32.047. B
C. the area under the load line up to that experiences . A. Conjugate beam method 3. A Hi. C 33. A 48. 0
point A. tensile stress B. Superposition method 4.A 19.A 34.C49. A
D. the length of the load line up to that B. compressive stress C. Conversion method 5. C 20. A 35. B 50. D
point C. axial stress D. Diversion method 6.8 21.0 36.851. 0
D. bearing stress 7. 8 22. C 37. C52. C
8. A 23. A 38. A 53. B
84. The maximum mome!lt occurs 97. When multiple loads act 9. A 24. 8 39. C 54. A
where 91. The upper surface of the beam simultaneously on a beam, all of the 10. C 25. A 40. A 55. A
A. the shear is maximum experiences loads contribute to deflection and this 11 .A 26.A 41.A56.D
B. the shear is zero A. tensile stress method permits the deflections at a 12.0 27. 8 42. A 57. A
C. the load is zero B. COmpressive stress 13.8 28. C 43. B 58. A
point to be calculated as the sum of the 14.C 29. A 44. A 59. A
D. the moment is zero C. axial stress deflections from each individual load 15.A 30. B 45. C 60. B

85. The moment diagram is straight and


D. bearing stress · acting singly.
A. Conjugate beam method
~I ,
sloping between 92. There is no stress along a horizontal B. Superposition method
A. concentrated loads
B. uniformly distributed loads
plane passing through the centroid of
·the cross section, a plane known as
C. Conversion method
D. Diversion method rn
C. variable loads A. the neutral plane or the neutral axis DAY 58- ANSWER KEY
D. moving loads B. top plane 98. The virtual work method (also known 61 . A 76.A 91 . B
C. bottom plane as the unit load method and the Hardy 62. C 77. A 92. A
63. C 78.8 93. B
86. The moment diagram is curved D. axial plane Cross method) is 64. A 79. A 94. A
(parabolic) upward over A. an extension of the strain energy 65. A 80. 8 95. A
A. concentrated loads 93. Bending stress varies with location method 66. 0 81. A 96. A
B. uniformly distributed loads (depth) within the beam. It is zero at B. deflection method 67.0 82.8 97.8
C. variable loads A. top surface C. division method 68. A 83. A 98. A
69. B 84. 8 99.0
D. moving loads B. the neutral axis D. internal energy method 70. B 85. A 100. A
C. lower surface 71. C 86. 8
87. The average shear stress D. horizontal axis 99. Beams can fail in different ways, 72. C 87.A
experienced at a point along the length including 73. D 88.8
74.A 89. B
of a beam 94. The vertical component of an axial A. excessive deflection 75. B 90.A
A. depends on the shear at that point member's force is B. local and lateral buckling
and the area of the beam A. equal to the vertical component of the C. rotation G
B. depends on the load at that point load applied to the member D. All of the above
C. depends on the volume of the beam B. equal to the sum vertical and
D. depends on the length of the beam horizontal component 100. Occurs when a beam bends more
C. equal to the horizontal component than a permitted amount
88. The shear stress is zero at the top D. Allofthese A. Excessive deflection
and bottom surfaces of the beam and is B. Overloading
maximum at 95. This method equates the external C. False reactions
A. the right end work to the total internal strain energy. D. lack of support
B. the neutral axis (center) A. strain energy method
C. the left end B. internal energy method
D. at either end C. diagram method
D. positioning method
89. Used to indicate the SGurce of the
stress

REFRESHER MANUAL 2nd Edftlon by JAS TORDILLO REFRESHER MANUAL 2nd Edition by JAS TORDILLO
58 - 10 1 Day 58 - Solution Day 58 - Solution I 58 - 11
· Machine Design
A. 148.36 mm "*
DAY 58 - SOLUTION B. 158.36 mm external pressure of 3,000 psi. C. 1350 psi
C. 168.36 mm Determine the hoop stress at the inner D. 1000 psi
1. A flat belt is to transmit 100 Hp to an D. 178.36 mm surface of the cylinder.
air compressor. The small sheave is A. 4,333.33 psi PD
180 mm in diameter and turns at 1200 St = 2tE
p = 120 hp = 89.52 B. 3,333.33 psi
rpm, while the larger sheave turns at P = 2nTN C. 5,333.33 psi "
450 rpm. The service factor may be 10,000 = 220(72)
D. 6,333.33 psi
89.52 = 2nTC ~)
6
taken as 2.0, the. center distance is t - 0 . 2t(0.90)
- .88 In
equal to two times the diameter of the
larger sheave. Determine the length of T = 1.2212 kN-m - p{rJ +rn-2Poro2 Solving the bursting pressure:
belt. . torque S"- . 2 2 S = PD
A. 1,579.64 mm F1- F~ =tangential force= - - . - ro - ri 1L 2t
B 1,979.64 mm rad1us 70 000 - P(?2)
Where: r, = 8" p; = 8,000 psi ' - 2(0.88)(0.90)
C. 2,579.64 mm 212 ro = 16" Po= 3,000 psi
= L2 = 3.2566 kN = 3256.6 N P = 1540 psi
D. 2,979.64 mm * 0.:]512

D1 = 180 mm F 0.30( 152x-


• 1t )
_ s.oo~16 2 +s 2)-2(3oooX16)2 Bursting Pressure should be greater
___l_ =ere = e 180 = 2.216 St;- - than 1540 psi.
D1N1 = DzNz F2 . 16 2 -8
180(1200) = Dz(450) 8. The Coriolis acceleration depends on
Dz = 480 mm F2 - Ft = 5,333.33 psi
2.216 the latitude,
C = 2Dz = 2(480) = 960 mm -
A. Because the earth's tangential
fl. . 6. A cylindrical tank is subjected to a
L=2C+1 .57(D .+D)+ (D2 -D1)2 F1 - - - = 3256.6 maximum pressure of 3 MPa. The velocity is less near the poles than at
2 1 4C 2.216 inside diameter is 2,000 mm while the the equator. *
=2(960) + 1.57(480+180) riveted joint has a longitudinal pitch of B. Because the earth's tangential
F1 =5934.72 N 70 mm· and there are two straps placed. velocity is greater near the poles than at
+ (480-180)2 the equator.
Stress= £l The design stress for the plate and the
4(960) rivets are 170 MPa and 220 MPa, C. Because the earth's tangential
bt
= 2,979.64 mm velocity is not enough near the poles
20 = 5934.72 respectively. Determine the thickness of
the plate. Assume 70% efficiency of the than at the equator.
2. In Problem No. 1, Find the arc of . b(20) D. . Because the earth's tangential
joints.
contact of small sheave in degrees. b = 148.36 mm A. 15.21 mm velocity is sometimes greater near .the
A. 121.25° B. 25.21 mm • poles than at the equator.
B. 141.25° 4. An 8 inches diameter pulley turning C. 35.21 mm
c. 161.25° .. at 600 rpm is belt connected to a 14 D. 39.21 mm 9. This is used when motion of a particle
D. 181.25° inches diameter pulley. If there is a 4% is described with respect to something
slip, find the speed of the 14 inches else in motion
0 pulley.
t = PD = 3(2000) = 25.21 mm
A. Relative motion *
Arc of Contact = 180o _ (D - d)6o A. 329 rpm*
2ste 2(170XO. 70)
B. Inertia
c B. 239 rpm C. Polar
= o _ (4so -tso )6oo _ 0 C. 932 rpm 7. A steel cylindrical air receiver with 6.0 D. Static
180 960 -161.25 D. 500 rpm feet diameter and pressure load of 220
psi, design stress of 10,000 psi 10. The particle's position, velocity, and
D1N1 = DzNz maximum. Assume a 90% weld joint acceleration may be specified with
3. A 750 mm steel pulley transmits 120
8(600)(1- 0.04) =(14)N2 efficiency. The lap welding tensile respect to another moving particle or
hp at 700 RPM. The arc of contact
N2 = 329 rpm strength is 70,000 psi. Determine the with respect to a moving frame of
between the belt and pulley is 152
bursting steam pressure of this air reference, known as
degrees, the coefficient of friction
5. An air cylinder having an internal receiver. A. an inertial frame of reference
between belt and pulley is 0.30 and the
diameter of 16 inches and an external A. 1530 psi B. a functional frame of reference
safe working stress of the belt is 2.0
diameter of 32 inches is subjected to an B. 1600 psi* C. a non-inertial frame of reference *
MPa. Find the width of the belt used in
mm if its thickness is 20 mm. internal pressure of 8,000 psi and an D. a pivot point

REFRESHER MANUAL 2nd Edition by JAS TORDILLO REFRESHER MANUAL 2nd Edition by JAS TORDILLO
58 -12 1 Day 58- Solution Day 58 - Solution 158 - 13
Machine Design
11 . It is a study of the geometry of B. dependent motion
motion without consideration of the C. radial motion 29. Another name for moment is
A. tangential force
causes of motion D. plane motion * B. central force A. torque •
A. Kinematics*
C. internal force * B. latent
B. Kinetics 17. A point at which the body could be D. external force C. resultant
C. Physics fixed (pinned) without changing the D. opposite
D. Translation instantaneous angular velocities are 23. These are the tensile and
concerned, the body seems to rotate compressive forces within parts of the 30. When acted upon by a moment,
12. Kinematics .. deals only with about a fixed instantaneous center. body,· as found from the product o.f unrestrained bodies will
relationships among A. The instantaneous center * stress and area. A. remain steady
A. weight, velocity, acceleration, and B. The pivot center
A. Internal forces * B. rotate*
time C:'"Fhe wheel center
B. External forces C. remain static
B. vectors, velo<!:ity, acceleration, and D. The.support center
C. Inherent forces D. maintain its position
time
D. Axial forces
C. forces, velocity, acceleration, and 18. ·for a rolling wheel, the 31. When a restrained body is acted
time instantaneous center is 24. A vector of unit length directed along upon by a moment
D. position, velocity, acceleration, A. The point of contact with the load A. there is no rotation •
and time* a coordinate axis.
surface B. rotate
A. unit length
B. The point of contact with the moving B. unit vector * C. move
13. Particles do not possess rotational surface
C. unit displacement D. divert
kinetic energy. All parts of a particle C. The point of contact with the
have D. unit direction
supporting surface * 32. Typical units of moments are
A. The different instantaneous D. The point of contact with the static A. foot-pounds
25. A push or pull that one body exerts
displacement, velocity and acceleration. surface
on another. B. inch-pounds
B. The s-lrme instantaneous C. newton-meters
A. Force*
displacement, velocity and 19. Used to compute· the absolute
B. Energy D. All of these *
acceleration• acceleration of a point as if a body is in
C. Moment
C. Unknown instantaneous pure rotation about that point.
D. Work ~- The line of action of the moment
displacement, velocity and acceleration. A.. instantaneous center of vector is -·"
D. All of these acceleration * A. Normal to the plane containing the
26. A vector having magnitude, direction
B.lnstarttaneous center of velocity and location in three-dimensional space. force vector and the position vector*
14. When the position of one particle in C. instantaneous center of displacement A. concentrated force/point force * B. Tangent to the plane containing the
a multiple-particle system depends on D. instantaneous center of rotation
B. pivot point force vector and the position vector
the position of one or more other
C. cE!ntral point C. Dependent to the plane containing
particles, the motions are said to be 20. Part of the subject known as
D. point of tangency the force vector
A. kinetics engineering mechanics and it is the D. Parallel to the plane containing the
B. independent study of rigid bodies that are stationary.
27. The line in the direction of the force position vector
C. dependent* A. ·statics*
D. plotted extended forward and backward.
B. Kinetics 34. Place the position and force vectors
A. moment of a force
C. Displacement tail to tail. Close your hand and position
B. line of action of a force *
15. A block-and-pulley system with one D. Force it over the pivot point. Rotate the'
fixed roped end is C. rotation of a force
D. all of these position vector into the force vector, and
A. a dependent system * 21. A force on a rigid body caused by position your hand such that your
B. an independent system other bodies.
28. The name given to the tendency of a fingers curl in the same direction as the
C. a popular system A. tangential force
force to rotate. turn, or twist a rigid body position vector rotates. Your extended
D. a plane system B. central force
about an actual or assumed pivot point. thumb will coincide with the direction of
C. internal force the moment.
16. The sum of a translational A. radial
D. external force * A. Rule ofThumb
component and a rotation about a fixed B. couple
C. moment* B. Newton's rule
axis 22. A force which holds parts of the rigid
D. positive C. Right hand rule *
A. circular motion body together. D. Magic rule
REFRESHER MANUAL 2nd Edition by JAS TORDILLO REFRESHER MANUAL 2nd Edition· by JAS TORDILLO
58- 141 Day 58- Solution Day 58 - Solution I 58 - 15
Machine Design
35. A statement of how the total moment A. linear force system *
is. derived from a number of forces B. circular force system C. general force system 55. Generally, the terms simple support
acting simultaneously at a point C. normal force system D. coplanar force system * and simply-supported refers to
A. Ptolemy'.s Theorem D. stationary force system A. a roller support *
B. Varignon's Theorem * 49. All other combinations of non- B. cantilever
C. Planck's Theorem 42. Force which is equal to the sum of concurrent, non-parallel, and non- C. single support
D. Moment Theorem the individual forces coplanar forces. D. elastic support
A. equivalent resultant force * A. general three-dimensional system*
36. The sum of. individual moments B. distributed force B: static system 56. Supports both vertical and horizontal
about a point caused by multiply C. redundant force C. support system forces.
concurrent forces is equal to the D. tangential force D. roller system A. roller support
moment of the resultant force about the B. friction support
same point. 43. If an object is continuously loaded 50. An object is static when it is C. block support
A. Plato's Theorem over a portion of its length or load result A. moving D. pinned support*
B. Varignon's Theorem " from ·· dead load and hydrostatic B. rolling
C. Planck's Theorem presS\Jre. C. pushed 57. When a body is in contact with a
D. Torque Rules A. Moving load D. stationary * frictionless surface, there is
B. Distributed load " A. no frictional force component
37. Any pair of equal, opposite, and C. Fixed load 51. To be stationary, all of the forces on parallel to the surface *
parallel forces constitute a D. Parallel load the object to be in equilibrium, the B. a frictional force component parallel
A. concurrent resultant force and moment vectors to the surface
B. diagonal 44. The equivalent load in a distributed must be C. more frictional force component
c. couple* load is A. equal parallel to the surface
D. equilibrium · A. the area under the loading curve * B. equal to one D. less frictional force . component
B. the graph under the loading curve C. parallel parallel to the surface
38. Two forces can be replaced by a C. the volume under the loading curve D. zero*
moment vector, which can be moved to D. the product under the loading curve 58. Frictionless contact is particularly
any location on ·a body. Such a moment 52. A two-force member can be in us,eful when dealing with systems of
is known as 45. All of the forces act at the same equilibrium only if the two forces have spheres and' cylinders in contact with
A. a free moment * point the same line of action and are rigid supports. Frictionless contact is
B. collinear A coplanar force system A. equal and parallel also assumed for
C. coplanar B. steady force system B. unequal and tangent A. roller and rocker supports *
D. stationary C. concurrent force system * C. equal but opposite * B. block support
D. stationary force-system D. unequal but opposite C. welded support
39. The combination of the moved force D. wooden support
and the couple is known as 46. All of the forces share. the same line 53. A three-force member can. be in
A. a moment system of action. equilibrium only if the three forces are 59. When the equations of equilibrium
B. a turning system A. collinear force system * A. linear ' are independent, a rigid body . force
C. a force-couple-system • B. steady force system system is said to be·
B. concurrent*
D. linear force system C. concurrent force syster:n C. opposite A. statically determinate *
D. stationary force system D. coplanar B. statically indeterminate
40. A force system in which all forces C. unbalanced system
are parallel and applied along a straight 47. All of the forces are parallel 54. For beams, the two most common D. port system
line. A. opposite force system
types of supports are the roller support
A. linear force system" B. parallel force system * and the 60. When the body has more supports
B. circular force system C. coplanar force system than are necessary for equilibrium, the
A. pinned support *
C. normal force system D. equal force system B. backed support force system is said to be
D. stationary force system C. block support A. statically determinate
48. All of the forces are in a plane. D. friction B. statically indeterminate *
41. A straight beam loaded by several A. steady force system ' C. unbalanced system
concentrated forces is an example of a B. concurrent force system D. port system

REFRESHER MANUAL 2nd Edl~ion by JAS TORDILLO REFRESHER MANUAL 2nd Edition by JAS TORDILLO
58 -161 Day 58- Solution Day 58 - Solution 1 58 - 17
Machine Design
61. In a statically indeterminate system, C. direction of the reaction
one or more of the supports or members D. magnitude of the reaction * 73. If the pulley is attached by a bracket C. load diagram
can be removed or reduced in restraint or cable to a fixed location, it is known D. graphical presentation
without affecting the equilibrium 67. These are added to structures to as
position. These supports and members prevent translation while permitting A. a free pulley 80. The shear at any point is equal to
are known as rotation. B. solid pulley A. the product of the loads and reactions
C. vertical pulley from the point to the left end
A. redundant members • A. rivets D. a fixed pulley * B. the sum of the loads and reactions
B. lever B. bolt from the point to the left end *
C. pulley C. lever 74. If the pulley is attached to a load, or C. the sum of the loads and reactions
D. continuous membtrs D. hinges* if the pulley is free to move, it is known from the point to the right end
as D. the product of the loads and
62. The number>of redundant members 68. A frictionless hinge A. a free pulley * reactions from the point to the midpoint
is known as the A. can support a force, but cannot B. a solid pulley
A. number of supports transmit a moment * C. a vertical pulley 81 . The magnitude of the shear at any
B. position of members B. cannot support a force, but can D. a fixed pulley point is equal to
C. degree of indeterminacy • transmit a moment . A. the slope of the moment line at
D. number of frames C. cannot support a load, but can 75.· This is used to account for rope that point * .
transmit a torque rigidity. B. the angle of the moment line at that
63. A body that is statically D. can support a load, but cannot A. breaking factor point.
indeterminate requires additional transmit a rotation B. loss factor * C. the value of shear st that point
equations to supplement the equilibrium C. power factor D. the slope of shear at that point
equations. The additional equations 69. A simple mechanical machine with D. friction factor
typically involve the ability to increase an applied force. 82. The moment at any point is equal to
A. moment A. hinge 76. For most wire ropes and chains w ith A. the product of the moments and
B. summation B. lever* 180° contact, the loss factor at low couples from the point to the left end
C. deflection * C. pin speeds varies between B. the sum of the moments and couples
D. elongation D. roller A. 1.03 and 1.06 * t,rom the point to the left end •
B. 1.5 and 2.0 C. the lengttr·of the moment diagram at
64. A representation of a body in 76. In a lever, the ratio of the load- C. 3 and 4 that point
equilibrium, showing all applied forces, bearing force to applied force is known D. 10and15 D. the total load at that point
moments and reactions. as the
A. free body diagram * A. power solution ·n . The loss factor is the 83. The magnitude of the moment at
B. body graph B. mechanical ad'4antage * A. reciprocal of the pulley efficiency * any point is equal to' · ~
C. line graph C. force diagram B. equal to the pulley efficiency A. the area under the shear line up to
D. body sketch D. loao implication C. two times the pulley efficiency that point*
D. None of these B. the length of the shear line up. to that
65. A graph of the magnitude of a ' 71. This is used to change the direction point
reaction as a function of the load of an applied tensi le force 78. An axial member is capable of C. the area under the load line up to that
placement. A. bearing supporting point
A. influence diagram • B. belt A. axial and radial forces D. the length of the load line up to that
B. magnitude diagram C. pulley or sheave * B. axial forces only * point
C. load diagram D. clutch C. radial forces
D. lever diagram D. triaxial forces 84. The maximum moment occurs
72. A series of pulleys working together where
66. In influence diagram, the x-axis of known as 79. Graphs of shear and moment as A. the shear is maximum
the graph corresponds to the location on A. train motion functions of position along the beam are B. the shear is zero *
the body (along the length of a beam) B. rope system known as C. the load is zero
and the y-axis corresponds to the C. a block and tackle * A. shear and moment diagrams * D. the moment is zero
A. deflection D. flywheel system B. schematic diagram
B. position of the reaction

REFRESHER MANUAL 2nd Edition by JAS TORDILLO REFRESHER MANUAL 2nd Edition by JAS TORDILLO
58- 18 1 Day 58- Solution Day 58 - Solution I 58 - 19
92. There is no stress along a horizontal
Machine Design
85. The _q10ment diagram is straight and
sloping between plane passing through the centroid of C. Conversion method
A. concentrated loads • the cross section, a plane known as D. Diversion method
B. uniformly distributed loads A. the neutral plane or the neutral
C. variable loads axis * 98. The virtual work method (also known
D. moving loads B. top plane as the unit load method and the Hardy
C. bottom plane Cross method) is
86. The moment diagram is curved D. axial plane A. an extension of the strain energy
(parabolic) upward.over method •
A. concentrated loads 93. Bending stress varies with location
B. deflection method
B. uniformly distributed loads * (depth) within the beam. It is zero at
C. division method
C. variable loads A. top surface D. internal energy method
D. moving loads B. the. neutral axis *
C. lower surface 99. Beams can fail in different ways,
87. The average shear stress D. horizontal axis including
experienced at a point along the length A excessive deflection
of a beam 94. The vertical component of an axial
B. local and lateral buckling
A. depends on the shear at that point member's force is C. rotation
and the area of the beam * A. equal to the vertical component of
D. All of the above *
B. depends on the load at that point the load apptied to the member •
C. depends on the volume of the beam B. equal to the sum vertical and
100. Occurs when a beam bends more
D. depends on the length of the beam horizontal component
than a permitted amount
C. equal to the horizontal component A. Excessive deflection *
88. The shear stress is zero at the top D. All of these B. Overloadir.g
and bottom surfaces of the beam and is C. False reactions
maximum at 95. This method equates the external
D. lack of support
A. the right end work to the total internal strain energy.
B. the neutral axis (center) * A. strain energy method *
C. the left end B. internal energy method
D. at either end c·. diagram method
D'. positioning method
89. Used to indicate the source of the
stress 96. The method has the advantage of
A. Shear stress being able to handle beams of varying
B. Bending or flexural stress * cross sections and materials and
C. Thermal stress changes a deflection problem into one
D. Modulus of rigidity of drawing moment diagrams.
A. Conjugate beam method *
90. The lower surface of the beam B. Superposition method
experiences C. Conversion method
A. tensile stress • D. Diversion method
B. compressive stress
C. axial stress 97. When multiple loads act
D. bearing stress simultaneously on a beam, all of the
loads contribute to deflection and this
91. The upper surface of the beam method permits the deflections at a
experiences point to be calculated as the sum of the
A. tensile stress deflections from each individual load
B. compressive stress * acting singly.
C. axial stress A. Conjugate beam method
D. bearing stress B. Superposition method*

REFRESHER MANUAL 2nd Editton by JAS TORDILLO REFRESHE~ MANUAL 2nd Edition by JAS TORDILLO

j
Day 59 - EXAM I 59 - 1
Machine Design
C. model
DAY 59- EXAM D. structure

1. Excessive deflection is elastic and no 8. The connection points of a truss are


yielding occurs. For this reason, the known as
failure mechanism is sometimes called A. columns
A. elastic failure B. joints
B. deflection error C. support
C. buckling D. beam
D. structural failure
9. These consist of all memt>ers in a
2. The beam does not yield, the closes loop of members. For the truss to
excessive deflection may cause be stable, all of these must be triangles.
A. crack in plaster and sheetrock A. structural cell
B. misalignment of doors and windows B. staiic member
C. occupant concern and lack of C. truss member
confidence in the structure · D. continuous beam
D. All of these
10. A braced structure spanning a
3. An overload that occurs near large ravine, gorge, or other land depression
concentrated loads I n order to support a road or rail line.
A. we.ak supports A. trestle
B. local buckling B. columns
C. rotation C. pier
D. unstable structure D. beam

4. Two types of local buckling 11. If the left-hand side is greater than
A Vertical buckling and web crippling the right-hand side (i.e., there are
B. Horizontal buckling and static re_dundant members), the truss is
crippling A. statically determinate
C. Trestle and static buckling B. statically indeterminate
D. Finite and sideways buckling C. stable joint
D. unstable joint
5. Occurs when a long, unsupported
member rolls out of its normal plane 12. If the left-hand side Is less than the
A. Random buckling right-hand side, the truss is
B. Lateral buckling A. unstable
C. Method buckling B. stable
D. Inelastic C. static
D. dynamic
6. An inelastic (plastic) failure of the
beam. 13. This is one of three methods which
A Rotation can be used to find the internal forces in
B. Buckling each truss member.
C. Vertical A. method of joints
D. Trestle B. method of truss
C. method of sections
7. A set of pin-connected axial members D. superposition .
A. figure
B. truss

REFRESHER MANUAL 2nd Editit>nby JAS TORDILLO


59-21 Day59-EXAM . Day 59 - EXAM I 59 - 3
14. This method can be used to find 20. Buckling failure is sudden, often
Machine Design
forces in inclined members. This method without significant sideways bending.
27. Columns that have a slenderness A. 100 feet
is strictly an application of the vertical The load at which a column fails is
ratio of less than the critical slenderness B. 500 feet
equilibrium condition (l:Fy = 0). known as the critical load or
ratio but which are too tall to be piers C. 1000 feet
A. method of joints A. Euler load
are known as D. 100m
B. method of sections B. Newtonian load
A. intermediate column
C. cut-and-sum method C. Board load
D. Euler method
B. a braced column 34. In Sl countries, the standard metric
D. Unbalanced load
C. composite column tape is
15. A direct approach to finding forces in 21. The theoretical maximum load that
D. spring column A. 10m
any truss member. This method is an initially straight column can support
B. 50 or 100 meters
28. One of the methods of determining ~25m
convenient when only a few truss without buckling.
the buckling load for an intermediate D. ,000 m
member forces are unknown. A. Francis load
A. method-of-sections column is
B. Euler load
A. torsion formula 35. The coefficient of linear -expansion,
B. method of joints C. Eutectic load
B. secant formu la a, for steel has an approximate value of
C. cut-and-sum method D. Shear load
C. shear formula A. 6.5'x 10'6 1f'F
D. Euler method
D. method of sections B. 65 x 10.s 1f'F
22. The quantity Uk is known .as
C. 650 X 10-6 1f'F
16. A term used to describe the process A. critical ratio
29. Shear stress occurs when a shaft is D. 100 x 10-6 1/"F
of determining member forces by B. slenderness ratio
considering loads one at a time. C. buckle ratio placed in
A. bending 36. One in which two or more different
A. superposition D. stiffness ratio
B. torsion materials are used
B. method of load
C. tension A. mono block structure
C. column method 23. Long columns have
D. compression B. composite structure
D. critical method A. low slenderness ratios
C. multiple structure
B.· high slenderness ratios
30. The shear modulus of a shaft is D. dual structure
17. Very short compression meml:)ers C. variable slenderness ratios
equal to
are known as D. typical slenderness ratio 6
A. 11 .5 x 10 psi 37. This method assumes that the
A. piers
B. 30 x 106 psi strains are the same in both materials at
B. columns 24. Typical critical slenderness ratios
C. 50 x 106 psi the interface between them. Most simple
C. square beam range from 6
D.1 x 10 psi composite structures can be analyzed
D. spring beam A. 80 to 120
using the
B. 1 to 5
31. The torque carried by a shaft A. the area transformation method
18. Long compression members are C. 10 to 30
spinning at revolutions per m inute is B. the strain deformation method
known as D. 40 to 60
related to the transmitted C. the stress analysis method
A. piers
A. horsepower D. the modulus system method
B. columns 25. The critical slenderness- ratio
B. shear stress
C. continuous beam becomes smaller as the compressive
C. moment 38. Assumed to be perfectly elastic
D. static member yield strength
D. tangential stress within its working range.
A. increases
A. threaded bolt
19. Failure in piers occurs B. decreases
32. Shear stress due to torsion in a thin- B. springs
A. when the applied stress exceeds the C. equal to shear strength
walled, noncircular shell (also known as C. ropes
yield strength of the material D. equal to bending strength
a closed box) acts D. steel shaft
B. when the applied stress is less than
the yield strength of the material 26. If a column is braced against A. around the perimeter of the shell
B. inside the surface of the shell 39. In engineering design, the term
C. when the applied load is less than buckling at some point between its two
C. at the center of the shell allowable always means that
yield strength of the material ends, the column is known as
D. at the top of the shell A. a factor of safety has been applied to
D: when the applied moment is less A. intermediate column
the governing material strength
than the strength of the material B. a braced column
33. The standard surveyor's tape in the B. the strength is. multiplied by the factor
C. composite column
United States is a flat steel ribbon with a of safety
D. spring column
length of
REFRESHER MANUAL 2nd Edition by JAS TORDILLO REFRESHER MANUAL 2nd Editlori by JAS TORDILLO
59-41 Day 59 - EXAM Day 59 - EXAM I 59 - 5
Machine Design
C. the strength is added by the factor of 46. A cylindrical tank is a thin-walled
safety tank if Its wall thickness-to-internal 52. The difference between the inner 58. If each of the coordinates is
D. all of these diameter ratio is and outer dimensions. independent of the other, the
A. less tban approximately 0.10 A. diametral interference coordinates are known as
40. A spring stores energy when it is B. greater than approximately 0.10 B. circumferential interference A. harmonic coordinates
compressed or extended. By the work C. greater than 0.15 C. offset interference B. holonomic coordinates
energy principle, the energy storage is D. less than 10 D. yield distance C. relative coordinates
A. equal to the work to displace the D. opposite coordinates
spring 47. The hoop stress also known as 53. The coefficient of friction for press
B. equal to the distance to displace the fl;... circumferential stress and tangential fits is high'ly variable, having been 59. The v(:lctor form of the particle's
spring stress reported in the range of position is where the vector
C. equal to the power to displace the B. shear stress A . 0.03 to 0.33 A. has magnitude only
spring C. longitudinal stress B.0.1to10 B. has both magnitude and direction
D. equal to the stress to displace the D. diametral stress C. has direction only
C. 10 to 100
spring D. 50 to 75 D. has coordinates
48. The axial forces on the ends of the
41. The ratio of the mean coil diameter cylindrical tank produce a stress 60. One in which particles move only in
54. For ductile materials, the material
to wire size is known as directed along the tank's longitudinal straight lines.
strength used is
A. stiffness of spring axis, known as A. shear strength A. coordinate system
B. spring index A. longitudinal stress B. yield strength B. linear system
C. diametral ratio B. tangential stress 9. point-slope system
C. tensile strength
·D. spring number C. bending stress D. dynamic system
D. elastic limit
D. linear stress
42. The spring index is optimum 55. For steel, the factor of safety ranges 61. When values of time are substituted
A. around 9 49. There is no unique axis in a into these equations, the position ,
from
B. around 15 spherical tank or in the spherical ends of velocity, and acceleration are known as
A. 1.5 to 2.5
C. around 5 a cylindricai tank. Therefore. the hoop B. 5to 10 A. instantaneous values
D. around 20 and long stresses are B. displacement values
C. 10 to 20
A. proportional 0. numerical values
D. 20 to 100
43. The useful range of spring index is B. identical 0-. cooroinate values
approximately C. similar 56. For brittle materials, the material
A. 1 to 10 D. different
strength used is 62. The ne~ change in a particle's
B. 4 to 12 position, as determined from the
A. ultimate strength
C. 9·to 20 50. A thick-walled cylinder has a wall position function
B. yield strength
D. 20 to 100 thickness-to-diameter ratio C. proportional limit A. displacel)"lent
A. greater than 0.1 D. elastic limit B. position
44. It is difficult to wind springs with B. less than 0.10 C. distance.
small spring indexes, but, on the other C. less than 0.01 D. slope
57. In general, the number of degrees of
hand, springs with large spring indexes D. greater than 0.01
freedom is
are A. equal to the number of coordinates 63. The accumulated length of the path
A. flimsy and tend to buckle 51. It is possible to install a shaft or
required to completely specify the state traveled during all direction reversals.
B. ideal and can support load cylinder in a hub whose opening is A. velocity
of an object
C. identical slightly smaller than the shaft. This can B. distance
B. more than the number of coordinates
D. can support heavy loads be done by force in a hydraulic press , or C. speed
required to complet.ely specify the state
by heating the hub and cooling the shaft D. motion
of an object
45. Tanks under internal pressure prior to assembly. The resulting C. less than the number of coordinates
experience assemblage is known as required to completely specify the state 64. Distance is always
A. circumferential stress A. a press fit or interference fit
of an object A. less than displacement
B. longitudinal stress B. dynamic fit B. greater than or equal to displacement
D. none of the above
C. radial stress C. diametral fit C. greater than linear velocity
D. All of these D. hydraulic fit D. less than linear velocity

REFRESHER MANUAL 2nd Edition by JAS TORDILLO REFRESHER MANUAL 2nd Edition by JAS TORDILLO
59 - 6 I Day 59 - EXAM Day 59 - EXAM I 59 - 7
Machine Design
65. A vector, having both magnitude and 72. A projectile is placed into motion by
direction A. an initial velocity 79. The study of a body's motion relationships between forces in the
A. Velocity B. an initfal impulse independent of the forces on the body mechanism can be determined by
B. speed C. an initial slope A. kinetics statics.
C. area D. an initial displacement B. kinematics A. one-dimensional mechanism
D. time C. dynamics B. two-dimensional mechanism
73. Kinematics deals only with dynamics D. statics C. three-dimensional mechanism
66: Brittle failure is sudden and during the flight, and the force acting on D. rigid mechanism
unpredictable, the factor of safety the projectile during the launch phase is 80. This design method does not use
ranges from covered in allowable stresses at all and the 86. A simple three-dimensional truss
A. 1 to 3 A. kinetics member is designed so that its actual (frame) which consists of three axial
B. 6to 10 B. dynamics nominal strength exceeds the required members.
C. 3 to 6 C. staticS ultimate strength. A. dual-pod
D. 2 to 5 D. motion system A. ultimate strength design method B. Tripod
B. load factor design method C. beam
67. If an actual stress is less than the 74. In projectile, the trajectory is C. plastic design method D. movable member
allowable stress A . circular D. all of these
A. the design is considered acceptable B. parabolic 87. Stress concentration occur
B. the design is unacceptable C. linear 81 . The ultimate strength of a member is whenever there is
C. the design will fail D. rectilinear calculated from the actual service loads A. non-uniformity in an object
D. the design is wrong and multiplicative factors known as B. uniformity in an object
75. In projectile, the impact velocity is A overload factors C. continuity in an object .
B. safety factors D. balance of an object
68. A scalar quantity, equal to the A equal to initial velocity
magnitude of velocity. B. equal to initial displacement C. lever factors
D. material strength factor 88. In projectile, the impact angle is
A. distance C. greater than initial velocity
B. speed D. less than initial velocity A. greater than the initial launch angle
C. time 82. Used to determine the actual stress B. equal to the initial launch angle
experienced on the surface · of a C. less than the initial launch angle
D. angle 76 . If an actual stress is less than the
member. It consists of a folded wire D. more or less than the initial angle
allowable stress, the design is
assembly that exhibits a resistance
69. Means uniform velocity or the considered acceptable. This is the
change as the wire length changes. 89. Deals with the elastic behavior of
velocity is constant and the acceleration principle of the allowable stress design
A. wire gage loaded engineering materials.
is zero. method, also known as
A . uniform motion A. working stress dl')sign method
B. strain gage A . Strength of materials
C. stress gage B. Materials engin~ing
B. variable speed B. short cut method
D. resistance gag1= C. Engineering mechanics
C. uniform velocity C. safety factor method
D. variable motion D. allowable 'method D. Fluid mechanics
83. Commonly used wire in strain gage.
A. Iron wire 90. A semi-graphical technique that is
70. For a constant velocity system the 77. The study of moving objects.
B. Nichrome wire applicable whenever slopes of deflection
position function varies A. stati~
C. Copper wire beams are not too great.
A . linearly with time B. dynamics
D. Live wire A. shear- deflection method
B. inversely w ith time C. rectilinear motion
C. proportional with time D. projectile B. moment- area method
84. The strain gage sensitivity also C. sum-product method
D. inversely proportional with time
known as D. load- area method
78. In dynamics, the subject is divided
A . the gage factor
71 . Linear acceleration means that the into two
B. the length factor 91. When more than one spring or
acceleration A. kinematics and kinetics
C. t~e wire factor resisting member share the load, the
A increases uniformly with time B. statics and dynamics
D. the usageiactor relative stiffnesses are known as
B. decreases uniformly with time C. rectilinear and angular
C. proportional with time D. linear and circular
A elasticities
85. A non-rigid structure. Altnough parts B. rigidities
D. inversely w ith time
of the mechanism move, the

REFRESHER MANUAL 2nd Edition by JAS TORDILLO .


REFRESHER MANUAL 2nd Edition by JAS TORDILLO
-
59 - 8 I Day 59 - EXAM Day 59 - Solution I 59 - 9
Machine Design
C. concentric C. modulus of elasticity
D. shear modulus D. tensile modulus B. truss*
99. These are correction factors used to DAY 59 - SOLUTION C. model
92. The area is normal to the force account for the non-uniform stress D. structure
carried distributions 1 . Excessive deflection is elastic and no
A. shear stress A. Stress concentration factors yielding occurs. For this reason, the 8. The connection points of a truss are
B. normal stress B. Shear factor failure mechanism is sometimes called known as
C. strain C. Beam factor A. elastic failure* A. columns
D. bearing strength D. Material deflection factor B. deflection error B. joints*
100. Values of the stress concentration C. buckling C. support
93. The area is parallel to the force. factor are almost always D. structural failure D. beam
A. normal stress A. greater than 1.0 and can run as high
B. shear stress as 3.0 and above 2. The beam does not yield, the 9. These consist of all members in a
C. tangential stress B. less than 1.0 excessive deflection may cause closed loop of members. For the truss to
D. longitudinal stress C. greater than 5.0 A. crack in plaster and sheetrock be stable, all of these must be triangles.
D. less than 10 B. misalignment of doors and windows A. structural cell *
94. Elongation expressed on a fractional C. occupant concern and lack of B. static member
or percentage basis (rl" ) confidence in the structure C. truss member
A. linear elongation D. All of these* D. continuous beam
B. strain DAY 59- ANSWER KEY
1. A 16.A 31.A46.A
C. stress 2. D 17. A 3'2.. A~.7. A 3. An overload that occurs near large 10. A braced structure spanning a
D. deflection • 3. 8 18. 8 33. A 48. A concentrated loads ravine, gorge, or other land depression
4. A 19. A 34. 849. B A. weak supports in order to support a road or rail line.
95. The sum of the strains in the three 5. 8 20. A 35. A 50. A
6. A 21.8 36. B51 . A
B. local buckling * A. trestle*
coordinate directions. C. rotation B. columns
7. 8 22.8 37. A 52. A
A . inertia 8. 8 23. 8 38. 8 53. A D. unstable structure C. pier
~-dilation 9. A 24. A 39. A 54. 8 D. beam
C. summation 10.A 25. A 40. A 55. A 4. Two types of local buckling
D. shear modulus 11.8 26_8 41 . 8 56. A
12.A 27:· A 42. A 57. A
A. Vertical buckling and web 11. If the left-hand side is greater than
13.A 28. 8 43. 8 58. 8 crippling* the right-hand side (i.e., there are
96. Hooke's law is a simple 14.C 29. 8 44. A 59. 8 B. Horizontal buckling and static redundant members), the truss is
mathematical statement of the 15.A 30. A 45. D60. B crippling A. statically determinate
relationship between elastic stress and C. Trestle and static buckling B. statically indeterminate *
strain G D. Finite and sideways buckling C. stable joint
A. stress is proportional to strain D. unstable joint
B. strain is proportional to stress (rl" ) 5. Occurs when a long, unsupported
C. stress is inversely proportional to DAY 59- ANSWER KEY member rolls out of its normal plane 12. If the left- hand side is less than the
strain 61 . A 76. A 91.8 A. Random buckling right-hand side, the truss is
D. stress is two times strain 62. A 77.8 92. 8 B. Lateral buckling * A.. unstable *
63. 8 78. A 93. 8
64. 8 79. 8 94. 8
C. Method buckling B. stable
97. For normal stress, the constant of D. Inelastic C. static
65. A BO.D 95.8
proportionality is 66.8 81.A 96.A D. dynamic
A. modulus of elasticity 67. A 82.8 97. A 6. An inelastic (plastic) failure of the
B. modulus of rigidity 68. 8 83. 8 98. A beam. 13. This is one of three methods which
C. tensile modulus 69. A 84. A 99. A
70. A 85. B 100. A A. Rotation * can be used to find the internal forces in
D. stress modulus B. Buckling each truss member.
71. A 86.8
72. 8 87. A C. Vertical A. method of joints *
98. For shear stress, the constant of 73. A 88.8 D. Trestle B. method of truss
proportionality is 74. 8 89. A
C. method of sections
A. the shear modulus 75.A 90. B
7. A set of pin-connected axial members D. superposition
B. stress modulus ~ I A. figure

REFRESHER MANUAL 2nd Edition by JAS TORDILLO REFRESHER MANUAL 2nd Edition by JAS TORDILLO

jj
-
59 - 10 1 Day 59 - Solution Day 59 - Solution 159 - 11
Machine Design
14. This method can be used to find 20. Buckling failure is sudden, often
forces in inclined members. This method without significant sideways bending. 27. Columns that have a slenderness A. 100 feet •
is strictly an application of the vertical The load at which a column fails is ratio of less than the critical slenderness B. 500 feet
equilibrium condition (LFy = 0). known as the critical load or ratio but which are too tall to be piers C. 1000 feet
A. method of joints A. Euler load * are known as D. 100m
B. method of sections B. Newtonian load A. intermediate column *
C. cut-and-sum method * C. Board load B. a braced column 34. In Sl countries, the standard metric
D. Euler method D. Unbalanced load C. composite column tape is
D. spring column A. 10m
15. A direct _
approach to finding forces in 21. The theore.icPI maximum load that B. 50 or 100 meters *
any truss member. This method is an initially straight column can support 28. One of the methods of determining C 25m
convenient when only a few truss without buckling. the buckling load for an intermediate D. 1000 m
member forces are unknown. A . Francis load column is
A. method-of-sections * B. Euler load * A. torsion formula 35. The coefficient of linear expansion,
B. method of joints c. Eutectic load B. secant formula * a, for steel has an approximate value of
C. cut-and-sum method D. Shear load C. shear formula A. 6.5 X 10"8 1/°F *
D. Euler method D. method of sections B. 65 x 10-e 1f'F
22. The quantity Uk is known as C. 650 X 10-e 1f'F
16. A term used to·describe the process A. critical ratio 29. Shear stress occurs when a shaft is D. 100 x 10-e 1f'F
of determining member forces by B. slenderness ratio * placed in
considering loads one at a time. C. buckle ratio A. bending 36. One in which two or more different
A. superposition * D. stiffness ratio B. torsion* materials are used
B. method of load C. tension A. mono block structure
C. column method 23. Long columns have D. compression B. composite structure *
D. critical method A. low slenderness ratios C. multiple structure
B. high slenderness ratios • 30. The shear modulus of a shaft is D. dual structure
17. Very short compression members C. variable slenderness ratios equal to
are known as D. typical slenderness ratio A. 11.5 X 106 psi* 37. This method assumes that the
6
A. piers* B. 30 X 10 psi strains are the same in both materials at
B. columns 24. Typical critical slenderness ratios C. 50 x 108 psi tbe interface between them. Most simple
C. square beam range from D. 1 X 108 psi composite structures can be analyzed
D. spring beam A. 80 to 120 * using the
B. 1 to 5 31. The torque carried by a shaft A. the area transfonnation method *
18. Long compression members are C.10to30 spinning at revolutions per m inute is B. the strain deformation method
known as D. 40 to 60 related to the transmitted C. the stress analysis method
A. piers A. horsepower* D. the modulus system method
B. columns* 25. The critical slenderness. ratio B. shear stress ,
C. continuous beam becomes smaller as the compressive C. moment 38. Assumed to be perfectly elastic
D. static member yield strength D. tangential stress within its working range.
A. increases* A. threaded bolt
19. Failure in piers occurs B. decreases 32. Shear stress due to torsion in a thin- B. springs *
A. when the applied stress exceeds C. equal to shear strength walled, noncircular shell (also known as C. ropes
the yield strength of the material * D. equal to bending strength a c losed box) acts D. steel shaft
B. when the applied stress is less than A. around the perimeter of the shell *
the yield strength of the material 26. If a column is braced against B. inside the surface of the shell 39. In engineering design, the term
C. when the applied load is less than buckling at some point between its two C. at the center of the shell allowable always means that
yield strength of the material ends, the column is known as D. ·at the top of the shell A. a factor of safety has been applied
D. when the applied moment is less A. intermediate column to the governing material strength *
than the strength of the material B. a braced column * 33. The standard surveyor's tape In the B. the strength _is multiplied by the factor
C. composite column United States is a flat steel nbbon with a of safety
D. spring column length of

REFRESHER MANUAL 2nd Edition by JAS TORDILLO REFRESHER MANUAL 2nd Edition by JAS TORDILLO

II
59 - 12 1 Day 59 - Solution Day 59 - Solution 159 - 13
· Machine Design
C. the strength is added by the factor of 46. A cylindrical tank is a thin-walled
safety tank if its wall thickness-to-internal 52. The difference between the inner 58. If each of the coordinates is
D. all of these diameter ratio is and outer dimensions. independent of the other, the
A. less than approximately 0.10 * A. diametral interference * coordinates are known as
40. A spring stores energy when it is B. greater than approximately 0.10 B. circumferential interference A. harmonic coordinates
compressed or extended. By the work C. greater than 0.15 C. offset interference B. holonomlc coordinates *
energy principle, the energy storage is D. less than 10 D. yield distance C. relative coordinates
A. equal to the work to displace the D. opposite coordinates
spring* 47. The hoop stress also known as 53. The coefficient of friction for press
B. equal to the distance to displace the A. circumferential · stress and fits is highly variable, having been 59. The vector form of the particle's
spring tangential stress * reported in the range of position is where the vector
C. equal to the power to displace the B. shear stress A. 0.03 to 0.33 * A. has magnitude only
spring C. longitudinal stress B.0.1to10 B. has both magnitude and direction*
D. equal to the stress to displace the D. diametral stress C. 10 to 100 C. has direction only
spring D. 50 to 75 D. has coordinates
48. The axial forces on the ends of the
41. The ratio of the mean coil diameter cylindrical tank produce a stress 54. For ductile materials, the material 60. One in which particles move only in
to wire size is known as directed along the tank's longitudinal strength used is straight lines.
A . stiffness of spring axis, known as A. shear strength A. coordinate system
B. spring index * A. longitudinal stress • B. yield strength • B. linear system *
C. diametral ratio B. tangential stress C . tensile strength C. point-slope system
D. spring number C. bending stress D. elastic limit D. dynamic system
D. linear stress
42. The spring index is optimum 55. For steel, the factor of safety ranges 61. When values of time are substituted
A. around 9 • 49. There is no unique axis in a from into these equations, the position,
B. around 15 spherical tank or in tHe spherical ends of A. 1.5 to 2.5 * velocity, and acceleration are known as
C. around 5 a cylindrical tank. Therefore, the hoop B. 5 to 10 A. instantaneous values *
D. around 20 and long stresses are c. 10 to 20 B. displacement values
A. proportional D. 20 to 1oo C. numerical values
43. The useful range of spring index is a. identical '* D. coordinate values
approximately C. similar 56. For brittle materials, the material
A. 1 to 10 D. different strength used is 62. The net change in a particle's
B. 4 to 12 * A. ultimate strength * position, as determined from the
C.~ to 20 50. A thick-walled' -cylinder has a wall B. yield strength position function
D. 20 to 100 thickness-to-diameter ratio C. proportional limit A. displacement*
A. greater than 0.1 * D. elastic limit B. position
44. It is difficult to wind springs with B. less than 0.10 C. distance
small spring indexes, but, on the other C. less than 0.01 57. In general, the number of degrees of D. slope
hand, springs with large spring indexes D. greater than 0.01 freedom is
are A. equal to the number of 63. The accumulated length of the path
A. flimsy and tend to buckle * 51 . It is possible to install a shaft •or coordinates required to completely traveled during all direction reversals. ,
B. ideal and can support load cylinder in a hub whose opening is specify the state of an object * A. velocity
C. identical slightly smaller than the shaft. This can B. more than the number of coordinates B. distance *
D. ca n support heavy loads be done by force in a hydraulic press, or required to completely specify the state C. speed
by heating the hub and cooling the shaft of an object D. motion
45. Tanks under internal pressure prior to assembly. The resulting C. less than the number of coordinates
. experience assemblage is known as required to completely specify the state 64. Distance is always
A. circumferential stress A. a press fit or interference fit * of an object A. less than displacement
B. longitudinal stress B. dynamic fit D. none. of the above B. greater than or equal to
C. radial stress C. diametral fit displacement*
D. All of these • D. hydraulic fit

REFRESHER MANUAL 2nd Edition by JAS TORDILLO REFRESHER MANUAL 2nd Edition by JAS TORDILLO
. 59 - 141 Day 59 - Solution Day 59- Solution 159 -15
Machine Design
C. greater than linear velocity B. decreases uniformly with time
D. less than linear velocity C. proportional with time C. rectilinear and angular 85. A non-rigid structure. Although parts
D. inversely with time D. linear and circu lar of the mechanism move, the
65. A vector, having both magnitude and relationships between forces in the
direction 72. A projectile is placed into motion by 79. The study of a body's motion mechanism can be determined by
A. Velocity ~ A. an initial velocity independent of the forces on the body statics.
B. speed B. an initial impulse* A. kinetics A. one-dimensional mechanism
C. area C. an initial slope B. kinematics • B. two-dimensional mechanism *
D. time D. an initial displacement C. dynamics C. three-dimensional mechanism
D. statics D. rigid mechanism
66. Brittle failure is sudden and 73. Kinematics deals only with dynamics ....
unpredictable, the factor of safety during the flight, and the force acting on 80. This design method does not use 86:- A simple three-dimensional truss
ranges from the projectile during the launch phase is allowable stresses at all and the (frame) which consists of three axial
A. 1 to 3 covered in member is designed so that its actual members.
B. 6 to 10 • A. kinetics * nominal strength exceeds the required A dual-pod
C. 3 to 6 B. dynamics ultimate strength. B. Tripod*
D. 2 to 5 C. statics A. ultimate strength design method C. beam
D. motion system B. load factor design r:nethod D. movable member
67. If an actual stress is less than the C. plastic design method
allowable stress 74. In projectile, the trajectory is D. all of these * 87. Stress concentration occur
A. the design is considered A. circular whenever there is
acceptable* B. parabolic * 81. The ultimate strength of a member is A. non-uniformity in an object*
B. the design is unacceptable C. linear calculated from the actual service loads B. uniformity in an object
C. the design will fail D. rectilinear and multiplicative factors known as C. continuity in an object
D. the design is wrong A. overload fattors * D. balance of an object
75. In projectile, the impact velocity is B. safety factors
68. A scalar quantity, equal to the A. equal to initial velocity * C. lever factors 88. In pmjectile, the impact angle is
magnitude of velocity. B. equal to initial displacement D. material strength factor A greater than the initial launch angle
A. distance C. greater tlmn initial velocity B. equal to the initial launch angle *
B. speed* D. less than initial velocity 82. Used to determine the actual stress G. less than the initial launch angle
C. time experienced on the surface of a D. more or less than the initial angle
D. angle 76. If an actual stress is less than the member. It consists of a folded wire
allowable stress, the design is assembly that exhibits a resistance 89. Deals with the elastic behavior of
69: Means uniform velocity or the considered acceptable. This is the change as the wire length changes. loaded englneerin~aterials.
velocity is constant and the acceleration principle of the allowable stress design A. wire gage A. Strength of materials *
is zero. method, also known as B. strain gage • B. Materials engineering
A. uniform motion ~ A. working stress design method * C. stress gage C. Engineering mechanics
B. variable speed B. short cut method D. resistance gage D. Fluid mechanics
C. uniform velocity C. safety factor method
D. variable motion D. allowable method 83. Commonly used wire in strain gage. 90. A semi-graphical technique that is
A. Iron wire applicable whenever slopes of deflection
70. For a constant velocity system the 77. The study of moving objects. B. Nichrome wire * beams are not too great.
position function varies A. statics C. Copper wire A. shear - deflection method
A. linearly with time • B. dynamics * D. Live wire B. moment- area method *
B. inversely with time C. rectilinear motion C. sum-product method
C. proportional with time D. projectile 84. The strain gage sensitivity also D. load- area method
D. inversely proportional with time known as
78. In dynamics, the subject is divided A. the gage factor • 91. When more than one spring or
71. Linear acceleration means that the into two B. the length factor resisting mem~er share the load, the
acceleration A. kinematics and kinetics * C. the wire factor relative stiffnesses are known as
A. increases uniformly with time * B. statics and dynamics D. the usage factor . A elasticities

REFRESHER MANUAL 2nd Edition by JAS TORDILLO REFRESHER MANUAL 2nd Edition by JAS TORDILLO
59 - 16 1 Day 59- Solution Day 60 - EXAM I 60 - 1
Machine Design
B . rigidities * B. stress modulus
C. concentric C. modulus of elasticity 7. Shear stress (also known as diagonal
D. shear modulus D. tensile modulus
DAY 60- EXAM tension) in concrete beams is resisted
by
92. The area is normal to the force 99. These are correction factors used to 1. For any point in a loaded specimen, a A. a combination of stirrups and bent-up
carried account for the non-uniform stress plane can be found where the shear bars
A . shear stress distributions stress is
B. horizontal bars
B. normal stress • A. Stress concentration factors '" A. zero
C. vertical bars
C. strain B. Shear factor B. maximum D. concrete
D. bearing strength C. Beam factor C. minimum
D. Material deflection factor D. 100 8. 'Stirrups pass underneath the bottom
93. The area is parallel to the force. steel f~r anchorage or are welded to the
A. normal stress 100. Values of the stress concentration 2. T he normal stresses associated with
longitudinal bars. The stirrups can be
B. shear stress * factor are almost always the plane are known as
placed at any angle but are usually
C. tangential stress A. greater than 1.0 and can run as A. alternating stress
A. verftcal
D. longitudinal stress high as 3.0 and above * B. principal stress B. horizontal
B. less than 1.0 · C. shear stress C. incline
94. Elongation expressed on a fractional C. greater than 5.0 D. tangential stress
D. diagonal
or percentage basis D. less than 10
A. linear elongation 3. T he alternating stress is 9. In projectile, the range is maximum
B. stra in * A. equal to the range stress when the angle is
C. stress B. half of the range stress A. 30 deg
D. deflection C. one tenth of the range stress B. 45deg
D. one and a half times the range stress C. 60deg
95. The sum of the strains in the three D. 90deg
coordinate directions. 4. A criterion for acceptable design (or
A. inertia for failure) is established by graphically 10. The time for the projectile to travel
B. dilation * relating the yield strength and the from the launch point to the apex- is
C. summation endurance limit. One method of relating A: equal to the time to travel from apex
D. shear modulus this information is to impact point
A. Soderberg line
B. greater than the time to travel from
96. Hooke's law is a simple B. Goodman line apex to impact point
mathematical statement of the C. Straight line C. less than the time to travel from apex
relationship between elastic stress and D. Willans line to impact point
strain D. more or less than the time to travel
A . stress is proportional to strain * 5. In des ign, not conservative, but it has from apex to impact point
B. strain is proportional to stress the disadvantage of requiring the
C. stress is inversely proportional to ultimate strength to be known. 11. Motion of a particle around a circular
strain A. Goodman line path.
D. stress is two times strain B. Soderberg line A. Rotational particle motion
C. Solid line B. Angular motion
97. For normal stress, the constant of D. tension line C. Circular motion
proportionality is D. All of these
A. modulus of elasticity * 6. Resist flexural tension in the central
B. modulus ohigidity portion of the beam. 12. A particles moving in a curvilinear
"C. tensile modulus A. Straight bars path will have
D. stress modulus B. Bottom steel A. instantaneous linear velocity and
C. tension reinforcement linear acceleration
98. For shear stress, the constant of D. all 'of these B. instantaneous speed
proportionality is C. instantaneous acceleration
A . t he shear modulus * D. instantaneous distance

REFRESHER MANUAL 2nd Edition by JAS TORDILLO. REFRESHER MANUAL 2nd Edition by JAS TORDILLO
60 - 2 I Day 60 - EXAM Day 60- EXAM 160- 3
Machine Design
13. The inward acceleration is C. Section modulus of an object
perpendicular to the tangential velocity D. Radius of gyration A. Mass B. The dangerous sectiqn of the beam
and acceleration, it is known as B. Weight C. The optimum section of the beam
A. Tangential acceleration 20. This is obtained when the ·origin of C. Force D. The section of maximum shear
B. Normal acceleration the axes coincides with the object's D. Velocity
C. Total acceleration center of gravity 32. If a load is applied through the
D. Angular acceleration A. Centroidal mass moment of inertia 26. The Goodman line can be used with centroid of a tension or compression
B. Central mass moment of inertia steel, aluminum, titanium and ductile member's cross section, the loading is
14. The vector sum of the tangential and C. Gravitational mass moment of inertia magnesium alloys, but should not be called
normal accelerations D. Frictional mass moment of inertia used with A. Axial loading
A. Angular acceleration A. plastic B. Tangential loading
B. Resultant acceleration 21. Once the centroid mass moment of B. cast iron C. Normal loading
C. Perpendicular acceleration inertia is known, this theorem is used to C. stainless D. Central loading
D. Rotational acceleration find the mass moment of inertia about D. molybdenum
any parallel axis. 33. If a ll)ember experience both axial
15. Coriolis acceleration also acts on A. Parallel axes theorem 27. The energy stored in a loaded stress and bending stress, it is known as
particles moving on B. Central axes theorem member is equal to A. Beam-column
A. Rotating cylinders C. Normal axes theorem A. the work required to deform the B. Pier
B. Rotating spheres D. Mass axes theorem member C. Dam
C. Rotating objects B. the moment required to deform the D. Spring-colull)n
D. Rotating motion 22. Represents the ~istance from the member
rotational axis at which the object's C. the torque required to deform the 34. In a rectangular member, the load
16. Used to calculate the horizontal entire mass could be located without memoer must be kept within a rhombus-shaped
pressure from the vertical pressure changing the mass moment of inertia. D. the force required to deform the area formed from the middle thirds of
A. Coefficient of earth pressure A. normal axes member the centroidal axes. This area is known
B. Coefficient of earth acceleration B. radius of gyration as
C. Coefficient of earth friction C. diametral axes 28. The amount of force required to A. Core
D. Coefficient of earth motion D. normal distance cause a unit of deformation B. Kern
A. Stiffness C. Kernel
17. The of an object 23. A device, such as a rotating shaft, B. Spring index D. All of these
depends on the strength of the flywheel, or crank, is said to be statically C. Spring number
gravitational field. balanced D. Strain 35. Shear at a point is
A. Mass A. if its center of mass lies on the axis of · A. The sum of all vertical and horizontal
B. Weight rotation 29. The centroidal moment of inertia of forces acting on al'l object
C. Volume B. if its center of mass lies normal on the beam's cross section is called B. The sum of all vertical forces acting
D. Surface area the axis of rotation A. Bending moment . on an object *
C. if its center of mass lies B. Flexural load C. The product of all vertical forces
18. The resistance the object offers to perpendicular on the axis of rotation C. Torque acting on an object
attempts to accelerate it {i.e., change its · D. if its center of mass lies tangent on D. Reaction D. The product of all horizontal forces
velocity) in a linear direction. the axis of rotation acting on an object
A. Friction · 30. The maximum bending stress will
B. Inertia 24. The earth's attractive force, which is occur where 36. Shear is taken as positive when
C. Drag force called weight, can be assumed to act A. The moment is minimum there is a
D. Weight through the B. The moment is maximum A. Net upward force to the right of a
A. center of gravity C. The moment is zero point
19. Measures a solid object's resistance B. center of gyration D. The moment is optimum B. Net upward force to the left of a point
to changes in rotational speed about a C. center of line C. Net downward force to the left of a
specific axis D. center of the computed volum~ 31 . The region immediately adjacent to point
A. Mass moment of inertia the point of maximum bending moment D. Net horizontal force of a point
B. Coefficient of friction 25. _ _ _ is independent of the is called
strength of the gravitational field. A. The safest section of the beam

REFRESHER MANUAL 2nd Edition by .JAS TORDILLO REFRESHER MANUAL 2nd Eqiflon by JAS TORDILLO
60 - 4 I Day 60 - EXAM Day 60 - EXAM I 60 - 5
Machine Design
37. Shear is negative when there is 43. Find the polar section modulus of a
A. Net upward, force to the right of a hollow shaft with outside diameter of 5 C. Right hand threaded 53. A .cylindrical shaft transmits power
point inches and inside diameter of 2 inches. D. Left hand threaded delivered by a 15 horsepower electric
B. Net upward force to the left of a point A. 23.91 in3 motor turning at 1,800 rpm. If the shaft
3
C. Net downward force to the right of a B. 17.31 in 49. A single square thread power screw is made from AISI 1117 hot-rolled steel,
3
point C. 14.51 in is to raise a load of 200 kN . The screw 1-inch in diameter and 12 inches length,
3
D. Net downward force between the D. 9.21 in has a major diarpeter of 36 mm and a what would the angular deflection of this
point and the left end pitch of 6 mm. The coefficient of thread shaft be?
44. A steel rod 4.0 em in diameter and friction and collar ''friction are 0.13 and A. 0.145 deg
38. The total bending moment acting on 1000 mm long has an allowable 0.10 respectively. If the collar mean B. 0.219 deg
an object elongation not to exceed 7 mm, find the diameter is 90 mm and the screw turns C 0.319 deg
A. Moment at a point allowable load in kN. at 60 rpm, find pqwer input to the screw. D. 0.892 deg
B. Torque at a point A. 507.5 kN A. 2.5 kw
C. Load at a point B. 909.2 kN B. 3.1 kw 54. A plate-type clutch has the following
D. Energy at a point C. 1,818.9 kN C. 4.2 kw properties: ro = 15 in, n = 8 in,
D. 2,945.2 kN D. 4.8 kw engagement force of 160 lb (normal
39. Wh.en the upper surface of the beam force), and turns at 3,000 rpm. The
is in compression and the lower surface 45. A spring with a force constant of 300 50. An automobile engine creates a friction disc has a coefficient of friction of
is in tension, the moment is N/m is compresses by 10 em. A 300-g torque of 400 foot-pounds at a rotational 0.3. Determine the power that can be
A. Positive ball is placed against the end of the speed of 4000 rpm. In fourth gear, the transmitted by this system.
B. Negative spring, which is then~released. What is transmission has a one-to-one ratio. A. 16.3 hp
C. .Plus or minus the ball's velocity when it leaves the The differential, however. has a ratio of 8. 26.3 hp
D. Zero spring? 4 to 1, which creates a torque in the. axle c. 36.3 hp
A. 3.16 m/s of 1,600 foot-pounds. What is the force D. 46.3 hp
40. What is the decrease in lateral B. 2.85 m/s being exerted on the road by the
dimension due to and axial tensile force C. 2.21 m/s automobile tire if the diameter of the tire 55. Determine the maximum pressure
of 100,000 lbs in a steel bar 3.5 inches D. 1.23 m/s is 36 inches? in a plate clutch where the axial force is
on each side and 20 feet long? A.6661bs 1300 lb, the inside radius of contact is
A . 1.67 x 10·4 in/in 46. Two springs, one of force constant B. 10661bs R; = 2.5 in, the outside radius is Ro = 5
B. 1.92 x 10· 4 in/in k1 and the other of force constant kz, are C. 15661bs in Uniform wear is assumed.
C. 2.72 x 10'4 in/in connected end-to-end. If k1 = 10 N/m, D. 36661bs A 23.1'psi
D. 3.33 x 10--4 in/in and kz = 15 N/m , find k. B. 33.1 psi
A. 3 N/m 51. A 30,000-pound compression load C. 43.1 psi
41. • A wire of length 5.0 . m has a B. 6 N/m is applied to a two-inch diameter steel D. 49.1 psi
percentage strain of 0.034% when C. 9 N/m bar that is made from AISI 1020 hot-
loaded with tensile force. Determine the D. 15 N/m rolled steel with Su = 55,000 psi and 56. Determine the minimum pressure in
extension of the wire. Sy = 30,000 psi. The original length of a plate clutch where the axial force is
A. 1.7 mm 47. ln'the thread specification: the bar is 10 inches. What is the stress 1300 lb, the inside radius of contact is
B. 2.2 mm 2-8UNC-2A-RH, what do you mean by in the bar? Rt = 2.5 in, the outside radius is Ro = 5
C. 3.5 mm the number 8? A . 9,549.3 psi in Uniform wear is assumed.
D. 5.2 mm A. Nominal size B. 5,949.3 psi A 16.55 psi
B. No. of threads C. 4,995.3 psi B. 19.55 psi
42. What is the bending moment in ft-lb C. No. of threads per inch D. 3,994.4 psi C. 23.55 psi
at the fixed end of a 12 ft truss with a D. Type of thread D. 33.55 psi
uniform weight of 100 lb/ft and a 52. From Problem No. 51, By how much
concentrated vertical load at the free 48. In the thread specification: =
did the bar shorten? Use E 30 x 106 57. Determine the average pressure in a
end of 2000 lbs? 2-8UNC-2A-RH, what do you mean by psi. plate clutch where the axial force is
A. 8,100 ft-lb A? A. 0.005 in 1300 lb, the inside radius of contact is
B. 15,200 ft-lb A. External thread B. 0.004 in R; = 2.5 in, the outside radius is Ro = 5
C. 24,500 ft-lb B. Internal thread C. 0.0032 in in Uniform wear is assumed.
D. 31,200 ft-lb D. 0.0015 in A. 17.22 psi

REFRESHER MANUAL 2nd Edition by JAS TORDILLO REFRESHER MANUAL 2nd Edition by JAS TORDILLO
60 • 6 I Day 60 - EXAM Day 60 - Solution 1 60 - 7
Machine Design
B. 22.07 psi
C. 2807 psi 7. Shear stress (also known as diagonal
lrj ') DAY 60 - SOLUTION tension) in concrete beams is resisted
D. 35.06 psi
DAY 60- ANSWER KEY by
58. A multiple dick clutch has 4 steel 1.A 16. A 31 . 646. 6 1. For any point in a loaded specimen, a A. a combination of stirrups and
disks and 3 bronze disks and each 2.6 .17. 8 32.A47. C plane can be found where the shear bent-up bars *
3. B 18. 8 33. A 48. A stress is
surface has a contact area of 4 in2 and a 4. A 19. A 34. 049. D
B. horizontal bars
mean radius of 2 in. The coefficient of A. zero • C. vertical bars
5. A 20. A 35. 6 50. 8
friction is 0.25. What is the horsepower 6. 0 21 . A 36.851 . A B. maximum D. concrete
capacity for an axial force of 150 lb if the 7. A 22. 8 37. 0 52. C C. minimum
B. A 23. A 38. A 53. C D. 100 8. Stirrups pass underneath the bottom
clutch rotates at 400 rpm? Assume 9. 8 24. A 39. A 54. 6
uniform wear in the clutch plates. 10.A 25. A 40. C 55. 6
steel for anchorage or are welded to the
A. 2.75 hp 11.026.6 41.A56.A
2. The normal stresses associated with longitudinal bars. The stirrups can be
B. 3.50 hp 12.A 27. A 42. D 57. 6 the plane are known as placed at any angle but are usually
c. 4.30 hp 13.8 28. A 43. A 58. C A. alternating stress A. vertical*
14.8 29. A 44. C 59. C B. principal stress • B. hO'Fizontal
D. 5.80 hp 15.8 30. 8 45. A 60. D
C. shear stress C. incline
59. Determine the required number of G D. tangential stress D. diagonal
coils and permissible force in a helical
spring made of 1/16 inch diameter steel 3. The alternating stress is 9. In projectile, the range is maximum
wire, assuming a spring index of 6 and A. equal to the range stress when the angle is
an allowance stress of 55,000 psi in B. half of the range stress * A. 30 deg
shear. The spring rate is to be 12 lb/in. C. one tenth of the range stress B. 45deg *
A. 7.51b D. one and a half times the range stress C. 60 deg
B. 9.51b D. 90 deg
C.11.251b 4. A criterion for acceptable design (or
D. 15.51b for failure) is established by graphically 10. The time . for the projectile to travel
relating the yield strength and the from the launch point to the apex is
60. A spur pinion 6 inches in diameter
I endurance limit. One· method of relating A· equal to the time to travel from
has a torque of-4500 in-lb applied to it. this information is apex to impact point *
The spur gear in mesh with it is 12 A. Soderberg line * B. greater than the time to travel from
inches in diameter. The pressure angle B. Goodman line apex to impact point
is 20° Determine the separating force C. Straight line C. less than the time to travel from apex
F,. D. Willans line to impact point
A. 246 1bs D. more or less than the time to travel
B. 346 1bs ' 5. In design, not conservative, but it has from apex to impact point
C. 446 lbs the disadvantage of requiring the
D. 546 1bs ultimate strength to be known. 11. Motion of a particle around a circular.
A. Goodman line * 'path .
B. Soderberg line A. Rotational particle motion
c. Solid line B. Angular motion
D. tension line C. Circular motion
D. All of these *
6. Resist flexural tension in the central
portion of the beam. 12. A particles moving in a curvilinear
A. Straight bars path will have
B. Bottom steel A. instantaneous linear velocity and
C. tension reinforcement linear acceleration *
D. all of these • B. instantaneous speed
C. instantaneous acceleration
D. instantaneous distance

REFRESHER MANUAL 2nd Edition by·JAS TORDILLO REFRESHER MANUAL 2nd fdition by JAS TORDILLO

II
60-8 I Day 60- Solution Day 60 - Solution 160 - 9
Machine Design
13. The inward acceleration is C. Section modulus of an object
perpendicular to the tangential velocity D. Radius of gyration 25. _ _ _ is independent of the 31. The region immediately adjacent to
and acceleration, it is known as strength of the gravitational field. the point of maximum bending moment
A. Tangential acceleration 20. This ~s obtained when the origin of A. Mass • is called
B. Normal acceleration * the axes coincides with the object's B. Weight A. The safest section of the beam
C. Total acceleration center of gravity C. Force B. The dangerous section of the
D. Angular acceleration A. Centroidal mass moment of D. Velocity beam*
inertia* C. The optimum section of the beam
14. The vector sum of the tangential and B. Central mass moment of inertia 26. The Goodman line can be used with D. The section of maximum shear
normal accelerations C. Gravitational mass moment of inertia steel, aluminum, titanium and ductile
A. Angular acceleration D. Frictional mass moment of inertia magnesium alloys, but should not be 32. If a load is applied through the
B. Resultant acceleration * used with · centroid of a tension or compression
C. Perpendicular acceleration 21. Once the centroid mass moment of A. plastic member's cross section, the loading is
D. Rotational acceleration inertia is known, this theorem is used to B. cast iron • called
find the mass moment of inertia about C. ·stainless A. Axial loading •
15. Coriolis acceleration also acts on any parallel axis. D. molybdenum B. Tangential loading
particles moving on A. Parallel axes theorem * C. Normal loading
A. Rotating cylinders B. Central axes theorem 27. The energy sto~d in a loaded D. Central loading
B. Rotating spheres * C. Normal axes theorem member is equal to
C. Rotating objects D. Mass axes theorem A. the work required to deform the 33. If a member experience both axial
D. Rotating motion member* stress and bending stress, it is known as
22. Represents the distance from the B. the moment required to deform the A. Beam-column •
16. Used to calculate the horizontal rotational axis at which the object's member B. Pier
pressure from the vertical pressure entire mass could be located without C. the torque required to deform the C. Dam
A. Coefficient of earth pressure * changing the mass moment of inertia. member D. Spring-column
B. Coefficient of earth acceleration A. normal axes D. the force required to deform the
C. Coefficient of earth friction B. radius of gyration " member 34. In a rectangular member, the load
D. Coefficient of earth motion Q. diametral axes must be kept within a rhombus-shaped
D. normal diStance 28. The amount of force required to area formed from the middle thirds of
17. The _ _ of an object cause a unit of deformation the centroidal axes. This area is known
depends on the strength of the 23. A device, such as a rotating shaft, A. Stiffness • as
gravitational field. flywheel, or crank, is said to be statically B. Spring index A. Core
A. Mass balanced C. Spring number B. Kern
B. Weight* A. if its center of mass lies on the D. Strain C. Kernel
C. Volume axis of rotation •· D. All of these •
D. Surface area B. if its center of mass lies normal on 29. The centroidal moment of inertia of
the axis of rotation the beam's cross section is called 35. Shear at a point is
18. The resistance the object offers to C. if its center of mass lies A. Bending moment * A. The sum of all vertical and horizontal
attempts to accelerate it (i.e., change its perpendicular on the axis of rotation B. Flexural load forces acting on an objeqt
velocity) in a linear direction. D. if its center of mass lies tangent on C. Torque B. The sum of all vertical forces
A. Friction the axis of rotation D. Reaction acting on an object •
B. Inertia " C. The product of all vertical forces
C. Drag force 24. The earth's attractive force, which is 30. The maximum bending stress will acting on an object·
D. Weight called weight, can be assumed to act occur where D. The product of all horizontal forces
through the A. The moment is minimum acting on an object
19. Measures a solid object's resistance A. center of gravity • B. The moment is maximum •
to changes in rotational speed about a B. center of gyration C. The moment is zero 36. Shear is taken as positive when
specific axis C. center of line D. The moment is optimum there is a
A. Mass moment of inertia • D. center of the computed volume A. Net upward force to the ·right of a
B. Coefficient of friction point

REFRESHER MANUAL 2nd Edition by JAS TORDILLO REFRESHER MANUAL 2nd Edition by JAS TORDILLO ·
60 -10 1Day 60 ·Solution Day 60 - Solution I GO - 11
Machine Design
B. Net upward force to the left of a 41 . A wire of length 5.0 m has a
point* percentage strain of 0.034% when
C. Net downward force to the left of a loaded with tensile force. Determine the
C. 1,818.9 kN * k1k2 _ 10(1)) =6 N/m
D. 2,945.2 kN k = k1 +k2 - 10+15
point extension of the wire.
D. Net horizontal force of a point A . 1.7mm*
B. 2.2 mm Y = FL 47. In the thread specification:
AE 2-8UNC- 2A-RH, what do you mean by
37. Shear is negative when there is C. 3.5mm
A Net upward force to the right of a D. 5.2 mm 6
the number 8?
Since E is not given, use E = 30 x 10 A. Nominal size
point
B. Net upward force to the left of a point psi for steel B. No. of threads
C . ,Net downward force to the right of a Strain= L C. No. of threads per inch "
point
L E = 30 x 105 psi= 206,786 Mpa D. Type of thread
D. Net downward force between the 0.034 =-y-
100 5000 Substitute: 48. In the thread specification:
point and the left end*
y = 1.7 mm = F(IOOO) 2-8UNC-2A-RH, what do you mean by
7
A?
38. The total bending moment acting on
an object · 42. What is the bending moment in ft-lb ~ (0.04)2 ( 206,786x!0 6 ) A. External thread *
A. Moment at a point* at the fixed end of a 12 ft truss with a B. Internal thread
uniform weight of 100 lb/ft and a F = 1,818,984.6 N = 1,818.9 kN
B. Torque at a point C. Right hand threaded
C. Load at a point concentrated vertical load at the free D. Left hand threaded
end of 2000 lbs? 45. A spring with a force constant of 300
D. Energy at a point N/m is compresses by 10 em. A 300-g
A. 8,100 ft-lb 49. A single square thread power screw
B. 15,200 ft-lb ball is placed against the end of the
39. When the upper surface .Qf the beam spring, which is then released. What is is to raise a load of 200 kN. The screw
is in compression and the lower surface C. 24,500 ft-lb has a major diameter of 36 mm and a
D. 31,2.00 ft-lb * the ball's velocity when it leaves the
is in tension, the moment is pitch of 6 mm. The coefficient of thread
spring?
A. Positive * friction and collar friction are 0.13 and
Bending Moment A. 3.16 m/s *
B. Negative B. 2.85 m/s 0.10 respectively. If the collar mean
C . Plus or niirws = 2000(12) + .!.(100x12) (12) diameter is 90 mm and the screw turns
C. 2.21 m/s
D. Zero 2 0
D 1.23 m/s at 60 rpm, find power input to the screw.
= 31,200 ft-lb A 2.5 kw
40. W hat is the €1ecrease in lateral The kinetic energy of the ball equals the B: 3.1 kw
dimension due to and axial tensile force 43. Find the polar section modulus of a elastic potential energy of the C. 4.2 kw
of 100,000 lbs in a steel bar 3.5 inches hollow shaft with outside diameter of 5 compressed spring. D. 4.8 kw*
on each side and.20 feet long? Inches and inside diameter of 2 inches.
4
A. 1.67 x 10' in/in A 23.91 in3 * KE = PE For square thread:
B. 1.92 x 10'4 in/in B. 17.31 in 3 )6 mv 2
= )6 ks 2 Depth of thread = Y. p = Y. (6) = 3 mm
C. 2.72 x 10-4 in/in* C. 14.51 in 3 Dm = 36 - 3 = 33 mm
D. 9.21 in3
~ JN&oo
4 Lead= 6 mm
D. 3.33 x 10' in/in V = s= --(0. 10) = 3.16 m/s
0.300

Stress
Force
= Area
· 1t
Zp-32
(D~ -D~)
I
24)
-~-(s4 _
= ~3~2~~~ 46. Two springs, one of force constant
Tan x = Lead = - 6 - = 0.057874
JtDm 7t(33)
k, and the other of force constant k2, are
100,000
D% ~ connected end-to-end. If k1 = 10 N/m,
X = 3.31 deg
tan x(1- ftan x)
and k 2 = 15 N/m, find k. Efficiency= f fcD£.(1-ftanx)
= 3.5(3.5) = 23.91 in3 A. 3 N/m tanx+ + Dm
= 8163.3 psi B. 6 N/m *
. . stress 44. A steel rod 4.0 em in diameter and C. 9 N/m =
Modulus of elast1c1ty, E = - - . - -1000 mm long has an allowable 0.057874[1- 0.13(0.057874)]
stram D. 15 N/m
elongation not to exceed -7 mm, find the 0.057874+ 0.13 + O.l0'(90} (1- 0.13(0.057874)]
3 X 101 = 8163.3 allowable load in kN. 33
strain
A 507.5 kN k= ~=~
Strain = 2.72 x 10'4 in/in s kt +k2
B. 909.2 kN = 0.12526 = 12.526%
REFRESHER MANUAL 2nd Edition by JAS TORDILLO , REFRESHER MANUAL 2nd Edition by JAS TORDILLO
60 - 12 1Day 60 - Solution Day 60 - Solution 1 60 - 13
Machine Design
v = velocity = 6(60) = 360 mm/min the bar is 10 inches. What is the stress
= 6 mm/sec in the bar? e= TL 56. Determine the minimum pressure in
A. 9,549.3 psi * JG a plate clutch where the axial force is
B. 5,949.3.psi 1300 lb, the inside radius of contact is
Power mput = 100(0.006) = 4.79 kW 525(12)
0.12526 C. 4,995.3 psi R; = 2.5 in, the outside radius is Ro = 5
D. 3,994.4 psi 0.0981(11.5xl o6 ) in Uniform wear is assumed.
- = 0.005584 rad = 0.319 deg A. 16.55 psi *
100 kN
B. 19.55 psi
s= F
-=
30,000 lb
2
= 9' 549.30 psi 54. A plate-type clutch has the following C. 23.55 psi
A ~(2 i~·)_ properties: ro 15 in, n =
engagement force of 160 lb (normal
8 in, = D. 33.55 psi

4 force), and turns at 3,000 rpm. The The minimum pressure occurs at r = Ro
friction disc has a coefficient of friction of 1300
52. From Problem No. 51, By how much 0.3. Determine the power that can be P(min) = ( X) =16.55 psi.
21t 5-2.5 5
6
did the bar shorten? Use E = 30 x 10 transmitted by this system.

@
Dm = 90 mm
psi.
A. 0.005 in
B. 0.004 in
C. 0.0032 in *
A 16.3 hp
B. 26.3 hp *
C. 36.3 hp
57. Determine the average pressure in a
plate clutch where the axial force is
1300 lb, the inside radi1,1s of.. contact is
D. 46.3 hp R; = 2.5 in, the outside radius is Ro = 5
D. 0.0015 in
Collar in Uniform wear is assumed.
_ FL - 30,00'0 lb( 10 in) Tr = fN (ro +f;) A. 17.22 psi
2 B. 22.07 psi *
Y=-
2
AE n(2 in) Tr=0.3(1601b) es;8) C. 28.07 psi
50. An automobile engine creates a ~--'--(30 x106 lb/in2 ) D. 35.06 psi
torque of 400 foot-pounds at a rotational 4 Tr =552 in-lb
speed of 4000 rpm. In fourth gear, the =- 0.0032 in Average pressure P
transmission has a one-to-one ratio. p _ TfN = F = 1300
The differential, however, has a ratio of
4 to 1, which creates a torque in the axle
53. A cylindrical shaft transmits pC»Ner
delivered by;--.a 15 horsepower electric
1
- 63,000 n(R5-Rn n(5 2 -2.s2)
of 1,600 foot-pounds. What is the force motor turning at 1,800 rpm. If the shaft p - 552(3,000)
I- 63,000
= 22.07 psi
being exerted on the road by the is made from AISI 1117 hot-rolled steel,
automobile tire if the diameter of the tire 1-inch in diameter and 12 inches length, Pr = 26.28 hp 58. A multiple dick clutch has 4 steel
is 3~ inches? what would the angular deflection of this ' disks and 3 ·bronze disks and each
A. 6661bs shaft be? 55. Determine the maximum pressure 2
surface has a contact area of 4 in and a
B. 1066 lbs * A. o.145 deg in a plate clutch where the axial force is mean radius of 2 in. The coefficient of
C. 15661bs B. 0.219 deg 1300 lb, the inside radius of contact is friction is 0.25. What is the horsepower
D. 3666 1bs C. 0.319 deg * R; = 2.5 in, the outside radius is Ro = 5 capacity for an axial force of 158 lb if the
D. 0.892 deg in Uniform wear is assumed. clutch rotates at 400 rpm? Assume
T = Fd A. 23.1 psi uniform wear in the clutch plates.
F = 2:_ B. 33.1 psi* A. 2.75 hp
T = 63,000 HP
d C. 43.1 psi B. 3.50 hp
N
ft -lb
1,600 D. 49.1 psi c. 4.30 hp *
F= = 1066.67 lbs T = 63,000 (15) = 525 in-lb D. 5.80 hp
36 in. (ft) 1,800 The pressure variation is given by
2 (12 in.) S= TL p = ~ = ---,,--F-~ T =F f Rr n =150(0.25)(2)(6) =450 in-lb
51 . A 30,000-pound compression load
JG r 2n(R 0 - Ri Xr)
is applied to a two-inch diameter steel _ nD4 n(l.O )4 HP = __.!!::!_ = 450(600) =4 _28
J--=-- The maximum pressure occurs at r = R; 63,000 63,000
bar that is. made from AISI 1020 hot- 32 32
rolled steel with Su = 55,000 psi and
=
Sy 30,000 psi. The original length of
J = 0.0981 in4
P(max) =
1300
(· .
21t)- 2.) 2.5
= 33.1 ps1
.
-x )
REFRESHER MANUAL 2nd Edition by JAS TORDILLO REFRESHER MANUAL 2nd Edition by)AS TORDILLO
60 -141 Day 60- Solution
59. Determine the required number of
coils and permissible force in a helical
spring made of 1/16 inch diameter steel
wire, assuming a spring index of 6 and
an allowance stress· of 55,000 psi in
shear. The spring rate is to be 12 lb/in.
A. 7.5 lb
B. 9.51b
c. 11.25 lb,.
D. 15.5 lb

C=~
d
6 =_E.__
1/16
D = 0.375 in
4C-l 0.615
Wahl factor F = - - + - - = 1.25.
4C-4 C
F Gd
Y= &C 3n
6
=· (In6XII.5xlo )
12 3
8(6 )n
n = 34.66 turns

Ss = K 8FD
3 .
7!d
55,000 = 1.25 8F(0.375)
1t(l116)3
F = 11.251b
I
60. A spur pinion 6 inches in diameter
has a torque of 4500 in-lb applied to it.
The spur gear in mesh with it is 12
inches in diameter. The pressure angle
is 20°. Determine the separating force
F,.
A. 246lbs
B. 3461bs
C. 4461bs
D. 546 lbs *

Ft = Mtfr = 4500/3 = 1500 lb

F, = Ft tan0= 1500 tan 20° = 545.951b

REFRESHER MANUAL 2nd Edition by JAS TORDILLO

S-ar putea să vă placă și